You are on page 1of 552

M THPI D

Tuyển tập một số bài toán dãy số từ các tạp chí


toán của Mỹ

Mathpiad - Tạp chí và tư liệu toán học


Ngày 24 tháng 7 năm 2021

1 Các bài toán


Bài toán 1
Cho {an }n>1 là một dãy đơn điệu tăng, với an > 0, đồng thời lim an = ` > 0. Chứng
n→∞
minh rằng

X a2k+1 − a2k a1 + a2
(a2k−1 + a2k ) · 6`−
k=1
a2k+1 + a2k 2

Spiros P. Andriopoulos, Third High School of Amalianaa, Eleia, Greece.

Lời giải. Paolo Perfetti, University of Rome Tor Vergata, Italy.


Từ điều kiện của bài toán, ta có
a2k−1 + a2k
0< 61
a2k+1 + a2k
Suy ra
n n
X a2k+1 − a2k X
(a2k−1 + a2k ) · 6 (a2k+1 − a2k ) = a2n+1 − a2
k=1
a2k+1 + a2k k=1

Cho n → ∞ ta được

X a2k+1 − a2k a1 + a2
(a2k−1 + a2k ) 6 ` − a2 6 ` − ,
k=1
a2k+1 + a2k 2

a1 + a2
vì ` − a2 6 ` − và {an } là dãy đơn điệu. 
2
Bài toán 2
Cho hai và hbi là hai tập vô hạn các số nguyên dương. Xét dãy hxi là dãy vô hạn các số
thực xác định bởi
ab11 · · · abnn
xn =  b +···+bn , n > 1
a1 b 1 + · · · + an b n 1
b1 + · · · bn

a, Chứng minh rằng tồn tại lim xn .


n→∞

LAT EX by Mathpiad
1
TUYỂN TẬP DÃY SỐ TRONG 10 NĂM TỪ CÁC TẠP CHÍ TOÁN CỦA MỸ

b, Tìm tất cả các số thực c thỏa mãn, tồn tại hai dãy hai and hbi sao cho lim xn = c.
n→∞

Vazgen Mikayelyan, Yerevan State University, Yerevan, Armenia.

Lời giải. Omran Kouba, Higher Institute for Applied Sciences and Technology, Damascus, Syria.

a, Đầu tiên, chúng ta sẽ chứng minh xn là dãy không tăng. Xét các dãy sau

n
! Λ1 n
n
bn Y 1 X
Λ n = b1 + · · · + bn , λn = , Gn = abkk , An = bk ak ,
Λn k=1
Λn k=1
 Λn
Gn
Khi đó, xn = . Do vậy,
An
λ
Gn+1 = G1−λ
n
n+1 n+1
an+1 và An+1 = (1 − λn+1 ) An + λn+1 an+1 ,

Dẫn đến
λ
Gn+1 G1−λ
n
n+1 n+1
an+1
=
An+1 (1 − λn+1 ) An + λn+1 an+1
 1−λn+1 λn+1
Gn An1−λn+1 an+1
= ,
An (1 − λn+1 ) An + λn+1 an+1
Từ đây suy ra
λ
!Λn+1
xn+1 A1−λ
n
n+1 n+1
an+1
=
xn (1 − λn+1 ) An + λn+1 an+1

Áp dụng bất đẳng thức AM − GM, với a > 0, A > 0, và 0 < λ < 1 thì

A1−λ aλ 6 (1 − λ)A + λa,

ta suy ra xn+1 6 xn . Để ý rằng x1 = 1 và xn > 0 với mọi n > 1. Do vậy, xn hội tụ đến một
giới hạn nằm trong đoạn [0, 1].

b, Chúng ta chứng minh rằng, với mọi c ∈ [0, 1], tồn tại dãy hai và hbi sao cho xn hội tụ đến
1
c. với c = 0, chọn bn = 1 và an = với mọi n, khi đó
n(n + 1)

(n + 1)n
xn = →0
n! · (n + 1)!

Với c = 1, chọn bn = 1 và an = 1 với mọi n, khi đó xn = 1 với mọi n và xn → 1.


Với 0 < c < 1, chọn bn = 1 với mọi n, và chọn
√ √
1+ 1−c 1− 1−c 1
a1 = √ , a2 = √ , an = √ với n > 3 .
c c c

Khi đó, xn = c với mọi n > 2 nên xn → c.

LAT EX by Mathpiad
2
TẠP CHÍ VÀ TƯ LIỆU TOÁN HỌC

Bài toán 3
Cho p là một số nguyên tố. Đặt hui Là dãy xác định bởi un = n với 0 6 n 6 p − 1 và
un = pun+1−p + un−p với n > p. Chứng minh rằng với mọi số nguyên dương n thì

vp (n) = vp (un ).

Bakir Farhi, University of Béjaia, Béjaia, Algeria.

Lời giải. John H. Lindsey II, Cambridge, Massachussetts.


Đầu tiên, chúng ta sẽ chứng minh bằng quy nạp rằng, với mọi n > 1 thì vp (n!) 6 n − 1 và đẳng
thức không xảy ra với p > 2 và n > 1. Khẳng định hiển nhiên đúng với n < p.
Với n > p, đặt n = j + ip với 0 6 j < p và i > 0. Áp dụng giả thiết quy nạp, ta có
vp ((j + ip)!) = vp (p(2p) · · · (ip)) = i + vp (i!) 6 i + i − 1 6 ip − 1
Hiển nhiên, dấu bằng không xảy ra với p > 2. Bây giờ, ta chứng minh
i  
X i k
uj+ip = p uj+k
k=0
k

với i > 0 và j > 0 bằng cách quy nạp theo i.


Khẳng định hiển nhiên đúng với i = 0, với i > 1, ta có
uj+ip = uj+(i−1)p + puj+1+(i−1)p
i−1   i−1  
X i−1 k X i − 1 1+k
= p uj+k + p uj+1+k
k=0
k k=0
k
i−1   i  
X i−1 k X i−1 k
= p uj+k + p uj+k
k=0
k k=1
k−1
i  
X i k
= p uj+k
k=0
k

Bây giờ, ta chứng minh vp (un ) = vp (n) với n > 1.


Với 1 6 j < p và i > 0, ta có
i  
X i k
uj+ip = uj + p uj+k
k=1
k

Do uj = j, nên uj+ip không chia hết cho p. Mà j + ip cũng không chia hết cho p. Do vậy
vp (uj+ip ) = 0 = vp (j + ip)
Với j = 0, vì u0 = 0 và u1 = 1, nên
i   i  
X i k X i k
u0+ip = p uk = ip + p uk
k=0
k k=2
k

Giả sử p > 2. Khi k > 1,


  
i k
vp p uk > vp (i) − vp (k!) + k > vp (i) − k + 1 + k = vp (ip)
k
Từ đây ta suy ra vp (uip ) = vp (ip).
Bây giờ ta xét p = 2.

LATEX by Mathpiad 3
TUYỂN TẬP DÃY SỐ TRONG 10 NĂM TỪ CÁC TẠP CHÍ TOÁN CỦA MỸ

• Với k = 2, ta có
  
i 2
v2 2 u2 > v2 (i) − 1 + 2 + 1 = v2 (2i) + 1
2
 
i
• Với k > 2, chúng ta sử dụng nhân tử i(i − 1)(i − 2) trong (với một trong 2 số
k
i − 1, i − 2 là số chẵn).

iik 2k uk > v2 (i) + v2 ((i − 1)(i − 2)) − v2 (k!) + k


 
v2
> v2 (i) + 1 − k + 1 + k = v2 (2i) + 1

Từ đây ta cũng suy ra v2 (u2i ) = v2 (2i).

Bài toán được giải quyết. 


Bài toán 4
Cho a ∈ R và hàm số f : (−1, 1) → R khả vi tại 0 . Tính
  
k
lim an − f
n→∞ n2

Dorin Andrica, Babes-Bolyai University, România

Lời giải. Daniel Lasaosa, Pamplona, Spain

k 1
Nhận thấy, 2
6 → 0 khi n → ∞, do vậy ta có khai triển sau
n n
   
k k 0 1
f 2
= f (0) + 2 f (0) + O .
n n n2

Do đó,
n n
f 0 (0) X
   
X k 1
f 2
= nf (0) + 2 k+O
k=1
n n k=1 n

Pn n(n + 1)
Do k=1 k= , nên
2
n
f 0 (0)
   
X k 1
f = (a − f (0))n − + O
k=1
n2 2 n

f 0 (0)
 
1
Theo RHS, lim O = 0 và là hằng số, ta có kết luận sau
n→∞ n 2

     −∞ khi f (0) > a
k 
lim an − f = ∞ khi f (0) < a
n→∞ n2  f 0
(0)
khi f (0) = a


2


LATEX by Mathpiad 4
TẠP CHÍ VÀ TƯ LIỆU TOÁN HỌC

Bài toán 5
 √
Cho dãy số sau an = ( 3 65 − 4)−n , với n ∈ N∗ . Chứng minh rằng an ≡ 2, 3( mod 15)


Vlad Matei, University of Wisconsin, USA

Lời giải. Daniel Lasaosa, Pamplona, Spain


√ √ √ 2 √
Đặt r = 3 65 và u = ( 3 65 − 4)−1 = 3 65 + 4 4 65 + 16 = r2 + 4r + 16. Để ý rằng
u2 = 48r2 + 193r + 776, u3 = 2316r2 + 9312r + 37441
Do đó, u là một nghiệm của phương trình
x3 − 48x2 − 12x − 1 = 0
Gọi hai nghiệm còn lại của phương trình trên là v, w, ta có
v + w = 48 − u = 32 − 4r − r2 = −12(r − 4) − (r − 4)2
1
và vw = = r − 4. Đầu tiên, ta có
u
3
3 · 42

3 3 3 1
r =4 +1=4 + < 4+
48 48
1
hay 0 < r − 4 < , suy ra 0 > v + w > −1 và 1 > vw > 0, dẫn đến v, w < 0. Và với mọi số
48
nguyên dương n, ta có
0 < |v n + wn | = |v n | + |wn | 6 |v| + |w| < 1
hay 1 > v n + wn > 0 với n chẵn và 0 > v n + wn > −1 với n lẻ. Ta lại có,
v 2 + w2 = (v + w)2 − 2vw = 1552 − 193r − 48r2
v 3 + w3 = (v + w)3 − 3vw(v + w) = 74882 − 9312r − 2316r2
nên
u + v + w = 48 ≡ 3 (mod 15),
u + v 2 + w2 = 2328 ≡ 3 (mod 15),
2

u3 + v 3 + w3 = 112323 ≡ 3 (mod 15).


Bây giờ, ta xét dãy (bn )n>1 xác định bởi
(
b1 = 48, b2 = 2328, b3 = 112323
bn+3 = 48bn+2 + 12bn+1 + bn

với các nghiệm của phương trình là u, v, w. Do vậy, bn = un + v n + wn với mọi n ∈ N∗ . Dễ thấy
bn với mọi n ∈ N∗ . Do b1 ≡ b2 ≡ b3 ≡ 3(mod15), và
bn+3 ≡ 3 (bn+2 − bn+1 ) + bn (mod15)
nên bn ≡ 3(mod15) với mọi n ∈ N∗ . Từ đây, ta kết luận rằng, với mọi n ∈ N∗ , n chẵn, thì
an = [un ] = un + v n + wn − 1 ≡ 2 (mod 15)
và với n ∈ N∗ , n lẻ, thì
an = [un ] = un + v n + wn ≡ 3 (mod 15)
Vậy an ≡ 2, 3(mod15). 

LATEX by Mathpiad 5
TUYỂN TẬP DÃY SỐ TRONG 10 NĂM TỪ CÁC TẠP CHÍ TOÁN CỦA MỸ

Bài toán 6
Xét dãy (an )n>0 xác định bởi a0 = 0, a1 = 1, a2 = 2, a3 = 6 và

an+4 = 2an+3 + an+2 − 2an+1 − an , n > 0.

Chứng minh rằng tồn tại vô hạn số nguyên dương n thỏa mãn n2 | an .

Dorin Andrica, Babes-Bolyai University, Cluj Napoca, Romania

Lời giải. Li Zhou, Polk State College, USA


Xét (Fn ) là dãy Fibonacci F0 = 0, F1 = 1, Fn+2 = Fn+1 + Fn với mọi n > 0. Dễ thấy an = nFn
với n = 0, 1, 2, 3. Giả sử rằng an = nFn với mọi n = 0, 1, 2, ..., k với k > 3, ta có
ak+1 = 2kFk + (k − 1)Fk−1 − 2(k − 2)Fk−2 − (k − 3)Fk−3
= 2kFk + 2Fk−1 − (k − 1)Fk−2
= (k + 1)Fk + (k + 1)Fk−1 = (k + 1)Fk+1
Do vậy an = nFn với mọi n > 0. Bây giờ, ta chỉ cần chỉ ra rằng có vô hạn n để n | Fn .
Thật vậy, dễ dàng chứng minh được 2m+2 | F3·2m với mọi m > 1. Lại có với mọi m > 2 thì
144 = F(12,3·2m ) = (F12 , F3·2m ) nên 3 | F3·2m . Do đó, n | Fn với mọi n = 3 · 2m , m > 2, điều phải
chứng minh. 
Bài toán 7
Cho dãy (An ) xác định bởi
n n n
An = + 2 + ··· + 2 .
n2 +1 2 n +2 2 n + n2
   1
π
Tìm lim n n − An − .
n→∞ 4 4

Yong Xi Wang, China

Lời giải. Brian Bradie, Christopher Newport University, Newport News, VA, USA
Xét f là hàm số có đạo hàm cấp 3 trên đoạn [0, 1] bất kỳ. Gọi [a, b] ⊂ [0, 1]. bất kỳ. Sử dụng
khai triển Taylor, ta có
1 1
f (x) = f (b) + f 0 (b)(x − b) + f 00 (b)(x − b)2 + f 000 (ξ)(x − b)3 ,
2 6
với ξ nào đó nằm giữa x và b. Suy ra
Z b
1 1
f (x)dx = f (b)(b − a) − f 0 (b)(b − a)2 + f 00 (b)(b − a)3 + O (b − a)4 .

a 2 6
Xét n là số nguyên dương. Với mọi k = 0, 1, 2, . . . , n − 1, ta có
Z k/n        
1 k 1 0 k 1 00 k 1
f (x)dx = f − 2f + 3f +O .
(k−1)/n n n 2n n 6n n n4

Cho k chạy từ 0 đến n − 1 rồi lấy tổng n đẳng thức trên, ta được
Z 1 n   n   n    
1X k 1 X 0 k 1 X 00 k 1
f (x)dx = f − 2 f + 3 f +O .
0 n k=1 n 2n k=1 n 6n k=1 n n3

LATEX by Mathpiad 6
TẠP CHÍ VÀ TƯ LIỆU TOÁN HỌC

Điều này tương đương với


Z 1 n   n    
0 1X 0 k 1 X 00 k 1
f (1) − f (0) = f (x)dx = f − 2 f +O
0 n k=1 n 2n k=1 n n2

và n
Z 1    
0 0 00 1 X 00 k 1
f (1) − f (0) = f (x)dx = f +O .
0 n k=1 n n
Kết hợp 3 đẳng thức trên, ta được
Z 1 n    
1X k 1 1 0 0 1
f (x)dx = f − [f (1) − f (0)] − 2
[f (1) − f (0)] + O .
0 n k=1 n 2n 12n n3

1
Xét f (x) = . Khi đó,
1 + x2
Z 1
π
f (x)dx = ;
0 4
n   n n
1X k 1X 1 X n
f = 2
= = An ;
n k=1 n n k=1
1 + (k/n) k=1
n + k2
2

1 1
f (1) − f (0) = − 1 = − ;
2 2
1 1
f 0 (1) − f 0 (0) = − − 0 = −
2 2
Do vậy,  
π 1 1 1
= An + + +O .
4 4n 24n2 n3
Suy ra  
π  1 1
lim n n − An − = .
n→∞ 4 4 24

Bài toán 8
Cho x > 1, xét dãy (an )n>1 thỏa mãn an = [xn ] với mọi n ∈ Z+ . Chứng minh rằng, nếu
(an )n>1 là cấp số nhân thì x là một số nguyên.

Marius Cavachi, Constanta, Romania

Lời giải. Robert Bosch, Archimedean Academy, Florida, USA


Gọi k là công bội của cấp số nhân trên. Ta có
[xn ] = k n−1 [x]
Chia hai vế cho xn , ta được  n−1
[xn ] k [x]
=
xn x x
xn − 1 [xn ] [xn ]
Nhận thấy, < 6 1 nên lim lim = 1. Lấy giới hạn hai vế khi n → ∞, ta được
xn xn x→∞ xn
k = x, do đó x = [x]. Hay x là số nguyên, điều phải chứng minh.


LATEX by Mathpiad 7
TUYỂN TẬP DÃY SỐ TRONG 10 NĂM TỪ CÁC TẠP CHÍ TOÁN CỦA MỸ

Bài toán 9
Cho a0 > 0 và an+1 = a0 · . . . · an + 4 với mọi n > 0. Chứng minh rằng
p
an − 4 (an+1 + 1) (a2n + 1) − 4 = 1

với mọi n > 1.

Titu Andreescu, University of Texas at Dallas

Lời giải. AN-anduud Problem Solving Group, Ulaanbaatar, Mongolia


Ta có

an+1 = a0 · . . . · an−1 an + 4
⇒ an+1 = (an − 4) an + 4
⇒ an+1 + 1 = a2n − 4an + 5
⇒ (an+1 + 1) a2n + 1 − 4 = a2n − 4an + 5 a2n + 1 − 4
  

⇒ (an+1 + 1) a2n + 1 − 4 = a4n − 4a3n + 6a2n − 4an + 1




⇒ (an+1 + 1) a2n + 1 − 4 = (an − 1)4



p
⇒ (an+1 + 1) (a2n + 1) − 4 = an − 1
p
⇒ an − 4 (an+1 + 1) (a2n + 1) − 4 = 1,

Điều phải chứng minh. 


Bài toán 10
Cho (an )n>0 là một dãy số thực thỏa mãn a0 = 1 và
an
an+1 =
n2 a 2
n + an + 1

Tìm lim n3 an .
x→∞

Khakimboy Egamberganov, Tashkent, Uzbekistan

Lời giải. Alessandro Ventullo, Milan, Italy



an 1 X
Nhận thấy an+1 < 2 = 2 , do vậy an hội tụ theo Tiêu chuẩn so sánh. Đặt
n an n n=0


X
an = c ∈ R
n=0

Ta có

1 1
= n2 + an +
an+1 an

LATEX by Mathpiad 8
TẠP CHÍ VÀ TƯ LIỆU TOÁN HỌC

Nên
1 1
= 02 + a0 +
a1 a0
1 1
= 12 + a1 +
a2 a1
......
...
1 1
= (n − 1)2 + an−1 +
an an−1
Cộng vế theo vế các đẳng thức trên, ta được
n−1
1 (n − 1)n(2n − 1) X
= + ak + 1
an 6 k=0

1 1
= (n − 1)2 + an−1 +
an an−1
Suy ra,
n−1
X
ak + 1
1 (n − 1)n(2n − 1) k=0
= +
n 3 an 6n3 n3

n−1
X
ak + 1
k=0 c+1
lim = lim =0
n→∞ n3 n→∞ n3

Nên
1 (n − 1)n(2n − 1) 1
lim = lim =
n→∞ n3 a n n→∞ 6n 3 3
Do vậy,
lim n3 an = 3
n→∞


Bài toán 11
Cho (an )n>1 là một dãy số thực dương đơn điệu tăng thỏa mãn lim an = ∞ và dãy
n→∞
(an+1 − an )n>1 đơn điệu. Tính
a1 + . . . + an
lim √ .
n→∞ n an

Mihai Piticari and Sorin Rădulescu, Romania

Lời giải. Paolo Perfetti, Universitá degli studi di Tor Vergata Roma, Roma, Italy
Dãy (an+1 − an )n>1 đơn điệu nên sẽ hội tụ đến L. Ta xét L > 0 hữu hạn. Sử dụng định lý
Trung bình Cesaro
an
L = lim (an+1 − an ) =⇒ L = lim
n→∞ n→∞ n


√ √ an+1 − an L
lim ( an+1 − an ) = lim √ √ = =0
n→∞ n→∞ an+1 + an ∞

LATEX by Mathpiad 9
TUYỂN TẬP DÃY SỐ TRONG 10 NĂM TỪ CÁC TẠP CHÍ TOÁN CỦA MỸ

Áp dụng định lý Stolz

(a1 + . . . + an+1 ) − (a1 + . . . + an ) an+1


lim √ √ = lim √ √
n→∞ (n + 1) an+1 − n an n→∞ (n + 1) an+1 − n an

Ta có √ √ √ 
(n + 1) an+1 − n an

n+1 √ √ an
lim = lim ( an+1 − an ) +
n→∞ an+1 n→∞ an+1 an+1
và √
n+1 √ √ 1 an
lim ( an+1 − an ) = · 0 = 0, lim =0
n→∞ an+1 L n→∞ an+1

an+1
Mà (an ) tăng và an > 0 nên √ √ > 0. Do đó,
(n + 1) an+1 − n an
an+1
lim √ √ = ∞.
n→∞ (n + 1) an+1 − n an

an
Bây giờ ta xét L = lim (an+1 − an ) = 0, khi đó 0 = lim . Ta có
n→∞ n→∞ n

(a1 + . . . + an+1 ) − (a1 + . . . + an ) an+1


lim √ √ = lim √ √
n→∞ (n + 1) an+1 − n an n→∞ (n + 1) an+1 − n an
a
= lim  √ n+1 √
an+1 − an

n→∞ √
an+1 n √ +1
an+1
√ √
an+1 − an
Bây giờ ta chứng minh dãy n √ bị chặn trên. Ta viết
an+1

n an+1 − an n (an+1 − an )
√ √ √ 6
an+1 an+1 + an 2an

n (an+1 − an )
Do đó, ta đi chứng minh dãy bị chặn trên.
2an 
np anp +1 − anp p
Phản chứng, giả sử với mọi số nguyên dương, p tồn tại np thỏa mãn > hay
2anp 2
panp
anp +1 − anp > . Do dãy an+1 − an đơn điệu nên
np
panp
an+1 − an > anp +1 − anp > ∀1 6 n 6 np − 1
np

Dẫn đến
np
X panp
a1 = an p + (ak − ak+1 ) < anp − np = np (1 − p) < 0,
k=1
np

n (an+1 − an )
mâu thuẫn với giả thiết. Do đó, tồn tại số thực dương C sao cho 0 < 6 C. Dẫn
2an
đến
a an+1
lim  √ n+1 √ >√ =∞
n→∞ √ an+1 − an an+1 (C + 1)
an+1 n √ +1
an+1

LATEX by Mathpiad 10
TẠP CHÍ VÀ TƯ LIỆU TOÁN HỌC

Cuối cùng, ta xét L = ∞. Suy ra, với mỗi p > 0, tồn tại np thỏa mãn với mọi n > np thì
an+1 − an > p. Dẫn đến
np −1 n
X X
an = a1 + (ak+1 − ak ) + (ak+1 − ak ) >
k=1 k=np

> a1 + (np − 1) min (ak+1 − ak ) + p (n − np + 1)


16k6np −1
n
> p (n − np + 1) > p
2
với n > 2 (np − 1). Từ đây ta có
 n
a1 + . . . + a2np −3 + a2np −2 + . . . + an > (2np − 3) a1 + (n − 2np + 3) p
2
2
n n
> (n − 2np + 3) p > p
2 4
với mọi n > 4np + 6. Do đó,
√ √ √
a1 + . . . + an a1 + . . . + an a1 + . . . + an a1 + . . . + an √ n 1 √
√ = √ > > p = p
n an n an n 2n

Cho p → ∞, ta được, với L = ∞ thì


a1 + . . . + an
lim √ = ∞.
n→∞ n an

Vậy
a1 + . . . + an
lim √ = ∞.
n→∞ n an

Bài toán 12
1 2 n
Chứng minh rằng dãy 22 + 1, 22 + 1, . . . , 22 + 1, . . . và một cấp số cộng đơn điệu tăng
bất kỳ hoặc cùng có chung vô số phần tử, hoặc chỉ có chung không quá một phần tử.

Nairi Sedrakyan, Yerevan, Armenia

Lời giải. Adnan Ali, Student in A.E.C.S-4, Mumbai, India


Phản chứng, giả sử tồn tại cấp số cộng {an }n>1 với công sai d > 0, có đúng a phần tử thuộc
1 2 n
dãy {Fn }n>1 = 22 + 1, 22 + 1, · · · , 22 + 1, · · · , 2 6 a < ∞.
j m
Giả sử 2 phần tử giống nhau cuối cùng là ak = 22 + 1 và a` = 22 + 1 với k < ` ⇔ j < m.
Ta có,  m j 
j
d | a` − ak ⇔ d | 22 22 −2 − 1

Nếu d = 2b , b 6 2j , khi đó
m
 m+1 −2m

d | 22 22 − 1 ⇔ d | Fm+1 − Fm

Do đó, ∃ u ∈ Z+ mà a`+u = Fm+1 , mâu thuẫn.


Do vậy, d có một ước lẻ la d0 > 1. Khi đó,
m j
 m j 2m−j
22 −2 ≡ 1 (modd0 ) ⇒ 22 −2 ≡ 1 (modd0 ) .

LATEX by Mathpiad 11
TUYỂN TẬP DÃY SỐ TRONG 10 NĂM TỪ CÁC TẠP CHÍ TOÁN CỦA MỸ

Dẫn đến  
m 2m−j −2m
d0 | 22 22 − 1 ⇔ d | F2m−j − Fm .

Suy ra, tồn tại số nguyên dương r thỏa mãn a`+r = F2m−j , mâu thuẫn.
Vậy giả sử trên là sai, ta có điều phải chứng minh. 
Bài toán 13
1 2
Cho {an } là dãy số thực xác định bởi a0 = 3 và an+1 = (a + 1) , ∀n > 0.
2 n
Chứng minh rằng  2
v 
u n
u
X t k Fk Fn−k 
 n u 
1 + 5  6 2n−1 Ln .

 k=0 1 + a k

Trong đó,

• Ln là số Lucas thứ n, tức số hạng thứ n của dãy L0 = 2, L1 = 1 và Ln = Ln−1 +Ln−2 .

• Fn là số F ibonacci thứ n, tức số hạng thứ n của dãy F0 = 0, F1 = 1 và

Fn = Fn−1 + Fn−2

Jose Luis Díaz-Barrero, Barcelona, Spain và Angel Plaza, Universidad de Las


Palmas de Gran Canaria, Spain.

Lời giải. Henry Ricardo, New York Math Circle.


Ta sẽ lần lượt chứng minh hai kết quả sau đây
n   n
X n 2n Ln − 2 X 1 1
Fk Fn−k = , 6 .
k=0
k 5 k=0
1 + ak 2

Một điều hiển nhiên, ta chứng minh được các công thức tổng quát của hai dãy F ibonacci và
Lucas, đó là
αn − β n
Fn = √ , Ln = αn + β n ,
5
√ √
1+ 5 1− 5
ở đây α = và β = . Bằng cách này, đẳng thức đầu tiên được chứng minh theo
2 2
khai tiển nhị thức N ewton. Kết quả thứ hai thu được dựa vào việc quan sát thấy {an } là một
dãy tăng, và
n n  
X 1 X 1 1 1 1 1
= − = − 6
k=0
1 + ak k=0
ak − 1 ak+1 − 1 a0 − 1 an+1 − 1 2

Ap dụng các kết quả thu được và bất đẳng thức Cauchy − Schwarz, ta có
 v  2
u n
u
!2
X t k Fk Fn−k 
 n u s 
 n r
X n 1
1 + 5  =1+5 Fk Fn−k

 k=0 1 + a k

 k=0
k 1 + ak

LATEX by Mathpiad 12
TẠP CHÍ VÀ TƯ LIỆU TOÁN HỌC

n  
! n
!
X n X 1
61+5 Fk Fn−k
k=0
k k=0
1 + ak
1
6 1 + (2n Ln − 2) = 2n−1 Ln .
2
Như vậy, bài toán đã cho được chứng minh. 
Bài toán 14
Cho x1 , x2 , . . . là một dãy các số thực dương thỏa mãn đồng thời hai điều kiện
log xn
lim xn = 0 và lim là số âm.
n→∞ n→∞ x1 + · · · + xn

Chứng minh rằng


log xn
lim = −1.
n→∞ log n

by George Stoica, University of New Brunswick, Saint John, Canada.

Lời giải. Lixing Han, University of Michigan, Flint, MI.


log xn
Ta đặt β = . Áp dụng định lý Stolz − Cesaro, ta có
x1 + · · · + xn
 
xn+1
log
xn log xn+1 − log xn log xn
lim = lim = lim = β. (1)
n→∞ xn+1 n→∞ xn+1 n→∞ x1 + · · · + xn

Kết quả trên cho ta biết xn+1 < xn với số nguyên dương 
n đủ lớn
 thỏa mãn xn > 0. Do xn tiến
xn+1
tới 0 khi n → ∞, từ (1) ta suy ra lim βxn+1 = lim log = 0, và điều này dẫn đến
n→∞ n→∞ xn
xn+1
lim =1
n→∞ xn

Theo như định lý giá trị trung bình, tồn tại dãy {ζn } thỏa mãn
1
log xn+1 − log xn = (xn+1 − xn ) , (2)
ζn
xn+1
ở đây ζn nằm giữa xn và xn+1 . Theo đó, với n đủ lớn, ta có < ζn /xn < 1. Từ (2), ta chứng
xn
ζn
minh được lim = 1. Sử dụng nhận định này kết hợp với (1), ta lại có
n→∞ xn

log xn+1 − log xn 1 xn+1 − xn xn xn+1 − xn


lim = lim · = lim · =β
n→∞ xn+1 n→∞ ζn xn+1 n→∞ ζn xn xn+1
Theo đó, với bất kì số  ∈ (0, |β|) nào, luôn tồn tại số nguyên dương N sao cho
1 1
β−< − < β +  , ∀n > N.
xn xn+1
Cộng theo vế các bất đẳng thức trên từ n = N tới N + m − 1, ta nhận được
1 1
(β − )m < − < (β + )m
xN xN +m

LATEX by Mathpiad 13
TUYỂN TẬP DÃY SỐ TRONG 10 NĂM TỪ CÁC TẠP CHÍ TOÁN CỦA MỸ

Chia các vế cho N + m và cho m → ∞, ta có


 
1 1
β −  6 lim − 6β+
m→∞ (N + m)xN (N + m)xN +m
1
Lại do lim = 0 nên
m→∞ (N + m)xN
1 1
6 − lim (N + m)xN +m = − lim nxn 6
β+ m→∞ n→∞ β−
−1
Khi cho  tiến đến 0, ta có thể suy ra lim nxn = > 0, vậy nên
n→∞ β
 
1
lim log (nxn ) = lim (log n + log xn ) = log −
n→∞ n→∞ β
Tuy nhiên, nếu đánh giá trên xảy ra, ta bắt buộc phải có
 
log n + log xn log − β1
lim = lim =0
n→∞ log n n→∞ log n
và vì vậy  
log xn
lim 1+ = 0.
n→∞ log n
Bài toán đã cho được chứng minh 
Bài toán 15
Cho các số thực dương a, b, c thỏa mãn b2 > 4ac. Cho dãy {λn } thỏa mãn λ0 > 0 và
cλ1 > bλ0 . Xét dãy {un } xác định bởi
(
u0 = cλ0 , u1 = cλ1 − bλ0
un = aλn−2 − bλn−1 + cλn , ∀n > 2

Chứng minh rằng, nếu un > 0 với mọi n > 0 thì λn > 0 với mọi n > 0.

H. A. ShahAli, Tehran, Iran.

Lời giải. J. C. Linders, Eindhoven, The Netherlands.


Do u0 > 0 và u1 > 0 nên cả λ0 và λ1 đều dương. Ta sẽ chứng minh bằng quy nạp với n rằng
n+1
cλn > bλn−1 ∀n > 1 và λn > 0, ∀n > 1.
2n
Hiển nhiên, cả hai bất đẳng thức trên đúng với n = 1. Trong trường hợp tổng quát, giả thiết
quy nạp cho ta
 
2n c 2nac
cλn+1 > bλn − aλn−1 > bλn − a · λn = 1 − 2 bλn
n+1 b b (n + 1)
 
n n+2
> 1− bλn = bλn .
2(n + 1) 2(n + 1)
Cả hai bất đẳng thức bên trên được chứng minh theo nguyên lí quy nạp. Bài toán được giải
quyết. 

LATEX by Mathpiad 14
TẠP CHÍ VÀ TƯ LIỆU TOÁN HỌC

Bài toán 16
p
Cho 0 6 a 6 2, dãy số han i được xác định bởi a1 = a và an+1 = 2n − 2n (2n − an ) với

X
n > 1. Tính a2n .
n=1

Pál Péter Dályay, Szeged, Hungary.

Lời giải. Hansruedi Widmer, Kantonsschule Baden, Baden, Switzerland.


Đầu tiên ta viết lại công thức thành
a2n+1 = 2n+1 an+1 − 2n an .
Thế vào biểu thức ta được
n
X
a2l = a21 − 2a1 + 2n an = a2 − 2a + 2n an
l=1

Ta cần tính lim 2n an .


n→∞
π p 1
Với 0 6 γ 6 là góc sao cho a1 = a = 2 sin2 γ, suy ra γ = arcsin a/2 = arccos(1 − a). Do
2 2
đó theo phép quy nạp ta được an = 2n sin2 (γ/2n−1 ) . Vì thế nên
 γ 
n 2n 2
lim 2 an = lim 2 sin n−1
= 4γ 2 = arccos2 (1 − a)
n→∞ n→∞ 2
X ∞
Do đó ta thu được a2n = a2 − 2a + arccos2 (1 − a). Bài toán kết thúc 
n=1

Bài toán 17
Cho {an } được xác định là dãy tuần hoàn cho bởi a1 = a3 = 1, a2 = 2, a4 = a6 = −1,
a5 = −2, và an = an−6 với n > 7. Gọi {Fn } là dãy Fibonacci với F1 = F2 = 1. Chứng
minh

! ∞ !
X ak Fk F2k−1 X (−1)kn π
=
k=1
2k − 1 F F
n=0 kn+2k−1 kn+3k−1
4

Bruce Burdick, Roger Williams University, Bristol, RI.

Lời giải. Roberto Tauraso, Universita di Roma “Tor Vergata”, Roma, Italy; Rituraj Nandan, St.
Peters, MO; và tác giả
FN FM +1 − FN +1 FM = (−1)M FN −M . Với M = kn + 2k − 1 và N = k(n + 1) + 2k − 1, ta có:
(−1)kn −Fk(n+1)+2k−1 Fkn+2k + Fk(n+1)+2k Fkn+2k−1
=
Fkn+2k−1 Fkn+3k−1 Fk Fkn+2k−1 Fk(n+1)+2k−1
 
1 Fk(n+1)+2k Fkn+2k
= −
Fk Fk(n+1)+2k−1 Fkn+2k−1
Do đó: ∞ ∞ 
(−1)kn

X 1 X Fk(n+1)+2k Fkn+2k
= −
n=0
Fkn+2k−1 Fkn+3k−1 Fk n=0 Fk(n+1)+2k−1 Fkn+2k−1
 
1 F2k
= φ−
Fk F2k−1

LATEX by Mathpiad 15
TUYỂN TẬP DÃY SỐ TRONG 10 NĂM TỪ CÁC TẠP CHÍ TOÁN CỦA MỸ


1+ 5
trong đó φ = . Khi đó:
2
∞ ∞ ∞
X ak Fk F2k−1 X (−1)kn X ak (φF2k−1 − F2k )
=
k=1
2k − 1 n=0
Fkn+2k−1 Fkn+3k−1 k=1
2k − 1
∞  2k−1
X ak 1
=
k=1
2k − 1 φ

vì Fn+1 − φFn = −φ−n . Chuỗi 1, 2, 1, −1, −2, −1, . . . là tổng của hai chuỗi có chu kỳ là
1, −1, 1, −1, 1, −1 và 0, 3, 0, 0, −3, 0. Tiếp tục tính ta được:
∞  2k−1 X ∞  2k−1 X ∞  6k−3
X ak 1 (−1)k+1 1 3(−1)k+1 1
= +
k=1
2k − 1 φ k=1
2k − 1 φ k=1
6k − 3 φ
∞  2k−1 X ∞  2k−1
X (−1)k+1 1 (−1)k+1 1
= +
k=1
2k − 1 φ k=1
2k − 1 φ3
   
1 1
= arctan + arctan
φ φ3

Áp dụng công thức cộng arctan và φ2 = φ + 1 bài toán trở thành:


 
1 1
 φ + φ3 
     3 
1 1 φ + φ
arctan + arctan = arctan   = arctan
φ φ3  1 1  φ4 − 1
1− · 3
φ φ
φ3 + φ
   3
φ +φ π
= arctan 3 2
= arctan 3
= arctan(1) =
φ +φ −1 φ +φ 4

Bài toán 18
Gọi p, a, và b là các số thực dương thỏa mãn a < b. Xét dãy hxn i được xác định bởi công
thức nxn+1 = (n + 1/p)xn và điều kiện x1 6= 0. Tính giới hạn:
xan + xan+1 + · · · + xbn
lim .
n→∞ nxan

Alina Sı̂ntămărian, Technical University of ClujNapoca, Cluj-Napoca, Romania.

Lời giải. Omran Kouba, H.I.A.S.T., Damascus, Syria.


b1+c − a1+c
Đáp án của bài toán là , với c = 1/p. Thật vậy, lời giải luôn đúng với mọi số thực
(1 + c)ac
1 n−1
không âm c thay thế , không chỉ là nghịch đảo của các số nguyên. Đặt yn = xn . Từ
p 1+c
nxn+1 − (n − 1)xn (n + c)xn − (n − 1)xn
yn+1 − yn = = = xn ,
1+c 1+c
ta thu được
bn bn
X X bnxbn+1 − (an − 1)xan
xk = (yk+1 − yk ) = ybn+1 − yan = .
k=an k=an
1+c

LATEX by Mathpiad 16
TẠP CHÍ VÀ TƯ LIỆU TOÁN HỌC

Vì vậy
bn
1 X b xbn+1 an − 1
xk = · −
nxan k=an 1 + c xan n(1 + c)

Do đó ta chỉ cần tìm lim zn , với zn = xbn+1 /xan .


bn bn 
Y xk+1 Y c
zn = = 1+ ,
k=an
x k
k=an
k

ta có  c bn  c bn   c
an − 1 Y k−1 Y c 1
zn = zn = 1+ 1− .
bn k=an
k k=an
k k
 c  c
 c 1 ∞
 c 1
= 1 + O (k −2 ), tích vô hạn k=1 1 +
Q
Do 1 + 1− 1− hội tụ. Do đó:
k k k k
 c bn   c
an − 1 Y c 1
lim zn = lim 1+ 1− =1
n→∞ bn n→∞
k=an
k k

Vậy lim zn = (b/a)c . Ta được:


n→∞

bn
1 X b a b1+c − a1+c
lim xk = lim zn − =
n→∞ nxan
k=an
1 + c n→∞ 1+c (1 + c)ac


Bài toán 19
1 2 
Cho dãy số hak i là dãy của các số thực dương được xác định bởi an = an−1 + 1 với
2
mọi n > 1, và a1 = 3. Chứng minh rằng
" n
! n
!# 21  
X ak X 1 1 a1 + an
6 √ .
k=1
1 + ak k=1
ak (1 + ak ) 4 a1 an

José Luis Díaz-Barrero and José Gibergans-Báguena, Universidad Politécnica de


Cataluña, Barcelona, Spain.

Lời giải. Jim Simons. Cheltenham, U.K.

Định nghĩa 1. Khi n = 9 thì V T > 1 mà V P > 1025 nên ta nhận thấy bất đẳng thức này
rất yếu. Đầu tiên ta chứng minh ak > 2k . Theo phép quy nạp, với n = 1, 2 thì a1 = 3 > 2 và
a2 = 5 > 4 đúng. Ta giả sử ak > 2k kết hợp với giả thiết ta được
1 2  1 2k
2 + 1 > 2k+1 (Đúng theo quy nạp)

ak+1 = ak + 1 >
2 2
k−1
32
(Ta dễ dàng có thể chứng minh bất đẳng thức mạnh hơn ak > 2k−1 −1 )
2
ak Pn ak
Vì < 1, nên k=1 < n.
(1 + ak ) (1 + ak )

LATEX by Mathpiad 17
TUYỂN TẬP DÃY SỐ TRONG 10 NĂM TỪ CÁC TẠP CHÍ TOÁN CỦA MỸ

1 1 1
Vì 1 + ak > 4 và ak > 2k , ta có < 2−k−2 và nk=1
P
< .
(ak (1 + ak )) r (ak (1 + ak )) 4
n
Kết hợp những chứng minh trên ta thu được V T < .
2
Với n > 8, ta có
  √ n r
1 a1 + an an 22 n
√ > √ > √ >
4 a1 an 4 3 4 3 2
Thử trực tiếp với n = 2, 3, 4, 5, 6, 7 ta thấy bất đẳng thức đúng.
Vậy bất đẳng thức được chứng minh. 
Bài toán 20
Cho số thực dương q 6= 1. Xét các dãy {un } và {λn } thỏa mãn công thức tổng quát
n n
q 2 − q− 2 n n
vn = 1
− 12
và µn = q 2 + q − 2
q −q
2

a) Chứng minh 2vn+1 = µ1 vn + µn với mọi n > 1.

b) Chứng minh 2vn−1 = µ1 vn − µn với mọi n > 2.

c) Rút gọn tổng


N
X 1
n=2
vn vn+1

Proposed by Boon Wee Ong, Behrend College, Erie, PA.

Lời giải. M. Bello, M. Benito, Ó. Ciaurri, E. Fernández, and L. Roncal, Logro no, Spain.

a) Ta nhận thấy rằng

2 q (n+1)/2 − q −(n+1)/2 − q n/2 + q −n/2 q 1/2 − q −1/2


  
2vn+1 − µn =
q 1/2 − q −1/2
q (n+1)/2 − q −(n+1)/2 + q (n−1)/2 − q −(n−1)/2
=
q 1/2 − q −1/2
q n/2 q 1/2 + q −1/2 − q −n/2 q 1/2 + q −1/2
 
= = µ1 vn .
q 1/2 − q −1/2

Dựa vào đây, đẳng thức đã cho được chứng minh.

b) Ta tiếp tục nhận thấy

2 q (n−1)/2 − q −(n−1)/2 + q n/2 + q −n/2 q 1/2 − q −1/2


  
2vn−1 + µn =
q 1/2 − q −1/2
q (n−1)/2 − q −(n−1)/2 + q (n+1)/2 − q −(n+1)/2
= = µ1 v n .
q 1/2 − q −1/2

Ở đây, đẳng thức cuối cùng được chứng minh dựa theo kết quả ý a, và đẳng thức đã cho,
theo đó, cũng được chứng minh.

LATEX by Mathpiad 18
TẠP CHÍ VÀ TƯ LIỆU TOÁN HỌC

c) Xuất phát từ
2 (q − q −1 )
vn+1 µn−1 − µn+1 vn−1 = = 2µ1 ,
q 1/2 − q −1/2
và kết hợp với kết quả ở ý a, ta có
µ1 vn−1 + µn−1 µ1 vn+1 + µn+1
vn+1 vn − vn−1 vn+2 = vn+1 − vn−1
2 2
vn+1 µn−1 − vn−1 µn+1
= = µ1 .
2

Biến đổi trên chỉ ra cho ta


µ1 vn+1 vn − vn−1 vn+2 vn vn−1
= = − .
vn+1 vn+2 vn+1 vn+2 vn+2 vn+1

Với việc v0 = 1, ta biết được rằng


N N    
X 1 1 X vn−1 vn−2 1 vN −1 v0 vN −1
= − = − = .
n=2
vn vn+1 µ1 n=2 vn+1 vn µ1 vN +1 v2 µ1 vN +1

Tổng đã cho được rút gọn.


Bài toán 21
Cho các số thực dương a, b, c thỏa mãn b2 > 4ac. Cho dãy {λn } xác định bởi λ0 > 0 và
cλ1 > bλ0 . Xét dãy {un } xác định bởi
(
u0 = cλ0 , u1 = cλ1 − bλ0
un = aλn−2 − bλn−1 + cλn , ∀n > 2

Chứng minh rằng, nếu un > 0 với mọi n > 0 thì λn > 0 với mọi n > 0.

H. A. ShahAli, Tehran, Iran.

Lời giải. J. C. Linders, Eindhoven, The Netherlands.


Do u0 > 0 và u1 > 0 nên cả λ0 và λ1 đều dương. Ta sẽ chứng minh bằng quy nạp với n rằng

n+1
cλn > bλn−1 ∀n > 1 và λn > 0, ∀n > 1.
2n

Hiển nhiên, cả hai bất đẳng thức trên đúng với n = 1. Trong trường hợp tổng quát, giả thiết
quy nạp cho ta
 
2n c 2nac
cλn+1 > bλn − aλn−1 > bλn − a · λn = 1 − 2 bλn
n+1 b b (n + 1)
 
n n+2
> 1− bλn = bλn .
2(n + 1) 2(n + 1)

Cả hai bất đẳng thức bên trên được chứng minh theo nguyên lí quy nạp. Bài toán được giải
quyết. 

LATEX by Mathpiad 19
TUYỂN TẬP DÃY SỐ TRONG 10 NĂM TỪ CÁC TẠP CHÍ TOÁN CỦA MỸ

Bài toán 22
Cho số nguyên dương k và dãy (an ) được xác định bởi a0 = 0, a1 = 1 và an+1 = kan +an−1
với mọi n > 1.

a) Chứng minh rằng nếu n, r và h là các số nguyên không âm thỏa mãn r + h 6 n thì
an+r an+r+h + (−1)h+1 an−r−h an−r = a2n a2r+h .

b) Chứng tỏ rằng nếu i và j là các số nguyên dương và i > j thì


j−1 
X ai aj+1 , nếu j lẻ;
k ai−r aj−r =
ai aj+1 − ai−j , nếu j chẵn.
r=0

Sergio Falcón and Ángel Plaza, University of las Palmas de Gran Canaria, Las
Palmas de Gran Canaria, Spain.

Lời giải. J. C. Linders, Eindhoven, The Netherlands.

√  √ 
a) Đặt φ = k + k 2 + 4 /2 và ω = k − k 2 + 4 /2 là các nghiệm của phương trình x2 −
kx − 1 = 0.
Dựa vào phương trình đặc trưng ta dễ dàng thấy rằng an = (φn − ω n ) /(φ − ω) với mọi
n > 0. Vì thế
φp − ω p φq − ω q φp+q − φp ω q − ω p φq + ω p+q
ap aq = · = ,
φ−ω φ−ω (φ − ω)2
với mọi p, q > 0. Lại có φω = −1 nên suy ra
φp+q + ω p+q − (−1)q (φp−q − ω p−q )
ap aq = . (1)
(φ − ω)2

Điều này dẫn tới


φp+q+2s + ω p+q+2s − (−1)s (φp+q + ω p+q )
ap+s aq+s − (−1)s ap aq =
(φ − ω)2
Áp dụng (1) thì với p, q, s > 0, ta có
ap+s aq+s − (−1)s ap aq = ap+q+s as (2)
Từ đây sẽ có điều phải chứng minh khi thay p = n − r − h, q = n − r, và s = 2r + h.
b) Thay q = 1 vào (2) ta có ap+s as+1 − (−1)s ap = ap+s+1 as . Từ đây rút ra được
kap+s as = (ap+s+1 − ap+s−1 ) as = ap+s as+1 − (−1)s ap − ap+s−1 as .
Vậy với mọi số nguyên dương j và số nguyên không âm p, ta có
j j j
X X X
k ap+s as = (ap+s as+1 − ap+s−1 as ) − (−1)s ap
s=1 s=1 s=1
 j
X
s ap+j aj+1 , nếu j lẻ;
= ap+j aj+1 − ap a1 − ap (−1) =
ap+j aj+1 − ap , nếu j chẵn.
s=1

Từ đây thay p = i − j và s = j − r sẽ thu được đpcm.




LATEX by Mathpiad 20
TẠP CHÍ VÀ TƯ LIỆU TOÁN HỌC

Bài toán 23
Cho dãy (fn ) xác định bởi công thức
 n
1 1
fn = 1 + ((2n − 1)!!Ln ) n
n
b(n−1)/2c
Y
Tìm lim (fn+1 − fn ). Ở đây n!! = (n − 2j) và Ln là số hạng thứ n của dãy Lucas.
n→∞
j=0

D.M.Bătinetu-Giurgiu, “Matei Basarab” National College, Bucharest, Romania,


and Neculai Stanciu, “George Emil Palade” School, Buzău, Romania.

Lời giải. László Lipták, Oakland University, Rochester, MI.


Theo công thức Stirling, ta có
√ 1  n
(2n)! 2π(2n)2n+ 2 e−2n eO(1/n) √ 2n
(2n − 1)!! = n = √ 1 = 2 eO(1/n) .
2 n! n n+
2 2πn 2 e e −n O(1/n) e
√ 2n O(1/n2 )
Vì thế ((2n − 1)!!)1/n = 2n
2ee . Lại có

 
2n ln 2 ln 2 1
2=e 2n =1+ +O ,
2n n2

điều này cho ta   


1/n 2n ln 2 1
((2n − 1)!!) = 1+ +O .
e 2n n2
2
Tương tự, sử dụng ln(1 + x) = x − x2 + O (x3 ) khi x → 0, ta có
 n   
1 1
n ln(1+ n 1 1
) = e 2n ( n2 ) = e 1 −
1− +O 1 1
1+ =e +O .
n 2n n2

Cuối cùng, Ln = ϕn + (−ϕ)−n nên


  
  1/n
2 −n 1
L1/n
n = ϕ 1 + −ϕ =ϕ 1+O
n2

Vậy khi n → ∞, ta có
        
1 1 2n ln 2 1 1
fn = e 1 − +O 1 + + O ϕ 1 + O
2n n2 e 2n n2 n2
 
1
= 2nϕ + ϕ(ln 2 − 1) + O
n

Suy ra fn+1 − fn = 2ϕ + O n1 , vậy nên lim (fn+1 − fn ) = 2ϕ.




n→∞

Bài toán 24
(k 2 + 1)2
Cho dãy số ak = , k = 1, 2, 3, . . .. Chứng minh rằng với số nguyên dương n
k4 + 1

LATEX by Mathpiad 21
TUYỂN TẬP DÃY SỐ TRONG 10 NĂM TỪ CÁC TẠP CHÍ TOÁN CỦA MỸ

bất kỳ thì
2n+1
an1 an−1
2 an−2
3 · · · an = .
n2 + 2n + 2

Nguyen Viet Hung, Hanoi University of Science, Vietnam

Lời giải. Brian Bradie, Christopher Newport University, Newport News, VA, USA
Đặt ck = k 2 + 1 và dk = k 4 + 1. Khi đó

ck−1 ck+1 = (k − 1)2 + 1 (k + 1)2 + 1


  

= k 2 − 2k + 2 k 2 + 2k + 2 = k 4 + 4 = dk ,
 

cho nên
c2k
ak = .
ck−1 ck+1
Vì vậy với số nguyên dương n bất kì, ta có
n n
Y Y c2n+2−k
an1 an−1
2 an−2
3 · · · an = an+1−k
k = k
n+1−k n+1−k
k=1
c
k=1 k−1
ck+1
2n−(n−1) n
c1 Y c2n+2−2k
k 1
= · ·
cn0 c
k=2 k
n+2−k n−k c
ck n+1

cn+1
1 1
= n
· 1n−1 ·
c0 cn+1
n+1
c
= n1
c0 cn+1

Mà lại có c0 = 1, c1 = 2 và cn+1 = (n + 1)2 + 1 = n2 + 2n + 2, từ đây suy ra

2n+1
an1 an−1
2 a3n−2 · · · an = .
n2 + 2n + 2
Vậy ta có điều phải chứng minh. 
Bài toán 25
Kí hiệu (Fn ), n > 0 là dãy số Fibonacci. Chứng minh các hệ thức sau
F3n 2 2

(a) = 2 Fn−1 + Fn+1 − Fn−1 Fn+1
Fn
       
2n + 1 2n + 1 2n + 1 2n + 1
(b) F2n+1 + F2n−1 + F2n−3 + · · · + F1 = 5n .
0 1 2 n

Nguyen Viet Hung, Hanoi University of Science, Vietnam

Lời giải. Brian Bradie, Christopher Newport University, Newport News, VA, USA
Chúng ta đã biết rằng công thức tổng quát của dãy Fibonacci là
√ √
αn − β n 1+ 5 1− 5
Fn = √ , trong đó α = và β = .
5 2 2

LATEX by Mathpiad 22
TẠP CHÍ VÀ TƯ LIỆU TOÁN HỌC

Lưu ý rằng
1 √ 1 √
αβ = −1, α+ = 5, β+ = − 5.
α β

1 1
α2 + 2
= β 2 + 2 = α2 + β 2 = 3.
α β

(a) Sử dụng công thức Binet, ta có

F3n α3n − β 3n
= n n
= α2n + αn β n + β 2n = α2n + β 2n + (−1)n .
Fn α −β

Tiếp theo,
2 1 2n−2
+ β 2n−2 + 2(−1)n , và

Fn−1 = α
5
2 1 2n+2
+ β 2n+2 + 2(−1)n

Fn+1 = α
5
Vì vậy
   
2 2 2 2 1 2n 2 1 8
β + 2 β 2n + (−1)n
2

2 Fn−1 + Fn+1 = α + 2 α +
5 α 5 β 5
6 2n 6 2n 8
= α + β + (−1)n
5 5 5
Hơn nữa,
1 2n
α − αn − 1β n+1 − αn+1 β n−1 + β 2n

Fn−1 Fn+1 =
5
1 2n
α − αn−1 β n−1 α2 + β 2 + β 2n
 
=
5
1 1 3
= α2n + β 2n + (−1)n
5 5 5
Suy ra
2 2 F3n
− Fn−1 Fn+1 = α2n + β 2n + (−1)n =

2 Fn−1 + Fn+1
Fn

(b) Trước hết để ý,

2n+1
X  n   2n+1
X 2n + 1
2n + 1 X 2n + 1
F2n+1−2k = F2n+1−2k + F2n+1−2k
k=0
k k=0
k k=n+1
k
n   n  
X 2n + 1 X 2n + 1
= F2n+1−2k + F2k−(2n+1)
k=0
k k=0
2n + 1 − k
n  
X 2n + 1
=2 F2n+1−2k ,
k=0
k

ở đây chúng ta sử dụng đến


   
n+1 n n
F−n = (−1) Fn và =
k n−k

LATEX by Mathpiad 23
TUYỂN TẬP DÃY SỐ TRONG 10 NĂM TỪ CÁC TẠP CHÍ TOÁN CỦA MỸ

Vậy ta có
2n+1 2n+1 2n+1 
!
X 2n + 1

1 X 
2n + 1 2n+1−2k X 2n + 1 2n+1−2k

F2n+1−2k = √ α − β
k=0
k 5 k=0
k k=0
k
2n+1   2n+1
!
1 X 2n + 1 X 2n + 1
=√ α−(2n+1) α4n+2−2k − β −(2n+1) β 4n+2−2k
5 k=0
k k=0
k
" #
2 2n+1 2 2n+1
  
1 1+α 1+β
=√ −
5 α β
1 √ 2n+1 √ 2n+1 
=√ 5 + 5 = 2 · 5n
5

Vì vậy chúng ta có
n   2n+1  
X 2n + 1 1 X 2n + 1
F2n+1−2k = F2n+1−2k = 5n
k=0
k 2 k=0
k


Bài toán 26
Dãy số {an }n>1 được xác định bởi công thức sau a1 = 4, 3an+1 = (an + 1)3 − 5 với mọi
n > 1. Chứng minh rằng an là số nguyên dương với mọi n và tính tổng

X an − 1
.
a2
n=1 n
+ an + 1

Albert Stadler, Herrliberg, Switzerland

Lời giải. Daniel Lasaosa, Pamplona, Spain


Ta đặt an = 3bn + 1, khi đó b1 = 1 và với mọi n > 1,

bn+1 = 3bn (bn + 1)2 + bn .

Vì b1 là số nguyên dương nên bằng quy nạp ta dễ dàng chỉ ra rằng bn là số nguyên dương, từ
đó suy ra an luôn là số nguyên dương với mọi n. Hơn nữa,

an − 1 bn bn (bn + 1) bn+1 − bn 1 1
= 2 = = = − ,
a2n + an + 1 3bn + 3bn + 1 bn+1 + 1 3 (bn + 1) (bn+1 + 1) 3 (bn + 1) 3 (bn+1 + 1)

hay ta có
N
X an − 1 1 1 1 1
= − = − ,
a2
n=1 n
+ an + 1 3 (b1 + 1) 3 (bN +1 + 1) 6 3 (bN +1 + 1)

và vì bn tiến tới dương vô cùng khi n ra vô cùng (dễ dàng chứng minh từ công thức truy hồi).
Vậy ta có

X an − 1 1 1 1
2
= − lim = .
a + an + 1
n=1 n
6 N →∞ 3 (bN +1 + 1) 6


LATEX by Mathpiad 24
TẠP CHÍ VÀ TƯ LIỆU TOÁN HỌC

Bài toán 27
Cho dãy số

r r
3 3 n+1 n+1
xn = 2 + + ··· + , n = 1, 2, 3, . . .
2 n
xn
Tính limn→∞ .
n

Nguyen Viet Hung, Hanoi University of Science, Vietnam

Lời giải. Li Zhou, Polk State College, USA


Với mỗi 1 6 k 6 n, dựa vào định lý khai triển nhị thức
 k+1
1 1 k+1
1+ =1+ + ··· > .
k(k + 1) k k

Vì vậy, r
k+1 k+1 1 1 1
1< <1+ =1+ − ,
k k(k + 1) k k+1
1 xn
nên suy ra n < xn < n + 1 − . Từ đây dễ dàng đi đến kết luận lim = 1. 
n+1 n→∞ n

Bài toán 28
Cho (xn )n>1 là dãy số tăng và tồn tại số a > 2 thỏa mãn xn+1 > axn − (a − 1)xn−1 , với
mọi n > 1. Chứng minh rằng dãy (xn )n>1 phân kỳ.

Mihai Piticari and Sorin Rădulescu, România

Lời giải. Daniel Lasaosa, Pamplona, Spain


Ta có
xn+1 − xn−1 > a (xn − xn−1 ) > 2 (xn − xn−1 ) , xn+1 − xn > xn − xn−1 .
Đặt yn = xn+1 − xn thì rõ ràng dãy (yn )n>1 là dãy số dương tăng ngặt nên nó có giới hạn hoặc
phân kỳ. Trong cả hai trường hợp đều sẽ tồn tại số thực dương r và số nguyên dương N sao
cho với mọi n > N thì yn > r. Từ đây ta sẽ rút ra được rằng với mọi n > N thì xn > (n − N )r,
suy ra đpcm. 
Bài toán 29
Cho dãy (xn )n>1 là một dãy đơn điệu và số thực a ∈ (−1, 0). Tìm

lim x1 an−1 + x2 an−2 + · · · + xn .



n→∞

Mihai Piticari and Sorin Rădulescu, România

Lời giải. Daniel Lasaosa, Pamplona, Spain


Đặt yn là biểu thức mà chúng ta cần đi tìm giới hạn.

• Trường hợp 1. Nếu dãy (xn ) bị chặn, suy ra dãy có giới hạn là L. Xét  > 0 bất kì, do |a| < 1
(1 − a)
nên tồn tại số nguyên dương N1 sao cho với mọi n > N1 , có |an | < . Lại có dãy
3L

LATEX by Mathpiad 25
TUYỂN TẬP DÃY SỐ TRONG 10 NĂM TỪ CÁC TẠP CHÍ TOÁN CỦA MỸ

(1 − |a|)
(xn ) có giới hạn là L nên tồn tại M1 sao cho với mọi n > M1 thì |xn − L| < .
3
Mặt khác do (xn ) là dãy đơn điệu nên D = |x1 − L| > |xn − L|. Lại dựa vào |a|n tiến tới
(1 − |a|)
0 nên tồn tại M2 sao cho với mọi n > M2 thì |a|n+1 < . Lấy M = max {M1 , M2 }
3D
và N = max {N1 , M + 2}. Vậy ta có với n > N thì

L an L 
− L 1 + a + a2 + · · · + an−1 =

< ,
1−a 1−a 3


xn + axn−1 + a2 xn−2 + · · · + aM xn−M − L 1 + a + a2 + · · · + aM 6


M 1 − |a|M +1  
6 |xn − L| + |a| |xn−1 − L| + · · · + |a| |xn−M − L| < < .
3 3
Ta cũng có

aM +1 xn−M −1 + · · · + an−1 x1 − L aM +1 + · · · + an−1




6 |a|M +1 |xn−M −1 − L| + · · · + |a|n−M −2 |x1 − L|




1 − |a|n−M −1 

M +1 n−M −2
 
6 |a| D 1 + |a| + · · · + |a| < < .
3 3
Từ ba bất đẳng thức trên ta suy ra
L
yn − < .
1−a

Từ đây suy ra nếu (xn ) bị chặn và có giới hạn là L thì


L
lim x1 an−1 + x2 an−2 + · · · + xn =

.
n→∞ 1−a

• Trường hợp 2. Nếu dãy (xn ) tăng và không bị chặn thì với số N > 0 bất kỳ thì tồn tại số
M thỏa mãn xn > |x1 + x2 + · · · + xN −1 | với mọi n > M và do 0 < a2 < 1 nên với mọi
số chẵn n > M + 2 suy ra

x1 an−1 + x2 an−2 + · · · + xn > (1 + a) xn + a2 xn−2 + · · · + an−2 x2 > (1 + a)xn ,




và với mọi số lẻ n > M + 2, ta có

x1 an−1 + x2 an−2 + · · · + xn > (1 + a) xn + a2 xn−2 + · · · + an−2 x2 + an−1 x1




> (1 + a)xn + an−1 x1 ,

Dựa vào hai bất đẳng thức trên cho thấy biểu thức vế trái luôn tiến ra dương vô cùng
hay
lim x1 an−1 + x2 an−2 + · · · + xn = +∞

n→∞

• Trường hợp 3. Nếu dãy (xn ) giảm và không bị chặn thì ta làm tương tự trường hợp 2 để
suy ra
lim x1 an−1 + x2 an−2 + · · · + xn = −∞

n→∞

LATEX by Mathpiad 26
TẠP CHÍ VÀ TƯ LIỆU TOÁN HỌC

Bài toán 30
Tính giới hạn của dãy số được xác định bởi a1 = 1, a2 = 2, và
a2k−1 + a2k √
a2k+1 = và a2k+2 = a2k a2k+1
2
với k là một số nguyên dương.

Peter Kórus, University of Szeged, Szeged, Hungary.

Lời giải. Robin Chapman, University of Exeter, Exeter, UK.


Giả sử √ √
3 π 3 π
a2k−1 = k−1 cot k−1
và a2k = k−1 csc
2 3·2 2 3 · 2k−1
với mỗi số k nguyên dương. Khi k = 1,
√ √ √ √
3 π 3 π 3 3
cot = = 1 = a1 và csc = = 2 = a2
2k−1 3 · 2k−1 tan π/3 2k−1 3 · 2k−1 sin π/3

Giả sử các công thức đúng với mọi số k và để thuận tiện, ta đặt θ = π/ 3 · 2k . Ta có:
√   √ √
a2k−1 + a2k 3 cos 2θ + 1 3 2 cos2 θ 3
a2k+1 = = k = k = k cot θ
2 2 sin 2θ 2 2 sin θ cos θ 2

và √ r √ r √
√ 3 cos θ 3 cos θ 3
a2k+2 = a2k a2k+1 = k−1 = k−1 2 = k csc θ .
2 2 sin θ sin 2θ 2 4 sin θ cos θ 2
Sử dụng phương pháp quy nạp toán học cho mọi số k. Vì x → 0, csc x ∼ x−1 và cot x ∼ x−1 .
Do đó √ √
3 π 3 3
lim a2k−1 = lim k−1 cot =
k→∞ k→∞ 2 3 · 2k−1 π
và √ √
3 π 3 3
lim a2k = lim csc = .
k→∞ k→∞ 2k−1 3 · 2k−1 π
Vậy √
3 3
lim an =
n→∞ π

Bài toán 31
p2
Gọi p và q là các số thực với p > 0 và q > − . Cho U0 = 0, U1 = 1, và Un+2 = pUn+1 +qUn
4
với n > 0. Tính: v
u v s
u u r
u u q
lim U1 + U2 + U4 + · · · + U22n−1
2 2 2
t t
n→∞

Hideyuki Ohtsuka, Saitama, Japan.

LATEX by Mathpiad 27
TUYỂN TẬP DÃY SỐ TRONG 10 NĂM TỪ CÁC TẠP CHÍ TOÁN CỦA MỸ

Lời giải. Said Amghibech,


r Quebec City, QC, Canada.
2
p p
Giới hạn là + + q + 1. Gọi α và β là nghiệm của phương trình đặc trưng x2 − px − q = 0
2 4 r r
p p2 p p2
của hệ thức truy hồi Un , với α = + + q > 0 và β = − + q; là nghiệm thực vì
2 4 2 4
p2
q > − . Từ điều kiện đã cho Un = (αn − β n ) /(α − β) với mọi n. Gọi r và s là nghiệm của
4
x2 −x−1/(α −β)2 = 0 với r > 0 > s; Chú ý r +s = 1 và rs = −1/(α −β)2 . Xét vn = rαn +sβ n ,
với n > 0 ta có:

vn2 = r2 α2n + 2rsαn β n + s2 β 2n


= r2 − r α2n + 2rsαn β n + s2 − s β 2n + rα2n + sβ 2n
 

α2n − 2αn β n + β 2n
= + v2n = Un2 + v2n
(α − β)2

Vì α > |β| và s < 0 nên vn = αn + s p


(β n − αn ) > 0. Xem xn biểu diễn cho biểu thức có giới
hạn cần tìm. Sử dụng công thức vn = Un2 + v2n nhiều lần, ta được

q r q
v1 = U1 + v2 = U1 + U22 + v4
2 2

v v
u u s
u u r q
u
= · · · = U + U + U + · · · + U2
2 2 2
t
t
1 2 4 2n−1 + v2n > xn

Nhân liên hợp liên tiếp:


s r q p
v2 − U22 + U42 + · · · + U22n−1
v12 − x2n
v1 − xn = 6
v1 + xn v1
r
q r q
p p
v22 − U22 − U42 + · · · + U22n−1 v4 − U42 + · · · + U22n−1
=   6
√ v1 v2
q p
v1 v2 + U22 + · · · + U2n−1
n−1 n−1 2 n−1
− U22k Y U222
Y v2k 
v2n Yv2k+1
6 · · · 6 Qn−1 = v1 = v1 = v1 1− 2 .
k=0 v2k k=0
v22k k=0
v22k k=0
v2k

Từ đó ta được
n−1
Y U 2k

0 6 v1 − xn 6 v1 1 − 22
k=0
v2k

Do
 2
k k
U 2k α2 − β 2 1 1
lim 22 = lim 2 = 2 2
= <1
k→∞ v k
2
k→∞ (α − β)2 rα 2k + sβ 2k (α − β) r (α − β)2 r + 1
p
Từ định lý kẹp ta suy ra lim xn = v1 = rα + sβ = p/2 + p2 /4 + q + 1. 
n→∞

LATEX by Mathpiad 28
TẠP CHÍ VÀ TƯ LIỆU TOÁN HỌC

Bài toán 32
n
1X
Cho x1 thuộc khoảng (0, 1), và đặt xn+1 = ln (1 + xk ) với n > 1. Tính lim xn ln n.
n k=1 n→∞

Moubinoul Omarjee, Lyceé Henri IV, Paris, France.

Lời giải. Brian Bradie, Christopher Newport University, Newport News, VA.
Ta tính được giới hạn là 2. Vì 0 < ln(1 + x) < x với mọi x > 0, áp dụng quy nạp toán học cho
xn ∈ (0, 1) với mọi n. Từ
n
X n−1
X
nxn+1 = ln (1 + xk ) và (n − 1)xn = ln (1 + xk )
k=1 k=1

Ta có nxn+1 − (n − 1)xn = ln (1 + xn ), hay


ln (1 + xn ) − xn
xn+1 − xn = <0
n
Do đó {xn } là dãy giảm. Vì tất cả các dãy trên đều dương nên nó hội tụ về một giới hạn không
âm. L biểu diễn giới hạn của dãy số. Theo định lý Stolz-Cesàro,
n−1
X
ln (1 + xk )
k=1
L = lim xn = lim
n→∞ n→∞ n−1
n
X n−1
X
ln (1 + xk ) − ln (1 + xk )
k=1 k=1
= lim = lim ln (1 + xn ) = ln(1 + L)
n→∞ n − (n − 1) n→∞

Nghiệm duy nhất của phương trình L = ln(1 + L) là L = 0, do đó xn → 0. Mặt khác, ta lại
xn+1 n − 1 ln (1 + xn )
có nxn+1 − (n − 1)xn = ln (1 + xn ) vì = + và lim (ln (1 + xn )) /xn = 1,
xn n nxn n→∞
khi đó lim xn+1 /xn = 1. Theo định lý Stolz-Cesàro,
n→∞

ln n ln(n + 1) − ln n
lim xn ln n = lim = lim
n→∞ n→∞ 1/xn n→∞ 1 1

xn+1 xn
   n
1 1
xn+1 xn ln 1 + xn+1 xn ln 1 +
n n
= lim = lim
n→∞ xn − xn+1 n→∞ x − ln (1 + xn )
2
 n n
xn+1 xn 1
= lim · · ln 1 + =1·2·1=2
n→∞ xn xn − ln (1 + xn ) n

Bài toán 33
xn+1 x2n
Cho dãy số (xn )n>1 gồm các số thực thỏa mãn → 1 do n → ∞ và a 6 6 b với
xn xn
a và b là các số thực dương và n đủ lớn. Chứng minh:

LATEX by Mathpiad 29
TUYỂN TẬP DÃY SỐ TRONG 10 NĂM TỪ CÁC TẠP CHÍ TOÁN CỦA MỸ


X 1
(a) xn < ∞ nếu b <
n=1
2

X 1
(b) xn = ∞ nếu a >
n=1
2

George Stoica, University of New Brunswick, New Brunswick, Canada.

Lời giải. Missouri State Problem Solving Group (Cách 1).


Angel Plaza, Universidad de Las Palmas de Gran Canaria, Spain (Cách 2).
xn+1
Vì > 0 với n đủ lớn, ta thấy xn không đổi. Tuy nhiên, (a) không đáng kể và (b) không
xn
tồn tại với xk âm, do đó giả sử với mỗi xk > 0
1 1 xn+1
(a) Cho b < 1/2. ta có > 1, và tồn tại một số c sao cho 1 < c < . Do → 1, ta có
2b 2b xn
xn+1 < cxn với n đủ lớn. Không mất tính tổng quát, giả sử
xn+1 < cxn và x2n 6 bxn với mọi n .
Chú ý với r = b(1 + c),  
1
0<r <b 1+ = b + 1/2 < 1
2b
với mỗi n > 0 ta xác định được
yn = x2n + x2n +1 + x2n +2 + · · · + x2n+1 −1
Khi đó
yn+1 = (x2n+1 + x2n+1 +1 ) + (x2n+1 +2 + x2n+1 +3 ) + · · · + (x2n+2 −2 + x2n+2 −1 )
< (1 + c) (x2n+1 + x2n+1 +2 + x2n+1 +4 + · · · + x2n+2 −2 )
6 b(1 + c) (x2n + x2n +1 + x2n +2 + · · · + x2n+1 −1 )
= ryn

Với mọi n, ta có yn 6 rn−1 y1 . Khi đó:



X ∞
X ∞
X
xn = yn 6 y1 rn−1 < ∞
n=1 n=0 n=0

1 1
(b) Cho a > 1/2. khi đó < 1, do vậy tồn tại một số c sao cho < c < 1. Lưu ý r = a(1 + c)
2a 2a
 
1
r >a 1+ = a + 1/2 > 1
2a
Trong phần này,lập luận tương tự, giả sử xn+1 > cxn và x2n > axn với mọi n. Lập luận với yn
như trên,
yn+1 = (x2n+1 + x2n+1 +1 ) + (x2n+1 +2 + x2n+1 +3 ) + · · · + (x2n+2 −2 + x2n+2 −1 )
> (1 + c) (x2n+1 + x2n+1 +2 + x2n+1 +4 + · · · + x2n+2 −2 )
> a(1 + c) (x2n + x2n +1 + x2n +2 + · · · + x2n+1 −1 )
= ryn .

LATEX by Mathpiad 30
TẠP CHÍ VÀ TƯ LIỆU TOÁN HỌC

Với mọi n, ta có yn > rn−1 y1 . Do đó



X ∞
X ∞
X
xn = yn > y1 rn−1 = ∞
n=1 n=0 n=0


Bài toán 34

X
Gọi an là dãy số với các số hạng là các số dương và Sn biểu diễn tổng thứ n của dãy.
n=1
∞ ∞
X an X
Chứng minh rằng nếu hội tụ, thì an cũng hội tụ.
S
n=1 n n=1

So Mi Lim (student) and Sung Soo Kim, Hanyang University, Ansan, South Korea.

Lời giải. Jim Hartman, The College of Wooster, Wooster, OH.


∞ ∞ n
X X an X ak
Ta chứng minh nếu an hội tụ, khi đó hội tụ. Chú ý cả Sn và Tn = là dãy
n=1
S
n=1 n k=1
Sk
tăng và dương. Vì lim Sn = ∞ nên tồn tại một dãy con {nk }∞
k=1 trong dãy số tự nhiên để
n→∞
Snk+1 > 2Snk . Ta lại có
nk+1 nk+1
X ap X ap
Tnk+1 = Tnk + > Tnk +
S
p=n +1 p
S
p=n +1 nk+1
k k

Snk+1 − Snk 1
= Tnk + > Tnk +
Snk+1 2
snk 1
do < . Vì Tn tăng, nên lim Tn = ∞. 
Snk+1 2 n→∞

Bài toán 35
Cho p là số thực dương. Dãy (an )n>1 có a1 = 0 và
 p
n+1
an = + ab n c
2 2

an
với n > 2. Tìm giá trị nhỏ nhất của với mọi số nguyên dương n.
np −1

Arkady Alt, San Jose, California, USA

Lời giải. Radouan Boukharfane, Polytechnique de Montreal, Canada


Ta thấy:  p
2n + 1
a2n = + ab 2n c = np + an
 2 p 2

2n + 2
a2n+1 = + ab 2n+1 c = (n + 1)p + an
2 2

Trừ hai đẳng thức trên ta có :

a2n+1 − a2n = (n + 1)p − np

LATEX by Mathpiad 31
TUYỂN TẬP DÃY SỐ TRONG 10 NĂM TỪ CÁC TẠP CHÍ TOÁN CỦA MỸ

Viết lại ta được : n p  n p


an+1 − an = +1 −
2 2
Từ đó ta chứng minh được an tăng với mọi n > 3 :
n−1  p  p 
X k k
an = 1 + +1 −
k=2
2 2
an
Dãy sn = giảm và giá trị nhỏ nhất của sn với mọi số nguyên dương n là
np −1
n−1  p  p 
X k k
1+ +1 −
k=2
2 2
lim sn = lim
n→∞ n→∞ np − 1
hay:
n−1  p  p  n−1
X k k X
+1 − ((k + 2)p − k p )
k=2
2 2 1 k=2
=
np −1 2p np − 1
hay:
n−1
X n−1
X
p p
[(k + 2) − k ] = [(k + 2)p − (k + 1)p + (k + 1)p − k p ]
k=2 k=2
n−1
X n−1
X
p p
= [(k + 2) − (k + 1) ] + [(k + 1)p − k p ]
k=2 k=2
p p p p
= (n + 1) − 3 + n − 2
Ta suy ra
n−1
X
[(k + 2)p − k p ]
k=2 (n + 1)p − 3p + np − 2p
lim = lim
n→∞ np − 1 n→∞ np − 1
(n + 1) + np
p
−3p − 2p (n + 1)p + np
= lim + lim = lim +0
n→∞ np − 1 n→∞ np−1 n→∞ np − 1
(n + 1)p + np (n + 1)p np
= lim = lim + lim
n→∞ np − 1 n→∞ np − 1

n→∞ np − 1
p
(n + 1)p n+1
lim
np np n→∞ n 1
= lim p + lim p = +1= +1=2
n→∞ n − 1 n→∞ n − 1 1 1
n p

Suy ra:
1
lim sn =
n→∞ 2p−1
Kết quả trên là giá trị nhỏ nhất cần tìm. 
Bài toán 36
Cho x1 và x2 là hai số thực dương xác định, với n > 2
√ √ √
xn+1 = n x1 + n x2 + · · · + n xn

LATEX by Mathpiad 32
TẠP CHÍ VÀ TƯ LIỆU TOÁN HỌC

xn − n
Tìm lim .
n→∞ ln n

Gabriel Dospinescu, Ecole Normale Supérieure, Lyon, France

Lời giải. Jedrzej Garnek, University of Adam Mickiewicz, Poznan, Poland


Đặt c = max (1, x1 , x2 , x23 /4) và d = min (1, x1 , x2 , x23 /4).Ta sẽ đi chứng minh
√ √ √
 
n! log(n − 1)!
d· n+ 6 xn+1 6 n · n n − 1 · n c
n
với n > 2. Cả√hai bất đẳng thức trên đều đúng với n = 2. Sử dụng bất đẳng thức AM-GM, bất
đẳng thức k−1 k − 2 6 2 (dễ được suy ra từ 2x > 1 + x ) và giả thiết quy nạp (IH):
n n
X √ √ √ X √ (IH)
xn+1 = n
xk = x1 + x2 +
n n n
xk
k=1 k=3
n q
√ √ X n n−1
√ √
n n
6 c+ c+ (k − 1) · k−2· k−1
c
k=3
n q
!
√ X n k−1

6 n
c· 1+1+ (k − 1) · k−2
k=3
v
n
u
u n X √
k−1
n 1 + 1n +
u (k − 1) · k−2
√ t
k=3
6 c·n·
n

v n
u Xn
u n
n 1 + 1n +
u (k − 1) · 2
√ t
k=3
√ √
6 n cn · =nnn−1· nc
n
Để chứng minh bất đẳng thức thứ hai, ta dùng bất đẳng thức

n
ln x ln x
x=e n >1+
n
Ta có n n
X √ √ √ X√
xn+1 = n
xk = n x 1 + n x 2 + n
xk
k=1 k=3
s
n
√ √
 
n n
X
n log(k − 2)!
> d+ d+ [(k − 1)!]d · (k − 1) +
k=3
k−1
n

n

n
X p n
> d+ d+ (k − 1)]d · (k − 1)
k=3
n
!

n
X √
n
> d· 1+1+ k−1
k=3
n  !

n! log(k − 1)
X
> d· 1+1+ 1+
k=3
n

 
n! log(n − 1)!
= d· n+
n

LATEX by Mathpiad 33
TUYỂN TẬP DÃY SỐ TRONG 10 NĂM TỪ CÁC TẠP CHÍ TOÁN CỦA MỸ

Do đó ta có:

 
log(n − 1)!
n!
d· n+ − (n + 1) √ √
n xn+1 − (n + 1) n · n n − 1 · n c − (n + 1)
6 6
ln(n + 1) ln(n + 1) ln(n + 1)
ex − 1
Sử dụng xấp xỉ Stirling và giới hạn lim =1:
x→0 x

 
n! log(n − 1)! √ √
d· n+ − (n + 1) n n
!
n n · ( d − 1) d log(n − 1)!
lim = lim + =
n→∞ ln(n + 1) n→∞ ln(n + 1) n ln(n + 1)
 
ln d exp ln d − 1
 

 n · n! n
p
n! d log n (n − 1)! 
= lim  · ln d
+ =
n→∞ ln(n + 1) n!
ln(n + 1) 


ln d √
n
p  
n· d log n 1/2
(2π(n − 1)) · ((n − 1)/e)n−1 · δn 
= lim 
 n! + =0+1
n→∞ ln(n + 1) ln(n + 1)

(để ý rằng lim δn = 1 ) và:


n→∞
√ √
n· nn−1· nc−n n · (exp(ln(c(n − 1))/n) − 1)
lim = lim
n→∞ ln(n + 1) n→∞ ln(n + 1)
n · ln(c(n − 1))/n) − 1) exp(ln(c(n − 1))/n) − 1
= lim ·
n→∞ ln(n + 1) ln(c(n − 1))/n
n · ln(c(n − 1))/n) − 1)
= lim =1
n→∞ ln(n + 1)
Do đó, theo định lý kẹp, giới hạn cần tìm bằng 1. 
Bài toán 37
Cho a là một số nguyên bất kỳ. Xét dãy truy hồi được xác định bởi
(
u0 = 4, u1 = 0, u2 = 2, u3 = 3
un+4 = un+2 + un+1 + a · un

với mọi số nguyên n > 0. Chứng minh rằng up chia hết p cho mọi p nguyên tố.

Éric Pité, Paris, France.

Lời giải. John L. Simons, University of Groningen, the Netherlands.


Xét α1 , α2 , α3 , α4 là nghiệm của đa thức đặc trưng
Yi=4
f (x) = (x − αi ) = x4 − x2 − x − a. Bằng cách so sánh các hệ số, chúng ta thu được:
i=1

α1 + α2 + α3 + α4 = 0,
α1 α2 + α1 α3 + α1 α4 + α2 α3 + α2 α4 + α3 α4 = −1
α1 α2 α3 + α1 α2 α4 + α1 α3 α4 + α2 α3 α4 = 1, và
α1 α2 α3 α4 = −a

LATEX by Mathpiad 34
TẠP CHÍ VÀ TƯ LIỆU TOÁN HỌC

i=4
X
Xét nghiệm của un = αin . Dễ dàng thấy rằng nghiệm này thỏa mãn phương trình truy hồi
i=1
un+4 = un+2 + un+1 + a · un . Xét các giá trị đầu, ta có:

u0 = 4,
u1 = α1 + α2 + α3 + α4 = 0,
u2 = α12 + α22 + α32 + α42 = (α1 + α2 + α3 + α4 )2 − 2 (α1 α2 + . . . + α3 α4 ) = 2,
u3 = α13 + α23 + α33 + α43
= (α1 + α2 + α3 + α4 )3 − 3 (α1 + α2 + α3 + α4 )
(α1 α2 + α1 α3 + α1 α4 + α2 α3 + α2 α4 + α3 α4 )
+ 3 (α1 α2 α3 + α1 α2 α4 + α1 α3 α4 + α2 α3 α4 ) = 3

i=4
X
Do đó un = αin là nghiệm tổng quát của phương trình truy hồi với các giá trị ban đầu đã
i=1  
p
cho. Đối với p nguyên tố, tất cả các hệ số nhị thức với 0 < j < p đều chứa một hệ số p.
j
Do đó,
up = α1p + α2p + α3p + α4p = (α1 + α2 + α3 + α4 )p − p · g (α1 , α2 , α3 , α4 )

trong đó g (α1 , α2 , α3 , α4 ) là một đa thức đối xứng có nghiệm αi . Một định lý cơ bản trong lý
thuyết về đa thức đối xứng phát biểu rằng một đa thức đối xứng với nghiệm αi có thể được
biểu diễn dưới dạng hệ số của đa thức đặc trưng. Từ định lý này, ta thu được:

up ≡ (α1 + α2 + α3 + α4 )p ≡ 0 (mod p)


Bài toán 38
Cho một số thực dương a và xét (xn ) là dãy được xác định bởi :

x 1 = 1

n
−1 ,n > 1
Y
xn+1 = xn + an
 xi n
i=1

1. Chứng minh rằng: lim xn = ∞.


n→∞

2. Tính lim xn / ln n.
n→∞

Angel Plaza, Universidad de las Palmas de Gran Canaria, Las Palmas, Spain.

Lời giải. The Iowa State University Student Problem Solving Group, Iowa State University, Ames,
IA.
Vì a > 0√và x1 = 1 > 0, theo dãy truy hồi ta thu được: {xn } là dãy tăng. Ta đi chứng minh
xn+1 > 2 an vì:
p
p xn 2 a(n − 1)
lim xn > lim 2 a(n − 1) = ∞ và lim > lim =∞
n→∞ n→∞ n→∞ ln n n→∞ ln n

LATEX by Mathpiad 35
TUYỂN TẬP DÃY SỐ TRONG 10 NĂM TỪ CÁC TẠP CHÍ TOÁN CỦA MỸ

Để chứng minh bất đẳng thức trên, ta sẽ dùng {xn } là dãy tăng và AM-GM như sau:
n
−1/n
Y
xn+1 = xn + an xi
i=1
n
Y p √
> xn + an x−1/n
n = xn + anx−1 −1
n > 2 xn anxn = 2 an
i=1


Bài toán 39
Cho số thực dương a và {xn }n∈N là dãy truy hồi được xác định bởi :
a
x1 = 1 , và xn+1 = xn + với mọi n > 1 .
x1 + x 2 + · · · + x n

1. Chứng minh rằng lim xn = ∞.


n→∞

xn
2. Tính lim √ .
n→∞ ln n

Ovidiu Furdui, Technical University of Cluj-Napoca, Cluj-Napoca, Romania.

Lời giải. Michel Bataille, Rouen, France; Antonio Trusiani, Universita di Roma, “Tor Vergata,” Rome,
Italy; and Haohao Wang and Jerzy Wojdylo, Southeast Missouri State University, Cape Girardeau,
MO (independently).
Đầu tiên, {xn }n∈N là dãy tăng vì :
 2
2 a 2a
xn+1 > xn + > x2n + ,
nxn n
n−1 n−1
2
X
2 2
 X 1
xn > 1 + xk+1 − xk > 2a ,
k=1 k=1
k

Do đó, lim xn = ∞. Áp dụng định lý Stolz–Cesaro, ta có:


n→∞

x2n x2n+1 − x2n


lim = lim
n→∞ ln n n→∞ ln(n + 1) − ln n
 2 !
a 2
= lim n xn + − xn
n→∞ x1 + x2 + · · · + xn
a2 n
 
2anxn
= lim +
n→∞ x1 + x2 + · · · + xn (x1 + x2 + · · · + xn )2
n 1
Vì xn > xn−1 > · · · > x1 = 1, 0 6 2 6 , do đó
(x1 + x2 + · · · + xn ) n

a2 n
lim =0
n→∞ (x + x + · · · + x )2
1 2 n

Vì {xn } là dãy tăng nên


xn+1 a a
16 =1+ 61+
xn (x1 + x2 + · · · + xn ) xn nxn

LATEX by Mathpiad 36
TẠP CHÍ VÀ TƯ LIỆU TOÁN HỌC

Áp dụng định lý Stolz − Cesaro:


2anxn (n + 1)xn+1 − nxn
lim = 2a lim
n→∞ x1 + x2 + · · · + xn n→∞ xn+1
 
nxn
= 2a lim n + 1 −
n→∞ xn+1
= 2a.
xn √
Vì vậy, lim √ = 2a. 
n→∞ ln n
Bài toán 40
Cho số thực a và số nguyên dương k. Xét dãy số thực {an } sao cho lim an = a. Tính
n→∞
 
a0 a1 a(n−1)k
lim + + ··· +
n→∞ n+k n+k+1 n + nk

Valery Karachik, South Ural State University, Chelyabinsk, Russia.

Lời giải. Hongwei Chen, Christopher Newport University, Newport News, VA.
Giá trị ` của giới hạn ta mong muốn bằng a ln(k + 1).
Đầu tiên, ta chứng minh ` = aL(k), khi L(k) = lim H(n, k) và
n→∞

nk
X
H(n, k) = (n + j)−1 .
j=k

Vì lim an = a, với mọi ε > 0, tồn tại một số nguyên dương N sao cho |an − a| < ε khi n > N .
n→∞
Như vậy,
(n−1)k   N (n−1)k
X ai a X |ai − a| X |ai − a|
− 6 +
i=0
n+k+i n+k+i i=0
n + k + i i=N +1 n + k + i
(N + 1) max |ai − a|
06i6N (n − 1)k − N
6 + ε.
n+k n+k+N +1
Cho n → ∞ ta thu được |` − aL(k)| 6 kε. Vì ε > 0 bất kỳ nên ta thu được ` = aL(k).
Hãy chú ý
k−1 nk
X 1 X 1
L(k) + lim = lim
n→∞
i=1
n + i n→∞ i=1 n + i
nk
1 X 1
= lim
n→∞ nk 1 i .
i=1 +
Z 1 k nk
1
= dx
0
1
+x
k
Ta có Z 1   1
1 1
L(k) = dx = ln +x = ln(k + 1)
0
1 k
+x 0
k

LATEX by Mathpiad 37
TUYỂN TẬP DÃY SỐ TRONG 10 NĂM TỪ CÁC TẠP CHÍ TOÁN CỦA MỸ

Vì vậy, ta thu được ` = aL(k) = a ln(k + 1). 


Bài toán 41
√
Cho dãy số thực an = n2 + 1 , biết {x} là phần thập phân của x. Tìm limn→∞ nan

Ivan Borsenco, Massachusetts Institute of Technology, USA

Lời giải. Arkady Alt, San Jose, California,


√ USA
2 2 2 2
Vì n < n + 1 < (n + 1) nên n + 1 = n. Do đó:
√ 1
an = n2 + 1 − n = √ .
n2 + 1 + n

Vì vậy:
n 1
lim nan = lim √ = .
n→∞ n→∞ n2 + 1 + n 2

Bài toán 42
Cho dãy (an )n>0 được xác định bởi:
1 1 1
e n+1 e n+2 e 2n
an = + + ··· +
n+1 n+2 2n

Chứng minh rằng (an )n>0 là dãy số giảm và tính giới hạn của nó.

Angel Plaza, Universidad de Las Palmas de Gran Canaria, Spain

Lời giải. Konstantinos Tsouvalas, University of Athens, Greece


ex − 1
Ta có lim+ = 1, do đó tồn tại số M > 0 sao cho:
x→0 x
ex 6 1 + 2x, x<M
1
Tồn tại một số m ∈ N sao cho < M n > m, ta thu được:
m
 
1
n exp n 2n
X n+k X 1 X 2
6 +
k=0
n+k k=0
n + k k=n k 2

Từ đó, ta có bất đẳng thức: ex > x + 1, x > 0. Ta thấy:


 
1
n exp n 2n
X n+k X 1 X 1
> +
k=0
n+k k=0
n + k k=n k 2
X
Vì n−2 là dãy hội tụ, ta có:
n
2n
X 1
lim =0
n→∞
k=n
k2

LATEX by Mathpiad 38
TẠP CHÍ VÀ TƯ LIỆU TOÁN HỌC


n n Z 1
X 1 1X 1 dx
= k
→ = ln 2
k=1
n+k n k=1 1 + n 0 1+x

Từ đây kết hợp với bất đẳng thức đầu tiên, ta nhận thấy:

n
!
X exp ((n + k)−1 )
lim sup 6 ln 2
n→∞
k=1
n+k

Do đó: !
n
X exp ((n + k)−1 )
lim inf > ln 2
n→∞ n=1
k=1
n+k

Vì vậy, ta thu được:


n
!
X exp ((n + k)−1 )
lim = ln 2
n→∞
k=1
n+k


Bài toán 43
Xét s(n) là tổng các chữ số của n2 + 1. Dãy (an )n>0 được định nghĩa bởi an+1 = s (an ),
với a0 là số nguyên dương bất kỳ. Chứng minh rằng tồn tại n0 sao cho an+3 = an với mọi
n > n0 .

Roberto Bosch Cabrera, Havana, Cuba

Lời giải. Alessandro Ventullo, Milan, Italy


Ta cần chứng minh dãy số tuần hoàn chu kỳ 3. Vì f (5) = 8, f (8) = 11 và f (11) = 5,từ đây ta
chứng minh được rằng với mọi số nguyên dương a0 , tồn tại các số n ∈ N sao cho an ∈ {5, 8, 11}.
Đặt m số các chữ số của a0 . Ta chứng minh quy nạp trên m. Với m 6 2 ta tính trực tiếp.
Nếu a0 ∈ {5, 8, 11} bài toán được chứng minh. Nếu a0 là số có 2 chữ số thì a20 6 10000, vì vậy
a1 6 37 và ta giảm được các trường hợp a0 6 37.

1. Nếu a0 ∈ {2, 7, 20}, thì a1 = 5. Nếu a0 ∈ {1, 10, 26, 28}, thì a1 ∈ {2, 20}, do đó a2 = 5.
Cuối cùng, nếu a0 ∈ {3, 6, 9, 12, 15, 18, 27, 30, 33}, thì a3 = 5

2. Nếu a0 ∈ {4, 13, 23, 32}, thì a1 = 8

3. Nếu a0 ∈ {17, 19, 21, 35, 37}, thì a1 = 11. Nếu a0 ∈ {14, 22, 24, 31, 36}, thì a1 ∈ {17, 19},
so a2 = 11. Cuối cùng, nếu a0 ∈ {16, 25, 29, 34}, thì a3 = 11

Vì vậy, ta chứng minh được rằng nếu a0 là số có 1 hoặc 2 chữ số, thì an ∈ {5, 8, 11} với n ∈ N,
hay là dãy tuần hoàn chu kỳ 3. Xét m > 2 và giả sử đúng với mọi k 6 m. Gọi a0 là số có
(m + 1) chữ số. Do đó, 10m 6 a0 < 10m+1 tức là 102m 6 a20 < 102(m+1) . Kết lại

a1 = f (a0 ) 6 9 · 2(m + 1) + 1 < 10m

và theo quy nạp, dãy (an )n>1 là dãy tuần hoàn chu kỳ 3, tức là dãy (an )n>0 cũng là dãy tuần
hoàn chu kỳ 3, điều phải chứng minh. 

LATEX by Mathpiad 39
TUYỂN TẬP DÃY SỐ TRONG 10 NĂM TỪ CÁC TẠP CHÍ TOÁN CỦA MỸ

Bài toán 44
Cho số thực a sao cho (bnac)n>1 là dãy cấp số cộng. Chứng minh rằng a là số nguyên.

Mihai Piticari, Campulung Moldovenesc, Romania

Lời giải. Corneliu Manescu-Avram, Transportation High School, Ploiesti, Romania


Ta có: a = bac + {a} và bnac = nbac + bn{a}c. Không mất tính tổng quát, ta giả sử 0 6 a < 1,
kể từ số hạng đầu tiên của dãy. Cần chứng minh f (n) = b(n + 1)ac − bnac liên tục chỉ khi
1 1
a = 0. Thật vậy, với a 6= 0, tồn tại số dương m sao cho 6 a < , tức là b(m − 1)ac =
m+1 m
bmac = 0, b(m + 1)ac = 1, vậy nên f (m − 1) = 0 và f (m) = 1 Với a 6= 0 thì hàm f không liên
tục, bài toán chứng minh xong. 
Bài toán 45
r
an−1
Cho a0 = 0, a1 = 2 và an+1 = 2− với n > 1. Tính lim 2n an .
an n→∞

Titu Andreescu, University of Texas at Dallas, USA

Lời giải. Brian Bradie, Christopher Newport University, Newport News, VA, USA
Đầu tiên ta chứng minh quy nạp với n:
π
an = 2 sin n
2
Chú ý rằng 2 sin 2π0 = 2 sin 0 = 0 = a0 và 2 sin 2π1 = 2 sin π2 = 2 · 1 = 2 = a1 . Do đó
s
r π
an−1 2 sin 2n−1
an+1 = 2 − = 2−
an 2 sin 2πn
r r
π π π
= 2 − 2 cos n = 4 sin2 n+1 = 2 sin n+1 ,
2 2 2
ta suy ra điều cần chứng minh. Vì vậy:
sin 2πn
 
n n+1 π
lim 2 an = lim 2 sin n = lim 2π · π = 2π
n→∞ n→∞ 2 n→∞
2n

Bài toán 46
Cho dãy số thực dương (an )n>1 sao cho a1 + a2 + . . . + an < n2 với mọi n > 1. Chứng
minh rằng:  
1 1 1
lim + + ... + =∞
n→∞ a1 a2 an

Mihai Piticari Campulung, Moldovenesc, Romania

Lời giải. Ángel Plaza, Universidad de Las Palmas de Gran Canaria, Spain
Bằng cách sắp xếp lại trình tự dãy ( nếu cần thiết), ta giả sử rằng (an )n>1 là một dãy tăng. Ta
sẽ chứng minh bài toán bằng phản chứng. Giả sử rằng
 
1 1 1
lim + + ... + < ∞.
n→∞ a1 a2 an

LATEX by Mathpiad 40
TẠP CHÍ VÀ TƯ LIỆU TOÁN HỌC

n
Bằng cách so sánh với chuỗi điều hòa lim = 0. Theo tiêu chuẩn Cezaro-Stolz, ta có:
n→∞ an

n2 2n − 1
lim = lim =0
n→∞ a1 + a2 + . . . + an n→∞ an
n2
Nhưng nó mâu thuẫn với giả thiết 1 6 . 
a1 + a2 + . . . + an
Bài toán 47
1
Cho dãy số {yn } xác định bởi y1 = 0 và yn = với mọi n > 2. Giả sử tồn tại dãy
3 − yn−1
{xn } xác định bởi 
x 1 ∈
/ {yn : n ∈ N}
1
x n = 3 − , ∀x > 2.
xn−1
Chững minh rằng dãy {xn } hội tụ.

Prapanpong Pongsriiam, Silpakorn University, Nakhon Pathom, Thailand.

Lời giải. Northwestern University Math Problem Solving Group, Evanston, IL.
1 1
Chú ý rằng, hàm ngược của hàm f (x) = 3 − là hàm f −1 (y) = . Theo đó, các điểm bất
x 3−y
động của f (x) (cũng như của f −1 (x)) gồm
√ √
3− 5 3+ 5
α= , β= .
2 2
Nếu x1 = α, dãy {xn } là dãy hằng với tất cả các số hạng bằng α. Phần sau của chứng minh
này, ta giả sử rằng x1 6= α. Ta sẽ đi chứng minh xn → β khi n → ∞.
1
Công thức tổng quát yn = cho ta
3 − yn−1
β − yn−1
yn − yn−1 = (α − yn−1 ) , ∀n > 2 (1)
3 − yn−1

Do y1 = 0 < α < β < 3, ta có thể suy ra từ (1) rằng dãy {yn } tăng thực sự và bị chặn bởi α.
Cụ thể, nhờ vào yn−1 < 3, ta có yn được xác định với mọi n > 1. Lại do (xn ) và (yn ) là các
dãy được sinh ra từ hai hàm ngược f và f −1 , một kết quả hiển nhiên là xn = y1 khi và chỉ khi
x1 = yn . Với giả thiết x1 6= yn , mọi số xn đều khác 0, và xn được xác định với mọi n > 1. Tiếp
theo, ta xét dãy {zn } xác định bởi
xn − β
zn = , ∀n > 1. (2)
xn − α
Với chú ý xn 6= α, ∀n > 1 suy ra từ x1 6= α, ta nhận thấy mọi số hạng trong (zn ) cũng được
xác định. Ta có  
1 1 1
3− −β −
xn − β xn−1 β xn−1
zn = = =
1 1

xn − α 1
3− −α −
xn−1 α xn−1
α xn−1 − β α
= · = zn−1 , ∀n > 2
β xn−1 − α β

LATEX by Mathpiad 41
TUYỂN TẬP DÃY SỐ TRONG 10 NĂM TỪ CÁC TẠP CHÍ TOÁN CỦA MỸ

 n−1
α
Đánh giá trên dẫn đến zn = z1 → 0 nếu như n → ∞. Ngoài ra, do phép đặt ở (2)
β
tương đương với
β − αzn
xn =
1 − zn
thế nên là
β − αzn β−α·0
lim xn = lim = = β.
n→∞ n→∞ 1 − zn 1−0
Bài toán được chứng minh. 
Bài toán 48
Với mỗi số nguyên dương n, ta đặt vn = k nếu n chia hết cho 3k nhưng không chia hết
cho 3k+1 . Xét dãy {Xn } thỏa mãn X1 = 2, đồng thời
2
Xn = 4vn + 2 − , ∀n > 2.
Xn−1

Chứng minh rằng mọi số hữu tỉ dương xuất hiện đúng một lần trong dãy trên.

Sam Northshield, SUNY Plattsburgh, Plattsburgh, NY.

Lời giải. László Lipták, Oakland University, Rochester, MI.


Xét các ánh xạ S, P, Q, and R đi từ tập hữu tỉ tới tập hữu tỉ và thỏa mãn
2x + 2 x 1
S(x) = x + 2, P (x) = , Q(x) = , R(x) = .
x+2 x+1 x
Dãy đã cho trở thành
Xn = 4vn + 2 − 2R (Xn−1 ) .
Mặt khác, ta chứng minh được một vài mối liên hệ giữa R, S, P và Q, đó là

• 2 − 2R(S(x)). −2x + 2
• P −1 (x) = .
x−2
• 2 − 2R(P (x)) = Q(x). −x
• Q−1 (x) = .
x−1
• 2 − 2R(Q(x)) = −2R(x). • S −1 (x) = x − 2.

Bây giờ, ta sẽ đi chứng minh rằng những đẳng thức sau đúng với mọi n > 1.

X3n = S (Xn ) , X3n+1 = P (Xn ) , X3n+2 = Q (Xn ) . (1)

Kiểm tra trực tiếp, tất cả đẳng thức trên đúng với n = 1, và ta chỉ cần chứng minh chúng cũng
đúng với n > 1. Nhờ vào giả thiết v3n = 1 + vn , ta có

X3n = 4 + 4vn + 2 − 2R (X3n−1 )


= 4vn + 2 + (2 − 2R (Q (Xn−1 )) + 2
= 4vn + 2 − 2R (Xn−1 ) + 2 = Xn + 2 = S (Xn )

Tiếp theo, giả thiết v3n+1 = 0 cho ta

X3n+1 = 2 − 2R (X3n ) = 2 − 2R (S (Xn )) = P (Xn ) .

LATEX by Mathpiad 42
TẠP CHÍ VÀ TƯ LIỆU TOÁN HỌC

Cuối cùng, từ giả thiết v3n+2 = 0, ta suy ra được

X3n+2 = 2 − 2R (X3n+1 ) = 2 − 2R (P (Xn )) = Q (Xn ) .

Với lẽ hiển nhiên là các số Xi hữu tỉ dương, chúng ta có

0 < X3n+2 < 1 < X3n+1 < 2 < X3n . (2)


a a
Với là một phân số tối giản, ta định nghĩa hàm σ như sau: σ = a + b.
b b
Ta xét hàm số  a a
 Q , nếu 0 < < 1
  ab  b



a
T (x) = P , nếu 1 < < 2
 b b
a a

 
R

, nếu 2 <
b b
  a  a
Ta sẽ đi chứng minh σ T −1 <σ . Theo đó
b b
 
a 
−1 a
  a a
• Nếu 0 < < 1, ta có a < b, thế nên σ Q =σ =b<σ .
b b b−a b
a
• Nếu 1 < < 2, ta có b < a < 2b, thế nên
b
 

−1 a
  2a − 2b a
σ P =σ =a<σ .
b 2b − a b

a
• Nếu > 2, ta có 2b < a, thế nên
b
 

−1 a
  a − 2b a
σ S =σ =a−b<σ .
b b b
a
−1
Chú ý rằng T cũng là thương của hai số dương nguyên tố cùng nhau. Để chứng minh
b
kết quả này, chú ý rằng (2) cho ta biết 2 và 1 không thể xuất hiện ở khúc sau của dãy. Viết tất
a
cả các phân số dưới dạng rút gọn . Khi T −1 được tác động liên tiếp, ta chỉ ra hàm σ giảm.
b
Như vậy, tồn tại c ∈ {1, 2} và dãy các hàm T1 , . . . , Ts với mỗi Ti ∈ {Q, P, S} sao cho
a
Ts−1 · · · T1−1 = c ∈ {1, 2}.
b

Sự tồn tại trên dẫn tới


a
= T1 · · · Ts (c).
b
a
Kết hợp với (1), ta nhận thấy tồn tại số nguyên dương m sao cho = Xm .
b
a
Vấn đề còn lại chỉ còn là chứng minh tính duy nhất của m. Với = Xm , từ (2) và (1), ta
b
nhận thấy T1 lần lượt là hàm S, P, và Q nếu như số dư của
j mm kkhi chia cho 3 là 0, 1 và 2. Một
cách tương tự, Tn cũng được xác định theo modulo 3 của i−1 . Biểu diễn của một số nguyên
3
dương trong hệ cơ số 3 là duy nhất và ta có điều phải chứng minh. 

LATEX by Mathpiad 43
TUYỂN TẬP DÃY SỐ TRONG 10 NĂM TỪ CÁC TẠP CHÍ TOÁN CỦA MỸ

Bài toán 49
Gọi Qn là số hạng thứ n của dãy Pell − Lucas:
(
Q0 = 2, Q1 = 2,
Qn = 2Qn−1 + Qn−2

Xét dãy an được xác định bởi


n
!2 n
X X
an = Qj − Q2j+1 .
j=0 j=0


X an
Hãy tính giới hạn lim .
n=0
n!

Brian Bradie, Christopher Newport University, Newport News, VA.

Lời giải. Kim McInturff, Santa Barbara, CA.


Với mỗi n = 0, 1, 2, . . ., ta gọi Pn là số Pell thứ n. Theo đó P0 = 0, P1 = 1, và Pn = 2Pn−1 +Pn−2
với mọi n > 2. Bằng kiểm tra trực tiếp, ta chỉ ra
n
X n
X
Qj = 2Pn+1 , Q2j+1 = (2Pn+1 )2 − 1 − (−1)n .
j=0 j=0

Hai đẳng thức trên dẫn đến


an = (2Pn+1 )2 − (2Pn+1 )2 + 1 + (−1)n = 1 + (−1)n .
Chính vì lẽ đó ∞ ∞ ∞
X an X 1 X (−1)n 1
lim = lim + lim =e+ .
n=0
n! n=0
n! n=0
n! e
Bài toán được giải quyết. 
∞ ∞
X 1 X (−1)n 1
Hai kết quả lim = e và lim = thu được ở trên là hệ quả của khai triển
n=0
n! n=0
n! e
Taylor:

f 0 (x0 ) f 00 (x0 )
! f (x) = f (x0 ) +
1!
(x − x0 ) +
2!
(x − x0 )2
f (n) (x0 )
+ ··· + (x − x0 )n + · · ·
n!
Cho f (x) = ex và lần lượt cho x0 = 1, x0 = −1,, các đẳng thức giới hạn được chứng minh.

Bài toán 50
Xét dãy {an }n=∞
n=−∞ được cho bởi a0 = 1, a1 = 0, a2 = 0 và với mọi số nguyên n thì
an+3 = an + an+1 + an+2 . Chứng minh rằng

an a−n = an+1 a−n+1 + an+2 a−n+2 + an+3 a−n+3

LATEX by Mathpiad 44
TẠP CHÍ VÀ TƯ LIỆU TOÁN HỌC

Michael Goldenberg, The Ingenuity Project, Baltimore Polytechnic Institute,


Baltimore MD and Mark Kaplan, Towson University, Towson MD.

Lời giải. James Duemmel, Bellingham WA and Michael Woltermann, Washington and Jefferson
College, Washington, PA
Đặt

dn = an a−n − (an+1 a−n+1 + an+2 a−n+2 + an+3 a−n+3 ) (3)

Ta sẽ chứng minh dn = 0 với mọi n ∈ N . Thay an+3 bởi an + an+1 + an+2 và a−n+3 bởi
a−n + a−n+1 + a−n+2 vào (3),

dn = −an a−n+1 − an a−n+2 − bn

với bn = an+1 a−n+1 + an+2 a−n+2 + an+2 a−n+3 + an+1 a−n+3 . Mặt khác, ta có

dn−1 = an−1 a−n+1 − (an a−n+2 + an+1 a−n+3 + an+2 a−n+4 ) (4)

Thay a−n+4 bởi a−n+3 + a−n+2 + a−n+1 và an−1 bởi an+2 − an+1 − an vào (4) ta được

dn−1 = −an a−n+1 − an a−n+2 − bn = dn .

Từ đây suy ra dn = d0 = 0. 
Bài toán 51
Xét dãy xn ∈ [0, 1] được cho bởi a0 = 1 và
xn + xn−1
(1 − xn )2 − (1 − xn−1 )2 = ,
α
với n là số tự nhiên, α > 1. Chứng minh rằng

X 1
xn = (α − 1) .
n=1
2

Northwestern University Math Problem Solving Group, Northwestern University,


Evanston IL.

Lời giải. Tom Beatty, Florida Gulf Coast University, Ft. Myers FL.
Đầu tiên chúng ta chứng minh rằng nếu xn−1 ∈ [0; 1] thì phương trình đặc trưng của dãy truy
hồi có một nghiệm duy nhất xn ∈ [0; 1]. Thực tế, xn là một nghiệm của đa thức bậc hai sau
đây với ẩn t:
t + xn−1
f (t) = (1 − t)2 − (1 − xn−1 )2 −
α
 
1 1 + xn−1
Vì f (0) = xn−1 2 − − xn−1 > 0 và f (1) = −(1 − xn−1 )2 − < 0 nên f (t) có đúng
α α
một nghiệm trong đoạn [0; 1]. Hơn nữa, xn ∈ [0; 1] với n > 1 vì f (t) 6= 0 khi t = 0 và t = 1.
Khi đó, tổng của n số hạng đầu tiên của dãy có thể được viết như sau:
N
X 1 1 1 1 1
Sn = xn = − x0 + (x1 + x0 ) + (x2 + x1 ) + · · · + (xn + xn−1 ) + xn
n=1
2 2 2 2 2

LATEX by Mathpiad 45
TUYỂN TẬP DÃY SỐ TRONG 10 NĂM TỪ CÁC TẠP CHÍ TOÁN CỦA MỸ

n
!
1 α X 1
(1 − xn )2 − (1 − xn−1 )2

=− + + xn
2 2 n=1
2
1 α 1
= − + (1 − xn )2 + xn
2 2 2
α
Vì các số hạng của tổng là số dương, Sn đơn điệu tăng và bị chặn trên tại . Do đó, Sn hội tụ.
2
Điều này có nghĩa là lim xn = 0. Từ đó ta có điều phải chứng minh. 
n→∞

Bài toán 52
Cho a1 là một số thực dương. Dãy {an } được xác định bởi an+1 = n2 /an với n = 1, 2, . . .
Tính n
1 X 1
lim
n→∞ ln n ak
k=1

Paul Bracken, University of Texas Rio Grande Valley, Edinburg, TX

Lời giải. Michael Goldenberg, The Ingenuity Project, Baltimore Polytechnic Insti- tute, Baltimore,
MD, and Mark Kaplan, Towson University, Baltimore, MD.
Tính một vài số hạng đầu, ta thấy
1 a1
=
a2 12
1 12
=
a3 22 a1
1 22 a1
=
a4 32 · 12
1 32 · 12
=
a5 42 · 22 a1
1 42 · 22 a1
= .
a6 52 · 32 · 12

Từ đó có thể dự đoán được


1 ((2k − 2)!!)2 a1
= và
a2k ((2k − 1)!!)2
1 ((2k − 1)!!)2
= .
a2k+1 ((2k)!!)2 a1
Ta sẽ chứng minh bằng quy nạp .Theo công thức Stirling thì

  
n+1/2 −n 1
n! = 2πn e 1+O
n

Từ đó
√ k+1/2 k+1/2 −k
  
k 1
(2k)!! = 2 k! = π2 k e 1+O
k


2π(2k)2k+1/2 e−2k 1 + O k1
   
(2k)! k+1/2 k −k 1
(2k − 1)!! = k = √ 1
=2 k e 1+O .
2 k! 2k 2πk k+1/2e (1+O( k ))
−k
k

LATEX by Mathpiad 46
TẠP CHÍ VÀ TƯ LIỆU TOÁN HỌC

Suy ra

((2k − 2)!!)2 a1 π22k−1 k 2k−1 e−2k (1 + O(1/k))


 
1 πa1 1
= 2
= 2k+1 2k −2k
a1 = +O
a2k ((2k − 1)!!) 2 k e (1 + O(1/k)) 4k k2

Bài toán 53
Cho a, b, c là các số thực dương không lớn hơn 2. Dãy số (xn )n>0 xác định bởi

x0 = a, x1 = b, x2 = c
xn+1 = xn + √xn−1 + xn−2
q

với mọi n > 2. Chứng minh (xn )n>0 hội tụ và tìm giới hạn của nó.

Mircea Becheanu, Canada and Nicolae Secelean, Romania

Lời giải. Daniel Lasaosa, Pamplona, Spain, Albert Stadler,


pHerrliberg, Switzerland

Chú ý nếu 0 < xn−2 , xn−1 , xn 6 2, khi đó 0 < xn+1 6 2 + 2 + 2 = 2, dễ thấy 0 < xn 6 2
với mọi n > 0.

• Nếu xn > 1, thì xn+1 > xn > 1.

• Nếu xn > 2−2 với m là số nguyên dương, thì


m

√ m−1
xn+1 > xn > 2−2
s r
1 1 1 1
ta có xn+m−1 > và xn+m > , với xn+m+1 > + = 1.
4 2 2 4
Như vậy xn > 1 với mọi n > N và N là số nguyên.
Đặt δn = 2 − xn , trong đó 0 6 δn < 2 với mọi số nguyên dương n, hoặc với n > N + 2 và N
xác định như trên, ta có:

4 − xn − xn−1 + xn−2
δn+1 = 2 − xn+1 = p √
2 + xn + xn−1 + xn−2
δn δn−1 + δn−2
= p √ + √  √
2 + xn + xn−1 + xn−2
p
2 + xn + xn−1 + xn−2 (2 + xn−1 + xn−2 )
δ δ +δ 6δ + 2δn−1 + 2δn−2
6 pn √ + p n−1 √ n−2 √ 6 n
2 + 1 + 2 (2 + 1 + 2)(2 + 2) 21
p √ 3
Từ việc chứng minh bất đẳng thức cuối cùng, ta đồng thời chứng minh được 1 + 2 >
√ √ 2
và 2 + 2 > 3. Dễ thấy 2 > 1, kết hợp với điều kiện trước, bất đẳng thức tương đương với
√ 5 √ √ √
2 > , đúng vì 4 2 = 32 > 25 = 5. Ta có 21δn+1 < 6δn + 2δn−1 + 2δn−2 , hoặc
4
 
10δn−1 2 10δn−2 2δn−2
21δn+1 + 8δn + 6 21δn + 8δn−1 + −
3 3 3 9
 
2 10δn−2
6 21δn + 8δn−1 + .
3 3

LATEX by Mathpiad 47
TUYỂN TẬP DÃY SỐ TRONG 10 NĂM TỪ CÁC TẠP CHÍ TOÁN CỦA MỸ

10δn−1 2∆n−1
Hoặc đặt ∆n = 21δn+1 + 8δn + , ta có 0 6 ∆n < với n > N + 2.
3 3
∆n−1
Vậy lim ∆n = 0, hoặc do 0 6 δn 6 , ta được lim δn = 0, và lim xn = 2. 
n→∞ 21 n→∞ n→∞

Bài toán 54
Chứng minh nếu Fn là số thứ n của dãy số Fibinacci và với p cố định thì:
n  
X n
FPk Fp−1
n−k
Fk = Fpn
k=1
k

Tarit Goswami, West Bengal, India

Lời giải. AN-anduud


√ Problem Solving Group, Ulaanbaatar, Mongolia
1+ 5
Đặt ϕ = . Sử dụng công thức
2
ϕm = Fm−1 + Fm ϕ
với m là số nguyên dương. Khi đó ta có:
Fpn · ϕ + Fpn−1 = ϕpn = (ϕp )n
= (Fp · ϕ + Fp−1 )n
n  
X n
= (Fp ϕ)k Fp−1
n−k

k=1
k
n  
X n
= Fpk Fp−1
n−k
(Fk · ϕ + Fk−1 )
k=1
k
n  
! n  
X n k n−k
X n
= Fp Fp−1 Fk ϕ + Fpk Fp−1
n−k
Fk−1 .
k=1
k k=1
k

Từ đó n  
X n
Fpn = Fpk Fp−1
n−k
Fk
k=1
k

Bài toán 55
Z x
sin x
Tính giá trị của lim dx.
n→∞ 0 1 + cos2 nx

Robert Bosch, USA

Lời giải. Robert Bosch, USA


Quy ước một số kí hiệu sau:
Z π n
sin x X
An = dx = Ik ,
0 1 + cos2 nx k=1

trong đó
Z kπ
n sin x
Ik = dx
(k−1)π
n
1 + cos2 nx

LATEX by Mathpiad 48
TẠP CHÍ VÀ TƯ LIỆU TOÁN HỌC


n
X
Bn = Jk ,
k=1

trong đó

sin kπ
Z
n
n
Jk = dx
(k−1)π
n
1 + cos2 nx

Đầu tiên ta sẽ chứng minh


lim (An − Bn ) = 0
n→∞

Xét hiệu Ik − Jk (k = 1, . . . , n) ta có:



sin x − sin kπ
Z
n
n
|Ik − Jk | 6 2
dx
(k−1)π
n
1 + cos nx

kπ/n − x
Z
n kπ/n + x
6 2 sin cos dx,
(k−1)π
n
2 2

kπ/n − x
Z
n
6 2 sin dx
(k−1)x
n
2

π2
Z  
n kπ
6 − x dx = 2 .
(k−1)π
n
n 2n

Do đó:
n n
X X π2 π2
|An − Bn | = (Ik − Jk ) 6 |Ik − Jk | 6 n · = .
k=1 k=1
2n2 2n

Nếu lim Bn tồn tại thì lim An cũng tồn tại và giới hạn của nó bằng nhau. Khi đó, ta có:
n→∞ n→∞

n Z π
1X kπ 1
Bn = sin dx
n k=1 n 0 1 + cos2 x

Ta tính được:
n Z π
1X kπ 1 2
lim sin = sin xdx =
n→∞ n n π 0 π
k=1

và Z x
1 π
dx = √
0 1 + cos2 x 2
Vậy
2 π √
lim Bn = · √ = 2,
n→∞ π 2
hay Z π
sin x √
lim dx = 2.
n→∞ 0 1 + cos2 nx


LATEX by Mathpiad 49
TUYỂN TẬP DÃY SỐ TRONG 10 NĂM TỪ CÁC TẠP CHÍ TOÁN CỦA MỸ

Bài toán 56
Cho dãy (an )n>1 được định nghĩa bởi
 n+an
1 1 1
1+ =1+ + ··· +
n 1! n!
1
(i) Chứng minh rằng (an )n>1 hội tụ và lim an = .
n→∞ 2
 
1
(ii) Tính giới hạn lim n an − .
n→∞ 2

Dorin Andrica, Babes-Bolyai University, Cluj-Napoca, România

Lời giải. Paolo Perfetti, Università degli studi di Tor Vergata Roma, Rome, Italy
n
X 1
(i) Đặt Sn = → e, ta có
k=0
k!
 
1
    ln Sn − n ln 1 +
1 1 n
n ln 1 + + an ln 1 + = ln Sn ⇔ an =  
n n 1
ln 1 +
n

Áp dụng định lý Cesaro − Stolz cùng với khai triển Maclaurin

x2 x3
+ o x3

ln (1 + x) = x − +
2 3
Ta được
   
1 1
ln Sn+1 − ln Sn − (n + 1) ln 1 + + n ln 1 +
n+1 n
lim     =
n→∞ 1 1
ln 1 + − ln 1 +
n+1 n
   
1 1 1 1 1 1
ln 1 + −1+ +− 2
+1− + 2 +o
Sn (n + 1)! 2(n + 1) 3(n + 1) 2n 3n n2
lim  
n→∞ 1
ln 1 −
(n + 1)2

Ta có
 
1
ln 1 +
Sn (n + 1)! (n + 1)2
1. lim   = − lim = 0.
n→∞ 1 n→∞ Sn (n + 1)!
ln 1 −
(n + 1)2
1 1 1
− −
2(n + 1) 2n (n + 1)2 −(n + 1)2 1 1
2. lim   = lim   lim =1· = .
n→∞ 1 n→∞ 1 n→∞ −2n(n + 1) 2 2
ln 1 − 2
ln 1 − 2
(n + 1) (n + 1)

LATEX by Mathpiad 50
TẠP CHÍ VÀ TƯ LIỆU TOÁN HỌC

1

−(n + 1)2 (2n + 1)
 
1 (n + 1)2
3. ln 1 − = lim   lim = 0.
(n + 1)2 n→∞ 1 n→∞ 3n2 (n + 1)2
ln 1 −
(n + 1)2
 
1
o
n2
4. lim   = 0.
n→∞ 1
ln 1 −
(n + 1)2

(ii) Áp dụng định lý Cesaro − Stolz , ta viết lại


 
1 1 2an − 1
lim n an − = lim
n→∞ 2 2 n→∞ 1
n
Xét giới hạn sau
1 2an+1 − 2an
lim =
2 n→∞ 1 1

n+1 n    
1 1

 ln Sn − n ln 1 + n ln Sn+1 − (n + 1) ln 1 +
n+1 
lim n(n + 1) 
   −   =
n→∞ 1 1 
ln 1 + ln 1 +
n n+1
 
ln S ln Sn+1
 n − 
 
lim n(n + 1)   + 1
=
n→∞  1 1
ln 1 + ln 1 +
n n+1
 
1
 
ln 1 +
ln Sn ln Sn S (n + 1)!
 n
 
lim n(n + 1) 
   −   −  + 1
n→∞ 1 1 1 
ln 1 + ln 1 + ln 1 +
n n+1 n+1

Ta có
   
1 1 1
ln 1 + ln 1 +
Sn (n + 1)! n(n + 1)2 n + 1 Sn (n + 1)!
lim n(n + 1) = lim
1
   
n→∞ 1 n→∞ Sn (n + 1)! 1
ln 1 + ln 1 +
n+1 n+1 Sn (n + 1)!
=0

Và đồng thời  
ln S ln Sn
 n − 
 
n(n + 1)   + 1
=
 1 1
ln 1 + ln 1 +
n n+1
 
 ln Sn ln Sn 
n(n+1)   + −   + 1
=
 1 1 1 1 1 1 1 1 1 1
− 2 + 3 − 4 +O − + − +O
n 2n 3n 4n n5 n + 1 2(n + 1)2 3(n + 1)3 4(n + 1)4 n5

LATEX by Mathpiad 51
TUYỂN TẬP DÃY SỐ TRONG 10 NĂM TỪ CÁC TẠP CHÍ TOÁN CỦA MỸ

 
 n ln Sn (n + 1) ln Sn 
n(n+1)     + −    + 1
=
 1 1 1 1 1 1 1 1
1− − + +O 1− − + +O
2n 3n2 4n3 n4 2(n + 1) 3(n + 1)2 4(n + 1)3 n4
Khai triển 1/(1 − x) = 1 + x + x2 + x3 + x4 + O (x5 ) ta thu được
   
1 1 1 1
n(n + 1) ln Sn − − + O
2 12n 12n2 n3
  
1 1 1 1
− ln Sn − − +O
2 12(n + 1) 12(n + 1)2 (n + 1)3
 
− ln Sn n(n + 1)(2n + 1) ln Sn 1 −1
= − 2 2
+O 2

12 12n (n + 1) n 12

Bài toán 57
Cho f : [0, 1] → R là một hàm liên tục, dãy (xn )n>1 được định nghĩa bởi
n   
X 1 k
xn = cos √ f − αnβ ,
k=0
n n

trogn đó α và β là 2 số thực. Tính giới hạn lim xn .


n→∞

Dorin Andrica, Babes-Bolyai University, Cluj-Napoca, Romania

Lời giải. AN-anduud Problem Solving Group, Ulaanbaatar, Mongolia


Sử dụng khai triển Taylor ta được
n       
X 1 2 k 1 4 k 1
xn = 1− f + 2
f +O 2
− αnβ
k=0
2n n 24n n n
n   n    
1 X 2 k 1 X 4 k 1
= (n + 1) − f + 2
f +O − αnβ
2n k=0 n 24n k=0 n n
Z 1  
1 1
∼ n − αnβ + 1 − f 2 (x)dx + O

2 0 n

Trường hợp 1. Nếu β < 1, α ∈ R, khi đó ta có lim xn = +∞.


n→∞
Z 1
1
Trường hợp 2. Nếu β = 1, α = 1, khi đó ta có lim xn = 1 − f 2 (x)dx.
n→∞ 2 0

Trường hợp 3. Nếu β = 1, α > 1, khi đó ta có lim xn = −∞


n→∞

Trường hợp 4. Nếu β = 1, α < 1, khi đó ta có lim = +∞


n→∞

Trường hợp 5. Nếu β > 1, α > 0, khi đó ta có lim = −∞


n→∞

Trường hợp 6. Nếu β > 1, α < 0, khi đó ta có lim = +∞.


n→∞

Bài toán được giải quyết. 

LATEX by Mathpiad 52
TẠP CHÍ VÀ TƯ LIỆU TOÁN HỌC

Bài toán 58
Cho p > 1 là một số tự nhiên. Chứng minh rằng
n
!
X 1 p  p−1 
lim √ − n p −1 ∈ (0, 1)
n→∞
k=1
p
k p−1

Alessandro Ventullo, Milan, Italy

Lời giải. Arkady Alt, San Jose, CA, USA


Ta xét 2 dãy số sau
n
X 1 p  p−1

an : = √ − (n + 1) P − 1
k=1
p
k p−1
n
X 1 p  p−1 
bn : = √ − n P −1
k=1
p
k p−1

Đầu tiên ta sẽ chứng minh rằng an ↑ N và bn ↓ N. Theo định lý giá trị trung bình, tồn tại một
số cn ∈ (n + 1, n + 2) sao cho
1 p  p−1 p−1

an+1 − an = p√ − (n + 2) p − (n + 1) p
n+1 p−1
1 1 1 1
= √
p
−√ p c
> √
p
− √p
n+1 n n+1 n+1
=0

tương tự như thế, với cn ∈ (n, n + 1) thì


1 p  p−1 p−1

bn − bn+1 = − √ + (n + 1) p − n p
p
n+1 p−1
1 1 1 1
= √
p c
−√p
> √
p
−√ p
n n+1 n+1 n+1
=0

Do an ↑ N, bn ↓ N và an < bn , n ∈ N nên an < bm với mọi n, m ∈ N. Do đó

an < an+m < bn+m < bm

Chú ý rằng an < b1 và a1 < bn , n ∈ N nên cả 2 dãy (an )n∈N và (bn )n∈N đều hội tụ. Đặt
l := lim an và u := lim bn . Vì
n→∞
lim (bn − an ) = 0
n→∞

nên l = u. Đặt c := lim an = lim bn , ta có an < c < bn bởi vì an ↑ N, bn ↓ N và an < bn , n ∈ N.


n→∞ n→∞
Từ đây suy ra
n n
X 1 p  p−1
  X 1 p  p−1 
an < c < bn ⇐⇒ √ − n + 1 p − 1 < c < √ − n p − 1
k=1
p
k p−1 k=1
p
k p−1
n
p  p−1  X 1 p  p−1
 
⇐⇒ n p −1 +c< √ < n+1 p − 1 + c.
p−1 k=1
p
k p−1

LATEX by Mathpiad 53
TUYỂN TẬP DÃY SỐ TRONG 10 NĂM TỪ CÁC TẠP CHÍ TOÁN CỦA MỸ

p  p−1 
Ta có 2 P − 1 ↑ p ∈ N\{1}, nên
p−1
1
X 1 p  p−1
 
a1 = √ − (1 + 1) p − 1
k=1
p
k p−1
p  p−1  2  2−1 
=1− 2 p −1 >1− 2 2 −1
p−1 2−1
√ 2
= 1 − 2( 2 − 1) = 1 − √
2+1

2−1
=√
2+1
>0

Do đó a1 6 an < c = c > 0. Tương tự, vì


1
X 1 p  p−1 
b1 = √ − 1 p −1 =1
k=1
p
k p−1

và đồng thời c < bn 6 b1 nên c < 1. Như vậy bài toán được giải quyết. 
Bài toán 59
Cho dãy (an ) được xác định bởi

a1 = 1
1 1 1
an = 1 + + + ··· + ,n > 1
a1 a2 an−1
 √ 
Tính giới hạn lim an − 2n .
n→∞

Robert Bosch, USA

Lời giải. Vincelot Ravoson, Lycée Henri IV, Paris, France


Ta dễ dàng chứng minh quy nạp rằng, ∀n ∈ N, an+1 − an > 0. Do đó
n
! n−1
!
X 1 X 1 1 1
an+1 − an = 1 + − 1+ = > 0 ⇔ an+1 = an +
k=1
ak k=1
ak an an

1 1
Giả sử rằng (an ) hội tự về giới hạn thực l. Khi đó ta có l = l + ⇔ = 0, điều này là vô lý,
l T
do vậy
lim an = +∞
n→+∞

Bây giờ, hãy chú ý rằng


1
a2n+1 = a2n + + 2,
a2n
1
và đồng thời lim = 0, lúc này
n→+∞ a2
n

1
a2n+1 − a2n = 2 + ∼ 2,
a2n

LATEX by Mathpiad 54
TẠP CHÍ VÀ TƯ LIỆU TOÁN HỌC

và theo định lý Stolz − Cesàro, ta có


n
X
a2k − a2k−1 ∼ 2(n − 1) ∼ 2n ⇔ a2n − a21 ∼ 2n

k=2

Do vậy √
a2n ∼ a2n − a21 ∼ 2n ⇔ an ∼ 2n
Từ đây ta suy ra
n−1 n−1
X  X 1
a2n − 2n ∼ a2n − 1 − 2(n − 1) = a2k+1 − a2k −2 =
k=1
a2
k=1 k
n−1
1X1

2 k=1 k
ln(n − 1)

2
ln(n)

2
Suy ra r
ln(n)
an = 2n + + o(ln(n))
s 2

 
ln(n) ln(n)
= 2n 1 + +o
4n n

  
ln(n) ln(n)
= 2n 1 + +o
8n n


 
2 ln(n) ln(n)
= 2n + √ +o √
8 n n
Như vậy √

 
2 ln(n) ln(n)
an − 2n = √ +o √
8 n n
Từ đây ta thu được  √ 
lim an − 2n = 0
n→+∞

Bài toán được giải quyết! 


Bài toán 60
Tính giới hạn √ √
cos x cos 2x · · · n cos nx − 1
lim
x→0 cos x cos 2x · · · cos nx − 1

Nguyen Viet Hung, Hanoi University of Science, Vietnam

Lời giải. AN-anduud Problem Solving Group, Ulaanbaatar, Mongolia


Trước tiên ta có đẳng thức sau
n−1 m
!
X Y
1 − cos x cos 2x · . . . · cos nx = cos kx · (1 − cos(m + 1)x)
m=0 k=0

LATEX by Mathpiad 55
TUYỂN TẬP DÃY SỐ TRONG 10 NĂM TỪ CÁC TẠP CHÍ TOÁN CỦA MỸ

Do đó
n−1 m
!
√ √ √
X Y 
k
p
m+1
1 − cos x · cos 2x · . . . n
cos nx = cos kx 1− cos(m + 1)x
m=0 k=0
n−1 m
!
X Y √
k (1 − cos(m + 1)x)
= cos kx · m
X p
m=0 k=0 m+1
(cos(m + 1)x)j
j=0

Ta có 2
m+1

x sin
1 − cos(m + 1)x 2  (m + 1)2 (m + 1)2
lim = lim · =

 m+1 
x→0 x2 x→0 2 2
x
2
Từ đây suy ra
n−1
X (m + 1)2 1
√ √ ·
cos x · cos 2x · . . . · n cos nx − 1 m=0
2 m+1
lim =
x→0 cos x · cos 2x · . . . · cos nx − 1 n−1
X (m + 1)2
m=0
2
n(n + 1)
2 3
= = .
n(n + 1)(2n + 1) 2n + 1
6

Như vậy bài toán được giải quyết. 


Bài toán 61
Với x0 > 1 là 1 số nguyên, ta định nghĩa xn+1 = d2 (xn ), trong đó d(k) làm hàm số các
ước dương của k. Chứng minh rằng

lim xn = 9
n→∞

Albert Stadler, Herrliberg, Switzerland

Lời giải. Li Zhou, Polk State College, Winter Haven, FL, USA
Vì x0 > 1, x1 = y 2 nên y > 2. Ta có pc11 · · · pemm là phân tích ra thừa số nguyên tố của y. Do đó

x2 = d2 y 2 = (2e1 + 1)2 · · · (2em + 1)2




Bây giờ ta sẽ chứng minh rằng với mọi số nguyên tố chẵn p và e > 1 thìp2e > (2e + 1)2 , đẳng
thức xảy ra khi và chỉ khi p = 3 và e = 1. Dễ thấy, p2 > 32 , bây giờ ta giả sử điều cần chứng
minh đúng với e > 1. Khi đó

p2(c+1) = p2 p2c > p2 (2e + 1)2 = (2pe + p)2 > (2e + 3)2


đúng theo quy nạp, do vậy xn > xn+1 với mọi n > 2, đẳng thức xảy ra khi và chỉ khi xn = 32 .
Do hxn in>2 bị chặn dưới nên tồn tại N > 2 sao cho x2 > x3 > · · · > xN và xN = 32 . Như vậy,
d2 (32 ) = 9, hoàn tất chứng minh. 

LATEX by Mathpiad 56
TẠP CHÍ VÀ TƯ LIỆU TOÁN HỌC

Bài toán 62
Cho α ∈ (0, 1) là một số vô tỉ bất kì. Ta xây dựng dãy {an } các số thực thỏa mãn hai
tính chất sau đây
   
1 n−1
• {an | n ∈ N} ⊂ |n∈N ∪ |n∈N ,
n n
a1 + a2 + · · · + an
• lim = α.
n→∞ n

Eugen J. Ionascu, Columbus State University, Columbus GA.

Lời giải. Northwestern University Math Problem Solving Group, Northwestern University, Evanston
IL.
Với mỗi n ∈ N, ta đặt 
 1

nếu bnαc = b(n − 1)αc
an = n − n .
1

 nếu bnαc > b(n − 1)αc
n
Trước tiên, ta có nhận xét sau đây
1 1
a1 + a2 + · · · + an = bnαc + 1 ± ± ··· ± . (1)
2 n
Với n = 1 thì b(n − 1)αc = 0 = b1 · αc, bởi vì α ∈ (0, 1). Từ đó ta thu được
1
a1 = = 0 + 1 = b1 · αc + 1.
1
1
Giả sử mệnh đề (1) đúng đến n = k. Nếu b(k + 1)αc = bkαc thì ta có ak+1 = và từ đó
k+1
ta thu được đẳng thức
1 1 1
a1 + a2 + · · · + ak + ak+1 = bkαc + 1 ± ± +
2 k k+1
1 1 1
= b(k + 1)αc + 1 ± · · · ± + .
2 k k+1
Mặt khác, nếu b(k + 1)αc > bkαc thì b(k + 1)αc = bkαc + 1, với α ∈ (0, 1), và do đó ta có
1 1 k
a1 + a2 + · · · + ak + ak+1 = bkαc + 1 ± ± +
2 k k+1
1 1 1
= bkαc + 1 ± · · · ± + 1 −
2 k k+1
1 1 1
= b(k + 1)αc + 1 ± · · · ± − .
2 k k+1
Do đó mệnh đề (1) cũng đúng với n = k + 1 nên theo nguyên lý quy nạp toán học mệnh đề
được chứng minh. Tiếp theo ta có
nα − bnαc 1 n→∞
06 < −→ 0.
n n
Từ đó ta thu được
bnαc n→∞
−→ 0,
n

LATEX by Mathpiad 57
TUYỂN TẬP DÃY SỐ TRONG 10 NĂM TỪ CÁC TẠP CHÍ TOÁN CỦA MỸ


1 1
1± ± ··· ±
2 n 6 1 + ln n n→∞
−→ 0.
n n

Do đó ta thu được kết quả bài toán. 


Bài toán 63
Chứng minh rằng với n > 4, tồn tại các số nguyên x1 , x2 , . . . , xn sao cho
n−2
x2n−1 + 1 Y x2k + 1
2
= 1,
x2n k=1
x k

và thỏa mãn các điều kiện dưới đây

• x1 = 1, xk−1 < xk < 3xk−1 với 2 6 k 6 n − 2.

• xn−2 < xn−1 < 2xn−2 với n > 3.

• xn−1 < xn < 2xn−1 với n > 2.

Herman Roelants, Catholic University of Leuven, Louvain, Belgium.

Lời giải. Chip Curtis, Missouri Southern State University, Joplin, MO.
Trước tiên ta đặt {Fn }n>0 là dãy Fibonacci, được định nghĩa như sau

F0 = 0, F1 = 1 và Fk+2 = Fk+1 + Fk , k > 0.

Tiếp theo ta đặt xn = F2n−3 , xn−1 = F2n−4 và xk = F2k−1 , 1 6 k 6 n − 2. Lưu ý sử dụng đồng
nhất thức sau đây
2
F2i−1 F2i+3 = F2i+1 + 1 và F2i−1 F2i+1 = F2i2 + 1,

đây là các trường hợp riêng của đẳng thức Catalan được phát biểu như sau

Fk2 − Fk+r Fn−r = (−1)k−r Fr2 .

Trước tiên, với k = 2i + 1 và r = 2, sau đó với k = 2i và r = 1 thì ta thu được


n−2 n−2 2
x2n−1 + 1 Y x2k + 1 2
F2n−4 + 1 F12 + 1 F32 + 1 Y F2k−1 +1
2 2
= 2
· 2
· 2 2
xn k=1
xk F2n−3 F1 F3 k=3 F2k−1
n−2
5 F2n−5 F2n−3 Y F2k−3 F2k+1
= · 2 2
2 F2n−3 k=3
F2k−1
n−3
Y n−1
Y
F2k−1 F2k−1
5 F2n−5 F2n−3 k=2 k=4
= · 2 n−2 n−2
2 F2n−3 Y Y
F2k−1 F2k−1
k=3 k=3
5 F2n−5 F2n−3 F3 F2n−3
= · 2
· · = 1.
2 F2n−3 F2n−5 F5

LATEX by Mathpiad 58
TẠP CHÍ VÀ TƯ LIỆU TOÁN HỌC

Với 2 6 k 6 n − 2 thì điều kiện xk−1 < xk < 3xk−1 tương đương với

F2k−3 < F2k−1 = F2k−3 + F2k−2 < 3F2k−3 ,

hoặc 0 < F2k−2 < 2F2k−3 . Điều kiện xn−1 < 2xn−2 trở thành

F2n−5 < F2n−4 < 2F2n−5 ,

đúng với n > 4. Cuối cùng, điều kiện xn−1 < xn < 2xn−1 trở thành

F2n−4 < F2n−3 < 2F2n−4 ,

cũng đúng với n > 4. Do đó ta thu được kết quả bài toán. 

2 Xấp xỉ Stirling
Ở trong tài liệu có một số bài toán sử dụng tới xấp xỉ Stirling, vì thế trong mục này chúng tôi
sẽ đề cập về nó cho bạn đọc. Xấp xỉ Stirling lần đầu được đưa ra bởi nhà toán học Abraham de
Moivre nhưng sau đó nó được đặt tên bởi nhà toán học James Stirling được ông công bố trong
công trình Methodus Differentialis sive Tractatus de Summatione et Interpolatione Serierum
Infinitarum (1730; "Phương pháp vi phân với một phân tích về tính tổng và nội suy của chuỗi
vô hạn "), một chuyên luận về chuỗi vô hạn, tổng, nội suy và cầu phương. Công thức xấp xỉ
Stirling này được phát biểu như sau
√  n n
n! ∼ 2πn ,
e
Công thức này nhìn có vẻ phát biểu phức tạp, nhưng đây lại là một trong những tiền đề quan
trọng để phát triển lý thuyết xác suất và thống kê, có ý nghĩa nhiều trong thống kê sinh học,
khoa học máy tính và nhiều ngành khoa học khác. Bây giờ vào phần chính của bài viết, ta sẽ
chứng minh xấp xỉ này! Trước tiên ta đề cập tới một công thức tính số π của nhà toán học
Wallis. Ta có r
2.4.6 . . . .2n 1 π
lim ·√ =
n→+∞ 3.5.7 . . . (2n − 1) 2n + 1 2
Để chứng minh đẳng thức này, trước tiên ta xét tích phân
Z π
2
In = (sin x)n dx
0
Z π
2
= (sin x)n−1 sin xdx
0
Z π
2
=− (sin x)n−1 d(cos x)
0
π
Z π
2
n−1
= − (sin x) 2
cos x 0 + cosxd(sin x)n−1
0
Z π
2
= (n − 1) cosx(sin x)n−2 cos xdx
0
Z π
2
1 − sin2 x (sin x)n−2 dx

= (n − 1)
0
Z π Z π
!
2 2
n−2 n
= (n − 1) (sin x) dx − (sin x) dx
0 0

LATEX by Mathpiad 59
TUYỂN TẬP DÃY SỐ TRONG 10 NĂM TỪ CÁC TẠP CHÍ TOÁN CỦA MỸ

Như vậy ta thiết lập được công thức truy hồi


n−1
In = (n − 1) (In−2 − In ) ⇔ nIn = (n − 1)In−2 ⇔ In = In−2
n

Ta có 
2n − 1

2n

 I 2n = I2n−2 
 I 2n+1 = I2n−1
2n 2n + 1

 

 
2n − 3 2n − 2

 

 

 I 2n−2 = I2n−4 
 I2n−1 = I2n−3
2n − 2 2n − 1

 


 

... ...

 


 

 
 3  4
I4 = I2 , I5 = I3

 4 
 5

 1 
 2

 I 2 = I0 
 I 3 = I1
2 3

 

 
π π

 


 Z2 
 Z2

 0 π 
 1

 I 0 = (sin x) dx = I1 = (sin x) dx = 1


2

 
 
0 0

Từ đây ta suy ra
2n − 1 2n − 3 3 1 π
I2n = · ··· · ·
2n 2n − 2 4 2 2

2n 2n − 2 4 2
I2n+1 = · ··· · · · 1
2n + 1 2n − 1 5 3
Do đó ta được
I2n+1 [2.4 . . . (2n − 2)(2n)]2 2
= 2 · (*)
I2n [3.5 . . . ..(2n − 1)] (2n + 1) π

I2n+1 2n
=
I2n−1 2n + 1
h πi
Ta có sin x ∈ [0, 1], ∀x ∈ 0, ⇒ (sin x)2n+1 6 (sin x)2n 6 (sin x)2n−1 , suy ra
2
2n I2n+1 I2n+1
= < 61
2n + 1 I2n−1 I2n

Mặt khác
2n 2 I2n+1
lim = lim = 1 ⇒ lim =1
n→+∞ 2n + 1 n→+∞ 1 n→+∞ I2n
2+
n
Kết hợp với điều kiện (∗) ta suy ra
r
2.4.6 . . . .2n 1 π
lim ·√ =
n→∞ 3.5.7 . . . .(2n − 1) 2n + 1 2

Điều phải chứng minh.


Bây giờ quay lại bài toán của chúng ta, chúng ta sẽ chứng minh một kết quả mạnh hơn
√  n n √  n n 1
2nπ < n! < 2nπ e 12n
e e

LATEX by Mathpiad 60
TẠP CHÍ VÀ TƯ LIỆU TOÁN HỌC

Ta có
Z k+1 Z k+1 Z k+1
ln k 6 ln x 6 ln(k + 1), ∀x ∈ [k, k + 1] ⇒ lnkdx 6 lnxdx 6 ln(k + 1)dx
k k k
Z k+1
k+1
⇒ ln kx|k 6 lnxdx 6 ln(k + 1)x|k+1
k
k
Z k+1
⇒ ln k 6 lnxdx 6 ln(k + 1)
k

Mặt khác  Z 1 Z 2
lnxdx 6 ln 1 6 lnxdx




 0 1
2 3

 Z Z

lnxdx 6 ln 2 6 lnxdx

1 2
···



 Z n Z n+1


lnxdx 6 ln n 6 lnxdx



n−1 n

Suy ra
Z 1 Z n Z 2 Z n+1
lnxdx + . . . + lnxdx 6 ln 1 + ln 2 + . . . + ln n 6 lnxdx + . . . + lnxdx
0 n−1 1 n

Như vậy ta được Z π Z n+1


lnxdx 6 ln(n!) 6 lnxdx
0 1

Sử dụng nguyên hàm từng phần ta có


Z Z Z
lnxdx = x ln x − xd(ln x) = x ln x − dx = x ln x − x

Suy ra Z n
lnxdx = (x ln x − x)|n0 = n ln n − n;
0

Hoàn toàn tương tự tích phân trên, ta cũng suy ra được


Z n+1
lnxdx = (x ln x − x)|n+1
1 = (n + 1) ln(n + 1) − n
1

Do đó
n ln n − n 6 ln(n!) 6 (n + 1) ln(n + 1) − n
Ta đặt  
1 2n + 1 n + 1
dn = ln n! − n + ln n + n ⇒ dn − dn+1 = ln −1
2 2 n
1 1 1+t t3 t5
Thay t = vào khai triển Maclaurin ln = t + + + ..., suy ra
2n + 1 2 1−t 3 5
1
1 n+1 1 1 + 2n + 1 1 1 1
ln = ln = + + 5 + ...
2 n 2 1 2n + 1 3(2n + 1) 3
5(2n + 1)
1−
2n + 1

LATEX by Mathpiad 61
TUYỂN TẬP DÃY SỐ TRONG 10 NĂM TỪ CÁC TẠP CHÍ TOÁN CỦA MỸ

Suy ra
2n + 1 n + 1 1 1
dn − dn+1 = ln −1= 2 + + ...
2 n 3(2n + 1) 5(2n + 1)4
Tóm lại ta có
   
1 1 1 1 1 1
0 < dn − dn+1 < + + ... = −
3 (2n + 1)2 (2n + 1)4 12 n n + 1

1 1
Từ đó suy ra dn+1 < dn là dãy giảm; dn − < dn+1 − là dãy tăng.
12n 12(n + 1)
Từ đây suy ra tồn tại một số c sao cho
 
1
lim dn − = lim dn = c
n→+∞ 12n n→+∞

và đồng thời
1 θ
dn − < c < d n ⇒ dn = c + , ∀θ ∈ (0, 1)
12n 12n
1 n!
⇒ ec = lim edn = lim eln n!−(n+ 2 ) ln n+n = lim 1
n→+∞ n→+∞ n→+∞ e−n nn+ 2

(n!)2 (2n)! (2n)!


⇒ lim −2π 2n+1
= e2c ; lim √ = lim
2n+ 12
= ec
n→+∞ e n n→+∞ e−2n n2n n2n 2n n→+∞ −2n
e (2n)

Từ công thức Wallis r


2.4.6 . . . .2n 1 π
lim ·√ =
n→∞ 3.5.7 . . . .(2n − 1) 2n + 1 2
Ta suy ra

(2.4.6 . . . .2n)2
r
π 1
= lim ·√
2 n→+∞ 2.3.4 . . . ..(2n) 2n + 1
n 2

(2 n!) 2n
= lim ·
n→+∞ (2n)! 2n
2

(n!) 22n 2n
= lim ·
n→+∞ 2n (2n)!
√ !
1 (n!)2 e−2n 22n n2n 2n
= lim · ·
n→+∞ 2 e−2n n2n+1 (2n)!
e2c
= lim
n→+∞ 2ec
ec
=
2

c
√ 2π dn n!
Suy ra e = 2π ⇒ c
e = edn = 1 , như vậy ta thu được
e e−n nn+ 2

−n n+ 12 2π dn
n! = e n e
ec√
1 θ
= nn+ 2 e 12n −n 2π(0 < θ < 1)

LATEX by Mathpiad 62
TẠP CHÍ VÀ TƯ LIỆU TOÁN HỌC

Vậy
√ θ √  n n θ √  n n √  n n 1
n! = nn e−n 2nπe l2n = 2nπ e 12n ⇒ 2nπ < n! < 2nπ e 12n
e e e
Vậy ta có điều phải chứng minh.

LATEX by Mathpiad 63
TUYỂN TẬP DÃY SỐ TRONG 10 NĂM TỪ CÁC TẠP CHÍ TOÁN CỦA MỸ

Tài liệu
[1] The American Mathematical Monthly −
https://www.tandfonline.com/loi/uamm20

[2] Mathematics Magazine −


https://www.tandfonline.com/loi/umma20

[3] The College Mathematics Journal −


https://www.tandfonline.com/loi/ucmj20

[4] Mathematical Reflections Journal −


https://www.awesomemath.org/mathematical-reflections/

LATEX by Mathpiad 64
VIỆT NGUYỄN

HÀM SỐ TRONG
IMO SHORTLIST

M THPI D
¨ PHƯƠNG TRÌNH HÀM TRONG IMO SHORTLIST
Hàm số trong IMO Shortlist

Nguyễn Hoàng Việt − A2K54 KHTN

Mathpiad − Tạp chí toán học +


Chương I
Lời nói đầu và các kí hiệu

¨ PHƯƠNG TRÌNH HÀM TRONG IMO SHORTLIST


Hàm số hiện là một phân môn vô cùng đặc sắc trong lĩnh vực toán olympic và cũng nhận được sự
quan tâm, yêu thích của rất nhiều học sinh chuyên toán trong và ngoài nước. Các bài toán hàm số
mang vẻ đẹp rất cuốn hút, với những phép thế vô cùng bất ngờ. Ở phân môn này cũng có các bài
toán khó và rất khó, đòi hỏi người làm phải tập trung cao độ mới có thể làm được. Bài toán khó và lạ
sẽ mang đến cho ta những phương pháp mới trong việc giải các bài toán. Một địa điểm chứa những
bài toán đó là IMO Shortlist. Nhận thấy các bài toán này đều lạ và khó nhưng được viết bằng tiếng
anh khiến cho nhiều học sinh kém tiếng anh "ngại" đọc do phải vừa đọc, vừa hiểu, vừa phải dịch,
tôi đã có ý tưởng dịch lại các bài hàm số này sang tiếng việt.(tại tôi dốt tiếng anh À‚). Nếu có sai
sót, các bạn đọc hãy liên hệ cho tôi tại https://www.facebook.com/nguyenhoangvietchuyenkhtn/.

¨ Một số kí hiệu trong sách:

1 f n (m) = f (f (...f (m)...)) là hàm hợp n lần của m.


| {z }
n

2 Kí hiệu P (a, b) là thay (x, y) bởi (a, b) vào đề bài với (x, y) là các biến thỏa mãn điều kiện
đề bài.
3 Z là tập số nguyên.
4 Z+ là tập số nguyên dương.
5 N là tập số tự nhiên.
6 R là tập số thực.
7 Q là tập số hữu tỉ.
8 P là tập số nguyên tố.

1
Chương II
Bài toán hàm số qua các năm

1. Bài toán hàm số đại số

 Bài 1 Cho hàm số f : Z+ → Z+ thỏa mãn

f (m + n) > f (m) + f (f (n)) − 1, ∀ m, n ∈ Z+


LATEX VÀ DỊCH THUẬT BỞI VIỆT

Tìm tất cả các giá trị có thể của f (2007)

A2 IMO Shortlist 2007

 Bài 2 Cho tập hợp S ⊆ R. Ta gọi một cặp hàm số (f, g) : S → S là có gấu nếu thỏa mãn
các điều kiện

(i) Cả hai hàm f, g đều tăng ngặt.

(ii) f (g(g(x))) < g(f (x)) với mọi x ∈ S

Hỏi liệu có tồn tại một cặp có gấu hay không nếu

(a) S = Z+
ß ™
1 +
(b) S = a − : a, b ∈ Z
b

A3 IMO Shortlist 2008

 Bài 3 Với mỗi m ∈ Z, đặt t(m) ∈ {1, 2, 3} sao cho 3 | m + t(m). Cho hàm số f : Z → Z
thỏa mãn f (−1) = 0, f (0) = 1, f (1) = −1 và

f (2n + m) = f (2n − t(m)) − f (m), ∀ m, n ∈ N với 2n > m

Chứng minh rằng f (3p) > 0 với mọi số nguyên p > 0

A4 IMO Shortlist 2008

2
 Bài 4 Cho hàm số f : R → Z+ thỏa mãn
Å ã Å ã
1 1
f x+ =f y+ , ∀ x, y ∈ R
f (y) f (x)

Chứng minh rằng tồn tại một số nguyên dương a sao cho không tồn tại n để f (n) = a.

A6 IMO Shortlist 2008

¨ PHƯƠNG TRÌNH HÀM TRONG IMO SHORTLIST


 Bài 5 Tìm tất cả các hàm số f : Z+ → Z+ sao cho với mọi x, y thì tồn tại một tam giác
có độ dài ba cạnh là
x, f (y) và f (y + f (x) − 1)

A3 IMO Shortlist 2009

 Bài 6 Cho hàm số f : R → R. Chứng minh rằng tồn tại hai số thực x, y mà

f (x − f (y)) > yf (x) + x

A4 IMO Shortlist 2009

 Bài 7 Tìm tất cả các hàm số f : R → R thỏa mãn

f (xf (x + y)) = f (yf (x)) + x2 , ∀ x, y ∈ R

A7 IMO Shortlist 2009

 Bài 8 Tìm tất cả các hàm số f : Q+ → Q+ thỏa mãn

f (f (x)2 y) = x3 f (xy), ∀ x, y ∈ Q+

A5 IMO Shortlist 2010

 Bài 9 Cho các hàm số f, g : Z+ → Z+ thỏa mãn

f (g(n)) = f (n) + 1 và g(f (n)) = g(n) + 1 với mọi n ∈ Z+

Chứng minh rằng f (n) = g(n), ∀ n ∈ Z+

A6 IMO Shortlist 2010

 Bài 10 Tìm tất cả các hàm số f, g : R → R thỏa mãn

g(f (x + y)) = f (x) + (2x + y)g(y), ∀ x, y ∈ R

A3 IMO Shortlist 2011

+ Do Math then Love Math 3 h Việt Nguyễn − Mathpiad


 Bài 11 Tìm tất cả các hàm số f, g : Z+ → Z+ thỏa mãn

f g(n)+1 (n) + g f (n) (n) = f (n + 1) − g(n + 1) + 1, ∀ n ∈ Z+

A4 IMO Shortlist 2011

 Bài 12 Tìm tất cả các hàm số f : R → R thỏa mãn

f (x + y) 6 yf (x) + f (f (x)), ∀ x, y ∈ R

Chứng minh rằng f (x) = 0 với mọi x 6 0

A6 IMO Shortlist 2011

 Bài 13 Tìm f : Z → Z sao cho với mọi a, b, c ∈ Z thỏa mãn a + b + c = 0 thì


LATEX VÀ DỊCH THUẬT BỞI VIỆT

f 2 (a) + f 2 (b) + f 2 (c) = 2f (a)f (b) + 2f (b)f (c) + 2f (c)f (a)

A1 IMO Shortlist 2012

 Bài 14 Tìm tất cả các hàm số f : R → R thỏa mãn f (−1) 6= 0 và

f (1 + xy) − f (x + y) = f (x)f (y), ∀ x, y ∈ R

A5 IMO Shortlist 2012

 Bài 15 Cho hàm số f : Z+ → Z+ . Giả sử rằng với mỗi số n ∈ Z+ , tồn tại một số k ∈ Z+
để f 2k (n) = n + k và đặt kn là số k nhỏ nhất có tính chất này. Chứng minh dãy k1 , k2 , ... không
bị chặn.

A6 IMO Shortlist 2012

 Bài 16 Tìm tất cả các hàm số f : Q>0 → R thỏa mãn


®
f (x)f (y) > f (xy) (1)
f (x + y) > f (x) + f (y) (2)

Với mọi số hữu tỉ dương x, y. Chứng minh rằng f (x) = x, ∀ x ∈ Q>0 , biết tồn tại một số hữu
tỉ dương a > 1 mà f (a) = a

A3 IMO Shortlist 2013

 Bài 17 Tìm tất cả các hàm số f : N → N thỏa mãn

f (f (f (n))) = f (n + 1) + 1, ∀ n ∈ N

A5 IMO Shortlist 2013

+ Do Math then Love Math 4 h Việt Nguyễn − Mathpiad


 Bài 18 Tìm tất cả các hàm số f : Z → Z thỏa mãn

f (f (m) + n) + f (m) = f (n) + f (3m) + 2014, ∀ m, n ∈ Z

A4 IMO Shortlist 2014

 Bài 19 Tìm tất cả các hàm số f : Z → Z thỏa mãn

n2 + 4f (n) = f (f (n))2 , ∀ n ∈ Z

¨ PHƯƠNG TRÌNH HÀM TRONG IMO SHORTLIST


A6 IMO Shortlist 2014

 Bài 20 Tìm tất cả các hàm số f : R → R thỏa mãn

f (x + f (x + y)) + f (xy) = x + f (x + y) + yf (x)

A4 IMO Shortlist 2015

 Bài 21 Kí hiệu 2Z + 1 là tập các số nguyên lẻ. Tìm tất cả các hàm số f : Z → 2Z + 1 thỏa
mãn
f (x + f (x) + y) + f (x − f (x) + y) = f (x + y) + f (x − y), ∀ x, y ∈ Z

A5 IMO Shortlist 2015

 Bài 22 Tìm tất cả các hàm số f : R+ → R+ thỏa mãn

xf (x2 )f (f (y)) + f (yf (x)) = f (xy) f (f (x2 )) + f (f (y 2 )) , ∀ x, y ∈ R+


 

A4 IMO Shortlist 2016

 Bài 23 Tìm tất cả các hàm số f : R → R thỏa mãn f (0) 6= 0 và

f (x + y)2 = 2f (x)f (y) + max{f (x2 ) + f (y 2 ), f (x2 + y 2 )}

A7 IMO Shortlist 2016

 Bài 24 Tìm tất cả các hàm số f : R → R sao cho

f (f (x)f (y)) + f (x + y) = f (xy), ∀ x, y ∈ R

A6 IMO Shortlist 2017

+ Do Math then Love Math 5 h Việt Nguyễn − Mathpiad


 Bài 25 Tìm tất cả các hàm số f : R → R thỏa mãn điều kiện sau

Với mọi x, y ∈ R sao cho (f (x) + y) (f (y) + x) > 0, ta có f (x) + y = f (y) + x

Chứng minh rằng f (x) + y 6 f (y) + x khi và chỉ khi x > y

A8 IMO Shortlist 2017

 Bài 26 Tìm tất cả các hàm số f : Q>0 → Q>0 thỏa mãn

f (x2 f (y)2 ) = f (x)2 f (y), ∀ x, y ∈ Q>0

A1 IMO Shortlist 2018

 Bài 27 Tìm tất cả các hàm số f : (0, ∞) → R thỏa mãn


LATEX VÀ DỊCH THUẬT BỞI VIỆT

Å ã
1 y
x+ f (y) = f (xy) + f , ∀ x, y ∈ R+
x x

A5 IMO Shortlist 2018

 Bài 28 Xét hàm số f : Z → Z thỏa mãn

f (f (x + y) + y) = f (f (x) + y), ∀ x, y ∈ Z

Ta gọi một số nguyên v là hiếm nếu tập hợp Xv = {x ∈ Z : f (x) = v} là hữu hạn và khác
rỗng.

1 Chứng minh rằng tồn tại một hàm số f thỏa mãn có một số hiếm.

2 Chứng minh rằng không tồn tại hàm số f nào có nhiều hơn 1 số hiếm.

A7 IMO Shortlist 2019

 Bài 29 Tìm tất cả các hàm số f : Z → Z sao cho

fa2 +b2 (a + b) = af (a) + bf (b), ∀ a, b ∈ Z

Với kí hiệu fn (x) là hợp n lần hàm số x.

A6 IMO Shortlist 2020

 Bài 30 Tìm tất cả các hàm số f : R+ → R+ thỏa mãn

f (x + f (xy)) + y = f (x)f (y) + 1, ∀ x, y ∈ R+

A8 IMO Shortlist 2020

+ Do Math then Love Math 6 h Việt Nguyễn − Mathpiad


2. Bài toán hàm số số học

 Bài 1 Tìm tất cả các hàm f : Z+ → Z+ toàn ánh sao cho với mọi m, n thì f (m + n) chia
hết cho p khi và chỉ khi f (m) + f (n) chia hết cho p

N5 IMO Shortlist 2007

 Bài 2 Với mọi n ∈ Z+ , kí hiệu d(n) là số ước dương của n. Tìm tất cả các hàm số

¨ PHƯƠNG TRÌNH HÀM TRONG IMO SHORTLIST


f : Z+ → Z+ thỏa mãn các tính chất sau

(i) d(f (x)) = x, ∀ x ∈ Z+ .

(ii) f (xy) | (x − 1)y xy−1 f (x), ∀ x, y ∈ Z+ .

N5 IMO Shortlist 2008

 Bài 3 Cho hàm số khác hằng f : Z+ → Z+ thỏa mãn

a − b | f (a) − f (b), ∀ a 6= b ∈ Z+

Chứng minh rằng tồn tại vô hạn số nguyên tố p mà p | f (c) với c ∈ Z+ .

N3 IMO Shortlist 2009

 Bài 4 Tìm tất cả các hàm số f : Z+ → Z+ sao cho (f (m) + n) (f (n) + m) là số chính
phương.

N5 IMO Shortlist 2010

 Bài 5 Tìm tất cả các hàm số f : Q → Z thỏa mãn


Å ã
f (x) + a x + a
f =f
b b

Với mọi x ∈ Q, a ∈ Z và b ∈ Z+ .

N6 IMO Shortlist 2013

+ Do Math then Love Math 7 h Việt Nguyễn − Mathpiad


 Bài 6 Xét hàm số f : Z+ → Z+ . Kí hiệu f n (m) = f (f (...f (m)...)). Giả sử f có hai tính
| {z }
n
chất sau
f n (m) − m
1 Nếu m, n ∈ Z+ thì ∈ Z+
n
2 Tập Z+ \ {f (n) : n ∈ Z+ } là hữu hạn.

Chứng minh rằng dãy f (1) − 1, f (2) − 2, ... tuần hoàn.

N6 IMO Shortlist 2015

 Bài 7 Với số nguyên dương k bất kì, ta gọi hàm f : Z+ → Z+ là k-nice nếu
gcd (f (n) + m, f (m) + n) 6 k với mọi n 6= m. Tìm tất cả các số nguyên dương k sao cho
tồn tại hàm k-nice.
LATEX VÀ DỊCH THUẬT BỞI VIỆT

N7 IMO Shortlist 2015

 Bài 8 Cho hàm số f : Z+ → Z>1 thỏa mãn f (m + n) | f (m) + f (n), ∀ m, n ∈ Z+ . Chứng


minh rằng tồn tại một số nguyên C sao cho C | f (x), ∀ x ∈ Z+

N6 IMO Shortlist 2018

 Bài 9 Tìm tất cả các hàm số f : Z+ → Z+ thỏa mãn tồn tại số nguyên C sao cho

a + f (b) | a2 + bf (a), với mọi a + b > C

N4 IMO Shortlist 2019

 Bài 10 Tìm tất cả các hàm số f : Z+ → N thỏa mãn

i) tồn tại t để f (t) 6= 0

ii) f (xy) = f (x) + f (y), ∀ x, y > 0

iii) Tồn tại vô hạn số nguyên dương n để f (k) = f (n − k) với mọi k < n

N5 IMO Shortlist 2020

+ Do Math then Love Math 8 h Việt Nguyễn − Mathpiad


Chương III
Lời Giải

¨ PHƯƠNG TRÌNH HÀM TRONG IMO SHORTLIST


1. Hàm số đại số

 Bài 1 Cho hàm số f : Z+ → Z+ thỏa mãn

f (m + n) > f (m) + f (f (n)) − 1, ∀ m, n ∈ Z+

Tìm tất cả các giá trị có thể của f (2007)

A2 IMO Shortlist 2007

Lời giải . Xét m > n, ta có

f (m) = f (n + (m − n)) > f (n) + f (f (m − n)) − 1 > f (n)

Nên f là hàm không giảm. Ta bỏ qua trường hợp tầm thường f ≡ 1.


Gọi a là số nhỏ nhất mà f (a) > 1. Khi đó f (b) > f (a) > 1 với mọi b > a.
Gọi A là tập các số n mà n ∈ Z+ và f (n) > n. Ta có

f (f (n)) = f ((f (n) − n) + n) > f (f (n) − n) + f (f (n)) − 1 ⇒ 1 > f (f (n) − n)

Suy ra f (n) − n 6 a với mọi n ∈ A. Vậy tồn tại

c = max{f (n) − n : n ∈ Z+ }

Giả sử c = f (k) − k > 0, ta có

2k + c > f (2k) = f (k + k) > f (k) + f (f (k)) − 1


> f (k) + f (k) − 1 = 2(k + c) − 1 = 2k + c + c − 1

Suy ra c 6 1 hay f (2007) 6 2008.


Ta xây dựng hàm f để f (2007) có thể nhận được tất cả các giá trị trong [1, 2008] như sau
®
n nếu 2007 - n
fj (n) = max{1, n + j − 2007} với j = 1, 2, ..., 2007; f2008 (n) =
n + 1 nếu 2007 | n

Ta chứng minh hàm số fj thỏa mãn nếu fj (2007) = j

¨ Trường hợp 1. j 6 2007. Ta có fj là hàm không giảm và fj (n) 6 n, ∀ n ∈ Z+ .

Nếu fj (m) = 1 thì

fj (m + n) > fj (n) > fj (fj (n)) = fj (m) + fj (fj (n)) − 1

9
Nếu fj (m) > 1 thì

fj (m) + fj (fj (n)) − 1 > (m + j − 2007) + n = (m + n) + j − 2007 = fj (m + n)

¨ Trường hợp 2. j = 2008. Ta có n + 1 > f2008 (n) > n, ∀ n ∈ Z+ .


Hơn nữa, ta cũng có f2008 (f2008 (n)) 6 n + 1(dễ chứng minh).

Nếu 2007 | m + n thì

f2008 (m + n) = m + n + 1 = (m + 1) + (n + 1) − 1 > f2008 (m) + f2008 (f2008 (n)) − 1

Nếu 2007 - m + n thì 2007 - n hoặc 2007 - m nên

[f2008 (m) + f2008 (f2008 (n))] − 1 6 (m + n + 1) − 1 = m + n = f (m + n)

â
LATEX VÀ DỊCH THUẬT BỞI VIỆT

 Bài 2 Cho tập hợp S ⊆ R. Ta gọi một cặp hàm số (f, g) : S → S là có gấu nếu thỏa mãn
các điều kiện

(i) Cả hai hàm f, g đều tăng ngặt.

(ii) f (g(g(x))) < g(f (x)) với mọi x ∈ S

Hỏi liệu có tồn tại một cặp có gấu hay không nếu

(a) S = Z+
ß ™
1 +
(b) S = a − : a, b ∈ Z
b

A3 IMO Shortlist 2008

Lời giải .
(a) Từ (i), ta được f (x) > x và g(x) > x với mọi x ∈ Z+ .
Ta chứng minh quy nạp rằng g k (x) 6 f (x) với mọi k > 0 và x ∈ Z+ . Ta có

g(g k+1 (x)) = g k (g 2 (x)) 6 f (g 2 (x)) < g(f (x)) ⇒ g k+1 (x) < f (x) (1)

Nếu g(x) = x, ∀ x ∈ Z+ thì f (g 2 (x)) = f (x) = g(f (x)), mâu thuẫn với (ii).
Nên tồn tại x0 mà g(x0 ) > x0 .
Xét dãy (xn ) xác định bởi xn = g n (x) với mọi n ∈ Z+ . Do g tăng ngặt nên dãy (xn ) cũng tăng
ngặt. Mặt khác, từ (1), ta thấy rằng dãy (xn ) bị chặn bởi g(f (x0 )), mâu thuẫn.
Vậy với S = Z+ thì không tồn tại cặp có gấu.
Å ã Å ã
1 1 1 1
(b) Đặt f a − = a + 1 − và g a − =a− với mọi a, b ∈ Z+ .
b b b b + 3a
Hiển nhiên hàm f là tăng ngặt. Ta chứng minh g cũng tăng ngặt.
1 1 1 1
Giả sử a − > c − với a, b, c, d ∈ Z+ . Nếu a 6 c − 1 thì c − > c − 1 > a > a − , vô lí.
b d d b
1 1
Vậy, ta được a > c. Ta có a − c > − . Ta có
b d
Å ã Å ã
1 1 1 1 1 1
g d− >g c− ⇔a− a
>c− c
⇔a−c> a

b d b+3 d+3 b+3 d + 3c

+ Do Math then Love Math 10 h Việt Nguyễn − Mathpiad


Vậy, ta chỉ cần chứng minh
1 1 1 1
− > −
b d b + 3a d + 3 c
Ta có
1 1 d − b + 3c − 3a d−b d−b 1 1
a
− c
= a c
6 a c
6 = −
b+3 d+3 (b + 3 )(d + 3 ) (b + 3 )(d + 3 ) bd b d
Cuối cùng, ta chỉ cần chứng minh f thỏa mãn điều kiện (ii)
Å Å Å ããã Å Å ãã
1 1 1 1
f g g a− = (a + 1) − < (a + 1) − =g f a−
b b + 2.3a b + 3a+1 b

¨ PHƯƠNG TRÌNH HÀM TRONG IMO SHORTLIST


â

 Bài 3 Với mỗi m ∈ Z, đặt t(m) ∈ {1, 2, 3} sao cho 3 | m + t(m). Cho hàm số f : Z → Z
thỏa mãn f (−1) = 0, f (0) = 1, f (1) = −1 và

f (2n + m) = f (2n − t(m)) − f (m), ∀ m, n ∈ N với 2n > m

Chứng minh rằng f (3p) > 0 với mọi số nguyên p > 0

A4 IMO Shortlist 2008

Lời giải . Với k > 0, ta chứng minh quy nạp

f (22k+1 − 3) = 0, f (22k+1 − 2) = 3k , f (22k+1 − 1) = −3k


f (22k+2 − 3) = −3k , f (22k+2 − 2) = −3k , f (22k+2 − 1) = 2.3k

Cho m = 0, n = 1 vào đề bài, ta được

f (21 ) = f (21 − 3) − 1 = f (−1) − 1 = −1

Cho m = n = 1 vào đề bài, ta được

f (3) = f (0) − f (1) = 1 − −1 = 2

Vậy ta đã chứng minh điều quy nạp với trường hợp k = 0.


Theo giả thiết quy nạp, ta có

f (22k+1 − 3) = f (22k + 22k − 3) = f (22k − t(22k − 3)) − f (22k − 3) = f (22k − 2) − f (22k − 3) = 0

Chứng minh tương tự với các trường hợp còn lại.


Như vậy, ta được f (2n − t(m)) > 3(n−1)/2 nếu 3 | 2n − t(m) và f (2n − t(m)) 6 0 nếu 3 - 2n − t(m).
Lại có 3 | m + t(m) nên ta được

1 f (2n − t(m)) > 3(n−1)/2 nếu 3 | 2n + m.

2 f (2n − t(m)) 6 0 nếu 3 - 2n + m.


2
Cũng từ khẳng định trên, ta được |f (2n − t(m))| 6 .3n/2 với mọi m, n > 0.
3
Bây giờ, ta sẽ chứng minh quy nạp theo n rằng |f (m)| 6 3n/2 với mọi m, n > 0 và m < 2n . (*)
Trường hợp n = 0, n = 1 là hiển nhiên. Giả sử (*) đúng đến n.

+ Do Math then Love Math 11 h Việt Nguyễn − Mathpiad


Theo giả thiết quy nạp, ta chỉ cần chứng minh (*) đúng với mọi 2n+1 > m > 2n .
2
Đặt m = 2n + k, ta có |f (2n − t(k)) | 6 .3n/2 nên
3
2
f (m) = |f (2n − t(k)) − f (k)| 6 |f (2n − t(k))| + |f (k)| 6 .3n/2 + 3n/2 < 3(n+1)/2
3
Vì 3p không là lũy thừa của 2 nên tồn tại a > b > 0 và 2b > c > 0 sao cho 3p = 2a + 2b + c.
Ta có
f (3p) = f (2a + 2b + c) = f (2a − t(2b + c)) − f (2b − t(c)) + f (c)
Do 3 | 2a + 2b + c ⇒ 3 | 2a − t(2b + c) nên

f (2a − t(2b + c)) > 3(a−1)/2

Vì 3 - 2b + c nên f (2b − t(c)) 6 0.


Ta lại có do c > 2b nên |f (c)| 6 3b/2 hay −f (c) > −3b/2 > −3(a−1)/2 do a − 1 > b.
Vậy, ta được
LATEX VÀ DỊCH THUẬT BỞI VIỆT

f (3p) > 3(a−1)/2 − 0 − 3(a−1)/2 = 0


Kết thúc chứng minh. â

 Bài 4 Cho hàm số f : R → Z+ thỏa mãn


Å ã Å ã
1 1
f x+ =f y+ , ∀ x, y ∈ R (1)
f (y) f (x)

Chứng minh rằng tồn tại một số nguyên dương a sao cho không tồn tại n để f (n) = a.

A6 IMO Shortlist 2008

Lời giải . Giả sử rằng f (R) = Z+ . Giả sử tồn tại a để f (a) = 1.


Xét hàm số g(x) = f (x + a). P (x + a, y + a), ta được
Å ã Å ã Å ã Å ã
1 1 1 1
g x+ =f x+a+ =f y+a+ =g y+
f (y) f (y + a) f (x + a) f (x)

Nên g cũng thỏa mãn các điều kiện như đề bài. Nên không mất tính tổng quát, giả sử f (0) = 1.
ß Å ã ™
1
! Ta chứng minh với một số c bất kì, ta có f c +
n
:n∈Z +
= Z+ .

Chứng minh. Thật vậy, ta có


ß Å ã ™
1
f (R) = f x + :x∈R
f (c)
ß Å ã ™ ß Å ã ™
1 1 +
f c+ :x∈R ⊂ f c+ :n∈Z ⊂ f (R)
f (x) n
1
Áp dụng với c = 0 và c = , ta được
3
ß Å ã ™ ß Å ã ™
1 + 1 1 +
f :n∈Z = f + :n∈Z = Z+ (2)
n 3 n

+ Do Math then Love Math 12 h Việt Nguyễn − Mathpiad


Nếu f (u) = f (v) thì f (u + q) = f (v + q) với mọi q ∈ Q >0 .
! Hơn nữa, nếu f (q) = 1 thì f (kq) = 1, ∀ k ∈ Z . +

Chứng minh. Với mọi x ∈ R, ta có


Å ã Å ã Å ã Å ã
1 1 1 1
f u+ =f x+ =f x+ =f v+
f (x) f (u) f (v) f (x)

Vì f (R) = Z+ , ta có Å ã Å ã
1 1
f u+ =f u+ , ∀ n ∈ Z+ (3)

¨ PHƯƠNG TRÌNH HÀM TRONG IMO SHORTLIST


n n
k
Đặt q = là một số hữu tỉ. Áp dụng (3) k lần, ta có
n
Å ã Å ã
k k
f (u + q) = f u + =f v+ = f (v + q) (*)
n n

! f (q) = f (q + 1) với mọi số hữu tỉ không âm q.


Å ã
1
Chứng minh. Theo (2), tồn tại số nguyên dương m để f = 1 = f (0).
m
Khi đó, theo (*), ta có Å ã Å ã
1 2
f (0) = f =f = ... = f (1)
m m
Vậy lại theo (*), ta được f (q) = f (q + 1).
Å ã
1
! f
n
= n với mọi n ∈ Z+

Chứng minh. Xét số q ∈ Q>0 bất kì, P (q, 0), ta được


Å ã
1
f (q + 1) = f = f (q)
f (q)
Å ã
+ + 1
Từ (2), với mỗi n ∈ Z , tồn tại k ∈ Z sao cho f = n. Ta có
k
Ü ê
Å ã Å ã
1 1 1
n=f =f Å ã =f
k 1 n
f
k
Å ã
+ 1 1
Trở lại bài toán. Theo (2), giả sử tồn tại n ∈ Z sao cho f + = 1.
3 n
1 1 s
Đặt + = với gcd(s, t) = 1. Dễ chứng minh t 6= 1. Ta chọn k, ` sao cho ks − `t = 1.
3 n t
Từ (*), ta suy ra nếu f (x) =Åf (0)
ã với x > 0 thì fÅ(kx)ã = 0. Å ã Å ã
s ks ks 1 1
Do f (0) = f = 1 nên f = 1. Lại có f =f +` =f = t, vô lí.
t t t t t
Vậy giả sử phản chứng sai. Kết thúc chứng minh. â

+ Do Math then Love Math 13 h Việt Nguyễn − Mathpiad


 Bài 5 Tìm tất cả các hàm số f : Z+ → Z+ sao cho với mọi x, y thì tồn tại một tam giác
có độ dài ba cạnh là
x, f (y) và f (y + f (x) − 1)

A3 IMO Shortlist 2009

Lời giải .
¨ Bước 1. Ta chứng minh f (1) = 1.
Giả sử f (1) = 1 + m với m > 0. Ta thấy rằng tồn tại một tam giác có ba cạnh là 1, f (y), f (y + m).
Như vậy 1 > |f (y) − f (y + m)| nên |f (y) − f (y + m)| = 0 hay f (y) = f (y + m).
Vậy hàm f tuần hoàn theo chu kì m. Hay f bị chặn.
Giả sử f (x) 6 B với mọi x thì nếu ta chọn x > 2B, ta được

x > 2B > f (y) + f (y + f (x) − 1)


LATEX VÀ DỊCH THUẬT BỞI VIỆT

Nên x, f (y), f (y + f (x) − 1) không phải là ba cạnh của một tam giác. Vậy m = 0 hay f (1) = 1.
¨ Bước 2. Ta có x, f (1) = 1, f (1 + f (x) − 1) = f (f (x)) là ba cạnh của một tam giác nên
f (f (x)) = x với mọi x.

¨ Bước 3.Ta chứng minh với mọi số nguyên z > 1 thì f (z) 6 z.

Giả sử tồn tại z để f (z) > z + 1. Đặt f (z) = A + 1. Ta có A > z > 2.


Đặt M = max{f (1), f (2), ..., f (A)}. Ta chứng minh không tồn tại số t mà
z−1
f (t) > t+M
A
Giả sử t = eA + f là số nhỏ nhất như vậy. Do cách chọn M nên ta được t > A.
Do tồn tại một tam giác có ba cạnh là z,f (t − A) và f (t − A + f (z) − 1) nên

z + f (t − A) > f (t)

Suy ra
z−1
f (t − A) > f (t) − (z − 1) > (t − A) + M
A
Mâu thuẫn với tính nhỏ nhất của t. Suy ra
z−1
f (t) 6 t + M với mọi t > 1 (1)
A
z−1
Do z 6 A nên < 1. Áp dụng (1), ta có
A
z−1 z−1 z−1
Å ã
t = f (f (t)) 6 f (t) + M 6 +M +M
A A A

Điều này là vô lí với mọi t. Vậy, giả sử phản chứng sai hay f (z) 6 z, ∀ z ∈ Z+ .
Áp dụng bước 3 và bước 2, ta có
t = f (f (t)) 6 f (t) 6 t
Nên f (t) = t, ∀ t ∈ Z+ .
â

+ Do Math then Love Math 14 h Việt Nguyễn − Mathpiad


 Bài 6 Cho hàm số f : R → R. Chứng minh rằng tồn tại hai số thực x, y mà

f (x − f (y)) > yf (x) + x

A4 IMO Shortlist 2009

Lời giải . Giả sử phản chứng

f (x − f (y)) 6 yf (x) + x, ∀ x, y ∈ R (1)

¨ PHƯƠNG TRÌNH HÀM TRONG IMO SHORTLIST


Đặt a = f (0). Thay y = 0 vào (1), ta được f (x − a) 6 x, ∀ x ∈ R. Như vậy

f (y) 6 y + a, ∀ y ∈ R (2)

Thay x = f (y) vào (1), ta được

a 6 yf (f (y)) + f (y) 6 yf (f (y)) + y + a, ∀ y ∈ R

Suy ra y [f (f (y)) + 1] > 0, ∀ y ∈ R hay f (f (y)) + 1 > 0 với mọi y > 0. (3)
Từ (2), (3), ta có

−1 6 f (f (y)) 6 f (y) + a ⇒ f (y) > −a − 1 với mọi y > 0 (4)


−a − x − 1
ß ™
Giả sử tồn tại x mà f (x) > 0. Chọn y < min x − a, , từ (2), ta được
f (x)
x − f (y) > x − (y + a) > 0

Mặt khác, từ (1) và (4), ta có

yf (x) + x > f (x − f (y)) > −a − 1


−a − x − 1
Suy ra y > , mâu thuẫn với cách chọn y. Như vậy, ta có f (x) 6 0, ∀ x ∈ R.
f (x)
Ta có a = f (0) 6 0 nên (2) tương đương f (x) 6 x, ∀ x ∈ R.
Ta chọn y sao cho y > max{0, −f (−1) − 1} và đặt x = f (y) − 1, ta có

f (−1) = f (x − f (y)) 6 yf (x) + x = yf (f (y) − 1) + f (y) − 1 6 y (f (y) − 1) − 1 6 −y − 1

Điều này là mâu thuẫn với cách chọn y. Vậy giả sử sai. Kết thúc chứng minh. â

 Bài 7 Tìm tất cả các hàm số f : R → R thỏa mãn

f (xf (x + y)) = f (yf (x)) + x2 , ∀ x, y ∈ R

A7 IMO Shortlist 2009

Lời giải . P (0, y), ta được


f (0) = f (yf (0))
Nếu f (0) 6= 0 thì f (x) = C, ∀ x ∈ R, không thỏa. Vậy f (0) = 0.
P (x, 0) và P (x, −x), ta được

f (xf (x)) = x2 và f (−xf (x)) = −x2 , ∀ x ∈ R

Như vậy, ta được f (a) = 0 ⇔ a = 0. Xét t 6= 0 bất kì, ta có

+ Do Math then Love Math 15 h Việt Nguyễn − Mathpiad


Nếu f (t) < 0. Đặt f (t) = −a2 .
Tồn tại số b để bf (a) = t do f (a) 6= 0 nên P (a, b), ta được

f (af (a + b)) = f (bf (a)) + a2 = f (t) + a2 = 0

Suy ra af (a + b) = 0. Suy ra a + b = 0. Hay t = −af (a). Suy ra

f (−t) = f (af (a)) = a2 = −(−a2 ) = −f (t)

Nếu f (t) > 0. Đặt f (t) = a2 .


Chọn b để af (b) = t. P (a, b − a), ta được

f ((b − a)f (a)) + a2 = f (af (b)) = f (t) = a2

Suy ra (b − a)f (a) = 0. Suy ra b = a. Hay t = af (a). Suy ra

f (−t) = f (−af (a)) = −a2 = −f (t)


LATEX VÀ DỊCH THUẬT BỞI VIỆT

Vậy ta đã chứng minh được f là hàm lẻ.


Thực hiện lần lượt P (y, −x − y) và P (−x − y, x), ta được

f (yf (−x)) = f ((−y − x)f (y)) + y 2


f ((−x − y)f (−y)) = f (xf (−x − y)) + (x + y)2

Từ tính lẻ, kết hợp với đề bài, ta được

f (yf (x)) − f (xf (x + y)) = −x2


f (yf (x)) − f ((x + y)f (y)) = −y 2
f ((x + y)f (y)) + f (xf (x + y)) = (x + y)2

yf (x)
Cộng theo vế, ta được 2f (yf (x)) = 2xy. Suy ra f (yf (x)) = yx = , ∀ x, y ∈ R. (*)
f (x)/x
Từ f (xf (x)) = x2 , ta được f (f (1)) = 1. Ta có

f (f (1)f (f (1))) = f (1)2 = f (f (1)) = 1

Nên f (1) = 1 hoặc f (1) = −1. Thay vào (*), ta được f (x) = x với mọi x hoặc f (x) = −x với mọi x.
â

 Bài 8 Tìm tất cả các hàm số f : Q+ → Q+ thỏa mãn

f (f (x)2 y) = x3 f (xy), ∀ x, y ∈ Q+

A5 IMO Shortlist 2010

Lời giải . P (x, 1), ta được


f (f (x)2 ) = x3 f (x), ∀ x ∈ Q+ (1)
Từ (1), ta dễ thấy f đơn ánh. Lại có

f (f (xy)2 ) = y 3 x3 f (xy) = y 3 f (f (x)2 y) = f (f (x)2 f (y)2 )

Suy ra
f (xy)2 = f (x)2 f (y)2 ⇔ f (xy) = f (x)f (y), ∀ x, y ∈ Q+

+ Do Math then Love Math 16 h Việt Nguyễn − Mathpiad


Từ đó, (1), tương đương
»
f (f (x))2 = x3 f (x) ⇔ f (f (x)) = x3 f (x), ∀ x ∈ Q+ (2)

Đặt g(x) = xf (x), (2) tương đương

g 2 (x) = g(g(x)) = g(x)f (g(x)) = xf (x)f (xf (x)) = xf (x)2 f (f (x))


»
xf (x)2 x3 f (x) = (xf (x))5/2 = g(x)5/2

Làm tương tự như vậy, ta được

¨ PHƯƠNG TRÌNH HÀM TRONG IMO SHORTLIST


n
g n+1 (x) = g(x)(5/2) ∈ Q+ (*)
1
Cho n → ∞, (*) chỉ xảy ra khi g(x) = 1 hay f (x) = .
x
1
Thử lại thỏa mãn. Vậy hàm số thỏa mãn đề bài là f (x) = , ∀ x ∈ Q+ . â
x

 Bài 9 Cho các hàm số f, g : Z+ → Z+ thỏa mãn

f (g(n)) = f (n) + 1 và g(f (n)) = g(n) + 1 với mọi n ∈ Z+

Chứng minh rằng f (n) = g(n), ∀ n ∈ Z+

A6 IMO Shortlist 2010

Lời giải .

¨ Bước 1.

Ta có nếu f (x) = f (y) thì g(x) = g(f (x)) − 1 = g(f (y)) − 1 = g(y).
Tương tự với g(x) = g(y).
Suy ra nếu f (f (x)) = f (f (y)) thì g(f (x)) = g(f (y)) ⇒ g(x) = g(y) ⇒ f (x) = f (y).

¨ Bước 2.

Gọi a, b lần lượt là giá trị nhỏ nhất mà f, g nhận và giả sử a 6 b.


Ta có f (a) > a vì nếu f (a) = a thì g(a) = g(f (a)) = g(a) + 1, vô lí.
Nếu tồn tại f (x) = m thì tồn tại f (t) = f (g(x)) = f (x) + 1 = m + 1. Như vậy, với mọi j 6 a thì luôn
tồn tại z để f (z) = j. Tương tự như vậy, ta suy ra tồn tại x để f (a) = f (x) + 1 hay f (a) = f (g(x)).
Ta lại có tồn tại t để f (t) = a và tồn tại y để f (y) = g(x) nên f (f (t)) = f (a) = f (g(x)) = f (f (y)).
Suy ra g(x) = f (y) = f (t) = a > b. Ta lại có b > a nên a = b.

¨ Bước 3.

Ta chứng minh nếu tồn tại f (x) = f (y) với y > x > a thì x = y.
Thật vậy, tồn tại f (h) = x và f (k) = y nên f (f (h)) = f (f (k)). Suy ra f (h) = f (k) = x = y.

¨ Bước 4.

Giả sử f (a) > a + 2 thì tồn tại x, y ∈ Z+ sao cho f (x) = f (a) − 2 và f (y) = g(x).
Như vậy, f (a) = f (x) + 2 = f (g(x)) + 1 = f (g 2 (x)). Lại có g 2 (x) > a nên g 2 (x) = a.
Suy ra a = g(f (y)) = 1 + g(y) > 1 + a, vô lí. Suy ra f (a) 6 a + 1.
Từ bước 2, ta có f (a) > a + 1 nên f (a) = a + 1. Tương tự, g(a) = a + 1. Vậy f (a) = g(a)

¨ Bước 5.

+ Do Math then Love Math 17 h Việt Nguyễn − Mathpiad


Giả sử f (n) = g(n) = n + 1, từ đề bài, ta được

f (n + 1) = f (g(n)) = f (n) + 1 = n + 2 và g(n + 1) = g(f (n)) = g(n) + 1 = n + 2

Như vậy, ta đã chứng minh được f (n) = g(n) với mọi n > a.
Với n bất kì thì g(n) > a nên ta được

f (n) + 1 = f (g(n)) = g(n) + 1

nên f (n) = g(n). Kết thúc chứng minh. â

 Bài 10 Tìm tất cả các hàm số f, g : R → R thỏa mãn

g(f (x + y)) = f (x) + (2x + y)g(y), ∀ x, y ∈ R

A3 IMO Shortlist 2011


LATEX VÀ DỊCH THUẬT BỞI VIỆT

Lời giải . P (x, −2x), ta được


g(f (−x)) = f (x), ∀ x ∈ R
Điều kiện đề bài tương đương

f (x) + (2x + y)g(y) = g(f (x + y)) = f (−x − y), ∀ x, y ∈ R (1)

Thay (x, y) với (−b, a + b) vào (1), ta được

f (−a) = f (−b) + (a − b)g(a + b), ∀ a, b ∈ R

Tương tự, ta có
f (−b) = f (−c) + (b − c)g(b + c)
f (−c) = f (−a) + (c − a)g(c + a)
Ta suy ra
(a − b)g(a + b) + (b − c)g(b + c) + (c − a)g(c + a) = 0, ∀ a, b, c ∈ R
Đặt (x, y, z) = (a + b, b + c, c + a), ta được

(z − y)g(x) + (x − z)g(y) + (y − x)g(z) = 0, ∀ x, y, z ∈ R

Như vậy, ba điểm A (x, g(x)), B (y, g(y)), C (z, g(z)) thằng hàng trên mặt phẳng tọa độ.
Vậy g(x) = ax + b, ∀ x ∈ R. Thay (x, y) bởi (0, −y) vào (1), ta được

f (0) − yg(−y) = f (y) = ay 2 − by + c

So sánh hệ số của x2 trong biểu thức g(f (−x)) = f (x), ta thu được a = a2 nên a = 0 hoặc a = 1.

¨ Trường hợp 1. Nếu a = 0 thì g(f (−x)) = b = f (x). Thử lại ta được b = 0.

¨ Trường hợp 2. Nếu a = 1 thì từ g(f (−x)) = f (x), ta được

x2 − bx + c = f (x) = g(f (−x)) = f (−x) + b = x2 + bx + c + b ⇒ 2bx = b ⇒ b = 0

Vậy f (x) = x2 + c và g(x) = x với mọi x ∈ R.

+ Do Math then Love Math 18 h Việt Nguyễn − Mathpiad


Vậy các hàm số thỏa mãn là (f, g) ∈ {(0, 0); (x2 + c, x)}. â

 Bài 11 Tìm tất cả các hàm số f, g : Z+ → Z+ thỏa mãn

f g(n)+1 (n) + g f (n) (n) = f (n + 1) − g(n + 1) + 1, ∀ n ∈ Z+

A4 IMO Shortlist 2011

Lời giải . Từ đề bài, ta có


Ä ä
f f g(n) (n) = f (n + 1) − g(n + 1) + 1 − g f (n) (n) 6 f (n + 1) − 1 + 1 − 1 < f (n + 1), ∀ n ∈ Z+

¨ PHƯƠNG TRÌNH HÀM TRONG IMO SHORTLIST


Giả sử y1 < y2 < ... là tập các giá trị của f . 
Nếu tồn tại a > 1 để f (a) = y1 thì f f g(a−1)+1 (a − 1) < y1 , mâu thuẫn với tính nhỏ nhất của y1 .

Từ đó suy ra f (x) = y1 ⇔ x = 1. Giả sử tồn tại a > 1 để f (a) = y2 thì f f g(a−1) (a − 1) = y1 nên
f g(a−1) = 1 hay y1 = 1.
Ta chứng minh quy nạp rằng nếu f (x) = yn thì x = yn = n. Giả sử điều này đúng đến n = k. Ta
chứng minh nó cũng đúng với n = k + 1.
Từ giả thiết quy nạp, với a < k + 1 thì nếu f t (x)  = a thì x = a nên f t (h) > a với h > a.
Giả sử tồn tại f (x) = yn+1 thì f f g(x−1) (x − 1) < yn+1 nên f f g(x−1) (x − 1) ∈ {y1 , ..., yn }
Do f (x) = yn+1 nên x > n hay
g(n+1)+1
 x − 1 > n = yn . Vậy x =g(n+1)+1
n + 1.
Vậy ta có f f (n + 1) < f (n + 2) = yn+2 nên f (n + 1) = yn+1 .
Suy ra f g(n+1) (n + 1) = n + 1 nên yn+1 = n + 1.
Vậy ta đã chứng minh được f (n) = n với mọi n nguyên dương.
Điều kiện đề bài tương đương

n + g n (n) = n + 1 − g(n + 1) + 1 ⇔ g n (n) + g(n + 1) = 2

Suy ra g(n + 1) = g n (n) = 1, ∀ n ∈ Z+ hay g(n) = 1, ∀ n ∈ Z+ .


Thử lại thỏa mãn. Vậy f (n) = n và g(n) = 1 với mọi n nguyên dương. â

 Bài 12 Tìm tất cả các hàm số f : R → R thỏa mãn

f (x + y) 6 yf (x) + f (f (x)), ∀ x, y ∈ R (1)

Chứng minh rằng f (x) = 0 với mọi x 6 0

A6 IMO Shortlist 2011

Lời giải . P (x, t − x), ta được

f (t) 6 tf (x) − xf (x) + f (f (x)), ∀ x, t ∈ R (2)

Thay (t, x) bởi (f (a), b) và (f (b), a), ta được

f (f (a)) 6 f (a)f (b) − bf (b) + f (f (b))

f (f (b)) 6 f (a)f (b) − af (a) + f (f (a))


Cộng theo vế cả hai bất đẳng thức trên, ta được

f (a)f (b) > af (a) + bf (b)

Cho b = 2f (a), ta thu được af (a) 6 0 nên f (a) > 0 với mọi a < 0 (*)
Giả sử tồn tại x để f (x) > 0.

+ Do Math then Love Math 19 h Việt Nguyễn − Mathpiad


xf (x) − f (f (x))
Từ (2), ta có nếu t < thì f (t) < 0, mâu thuẫn với (*)
f (x)
Như vậy, ta được f (x) 6 0, ∀ x ∈ R. Kết hợp với (*), ta được f (x) = 0 với mọi x < 0.
Chọn x < 0, y = 0 vào đề bài, ta được

0 = f (x) 6 f (f (x)) = f (0)

Mà f (0) 6 0 nên f (0) = 0. Kết thúc chứng minh. â

 Bài 13 Tìm f : Z → Z sao cho với mọi a, b, c ∈ Z thỏa mãn a + b + c = 0 thì

f 2 (a) + f 2 (b) + f 2 (c) = 2f (a)f (b) + 2f (b)f (c) + 2f (c)f (a) (*)

A1 IMO Shortlist 2012

Lời giải . Kí hiệu P (x, y, z) là thay lần lượt a, b, c bởi x, y, z vào phương trình (∗).
P (0, 0, 0) ⇒ 3f (0)2 = 6f (0)2 ⇒ f (0) = 0
LATEX VÀ DỊCH THUẬT BỞI VIỆT

P (a, 0, −a) ⇒ (f (a) − f (−a))2 = 0 ⇒ f (a) = f (−a)


P (a, b, −a − b) ⇒ f 2 (a) + f 2 (b) + f 2 (a + b) = 2f (a)f (b) + 2(f (b) + f (a))f (a + b), ∀ a, b ∈ Z (1)
Ta có các trường hợp sau :

¨ Trường hợp 1. Nếu không tồn tại x sao cho f (x) = 0 hay f (a) = 0 ⇔ a = 0

Trong phương trình (1), ta thay b bởi −b, ta có

f 2 (a + b) − f 2 (a − b) = 2(f (a) + f (b))(f (a + b) − f (a − b)) (2)

Nếu tồn tại f (x) = f (x + c) và x, c, 2x + c 6= 0 ta có

f 2 (x) + f 2 (c) + f 2 (x + c) = 2f 2 (x) + f 2 (c)


= 2f (x)f (c) + 2(f (x) + f (c))f (x + c) = 2f (x)f (c) + 2(f (x) + f (c))f (x)

⇒ f 2 (c) = 4f (x)f (c) ⇒ f (c) = 4f (x)


Mặt khác

f 2 (x) + f 2 (x + c) + f 2 (2x + c) = 2f 2 (x) + f 2 (2x + c)


= 2f (x)f (x + c) + 2(f (x) + f (x + c))f (2x + c) = 2f 2 (x) + 4f (x)f (2x + c)

⇒ f (2x + c) = 4f (x) = f (c) ⇒ f (2x) = 4f (c)


Trong phương trình (1), thay b bởi a, ta được f (2a) = 4f (a)
Từ đó f (x) = f (c) = 4f (c), vô lý. Từ đó suy ra f (a + b) 6= f (a − b)
Từ đó, ta có f (a + b) + f (a − b) = 2f (a) + 2f (b), ∀ a, b ∈ Z.
Dễ dàng quy nạp f (x) = dx2 . Thử lại ta thấy hàm số này thỏa mãn.

¨ Trường hợp 2. Nếu tồn tại x 6= 0, f (x) = 0 ⇒ f (b) = f (x + b), ∀ b ∈ Z

Từ phương trình (1), ta có f 2 (2b) = 4f (b)f (2b), ta lại có 3 khả năng sau :

Khả năng 1. Nếu f (2) = 0 thì f (a) = f (a + 2).


Vậy f (a) = 0 nếu a chẵn và f (a) = e nếu a lẻ.

+ Do Math then Love Math 20 h Việt Nguyễn − Mathpiad


f (2)
Khả năng 2. Nếu f (2) 6= 0, f (4) 6= 0 thì f (1) = 6= 0 và f (4) = 4f (2) = 16f (1) = 16t
4
Thay a = 1 và b = 2 vào phương trình (1), ta có

17t2 + f (3)2 = 8t2 + 10t.f (3)

Từ đó suy ra f (3) = 9t hoặc f (3) = t


Nếu f (3) = 9t thì thay a = 1 và b = 3 vào phương trình (1), ta có

82t2 + 162 t2 = 18t2 + 2.10t.16t, vô lý do t 6= 0

¨ PHƯƠNG TRÌNH HÀM TRONG IMO SHORTLIST


Vậy f (3) = t. Thay a = 1 và b = 2 vào phương trình (1), ta có

2t2 + 16t2 = 2t2 + 4t.16t, vô lý do t 6= 0

Vậy trường hợp này không thỏa mãn.

Khả năng 3. Nếu f (2) 6= 0 và f (4) = 0 thì f (x + 4) = f (x), ∀ x ∈ Z


Ta vẫn có f (3) = 9t hoặc f (3) = t.
Nếu f (3) = 9t thì thay a = 1 và b = 3 vào phương trình (1), ta có

t2 + 81t2 = 2.9t.t, vô lý do t 6= 0

Vậy f (3) = f (1) = t. Từ đó hàm f được xác định như sau thỏa mãn yêu cầu đề bài:

f (4k) = 0; f (4k + 1) = f (4k + 3) = t; f (4k + 2) = 4t, ∀ k, t ∈ Z

Vậy tất cả các hàm f thỏa mãn là


1 f (x) = dx2 , ∀ x, d ∈ Z

2 f (2k) = 0; f (2k + 1) = c, ∀ k, c ∈ Z

3 f (4k) = 0; f (4k + 1) = f (4k + 3) = t; f (4k + 2) = 4t, ∀ k, t ∈ Z

 Bài 14 Tìm tất cả các hàm số f : R → R thỏa mãn f (−1) 6= 0 và

f (1 + xy) − f (x + y) = f (x)f (y), ∀ x, y ∈ R

A5 IMO Shortlist 2012

Lời giải . P (x, −1), ta được

f (1 − x) = f (x − 1) + f (x)f (−1) ⇒ f (−x) = f (x) + f (x + 1)f (−1), ∀ x ∈ R

Thế x bởi −x, ta được f (x) = f (−x) + f (1 − x)f (−1) nên

f (x + 1)f (−1) + f (1 − x)f (−1) = 0 ⇒ f (x + 1) + f (x − 1) = 0 ⇒ f (x) + f (2 − x) = 0, ∀ x ∈ R (1)

P (x, 0) ⇒ f (1) = f (x) + f (x)f (0), ∀ x ∈ R. Nếu f (0) 6= −1 thì f (x) ≡ C.


Thế lại vào đề bài, ta được C = C + C 2 ⇒ C = 0, vô lí do f (−1) 6= 0. Vậy f (0) = −1 Ta chọn x, y
sao cho x + y = 1 và 1 + xy = t < 1. P (2 − x, 2 − y), ta được

f (xy + 3) = f (3) + f (2 − x)f (2 − y) = f (3) + f (x)f (y) ⇒ f (xy + 3) − f (xy + 1) = f (3)

+ Do Math then Love Math 21 h Việt Nguyễn − Mathpiad


Vậy ta có f (t + 2) − f (t) = f (3) với mọi t < 1.
Kết hợp với (1), ta được f (t + 2) − f (t) = f (3) = C, ∀ x ∈ R.
Ta có f (2) = f (0) + C = C − 1. f (4) = f (2) + C = 2C − 1. P (2, 2), ta có

f (5) = f (4) + f (2)2 = (2C − 1) + (C − 1)2 = f (3) + C = 2C ⇒ C 2 = 2C

Lại có C 6= 0 vì nếu C = 0 thì f (x + 2) = f (x), ∀ x ∈ R nên f (−1) = f (1) 6= 0, vô lí.


Vậy C = 2 hay f (x + 2) − f (x) = 2, ∀ x ∈ R. Kết hợp với (1), ta được

0 = f (x) + f (2 − x) = 2 + f (−x) + f (x) ⇒ f (−x) = −f (x) − 2, ∀ x ∈ R

P (x, y) và P (−x, −y), ta có

f (1 + xy) = f (−x − y) + f (−x)f (−y) = −f (x + y) − 2 + (f (x) − 2) (f (y) − 2)


= f (x + y) + f (x)f (y)

⇒ f (x + y) = f (x) + f (y) + 1, ∀ x, y ∈ R (1)


LATEX VÀ DỊCH THUẬT BỞI VIỆT

Suy ra đề bài tương đương

f (1) + f (xy) + 1 = f (1 + xy) = [f (x) + 1] [f (y) + 1] , ∀ x, y ∈ R (2)

Đặt g(x) = f (x) + 1, ∀ x ∈ R, ta được


®
g(xy) = g(x)g(y)
, ∀ x, y ∈ R
g(x + y) = g(x) + g(y)

Ta suy ra g(x) = x, ∀ x ∈ R. Suy ra f (x) = x − 1, ∀ x ∈ R. â

 Bài 15 Cho hàm số f : Z+ → Z+ . Giả sử rằng với mỗi số n ∈ Z+ , tồn tại một số k ∈ Z+
để f 2k (n) = n + k và đặt kn là số k nhỏ nhất có tính chất này. Chứng minh dãy k1 , k2 , ... không
bị chặn.

A6 IMO Shortlist 2012

Lời giải . Đặt S = {1, f (1), f 2 (1), ..., }. Khi đó S không bị chặn do với n ∈ S thì tồn tại số k > 0
sao cho f 2k (n) = n + k > n.
Như vậy, ta suy ra các phần tử trong f là khác nhau. Thật vậy, nếu tồn tại f i (1) = f j (1) với i 6= j
thì các giá trị f m (1) sẽ tuần hoàn, vô lí.
Đặt g : S → S với g(n) = f 2kn (n) = n + kn . Khi đó g đơn ánh. Thật vậy, giả sử tồn tại a < b mà
g(a) = g(b). Khi đó a + ka = f 2ka (a) = f 2kb (b) = b + kb nên ka > kb .
Do f đơn ánh nên ta có

f 2ka −2kb (a) = b = a + (ka − kb ), mâu thuẫn với tính nhỏ nhất của ka

Ta kí hiệu T là tập các phần tử thuộc S nhưng không có dạng g(n) với n ∈ S.
Ta có 1 ∈ T do g(n) > 1, ∀ n ∈ Z+ nên T 6= Ø.
Với mỗi t ∈ T , đặt Ct = {t, g(t), ...} và gọi Ct là chuỗi bắt đầu từ t. Ta thấy rằng các chuỗi khác
nhau thì không có phần tử chung do g đơn ánh.
Với mỗi n ∈ S \ T , n = g(n0 ) với n0 < n và n0 ∈ S. Lặp lại tương tự như vậy, ta suy ra n ∈ Ct với
t ∈ T nào đó.
Giả sử f n (1) nằm trong chuỗi bắt đầu từ t = f nt (1) nào đó, ta được

f n (1) = g j (f nt (1)) = f 2kj f 2kj−1 ...f 2k1 (f nt (1)) = f nt (1) + k1 + k2 + ... + kj




+ Do Math then Love Math 22 h Việt Nguyễn − Mathpiad


Mà n = nt + 2k1 + 2k2 + ... + 2kj nên
n − nt
f n (1) = f nt (1) + (1)
2
Bây giờ, ta chứng minh T là tập vô hạn. Giả sử phản chứng. Giả sử chỉ có hữu hạn chuỗi là
Ct1 , Ct2 , ..., Ctr với t1 < ... < tr .
Cố định N . Nếu f n (1) với 1 6 n 6 N nằm trong chuỗi Ct thì
n − nt N
f n (1) = t + 6 tr +
2 2

¨ PHƯƠNG TRÌNH HÀM TRONG IMO SHORTLIST


Mặt khác, 1, f (1), ..., f N (1) là các số khác nhau nên ta phải có
N N
N + 1 6 tr + ⇒ tr > +1
2 2
Điều này là vô lí do N có thể lớn tùy ý. Vậy tập T là tập vô hạn.
Trở lại bài toán. Chọn số nguyên dương k bất kì và xét k + 1 chuỗi bắt đầu từ k + 1 số đầu tiên
của tập T . Gọi t là số lớn nhất trong k + 1 số đó.
Khi đó mỗi chuỗi đều chứa một số không vượt quá t và tồn tại một chuỗi không chứa tất cả các số
nằm giữa (t + 1) và (t + k). Nói cách khác, tồn tại một chuỗi mà chứa số a 6 t và g(a) > t + k + 1.
Suy ra g(a) − a > k hay ka > k.
Vậy ta đã chứng minh được dãy (kn ) không bị chặn. â

 Bài 16 Tìm tất cả các hàm số f : Q>0 → R thỏa mãn


®
f (x)f (y) > f (xy) (1)
f (x + y) > f (x) + f (y) (2)

Với mọi số hữu tỉ dương x, y. Chứng minh rằng f (x) = x, ∀ x ∈ Q>0 , biết tồn tại một số hữu
tỉ dương a > 1 mà f (a) = a

A3 IMO Shortlist 2013

Lời giải . Thay x = 1, y = a vào (1), ta được af (1) > a ⇒ f (1) > 1. Từ (2), ta dễ thấy

f (nx) > nf (x) với mọi n ∈ Z+ và x ∈ Q>0 (3)

Ta suy ra
f (n) > nf (1) > n, ∀ n ∈ Z+ (4)
m
Từ (1), ta có f f (n) > f (m) nên f (q) > 0, ∀ q ∈ Q>0 .
n
Như vậy, từ (2), ta được hàm f tăng ngặt. Kết hợp với (4), ta được

f (x) > f ([x]) > [x] > x − 1 với mọi x > 1

Từ (1), ta được f (x)n > f (xn ) nên



f (x)n > f (xn ) > xn − 1 ⇒ f (x) > n
xn − 1 với mọi x > 1 và n ∈ Z+

Cho n → ∞, ta được f (x) > x với mọi x > 1. (5)


Từ (1) và (5), ta được an = f (a)n > f (an ) > an nên f (an ) = an , ∀ n ∈ Z+ .
Với x > 1, tồn tại n sao cho an − x > 1. Từ (2) và (5), ta được

an = f (an ) > f (x) + f (an − x) > x + (an − x) = an

+ Do Math then Love Math 23 h Việt Nguyễn − Mathpiad


Vậy f (x) = x với mọi x > 1. Khi đó, với y > 1, (2) tương đương

x + y > f (x) + y ⇒ x > f (x)

Kết hợp với (5), ta suy ra f (x) = x với mọi x ∈ Q>0 . â

 Bài 17 Tìm tất cả các hàm số f : N → N thỏa mãn

f (f (f (n))) = f (n + 1) + 1, ∀ n ∈ N

A5 IMO Shortlist 2013

Lời giải . Ta có

f 4 (n) = f (f 3 (n)) = f (f (n + 1) + 1) và f 4 (n + 1) = f 3 (f (n + 1)) = f (f (n + 1) + 1) + 1

Nên
LATEX VÀ DỊCH THUẬT BỞI VIỆT

f 4 (n) + 1 = f 4 (n + 1) (*)

¨ Bước 1. Ta chứng minh f là hàm đơn ánh.

Ta kí hiệu R(i) là tập giá trị của hàm f i . Khi đó R(0) = N và R(0) ⊇ R(1) ⊇ R(2) ⊇ ...
Từ (*), ta thấy rằng nếu a ∈ R(4) thì a + 1 ∈ R(4). Suy ra tập N \ R(4) là tập hữu hạn.
Do R(4) ⊇ R(1) nên tập N \ R(1) là hữu hạn. Hay R(1) không bị chặn trên. (1)
Giả sử tồn tại f (m) = f (n) với m 6= n. Từ đề bài ta có f (m + 1) = f (n + 1).
Tương tự như vậy, ta có f (m + c) = f (n + c) với mọi c ∈ N nên f tuần hoàn theo công sai |m − n|,
điều này mâu thuẫn với (1). Vậy f là hàm đơn ánh.

¨ Bước 2. Tồn tại một số a sao cho f (a) = a + 1

Đặt S(i) = R(i − 1) \ R(i). Từ f đơn ánh, ta có n ∈ S(i) ⇐⇒ f (n) ∈ S(i + 1).
Như vậy, ta được |S(1)| = |S(2)| = ... = k
Nếu 0 ∈ R(3) thì tồn tại n để f 3 (n) = 0 ⇒ f (n + 1) = −1, vô lí.
Suy ra 0 = R(0) \ R(3) = S(1) ∪ S(2) ∪ S(3) nên k > 0.
Ta chứng minh mỗi phần tử b ∈ S(1) ∪ S(2) ∪ S(3) phải thỏa mãn một trong ba điều kiện sau:

1 b=0

2 b = f (0) + 1

3 b − 1 ∈ S(1)

Thật vậy, nếu b − 1 ∈/ S(1) thì tồn tại n sao cho f (n) = b − 1.
3
Suy ra f (n − 1) = f (n) + 1 = b nên b ∈ R(3).
Vì vậy b ∈/ S(1) ∪ S(2) ∪ S(3). Suy ra các số lớn hơn b đều không thuộc S(1). Suy ra

3k = |S(1) ∩ S(2) ∩ S(3)| 6 1 + 1 + S(1) = k + 2 ⇒ k 6 1 ⇒ k = 1

Như vậy, ta có S(1) = {a}, S(2) = {f (a)}, S(3) = {f 2 (a)} với số nguyên dương a nào đó.
Từ 3 điều kiện như trên mà mỗi hàm chỉ được chọn 1. Vậy, ta được

{a, f (a), f 2 (a)} = {0, a + 1, f (0) + 1} (2)

Do a + 1 6= a nên a + 1 ∈ {f (a), f 2 (a)}.


Nếu a + 1 = f 2 (a) thì f (a + 1) = f 3 (a) = f (a + 1) + 1, vô lí. Vậy f (a) = a + 1

+ Do Math then Love Math 24 h Việt Nguyễn − Mathpiad


¨ Bước 3. Tìm tất cả các hàm thỏa mãn.

Cũng từ (2), ta được 0 ∈ {a, f 2 (a)}.

Trường hợp 1. a = 0. Vậy, ta được f (0) = f (a) = a + 1 = 1.


Cũng từ (2), f (1) = f 2 (a) = f (0) + 1 = 2. Ta chứng minh quy nạp f (n) = n + 1. Thật vậy

n + 1 = f (n − 1) + 1 = f 3 (n − 2) = f 2 (n − 1) = f (n)

Thử lại thỏa mãn.

¨ PHƯƠNG TRÌNH HÀM TRONG IMO SHORTLIST


Trường hợp 2. f 2 (a) = 0. Từ (2), ta có a = f (0) + 1.
Do f (a) = a + 1 nên f (a + 1) = f 2 (a) = 0 suy ra f (0) = f 3 (a) = f (a + 1) + 1 = 1.
Vậy a = f (0) + 1 = 2 và f (2) = 3. Ta đã tìm được f (0) = 1, f (2) = 3, f (3) = 0.
Ta sẽ chứng minh quy nạp kết quả sau

 n + 1 với 2 | n

f (n) = n + 5 với n ≡ 1 (mod 4) , ∀ n ∈ N (**)

n − 3 vói n ≡ 3 (mod 4)

Giả sử (∗∗) đúng với n = 4k + 2, 4k + 3, 4k với k 6 m và n = 4k + 1 với k < m. Ta chứng minh


điều này đúng với m + 1.
Thật vậy, từ đề bài, ta có f 3 (4m + 1) = f (4m + 2) + 1 = 4m + 3 + 1 = 4m + 4 nên áp dụng (*),
ta có
f (4m + 4) = f 4 (4m + 1) = f 4 (4m) + 1 = f 3 (4m + 1) + 1 = 4m + 5
Từ đó, theo đề bài, ta lần lượt có

f (4m + 1) = f (f (4m)) = f 3 (4m + 3) = f (4m + 4) + 1 = 4m + 5 + 1 = 4m + 6


f (4m + 6) = f (f (4m + 1)) = f 3 (4m) = f (4m + 1) + 1 = 4m + 7
f (4m + 7) = f (f (4m + 6)) = f 3 (4m + 1) = f (4m + 2) + 1 = 4m + 3 + 1 = 4m + 4

Vậy các hàm số thỏa mãn đề bài là

1 f (n) = n + 1, ∀ n ∈ N

 n + 1 với 2 | n

2 f (n) = n + 5 với n ≡ 1 (mod 4) ,∀n ∈ N

n − 3 vói n ≡ 3 (mod 4)

 Bài 18 Tìm tất cả các hàm số f : Z → Z thỏa mãn

f (f (m) + n) + f (m) = f (n) + f (3m) + 2014, ∀ m, n ∈ Z (1)

A4 IMO Shortlist 2014

Lời giải . Đặt C = 1007 và kí hiệu g(m) = f (3m) − f (m) + 2C, ∀ m ∈ Z. Khi đó g(0) = 2C.
Phương trình (1) có thể viết lại thành

f (f (m) + n) = g(m) + f (n), ∀ m, n ∈ Z ⇒ f (tf (m) + n) = tg(m) + f (n), ∀ m, n, t ∈ Z (2)

+ Do Math then Love Math 25 h Việt Nguyễn − Mathpiad


Kí hiệu P (a, b, c) là thay (m, n, t) bởi (a, b, c) vào (2).
Với r là số nguyên bất kì, P (r, 0, f (0)) và P (0, 0, f (r)), ta được
f (0)g(r) = f (f (r)f (0)) − f (0) = f (r)g(0)
Nếu f (0) = 0 thì do g(0) 6= 0 nên f (r) = 0, không thỏa mãn.
Ta xét f (0) 6= 0. Khi đó g(r) = αf (r). Như vậy, ta được f (3r) = (1 + α)f (r) − 2C. Suy ra
2C
f (3m) − β = (1 + α) (f (m) − β) , ∀ m ∈ Z với β =
α
Thực hiện k lần, ta được
f (3k m) − β = (1 + a)k (f (m) − β) , ∀ m, k > 0 ∈ Z (3)
Vì 3 - C nên từ đề bài, tồn tại một giá trị a sao cho f (a) = d không chia hết cho 3.
Từ (2), ta có
f (n + td) = f (n) + tg(a) = f (n) + tαf (a) = f (n) + α · td, ∀ n, t ∈ Z (4)

Chọn k = 3ϕ(|d|) , ta có d | 3k − 1. Áp dụng (4), ta có


LATEX VÀ DỊCH THUẬT BỞI VIỆT

f (3k m) = f (m) + α.(3k − 1)m (5)


Như vậy, từ (3) và (5), ta suy ra

k k α 3k − 1
(1 + a) (f (m) − β) + β = f (m) + α.(3 − 1)m ⇒ f (m) = ·m+β
(1 + α)k − 1
Vậy f (x) = Ax + B, ∀ x ∈ Z. Thử lại vào đề bài, ta có
(A2 − 2A)m + AB − 2014 = 0, ∀ m ∈ Z
Điều này chỉ xảy ra khi A2 = 2A và AB = 2014 hay (A, B) = (2, 1007).
Vậy hàm số thỏa mãn yêu cầu đề bài là f (x) = 2x + 1007, ∀ x ∈ Z â

 Bài 19 Tìm tất cả các hàm số f : Z → Z thỏa mãn

n2 + 4f (n) = f (f (n))2 , ∀ n ∈ Z (*)

A6 IMO Shortlist 2014

Lời giải .
¨ Bước 1. Ta chứng minh f (n) = n + 1, ∀ n ∈ Z+
Xét dãy (ak ) được xác định bởi ak = f k (1) với mọi k > 0. Từ (∗), suy ra
a2k + 4ak+1 = a2k+2 , ∀ k ∈ Z
Ta có a0 = 1 nên a22 = 1 + 4a1 là số lẻ. Suy ra a2 = 2r + 1 với r ∈ Z. Khi đó a1 = r2 + r và
a23 = a21 + 4a2 = (r2 + r)2 + 8r + 4
Lúc này a23 là một số chính phương chẵn mà 8r + 4 6= 0 nên
|8r + 4| = |a23 − (r2 + r)2 | > (r2 + r)2 − (r2 + r − 2)2 = 4(r2 + r − 1) (1)
Nếu |r| > 4 thì
4r2 > 16|r| > 12|r| + 16 > 8|r| + 4 + 4|r| + 4 > |8r + 4| − |4r| + 4
Mâu thuẫn với (1). Như vậy ta có |r| 6 3.
Kiểm tra trực tiếp, để (r2 + r)2 + 8r + 4 là số chính phương thì r ∈ {−3, 0, 1}.

+ Do Math then Love Math 26 h Việt Nguyễn − Mathpiad


Trường hợp 1. r = −3. Khi đó a1 = 6 và a2 = −5. Với mọi k > 1, ta có
»
ak+2 = ± a2k + 4ak+1 ∈ Z

Nên a2k + 4ak+1 là số chính phương với mọi k > 1.


Điều này dẫn đến a3 = −4, a4 = −3, a5 = −2, a6 = −1, a7 = 0, a8 = 1, a9 = 2.
Điều này là mâu thuẫn do f (1) = a1 = 6 và f (1) = f (a8 ) = a9 = 2.

Trường hợp 2. r = 0. Khi đó a1 = 0 và a2 = 1. Khi đó a23 = a21 + 4a2 = 4.


Điều này là mâu thuẫn do f (1) = a1 = 0 = f (a2 ) = a3 .

¨ PHƯƠNG TRÌNH HÀM TRONG IMO SHORTLIST


Trường hợp 3. r = 1. Khi đó f (1) = 2 và f (2) = 3.
Ta chứng minh f (n) = n + 1, ∀ n ∈ Z+ bằng quy nạp.
Giả sử điều này đúng đến k. Ta có
» »
2
ak+1 = ± ak−1 + 4ak = ± k 2 + 4(k + 1) = ±(k + 2)

Nếu ak+1 = −(k + 2) thì

a2k+2 = a2k + 4ak+1 = (k + 1)2 − 4(k + 2) = (k − 1)2 − 8

(k − 1)2 − 8 là số chính phương khi k = 4. Vậy a6 = ±1.


Ta có a5 = −6 và a6 = ±1 nên
a27 = a25 + 4a6 = 36 ± 4
không phải là số chính phương, vô lí. Vậy ak+1 = k + 2.

¨ Bước 2. Ta chứng minh f (0) = 1 hoặc; f (0) = 0 và f (n) 6= 0 với mọi n 6= 0

Thay n bởi 0 vào (∗), ta được


4f (0) = f (f (0))2
Suy ra f (0) > 0. Nếu f (0) = a > 0 thì từ bước 1, ta có

4a = f (a)2 = (a + 1)2 ⇒ a = 1

Vậy f (0) = 0 hoặc f (0) = 1.

¨ Bước 3. Ta chứng minh f (−n) = n + 1 hoặc f (−n) = −n + 1.

Xét n > 1. Nếu f (−n) > 0 thì ta có

n2 + 4f (−n) = f (f (−n))2 = (f (−n) + 1)2 ⇒ n2 = (f (−n) − 1)2 ⇒ f (−n) = n + 1

Xét n > 2. Nếu f (−n) = −n + 1 thì ta có

n2 + 4(−n + 1) = f (−n + 1)2 = (n − 2)2

Nếu f (−n + 1) > 0 thì f (−n + 1) = n − 1 + 1 = n. Suy ra n2 = (n − 2)2 , vô lí.


Suy ra f (−n + 1) 6 0. Suy ra f (−n + 1) = −n + 2.
Thực hiện nhiều lần như vậy, ta được f (−1) = 0 và f (0) = 1.
Vậy nếu f (0) = 0 thì f (−n) = n + 1, ∀ n ∈ Z+ .

+ Do Math then Love Math 27 h Việt Nguyễn − Mathpiad


Xét f (0) = 1. Khi đó f (a) = a + 1 với mọi số nguyên a không âm.
Xét n > 2. Giả sử f (n) < 0. Ta có

f k (−n)2 + 4f k+1 (−n) = f k+2 (−n)2 , ∀ k ∈ N

Gọi k là số tự nhiên nhỏ nhất mà f k+1 (−n) > 0. Khi đó


2
f k (−n)2 = f k+1 (−n) − 1 ⇒ f k+1 (−n) = f k (−n) + 1

Tương tự như vậy, ta cũng được


2 2
f k−1 (−n)2 + 4f k (−n) = f k+1 (−n)2 = f k (−n) + 1 ⇒ f k−1 (−n)2 = f k (−n) − 1

Suy ra f k (−n) − 1 = −f k−1 (−n) ⇒ f k (−n) = 1 + f k−1 (−n) + 1.


Làm tương tự như vậy, cuối cùng, ta được f (−n) = −n + 1.

Kết luận. Vậy có ba hàm số thỏa mãn yêu cầu đề bài là


LATEX VÀ DỊCH THUẬT BỞI VIỆT

1 f (n) = n + 1, ∀ n ∈ Z
®
f (n) = n + 1 với n > −a
2 với a là hằng số lớn hơn −1.
f (n) = −n + 1 vói n 6 −a

 f (n) = n + 1 với n > 0

3 f (0) = 0

f (n) = −n + 1 với n < 0

 Bài 20 Tìm tất cả các hàm số f : R → R thỏa mãn

f (x + f (x + y)) + f (xy) = x + f (x + y) + yf (x)

A4 IMO Shortlist 2015

Lời giải . P (x, 1), ta được f (x + f (x + 1)) = x + f (x + 1), ∀ x ∈ R (1)

¨ Trường hợp 1. f (0) 6= 0. P (0, y), ta được

f (f (y)) + f (0) = f (y) + yf (0), ∀ y ∈ R

Thay y bởi x + f (x + 1), ta được

f (0) = [x + f (x + 1)] f (0) ⇔ f (x + 1) + x = 1

Vậy ta suy ra f (x) = 2 − x, ∀ x ∈ R

¨ Trường hợp 2. f (0) = 0. P (x + 1, 0), ta được

f (x + 1 + f (x + 1)) = x + 1 + f (x + 1) (2)

P (1, y), ta được


f (1 + f (1 + y)) + f (y) = 1 + f (y + 1) + yf (1) (3)

+ Do Math then Love Math 28 h Việt Nguyễn − Mathpiad


Thay x bởi −1 vào (1), ta được f (−1) = −1. Thay y bởi −1 vào (3), ta được
f (1) − 1 = 1 − f (1) ⇔ f (1) = 1
Từ (3), ta thấy rằng nếu f (y) = y và f (y + 1) = y + 1 thì f (y + 2) = y + 2. Nên từ (1) và (2),
ta suy ra
f (x + f (x + 1) + 2) = x + f (x + 1) + 2, ∀ x ∈ R (4)
Thay x bởi x − 2 vào (4), ta được
f (x + f (x − 1)) = x + f (x − 1)

¨ PHƯƠNG TRÌNH HÀM TRONG IMO SHORTLIST


P (x, −1), ta được
f (x + f (x − 1)) + f (−x) = x + f (x − 1) − f (x) ⇔ f (−x) = −f (x), ∀ x ∈ R
P (−1, −y), ta được
−f (1 + f (y + 1)) + f (y) = −1 − f (y + 1) + y (5)
Từ (3) và (5), ta suy ra f (y) = y, ∀ y ∈ R.
Vậy các hàm số thỏa mãn là f (x) = x và f (x) = 2 − x â

 Bài 21 Kí hiệu 2Z + 1 là tập các số nguyên lẻ. Tìm tất cả các hàm số f : Z → 2Z + 1 thỏa
mãn
f (x + f (x) + y) + f (x − f (x) + y) = f (x + y) + f (x − y), ∀ x, y ∈ Z (1)

A5 IMO Shortlist 2015

Lời giải . Với hàm số g và số nguyên t 6= 0 bất kì, kí hiệu


∆t g(x) = g(x + t) − g(x)
Với hai số nguyên a, b 6= 0 bất kì, ta có rằng
∆a ∆b g(x) = ∆b ∆a g(x).
Nếu ∆a g = 0 và ∆b g = 0 thì ∆a+b g = 0 và ∆at g = 0 với mọi số nguyên t 6= 0
Ta gọi g là t-quasi-periodic nếu ∆t g là hàm hằng ( Nói các khác là khi ∆1 ∆t g = 0 và gọi ∆t g
là t-periodic). Lúc này, ta gọi t là một quasi-period của g. Ta gọi g là một quasi-periodic nếu là
t-quasi-periodic của một vài số nguyên không âm t.
Chú ý rằng một quasi-period của g là một chu kì của ∆1 g. Nên nếu g là quasi-periodic thì số t là
quasi-period nhỏ nhất là ước của mọi quasi-period khác.
Đặt a = x + y, điều kiện bài toán tương đương
∆f (x) f (a) = ∆f (x) f (2x − a − f (x)), ∀ x, a ∈ Z (2)
Xét b, k là hai số nguyên dương bất kì với k > 0.
Áp dụng (2) khi a lần lượt là b, b + f (x), ..., b + (k − 1)f (x) sau đó cộng theo vế, ta được
∆kf (x) f (b) = ∆kf (x) f (2x − b − kf (x))
Chứng minh tương tự với k < 0. Vậy ta có
∆M f (b) = ∆M f (2x − b − M ) với mọi số nguyên M 6= 0 mà f (x) | M (3)

¤ Bổ đề 21.1. Với hai số nguyên khác nhau x, y bất kì thì hàm số ∆lcm(f (x),f (y)) f là một
2(y − x)-periodic

+ Do Math then Love Math 29 h Việt Nguyễn − Mathpiad


Chứng minh. Đặt L = lcm (f (x), f (y)). Áp dụng (3) hai lần, ta có

∆L f (b) = ∆L f (2x − b − L) = ∆L f (2y − (b + 2(y − x)) − L) = Cf (b + 2(y − x))

Vậy hàm số ∆L f là một 2(y − x)-periodic.

¤ Bổ đề 21.2. Cho g là một hàm số. Nếu s, t 6= 0 mà ∆ts g = 0 và ∆t ∆t g = 0 thì ∆t g = 0

Chứng minh. Không mất tính tổng quát, giả sử s > 0. Xét a là số nguyên bất kì.
Vì ∆t ∆t g = 0, ta có
∆t g(a) = ∆t g(a + t) = ... = ∆t g(a + (s − 1)t)
Tổng của s số trên là ∆ts g = 0 nên mỗi số trong tổng này đều bằng 0.
Trở lại bài toán. Ta làm theo 3 bước
¨ Bước 1. f là một quasi-periodic.
Đặt Q = lcm (f (0), f (1)). Áp dụng bổ (21.1), ta suy ra ∆Q f là một 2-periodic.
LATEX VÀ DỊCH THUẬT BỞI VIỆT

Nói cách khác, giá trị của g là giống nhau với các số chẵn và giống nhau với các số lẻ.
Thay M = Q và x = b = 0 vào (3), ta suy ra g(0) = g(−Q). Vì 0 và −Q khác tính chẵn lẻ nên
giá trị của g ở số chẵn bằng giá trị của g ở số lẻ. Vậy g là hằng số hay Q là một quasi-period
của f .

¨ Bước 2. Gọi quasi-period dương nhỏ nhất của f là T . Ta chứng minh T | f (x), ∀ x ∈ Z.
Vì số lẻ Q là một quasi-period của f nên số T cũng phải là số lẻ.
Giả sử tồn tại một số nguyên tố lẻ p, một số nguyên dương α bất kì và một số nguyên u sao
cho pα | T và pα - f (u). Cho (x, y) = (u, 0) vào (1), ta được

2f (u) = f (u + f (u)) + f (u − f (u))

Suy ra pα không là ước của f tại một trong hai điểm u + f (u) và u − f (u). Ta gọi điểm đó là v.
Đặt L = lcm (f (u), f (v)). Vì |u − v| = f (u) nên theo bổ đề (21.1), ta có ∆2f (u) ∆L f = 0.
Vì hàm số ∆L f là một 2f (u)-periodic và là một T -periodic nên ∆L f là một gcd(T, 2f (u))-
periodic hay ∆gcd(T,2f (u)) ∆L f = 0. Tương tự, vì hàm số ∆gcd(T,2f (u)) là một L-periodic và
T -periodic nên ta suy ra ∆gcd(T,L) ∆gcd(T,2f (u)) f = 0.
Vì pα - L nên gcd(T, L) và gcd(T, 2f (u)) là ước của T /p.
Vậy ta suy ra ∆T /p ∆T /p f = 0. Suy ra ∆T /p ∆T /p ∆1 f = 0.
Vì ∆T ∆1 f = 0 nên áp dụng bổ đề (21.2) cho hàm số ∆1 f , ta được ∆T /p ∆1 f = 0.
Như vậy, f là một (T /p)-quasi-periodic, mâu thuẫn với tính nhỏ nhất của T .

¨ Bước 3. Tìm tất cả các hàm f .


Gọi d là ước chung lớn nhất của f (x) với x ∈ Z. Khi đó d là số lẻ.
Theo bước 2, ta có d là một quasi-period của f nên ∆d f là hằng số.
Vì ∆d f là số chẵn chia hết cho d nên ta đặt ∆d f = 2dk với k là một số nguyên.
f (i)
Tiếp theo, với mọi i = 0, 1, ..., d − 1, đặt hi = . Khi đó hi là số lẻ. Ta có
d
f (md + i) = ∆md f (i) + f (i) = 2kmd + hi d với mọi m ∈ Z và i = 0, 1, ..., d − 1

Ta mô tả hàm số thỏa mãn yêu cầu đề bài:


Cố định một số nguyên dương d lẻ, một số nguyên bất kì k và các số lẻ h0 , h1 , ..., hd−1 . Khi đó

f (md + i) = ∆md f (i) + f (i) = 2kmd + hi d với m ∈ Z và i = 0, 1, ..., d − 1

Thử lại thỏa mãn. â

+ Do Math then Love Math 30 h Việt Nguyễn − Mathpiad


 Bài 22 Tìm tất cả các hàm số f : R+ → R+ thỏa mãn

xf (x2 )f (f (y)) + f (yf (x)) = f (xy) f (f (x2 )) + f (f (y 2 )) , ∀ x, y ∈ R+


 

A4 IMO Shortlist 2016

Lời giải . P (1, 1) ⇒ f (1)f (f (1)) + f (f (1)) = 2f (1)f (f (1)) ⇒ f (1) = 1.


Mặt khác, hoán đổi vai trò của x và y, ta được

xf (x2 )f (f (y)) + f (yf (x)) = yf (y 2 )f (f (x)) + f (xf (y)), ∀ x, y ∈ R+ (*)

¨ PHƯƠNG TRÌNH HÀM TRONG IMO SHORTLIST


Thay y bởi 1 vào (∗), ta được

xf (x2 ) + f (f (x)) = f (f (x)) + f (x) ⇔ xf (x2 ) = f (x), ∀ x ∈ R+ (1)

Thay y bởi 1 vào đề bài và áp dụng (1), ta được

f (x) + f (f (x)) = f (x) f (f (x2 )) + 1 ⇔ f (f (x)) = f (x)f (f (x2 )), ∀ x ∈ R+


 
(2)
Å ã
f (x)
¨ Từ xf (x ) = f (x), ta có f (f (x )) = f
2 2
x
Å ã
1
¨ P x, , kết hợp với (1), ta được
x
Å Å ãã Å ã Å Å ãã
1 f (x) 2 1
f (x)f f +f = f (f (x )) + f f
x x x2

Kết hợp với đẳng thức ở trên, ta suy ra


Å Å ãã Å Å ãã
1 1
f (x)f f =f f (3)
x x2

1
¨ Thay x bởi vào (2), ta được
x
Å Å ãã Å ã Å Å ãã
1 1 1
f f =f f f (4)
x x x2

Từ (3) và (4), ta suy ra Å ã


1
f (x)f = 1, ∀ x ∈ R+ (5)
x
Å ã
1 1
Thay (x, y) bởi , vào đề bài, kết hợp với (5), ta được
x y
ï ò
1 1 1 1 1
+ = +
xf (x2 )f (f (y)) f (yf (x)) f (xy) f (f (x2 )) f (f (y 2 ))

Sau khi quy đồng, ta suy ra

f (xy)2 f (f (x2 ))f (f (y 2 )) = xf (x2 )f (f (y))f (yf (x)), ∀ x, y ∈ R+

Áp dụng (1) và (2), ta được

f (xy)2 f (f (x)) = f (x)2 f (y)f (yf (x)), ∀ x, y ∈ R+ (**)

+ Do Math then Love Math 31 h Việt Nguyễn − Mathpiad


Thay y bởi f (x) vào (**) và áp dụng (1), ta được

f (xf (x))2 f (f (x)) = f (f (x))f (x)2 f (f (x)2 ) ⇒ f (xf (x))2 = f (x)f (f (x)), ∀ x ∈ R+ (6)

Thay y bởi x vào (**), áp dụng (6), ta được

f (x2 )2 f (f (x)) = f (x)3 f (xf (x))

f (x)2
⇔ f (f (x)) = f (x)3 f (xf (x)) ⇔ f (f (x)) = x2 f (x)f (xf (x))
x2
⇔ f (f (x))2 = x4 f (x)2 f (xf (x))2 = x4 f (x)3 f (f (x)) ⇔ f (f (x)) = x4 f (x)3 , ∀ x ∈ R+ (7)

Áp dụng (1), (2) và (7), ta suy ra


1
x4 f (x)3 = f (f (x)) = f (x)f (f (x2 )) = f (x)x8 f (x2 )3 = x5 f (x)4 ⇔ f (x) = , ∀ x ∈ R+
x
1
LATEX VÀ DỊCH THUẬT BỞI VIỆT

Thử lại thỏa mãn. Vậy hàm số thỏa mãn đề bài là f (x) = , ∀ x ∈ R+ â
x

 Bài 23 Tìm tất cả các hàm số f : R → R thỏa mãn f (0) 6= 0 và

f (x + y)2 = 2f (x)f (y) + max{f (x2 ) + f (y 2 ), f (x2 + y 2 )}

A7 IMO Shortlist 2016

Lời giải . P (0, 0), ta được f (0)2 = 2f (0)2 + max{2f (0), f (0)}.

¨ Nếu f (0) < 0 thì f (0)2 + f (0) = 0 ⇒ f (0) = −1

¨ Nếu f (0) > 0 thì f (0)2 + 2f (0) = 0 ⇒ f (0) = −2, vô lí.

P (x, 0), ta được


f (x)2 = −2f (x) + f (x2 ) (1)
Từ (1), ta suy ra f (x)2 + 2f (x) = f (−x)2 + 2f (−x). Từ đó, ta được

f (x) = f (−x) hoặc f (x) + f (−x) = −2 (2)

P (x, x), ta được f (2x)2 − 2f (x)2 = max{2f (x2 ), f (2x2 )}.


P (x, −x), ta được 1 − 2f (−x)f (x) = max{2f (x2 ), f (2x2 )}.
Vậy ta suy ra
f (2x)2 − 2f (x)2 = 1 − 2f (−x)f (x) (3)

Nhận xét. Nếu f (x) + f (−x) = 2 thì ta có


! f (2x)2 = 4f (x)2 − 2f (−x)f (x) − 2f (x)2 + 1 = 4f (x)2 + 4f (x) + 1 = [2f (x) + 1]2

Bây giờ, ta sẽ chứng minh f (x) + f (−x) = −2, ∀ x ∈ R. Thật vậy, giả sử tồn tại số a sao cho
f (a) + f (−a) 6= −2. Suy ra f (a) = f (−a) 6= −1. Ta giả sử a > 0

¨ Bước 1. Ta sẽ chứng minh f (a) 6= 1. Thật vậy, giả sử f (a) = 1. Thay a bởi 1 vào (3), ta được
f (2a)2 = 1. P (a, a), ta được 1 = 2 + max{2f (a2 ), f (2a2 )}.
Mặt khác từ (1), ta có f (a2 ) = 3. Vậy ta có 1 > 2 + 6, vô lí.

+ Do Math then Love Math 32 h Việt Nguyễn − Mathpiad


a  −a 
¨ Bước 2. Từ bước 1, ta suy ra f (a) 6= ±1. Nên theo (3), f 6= f .
2 2  
 a  −a 
Nên theo nhận xét thì f (a) = ± 2f + 1 . Tương tự, f (−a) = ± 2f +1 .
a  −a  2 2
Do f 6= f và f (a) = f (−a) nên ta phải có
2 2
a  −a 
2f + 1 + 2f + 1 = 0 ⇔ −4 + 2 = 0, vô lí.
2 2

Vậy ta đã chứng minh được f (x) + f (−x) = −2, ∀ x ∈ R

¨ PHƯƠNG TRÌNH HÀM TRONG IMO SHORTLIST


P (−x, −y), ta được

f (−x − y)2 = 2f (−x)f (−y) + max{f (x2 ) + f (y 2 ), f (x2 + y 2 )}

Kết hợp với đề bài, ta suy ra

f (−x − y)2 − 2f (−x)f (−y) = f (x + y)2 − 2f (x)f (y)

⇔ [f (x + y) + 2]2 − 2 [f (x) + 2] [f (y) + 2] = f (x + y)2 − 2f (x)f (y)


⇔ f (x + y) = f (x) + f (y) + 1
Đặt g(x) = f (x) + 1, ta thu được g(x + y) = g(x) + g(y), ∀ x, y ∈ R.
Hơn nữa, (1) tương đương g(x)2 = f (x)2 + 2f (x) + 1 = f (x2 ) + 1 = g(x2 ), ∀ x ∈ R
Từ đó suy ra g(x) = Cx, ∀ x ∈ R với C là một hằng số.
Thử lại, ta có g(x)2 = C 2 x2 = g(x2 ) = Cx2 nên C = 0 hoặc C = 1.
Khi đó f (x) = −1 hoặc f (x) = x − 1. Hai hàm này đều thỏa mãn yêu cầu đề bài. â

 Bài 24 Tìm tất cả các hàm số f : R → R sao cho

f (f (x)f (y)) + f (x + y) = f (xy), ∀ x, y ∈ R

A6 IMO Shortlist 2017

Lời giải . Ta nhận xét rằng nếu hàm số f (x) thỏa mãn yêu cầu đề bài thì hàm số −f (x) cũng thỏa
mãn yêu cầu đề bài.
Vì vậy, không mất tính tổng quát, giả sử f (0) 6 0.
Ta bỏ qua trường
Å hợp tầmã thường f là hằng số 0.
x
Xét x 6= 1, P x, , ta được
x−1
Å Å ãã
x
f f (x)f =0 (1)
x−1

Thay x bởi 0 vào (1), Ta được f (f (0)2 ) = 0


¨ Trường hợp 1. f (0) = 0.
P (x, 0) ⇒ f (x) = 0, ∀ x ∈ R

¨ Trường hợp 2. f (0) < 0.


Ta thấy rằng tồn tại a sao cho f (a) = 0. Nếu a 6= 1 thì thay x bởi a vào (1), ta được f (0) = 0,
mâu thuẫn. Vậy f (x) = 0 ⇔ x = 1.
Suy ra f (0)2 = 1 mà f (0) < 0 nên f (0) = −1.
P (x, 1) ⇒ f (0) + f (x + 1) = f (x) ⇒ f (x + 1) = f (x) + 1, ∀ x ∈ R.
Tiếp theo, ta chứng minh f là một hàm đơn ánh.

+ Do Math then Love Math 33 h Việt Nguyễn − Mathpiad


Thật vậy, giả sử tồn tại a 6= b mà f (a) = f (b).
Từ f (x + 1) = f (x) + 1, ta suy ra f (x + n) = f (x) + n, ∀ n ∈ Z. Ta có

f (a + N + 1) = f (a) + N + 1 = f (b) + N + 1 = f (b + N ) + 1, ∀ N ∈ N
®
x0 + y0 = a + N + 1
Chọn số nguyên N < −b, tồn tại hai số thực x0 , y0 sao cho
x0 y0 = b + N
P (x0 , y0 ), ta được f (f (x0 )f (y0 )) + f (a + N + 1) = f (b + N ). Suy ra

f (f (x0 )f (y0 )) + 1 = 0 ⇔ f (f (x0 )f (y0 ) + 1) = 0 ⇔ f (x0 )f (y0 ) + 1 = 1

Vậy f (x0 ) hoặc f (y0 ) bằng 0 hay x0 hoặc y0 bằng 1, vô lí do a 6= b.


Vậy f là hàm đơn ánh. P (x, −x), ta được

f (f (x)f (−x)) + f (0) = f (−x2 ) ⇔f (f (x)f (−x)) = f (−x2 ) + 1 = f (1 − x2 )


⇔f (x)f (−x) = 1 − x2
LATEX VÀ DỊCH THUẬT BỞI VIỆT

P (x, 1 − x), kết hợp với tính chất trên, ta được

f (f (x)f (1 − x)) + f (1) = f (x(1 − x)) ⇔ f (f (x)f (1 − x)) = f (x(1 − x))


⇔ f (x)f (1 − x) = x(1 − x)
⇔ f (x) (1 + f (−x)) = x − x2
⇔ f (x) − x2 + 1 = x − x2
⇔ f (x) = x − 1

Vậy tất cả các hàm số thỏa mãn là f (x) ∈ {0, x − 1, 1 − x} â

 Bài 25 Tìm tất cả các hàm số f : R → R thỏa mãn điều kiện sau

Với mọi x, y ∈ R sao cho (f (x) + y) (f (y) + x) > 0, ta có f (x) + y = f (y) + x

Chứng minh rằng f (x) + y 6 f (y) + x khi và chỉ khi x > y

A8 IMO Shortlist 2017

Lời giải . Đặt g(x) = x − f (x). Điều kiện của đề bài có thể viết lại thành

Với mọi x, y ∈ R mà (x + y − g(x)) (x + y − g(y)) > 0, ta có g(x) = g(y)

Điều kiện này có thể viết lại thành

Nếu g(x) 6= g(y) thì x + y nằm giữa g(x) và g(y) (*)

Ta cũng dễ thấy rằng hàm số g1 (x) = −g(−x) cũng thỏa mãn điều kiện (∗)
Ta cần phải chứng minh g(x) 6 g(y) khi và chỉ khi x < y.

¨ Bước 1. Giả sử tồn tại số x sao cho X = g(x) < 2x. Khi đó trên đoạn (X − x, x] hàm số g chứa
nhiều nhất hai giá trị là X và một số Y > X. Tương tự, nếu X > 2x, trên đoạn [x, X − x) g
chứa nhiều nhất hai giá trị là X và một số Y > X
Chứng minh. Giả sử tồn tại một số a ∈ (X − x, x) mà g(a) 6= X.
Nếu g(a) < X thì từ điều kiện (∗) suy ra g(a) < a + x 6 g(x) = X nên a 6 X − x, vô lí.
Vậy g(a) > X và từ (*), ta có X 6 a + x 6 g(a).

+ Do Math then Love Math 34 h Việt Nguyễn − Mathpiad


Bây giờ giả sử tồn tại một số b 6= a nằm trong khoảng (X − x, x) mà g(b) 6= X. Tương tự, ta
cũng có b + x 6 g(b). Khi đó do a + b < a + x 6 g(a) và a + b < b + x 6 g(b) nên từ điều kiện
(∗), ta được g(a) = g(b).
Vậy trên đoạn (X − x, x] hàm số g chứa nhiều nhất hai giá trị là X và một số Y > X.
Để chứng minh tính chất thứ hai, ta để ý rằng g1 (−x) = −X < −2x nên hàm số g1 chứa nhiều
nhất hai giá trị trên đoạn (−X + x, −x] là −X và một số −Y > −X. Thay lại vào hàm g, ta
suy ra được điều phải chứng minh.
¨ Bước 2. Nếu X < 2x thì g là hằng số trên (X − x, x). Trường hợp X > 2x, ta chứng minh
tương tự như trên nên ta chỉ cần chứng minh trường hợp đầu.

¨ PHƯƠNG TRÌNH HÀM TRONG IMO SHORTLIST


Chứng minh. Giả sử tồn tại hai số a, b mà g(a) = X và g(b) = Y > X.
Ta có min{X−a, X−b} > X−x nên tồn tại một số u ∈ (X−x, x) sao cho u < min{X−a, X−b}.
Vậy từ bước 1, ta có g(u) = X hoặc g(u) = Y .
Từ điều kiện (∗), ta có X 6 u + b 6 Y , mâu thuẫn với điều kiện u < X − b
¨ Bước 3. Nếu X < 2x, thì g(a) = X, ∀ a ∈ (X − x, x).
Nếu X > 2x thì g(a) = X, ∀ a ∈ (x, X − x).
Chứng minh. Ta chỉ cần chứng minh trong trường hợp X < 2x.
Giả sử g(t) = Y > X, ∀ t ∈ (x, X − x).
Chọn a < b ∈ (X − x, x). Từ điều kiện (∗), ta suy ra Y > b + x > X
Ta có Y > b + x > 2b > 2a. Từ đó suy ra a < b < x 6 Y − b < Y − a nên x, b ∈ (a, Y − a).
Ta có g là hằng số trên (a, Y − a) nên ta suy ra g(b) = g(x) = X.
Vậy g là hằng số trên (X − x, x)
Giả sử tồn tại y > x sao cho g(x) > g(y) 6= 2y. Đặt X = g(x) và Y = g(y).
Từ điều kiện (∗), ta có X > x + y > Y nên Y − y 6 x < y 6 X − x.
Từ đó suy ra (Y − y, y) ∩ (x, X − x) = (x, y) 6= Ø.
Từ bước 3, ta suy ra g là hằng số trên đoạn (Y − y, y) và (x, X − x).
Suy ra g là hằng số trên (x, y), vô lí.
Nếu g(y) = 2y và y > x thì x + y − g(y) = x − y < 0 nên g(y) = 2y > x + y > g(x).
Vậy ta chứng minh được g(x) 6 g(y) ⇔ x < y. Kết thúc chứng minh. â

 Bài 26 Tìm tất cả các hàm số f : Q>0 → Q>0 thỏa mãn

f (x2 f (y)2 ) = f (x)2 f (y), ∀ x, y ∈ Q>0

A1 IMO Shortlist 2018

Lời giải . Ta có
f (f (a))2 f (f (b))2
f (f (a)2 f (b)2 ) = f (f (a))2 f (b) = f (f (b))2 f (a) ⇒ = , ∀ a, b ∈ Q>0
f (a) f (b)
f (f (a)) 2 f (a)
Å ã
2
Suy ra tồn tại hằng số C mà f (f (a)) = Cf (a). Suy ra = , ∀ a ∈ Q>0 .
C C
Kí hiệu f n (x) là hợp của n lần hàm f . Ta có
Å 2 ã2 Å 3 ã4 Å n+1 ã2n
f (a) f (a) f (a) f (a)
= = = ... = ,∀n ∈ N
C C C C

2n f (a) f n+1 (a)
Suy ra = là một số hữu tỉ với mọi số nguyên dương n.
C C
f (a)
Điều này chỉ xảy ra khi = 1 hay f (a) = C. Thử lại, ta được C = C 3 ⇔ C = 1.
C
Vậy hàm số thỏa mãn là f (x) = 1, ∀ x ∈ Q>0 â

+ Do Math then Love Math 35 h Việt Nguyễn − Mathpiad


 Bài 27 Tìm tất cả các hàm số f : (0, ∞) → R thỏa mãn
Å ã
1 y
x+ f (y) = f (xy) + f , ∀ x, y ∈ R+
x x

A5 IMO Shortlist 2018

Lời giải . Cố định một số a > 1, ta có


Å ã
1
P (x, x) ⇒ x + f (x) = f (x2 ) + f (1) (1)
x
x  x a
P , ax ⇒ + f (ax) = f (x2 ) + f (a2 ) (2)
a Å a xã Å ã
2 2 1 2 2 1
P (a x, x) ⇒ a x + 2 f (x) = f (a x ) + f (3)
ax a2
Å ã
1
(ax, ax) ⇒ ax + f (ax) = f (a2 x2 ) + f (1) (4)
LATEX VÀ DỊCH THUẬT BỞI VIỆT

ax
Từ (1) và (2), ta có
Å ã
1 x a
x+ f (x) − + f (ax) = f (1) − f (a2 ) = A (5)
x a x
Từ (3) và (4), ta có
Å ã Å ã Å ã
2 1 1 1
a x+ 2 f (x) − ax + f (ax) = f − f (1) = B (6)
ax ax a2
Từ (5) và (6), ta suy ra
ïÅ ãÅ ã  Å ãò Å ã
1 1 x a 1 1 x a
ax + x+ − + ax + f (x) = A ax + +B + (7)
ax x a x ax ax a x
Mặt khác, ta có
Å ãÅ ã  Å ã
1 1 x a 1 1 1
ax + x+ − + ax + = a + − a3 − 3 = C < 0
ax x a x ax a a
Vậy phương trình (7) tương đương
E C2
Cf (x) = Dx + ⇒ f (x) = C1 x +
x x
Thử lại ta thấy hàm này thỏa mãn. â

 Bài 28 Xét hàm số f : Z → Z thỏa mãn

f (f (x + y) + y) = f (f (x) + y), ∀ x, y ∈ Z

Ta gọi một số nguyên v là hiếm nếu tập hợp Xv = {x ∈ Z : f (x) = v} là hữu hạn và khác
rỗng.

1 Chứng minh rằng tồn tại một hàm số f thỏa mãn có một số hiếm.

2 Chứng minh rằng không tồn tại hàm số f nào có nhiều hơn 1 số hiếm.

A7 IMO Shortlist 2019

Lời giải .

+ Do Math then Love Math 36 h Việt Nguyễn − Mathpiad


1 Ta xét hàm số f sao cho f (0) = 0 và f (x) = 2v2 (2x) , ∀ x ∈ Z \ {0}.
Như vậy ta có số 0 là số hiếm.
Bây giờ ta chứng minh hàm số f như cách chọn trên thỏa mãn yêu cầu đề bài.

¨ Nếu y = 0, hàm số thỏa mãn.


¨ Nếu x = 0, khi đó f (f (y) + y) = f (y) do v2 (f (y) + y) = v2 (y)
¨ Nếu x, y 6= 0. Ta có f (2x) = 2f (x), ∀ x ∈ Z \ {0} nên ta chỉ cần chứng minh hàm số này
thỏa mãn điều kiện với ít nhất một trong hai số đều lẻ x, y là các số nguyên lẻ. Thật vậy
ta có hai trường hợp sau

¨ PHƯƠNG TRÌNH HÀM TRONG IMO SHORTLIST


Trường hợp 1. y là số lẻ. Khi đó f (x + y) + y và f (x) + y là số lẻ nên

f (f (x + y) + y) = f (f (x) + y) = 21 = 2

Trường hợp 2. y là số lẻ, x là số chẵn. Khi đó

f (f (x + y) + y) = f (f (x) + y) = f (2 + y)

2 Giả sử tồn tại nhiều hơn 1 số hiếm. Ta dễ thấy

f (f (x + ky) + y) = f (f (x) + y), ∀ x, y, k ∈ Z (*)

Nếu v là một số hiếm và a là phần tử nhỏ nhất thuộc Xv , thay y bởi a − f (x), ta được

f (f (x + k(a − f (x))) − f (x) + a) = f (a) ∈ Xv ⇒ f (x + k(a − f (x))) − f (x) + a > a, ∀ x, k ∈ Z

⇒ f (x + k(a − f (x))) > f (x), ∀ x, k ∈ Z (1)


Tương tự, nếu ta gọi b là phẩn tử lớn nhất thuộc Xv , ta cũng có

f (x + k(b − f (x))) 6 f (x), ∀ x, k ∈ Z (2)

Từ (1) và (2), ta suy ra

f (x + k(a − f (x))(b − f (x))) = f (x), ∀ x, k ∈ Z

Từ đẳng thức trên, nếu tồn tại một số t sao cho f (t) 6= a, b thì tập hợp Xf (t) là tập vô hạn nên
f (t) không thể là số hiếm. Vậy ta chỉ có hai số hiếm tối đa có thể là a và b với a 6= b.
Ta có v cũng là số hiếm nên v = a hoặc v = b. Ta có f (v) = f (a) = f (b) = v nên không thể
tồn tại số hiếm u 6= v vì nếu u là số hiếm thì f (u) = f (a) = f (b) = u 6= v, mâu thuẫn.
â

 Bài 29 Tìm tất cả các hàm số f : Z → Z sao cho

fa2 +b2 (a + b) = af (a) + bf (b), ∀ a, b ∈ Z

Với kí hiệu fn (x) là hợp n lần hàm số x.

A6 IMO Shortlist 2020

Lời giải . P (0, b) ⇒ fb2 (b) = bf (b). P (0, −1) ⇒ f (−1) = −f (−1) ⇒ f (−1) = 0
P (a, −1) ⇒ fa2 +1 (a − 1) = af (a) = fa2 (a).
Ta gọi O(x) = {x, f (x), f (f (x)), ...} ⊂ Z là orbit của x. Từ trên, ta có O(x) và O(x − 1) khác nhau
một lượng phần tử hữu hạn. Vậy ta suy ra O(a) và O(b) khác nhau một lượng phần tử hữu hạn với
a, b bất kì.

+ Do Math then Love Math 37 h Việt Nguyễn − Mathpiad


¨ Trường hợp 1. Các orbit có hữu hạn giá trị. O(0) cũng có hữu hạn giá trị
P (a, −a) ⇒ f2a2 (0) = af (a) − af (−a) ∈ O(0)
Chọn a > max |z|, ta suy ra f (a) = f (−a) nên f2a2 (0) = 0. Như vậy, ta được dãy fk (0) tuần
z∈O(0)
hoàn theo chu kì T là ước của 2a2 .
Tương tự, ta cũng có T là ước của 2(a + 1)2 nên T | gcd (2a2 , 2(a + 1)2 ) = 2.
Vậy ta suy ra f (f (0)) = 0 nên af (a) − af (−a) = f2a2 (0) = 0
⇒ f (a) = f (−a), ∀ a ∈ Z \ {0} ⇒ f (1) = f (−1) = 0 (1)
Xét n là số nguyên dương bất kì, P (n, 1 − n). Suy ra
nf (n) + (1 − n)f (1 − n) = fn2 +(1−n)2 (1) = f2n2 −2n (0) = 0 (2)
Ta gọi m 6= 0 là số mà |m| nhỏ nhất và f (m) 6= 0. Từ (1), ta có m > 1.
Do f (a) = f (−a), ∀ a ∈ Z \ {0} nên f (|m|) 6= 0. Từ (2), suy ra f (1 − |m|) 6= 0.
Suy ra f (|m| − 1) 6= 0, mâu thuẫn với tính nhỏ nhất của |m|. Vậy f (n) = 0, ∀ n 6= 0. Ta có
f (0) = f3 (0) = f4 (2) = 2f (2) = 0
Vậy hàm số thỏa mãn đề bài trong trường hợp này là f (x) = 0, ∀ x ∈ Z
LATEX VÀ DỊCH THUẬT BỞI VIỆT

¨ Trường hợp 2. Các orbit chứa vô hạn giá trị.


Cố định a, b ∈ Z, ta suy ra rằng các cặp (m, n) không âm thỏa mãn fn (a) = fm (b) có hiệu
n − m giống nhau.
Thật vậy nếu tồn tại fn (a) = fm (b) và fp (a) = fq (b) mà n − m > p − q thì khi đó fp+m+k (b) =
fp+n+k (a) = fq+n+k (b). Suy ra dãy fl (b) tuần hoàn hay O(b) có hữu hạn giá trị, vô lí.
Bây giờ, với mọi a, b ∈ Z, ta kí hiệu X(a, b) là hiệu m − n. Ta có X(a − 1, a) = 1
Ta cũng suy ra được X(a, b) + X(b, c) = X(a, c).
Thật vậy, nếu fn (a) = fm (b) và fp (b) = fq (c) thì fp+n (a) = fp+m (b) = fq+m (c).
Từ đó suy ra X(a, b) = b − a, ∀ a, b ∈ Z. Suy ra
fa2 +1 (f (a − 1)) = fa2 (f (a)) ⇒ 1 = X (f (a − 1), f (a)) = f (a) − f (a − 1), ∀ a ∈ Z
Kết hợp với f (−1) = 0, ta có f (x) = x + 1, ∀ x ∈ Z.
Thử lại, fa2 +b2 (a + b) = (a + b) + (a2 + b2 ) = a(a + 1) + b(b + 1) = af (a) + bf (b), đúng.
Vậy ta có hai hàm số f thỏa mãn đề bài là f (x) = 0, ∀ x ∈ Z và f (x) = x + 1, ∀ x ∈ Z. â

 Bài 30 Tìm tất cả các hàm số f : R+ → R+ thỏa mãn

f (x + f (xy)) + y = f (x)f (y) + 1, ∀ x, y ∈ R+

A8 IMO Shortlist 2020

Lời giải . P (1, y) ⇒ f (1 + f (y)) + y = f (1)f (y) + 1. Suy ra f là hàm đơn ánh.
Với y cố định, đặt g(x) = f (x + f (xy)) = f (x)f (y) + 1 − y thì khi đó g là hàm đơn ánh.
Suy ra x1 + f (x1 y) 6= x2 + f (x2 y), ∀ x1 6= x2 , y ∈ R+ . Đặt zi = xi y, ta có
z1 − z2 1 f (z2 ) − f (z1 )
6= f (z2 ) − f (z1 ) hay 6=
y y z1 − z2
f (z2 ) − f (z1 )
Điều này chỉ xảy ra khi < 0. Nói cách khác, ta suy ra f tăng ngặt.
z1 − z2
Do f tăng ngặt nên đặt p = lim f (x) và q = lim f (x).
x→0 x→p
Cố định y, cho x → 0, ta được
q+y−1
q + y = pf (y) + 1 ⇒ f (y) = = ay + b, ∀ y ∈ R+
p

+ Do Math then Love Math 38 h Việt Nguyễn − Mathpiad


Thay vào đề bài, ta có

a(x + axy + b) + b + y = (ax + b)(ay + b) + 1

Tương đương với


(a − ab)x + (1 − ab)y + ab + b − b2 − 1 = 0, ∀ x, y ∈ R+
Điều này chỉ xảy ra khi và chỉ khi a − ab = 1 − ab = ab + b − b2 − 1 = 0 ⇔ a = b = 1.
Vậy ta kết luận hàm số thỏa mãn yêu cầu đề bài là f (x) = x + 1, ∀ x ∈ R+ â

¨ PHƯƠNG TRÌNH HÀM TRONG IMO SHORTLIST


2. Hàm số số học

 Bài 1 Tìm tất cả các hàm số f : Z+ → Z+ toàn ánh sao cho với mọi m, n ∈ Z+ thì
f (m + n) chia hết cho p khi và chỉ khi f (m) + f (n) chia hết cho p

N5 IMO Shortlist 2007

Lời giải . Xét một số nguyên tố p bất kì. Vì f toán ánh nên tồn tại x để p | f (x). Đặt

d = min{x ∈ Z+ : p | f (x)}

Ta dễ dàng suy ra p | f (kd) với k là số nguyên dương bất kì bằng quy nạp.
Giả sử tồn tại một số x mà d - x và p | f (x). Đặt

y = min{x ∈ Z+ : d - x, p | f (x)}

Từ cách chọn d, ta được y > d nên y − d là số nguyên dương không chia hết cho d. Suy ra p - f (y − d).
Mặt khác
p | f (y) ⇔ p | f (y − d) + f (d) ⇔ p | f (y − d), mâu thuẫn.
Từ đó ta suy ra được
p | f (x) ⇐⇒ d | x (1)
Chọn x ≡ y (mod d), áp dụng (1), ta có

p | f (2xd + y − x) ⇔ p | f (y) + f (2xd − x)


p | f (2xd) ⇔ p | f (x) + f (2xd − x)

Suy ra f (x) ≡ f (y) (mod p). Như vậy, ta được

x≡y (mod d) ⇐⇒ f (x) ≡ f (y) (mod p) (2)

Từ (2) ta có số dư của f (x) khi chia cho p nằm trong tập hợp

A = {f (1), f (2), ..., f (d)}

Vì f toàn ánh nên A phải chứa hệ thặng dư modulo p nên d > p.


Nếu d > p thì tồn tại x 6= y 6 d mà f (x) ≡ f (y) (mod p). Suy ra x ≡ y (mod d), vô lí.
Vậy ta đã chứng minh được d = p hay p | f (x) ⇐⇒ p | x. (3)
Do không tồn tại số nguyên tố p nào là ước của 1 nên f (1) không có ước nguyên tố hay f (1) = 1.
Ta chứng minh f (n) = n theo quy nạp. Xét n > 1. Đặt k = f (n), ta có hai trường hợp
¨ Trường hợp 1. Nếu k > n thì k − n + 1 > 1 nên tồn tại số nguyên tố p | k − n + 1.
Khi đó k ≡ n − 1 (mod p) nên f (n − 1) = n − 1 ≡ k = f (n) (mod p) ⇒ p | 1, vô lí.

+ Do Math then Love Math 39 h Việt Nguyễn − Mathpiad


¨ Trường hợp 2. Nếu k < n thì n − k + 1 > 1 nên tồn tại số nguyên tố q | n − k + 1.
Khi đó k = f (n) ≡ f (k − 1) = k − 1 (mod p), vô lí.

Vậy f (x) = x, ∀ x ∈ Z+ . â

 Bài 2 Với mọi n ∈ Z+ , kí hiệu d(n) là số ước dương của n. Tìm tất cả các hàm số
f : Z+ → Z+ thỏa mãn các tính chất sau

(i) d(f (x)) = x, ∀ x ∈ Z+ .

(ii) f (xy) | (x − 1)y xy−1 f (x), ∀ x, y ∈ Z+ .

N5 IMO Shortlist 2008

Lời giải . Ta có d(f (1)) = 1 nên f (1) = 1. Ta có công thức sau

d(p1b1 pb22 ...pbkk ) = (b1 + 1) (b2 + 1) ... (bk + 1) vớib1 , ..., bk ∈ Nvàp1 , ..., pk ∈ P
LATEX VÀ DỊCH THUẬT BỞI VIỆT

Gọi p là số nguyên tố, ta có

d(f (p)) = p ⇒ f (p) = q p−1 với q ∈ P (1)

Ta chứng minh q = p hay f (p) = pp−1 , ∀ p ∈ P.


Áp dụng (ii) với (x, y) = (2, p) và (x, y) = (p, 2), ta được

f (2p) là ước của p2p−1 f (2) và (p − 1)22p−1 f (p) = (p − 1)22p−1 q p−1

Nếu q 6= p thì p - (p − 1)22p−1 q p−1 . Nếu p lẻ, ta có

gcd p2p−1 f (2), (p − 1)22p−1 q p−1 = d | f (2)




Suy ra f (2p) | f (2) với f (2) là số nguyên tố. Vì f (2p) > 1 nên f (2p) = f (2).
Suy ra 2p = d(f (2p)) = d(f (2)) = 2 ⇔ p = 1, vô lí. Vậy q = p hay f (p) = pp−1 , ∀ p ∈ P \ {2}.
Áp dụng (ii) với (x, y) = (2, 3) và (x, y) = (3, 2), ta được

f (6) là ước của 35 f (2) và 26 f (3) = 26 .32

Từ (1), ta có f (2) = q 2−1 = q là số nguyên tố.


Nếu f (2) lẻ thì f (6) | gcd (35 f (2), 26 .32 ) | 32 = 9 nên f (6) ∈ {1, 3, 9}.
Lại có 6 = d(f (6)) ∈ {d(1), d(3), d(9)}, vô lí. Vậy f (2) chẵn nên f (2) = 2.
Xét số n bất kì, gọi p là ước
Å nguyênã tố nhỏ nhất của n.
n
Áp dụng (ii) với (x, y) = p, , ta được
p

f (n) | (p − 1)y n−1 f (p) = (p − 1)y n−1 pp−1

Đặt f (n) = `.P với gcd(`, n) = 1 và P | nn−1 . Ta có ` | p − 1 nên d(`) 6 ` < p.


Từ điều kiện (i), ta có
n = d(f (n)) = d(`.P ) = d(`).d(P )
Suy ra d(`) là ước của n mà lại bé hơn p. Suy ra d(`) = 1 hay ` = 1.
Ta suy ra f (n) chỉ có các ước nguyên tố là ước của n.
Vậy với p là số nguyên tố và a > 1 thì ước nguyên tố của f (pa ) chỉ có thể là b hay f (pa ) = pb .
a
Áp dụng (i), ta được b + 1 = d(pb ) = d(f (pa )) = pa . Suy ra f (pa ) = pp −1 .

+ Do Math then Love Math 40 h Việt Nguyễn − Mathpiad


Bây giờ, giả sử n có phân tích tiêu chuẩn là pa11 pa22 ...pakk . Đặt f (n) = pb11 pb22 ...pbkk .
Với i = 1, 2, ..., k, áp dụng (ii) với (x, y) = (pai i , n/x), ta có
a
p i −1
pbi i | f (n) | (pai i − 1)y n−1 f (pai i ) ⇒ pbi i | f (pai i ) = pi i ⇒ bi 6 pai i − 1

Áp dụng vừa suy ra và (i), ta được

pa11 pa22 ...pkak = n = d(f (n)) = d(pb11 pb22 ...pbkk ) = (b1 + 1) (b2 + 1) ... (bk + 1) 6 pa11 pa22 ...pakk

Vậy dấu bằng phải xảy ra hay bi = pai i − 1 với i = 1, 2, ..., k.

¨ PHƯƠNG TRÌNH HÀM TRONG IMO SHORTLIST


Kết luận. Hàm số thỏa mãn đề bài là
k a
p i −1
Y
Nếu n = pa11 pa22 ...pakk thì f (n) = pi i
i=1

 Bài 3 Cho hàm số khác hằng f : Z+ → Z+ thỏa mãn

a − b | f (a) − f (b), ∀ a 6= b ∈ Z+

Chứng minh rằng tồn tại vô hạn số nguyên tố p mà p | f (c) với c ∈ Z+ .

N3 IMO Shortlist 2009

Lời giải . Giả sử rằng f (Z+ ) chỉ có các ước nguyên tố là p1 , p2 , ..., pm .
Ta chọn số a sao cho vpi (a) > vpi (f (1)) với i = 1, 2, ..., m, ta có

a | f (a + 1) − f (1)

Nếu f (a + 1) 6= f (1) thì tồn tại một số i để vpi (f (a + 1)) 6= vpi (f (1)).
Khi đó
vpi (f (a + 1) − f (1)) = min{vpi (f (a + 1)), vpi (f (1))} 6 vpi (f (1)) < vpi (a)
Mà a | f (a + 1) − f (1) nên vpi (f (a + 1) − f (1)) > vpi (a), mâu thuẫn.
Vậy ta có f (a + 1) = f (1). Từ đề bài, ta được

(a + 1) − b | f (a + 1) − f (b) ⇒ (a + 1) − f (b) | f (1) − f (b)

Cho a → ∞, ta được f (b) = f (1), ∀ b ∈ Z+ , mâu thuẫn đề bài.


Vậy giả sử phản chứng sai. Kết thúc chứng minh. â

 Bài 4 Tìm tất cả các hàm số f : Z+ → Z+ sao cho (f (m) + n) (f (n) + m) là số chính
phương.

N5 IMO Shortlist 2010

Lời giải .

! Nếu p | f (k) − f (`) với p là số nguyên tố thì p | k − `


Chứng minh.

+ Do Math then Love Math 41 h Việt Nguyễn − Mathpiad


¨ Trường hợp 1. Nếu p2 | f (k) − f (`) thì f (`) = f (k) + ap2 .
Chọn D > max{f (k), f (`)} không chia hết cho p và đặt n = pD − f (k).
Ta được n + f (`) = pD + f (`) − f (k) = p(D + pa).
Ta có (f (k) + n) (f (n) + k) và (f (`) + n) (f (n) + `) là số chính phương chia hết cho p nên
f (n) + k và f (n) + ` chia hết cho p. Suy ra p | k − `.
¨ Trường hợp 2. Nếu p2 - f (k) − f (`) thì chọn số D tương tự và n = p3 D − f (k).
Tương tự, ta được f (k) + n và f (`) + n có vp lẻ.
Suy ra p là ước của f (n) + k và f (n) + ` nên p | k − `.
Giả sử tồn tại f (k) = f (`) thì p | f (k) − f (`) nên p | k − ` với mọi p nguyên tố. Điều này chỉ xảy ra
khi k = `. Vậy f là hàm đơn ánh.
Xét hai số f (k) và f (k + 1). Nếu tồn tại số p là ước của f (k + 1) − f (k) thì p | (k + 1) − k = 1, vô lí.
Vậy ta suy ra |f (k + 1) − f (k)| = 1. Đặt q = f (2) − f (1) với |q| = 1.
Khi đó, do f đơn ánh nên ta được f (n) = f (1) + (n − 1)q với mọi n.
Do f (n) ∈ Z+ nên q = 1 hay f (n) = n + c với c > 0.
Vậy hàm số thỏa mãn là f (x) = x + c, ∀ x ∈ Z+ và c > 0. â
LATEX VÀ DỊCH THUẬT BỞI VIỆT

 Bài 5 Tìm tất cả các hàm số f : Q → Z thỏa mãn


Å ã
f (x) + a x + a
f =f
b b

Với mọi x ∈ Q, a ∈ Z và b ∈ Z+ .

N6 IMO Shortlist 2013

Lời giải .
¨ Trường hợp 1. Tồn tại số nguyên m mà f (m) 6= m.
P (m, rb − f (m), b) với r ∈ Z và b = |m − f (m)| thì ta có
m + rb − f (m)
Å ã
f (r) = f = f (r + u) với u ∈ {1, −1}
b
P (r, a, 1) và P (r + u, a, 1), ta được

f (r + a) = f (r + u + a), ∀ a ∈ Z

Suy ra f (x) = C với mọi số nguyên x.


p
Đặt một số hữu tỉ y = . P (C − p, p − C, q), ta được
q
C +p−C C −p+p−C
Å ã Å ã
f =f ⇒ f (y) = f (0) = C
q q
Vậy f (x) là hàm hằng.
¨ Trường hợp 2. f (m) = m, ∀ m ∈ Z
P (x, a, 1) và do f (x) ∈ Z nên ta được

f (x) + a = f (x + a), ∀ (x, a) ∈ Q × Z (1)


Å ã
1
Đặt f = A, ta chứng minh bài toán theo ba bước
2

+ Do Math then Love Math 42 h Việt Nguyễn − Mathpiad


Å ã
1
Bước 1. Nếu A 6 0 thì P , −A, 1 − 2A , ta được
2
1
Ö è
Å
A−A
ã −A Å ã
1
f =f 2 =f = A = f (0) = 0
−1 − 2A −1 − 2A 2

Å ã
1
Nếu A > 0 thì P , A − 1, 2A − 1 , ta được A = 1.
2
Vậy A ∈ {0, 1}.

¨ PHƯƠNG TRÌNH HÀM TRONG IMO SHORTLIST


Bước 2. f (x) = A với mọi 0 < x < 1.
a a
Giả sử tồn tại số hữu tỉ ∈ (0, 1) với gcd(a, b) = 1 và b > 2 nhỏ nhất mà f 6= A.
b b
1 a−1 b
Å ã
Nếu b là số chẵn thì a là số lẻ. P , , , ta được
2 2 2
a−1
Ö è
Å
2A + a − 1
ã A + a
f =f 2 =f 6= A
b b b
2
a−1
a Å ã Å ã
a+1
Điều này có nghĩa là f =f nếu A = 0 và bằng f nếu A = 1.
b b b
Làm tương tự như vậy, ta được A 6= 0 và A 6=Å1, vô lí.ã
1
Suy ra b là số lẻ, đặt b = 2k + 1 với k > 1. P , k, b , ta được
2
Å ã Å ã
A+k 1
f =f =A
b 2

Vì gcd(a, b) = 1 nên tồn tại r ∈ {1, 2, ..., b} và m sao cho ra − mb = k + A.


Nếu m < 0 thì ra − mb > b = 2k + 1 > k + A, vô lí.
Å tự, nếu m
Tương ã > r thì ra − mb < ra − rb = r(a − b) < 0, vô lí. Vậy 0 6 m 6 r − 1.
k+A
P , m, r , ta được
b Å ã
A+m a
f =f 6= A
r b
Điều này mâu thuẫn với tính nhỏ nhất của b.

Bước 3. Từ (3), ta được f (x + 1) = f (x) + 1, ∀ x ∈ Q. Như vậy ta có

¨ Nếu A = 0 thì f (x) = bxc


¨ Nếu A = 1 thì f (x) = dxe

Thử lại thỏa mãn.

Vậy các hàm số thỏa mãn đề bài là

1 f (x) = C, ∀ x ∈ Q

2 f (x) = bxc, ∀ x ∈ Q

3 f (x) = dxe, ∀ x ∈ Q

+ Do Math then Love Math 43 h Việt Nguyễn − Mathpiad


â

 Bài 6 Xét hàm số f : Z+ → Z+ . Kí hiệu f n (m) = f (f (...f (m)...)). Giả sử f có hai tính
| {z }
n
chất sau
f n (m) − m
1 Nếu m, n ∈ Z+ thì ∈ Z+
n
2 Tập Z+ \ {f (n) : n ∈ Z+ } là hữu hạn.

Chứng minh rằng dãy f (1) − 1, f (2) − 2, ... tuần hoàn.

N6 IMO Shortlist 2015

Lời giải .
¨ Bước 1. Ta chứng minh hàm f là đơn ánh.
LATEX VÀ DỊCH THUẬT BỞI VIỆT

Thật vậy, giả sử tồn tại m, k ∈ Z+ mà f (m) = f (k). Từ điều kiện (1), ta suy ra
k−m f n (m) − m f n (k) − k
= − ∈ Z, ∀ n ∈ Z+
n n n
Điều này chỉ xảy ra khi k = m.
¨ Bước 2. Gọi {a1 , a2 , ..., ak } = Z+ \ {f (n) : n ∈ Z+ }.
Khi đó mọi số nguyên dương n đều biểu diễn được dưới dạng f j (ai )
Từ điều kiện (1), ta suy ra f (m) > m, ∀ m ∈ Z+ .
Nếu n = f (n0 ) thì ta có n0 < n. Lùi vô hạn như vậy, ta chứng minh được điều trên.
Như vậy, Ta có thể xếp các số nguyên dương vào một bảng
a1 f (a1 ) f 2 (a1 ) f 3 (a1 ) ...
a2 f (a2 ) f 2 (a2 ) f 3 (a2 ) ...
.. .. .. ..
. . . .
ak f (ak ) f 2 (ak ) f 3 (ak ) ...

¨ Bước 3. Ta chứng minh mỗi hàng của bảng trên là một cấp số cộng.

Không mất tính tổng quát, giả sử t hàng đầu là cấp số cộng. Ta gọi công sai của t hàng này là
T1 , T2 , ..., Tt . Ta viết T = lcm (T1 , T2 , ..., Tt ) và A = max{a1 , a2 , ..., at } nếu t > 0; và T = 1 và A = 0
nếu t = 0.
T
Với mọi số nguyên dương n > A thì đoạn ∆n = [n + 1, n + T ] chứa chính xác số thuộc hàng thứ
Ti
i.
Vì vậy, số phần tử thuộc k − t hàng còn lại không phụ thuộc vào n > A.
Hay nói cách khác trong k − t hàng này luôn chứa một số nằm trong ∆n .
Với số nguyên dương d bất kì, ta xét khoảng [A + 1, A + (d + 1)(k − t)T ]. Như vậy, trong k − t hàng
còn lại phải chứa ít nhất (d + 1)(k − t) số thuộc khoảng này. Theo nguyên lí Dirichlet, thì tồn tại
chỉ số x0 mà t + 1 6 x0 6 k sao cho hàng thứ x0 chứa ít nhất d + 1 số. Như vậy, ta có:

f d (ax0 ) 6 A + (d + 1)(k − t)T

Vì số x0 là hữu hạn nên tồn tại một chỉ số x > t + 1 để tập

X = d ∈ Z+ | f d (ax ) 6 A + (d + 1)(k − t)T




+ Do Math then Love Math 44 h Việt Nguyễn − Mathpiad


chứa vô hạn phần tử. Từ điều kiện (1), ta suy ra

f d (ax ) − ax
βd = ∈ Z+
d
A + (d + 1)(k − t)T Ad + 2d(k − t)T
Ta lại có βd không vượt quá 6 = A + 2(k − t)T
d d
Như vậy, tồn tại số Tx sao cho tập Y = {d ∈ X | βd = Tx } chứa vô hạn phần tử.
Chú ý rằng f d (ax ) = ax + d · Tx với mọi d ∈ Y.
Với j bất kì, ta chọn y ∈ Y sao cho y − j > |f j (ax ) − (ax + jTx )|.
Ta có cả hai số

¨ PHƯƠNG TRÌNH HÀM TRONG IMO SHORTLIST


f y (ax ) − f j (ax ) = f y−j f j (ax ) − f j (ax ) và f y (ax ) − (ax + jTx ) = (y − j)Tx

đều chia hết cho y − j nên y − j | f j (ax ) − (ax + jTx ).


Điều này chỉ xảy ra khi f j (ax ) = (ax + jTx ) hay hàng thứ x là một cấp số cộng.
Làm tiếp tục như vậy, ta nhận được tất cả các hàng của bảng đều là cấp số cộng.

¨ Bước 4. Ta kí hiệu công sai của hàng thứ i là Ti . Ta sẽ chứng minh rằng

f (n) − n = f (n + T ) − (n + T ), ∀ n ∈ Z+ với T = lcm (T1 , T2 , ..., Tk )

Giả sử f j (n) = n + j.Ti , ∀ j ∈ Z+ (n nằm trên hàng thứ i). Ta có

f (n + T ) − f (n) = f 1+T /Ti (n) − f (n) = n + T + Ti − (n + Ti ) = (n + T ) − n

Kết thúc chứng minh. â

 Bài 7 Với số nguyên dương k bất kì, ta gọi hàm f : Z+ → Z+ là k-nice nếu
gcd (f (n) + m, f (m) + n) 6 k với mọi n 6= m. Tìm tất cả các số nguyên dương k sao cho
tồn tại hàm k-nice.

N7 IMO Shortlist 2015

Lời giải . Ta thấy rằng nếu f là một hàm k-nice thì f cũng là một hàm (k + n)-nice với mọi số
nguyên dương n.

¨ Bước 1. Ta sẽ chứng minh không tồn tại hàm 1-nice.

Chứng minh. Đặt Gf (m, n) = gcd (f (n) + m, f (m) + n). Giả sử tồn tại hàm số f thỏa mãn
Gf (m, n) = 1 với mọi n 6= m.
Nếu tồn tại hai số chẵn khác nhau là m, n mà f (m) và f (n) đều là số chẵn thì Gf (m, n) > 2, vô lí.
Nên ta có thể chọn được m chẵn mà f (m) lẻ.
Tương tự ta có thể chọn được n lẻ mà f (n) chẵn. Khi đó, ta cũng có Gf (m, n) > 2, vô lí.

¨ Bây giờ, ta chứng minh tồn tại hàm 2-nice.



Chọn f (n) = 2g(n)+1 − n − 1 với g(1) = 1 và g(n + 1) = 2g(n)+1 ! với mọi n ∈ Z+ .
Với mỗi số nguyên m > n, ta có
®
A = f (m) + n = 2g(m)+1 − m + n − 1
B = f (n) + m = 2g(n)+1 − n + m − 1

Đầu tiên, ta có A + B = 2g(m)+1 + 2g(n)+1 − 2 không chia hết cho 4 nên 4 - gcd(A, B).

+ Do Math then Love Math 45 h Việt Nguyễn − Mathpiad


Giả sử tồn tại một số nguyên tố p > 2 mà p | gcd(A, B).
Vì g(k + 1) > g(k), ∀ k ∈ Z+ nên 2g(k+1)+1 > 2g(k)+1 + 1, ∀ k ∈ Z+ .
Suy ra 2g(m−1)+1 > 2g(n)+1 + (m − 1) − n = B.
Vì p | B nên p − 1 < B 6 2g(m−1)+1 . Suy ra p − 1 | (2g(m−1)+1 )! = g(m).
Suy ra 2g(m) ≡ 1 (mod p). Suy ra A + B = 2g(m)+1 + 2g(n)+1 − 2 ≡ 2g(n)+1 ≡ 0 (mod p).
Điều này là vô lí. Suy ra gcd(A, B) không có ước nguyên tố khác 2 hay gcd(A, B) 6 2.

Kết thúc chứng minh. Vậy các số k thỏa mãn đề bài là k > 2. â

 Bài 8 Cho hàm số f : Z+ → Z>1 thỏa mãn f (m + n) | f (m) + f (n), ∀ m, n ∈ Z+ . Chứng


minh rằng tồn tại một số nguyên C sao cho C | f (x), ∀ x ∈ Z+

N6 IMO Shortlist 2018

Lời giải . Với mọi số nguyên dương m, kí hiệu Sm = {n : m | f (n)}

! Nhận xét. Nếu tập S m có vô hạn phần tử thì Sm = {d, 2d, 3d, ...} = d · Z+ với d ∈ Z+
LATEX VÀ DỊCH THUẬT BỞI VIỆT

Chứng minh. Giả sử d = min Sm . Theo kí hiệu, ta có m | f (d).


Với n > d ∈ Sm , ta có m | f (n) | f (n − d) + f (d) nên m | f (n − d). Suy ra n − d ∈ Sm .
Giả sử n = kd + r. Nếu r > 0 thì tương tự, ta có r = n − kd ∈ Sm , trái với tính nhỏ nhất của d. Vậy
r = 0, d | n. Ta đã chứng minh xong nhận xét.

¨ Trường hợp 1. hàm f bị chặn trên.


Ta gọi một số nguyên tố p là xinh nếu tập Sp có vô hạn phần tử. Ngược lại gọi là xấu.
Do f bị chặn trên nên chỉ tồn tại hữu hạn số nguyên tố là ước của {f (n) : n ∈ Z+ }.
Ta có hữu hạn số xấu mà mỗi tập Si với i xấu thì có hữu hạn phần tử nên chỉ có hữu hạn số
nguyên dương n sao cho f (n) có ước nguyên tố là số xấu. Vậy với mọi n > N thì f (n) chỉ có
ước nguyên tố là các số xinh.
Gọi các số xinh là p1 , p2 , ..., pk . Theo nhận xét, ta có Spi = di · Z+ .
Xét số nguyên T = Ad1 d2 ...dk + 1 với A > N . Vậy f (T ) chia hết cho một số xinh.
Nói cách khác, tồn tại j ∈ {1, 2, ..., k} sao cho f (T ) ∈ Spj .
Khi đó dj | T = Ad1 d2 ...dk + 1 ⇒ dj | 1. Vậy dj = 1 hay Sdj = Z+ . Nên dj | f (x), ∀ x ∈ Z+

¨ Trường hợp 2. hàm f không bị chặn trên.


Ta chứng minh f (1) là ước của mọi f (n).
Đặt a = f (1). Vì 1 ∈ Sa nên ta sẽ chứng minh tập Sa có vô hạn phần tử.
Gọi một số nguyên dương p là mạnh mẽ nếu f (p) > max{f (1), ..., f (p − 1)}.
Vì hàm f không bị chặn trên nên tồn tại vô hạn số mạnh mẽ.
Gọi dãy các số mạnh mẽ là q1 < q2 < ... < qk < ... và đặt hk = f (qk )
Với qk mạnh mẽ và h < qk bất kì, ta có

f (qk ) | f (h) + f (qk − h) < 2f (qk )

Nên f (qk ) = f (h) + f (qk − h) = hk .


Do dãy hk là vô hạn nên tồn tại vô số số trong dãy này đồng dư với nhau modulo a.
Gọi các số này là hk0 ≡ hk1 ≡ ... (mod a) với k0 < k1 < .... Khi đó

f (pki − pk0 ) = f (pki ) − f (pk0 ) = hki − hk0 ≡ 0 (mod a) với mọi i > 0

Suy ra pki − pk0 ∈ Sa với mọi i > 0. Vậy Sa có vô hạn phần tử.

Kết thúc chứng minh. â

+ Do Math then Love Math 46 h Việt Nguyễn − Mathpiad


 Bài 9 Tìm tất cả các hàm số f : Z+ → Z+ thỏa mãn tồn tại số nguyên C sao cho

a + f (b) | a2 + bf (a), với mọi a + b > C

N4 IMO Shortlist 2019

Lời giải . Với a = 1, b đủ lớn, ta có được 1 + f (b) | 1 + bf (1) nên f (b) 6 bf (1).
Xét b bất kì. Chọn n ∈ Z đủ lớn để a = nb − f (b) > C, ta được

b | nb | (nb − f (b))2 + bf (nb − f (b)) ⇒ b | f (b)2

¨ PHƯƠNG TRÌNH HÀM TRONG IMO SHORTLIST


Vậy ta suy ra p | f (p) với p là số nguyên tố. Đặt f (p) = k(p) · p
Do f (p) 6 f (1)p nên k(p) 6 f (1). Vậy trong tập hợp {k(p) : p ∈ P} có một số k xuất hiện vô hạn.
Ta chứng minh f (x) = kx với mọi x nguyên dương.
Với k(p) = k và p > max{C, a}, ta có

a + kp | a2 + pf (a) ⇒ a + kp | a2 + pf (a) − a(a + kp) = p(f (a) − ka)

Do gcd(a + kp, p) = gcd(a, p) = 1 nên a + kp | f (a) − ka


Chọn p đủ lớn, ta được a + kp > |f (a) − ka| nên f (a) − ka = 0 ⇔ f (a) = ka.
Thử lại thỏa mãn. Vậy hàm số thỏa mãn đề bài là f (x) = kx, ∀ x ∈ Z+ â

 Bài 10 Tìm tất cả các hàm số f : Z+ → N thỏa mãn

i) tồn tại t để f (t) 6= 0

ii) f (xy) = f (x) + f (y), ∀ x, y > 0

iii) Tồn tại vô hạn số nguyên dương n để f (k) = f (n − k) với mọi k < n

N5 IMO Shortlist 2020

Lời giải . Từ điều kiện ii), ta có f (1) = f (1) + f (1) nên f (1) = 0
Nếu n có phân tích tiêu chuẩn n = pα1 1 pα2 2 ...pαk 2 thì

f (n) = α1 f (p1 ) + α2 f (p2 ) + ... + αk f (pk ) (*)

Ta gọi n là một số đáng yêu nếu f (k) = f (n − k) với mọi k < n.

! Nhận xét. Nếu n là một số đáng yêu thì với d | n thì d cũng là một số đáng yêu.
Chứng minh. Đặt n = dm, ta có

f (k) = f (mk) − f (m) = f (n − mk) − f (m) = f (m(d − k)) − f (m) = f (d − k) với 0 < k < d

Từ (∗), ta suy ra nếu f (n) = 0 thì f (d) = 0 với mọi d | n. Nên theo điều kiện i), ta suy ra tồn tại số
nguyên tố p mà f (p) 6= 0. Giả sử p là số nhỏ nhất mà f (p) 6= 0.
Khi đó, ta có f (r) = 0 với mọi r < p.
Giả sử n là một số đáng yêu bất kì lớn hơn p. Đặt n = pk + r. Nếu r > 0 thì

f (p) 6 f (pk) = f (n − pk) = f (r) = 0, mâu thuẫn.

Vậy ta suy ra nếu n là một số đáng yêu lớn hơn p thì p | n.


Nếu n có ước nguyên tố q lớn hơn p thì theo nhận xét, ta được q cũng là một số đáng yêu. Khi đó

+ Do Math then Love Math 47 h Việt Nguyễn − Mathpiad


p | q, mâu thuẫn. Vậy n không có ước nguyên tố lớn hơn q. Vậy n có dạng r · pk với 0 < r < p.
Vì n là số đáng yêu và pk | n nên pk cũng là số đáng yêu.
Theo điều kiện iii), ta thấy rằng k → ∞ nên tất cả các lũy thừa của p đều là số đáng yêu.
Xét số nguyên tố q 6= p. Ta có

f (q) 6 f (pq−1 − 1) = f (1) = 0 ⇒ f (q) = 0

Vậy ta suy ra f (n) = f (p).vp (n) = c.vp (n) với c 6= 0. Thử lại thỏa mãn.
Vậy hàm số thỏa mãn đề bài là f (n) = c.vp (n) với c 6= 0 và p ∈ P bất kì. â
LATEX VÀ DỊCH THUẬT BỞI VIỆT

Hết!

+ Do Math then Love Math 48 h Việt Nguyễn − Mathpiad


CHINH PHỤC OLYMPIC TOÁN
NGUYỄN MINH TUẤN
DOÃN QUANG TIẾN
TÔN NGỌC MINH QUÂN

PHƯƠNG TRÌNH HÀM


TRÊN TẬP RỜI RẠC

TẠP CHÍ VÀ TƯ LIỆU TOÁN HỌC


Omaths

Littited Chuyên đề
Bồi dưỡng
Edition Học sinh giỏi

Phương trình hàm


Trên tập rời rạc
Chinh phục Olympic toán

TẠP CHÍ VÀ TƯ LIỆU TOÁN HỌC


TẠP CHÍ VÀ TƯ LIỆU TOÁN HỌC

Copyright © 2019 by Tap chi va tu lieu toan hoc.

All rights reserved. No part of this book may be reproduced or distributed in any form
or by anymeans, or stored in data base or a retrieval system, without the prior written
the permission of the author.
LỜI GIỚI THIỆU
Những bài toán phþơng trình hàm ngày nay đã trở nên rất phổ biến đối với các bän
học sinh yêu Toán vì chúng đã xuất hiện thþờng xuyên trong các đề thi học sinh giỏi
TẠP CHÍ VÀ TƯ LIỆU TOÁN HỌC

các cấp cüng nhþ kì thi chọn đội tuyển quốc gia, VMO hay các kì thi khu vực và
quốc tế mà ta đþợc biết đến. Đặc biệt, trong các lớp däng phþơng trình hàm, thì
däng phþơng trình hàm trên các tập rời räc là một mâng đþợc ít các học sinh chú ý
tới bởi độ khó và chþa đþợc tiếp xúc nhiều đồng thời ngoài việc sử dýng các kï thuật
xử lý phþơng trình hàm cơ bân chúng ta còn phâi sử dýng các tính chất số học rất
đặc sắc cûa tập rời räc nhþ là: tính chia hết, tính chất cûa số nguyên tố, cûa số
chính phþơng,... Trong ebook này chúng tôi sẽ mang tới cho bän đọc tuyển tập các
bài toán phþơng trình hàm trên tập rời räc và một số bài toán phþơng trình hàm
khác hay và khó với những lời giâi vô cùng đặc sắc nhằm giúp bän đọc có thể có
nhiều cách nhìn khác về mâng toán này đồng thời cüng nhþ chuẩn bð cho các kì
học sinh giỏi, olympic.

Mình xin gửi lời câm ơn tới


HỌCHỌC

1. Thầy Huỳnh Kim Linh – THPT chuyên Lê Quý Đôn – Khánh Hòa – Đã góp ý
giúp bọn mình về phần nội dung.
2. Bän La Thð Đông Phþơng – Đäi học Hoa Sen – Đã giúp bọn mình chînh sửa bân
thâo đề hoàn thiện hơn.

Một lần nữa gửi lời câm ơn các bän, các thầy cô đã ûng hộ và theo dõi fanpage suốt
thời gian qua. Hy vọng ebook này sẽ giúp ích đþợc cho mọi ngþời. Thank you!

Nhóm tác giả


Nguyễn Minh Tuấn
Doãn Quang Tiến
Tôn Ngọc Minh Quân
PHƯƠNG TRÌNH HÀM TRÊN TẬP RỜI RẠC
TẠP CHÍ VÀ TƯ LIỆU TOÁN HỌC

CHINH PHỤC OLYMPIC TOÁN


Phương trình hàm trên tập rời rạc

Chuyên đề
PHƯƠNG TRÌNH HÀM TRÊN TẬP RỜI RẠC
Tạp chí và tư liệu toán học

Để giải quyết các bài toán phương trình hàm trên tập rời rạc mà có thể giải bằng các tính
chất số học thì nên lưu ý đến một số dấu hiệu sau:
 Nếu xuất hiện các biểu thức tuyến tính chứa lũy thừa, có thể nghĩ đến các bài toán
liên quan đến cấp của phần tử, các phương trình đặc biệt như phương trình Pell
hay phương trình Pythagore,<hay đưa về việc xử lý các phương trình vô định
nghiệm nguyên.
 Nếu hàm số đã cho là hàm nhân tính, ta thường hay xét đến giá trị hàm số tại các
CHINH PHỤC OLYMPIC TOÁN

điểm là số nguyên tố hoặc dãy vô hạn các số nguyên tố.


 Sử dụng các đẳng thức và bất đẳng thức số học.
 Và đặc biệt nhất, trong một số bài toán, hệ cơ số đếm có thể dùng để xây dựng
nhiều dãy số có tính chất số học thú vị. Trong hệ cơ số 10 chúng ta có thể rất khó
nhận ra quy luật của dãy, nhưng nếu chọn được hệ cơ số phù hợp thì bài toán có
thể giải quyết đơn giản hơn rất nhiều.
Nếu g  2, g  , với g là cơ số đếm, thì mọi số nguyên dương M đều biểu diễn
duy nhất dưới dạng:
M  a1 a2 ...an g  a1 g n1  a2 g n 2  ...  an1 g  an với 1  a1  g  1;0  ai  g  1, i  2, n.
Cơ số đếm mà hay được sử dụng trong các bài toán phương trình hàm trên tập rời rạc là 2
và 3.
Sau đây, chúng tôi sẽ đề cập đến các bài toán phương trình hàm mà sử dụng các tính chất
cũng như các phương pháp trong số học để giải, nhằm giúp bạn đọc hiểu rõ hơn và có một
cái nhìn mới mẻ hơn về các phương pháp khác để giải phương trình hàm, bên cạnh đó
chúng tôi cũng sẽ giới thiệu cho bạn đọc các bài toán phương trình hàm và khó trong tài
liệu này. Nào cùng bắt đầu nhé!

Chinh phục olympic toán| 1


Bồi dưỡng học sinh giỏi

I. ĐỀ BÀI
Câu 1. Tìm tất cả các hàm số f :  thỏa mãn điều kiện sau:
3 f  n   2 f  f  n    n , n 
Câu 2. Tìm tất cả các hàm số f :  thỏa mãn điều kiện sau
 m  n  f  m2  n2   mf  n   nf  m  , m, n   1
Câu 3. Cho hàm số f : *
 *
thỏa mãn điều kiện sau:
f  n  1   f  f  n   , n  *

Chứng minh rằng f  n   n , n  *


.
Câu 4. Tìm tất cả các hàm số f : *
 *
thỏa mãn điều kiện sau:
x 2  f  y  f 2  x   y , x , y  *

Câu 5. Tìm tất cả các hàm số f : *
 *
thỏa mãn điều kiện sau:

TẠP CHÍ VÀ TƯ LIỆU TOÁN HỌC


f 2  m   f  n   m2  n  , m, n 
2 *
*
Câu 6. Tìm tất cả các hàm f :  thỏa mãn tồn tại số k  và số nguyên tố p sao cho
với mọi n  k , f  n  p   f n  và nếu m n thì f  m  1  f  n   1.
Câu 7. Cho p là số nguyên tố lẻ. Tìm tất cả các hàm f :  thỏa mãn đồng thời các
điều kiện:
i) f  m   f  n  với m  n  mod p 
ii) f  mn   f  m  f  n  , m , n 
Câu 8. Tìm số nguyên không âm n nhỏ nhất sao cho tồn tại hàm số f :   0,   khác
hằng số thỏa mãn đồng thời các điều kiện:
i) f  xy   f  x  f  y  , x , y 
ii) 2 f  x 2  y 2   f  x   f  y   0, 1,..., n , x , y 
Với số n tìm được, hãy tìm tất cả các hàm số thỏa mãn.
Câu 9. Giả sử hàm số f : *
 thỏa mãn các điều kiện sau:
  n1
1  f  if n  2 m  1
  2 
f  1   1 và f  n   
1  n
f  if n  2 m
 2
Tìm các giá trị của n sao cho f  n   2019.
Câu 10. Tìm tất cả các hàm số f : *
 *
thỏa mãn các điều kiện sau:

2 | Tạp chí và tư liệu toán học


Phương trình hàm trên tập rời rạc

f  1  1, f  3   3

f  2n   f  n 
 với mọi số nguyên dương n .
f  4n  1   2 f  2 n  1   f  n 
f  4n  3   3 f  2 n  1   2 f  n 

 
Câu 11. Cho hàm số f :  thỏa mãn đồng thời các điều kiện:
 f  n  là ước của n 2018 với mọi n
 f  a  . f  b   f  c  với mọi a , b , c  
và a 2  b 2  c 2
a) Chứng minh rằng nếu n lẻ hoặc n 4 thì f  n   1
b) Gọi A là tập hợp giá trị có thể có của f  2   f  2018  . Tính A
Câu 12. Có tồn tại hàm số f : S  S thỏa mãn điều kiện
f  a  f  b   f  a 2 b 2  , a , b  S , a  b không, trong đó S  *
\1 ?
Câu 13. Tìm tất cả các hàm số f : *
 *
CHINH PHỤC OLYMPIC TOÁN

thỏa mãn điều kiện

 n  1  f  n  f  f  n    n 2  n , n 
2 *
.
Câu 14. Tìm tất cả hàm số f :  thỏa mãn đồng thời hai điều kiện sau:
i) x  f  y  f  x    y  f  x  f  y   với mọi x , y  ;
 f  x   f  y  
ii) Tập hợp I   , x , y  , x  y  là một khoảng
 xy 
thỏa mãn f 2  m   f  n   m2  n  , m, n 
2
Câu 15. Tìm các hàm số f : *
 * *

Câu 16. Cho hàm f  x , y  thỏa mãn các điều kiện:


 f  0, y   y  1; f  x  1, 0   f  x , 1 
 f  x  1, y  1   f  x , f  x  1, y  
Với mọi số nguyên không âm x , y . Tìm f  4, 1981  ?
 
Câu 17. Cho hàm f :  thỏa mãn các điều kiện sau:
i) f  n  1   f  n  ; n  

ii) f  f  n    3n , n  Z  .
Hãy tính f  2003  .
Câu 18. Cho f  n  là hàm số xác định với mọi n  *
và lấy giá tị không âm thỏa mãn tính
chất:
 n , m   * : f  m  n   f  m   f  n  lấy giá trị 0 hoặc 1
 f  2   0 và f  3   0 .
 f  9999   3333 .

Chinh phục olympic toán| 3


Bồi dưỡng học sinh giỏi

Tính f  2000  .
Câu 19. Cho f , g là các hàm xác định trên thỏa mãn điều kiện
f  x  y   f  x  y   2 f  x  .g  y  , x , y 

Chứng minh rằng nếu f  x   0 và f  x   1, x  thì g  y 0   a  1

Câu 20. Cho hàm số f :  thỏa 2 điều kiện


i) f  x   1  x ; x 
ii) f  x  y   f  x  . f  y  ; x , y 
Chứng minh rằng không thể tồn tại hai số a ; b  mà f  a  . f  b   0
2003
Câu 21. Cho f  x , y   cos 2  x  y   a cos  x  y    với a ,   .
2
Chứng minh rằng min  f  x , y     max f  x , y    2003.
2 2

TẠP CHÍ VÀ TƯ LIỆU TOÁN HỌC


x2  1
Câu 22. Cho hàm số f  x   , x  0.
2x
Giả sử f 0  x   x và f n  x   f  f n1  x   n   * , x  0 .
fn  x  1
Chứng minh n  , x  1, 0, 1  1
f n1  x   x1
2n

f 
 x1
Câu 23. Cho hàm số f : *
 *
 *
là hàm số thỏa mãn đồng thời các điều kiện sau:
i) f  1, 1   2
ii) f  m  1, n   f  m , n   m , m , n  *

iii) f  m, n  1   f  m, n   n , m , n  *

Tìm tất cả các cặp số  p , q  sao cho f  p , q   2019.


Câu 24. Tìm tất cả các hàm số f :  thỏa mãn các điều kiện sau:
i) 0  f  x   x 2 , n 
ii) f  x   f  y  chia hết cho x  y với mọi x , y  , x  y
Câu 25. Tìm tất cả các hàm số f : *
  *
 mà tập *
  x  x  0 thỏa mãn:
f  xy 
f  x   f  y   2 xyf  xy   , x , y  *
 1
f x  y


Câu 26. Cho hàm f :  là một hàm số thỏa mãn với mọi n  1 thì có một số nguyên
n
tố p là ước của n sao cho: f  n   f  p   f    1 và
 p
f  32018   f  52019   f  7 2020   2017.

4 | Tạp chí và tư liệu toán học


Phương trình hàm trên tập rời rạc

Hãy tính giá trị của biểu thức G  f  2018 2018   f  2019 2019   f  2020 2020 
Câu 27. Tìm tất cả các hàm số f : *
 *
thỏa mãn:
2 f 3  m2  n2   f 2  m  f  n   f  m  f 2  n  , m , n  *


Câu 28. Giả sử f :  là hàm liên tục và giảm sao cho với mọi x , y  ta có
f  x  y  f  f  x   f  y   f  y  f  x 
Chứng minh rằng f  f  x    x .
Câu 29. Cho song ánh f :  . Chứng minh rằng tồn tại vô số bộ  a , b , c  với a , b , c 
thỏa mãn a  b  c và 2 f  b   f  a   f  c  .
Câu 30. Có bao nhiêu hàm f : * * thoả mãn đồng thời các điều kiện sau
a) f  1   1

b) f  n  f  n  2   9  f  n  1    1997, n 
2
*.
CHINH PHỤC OLYMPIC TOÁN

Câu 31. Tìm tất cả các hàm số f : *


 *
sao cho.
a) f  2   2
b) f  m.n   f  m  . f  n  với mọi m, n  *
, UCLN  m, n   1
c) f  m   f  n  m , n  *
,m  n.
Câu 32. Tìm tất cả các hàm số f :  thỏa mãn
f  m  n   f  mn   f  m  f  n   1, m , n 
Câu 33. Tìm tất cả các hàm số f :  thỏa mãn f  0   2 và

 
f x  f  x  2 y   f  2 x   f  2 y  , x , y   1
Câu 34. Tìm tất cả hàm số f :  sao cho f  f  n    f  n   2n  3, n   1
Câu 35. Chứng minh rằng tồn tại duy nhất hàm số f : *  * thỏa mãn

f  m  f  n    n  f  m  b  , m , n  * b   i
Câu 36. Hãy xác định tất cả hàm số f : *
 *
thỏa mãn đẳng thức:
f  n   f  n  1  f  n  2  . f  n  3   a  1
Với a là số tự nhiên thỏa mãn a  1 là số nguyên tố.
Câu 37. Tìm tất cả các hàm số f : *
 *
thỏa mãn f t  n    a  1  . f  n   an   t  a  k với

 
f t  n   f f  ... f  n   với a , t là số tự nhiên tùy ý thỏa mãn k  2t  1   a  1 .
t

Câu 38. Cho hàm số f :  thỏa mãn:

 f  2 n  1   f  2 n   1   f  2 n  1   f  2 n   1   3  1  2 f  n  
 ,n
 f  2 n   f  n 

Chinh phục olympic toán| 5


Bồi dưỡng học sinh giỏi

Tìm n sao cho f  n   2009 .


Câu 39. Tìm tất cả các hàm số f :  thoả mãn:
1 1 1
f  xy   f  xz   f  x  f  yz   , x , y , z  .
3 3 9
Câu 40. Cho n   n  2  và hàm số f :  sao cho:
f  x n  y   x n1 f  x  f  f  y   ; x , y  *
a) Giả sử rằng f  2002   0. Tính f  2002  .
b) Tìm hàm số f .
Câu 41. Tìm tất cả các hàm số f :  thỏa mãn
f  x  y 2  z3   f  x   f 2  y   f 3  z  x , y , z 
Câu 42. Cho hàm số f : *
 *
thỏa mãn đồng thời hai điều kiện:
a) f  ab   f  a, b f   a, b   với mọi a, b  *
, a  b ; trong đó  a , b  ,  a , b  lần lượt là bội

TẠP CHÍ VÀ TƯ LIỆU TOÁN HỌC


chung nhỏ nhất, ước chung lớn nhất của hai số nguyên dương a, b ;
b) f  p  q  r   f  p   f  q   f  r  với mọi số nguyên tố p , q , r .
Tính giá trị của f  2013  ? Kí hiệu *
là tập hợp tất cả các số nguyên dương.
Câu 43. Đặt F  f :  0, 1   0, 1 và n  2. Tìm giá trị nhỏ nhất của c thỏa mãn điều kiện

 f  x  dx  c  f  x  dx
1 1
n
0 0

Với f  F và f là hàm liên tục.


Câu 44. Tìm tất cả các hàm f :  1, 1  liên tục, thỏa mãn:
 2x 
f x  f  2 
, x   1, 1
 1 x 
Câu 45. Có thể tồn tại hay không một hàm số f :  , liên tục trên và thỏa mãn điều
kiện: Với mọi số thực x , ta có f  x  là số hữu tỉ khi và chỉ khi f  x  1  là số vô tỉ.
Câu 46. Tìm tất cả các hàm số f :  thỏa mãn điều kiện f  x   f  t   f  y   f  z  với
mọi số hữu tỉ x  y  z  t và x , y , z , t theo thứ tự lập thành cấp số cộng.
Câu 47. Giả sử r , s  là hai số cho trước. Tìm tất cả các hàm số f :  thỏa mãn điều
kiện f  x  f  y    f  x  r   y  s , x , y  ?
Câu 48. Tìm tất cả các hàm số f :  sao cho với tất cả các số nguyên a , b , c thỏa mãn
a  b  c  0 , đẳng thức sau là đúng:
 f  a   f  b    f c   2 f  a  f  b   2 f  b  f c   2 f c  f  a 
2 2 2

 
Câu 49. Tìm tất cả các hàm f , g :  có đạo hàm trên thỏa mãn
g x f x
f ' x    ; g'  x    x  

x x

6 | Tạp chí và tư liệu toán học


Phương trình hàm trên tập rời rạc

Câu 50. Tìm tất cả các hàm f : *


  có đạo hàm trên *
 thỏa mãn
f  xy   f  x   f  y  x , y  *
  1
Câu 51. Tìm tất cả các hàm f :  thỏa mãn
f  f  n    n  b n   1
trong đó b là số nguyên dương chẵn.
 
Câu 52. Tìm tất cả các hàm f :  thỏa mãn:
i) f  xf  y    yf  x  x , y  
 1
ii) lim f  x   0
x 
 
Câu 53. Chứng minh rằng tồn tại song ánh f :  sao cho
f  3mn  m  n   4 f  m  f  n   f  m   f  n  m , n  

Câu 54. Tìm tất cả các hàm f :  thỏa:

 
3 f f  f  n    2 f  f  n    f  n   6n , n 
CHINH PHỤC OLYMPIC TOÁN

Câu 55. Tìm tất cả các hàm số f :  0;     0;   thỏa mãn điều kiện:
f  f  x    yf  yf  x   x , y   0;    1 
Câu 56. Chứng minh rằng tồn tại duy nhất một hàm số f xác định trên tập các số thực
dương, nhận giá trị thực dương và thỏa mãn f  f  x    6x  f  x  .
Câu 57. Hàm số f :  thỏa mãn đồng thời các điều kiện sau:
 i  : f  f  n    n, n   1
 ii  : f  f  n  2   2   n, n  2
 iii  : f  0   1  3
Tìm giá trị f  1995  , f  2007 
Câu 58. Tìm f :  0, 1   thỏa mãn f  xyz   xf  x   yf  y   zf  z  x , y , z   0, 1 
Câu 59. Tìm tất cả các hàm f xác định trên và thỏa mãn đồng thời các điều kiện sau:

2 f  n  f  k  n   2 f  k  n   3 f  n  f  k  , k  n


 f  1  1

Câu 60. Tìm tất cả các hàm số f : *
 *
thỏa mãn đồng thời hai điều kiện sau:
 f  f  n    n  2 k ,  n  *
,k *


 f  n  1   f  n  ,  n 
*

Câu 61. Tìm tất cả các hàm số f :  thỏa mãn đồng thời hai điều kiện sau:
 f  2013   2016

 f  f  n    n  4, n 
Câu 62. Tìm tất cả các hàm số f :  *
thỏa mãn điều kiện sau:

Chinh phục olympic toán| 7


Bồi dưỡng học sinh giỏi

f  n   f  n  1   f  n  1  . f  n  3  , n   1
 
Câu 63. Tìm tất cả các hàm f :  thỏa mãn:
f  x  f  y   f  x  y   f  y 
Câu 64. Tìm số nguyên dương m nhỏ nhất sao cho tồn tại hàm số f : *
 \1; 0; 1
thỏa mãn đồng thời các điều kiện sau
i) f  m   f  2015  , f  m  1   f  2016  ;
f n  1
ii) f  n  m   , n  1, 2,....
f n  1
Câu 65. Xác định hàm số f  x  liên tục 
 
thỏa mãn đồng thời các điều kiện:
 f  2 x   2 f  x  với mọi x  
,  1

   
f f 3  x  e f  x   1  x 2  e x  1  f  x  với mọi x  
, 2

TẠP CHÍ VÀ TƯ LIỆU TOÁN HỌC


 f  e  1   e  1 f  1 ,  3 
 f  k  là số nguyên dương với mọi số nguyên dương k ,  4 
Câu 66. Tìm tất cả các hàm f :  thỏa mãn đồng thời hai điều kiện sau:
 Với mọi cặp a, b nguyên dương không nguyên tố cùng nhau, có f  a  . f  b   f  ab 
 Với mọi bộ a, b nguyên dương tồn tại một tam giác không suy biến có độ dài ba
cạnh là f  a  , f  b  và f  a  b  1  .
Câu 67. Tìm các hàm số f :  1;    thoả mãn điều kiện:
f  x   f  y    y  x  f  xy  với mọi x , y  1  1 
Câu 68. Tìm tất cả các hàm f : *
 *
thỏa mãn đẳng thức:
f  f 2  m   2 f 2  n    m2  2n2 , với mọi m , n  *
.
Câu 69. Tìm tất cả các số nguyên không âm n sao cho tồn tại một hàm f :   0;   khác
hằng thỏa mãn đồng thời 2 điều kiện sau
i) f  xy   f  x  f  y  , x , y 


ii) 2 f  x 2  y 2   f  x   f  y  x , y    0; 1; 2;...; n.
Câu 70. Tìm tất cả các hàm số f : *  * thoả mãn điều kiện:


2 f  m2  n 2  
3
 f 2  m  . f  n   f 2  n  . f  m  , m , n  *

Câu 71. Tìm tất cả các hàm số f :  thoả mãn điều kiện:
 f 0  c

3 f n  1
 f  n  1  , n  *  1
3  f  n

8 | Tạp chí và tư liệu toán học


Phương trình hàm trên tập rời rạc

Câu 72. Tìm tất cả các hàm số f :  thỏa:


f  2 a   2 f  b   f  f  a  b   a , b 
Câu 73. Có tồn tại hay không hàm số f :  sao cho
f  m  f  n    f  m   n , m , n   1
Câu 74. Cho hàm số f :  là hàm số thỏa mãn các điều kiện sau:
i) f  mn   f  m  f  n  , m , n 
ii)  m  n  là ước của f  m   f  n  với mọi m , n 
Chứng minh rằng tồn tại một số tự nhiên lẻ k sao cho f  n   n k , n  .
Câu 75. Tìm tất cả các hàm số f : *
 *
thỏa mãn đồng thời các điều kiện sau:
i) f  0   0, f  1   1
ii) f  0   f  1   f  2   ...
iii) f  x 2  y 2   f 2  x   f 2  y  , x , y 
CHINH PHỤC OLYMPIC TOÁN

Câu 76. Tìm tất cả các hàm số f :  thỏa mãm các điều kiện sau:
i) Nếu a b thì f  a   f  b 
ii) f  ab   f  a 2  b 2   f  a   f  b  , a , b 
Câu 77. Tồn tại hay không hàm số f : 1, 2,..., n  thỏa mãn điều kiện:
i) f là hàm đơn ánh
ii) f  ab   f  a   f  b  với mọi a , b  1, 2,..., n và ab  n
Câu 78. Giả sử Josephus có  n  1  người bạn, n người này đúng thành một vòng tròn
đánh số từ 1 đến n theo chiều kim đồng hồ, tự sát theo nguyên tắc, người thứ nhất cầm
dao đếm 1 rồi tự sát, người thứ hai đếm 2 rồi tự sát,<Quá trình dừng lại khi còn một
người. Gọi f  n  là hàm số biểu thị vị trí cùa người sống sót đó. Câu hỏi đặt ra là, hãy tính
f n ?
Câu 79. Cho hai hàm số f , g : *
 *
là hai hàm số thỏa mãn đồng thời các điều kiện:
i) g là hàm số toàn ánh
ii) 2 f 2  n   n2  g 2  n  , n  

Nếu f  n   n  2019 n , n  
thì f có vô số điểm bất động.
Câu 80. Tìm tất cả các hàm số g : *
 *
thỏa mãn điều kiện sau:
g  g  n   n   g  n  1   3  n  g  n  , n 

Câu 81. Cho ba số thực a , b , c không âm, phân biệt sao cho tồn tại hàm f , g :  thỏa
x
mãn af  xy   bf    cf  x   g  y  với mọi số thực dương x  y .
y

Chinh phục olympic toán| 9


Bồi dưỡng học sinh giỏi


Chứng minh rằng tồn tại hàm h :  sao cho:
x
f  xy   f    2 f  x   h  y  , x  y  0
y
Câu 82. Tìm tất cả hàm số f :  thỏa mãn:
n ! f  m  ! f  n  ! f  m ! , m, n 
Câu 83. Tồn tại hay không hàm số f : *
 *
thỏa mãn điều kiện sau:
f  f  n    3n  2 f  n  , n  *

Câu 84. Tìm tất cả các hàm số tăng thực sự f : *


 *
thỏa mãn điều kiện sau:
f  n  f  n    2 f  n  , n  *

Câu 85. Tìm tất cả các toàn ánh f :  sao cho với mọi m , n  thỏa mãn:
f m f n  m n

TẠP CHÍ VÀ TƯ LIỆU TOÁN HỌC

10 | Tạp chí và tư liệu toán học


Phương trình hàm trên tập rời rạc

II. LỜI GIẢI.

Câu 1. Tìm tất cả các hàm số f :  thỏa mãn điều kiện sau:
3 f  n   2 f  f  n    n , n 
Lời giải
Giả sử f là hàm số thỏa mãn điều kiện bài toán.
Đặt g  n   f  n   n , n  .
Khi đó, thì ta được 2 g  f  n    2  f  f  n    f  n    f  n   n  g  n  , n   1
Áp dụng liên tiếp  1  ta được

    
g  n   2 g  f  n    2 2 g f  f  n    ...  2 m g f f  ... f  n  ... , trong đó có m dấu f .

Như vậy thì g  n  chia hết cho 2 m , m   g  n   0, n  hay f  n   n , n 


Thử lại thì thấy hàm số f  n   n , n 
CHINH PHỤC OLYMPIC TOÁN

thỏa mãn yêu cầu đề bài.


Vậy tất cả các hàm số thỏa mãn đề bài là: f  n   n , n  .
Nhận xét. Việc đặt hàm phụ g  n   f  n   n , n  giúp ta đưa phương trình hàm ban
đầu về dạng mới đẹp hơn. Và khi đó ta phát hiện ra thêm được các tính chất của hàm mới
g  n  để từ đó ta áp dụng liên tiếp các tính chất ấy và kết hợp với các tính chất số học chia
hết để suy ra được hàm thỏa mãn yêu cầu đề bài.

Câu 2. Tìm tất cả các hàm số f :  thỏa mãn điều kiện sau
 m  n  f  m2  n2   mf  n   nf  m  , m, n   1
Lời giải
Giả sử f là hàm số thỏa mãn điều kiện bài toán.
Kí hiệu P  u , v  là phép thế u, v vào  1  thì ta được:
P  0, n   nf  n2   nf  0  , n 

Do đó f  n2   f  0  , n  .
Đặt g  n   f  n   f  0  , n  .
Khi đó, ta thay vô  1  ta được  m  n  g  m2  n2   mg  n   ng  m  , m , n  2
Hơn nữa, ta còn có g  0   0 và g  n2   0, n 
Kí hiệu Q  u , v  là phép thế m  u, n  v vào  2  thì
Q  n , n   2ng  2n2   2ng  n  , n 

Do đó ta được g  2n2   g  n  , n  và

Chinh phục olympic toán| 11


Bồi dưỡng học sinh giỏi

Q  2n2 , n2   3n2 g  5n 4   n2 g  n  , n 

Từ đó suy ra g  n   3 g  5n 4  , n 
Từ đây ta áp dụng liên tục các tính chất trên, thì ta đó ta suy ra
g  n  3k , n  , k  *

Suy ra: g  n   0, n  hay f  n   f  0   const , n  .


Thử lại thì ta thấy hàm này thỏa mãn yêu cầu bài toán.
Vậy tất cả các hàm số thỏa mãn yêu cầu bài toán là f  n   f  0   const, n  .
Nhận xét. Cũng tương tự như bài toán 1 ta nhìn phương trình hàm ban đầu dưới một hàm
phụ khác, bằng các phép thế cơ bản ta phát hiện ra được một số tính chất sơ khai ban đầu.
Và bằng phép đặt g  n   f  n   n, n  ta được một phương trình hàm có dạng y chang
phương trình hàm ban đầu, nhưng ta lại được thêm các điều kiện ràng buộc là g  0   0 và
g  n2   0, n 

TẠP CHÍ VÀ TƯ LIỆU TOÁN HỌC


nên từ đó ta đã được thêm các ràng buộc, thuận lợi cho việc giải phương
trình. Phép đặt này rất hay, nó vừa bảo toàn phương trình hàm có dạng y chang ban đầu
và kèm theo là các điều kiện rang buộc mà phương trình hàm ban đầu không có. Từ đấy,
tương tự bài toán 1, ta phát hiện các tính chất của hàm g  n  và sử dụng liên tục chúng và
kết hợp cùng với các tính chất chia hết để suy ra hàm số cần tìm.

Câu 3. Cho hàm số f : *


 *
thỏa mãn điều kiện sau:
f  n  1   f  f  n   , n  *

Chứng minh rằng f  n   n , n  *


.
IMO 1977
Lời giải
Giả sử f là hàm số thỏa mãn điều kiện bài toán.
Đặt d  min  f  n  , n  *
 , theo nguyên lý cực hạn thì d tồn tại và duy nhất.
Gọi m  *
sao cho: f  m   d.
Nếu m  1 thì d  f  m   f  f  m  1   , mâu thuẫn.
Do đó f  n  đạt giá trị nhỏ nhất duy nhất tại n  1
Lập luận tương tự thì ta có f  2   min  f  n  , n  *
, n  2
Và lập luận lại quá trình tương tự như trên ta được:
f  1   f  2   f  3  ...  f  n   ...

Ta có f  1   1 nên f  n   n , n  *

Nếu tồn tại n0  *


mà f  n0   n0 thì f  n0   n0  1.

12 | Tạp chí và tư liệu toán học


Phương trình hàm trên tập rời rạc

Suy ra f  f  n0    f  n0  1  , mâu thuẫn


Do đó, f  n   n , n  *
, thử lại thấy thỏa mãn yêu cầu bài toán.
Vậy tất cả các hàm số thỏa mãn yêu cầu bài toán là: f  n   n , n  *
.
Nhận xét. Đây là một bài toán phương trình hàm trong kì thi Toán Quốc Tế - IMO năm
1977, một bài toán phương trình hàm với điều kiện rang buộc là ở dạng bất đẳng thức, rất
lạ và mới. Làm ta nãy ra ý tưởng sử dụng nguyên lý cực hạn để đánh giá để có điều vô lý
và suy ra được hàm số thỏa mãn yêu cầu đề bài.

Câu 4. Tìm tất cả các hàm số f : *


 *
thỏa mãn điều kiện sau:
x 2  f  y  f 2  x   y , x , y  *

Lời giải
Giả sử f là hàm số thỏa mãn điều kiện bài toán.
CHINH PHỤC OLYMPIC TOÁN

Trong    ta thế x  y  1 ta được 1  f  1  f 2  1   1  f  1   1

Trong    ta thế x  1 ta được 1  f  y  f 2  1   y  1  y , y  *


 y  f  y  , y  *
1 
Trong    ta thế y  1 ta được

x 2  f  1  f 2  x   1, x , y  *
 x 2  1 f 2  x   1 , x , y  *
 f  x   x , x  *
2 
Từ  1  và  2  ta suy ra f  x   x , x  *
, thử lại ta thấy thỏa mãn yêu cầu bài toán.
Vậy tất cả các hàm số thỏa mãn yêu cầu bài toán là f  x   x , x  *
.
Nhận xét. Đây là một bài toán phương trình hàm trên tập rời rạc, mà cho dưới dạng chia
hết. Bằng các phép thế đơn giản cùng với các đánh giá số học không quá khó khan, ta có
thể nhanh chóng đánh giá được biên của hàm f và để từ đó ta suy ra được hàm số thỏa
mãn đề bài.

Câu 5. Tìm tất cả các hàm số f : *


 *
thỏa mãn điều kiện sau:

f 2  m   f  n   m2  n  , m, n 
2 *
*
IMO Shortlist 2004
Lời giải
Giả sử f là hàm số thỏa mãn điều kiện bài toán.
Trong  *  ta thế m  n  1 ta được:

f 2  1   f  1   12  1   4  f  1   1, do f  1  
2
và f  1   1

Trong  *  ta thế m  1 ta được:

f 2  1   f  n   12  n  , m, n 
2
 1  f  n   1  n  , n 
* 2 *

Chinh phục olympic toán| 13


Bồi dưỡng học sinh giỏi

Trong  *  ta thế n  1 ta được:

f 2  m   f  1  m2  1  , m   f 2  m   1  m2  1  , m 
2 * 2 *

Với p là một số nguyên tố bất kì thì:


Trong  *  ta thế m  1, n  p  1 ta được:
1  f  p  1  p
1  f  p  1 p2  
 1  f  p  1   p
2

Trường hợp 1. 1  f  p  1   p 2  f  p  1   p 2  1.
Ta thế m  p  1, n  1 vào  *  ta được:

  p  1  1   p  1  p  1  1
2 2
f 2  p  1  1
2 2 2
2
1

Mà ta lại có đánh giá sau đây:

TẠP CHÍ VÀ TƯ LIỆU TOÁN HỌC


p  1  1   p  1  p  1  p2  p  1   p2  p     p  1 
2 2 2 2 2 2 2
2
 1 , mâu thuẫn

Do đó, ta phải xảy ra trường hợp còn lại.


Trường hợp 2. 1  f  p  1   p  f  p  1   p  1, với mọi p là số nguyên tố
Hay tồn tại k sao cho f  k   k.
Với mỗi k như thế và số tự nhiên n  0 bất kì thì ta có:
k2  f n  k2  n  k2  f n
2
  p  1 2
 f n    p  1 2

 2n  f  n    f  n   n 
2

Khi ta chọn k là một số đủ lớn thì ta bắt buộc phải có: f  n   n , n  *


, thử lại thỏa.
Vậy tất cả các hàm số thỏa mãn yêu cầu bài toán là: f  n   n , n  *
.
Nhận xét. Cũng tương tự như ở bài toán 4, đây là một bài phương trình hàm trên tập rời
rạc có dạng chia hết. Cũng tương tự ở bài trên, ta cũng thế bằng các phép thế đơn giản để
phát hiện một số tính chất của đề bài. Nhưng ở bài toán 5 này khó hơn ở bài toán 4 rất
nhiều, vì từ các tính chất ta tìm được, ta không thể chặn được khoảng của hàm f để rồi
suy ra f  n   n , n  *
như ở bài toán trên được. Vì thế mà ta phải xét giá trị của hàm số
f tại các giá trị là số nguyên tố để xử lý bài toán và bằng một số kiến thức đơn giản về
giới hạn ta có thể suy ra được f  n   n , n  *
một cách dễ dàng, từ đó kết thúc bài toán.

14 | Tạp chí và tư liệu toán học


Phương trình hàm trên tập rời rạc

Câu 6. Tìm tất cả các hàm f :  thỏa mãn tồn tại số k  và số nguyên tố p sao
cho với mọi n  k , f  n  p   f  n  và nếu m n thì f  m  1  f  n   1.
Iran TST 2005
Lời giải
Giả sử f là hàm số thỏa mãn điều kiện bài toán.
Giả sử n  k và p không chia hết cho n  1 thì khi đó tồn tại k sao cho n  1 n  kp.
Suy ra ta được f  n  f  n  kp   1

Mặt khác ta lại có f  n   f  n  kp  nên f  n  f  n   1  f  n  1  f  n   1

Với n  1 bất kì thì n  1  n  1 kp  f  n  f   n  1 kp   1  2


Do đó với n  1 thì ta có: f  n   1, 2 .
Bây giờ ta sẽ xét hai trường hợp sau:
Trường hợp 1. f  n   2, n  k và p n  1.
CHINH PHỤC OLYMPIC TOÁN

Xác định n  k và p không chia hết cho n  1 khi đó tồn tại m sao cho: n  1 m và p m  1.
Suy ra f  n  f  m   1  3 hay f  n   1
Ta xác định hàm f như sau:
 f  n   2, n  k và p n  1.
 f  n   1, n  k và p không là ước của n  1.
 f  i   f  i  p  , i  k.
Trường hợp 2. f  n   1, n  k và p n  1.

 
Trong trường hợp này f  n   1, n  k và nếu giả sử S  a f  a   2 thì sẽ không tồn tại
m , n  S thỏa mãn m  1 n.
Ta xác định hàm f như sau f  n   1, 2 , n  .
Với S là một tập con vô hạn của sao cho không tồn tại m , n  S thỏa mãn m  1 n và với
n  1 thì f  n   2  n  S ; f  x   1, với các giá trị x  1 còn lại và f  1  là một số bất kì

xác định bởi f  2  f  1   1.


Từ đây ta thử lại đề bài và thấy thỏa mãn nên ta hoàn thành bài toán.
Nhận xét. Đây là một bài toán phương trình hàm trên tập rời rạc khó và điều kiện ràng
buộc khá là khó chịu. Và bằng các phép thế để tìm ra các tính chất của hàm, cùng với các
kĩ thuật xử lý rất khó khan, chúng ta đã xử lý được bài toán. Đây là một bài toán khó, các
bạn đọc cần nghiên cứu và đọc thật kĩ.

Chinh phục olympic toán| 15


Bồi dưỡng học sinh giỏi

Câu 7. Cho p là số nguyên tố lẻ. Tìm tất cả các hàm f :  thỏa mãn đồng thời các
điều kiện:
i) f  m   f  n  với m  n  mod p 
ii) f  mn   f  m  f  n  , m , n 
USA TST
Lời giải
Giả sử f là hàm số thỏa mãn điều kiện bài toán.
Với k  , thì ta có f  p  k  1    f  pk   f  p   f  k  1   f  k    0
Bây giờ ta sẽ xét hai trường hợp sau
Trường hợp 1. f  p   0
Dễ thấy nếu f  1   0 thì f  n   0, n  , mâu thuẫn với f  p   0.
Xét riêng khi f  1   1.

TẠP CHÍ VÀ TƯ LIỆU TOÁN HỌC


Với mỗi x  và p khong chia hết cho x ta có y  sao cho xy  1  mod p  .
Do đó ta có f  x  f  y   f  xy   f  1   1, x , y 
Suy ra: f  n   1 và p không chia hết cho n.
Mặt khác ta lại có f  n2   f 2  n   1 với p không chia hết cho n nên f  m   1, nếu m là
một số chính phương mod p và p không chia hết cho m.
Nếu không tồn tại i , với p không chia hết cho i sao cho f  i   1 thì ta có ngay
f  n   1, n  và p không chia hết cho n.
Xét i là một số không chính phương mod p và k là một số không chính phương mod p
và p không chia hết cho k bất kì thì ta suy ra ik là số chính phương mod p.
Mặt khác ta lại có f  k    f  i  f  k    f  ik   1
Hay
 f  x   1, nếu x là một số chính phương mod p và p không chia hết cho x
 f  x   1, nếu x là một số không chính phương mod p
Xét số x 0 sao cho f  x0   1.
Bây giờ từ điều kiện ii) ta thay m  x0 , n  p ta được:
f  p   f  px0   f  p  f  x0  hay f  p   1
Suy ra:
 f  x   1, nếu x là số chính phương mod p
 f  x   1, nếu x là một số không chính phương mod p
Trường hợp 2. f  p   0 suy ra f  n   0, p n .
Khả năng 1. Nếu f  1   0 thì f  n   0, n  .

16 | Tạp chí và tư liệu toán học


Phương trình hàm trên tập rời rạc

Khả năng 2. Nếu f  1   0


Giả sử tồn tại x 0 sao cho f  x0   0 và p không chia hết cho x 0
Suy ra f  nx0   0, n 
Ta có dãy x0 , 2 x0 ,...,  p  1  x0  là một hệ thặng dư đầy đủ mod p
Suy ra f  1   0, điều này mâu thuẫn.
Vậy ta có f  x   0  p x và f  x   1, với các giá trị x còn lại.
Từ các kết quả trên đây, ta thấy có 4 hàm số thỏa mãn yêu cầu bài toán:
0 if p n
f  n   0, n  f n  
1 if n p
1 if n la mot so chinh phuong mod p
f  n   1, n  f  n  
1 if n khong la mot so chinh phuong mod p
CHINH PHỤC OLYMPIC TOÁN

Vậy đây là tất cả các hàm thỏa mãn yêu cầu bài toán.
Nhận xét. Đây là một bài toán khó, với điều kiện hàm rất khó xử lý, một bài toán khó
trong kì thi chọn đội tuyển IMO của Mỹ, và việc ứng dụng sâu sắc các kiến thức Số Học
tổng hợp trong lời giải, nó có vẻ khá phức tạp. Mong bạn đọc suy nghĩ và đọc thật kĩ, và
mong bạn đọc có một lời giải khác ngắn gọn và hay hơn cho bài toán.

Câu 8. Tìm số nguyên không âm n nhỏ nhất sao cho tồn tại hàm số f :   0,   khác
hằng số thỏa mãn đồng thời các điều kiện:
i) f  xy   f  x  f  y  , x , y 
ii) 2 f  x 2  y 2   f  x   f  y   0, 1,..., n , x , y 
Với số n tìm được, hãy tìm tất cả các hàm số thỏa mãn.
Lời giải
Với n  1 xét hàm f được xác định như sau:
0 if p x
f x   , với p là số nguyên tố có dạng 4 k  3
 1 if x p
Hiển nhiên hàm số trên thỏa mãn yêu cầu bài toán.
Giả sử với n  0 thì cũng tồn tại hàm số f thỏa mãn yêu cầu bài toán.
Khi đó thì ta có:
2 f  x 2  y 2   f  x   f  y   0, x , y   2 f  x 2  y 2   f  x   f  y  , x , y  
Từ điều kiện i) ta thế x  y  0 ta được:
f  0   f 2  0   f  0   0 hoặc f  0   1
Trường hợp 1. f  0   1

Chinh phục olympic toán| 17


Bồi dưỡng học sinh giỏi

Ta thế y  0 vào    thì ta được 2 f  x 2   f  x   f  0   f  x   1, x 

Mà f  x 2   f 2  x  , x  nên ta suy ra:

2 f 2  x   2 f  x 2   f  x   1, x   f  x   1, x  , do f  x   0, x 
Điều này lại trái với giả thiết f khác hằng số.
Trường hợp 2. f  0   0
Ta thế y  0 vào    thì ta được 2 f  x 2   f  x   f  0   f  x  , x 

Mà f  x 2   f 2  x  , x  nên ta suy ra 2 f 2  x   2 f  x 2   f  x  , x 
1
Suy ra với mỗi x  thì ta phải có f  x   0 hoặc f  x   .
2
1
Nếu tồn tại x 0 sao cho f  x0   .
2
Ta thế x  y  x0 vào    thì ta được:

TẠP CHÍ VÀ TƯ LIỆU TOÁN HỌC


2 f  2 x0   2 f  2  f  x0   2 f  x02  x02   f  x0   f  x0   2 f  x0     
Từ    ta thay x  1, y  0 thì ta được: f  1   0.
Từ    ta thay x  1, y  1 thì ta được: f  2   0.
Từ đây ta thay f  2   0 vào     thì ta được:
f  x   0, x  , điều này lại mâu thuẫn với f khác hằng số.
Vậy từ đây ta khẳng định được rằng n  1 là giá trị nhỏ nhất thỏa mãn yêu cầu bài toán.
Khi ta tìm được n  1 ta sẽ quay lại việc giải quyết bài toán đề bài.

Tìm tất cả các hàm số f :   0,   khác hằng số thỏa mãn đồng thời các điều kiện:
i) f  xy   f  x  f  y  , x , y 
ii) 2 f  x 2  y 2   f  x   f  y   0, 1 , x , y 

Giả sử f là hàm số thỏa mãn điều kiện bài toán.


Ta dễ dàng chứng minh được rằng: f  0   0, f  1   1.
Trong i) ta thế y  x thì ta được f  x 2   f 2  x  , x 
Trong ii) ta thế y  0 thì ta được
2 f  x 2   f  x   2 f 2  x   f  x   0, 1  f  x   0, 1
Trong i) ta thế x  y  1 thì ta được
f 2  1   f  1   1  f  1   1
Trong i) ta thế x  1, y   x thì ta được:

18 | Tạp chí và tư liệu toán học


Phương trình hàm trên tập rời rạc

f  x   f  1  f  x  , x   f  x   f  x  , x 
Trường hợp 1. Tồn tại số nguyên tố p sao cho f  p   0.
Giả sử cũng tồn tại số nguyên tố q  p sao cho f  q   0.
Trong ii) ta thế x  p , y  q thì ta được
2 f  p2  q 2   f  p   f  q   0  f  p2  q 2   0
Do đó với mỗi a , b  thì ta luôn có:
2 f  a2  b 2  f  p2  q 2   2 f  a 2

 b 2  p 2  q 2   2 f  ap  bq  2
  aq  bp 
2
0
Lưu ý rằng. a 2
 b 2  p 2  q 2    ap  bq    aq  bp 
2 2
là đẳng thức Brahmagupta –
Fibonacci nổi tiếng, cũng đã được đề cập đến trong nhiều cuốn sách, mong bạn đọc lưu ý
chi tiết này để giải toán.
Vì 0  f  x   f  y   2 f  x 2  y 2  nên f  aq  bp   0.
CHINH PHỤC OLYMPIC TOÁN

Do  p , q   1 nên tồn tại a , b  sao cho aq  bp  1.


Suy ra được 1  f  1   f  aq  bp   0, điều này là vô lý.
Vậy tồn tại duy nhất số nguyên tố p sao cho f  p   0.
Khả năng 1. Nếu p là số nguyên tố có dạng 4 k  1, k  thì tồn tại a  sao cho p a 2  1

hay f  a 2  1   0.
Lưu ý rằng. Kết quả này các bạn có thể tham khảo trong phần chuyên đề Thặng dư bình phương.
Mặt khác, trong ii) ta thế x  1, y  a thì ta được:
2 f  12  a2   f  1   f  a   2 f  a 2  1   f  1   f  a   1  f  a 2  1   1,
Điều này là mẫu thuẫn.
Vậy từ đấy chỉ xảy ra khả năng còn lại.
Khả năng 2. Nếu p là số nguyên tố có dạng 4 k  3 thì
Từ đó ta có f  x   0  p x và f  x   1 với các giá trị x còn lại.
Trường hợp 2. f  p   1 với mọi số nguyên tố p.
Khi đó f  x   1, x  \0
Vậy từ đó có hai hàm số thỏa mãn yêu cầu bài toán là:
0 if p x
 f x   , trong đó p là một số nguyên tố bất kì có dạng 4 k  3, k  .
1 if x p
0 if x  0
 f x  
1 if x  0
Nhận xét. Đây là một bài toán phương trình hàm trên tập rời rạc phải nói là rất rất khó, sử
dụng rất nhiều kiến thức trong Số Học cũng như kĩ năng phán đoán và biến đổi thuần

Chinh phục olympic toán| 19


Bồi dưỡng học sinh giỏi

thục. Sử dụng rất nhiều các mạng kiến thức liên quan đến số nguyên tố, thặng dư bình
phương hay các đẳng thức rất nổi tiếng trong Toán học. Thực sự đây là một bài hàm liên
quan đến số học tổng hợp, rất hay và thú vị, mong bạn đọc nghiên cứu thật kĩ càng và cẩn
thận bài toán này.

Câu 9. Giả sử hàm số f : *


 thỏa mãn các điều kiện sau:
  n1
1  f  if n  2 m  1
  2 
f  1   1 và f  n   
1  n
f  if n  2 m
 2
Tìm các giá trị của n sao cho f  n   2019.
Lời giải
Từ cách xác định của hàm f ta dễ dàng tính được:

TẠP CHÍ VÀ TƯ LIỆU TOÁN HỌC


f  2   f  3   2; f  4   f  5   f  6   f  7   3
Bây giờ ta sẽ viết dưới dạng nhị phân như sau:
     
f  1   f 12  1; f  2   f 10 2  2; f  3   f 112  2

f  4   f  100   3; f  5   f  101   3; f  6   f  110   3;...


2 2 2

Từ cách việt dưới dạng nhị phân như trên, ta dự đoán f  n  là số chữ số trong biễu diễn
nhị phân của số n.
Ta sẽ chứng minh dự đoán này bằng quy nạp như sau.
Thật vậy, ta thấy khẳng định đúng với n  1, n  2.
Giả sử khẳng định đúng đến n. Ta sẽ chứng minh khẳng định đúng đến n  1.
Nếu n là số chẵn thì n  ak ak 1 ...a1 0 2  f  n   k  1.
Khi đó thì ta có n  1  ak ak 1 ...a1 12
n n n
Từ đấy ta có   ak ak 1 ...a1 2  f    k  f  n   1  f    k  1, tức là bằng số chữ số
2 2 2
trong biểu diễn nhị phân của số n.
Nếu n là số lẻ thì bằng cách làm tương tự ta cũng được kết quả tương tự.
Vậy theo nguyên lý quy nạp ta suy ra f  n  là số chữ số trong biểu diễn nhị phân của n.
Ta đó ta suy ra nếu f  n   2019 thì biểu diễn của n trong hệ nhị phân chứa đúng 2019
chữ số.
Vậy từ đó ta suy ra: 2 2018  n  2 2019.
Nhận xét. Đây là một bài toán khá hay, với tư tưởng giải là đưa về hệ nhị phân. Bằng cách
biểu thị bình thường thì ta không thể tìm ra được tính chất của dãy, bởi điều kiện nó xen
kẽ với tính chẵn lẽ, rất khó chịu và phức tạp. Mà chỉ bằng cách đưa về hệ nhị phân ta đã

20 | Tạp chí và tư liệu toán học


Phương trình hàm trên tập rời rạc

nắm được quy luật của dãy số mà tác giá đã ẩn đi trong bài toán, và từ đó ta nảy ra ý
tưởng giải và chỉ việc đi triển khai, cuối cùng ta thu được kết quả của bài toán.

Câu 10. Tìm tất cả các hàm số f : *


 *
thỏa mãn các điều kiện sau:
f  1  1, f  3   3

f  2n   f  n 
 với mọi số nguyên dương n .
f  4n  1   2 f  2 n  1   f  n 
f  4n  3   3 f  2 n  1   2 f  n 

IMO 1988
Lời giải
Giả sử f là hàm số thỏa mãn điều kiện bài toán.
Một số nguyên dương k chỉ có thể có một trong bốn dạng sau:
k  4n , k  4n  1, k  4n  2, k  4n  3; k , n 
CHINH PHỤC OLYMPIC TOÁN

Do đó, từ giả thiết của bài toán, hàm số f được xác định một cách duy nhất. Ta sẽ sử
dụng biểu diễn cơ số 2 để tìm biểu diễn của hàm số f .
Ta có các nhận xét sơ bộ như sau:
   
f 12  f  1   1  12 , f 10 2  f  2   1  012

f  11   f  3   3  11 , f  100   f  4   1  001 ,...


2 2 2 2

Từ đấy ta thấy được quy luật như sau:


Quy luật. Biểu diễn của f  n  trong hệ cơ số 2 chính là cách viết ngược lại của biểu diễn

của n trong hệ cơ số 2 tức là f  a a


k k 1 
...a1 a0 2   a0 a1 ...ak 1 ak 2 .

Bây giờ ta sẽ chứng minh dự đoán này bằng quy nạp như sau.
Chứng minh.
Với n  1, 2, 3, 4 thì hiển nhiên đúng, do ta đã thử kiểm tra ở trên.
Giả sử tính chất đã đúng cho với mọi k  n. Ta sẽ chứng minh tinh chất cũng đúng với n.
Trường hợp 1. Nếu n  2 m thì theo giả thiết ta có f  m   f  n  . Vì n  2 m nên nếu m được

biễu diễn trong hệ cơ số 2 dưới dạng m   ak ak 1 ...a1 a0 2 thì n   ak ak 1 ...a1 a0 0 2 .


Mà theo giả thiết quy nạp thì ta có:
f  a a
k k 1 
...a1 a0 0 2  f  n   f  m   f  a a k k 1 
...a1 a0 2   a0 a1 ...ak 1 ak 2   0a0 a1 ...ak 1 ak 2

Từ đây, trong trường hợp này, tính chất được chứng minh.
Trường hợp 2. Nếu n  4m  1 với m   ak ak 1 ...a1 a0 2 thì n   ak ak 1 ...a1 a0 01 2 và

2m  1   ak ak 1 ...a1 a0 1 2 .
Mà theo giả thiết quy nạp thì ta có:

Chinh phục olympic toán| 21


Bồi dưỡng học sinh giỏi

f  a a ...a a 01   f  n   f  4m  1  2 f  2m  1  f  m 
k k 1 1 0 2

 2 f   a a ...a a 1    f   a a ...a a  
k k 1 1 0 2 k k 1 1 0 2

 2 1a0 a1 ...ak 1 ak 2   a0 a1 ...ak 1 ak 2


  1a0 a1 ...ak 1 ak 0 2   a0 a1 ...ak 1 ak 2
  10...0 2   a0 a1 ...ak 1 ak 0 2   a0 a1 ...ak 1 ak 2
  10...0 2   a0 a1 ...ak 1 ak 2   10 a0 a1 ...ak 1 ak 2 .
Từ đây, trong trường hợp này, tính chất được chứng minh.
Trường hợp 3. Nếu n  4m  3 với m   ak ak 1 ...a1 a0 2 thì n   ak ak 1 ...a1 a0 11 2 và

2m  1   ak ak 1 ...a1 a0 1 2 .
Mà theo giả thiết quy nạp thì ta có:

 a a 
...a1 a0 11 2  f  n   f  4m  3   3 f  2m  1   2 f  m 

TẠP CHÍ VÀ TƯ LIỆU TOÁN HỌC


f k k 1

 f  2m  1  2 f  2m  1  2 f  m 
  1a0 a1 ...ak 1 ak 2   1a0 a1 ...ak 1 ak 0 2   a0 a1 ...ak 1 ak 0 2
  1a0 a1 ...ak 1 ak 2   10...0 2
  11a0 a1 ...ak 1 ak 2 .
Từ đây, trong trường hợp này, tính chất được chứng minh.
Vậy theo nguyên lý quy nạp thì quy luật của chúng ta đã được chứng minh.
Vậy tất cả các hàm f  n  thỏa mãn đề bài là:

f  a ak k 1 
...a1 a0 2   a0 a1 ...ak 1 ak 2 ,

trong đó n   ak ak 1 ...a1 a0 2 là biễu diễn của số n trong hệ cơ số 2.


Nhận xét. Cũng với ý tưởng tương tự như ở bài toán 9 là sử dụng hệ nhị phân để tìm ra
quy luật của dãy số. Nhưng ở bài toán 10 thì khó hơn ở các bài trước rất nhiều, do điều
kiện của đề bài khá nhiều, làm ta khá hoang mang, và khá nhiều trường hợp cần xét của n
nên lời giải có vẻ phức tạp. Ở bài này, các bạn cần có một cách nhìn tổng quan để khám
phá ra quy luật và kiểm nghiệm sự chính xác của nó, và từ đó đi chứng minh phỏng đoán
đó bằng phương pháp quy nạp, bên cạnh đó cũng cần sự biến đổi điêu luyện và thật chính
xác thì mới thu được kết quả của bài toán.

22 | Tạp chí và tư liệu toán học


Phương trình hàm trên tập rời rạc

 
Câu 11. Cho hàm số f :  thỏa mãn đồng thời các điều kiện:
 f  n  là ước của n 2018 với mọi n
 f  a  . f  b   f  c  với mọi a , b , c  
và a 2  b 2  c 2
a) Chứng minh rằng nếu n lẻ hoặc n 4 thì f  n   1
b) Gọi A là tập hợp giá trị có thể có của f  2   f  2018  . Tính A
Lời giải
2018
 n2  1 n2  1   a2  1   a2  1 
a) Nếu n  1  mod 2  ta chọn  a , b , c    n, ,   f  n | f  | 
 2 2   2   2 
 2018
   n2  1  
2018  n  1 
2
Ta có f  n |n 2018
 f  n |gcd n , 
    gcd  n , 
2018
   1  f n  1
  2     2 

Nếu n  0  mod 4  ta chọn  a , b , c    4 k , 4 k 2  1, 4 k 2  1   f  4 k | f  4 k 2  1 | 4 k 2  1 
2018

 1
CHINH PHỤC OLYMPIC TOÁN

,  4k 2  1
2018
Ta có f  4 k | 4 k   f  4 k |gcd  4 k 
2018 2018

b) Theo câu a, ta có f  2  2
2018
 f  2   2 0 , 2 1 ,..., 2 2018 

  
 f  2018   f 1009 2  1 1009 2  1 2018

Với  a , b , c    2018, 1018080, 1009 2  1  ta có 
 f  2018  20182018

 f  2018  gcd2018  2018, 1009 2  1  2 2018  f  2018   2 0 , 2 1 ,..., 2 2018 

 a 2  b 2  c 2  * 
Với  a , b , c  thoả 
gcd  a , b , c   1

 Nếu a , b lẻ, c chẵn.


Do a2  b 2  c 2   2 a    2b    2c  nên 1  f  2c   f  2 a  f  2b  .
2 2 2

Mà f : 
 
nên f  2 a   f  2b   f  2c   1 .

 a  c  1  mod 2 

 Nếu  .

 b  0  mod 2 
Khi đó: f  2c   f  2 a  f  2b   f  2 a  , a , c  1 lẻ thỏa mãn (*).

Do đó f  2c   f  2 a  * *   b  
: a 2  b 2  c 2 , a , c  1 .
 2018 1018080 10092  1 
Ta có f  2018  thỏa  * *  với bộ  a , b , c    , ,  mà f  2  không thỏa
 2 2 2 
* * do a , c  1 nên f  2   f  2018 
Dễ dàng chứng minh được 2 a  2 b  2 c  2 d với  a , b    c , d  bằng cách chia cả hai vế cho
2  với   min a , b , c , d nên T  C 2019
2
.

Chinh phục olympic toán| 23


Bồi dưỡng học sinh giỏi

Câu 12. Có tồn tại hàm số f : S  S thỏa mãn điều kiện


f  a  f  b   f  a 2 b 2  , a , b  S , a  b không, trong đó S  *
\1 ?
Lời giải
Giả sử tồn tại hàm số thỏa mãn yêu cầu bài toán. Ta sẽ tìm tính chất đặc biệt của hàm số f
Xét hai biểu thức sau f  a  f  d  f  c  , f  a  f  b  f  c  .
Ta có f  a  f  d  f  c   f  a 2 d 2  f  c   f  a 4 d 4c 2  và

f  a  f  b  f  c   f  a  f  b 2 c 2   f  a 2 b 4c 4  .
Ta sẽ chọn c , d sao cho a 4 d 4c 2  a2 b 4c 4  a2 d 4  b 4 c 2  ad 2  b 2 c  1  .
Với c , d thỏa mãn (1) thì
f  a 4 d 4c 2   f  a 2 b 4c 4   f  a  f  d  f  c   f  a  f  b  f  c   f  b   f  d  2
Tất nhiên ta cần chọn d  b và c thỏa mãn  1  chẳng hạn d  b 2 , c  ab 2 .

TẠP CHÍ VÀ TƯ LIỆU TOÁN HỌC


Nên từ  2  ta được f  b   f  b 2  , b  S (3).
Từ tính chất  3  ta có
f  a  f  b   f  a 2 b 2  , a , b  S , a  b  f  a  f  b   f  ab  , a , b  S , a  b  4
Sử dụng tính chất  3  ,  4  ta được f  16   f  4 2   f  4   f  2 2   f  2  .
Mặt khác f  16   f  2.8   f  2  f  8  .
Từ hai đẳng thức trên ta được f  2  f  8   f  2   f  8   1 (vô lí.)
Vậy không tồn tại hàm số f thỏa mãn yêu cầu bài toán.

Câu 13. Tìm tất cả các hàm số f : *


 *
thỏa mãn điều kiện

 n  1  f  n  f  f  n    n 2  n , n 
2 *
.
Lời giải
Với dạng toán mà giả thiết về bất đẳng thức thì việc dự đoán được nghiệm đóng vai trò
quan trọng trong việc định hướng lời giải. Việc dự đoán nghiệm thường ta thông qua việc
tính các giá như f  1  , f  2  , f  3  ,...
Thay n  1 vào phương trình ban đầu ta được 0  f  1  f  f  1    2  f  1   1 .
Thay n  2 vào phương trình ban đầu ta được 1  f  2  f  f  2    6 .
Từ bất đẳng thức này suy ra f  2   1 , nếu f  2   3 .
Từ điều kiện ban đầu ta thay n bởi f  n  thu được

 f  n   1  
 f  f  n   f f  f  n    f  n   f  n  , n 
2 2 *
(1).


Từ (1), thay n  2 ta được  f  2   1   f  f  2   f f  f  2    f  2   f  2  
2 2

24 | Tạp chí và tư liệu toán học


Phương trình hàm trên tập rời rạc

 
 4   f  2   1  f  f  2   f f  f  2    f  f  2    2
2

Điều này mâu thuẫn với 1  f  2  f  f  2    6 . Do đó f  2   2 .


Do đó ta dự đoán f  n   n , n  *
. Ta sẽ chứng minh bằng phương pháp quy nạp.
Giả sử f  1   1, f  2   2,..., f  n  1   n  1 . Ta chứng minh f  n   n
Với mỗi số nguyên dương n , đặt f  n   m . Ta xét các trường hợp sau:

Nếu m  n  1  f  n  f  f  n    mf  m   m 2   n  1 vô lí.
2

 Nếu f  n   m  n  1   n  1  f  f  n    f  n  f  f  n    n 2  n  f  f  n    n

  
f f  f  n  f  f n  f  f n . f f  f n  f  f n   
2
 n2 ,

 
Kết hợp với f f  f  n   f  f  n     f  n   1 ta được n2   f  n   1   f  n   n  1 vô lí.
2 2

Do đó f  n   n . Vậy f  n   n, n  *
.
CHINH PHỤC OLYMPIC TOÁN

Câu 14. Tìm tất cả hàm số f :  thỏa mãn đồng thời hai điều kiện sau:
i) x  f  y  f  x    y  f  x  f  y   với mọi x , y  ;

 f  x   f  y  
ii) Tập hợp I   , x , y  , x  y  là một khoảng
 xy 
Lời giải
Phân tích. Điểm mấu chốt để giải bài toán chính là hiểu và tận dụng được giả thiết ii).
Tập hợp I là một khoảng có nghĩa là nếu a , b  I , a  b thì  a , b   I .
Từ điều này ta suy ra tính chất quan trọng của tập hợp I là nếu có số thực a  I thì hoặc
là x  a , x  I hoặc là x  a , x  I .
Khi đó ta có lời giải sau.
Để ý rằng nếu f  x   f  y   x  y thì x  y ( do i) ).
Do đó 1  I .
Giả sử t  I . Khi đó t  1 và tồn tại x , y  , x  y sao cho f  x   f  y   t  x  y  .
Vì x  y nên f  y  f  x    f  x  f  y   và
f  y  f  x   f  x  f  y  yx 1
  .
 y  f  x    x  f  y  y  x  t x  y 1  t
1 1
Do đó  I . Mà I là một khoảng nên mọi phần từ x nằm giữa t và cũng thuộc I
1t 1t
1
Nếu t  1 thì  0 (mâu thuẫn vì khi đó ta lại có 1  I )
1t
1
Do đó t  1 . Tương tự ta cũng có  1 , tức là t  0 .
1t

Chinh phục olympic toán| 25


Bồi dưỡng học sinh giỏi

1
Như vậy ta đã chứng minh được rằng nếu t  I thì  I và t  0 .
1t
1
Mà khi t  0 thì  0 nên ta có mâu thuẫn.
1t
Như vậy I phải là tập rỗng mà rõ ràng I   , chẳng hạn  f  1   f  0    I .
Do đó không tồn tại hàm số thỏa mãn yêu cầu bài toán.
Ngoài ra ta cũng có thể giải bài toán này như sau.
f x  f  y
Đặt F  x , y   với mọi x , y  , x  y .
xy
Tương tự cách đầu, ta có F  x , y   1, x , y  , x  y
Nếu tồn tại x  y sao cho F  x , y   0 thì F  y  f  x  , x  f  x    1 (vô lý).
Do đó F  x , y   0; 1 , x , y  , x  y  * 
Suy ra f là đơn ánh. Do đó với mọi x , y  , x  y ta có

TẠP CHÍ VÀ TƯ LIỆU TOÁN HỌC


f  x  f  y   f  y  f  y   f  y  f  x   f  y  f  y   x  y

Hay F  x  f  y  , y  f  y    F  y  f  x  , y  f  y   F  x , y   1
Từ đây, kết hợp với  *  và ii) ta suy ra F  x , y   1, x , y  , x  y
Suy ra f đơn điệu tăng trên .
 Nếu x  y thì f  x   f  y   x  y hay y  f  x   x  f  y  .
Suy ra f  y  f  x    f  x  f  y   , kết hợp với i), ta được x  y .
 Tương tự nếu x  y ta cũng có mâu thuẫn.
Vậy không tồn tại hàm số thỏa mãn yêu cầu bài toán.

thỏa mãn f 2  m   f  n   m2  n  , m, n 


2
Câu 15. Tìm các hàm số f : *
 * *

Lời giải
Giả sử tồn tại hàm f thỏa mãn yêu cầu bài toán
Ta có f 2  1   f  1  4  f  1   1

Thay m  1  1  f  n   1  n  , n 
2

 f  p  1  1  p
Thay m  1, n  p  1  p  P   1  f  p  1 p 2  
 f  p  1  1  p
2

Nếu p  P : f  p  1   p 2  1 .

 
2
Thay m  p  1, n  1  f 2  p  1   1|  p  1   1
2

Suy ra  p 2  1   1  p 2  2 p  2 
2 2

26 | Tạp chí và tư liệu toán học


Phương trình hàm trên tập rời rạc

Có p 2  1  p 2  2 p  2 nên có mâu thuẫn


Vậy f  p  1   p  1, p  P
2
Thay m  p  1   p  1  f  n   p  1  n 
2 2

 


Suy ra n   p  1    0  mod  p  1  f  n
2 2 2

Mà  f  n   n   0  mod  p  1   f  n  
2 2

Suy ra  f  n   n   0  mod  p  1   f  n  
2 2

Nếu f  n   n , khi đó ta có  f  n   n   f (n)   p  1  , p  P  * 


2 2

Mà tập các số nguyên tố là vô hạn nên f  n    p  1    .


2

Mà  f  n   n  cố định với n xác định.


2

Do đó chỉ cần chọn p nguyên tố đủ lớn ta có mâu thuẫn  *  .


CHINH PHỤC OLYMPIC TOÁN

Vậy f  n   n, n  *
(thử lại thỏa mãn)

Câu 16. Cho hàm f  x , y  thỏa mãn các điều kiện:


 f  0, y   y  1; f  x  1, 0   f  x , 1 
 f  x  1, y  1   f  x , f  x  1, y  
Với mọi số nguyên không âm x , y . Tìm f  4, 1981 
Lời giải
Ta có f  1, n   f  0, f  1, n  1    1  f  1, n  1 
Do đó f  1, n   n  f  1, 0   n  f  0, 1   n  2
Ta lại có f  2, n   f  1, f  2, n  1    f  2, n  1   2
Do đó f  2, n   2n  f  2, 0   2n  f  1, 1   2 n  3
Bây giờ f  3, n   f  2, f  3, n  1    2 f  3, n  1   3
Đặt un  2un1 và u0  f  3, 0   3  f  2, 1   3  0
Do vậy un  2 n 3 f  3, n   2 n 3  3
 f  4, n   f  3, f  4, n  1    2 f  4,n1 3  3

Ta có  f  4, 0   f  3, 1   2 4  3  13

 f  4, 2   2  3
224

Bằng qui nạp ta chứng minh được f  4, n   2 22..24  3


Trong đó số mũ chứa  n  2  chữ số 2. Từ đó f  4, 1981   2 22.24  3 với số mũ chứa 1983
chữ số 2.

Chinh phục olympic toán| 27


Bồi dưỡng học sinh giỏi

 
Câu 17. Cho hàm f :  thỏa mãn các điều kiện sau:
iii) f  n  1   f  n  ; n  

iv) f  f  n    3n , n  Z  .
Hãy tính f  2003  .
Lời giải
Từ  i  ,  ii   f  1   f  f  1    3  f  1   2
Ta có f  2   f  f  1    3.1  3
f  3   f  f  3    3.2
f  2.3   f  f  3    3.3  32
.............
Suy ra f  2.3n   3n 1 , n  
; f  3n   2.3n ; n  Z 

   

TẠP CHÍ VÀ TƯ LIỆU TOÁN HỌC


Nên có f  3n 1   f f  2.3n   2.3n 1 và f  2.3n 1   f f  3n 1   3.3n 1  3n 2

Do đó khẳng định đúng với mọi n


Ta có  3n  1  số nguyên m nằm giữa 3 n và 2. 3 n và do giả thiết  i  f  n  1   f  n  nên có

3 n
 1  số nguyên m nằm giữa f  3n  và f  2.3n  suy ra 0  m  3n  f  3n  m   2.3n  3n .


Do giả thiết  ii  suy ra f  2.3n  m   f f  3n  m   3  3n  m  
Vậy f  2.3n  m   3  3n  m  với 0  m  3n

Suy ra n  2003  2.36  545  f  2003   3  36  545   3822 .

Câu 18. Cho f  n  là hàm số xác định với mọi n  *


và lấy giá tị không âm thỏa mãn
tính chất:
 n , m   * : f  m  n   f  m   f  n  lấy giá trị 0 hoặc 1
 f  2   0 và f  3   0 .
 f  9999   3333 .
Tính f  2000  .
Lời giải
Vì f  m  n   f  m   f  n  lấy giá trị 0 hoặc 1 nên ta suy ra f  m  n   f  m   f  n 
 f  2   2 f  1  f  1  0  f  3   1
Ta có f  6  f  3  f  3  2
f  9  f  6  f  3  3
.................
f  9999   f  9996   f  3   3333

28 | Tạp chí và tư liệu toán học


Phương trình hàm trên tập rời rạc

Vì giả thiết cho f  9999   3333 nên ta có dấu “=” ở các bất đẳng thức trên xảy ra, tức là
f  3n   n , n  1, 2,..., 3333  f  1998   666, f  2001   667
Mặt khác nếu a a , b   * và a  b  f  a   f  b   f  a  b   f  b  .
 666  f  2000   667  f  2000   666 hoặc 667
Giả sử f  2000  667   f  4000   1334  f  6000   1334  667  2001 .
Mà f  6000   2000 , mâu thuẫn. Vậy f  2000   666 .

Câu 19. Cho f , g là các hàm xác định trên thỏa mãn điều kiện
f  x  y   f  x  y   2 f  x  .g  y  , x , y 

Chứng minh rằng nếu f  x   0 và f  x   1, x  thì g  y 0   a  1

Lời giải
CHINH PHỤC OLYMPIC TOÁN

Ta dùng phương pháp phản chứng


Giả sử lại một điểm y0  : g  y0   a  1

Ta lấy x0 : f  x0   0 và xây dựng dãy xk  k  0, 1, 2.... như sau:


x k  y 0 , khi f  x k  y 0   f  x k  y 0 

xk 1 
x k  y 0 , khi f  x k  y 0   f  x k  y 0 

Theo giả thiết ta có 2 f  xk  1   f  xk  y0   f  xk  y0   f  x k  y 0   f  x k  y 0 

 2 f  xk  g  y0   2 a f  xk 

Nên f  xk  1   a f  x k  với a  1; k  1, 2, 3...

Do đó ta có: f  xk   a k f  x0  .

Nhưng vì f  x0   0 và a  1 nên có thể chọn k sao cho a k f  x0   1 dó đó f  xk   1

Mâu thuẫn với giả thiết.


Vậy g  y   1, y  R

Câu 20. Cho hàm số f :  thỏa 2 điều kiện


iii) f  x   1  x ; x 
iv) f  x  y   f  x  . f  y  ; x , y 
Chứng minh rằng không thể tồn tại hai số a ; b  mà f  a  . f  b   0
Lời giải
Ta sẽ chứng minh f  x   0, x 
Thật vậy! Với x  1 thì theo điều kiện  i  ta có ngay f  x   0

Chinh phục olympic toán| 29


Bồi dưỡng học sinh giỏi

Với x  1 , trước hết ta sẽ chứng minh bất đẳng thức:


2n
  x 
f  x    f  n   , x  , n   1
  2 
Với n  0 thì bất đẳng thức đúng!
2k
  x 
Giả sử  1  đúng với n  k  0 tức f  x    f  k  2
 2 
2k 2 k .2 2 k 1
  x   x x    x   x 
Ta có  f  k   f  k 1  k 1    f  k 1   f  k 1  tức  1  đúng với n  k  1
 2  2 2    2  2 
Theo nguyên lý quy nạp toán học bất đẳng thức  1  đúng.
x  x 
Bây giờ chọn n đủ lớn để x  2 n , x  tùy ý, khi đó 1 f  n 0
2n 2 
2n
  x 

TẠP CHÍ VÀ TƯ LIỆU TOÁN HỌC


Do đó  f  n    0 tức f  x   0, x 
  2 
Như vậy không thể tồn tại hai số a ; b  mà f  a  . f  b   0 .

2003
Câu 21. Cho f  x , y   cos 2  x  y   a cos  x  y    với a ,   .
2
Chứng minh rằng min  f  x , y     max f  x , y    2003.
2 2

Lời giải
  2003
Ta có f  0, 0   f  ;   2
2 2 2
     2003 2003
Nên max f  x , y   max  f  0, 0  , f  x , y     max f  x , y  
2
 ,   
  2 2  2 2
  2003    2003
Ta lại có f  ;     a.sin  , f   ;      a.sin 
4 4 2  4 4 2
     2003
Nên f  ;   f  ;    .
4 4  4 4 2
         2003
Suy ra min f  x , y   min  f  ;  , f   ;      x , y  
  4 4   4 4  2


 min  f  x , y  
2
  2003
2
.

Do đó min  f  x , y     max f  x , y    2003.


2 2

30 | Tạp chí và tư liệu toán học


Phương trình hàm trên tập rời rạc

x2  1
Câu 22. Cho hàm số f  x   , x  0.
2x
Giả sử f 0  x   x và f n  x   f  f n1  x   n   * , x  0 .
fn  x  1
Chứng minh n  , x  1, 0, 1  1
f n1  x   x1
2n

f 
 x1
Lời giải
1 n n
 và q  x   1  x  12   x  12  , x , y  
n n

Đặt pn  x    x  1   x  1
2 2

2   2  
n



 pn  1  x   p n  x   q n  x 
2 2


Ta có: qn  1  x   2 pn  x  qn  x  , x , y  

 f  x   x  x  p0  x  , x  0
CHINH PHỤC OLYMPIC TOÁN

 0 1 q0  x 
2
 pk  x  
  1
pk  x   qk  x   pk2  x   q k2  x  pk  1  x 
Giả sử f k  x    f k 1  x    
qk  x  pk  x  2 pk  x  q k  x  q k  1  x 
2.
qk  x 
pn  x 
Do đó f n  x   , n   , x  0
qn  x 
Ta có n   , x  1, 0, 1 thì có:
 2  2 1 
n n n1 n

f n  x   x  1    x  1    x  1    x  1  
2 2


f n  1  x   x  1  2   x  1  2   x  1  2   x  1  2  1 
n n n 1 n

   
2
 x  1  2   x  1  2 
n n
2n 2n

 
 2  x  1  x  1 1
 2 n 1 2n  1
 1 2 n 1 2n  1
 1 2n
 x  1   x  1  x  1   x  1  x1
f 
 x1
Câu 23. Cho hàm số f : *
 *
 *
là hàm số thỏa mãn đồng thời các điều kiện sau:
i) f  1, 1   2
ii) f  m  1, n   f  m , n   m , m , n  *

iii) f  m, n  1   f  m, n   n , m , n  *

Tìm tất cả các cặp số  p , q  sao cho f  p , q   2019.


Doãn Quang Tiến
Lời giải
Áp dụng điều kiện ii) ta có:

Chinh phục olympic toán| 31


Bồi dưỡng học sinh giỏi

p  p  1
f  p , q   f  p  1, q    p  1   f  p  2, q    p  2    p  1   ...  f  1, q  
2
p  p  1 q  q  1 p  p  1
 f  1, q  1   q  1    ...  f  1, 1     2019, áp dụng điều kiện
2 2 2
iii)
Từ điều kiện i) ta có f  1, 1   4 nên từ đẳng thức trên ta suy ra:
q  q  1 p  p  1 q  q  1 p  p  1
f  1, 1    2   2019
2 2 2 2
p  p  1 q  q  1
   2017   p  q  p  q  1   2.2017
2 2
Vì 2017 là số nguyên tố và  p  q    p  q  1  nên có thể xảy ra các trường hợp sau:
Trường hợp 1. p  q  1 và p  q  1  4034 thì từ đó ta được: p  2018, q  2017.
Trường hợp 2. p  q  2 và p  q  1  2017 thì từ đó ta được: p  1010, q  1008.

TẠP CHÍ VÀ TƯ LIỆU TOÁN HỌC


Vậy tất cả các cặp số  p , q  sao cho  p , q   2019 là:  p , q    2018, 2017  ,  1010, 1008  .

Câu 24. Tìm tất cả các hàm số f :  thỏa mãn các điều kiện sau:
i) 0  f  x   x 2 , n 
ii) f  x   f  y  chia hết cho x  y với mọi x , y  , x  y
Lời giải
Ta sẽ phải xét hai trường hợp dưới đây.
Trường hợp 1. f là hàm số hằng.
Giả sử f  x   c  const, c là hằng số thuộc .
Cho x  0 thì từ i) ta suy ra: 0  f  0   0  f  0   0. Vậy f  x   0, x  .
Trường hợp 2. f không là hàm số hằng.
Vì f  0   0 nên khi cho y  0 thì từ điều kiện ii) ta được: x f  x  , x  \0 .
Với x  1 thì từ điều kiện i) ta suy ra: f  1   0 hoặc f  1   1.
Khả năng 1. f  1   0.
Cho y  1 thì từ điều kiện ii) ta suy ra:  x  1  f  x  , x  \1 .

Mà ta đã có x f  x  , x  \0 và  x , x  1   1

Nên x  x  1  f  x   x  x  1   x 2  x  f  x 

 f  x   x 2  f  x   x2  x
Mặt khác ta có  
 x f  x   f  x   x
2

Ta loại trường hợp f  x   x 2 vì f  1   1  0.

32 | Tạp chí và tư liệu toán học


Phương trình hàm trên tập rời rạc

Vậy trong khả năng 1 này chỉ có hàm số f  x   x 2  x , x  thỏa mãn yêu cầu bài toán.
Khả năng 2. f  1   1.
Khi x f  x  , x  \0 và  x  1  f  x  , x  \1 thì ta lấy k tùy ý sao cho f  k   mk ,
với m tùy ý
Từ điều kiện ii) có  k  1   mk  1    k  1   mk  1    mk  m   m  1   k  1   m  1 

Mà từ điều kiện i) ta được f  k   k 2  mk  k 2  m  k


 k  1   m  1 
Mặt khác ta lại có   k1 m1 k  m
 m  1   0
Do đó m  1  0 nên suy ra: m  1 hoặc m  k.
Vậy từ đó ta được f  x   x 2 , x  hoặc f  x   x , x  .
Thử lại thì thấy các hàm số này thỏa mãn yêu cầu bài toán.
CHINH PHỤC OLYMPIC TOÁN

Vậy tất cả các hàm số thỏa mãn đề bài là


f  x   0, f  x   x , f  x   x 2 , f  x   x 2  x , x  .

Câu 25. Tìm tất cả các hàm số f : *


  *
 mà tập *
  x  x  0 thỏa mãn:
f  xy 
f  x   f  y   2 xyf  xy   , x , y  *
 1
f x  y

Lời giải
Ta sẽ giải bài toán này thông qua ba bước sau đây.
Bước 1. Ta sẽ chứng minh f  1   1.
Thật vậy, cho y  1 vào  1  và đặt f  1   a thì ta được
f x f x
f  x   a  2 xf  x    f  x  1  2
f  x  1  1  2x  f  x   a
1
a 1 1 1
Từ đó ta suy ra f  2    , f  3  4  , f  4  2  
*
4a 4 5 5  4 a 7  5 a  4 a
a
4
f  4
Mặt khác, ta cho x  y  2 vào  1  thì ta được 2 f  2   8 f  4   1
f  4
1 1
Mà từ  *  ta suy ra 2.  8.  1  a  1  f  1   1.
4 7  5a  4 a2
Từ đây ta xong bước 1.
f x
Bước 2. Ta sẽ chứng minh f  x  n   , n  1,   3 
n 2
 2nx  f  x   1
Ta sẽ dùng phương pháp quy nạp để chứng minh khẳng định này.

Chinh phục olympic toán| 33


Bồi dưỡng học sinh giỏi

Từ  2  suy ra  3  đúng khi n  1. Giả sử đúng đến n  k.


f x  k
Thì ta có f  x  k  1 
1  2  x  k  f  x  k   1
f x


k 2
 2 kx  f  x   1

f x
 1  2  x  k  f  x 
 2

 1
  k  1 2

 2  k  1 x f  x   1
  k  2 kx  f  x   1 
 
Từ đây theo nguyên lý quy nạp ta suy ra điều phải chứng minh.
Vậy khẳng định ở bước 2 được chứng minh.
f  1 1 1
Vậy từ đó ta suy ra f  n  1   hay f  n   , n  1, 
n 2
 2n  f  1  1  n  1
2
n2

1 1
Bước 3. Ta sẽ chứng minh: f    n2  , n  1,   4 

TẠP CHÍ VÀ TƯ LIỆU TOÁN HỌC


2
n 1
 
n
1
f 
1  1 n
Thật vậy, trong  3  ta thay x  thì được f  n   
n  n
 n2  2  f  n1   1
1 1 1
Tiếp theo, ta thay y  vào  1  thì được f  x   f    2 
x x  1
f x  
 x
1 1 1
Vậy f  n   f    2   n2  2 
n  1 1
f n   f 
 n n
1 1
Mà ta có: f  n   2
nên suy ra: f    n2 .
n n
Nên từ đây bước 3 được chứng minh hoàn toàn.
Thử lại thì thấy hàm số này thỏa mãn yêu cầu bài toán.
1
Vậy tất cả các hàm số thỏa mãn yêu cầu bài toán là f  n   , n  *
 .
n2

34 | Tạp chí và tư liệu toán học


Phương trình hàm trên tập rời rạc


Câu 26. Cho hàm f :  là một hàm số thỏa mãn với mọi n  1 thì có một số
n
nguyên tố p là ước của n sao cho: f  n   f  p   f    1 và
 p
f  32018   f  52019   f  7 2020   2017.

Hãy tính giá trị của biểu thức G  f  20182018   f  2019 2019   f  2020 2020 
Doãn Quang Tiến
Lời giải
f  1
Thay n  p vào  1  thì ta được f  p   f  1  f  p   f  p  
2
Thay n bởi pn thì ta được f  pn   f  pn1   f  p 
Bằng phương pháp quy nạp thì ta chứng minh được.
 2n
f  pn     f  1  * 
CHINH PHỤC OLYMPIC TOÁN

 2 
Thật vậy, thì  *  đúng với n  1.
Giả sử  *  đúng với n  k  1.
Ta sẽ chứng minh  *  cũng đúng với n  k  1 hay ta có

2k f  1  2   k  1 
f  pk 1   f  p k   f  p     f  1    f  1
 2  2  2 
Từ đây ta suy ra  *  cũng đúng trong trường hợp n  k  1.
Vậy theo nguyên lý quy nạp thì  *  đúng với mọi n.
Khi đó thì ta suy ra
f  32018   f  52019   f  7 2020   2017
 2  2018   2  2019   2  2020 
  f  1    f  1    f  1   2017
 2   2   2 
6051 2
 f  1   2017  f  1    .
2 3
2 f  1 1
Từ f  1    và f  p   suy ra: f  p    .'
3 2 3
1 n 1
Khi f  p    thì  1  được viết lại như sau f  n   f    , n  2
3 p 3
n
Cho k là số các thừa số nguyên tố của n , khi đó số lượng thừa số nguyên tố của   là
p
 k  1 .
1 12
 Với k  1 thì ta chọn n  2 thì theo  2  ta được: f  2     .
3 3

Chinh phục olympic toán| 35


Bồi dưỡng học sinh giỏi

 Với k  2 thì ta chọn n  10  2.5 thì theo .  2  ta được:


.

 10  1 1 1 1 22
f  10   f     f  5       0 
 2  3 3 3 3 3

 Với k  3 thì ta chọn n  12  2.3.4 thì theo  2  ta được:

 12  1 1 6 1 1 1 1 1 1 1 1 32
f  12   f     f  6    f      f 2        
 2  3 3 3 3 3 3 3 3 3 3 3 3
Từ đó, bằng phương pháp quy nạp ta suy ra được:
k2
f n  , với k là số các thừa số nguyên tố của n  * * 
3
Mà lưu ý rằng 2018  2.1009, 2019  3.673, 2020  2 2.5.101.
Do đó suy ra: 2018 2018 có 4036 thừa số nguyên tố, 2019 2019 có 4038 thừa số nguyên tố,
2020 2020 có 8080 thừa số nguyên tố.

TẠP CHÍ VÀ TƯ LIỆU TOÁN HỌC


Từ đấy theo công thức  * *  ta suy ra được:
4036  2 4038  2 8080  2 16148
G  f  20182018   f  20192019   f  20202020      .
3 3 3 3
Vậy từ đấy ta có kết quả của bài toán.
Nhận xét. Một điều thú vị là khi ta thay 2017, 2018, 2019, 2020 bằng x  1, x , x  1, x  2 thì
2
ta được kết quả của f  1    không thay đổi.
3

Câu 27. Tìm tất cả các hàm số f : *


 *
thỏa mãn:
2 f 3  m2  n2   f 2  m  f  n   f  m  f 2  n  , m , n  *

Lời giải
Giả sử tồn tại hàm f thỏa mãn yêu cầu bài toán.
Trường hợp 1. Nếu f là hàm số hằng.
Tức là f  n   const  c , với c là hằng số thì hiển nhiên thỏa mãn yêu cầu bài toán.
Trường hợp 2. Nếu f không là hàm số hằng.
Nếu tồn tại m , n  *
sao cho f  m   f  n  thì ta gọi a , b là hai số thỏa mãn:
f  a   f  b   min f  m   f  n  , m, n  *
 1
Giả sử f  a   f  b  thì ta có 2 f 3  b   f 2  a  f  b   f  a  f 2  b   2 f 3  a 

Suy ra f  b   f  a2  b 2   f  a   f  a2  b 2   f  b   f  a   f  b 

Từ đó thì ta suy ra f  a   f  b   f  a   f  b   f  a 2  b 2   f  b   f  a 2  b 2   f  b  2


Rõ ràng thì ta thấy  2  mâu thuẫn với  1  .

36 | Tạp chí và tư liệu toán học


Phương trình hàm trên tập rời rạc

Do đó chỉ có f là hàm số hằng thỏa mãn yêu cầu bài toán.


Vậy tất các hàm số thỏa mãn đề bài là: f  n   const  c , với c là hằng số.


Câu 28. Giả sử f :  là hàm liên tục và giảm sao cho với mọi x , y  ta có
f  x  y  f  f  x   f  y   f  y  f  x 
Chứng minh rằng f  f  x    x
Lời giải

Cho y  x ta được: f  2 x   f  2 f  x    f 2 f  x  f  x   
   
Thay x bằng f  x  ta có f  2 f  x    f 2 f  f  x    f 2 f f  x   f  f  x   
Trừ hai phương trình trên ta suy ra:
     
f 2 f  f  x    f  2x   f 2 f f  x   f  f  x    f 2 f  x  f  x   
CHINH PHỤC OLYMPIC TOÁN

Nếu f  f  x    x , vế trái của phương trình trên âm, do đó:

 
f f  x   f  f  x    f  x  f  x   và f  x   f  f  x    x  f  x 

là điều mâu thuẫn.


Tương tự, ta cũng có điều mâu thuẫn xảy ra khi f  f  x    x
Vậy f  f  x    x , điều phải chứng minh.

Câu 29. Cho song ánh f :  . Chứng minh rằng tồn tại vô số bộ  a, b , c  với
a, b , c  thỏa mãn a  b  c và 2 f  b   f  a   f  c 
Lời giải
Ta xây dựng dãy an  như sau:

Trong các số từ 0, 1, 2,..., m chọn số a1 sao cho f  a1   f  i  i  0; a1  m   

Chọn a2  a1 sao cho f  a2   f  i  , i  0; a2

Chọn ak  ak  1 sao cho f  ak   f  i  , i  0; ak

Vậy ta có dãy a1  a2  ...  ak  ak  1 và f  a1   f  a2   ...  f  ak   f  ak 1 

Trong đó ai   và f  ai   f  j  j  0; ai

Vì f là song ánh nên f  ak  1   f  ak   p , p  N *

Và c  để f  c   f  ak  1   p  f  ak  1 

 ak  1  ai , i  1, k
Mặt khác 
 f  ak  1   f  i  ,  i  1, an  1

Chinh phục olympic toán| 37


Bồi dưỡng học sinh giỏi

 p  ak   f  ak  1   p
Nên c  ak  1    2 f  ak  1   f  a k   f  c 
 f  c   f  ak  1   p
Do cách xây dựng, dãy an  là dãy vô hạn nên tồn tại vô số bộ  a , b , c  thỏa điều kiện đã
nêu.

Câu 30. Có bao nhiêu hàm f : * * thoả mãn đồng thời các điều kiện sau
a) f  1   1

b) f  n  f  n  2   9  f  n  1    1997, n 
2
*.
Lời giải
Gọi D là tập hợp tất cả các hàm số f thoả mãn điều kiện bài toán.
Theo giả thiết b) ta có
f  n  f  n  2    f  n  1    1997 ;
2

TẠP CHÍ VÀ TƯ LIỆU TOÁN HỌC


f  n  1  f  n  3    f  n  2    1997
2

Suy ra f  n  f  n  2    f  n  1    f  n  1  f ( n  3    f  n  2    1997
2 2

f n  f n  2  f  n  1  f  n  3 
  , n  *.
f  n  1 f n  2
f  1  f  3  f 2  f  4 f n  f n  2 
Vì vậy ta có   ...   ...
f 2 f  3 f  n  1
f  1  f  3 
Đặt c   1 suy ra f  n  2   cf  n  1  f  n  , n  * 2
f 2
p
Ta chứng minh c  * . Thật vậy, nếu c  với p , q  và  p , q   1 thì từ  2  ta có
q
q  f  n   f  n  2    pf  n  1  , n  *
Suy ra q f  n  1  , n  * hay q 2 f  n  f  n  2  , n  * và n  2.


Vì 1997  f  n  f  n  2    f  n  1  
2
q.
2

Mà 1997 là số nguyên tố nên q 2  1 hay q  1 suy ra c  *


Gọi f  2   a , do  1  ta có ac  1  f  3  suy ra ac  1  f  3   f  1 f  3    f  2    1997
2

 ac  1  a 2  1997  a  c  a   1998
Ta được a 1998 , hay f  2  là một ước dương của 1998.
Ngược lại với mỗi ước dương a của 1998 ta xây dựng hàm f : *  * như sau
f  1   1; f  2   a
1998
f  n  2    a  b  f  n  1   f  n  , n  *
; trong đó b   *.
a

38 | Tạp chí và tư liệu toán học


Phương trình hàm trên tập rời rạc

Ta chứng minh f thoả mãn điều kiện đề bài, nghĩa là f  D .


Thật vậy
f  n  1 f  n  3    f  n  2    f  n  1   a  b  f  n  2   f  n  1    f  n  2  
2 2

  a  b  f  n  1 f  n  2    f  n  1    f  n  2  
2 2

 f  n  2    a  b  f  n  1  f  n  2     f  n  1 
2

 f  n  2  f  n    f  n  1   , n 
2
*
Suy ra f  n  1  f  n  3    f  n  2    f  n  2  f  n    f  n  1  
2 2

...
 f  3  f  1   f  2    f  3    f  2  
2 2

Từ đó ta có f  n  f  n  2    f  n  1    f  3    f  2  
2 2

  a  b  f  2   f  1   f  2     a  b  a  1  a2
2
CHINH PHỤC OLYMPIC TOÁN

 ab  1  1998  1  1997.
Vậy ta được f  n  f  n  1    f  n  1    1997 hay f  D
2

Ta có tương ứng, mỗi f  D với một giá trị f  2  1998 là một song ánh giữa D và tập các
ước dương của 1998 .
Do đó số phần tử của D là: D  d  1998   d  2.33.37    1  1  3  1  1  1   16.
Vì vậy có tất cả 16 hàm số thoả mãn đề bài.

Câu 31. Tìm tất cả các hàm số f : *


 *
sao cho.
a) f  2   2
b) f  m.n   f  m  . f  n  với mọi m, n  *
, UCLN  m, n   1
c) f  m   f  n  m , n  *
,m  n.
Lời giải
Chọn n  1 , thay vào f  m.n   f  m  . f  n   f  1   1 .
Ta để ý rằng f  3  . f  5   f  15   f  2  . f  9   f  2  . f 10   f  2  . f  2  f  5  .
Vậy f  3   f  2  . f  2   4 . Mà 2  f  2   f  3   4 nên f  3   3 .
Từ đó ta tính được f  4   4, f  5   5, f  6   6, f 7   7, f 8   8, f 9   9, f 10   10 .
Dự đoán f  n   n với n  *
.
Giả sử f  k   k với k  *
,10  k  n . Ta chứng minh điều khẳng định vẫn còn đúng với
k  n1 .
Nếu k là số chẵn, ta xét hai trường hợp sau:
 k  2   2l  1 ;  , l  *
.

Chinh phục olympic toán| 39


Bồi dưỡng học sinh giỏi

   
Lúc này f  k   f 2  2l  1  f 2 f  2l  1  2  2l  1  k .
 k  2 ;   *

    
Lúc này f  k  2   f 2  2  f 2 2 1  1  f  2  f 2 1  1  2 2 1  1  k  2 .   
Mặt khác k  1  f  k  1  f  k   f  k  1  f  k  2   k  2 .
Do đó f  k   k , f  k  1  k  1 .
Nếu k là số lẻ thì k  1 là số chẵn, ta xét hai trường hợp sau:
 k  1  2  2l  1 ;  , l  *
. Khi đó 0  2  n,0  2l  1  n .

   
Theo giả thiết quy nạp f  k  1  f 2  2l  1  f 2 f  2l  1  2  2l  1  k  1 .

Mà k  1  f  k  1  f  k   f  k  1  k  1  f  k   k .
 k  1  2 ;   *
. Lúc này
 k  1  2   f  2 
     
 2  f 2 2 1  1  f  2  f 2 1  1  2 2 1  1  k  3 

TẠP CHÍ VÀ TƯ LIỆU TOÁN HỌC


f

Mặt khác k  1  f  k  1  f  k   f  k  1  f  k  2   f  k  3   k  3 .
Do đó f  k   k , f  k  1  k  1, f  k  2   k  2 .
Theo nguyên lí quy nạp f  n   n với n  *
.

Câu 32. Tìm tất cả các hàm số f :  thỏa mãn


f  m  n   f  mn   f  m  f  n   1, m , n   1
Lời giải
Thay m  n  0 vào  1  ta được 2 f  0   f 2  0   1  f  0   1
Thay m  1, n  1 vào  1  ta lại có f  1   f  1  f  1   1
Vậy f  1   0 . hoặc f  1   1
 Xét f  1   1 . Thay n  1 vào  1  ta có : f  m  1  f  m   f  m   1, m 
Suy ra f ( m  1)  1, m  hay f ( m)  1, m  . Thử lại thỏa mãn.
 Xét f  1   0
Thay n  1 vào  1  ta được f  m  1  f  m   1, m  2
Thay n  1 vào  1  ta lại được f  m  1  f  m   f  1 f  m   1, m 
Đặt a  f  1 thì phương trình trên trở thành f  m  1  f  m   af  m   1, m   3
+ Với a  2 . Nếu a  1 thì dẫn tới f  1   1 (trường hợp này đã giải ở trên)
Do đó ta xét a  1 .
 1 
Khi đó  3   f  m  1    a  1  f  m  
1
, m 
2a  2  a 

40 | Tạp chí và tư liệu toán học


Phương trình hàm trên tập rời rạc

 1 
 f  n    a  1  f  n  1 
1
, n 
2a  2  a 
n 1  1 
 f  n    a  1  f  1 
1
, n 
2a  2  a 
1   a  1
n 1

 f  n  , n   4
2a
1  bn 1
Đặt b  a  1  b  \0,1 , f  n   , n  .
1 b
1  b m 1  b1m
Thay vào  2  ta được   1, m   1  b m  1  b 1m  1  b , m 
1b 1 b
 bm  b1 m  b  1, m   5
Thay m  2 vào  5  ta được b2   b  1   b  1  b  1  0  b  1  b  1
1 2

b
1   1
n 1
CHINH PHỤC OLYMPIC TOÁN

Từ đó suy ra f  n   , n  , hay f  n   0 khi n lẻ và f  n   1 khi n chẵn.


2
Thử lại thỏa mãn.
+ Với a  2 thì f  1  2 .
Thay n  1 vào  1  ta được f  m  1  f  m   1, m  6
Từ f  1  2 và  6  ta dùng phương pháp quy nạp toán học thì sẽ chứng minh được
f  n   n  1, n  .
Thử lại thấy thỏa mãn. Vậy các hàm số thỏa mãn yêu cầu đề bài là
1   1
n 1

f  n  1 , f  n  , f  n   n  1 với n  .
2
Câu 33. Tìm tất cả các hàm số f :  thỏa mãn f  0   2 và

 
f x  f  x  2 y   f  2 x   f  2 y  , x , y   1
Lời giải
Thay x  y  0 vào  1  ta được f  f  0    2 f  0   f  2   4
Thay x  0 và y  1 ta được f  f  2    2  f  2   f  4   6
Ta sẽ chứng minh bằng quy nạp rằng với x  thì f  2 x   2 x  2 2
Theo trên thì  2  đúng khi x  0 .
Giả sử  2  đúng tới x  k  k  , k  0  .

  
Thay x  0, y  k vào  1  ta được f f  2 k   f  0   f  2k   f 2  k  1  2  k  1   2 
Vậy  2  cũng đúng khi x  k  1 , suy ra  2  đúng với x  , x  0.
Với x  , x  0, thay  x , y    2 x ,  x  vào  1  ta được

Chinh phục olympic toán| 41


Bồi dưỡng học sinh giỏi

 
f 2 x  f  0   f  4 x   f  2 x  , x  , x  0

   
 f 2  x  1  f 2  2 x   f  2 x  , x  , x  0
 2  x  1  2  2  2 x   2  f  2 x  , x  , x  0
 f  2 x   2 x  2, x  , x  0
 f  2 x   2 x  2, x  Z , x  0
 f (2 x)  2 x  2, x  Z
 
Dó đó (1)  1  f x  f  x  2 y   2 x  2 y  4, x , y   3
Ta sẽ chứng minh nếu x là số nguyên lẻ thì f  x  cũng là số nguyên lẻ.
Thật vậy, nếu f  x   2 k , với k  , thay x  x  2 k (với x lẻ) và y  k ta được :


f x  2k  f  x    2  x  2 k   2 k  4, x  , x lẻ

 f  x   2 x  2 k  4, x  , x lẻ

TẠP CHÍ VÀ TƯ LIỆU TOÁN HỌC


 2 k  2 x  2 k  4, x  , x lẻ
Do đó 4 k  2 x  4, với mọi số nguyên lẻ x.
Điều này vô lí vì 2 x  4 không phải lúc nào cũng chia hết cho 4.
Như vậy nếu x lẻ thì f  x  lẻ.
Từ đó nếu x lẻ thì x  2 y lẻ, dẫn đến f  x  2 y  lẻ, do đó x  f  x  2 y  chẵn

 
Do đó f x  f  x  2 y   x  f  x  2 y   2 .

Kết hợp với  3  ta được x  f  x  2 y   2  2 x  2 y  4  f  x  2 y   x  2 y  2, x , y  , x lẻ


 f  x   x  2, x 
Thử lại thỏa mãn.

Câu 34. Tìm tất cả hàm số f :  sao cho f  f  n    f  n   2n  3, n   1


Lời giải
Giả sử tồn tại hàm số f  n  thỏa mãn yêu cầu bài toán.

Cho n  0 , từ  1  có f  f  0    f  0   3  0  f  n   3 2
Nếu f  0   0 thì f  f  0    f  0   0 mâu thuẫn  2  . Vậy f  0   0

Nếu f  0   2 thì từ  2  ta có

f  f  0    1  f  2   f  f  0    1  f  1   f  f  2    2.2  3  f  2   6

 f  6   f  f  1    2.1  3  f  1   1

Suy ra f  6   , loại.

Như vậy f  0   2 . Tương tự cũng có f  0   3 .

42 | Tạp chí và tư liệu toán học


Phương trình hàm trên tập rời rạc

Do đó f  0   1 . Khi đó từ  2  ta có

f  1   f  f  0    2  f  2   f  f  1    2.1  3  f  1   3

Ta sẽ chứng minh hàm cần tìm là f  n   n  1, n  bằng quy nạp toán học.

Thật vậy. Với n  0 thì f  0   1  0  1 .

Giả sử khẳng định đúng tới n  k ,( k  ) . Tức là: f ( k )  k  1


Với n  k  1 ta có f  k  1   f  f  k    2 k  3  f  k   2 k  3   k  1    k  1   1

Vậy khẳng định đúng với n  k  1


Theo nguyên lý quy nạp toán học, ta có f  n   n  1, n  .

Thử lại hàm tìm được thỏa mãn yêu cầu bài toán.

Câu 35. Chứng minh rằng tồn tại duy nhất hàm số f : * 
CHINH PHỤC OLYMPIC TOÁN

* thỏa mãn

f  m  f  n    n  f  m  b  , m , n  * b   i
Lời giải
Giả sử tồn tại hàm số f  n  thỏa mãn yêu cầu bài toán.

Ta chứng minh f là đơn ánh.


Thật vậy, giả sử f  n1   f  n2  , n1 , n2  *.

Từ  i  ta có f  m  f  n1    f  m  f  n2    n1  f  m  b   n2  f  m  b   n1  n2

Vậy f là đơn ánh.


Với n  * , ta có f  f  1   f  n    n  f  f  1   b   n  1  f  b  b   f  b  f  n  1  

 f  n  1   b  f  n   f  1  , vì f là đơn ánh

 f  n  1   f  n   f  1   b  a , với a  f  1   b .

Suy ra f  n   f  n  1   ...  f  2   f  1   f  1   b  a , n  *

Từ đó: f  n   b  na  f  n   na  b , n  *.

Lúc này f  m  f  n    n  f  m  b   f  m  na  b   n   m  b  a  b

  m  na  b  a  b  n  ma  ba  b

 na 2  n  a 2  1  a  1 , vì nếu a  1 thì f  n   * khi n  b .

Suy ra f  n   n  b , n  * b  .
Thử lại hàm vừa tìm được thỏa mãn yêu cầu đề.

Chinh phục olympic toán| 43


Bồi dưỡng học sinh giỏi

Câu 36. Hãy xác định tất cả hàm số f : *


 *
thỏa mãn đẳng thức:
f  n   f  n  1  f  n  2  . f  n  3   a  1
Với a là số tự nhiên thỏa mãn a  1 là số nguyên tố
Lời giải
Giả sử phương trình có nghiệm.
Thay n bởi  n  1  lúc đó  1  trở thành f  n  1   f  n  2   f  n  3  f  n  4   a 2
Lấy  2    1  vế với vế ta được: f  n  2   f  n   f  n  3   f  n  4   f  n  2    3 
Thay n bởi 1 lúc này  3  trở thành: f  3   f  1   f  4   f  5   f  3  
Thay n bởi 2 lúc này  3  trở thành: f  4   f  2   f  5   f  6   f  4  
Thay n bởi 3 lúc này  3  trở thành: f  5   f  3   f  6   f  7   f  5  
Thay n bởi 4 lúc này  3  trở thành: f  6   f  4   f  7  .  f  8   f  6  
Từ đây ta đã nhận ra quy luật đặc biệt của bài toán này đó chính là nếu thay n bởi số lẻ thì

TẠP CHÍ VÀ TƯ LIỆU TOÁN HỌC


ta luôn biểu thị được: f  3   f  1   f  4  . f  6  ... f  2n  .  f  2n  1   f  2n  1    4 
Nếu thay n bởi các số chẵn ta sẽ được một đặc biệt khác
f  4   f  2   f  5  f  7  ... f  2n  1   f  2n  2   f  2n    5 
 Nếu f  1   f  3  thì lúc này ta sẽ lậy tức có ngay f  2n  1   f  2n  1  lúc này sẽ có
vô số số bé hơn f  1  mà f  1  là 1 số hữu hạn. Suy ra vô lí. Tức là f  3   f  1  .
Tương tự ta cũng sẽ có f  4   f  2  .
 Nếu f  1   f  3  và f  2   f  4  lúc này f  2n  1   f  2n  1  và f  2n   f  2n  2  .
Suy ra f  3   f  1  và f  4   f  2  sẽ có vô số ước số khác nhau (vô lí).
Từ đó chúng ta sẽ có 3 trường hợp
 f  3   f  1  f  2n  1   f  2n  1   f  1  f  2n  1
Trường hợp 1.   
, lúc này
 f  4   f  2   f  2n  2   f  2n   f  2   f  2n 
Thay bởi 1 vào phương trình  1  lúc này f  1   f  2   f  3  . f  4   a
 f  3   f  1
Mà lại có   f  1  f  2   f  2  . f  1  a   f  1  1  f  2   1  a  1
 f  4   f  2 
  f  1  2  2 khi n  2 k
  f n  
  f 2  a   a khi n  2 k  1
Mà lại có a  1 là số nguyên tố nên ta có  
  f  1  a  f n   a khi n  2 k
 f
  
   2   2  2 khi n  2 k  1
 f  1   f  3 
Trường hợp 2.  lúc này f  1   f  2n  1  với mọi n  *
.
 f  2   f  4 

44 | Tạp chí và tư liệu toán học


Phương trình hàm trên tập rời rạc

 f  2n  2   f  2n 
Thay vào  5  ta được 
 f  4   f  2    f  1    f  2n  2   f  2n  
n1

Lúc này f  4   f  2  có vô số các ước số nguyên dương đôi một khác nhau, điều này
không thể xảy ra nên phải có f  1   1 .
Thay n bởi 1 vào phương trình  1  thì ta sẽ có được f  1   f  2   f  3  . f  4   a
Mà lại có f  2 n  1   1
1  n  2 k  1 

 f  4   f  2   a  1  f  2n  2   f  2n   a  1  f  n     a  1  n  2 
x  n  2k 
 2
Với x  *

 f  2   f  4 
Trường hợp 3. Nếu  .
 f  1   f  3 
CHINH PHỤC OLYMPIC TOÁN

1 khi n  2 k

Lập luận tương tự như trên f  n     a  1 n  1 khi n  2 k  1 trong đó y 
*
.
y 
 2
Câu 37. Tìm tất cả các hàm số f : *
 *
thỏa mãn f t  n    a  1  . f  n   an   t  a  k

 
với f t  n   f f  ... f  n   với a , t là số tự nhiên tùy ý thỏa mãn k  2t  1   a  1 .
t

Lời giải
Đặt f  n   n  k  an . Ta sẽ chứng minh an  0 với mọi số tự nhiên dương n bất kì
Từ giả thiết ta suy ra
an  t  a  1 k n   t  a  1 k
 a  1 f  n   an   t  a  k  f  n    n  1
a1 a1 a1
Với n  2 a  2  tk thì
n  t  1  a k
n   t  1  a  k  2 a  2  ak  k   2  k  f n  n  k  2
a1
Như vậy với n  2 a  2  tk  f n   n  k  1
Khi đó với n  2 a  1  2 kt  k  t thì f  n   n  k  1  2 a  2 kt  2 k  t  2 a  2  tk lúc đó ta

 
cũng có f f  f  2    f  f  n    k  1  f  n   2  k  1 
Tiếp tục làm như thế cho đến lần thứ t  1 lúc đó ta sẽ có
f t 1  n   n   t  1  k  1   2 a  2  tk

 
Và cuối cùng f t   n   f f t 1  n   f t 1  n   k  1  n  t  k  1 
Lại có với n   t  1  k thì  a  1  f  n   f t   n    a  1  f  n   n  t  k  1    a  1  f  n 
Mà f t   n   an   t  a  1  k   a  1  f  n  nên bất đẳng thức trên cho ta

Chinh phục olympic toán| 45


Bồi dưỡng học sinh giỏi

 a  1 n  k  an   an   t  a  1 k  n  t  k  1  97  n  2  an   2 
tk  n k  2t  1
Từ  1  ta có  a  1 an   a  1 k  n   t  1  a  k  an    1  k  2t  1   a  1 
a1 a1
Mà từ  2  ta có
t
 a  1 an   a  1 k  t  k  1   t  a  1 k   a  1 an  t  an   an  0
a1
Từ đó ta sẽ có an  0 .
Xét với n   t  1  k . Ta sẽ chứng minh phần này của bài toán bằng phương pháp quy nạp.
Giả sự bài toán đúng đến m  n . Khi đó
f t 1  n   t  1 k   f t  2   n   t  2  k   f t  3  n   t  3  k   ...  f  n  2   n

 
f  n   f f t 1  n   t  1  k   f t  n   t  1  k   a  n   t  1  k    t  1  a  k   a  1  f  n  t  1  k 

Theo giả thiết quy nạp f (n  (t  1)k )  n  (t  2)k

TẠP CHÍ VÀ TƯ LIỆU TOÁN HỌC


Do đó f  n   an  a  t  1  k  tk  ak  k  an  a  t  2  k  n   t  2  k  n  k
Vậy f  n   n  k . Thử lại thấy thỏa mãn.

Câu 38. Cho hàm số f :  thỏa mãn:

 f  2 n  1   f  2 n   1   f  2 n  1   f  2 n   1   3  1  2 f  n  
 ,n
 f  2 n   f  n 
Tìm n sao cho f  n   2009 .
Lời giải
Giả sử tồn tại hàm f thỏa mãn đề bài
Vì 3  1  2 f  n   là số nguyên dương lẻ f  2n  1   f  2n   1 là số nguyên dương lẻ
 f  2n  1  f  2n   2  f  n 
 f  2 n  1   f  2 n   1  1
 f  2n  1   f  2n   1  2 f  n   1  
 f  2 n  1   f  2 n   1  3  1  2 f  n  
 f  2 n  1   f  2 n   2  f  2 n  1   f  2 n   2
 
2  f  2 n   1   2  3 f  n   1   f  2 n   3 f  n 
Ta sẽ chứng minh f  n   f  n  1  (1)
Với n  1  f  1   f  0   2  f 0 
Giả sử f  n   f  n  1  đúng tới k ( k  N *)  f  0   f  1   f  2    <f  k 
 Nếu k chẵn. Đặt k  2m  m  *   f  k  1   f  2 m  1  f  2 m   2  f  2m  f  k 
 Nếu k lẻ. Đặt k  2m  1  m  
 f  k  1   f  2m  2   3 f  m  1   3 f  m   2  f  2m   2  f  2m  1   f  k 
Như vậy trong mọi trường hợp khẳng định (2) đúng

46 | Tạp chí và tư liệu toán học


Phương trình hàm trên tập rời rạc

Do đó f  0   3 f  0   f  0   0, f  1   2; f  2   3 f  1   6
f  3   f  2   2  8, f  13   f  12   2  9 f  3   2  74, f  27   3 f  13   2  224
f  53   9 f  13   2  668, f  107   3 f  53   2  2006, f  108   9 f  27   2016
 f  107   2009  f  108 
Do f  n   2009  n  0; 1; 2;...; 107

Câu 39. Tìm tất cả các hàm số f :  thoả mãn:


1 1 1
f  xy   f  xz   f  x  f  yz   , x , y , z  .
3 3 9
Lời giải
2
1 1 1  1 1
Cho x  y  z  0 thì f 0  f 0  f 2 0    f 0    0  f 0  .
3 3 9  3 3
2
1 1 1  1 1
Cho x  y  z  1 thì f  1  f  1  f 2  1    f  1    0  f  1  .
CHINH PHỤC OLYMPIC TOÁN

3 3 9  3 3
2 1
Cho y  z  0 thì f  0   f  x  f  0   .
3 9
1 1
Do f  0   nên f  x   , x  .  1
3 3
1 1 1
Cho y  z  1 , ta có f  x   f  x   f  x  f  1  .
3 3 9
1 1
Do f  1  nên f  x   , x  . 2
3 3
1
Từ  1  và  2  ta được f  x   , x  .
3

Câu 40. Cho n   n  2  và hàm số f:  sao cho:


f  x n  y   x n1 f  x  f  f  y   ; x , y  *
a) Giả sử rằng f  2002   0. Tính f  2002  .
b) Tìm hàm số f .
Lời giải
a) Từ  *  ta được
 Với x  0; f  y   f  f  y   , y 
 Với x  1; y  0 : f  f 0    0  f 0   0.
 Với x  1, y  : f  1  y   f 1   f  y . 1 
Do đó, chứng minh bằng quy nạp ta được f  n   nf  1  , n  2
Từ  1  ta có f  0   f  1   f  1   f  1   f  0   f  1    f  t  ; f  x  1   f  x   f  1  .
Do đó, chứng minh bằng quy nạp ta được f  n   nf  1  , n  3

Chinh phục olympic toán| 47


Bồi dưỡng học sinh giỏi

Từ  2  ,  3  ta được f  n   nf  1  , n   4
p
Đặt f  1  ;p  , q  *
và ta được n  *
và n chia hết p nên nf  1   .
q
 f  1  0
Do đó ta được f  f  n    f  n   n  f  1   nf  1   f  1  f  1  
2 2
.
 f  1  1
Do đó, từ  4  ta được f  2002   1 hay f  2002   0 (loại).
Vậy f  2002   2002 .
b) Từ  *  ta được y  0 : f  x n   x n1 f  x  , x  1
f  xn  f  xn 
 Nếu n chẵn: x  0 : f  x   , f  x     f x
x n1 x n1
 Nếu n lẻ thì từ  *  và  1  ta được f  x n  y   f  x n   f  y  2

Suy ra f  x n
  f  x   0  f  x  
n
f   x     f  x    f  x  .
n
n

TẠP CHÍ VÀ TƯ LIỆU TOÁN HỌC


 x 
n1
x n1
Do đó f  x   f  x  , x 
Từ  2  , chứng minh bằng quy nạp ta được f  px n   pf  x n  , p  , x 
Có p  *
: f   px n    f  px n    pf  x n 
Vậy f  px n   pf  x n  , p  , x   3
u u  u..vn1   1 
Từ  3  ta có  
v
u  ,v *
 ta được f  
v
f n
n1
  uv f  n
 v  v 
 un   1   1  f  1
Mà f  1  f  n   vn f  n   f  n   n
v  v  v  v
u f  1 u
Vậy f    u.vn1 n  f  1  4
v v v
Ta có f  1   0 hay f  1   1 từ  4  suy ra f  x   0, x  hay f  x   x , x 
Thử lại thỏa mãn  *  .
Vậy f  x   0, x  .

Câu 41. Tìm tất cả các hàm số f :  thỏa mãn


f  x  y 2  z3   f  x   f 2  y   f 3  z  x , y , z 
Lời giải
Đặt P  u , v , t  là phép thế x bởi u , y bởi v , z bởi t , ta có:
 P  0, 0, 0   f  0   0

P  x , 1, 0   f  x  1   f  x   f  1 
2

48 | Tạp chí và tư liệu toán học


Phương trình hàm trên tập rời rạc

 f  1  0
 P  0, y , z   
 f  1  1
Với f  1   1 , ta chứng minh f  x   xf 2  1  , x 
Giả sử điều phải chứng minh đúng đến x  k , k  : f  k   kf 2  1 
Ta chứng minh đúng đến x  k  1 k  , thực vậy:
f  k  1   f  k   f 2  1   kf 2  1   f 2  1    k  1  f 2  1 
Với f  1   0 , ta chứng minh bằng quy nạp f  x   0 x 
Thế vào giả thiết ban đầu, ta nhận được hai hàm thỏa đề: f  x   0 và f  x   x x  .

Câu 42. Cho hàm số f : *


 *
thỏa mãn đồng thời hai điều kiện:
a) f  ab   f  a , b  f   a , b   với mọi a , b  *
, a  b ; trong đó  a , b  ,  a , b  lần lượt là bội
chung nhỏ nhất, ước chung lớn nhất của hai số nguyên dương a, b ;
CHINH PHỤC OLYMPIC TOÁN

b) f  p  q  r   f  p   f  q   f  r  với mọi số nguyên tố p , q , r .


Tính giá trị của f  2013  ? Kí hiệu *
là tập hợp tất cả các số nguyên dương.
Lời giải
Đặt f  2   a , f  3   b . Khi đó ta có các đẳng thức sau:
f 7   f  2  2  3  2 f  2   f  3  2 a  b
f  8   f  2  3  3   f  2   2 f  3   a  2b
f  16   f  7  7  2   2 f  7   f  2   2  2 a  b   a  5a  2b
f  16   f  2  f  8   a  a  2b  .
Do đó ta có 5a  2b  a 2  2 ab  1  .
Mặt khác ta có các đẳng thức sau:
f  12   f  2  3  7   f  2   f  3   f  7   3 a  2 b
f  12   f  2  f  6   a  f  2  2  2    3 a 2
Suy ra 3a  2b  3a 2  2  .
5a  2b  a2  2 ab a  2
Từ  1  ,  2  ta có   ...    f  7   7, f  8   8
3a  2b  3a b  3
2

Ta có 2003 là số nguyên tố nên


f  2013   f  2003  3  7   f  2003   f  3   f  7   f  2003   10  3 
f  2025   f  2003  5  17   f  2003   f  5   f  17   4 
f  9  3 f  3  9  f  5  2  2   f  5  2 f  2   f  5  5
f  17   f  7  7  3   2 f  7   f  3   17
Kết hợp với  4  ta được f  2003   f  2025   22  5 

Chinh phục olympic toán| 49


Bồi dưỡng học sinh giỏi

Mặt khác f  2025   f  9.9.25   f  9  f  9.25   9. f  5.5.9   9 f  5  f  45 


 9 f  5  f  3.15   45 f  3  f  15   45 f  3   f  7  5  3  
 45.3  f  7   f  5   f  3    2025
Do đó f  2025   2025 , kết hợp với (5) ta được f  2003   2003 .
Do đó từ đẳng thức  3  ta được f  2013   2013 .

Câu 43. Đặt F  f :  0, 1   0, 1 và n  2. Tìm giá trị nhỏ nhất của c thỏa mãn điều kiện

 f  x  dx  c  f  x  dx
1 1
n
0 0

Với f  F và f là hàm liên tục.


Lời giải

 f  x  dx  n  x  0, 1 , vậy c  n .


1 1 1 1
Ta có n
y n1 f  y  dy  n  x n1 f  x  dx  n  f  x  dx
0 0 0 0

TẠP CHÍ VÀ TƯ LIỆU TOÁN HỌC


n  p  1
Với p  0 , ta chọn hàm f  x   x p , khi đó: c 
n p
n  p  1
Do đó: c  lim n
p  pn
Vậy c  n , lại có n  2 nên giá trị c cần tìm là 2.

Câu 44. Tìm tất cả các hàm f :  1, 1  liên tục, thỏa mãn:
 2x 
f x  f  2 
, x   1, 1
 1 x 
Lời giải
2x
Đặt g  x   . Bài toán trở thành: f  x   f  g  x   với mọi 1  x  1 .
1  x2
Ta chứng minh bài toán nhỏ: Gọi dãy số an  g  an1  . Khi đó, với mọi giá trị dương
0  a1  1 , ta có: lim an  1
n 

2a
Chứng minh. Với mọi số thực 0  a  1 , ta có: a   a  g  a .
1  a2
Lại có, với mọi 0  a  1 , ta thấy 0  g  a   1 .

Vì vậy an n 1 là dãy tăng nghiêm ngặt và có giới hạn.


2L
Đặt lim an  L . Ta có: L  g  L   L1
n  1  L2
Do L  0 nên lim an  1
n 

50 | Tạp chí và tư liệu toán học


Phương trình hàm trên tập rời rạc

Vì f là hàm liên tục nên: lim f  1  h   f  1  , nói cách khác với mọi  0 luôn tồn tại 
h 0

sao cho với mọi 0  h   : f  1     f  1  


Đặt x  1   , từ chứng minh trên ta có các hàm x , g  x  , g 2  x  ,  đều tiến đến 1.
Vậy tồn tại số nguyên dương k sao cho g k  x   1   . Điều này đồng nghĩa:

f  g k  x    f  1 

 
Vì f  x   f g k  x  nên f  x   f  1   , do có thể chọn giá trị vô cùng bé, ta được:

f  x   f  1   0  f  x   f  1  với mọi 0  x  1 .
Chứng minh tương tự, ta được f  x   f  1  với mọi 1  x  0
Vì f là hàm liên tục nên lim f  x   f  0   f  1   f  1   f  0 
x 0

Vậy f  x   c với c là hằng số bất kỳ.


CHINH PHỤC OLYMPIC TOÁN

Câu 45. Có thể tồn tại hay không một hàm số f :  , liên tục trên và thỏa mãn
điều kiện: Với mọi số thực x , ta có f  x  là số hữu tỉ khi và chỉ khi f  x  1  là số vô tỉ.
Lời giải
Giả sử tồn tại hàm số liên tục f :  thỏa mãn điều kiện:
x  : f x   f  x  1  \ *
Xét các hàm số g  x   f  x  1   f  x  , h  x   f  x  1   f  x 
Khi đó g và h là những hàm số liên tục trên . Ta có g và h không thể đồng thời là
hàm hằng. Thực vậy, giả sử g  x   C 1 , h  x   C 2 . Khi đó:
2 f  x   C 2  C 1  f  x   C (C là hằng số)
Vì thế với   thì f     f    1   C , điều này mâu thuẫn với  *  .
Giả sử h không là hàm hằng, không mất tính tổng quát, khi đó tồn tại x1 , x2 , x1  x2 sao
cho h  x1   h  x2  .

Lúc này tồn tại số hữu tỉ r   h  x1  , h  x2   .

Ta có  h  x1   r   h  x2   r   0
Lại có h  x   r là hàm liên tục với mọi x thuộc nên phương trình h  x   r  0 có
nghiệm, tức là tồn tại x0  sao cho h  x0   r , từ đó f  x0  1   f  x0   r .
Mà r  nên f  x0  1  , f  x0  hoặc đồng thời hữu tỉ hoặc đồng thời vô tỉ, điều này mâu
thuẫn với  *  .
Vậy không tồn tại hàm số thỏa đề.

Chinh phục olympic toán| 51


Bồi dưỡng học sinh giỏi

Câu 46. Tìm tất cả các hàm số f :  thỏa mãn điều kiện f  x   f  t   f  y   f  z 
với mọi số hữu tỉ x  y  z  t và x , y , z , t theo thứ tự lập thành cấp số cộng.
USAJMO 2015
Lời giải
Do x , y , z , t theo thứ tự lập thành cấp số cộng nên x  t  y  z , kết hợp với phương trình đã
cho ta được f  x   f  y  z  x   f  y   f  z   1 với mọi x  y  z và x, y, z theo thứ tự
lập thành cấp số cộng.
Thay x  0 vào (1) ta được f  0   f  y  z   f  y   f  z  với mọi 0  y  z .
Đặt g  x   f  x   f  0  ta được g  y  z   g  y   g  z  0  y  z 2
Với số nguyên dương n  3 , ta sẽ tìm cách biểu diễn g  n  theo g  1 , g  2  .
Ta có g  3   g  1   g  2  , g  4   g  3   g  1   g  2   2 g  1  , dùng quy nạp ta chứng minh
được g  n   g  2    n  2  g  1  3 .

TẠP CHÍ VÀ TƯ LIỆU TOÁN HỌC


Lấy 2  y  z là các số nguyên dương ta được:
g  y   g  2    y  2  g  1 , g  z   g  2    z  2  g  1 , g  y  z   g  2    y  z  2  g  1 ,
Thay vào  2  ta được
g  2    y  z  2  g  1  g  2    y  2  g  1  g  2    z  2  g  1  g  2   2 g  1 .  * 
Kết hợp với  3  ta được g  n   ng  1 
Ta biểu diễn g  nx  theo g  x  , g  2 x  .
Bằng quy nạp ta dễ dàng chứng minh g  nx   g  2 x    n  2  g  x   4 .
Lấy 2  n  m là các số nguyên dương, thực hiện các thao tác như  *  ta được
g  2x    m  n  2  g  x   g  2x    n  2  g  x   g  2x    m  2  g  x   g  2x   2 g  x 
Kết hợp với  4  ta được g  nx   ng  x   5 
m m m
Vậy với m , n nguyên dương: g  m   ng    g    g  1  g  x   xg  1 , với x  

n n n
Do f  x   f  t   f  y   f  z   g  x   g  t   g  y   g  z  x  y  z  t lập thành CSC
Với x  0 bất kỳ, ta xét CSC x  0  y  z ta được
g  x   g  t   g  0   g  z   g  x   tg  1   g  0   zg  1   g  x   g  0   zg  1   tg  1   xg  1 
Vậy g  x   xg  1  , x   f  x   f  0   x  f  1  f  0    ax  b với x , a , b  .
Thử lại ta thấy thỏa mãn.

52 | Tạp chí và tư liệu toán học


Phương trình hàm trên tập rời rạc

Câu 47. Giả sử r , s  là hai số cho trước. Tìm tất cả các hàm số f :  thỏa mãn
điều kiện f  x  f  y    f  x  r   y  s , x , y 
Romania 2006
Lời giải
Thay x  0 vào phương trình ban đầu ta được
f  f  y    f  r   y  s , y   1
Thay y bởi f  y  thu được:

 
f x  f  f  y    f  x  r   f  y   s , x , y 

 f  x  f  r   y  s   f  x  r   f  y   s , x , y 
 f  x  r  y  f  r   r  s   f  x  r   f  y   s , x , y 
 f  x  y  f  r   r  s   f  x   f  y   s , x , y 
Đặt a  f  r   r  s thay vào phương trình trên ta được:
CHINH PHỤC OLYMPIC TOÁN

f  x  y  a   f  x   f  y   s , x , y  2
Thay y  0 vào phương trình (2) ta được: f  x  a  f  x   f  0   s, x   3 .
Từ  2  ,  3  ta được:
f  x  y   f  0   s  f  x   f  y   s , x , y 
 f  x  y   f  0   f  x   f  y  , x , y 
 f  x  y   f  0   f  x   f  0   f  y   f  0  , x , y 
Đặt f  x   f  0   g  x  , x  và thay vào phương trình trên ta được
g  x  y   g  x   g  y  , x , y 
 4
Từ  4  , theo kết quả về phương trình hàm Cauchy ta được g  x   bx , x , b 
Từ cách xác định hàm số g ta được f  x   bx  f  0   bx  c  5 
Từ  5  thay lại vào phương trình ban đầu ta được:
b  x  by  c   c  b  x  r   y  s  c , x , y 
 bx  b 2 y  bc  c  bx  y  br  s  c , x , y 
  b  r  s
 b2  1 
   b  1
bc  c  br  s  c  c  r  s
 
Vậy bài toán có hai nghiệm f  x   x  r  s , f  x   x  r  s , x  .

Chinh phục olympic toán| 53


Bồi dưỡng học sinh giỏi

Câu 48. Tìm tất cả các hàm số f :  sao cho với tất cả các số nguyên a , b , c thỏa
mãn a  b  c  0 , đẳng thức sau là đúng:
 f  a   f  b    f c   2 f  a  f  b   2 f  b  f c   2 f c  f  a 
2 2 2

IMO 2012
Lời giải
Lời giải 1. Tôn Ngọc Minh Quân
Giả sử hàm f :  thỏa mãn điều kiện đề bài.
Cho a  b  c  0 , ta được f  0   0 .
Cho a  n , b  n , c  0  n 
 ta được f  n   f  n  . Đặt f  1  t  t   .
Cho a  2, b  1, c  1 ta có f  2   0 hoặc f  2   4t .
 Trường hợp 1. f  2   0  f  3   t

Ta có  f  4     f  2     f (2)   2 f  2  f  4   2 f  2  f  4   2 f  2  f  2   f  4   0
2 2 2

TẠP CHÍ VÀ TƯ LIỆU TOÁN HỌC


Giả sử f  2i   0, f  2i  1   t  1  i  k 

  f  2k  2    f  2k    f  2   0  f  2k  2   0
2 2 2

Ta có  f  2 k  3     f  2 k     f  3    2 f  3  f  2 k  3  f  2 k  3   f  3   t
2 2 2

Vậy f  2 i   0, f  2i  1  t , i  N  f  2 i   0, f  2i  1  t , i 
 Trường hợp 2. f  2   4t  t  ,t  0 

Ta có  f  3     f  2     f  1    2 f  1  f  2   2 f  1  f  3   2 f  2  f  3 
2 2 2

Suy ra f  3   t hoặc f  3   9t
a) f  3   9t , f  2   4t , f  1   t . Ta chứng minh f  n   n 2 t , n  *

Thật vậy mệnh đề đúng với n  1, 2, 3 . Giả sử mệnh đề đúng đến n  3


Ta có  f  n  1     f  n     f  1    2 f  1  f  n   2 f  1  f  n  1   2 f  n  f  n  1 
2 2 2

  f  n  1    2t  n 2  1  f  n  1   t 2  n 2  1   0
2 2

 f  n  1  t  n  1 hoặc f  n  1   t  n  1 
2 2

Giả sử f  n  1   t  n  1   f  n  1 
2

Ta có  f  n  1     f  2     f  n  1    2 f  2  f  n  1   2 f  2  f  n  1   2  f  n  1  
2 2 2 2

  f  2    2 f  2   f  n  1  f  n  1 
2

 16t 2  8t.2  n  1  t  16t 2  16t 2  n  1  .


2 2

Đó là điều vô lý (vì n  3 ).
Vậy f  n   n2 t , n  *
 f  n   n 2t , n 
b) f  3   t , f  0   0, f  1   t , f  2   4t

54 | Tạp chí và tư liệu toán học


Phương trình hàm trên tập rời rạc

 f  4   f  2    f  2   2 f  2  f  2   2 f  2  f (  4 )  2 f  2  f  4 
2 2 2

 f  4   0 hoặc f  4   16t

Giả sử f  4   16t . Ta có  f  4     f  3     f  1    2 f  1  f  4   2 f  3  f  4   2 f  1  f  3 
2 2 2

 256t 2  2t 2  32t 2  32t 2  2t 2  192t 2  0 (vô lý).


Vậy f  4   0 .

Ta có  f  5     f  4     f  1   2 f  1  f  5   f (5)  f (1)  t ,
2 2 2

 f  6   f  4   f  2   2 f  2  f  6   f  6   f  2   4t ,
2 2 2

 f 7    f  4   f  3  2 f  3  f  7   f (7)  f (3)  t ,


2 2 2

 f  8   f  4    f  4   0  f  8  0
2 2 2

Bằng phương pháp quy nạp toán học ta chứng minh được
f  4 i  1   t i  N ; f  4 i  3   t i 
CHINH PHỤC OLYMPIC TOÁN

f  4 i   0  i  N ; f  4 i  2   4t  i 
Thật vậy giả sử f  4 k   0, f  4 k  1   t , f  4 k  2   4t , f  4 k  3   t  k  N 

Ta có  f  4 k  1     f  4 k     f  1    2 f  1  f  4 k   2 f  4 k  f  4 k  1   2 f  1  f  4 k  1 
2 2 2

 f  4k  1  f  1

 f  4k  2    f  4k    f  2   2 f  2  f  4k   2 f  4k  f  4k  2   2 f  2  f  4k  2 
2 2 2

 f  4 k  2   f  2   4t

 f  4k  3    f  4k     f  3   2 f  3  f  4k   2 f  4k  f  4k  3   2 f  3  f  4k  3 
2 2 2

 f  4k  3  f  3  t

 f  4k  4    f  4k    f  4   2 f  4  f  4k   2 f  4k  f  4k  4   2 f  4  f  4k  4 
2 2 2

 f  4k  4   f  4   0 .
Suy ra f  4i   0, f  4i  1   t , f  4i  2   4t , f  4i  3   t  t  ,t  0  i 
Ngược lại, giả sử hàm f :  thỏa mãn f  2i   0, f  2i  1   t  t   với mọi i 
Giả sử a , b , c  , a  b  c  0 . Suy ra trong 3 số a,b,c có ít nhất một số chẵn
+) Nếu a,b,c cùng chẵn thì f  a   f  b   f  c   0

  f  a     f  b     f c    2 f  a  f  b   2 f  b  f c   2 f c  f  a 
2 2 2

+) Nếu a chẵn và b,c lẻ thì f  a   0 , f  b   f  c   t

  f  a     f  b     f  c    2t 2
2 2 2

2  f  a  f  b   f  a  f  c   f  b  f  c    2t 2

  f ( a)    f  b     f  c    2 f  a  f  b   2 f  b  f  c   2 f  c  f  a 
2 2 2

Chinh phục olympic toán| 55


Bồi dưỡng học sinh giỏi

Tương tự nếu b chẵn a,c lẻ hoặc c chẵn a,b lẻ thì ta cũng có:
 f  a   f  b    f c   2 f  a  f  b   2 f  b  f c   2 f c  f  a 
2 2 2

Vậy hàm f :  sao cho f  2i   0 , f  2i  1   t  t   với mọi i  thỏa mãn điều


kiện đề bài.
 Xét hàm số f :  thỏa mãn f  n   n2 t  t  ,t  0  n 
Giả sử a , b , c  thỏa mãn a  b  c  0
Ta có f  a   a 2 t , f  b   b 2t , f  c   c 2t

Suy ra  f  a     f  b     f  c     a 4  b 4  c 4  t 2
2 2 2

Có a  b  c  0  a 2  b 2  c 2  2 ab  2bc  2ca
 a 4  b 4  c 4  2 a 2 b 2  2b 2c 2  2 a 2c 2  4 a 2 b 2  4b 2c 2  4 a 2c 2  8abc  a  b  c 
 a 4  b 4  c 4  2 a 2 b 2  2b 2c 2  2 a 2c 2
  f  a     f  b     f  c    2 a 2 b 2 t 2  2b 2c 2t 2  2 a 2c 2t 2
2 2 2

TẠP CHÍ VÀ TƯ LIỆU TOÁN HỌC


 2 f  a  f  b   2 f  b  f c   2 f c  f  a 
Vậy hàm f :  sao cho f  n   n2 t  t  ,t  0  n  thỏa mãn đề bài.
 Xét hàm f :  thỏa mãn
f  4i  1   t , f  4i  2   4t , f  4i  3   t , f  4i   0  t  ,t  0  i 
Giả sử a , b , c  sao cho a  b  c  0
  b  c  0  mod 4 
+ Nếu a  4i  i 
+ Nếu b,c đều chia hết cho 4 thì f  a   f  b   f  c   0

  f  a     f  b     f  c    2 f  a  f  b   2 f  b  f  c   2 f  c  f  a   0 
2 2 2

+ Nếu b  2  mod 4  và c  2  mod 4  thì f  a   0, f  b   4t , f  c   4t

  f  a     f  b     f  c    32t 2
2 2 2

2 f  a  f  b   2 f  b  f  c   2 f  c  f  a   32t 2

  f  a     f  b     f c    2 f  a  f  b   2 f  b  f c   2 f c  f  a 
2 2 2

+ Nếu b  1  mod 4  và c  3  mod 4  thì f  b   t , f  c   t

  f  a     f  b     f  c    2t 2
2 2 2

2 f  a  f  b   2 f  b  f  c   2 f  c  f  a   2t 2

  f  a     f  b     f c    2 f  a  f  b   2 f  b  f c   2 f c  f  a 
2 2 2

+ Nếu a  1  mod 4  , b  0  mod 4  và c  3  mod 4  , tương tự như trên ta cũng có:

 f  a   f  b    f c   2 f  a  f  b   2 f  b  f c   2 f c  f  a 
2 2 2

+ Nếu a  1( mod 4) , b  3  mod 4  và c  0  mod 4  , tương tự như trên ta cũng có:

56 | Tạp chí và tư liệu toán học


Phương trình hàm trên tập rời rạc

 f  a   f  b    f c   2 f  a  f  b   2 f  b  f c   2 f c  f  a 
2 2 2

+ Nếu a  1  mod 4  , b  2  mod 4  và c  1  mod 4 

 f  a   t , f  b   4t , f  c   t   f ( a)   f (b )   f (c )   18t 2
2 2 2

2 f  a  f  b   2 f  b  f  c   2 f  c  f  a   8t 2  8t 2  2t 2  18t 2

  f  a     f  b     f c    2 f  a  f  b   2 f  b  f c   2 f c  f  a 
2 2 2

+ Nếu a  1  mod 4  , b  1  mod 4  và c  2  mod 4  , tương tự như trên ta cũng có:

 f  a   f  b    f c   2 f  a  f  b   2 f  b  f c   2 f c  f  a 
2 2 2

+ Nếu a  2  mod 4  , b  0  mod 4  và c  2  mod 4  hoặc a  2  mod 4  , b  1  mod 4  và


c  1  mod 4  ; hoặc a  3  mod 4  , b  0  mod 4  và c  1  mod 4  hoặc a  3  mod 4  ,
b  1  mod 4  và c  0  mod 4  , tương tự như trên ta cũng có:

 f  a   f  b    f c   2 f  a  f  b   2 f  b  f c   2 f c  f  a 
2 2 2
CHINH PHỤC OLYMPIC TOÁN

+ Nếu a  3  mod 4  , b  3  mod 4  , c  2  mod 4  thì f  a   f  b   t , f  c   4t

  f  a     f  b     f  c    18t 2 ; 2 f  a  f  b   2 f  b  f  c   2 f  c  f  a   18t 2
2 2 2

  f  a     f  b     f  c    2 f  a  f  b   2 f  b  f ( c )  2 f  c  f  a 
2 2 2

Vậy tất cả các hàm f :  thỏa mãn đề bài là:


f:  : f  2i   0, f  2i  1   t  t   i 
f:  : f  n   n2 t  t  ,t  0  n 
f:  : f  4i   0, f  4i  1   t , f  4i  2   4t , f  4i  3   t  t  ,t  0  i  .
Cách 2.
Thay a  b  c  0 vào phương trình ban đầu ta được:
f 0  f  0  f  0  2 f 0 f 0   2 f 0  f 0   2 f 0  f 0   3 f 0   6 f 0   f 0   0
2 2 2 2 2

Thay b   a , c  0 vào phương trình ban đầu ta được:


f  a   f  a   f  0   2 f  a  f  a   2 f  a  f  0   2 f  0  f  a 
2 2 2

 f  a   f  a   2 f  a  f  a   f  a   2 f  a  f  a   f  a   0  f  a   f  a 
2 2 2 2

Ta có thể viết lại phương trình ban đầu dưới dạng:


f c   2 f c   f  a  f b    f  a  f b   0
2 2

2  f  a  f b   4  f  a  f b   4  f  a  f b 
2 2

 f  c   f  c   f  a  b  
2
 f  a  b  f  a  f b  2 f  a f b 
Nếu f  b   0 : f  a  b   f  a   f   a  mod  b  
Trường hợp 1. f  1   0  f  x   0x .

Chinh phục olympic toán| 57


Bồi dưỡng học sinh giỏi

Trường hợp 2. f  1   0, ta có f  2   f  1   f  1   2 f  1  f  1   f  2   0 or f  2   4 f  1 
Ta xét hai trường hợp nhỏ:
Trường hợp 2.1: f  1   0, f  2   0  f  x   f  x mod 2   f  x   f (1) nếu x lẻ và f  x   0
nếu x chẵn.
Trường hợp 2.2: f  1   0, f  2   4 f  1   f  3   f  2   f  1   2 f  2  f  1 
 f  3   5 f  1  4 f  1  f  3   f  1  9 f  1
 Nếu f  1   0, f  2   4 f  1  , f  3   f  1  :

 f  4   f  1   f  3   2 f  1  f  3  và f  4   f  2   f  2   2 f  2  f  2 
 f  4   f  1  hoặc 0 và f  4   16 f  1  hoặc 0
 f  4   0  f  x   f  x mod 4  .
 Nếu f  1   0, f  2   4 f  1  , f  3   9 f  1  :

TẠP CHÍ VÀ TƯ LIỆU TOÁN HỌC


 f  4   f  1  3   f  1   f  3   2 f  1  f  3   16 f (1) hoặc 4 f  1 

và f  4   f  2   f  2   2 f  2   16 f  1  hoặc 0.  f  4   16 f  1 
 Nếu x  4 , khi đó f  x   f  1  x 2
Dùng quy nạp, ta chứng minh: f  x   f  1  x 2 x
 Nếu . x  m ., khi đó f  x   f  1  x 2 x , đúng với một số giá trị m .
Giả sử điều phải chứng minh đúng với m  k :
 f  k  1   f  k   f  1   2 f  k  f  1   f  1  k  1  hoặc f  1  k  1 
2 2

Và f  k  1   f  k  1   f  2   2 f  k  1  f  2   f  1  k  1  hoặc f  1  k  3 
2 2

f  k  1   f  1  k  1  .
2

Vậy điều phải chứng minh đúng với m  k  1 .


Vì nó vẫn đúng với m  4 , theo quy nạp toán học ta có thể kết luận f  x   f  1  x 2 x .

 
Câu 49. Tìm tất cả các hàm f , g :  có đạo hàm trên thỏa mãn
g x f x
f ' x    ; g'  x    x  

x x
Lời giải
Ta có  x.  f  x   g  x    '  x  f '  x   g '  x    f  x   g  x 
 g x f x 
 x    f  x   g  x   0, x  0
 x x 
a
 x  f  x   g  x    ax  0  f  x   g  x    x  0  1
x

58 | Tạp chí và tư liệu toán học


Phương trình hàm trên tập rời rạc

 f x  g x 
Tương tự ta có   '  0, x  0  f  x   g  x   bx , x  0  x  0 2
 x 
1 a  1 a 
Từ  1  ,  2   f  x     bx  ; g  x     bx  , x  0, a , b 
2x  2x 

Câu 50. Tìm tất cả các hàm f : *


  có đạo hàm trên *
 thỏa mãn
f  xy   f  x   f  y  x , y  *
  1
Lời giải
Lấy đạo hàm hai vế  1  lần lượt theo biến x , y ta có
yf '  xy   f '  x  x , y  *

xf '  xy   f '  y  x , y  *

 x. f '  x   y. f '  y  x , y  *

CHINH PHỤC OLYMPIC TOÁN

 x. f '( x )  a x  *
  f  x   a.ln x  b x  R*
Thử lại  b  0 .
Vậy f  x   a.ln x x  *
 .
Câu 51. Tìm tất cả các hàm f :  thỏa mãn
f  f  n    n  b n   1
trong đó b là số nguyên dương chẵn.
Lời giải
Giả sử f thỏa mãn đề bài. Dễ thấy f đơn ánh
Trong  1  thay n bởi f  n  ta được f  f  f  n     f  n   b
Suy ra f  n  b   f  n   b n 

Vậy nếu m  qb  r , 0  r  b  1 thì f  m   f  qb  r   f   q  1  b  r   b  f  r   qb


Bây giờ ta chỉ cần xác định hàm f trên tập A  0, 1, 2,..., b  1 .
Xét x  A , đặt f  x   y thì f  y   x  b
Giả sử y  qb  r thì f  y   f  r   qb suy ra x  b  f  r   qb  qb  x  b  2b nên q  0
hoặc q  1 .
Do đó nếu x  A thì
i) f  x   r  b , r  A, f  r   x
ii) f  x   r , r  A, f  r   x  b
 f  qb  r   qb  f  r 

Vậy hàm f được xác định như sau  f  ai   bi với ai   bi   A .

 f  bi   ai  b

Chinh phục olympic toán| 59


Bồi dưỡng học sinh giỏi

 
Câu 52. Tìm tất cả các hàm f :  thỏa mãn:
i) f  xf  y    yf  x  x , y  
 1
ii) lim f  x   0
x 

IMO 1983
Lời giải
Thế x  1 ta được f  f  y    yf  1 
Nếu f  a   f  b  , thì af  1   f  f  a    f  f  b    bf  1   a  b vì f  1  tiến tới số nguyên
dương. Vì vậy hàm f đơn ánh.
Thế x  y ta được f  xf  x    xf ( x )
Vậy xf  x  là điểm bất động của hàm f .
Khi đó, thay y  1 thu được f  xf  1    f  x   f  1   1 vì x khác 0 nên 1 là điểm bất

TẠP CHÍ VÀ TƯ LIỆU TOÁN HỌC


động của hàm f. Ta chứng minh 1 là điểm bất động duy nhất của hàm f .
Giả sử tồn tại a,b là điểm bất động của f , ta có f  a   a và f  b   b .
Khi đó, thế x  a , y  b vào  i  ta được f  ab   f  af  b    bf  a   ab
Suy ra ab cũng là điểm bất động của f .
1 1  1  1 1 1
Thế x  , y  a thu được 1  f  1  f   a  f   f  a    af    f  
a a  a   a  a a
1
Vậy là điểm bất động của f .
a
 
Nếu f  a   a với a  1 , khi đó f an là điểm bất động của f , mâu thuẫn  ii 

 1 1
Nếu f  a   a với 0  a  1 , khi đó f  n   n là điểm bất động của f , mâu thuẫn  ii 
a  a
1
Từ các kết quả trên ta thu được điểm bất động duy nhất là 1, suy ra xf  x   1  f  a   .
x
 
Câu 53. Chứng minh rằng tồn tại song ánh f :  sao cho
f  3mn  m  n   4 f  m  f  n   f  m   f  n  m , n  

IMO Shortlist 1996


Lời giải
 x1
Xét hàm g : 3 
1 4 
 1 thỏa mãn: g  x   4 f    1 thì ta có g là song ánh và
 3 
g   3m  1  3n  1    g  3m  1  g  3n  1  m, n  
.
Thật vậy g   3m  1  3n  1    g  3  3mn  m  n   1   4 f  3mn  m  n   1
 4  4 f  m  f  n   f  m   f  n    1   4 f  m   1   4 f  n   1   g  3m  1  g  3n  1 
Vậy với mọi x , y  3 
 1 , ta có g  xy   g  x  .g  y 

60 | Tạp chí và tư liệu toán học


Phương trình hàm trên tập rời rạc

g  3x  1   1
Như thế, ta chỉ cần chỉ ra một song ánh g là đủ, vì khi đó f  x   thỏa mãn
4
đề bài.
Xét P1 , P2 là tập các số nguyên tố dạng 3 k  1, 3 k  2 tương ứng và Q1 , Q2 là tập các số
nguyên tố dạng 4 k  1, 4 k  3 tương ứng, ta xét song ánh h : P1  P2  Q1  Q2
Sao cho h  P1   Q1 , h  P2   Q2 và xác định g như sau:
 g  1  1
 nguyên tố thì g  n    h  pi 
Rõ ràng song ánh g như vậy thỏa mãn đề bài.

Câu 54. Tìm tất cả các hàm f :  thỏa:

 
3 f f  f  n    2 f  f  n    f  n   6n , n 
CHINH PHỤC OLYMPIC TOÁN

Lời giải
 
Đặt ak  f k  n   f f ...  f  n    (lặp k lần).
Ta lặp được dãy 3ak  2 ak 1  ak  2  6 ak  3 với mọi n
Xét tập S  a0 , a1 , a2 ,... vì đây là tập các số nguyên dương nên sẽ tồn tại một phần tử có
giá trị nhỏ nhất. Đặt j là số sao cho a j là giá trị nhỏ nhất trong tập S .
Ta có bất đẳng thức 3ak  2 ak 1  ak  2  6 ak  3  6 a j
Đẳng thức xảy ra khi ak  ak 1  ak  2  a j
Lại có khi đặt k  j  3 thì đẳng thức xảy ra, vậy ta có a j  a j  1  a j  2  a j  3 .
Tịnh tiến giá trị ta thu được ak  a j với mọi k  j .
3 ak  3  2 ak  2  a k  1
Thực hiện tương tự cho tịnh tiến lùi, lưu ý rằng ak  , ta thu được ak  a j
6
với mọi k  j . Vậy ak  a j với mọi k  0 .
Từ chứng minh trên dẫn đến a1  a0 hay f  n   n , n  .

Câu 55. Tìm tất cả các hàm số f :  0;     0;   thỏa mãn điều kiện:
f  f  x    yf  yf  x   x , y   0;    1 
Lời giải
f u
Với mọi t   0;   , ta chọn tùy ý một x 0 cố định và u  f  x0  , v  tf  x0  thì t 
f  v

1 1  f x 
Ta thay trong  1  y bởi ta có f  f  x    f
f y f y  f  y  
 

Chinh phục olympic toán| 61


Bồi dưỡng học sinh giỏi

 f x 
Hay f   f  y  . f  f  x   2 
 f  y  
 
1 1
Trong  1  , thay y bởi ta được f  f  x    f  1 x   0;    3
f x f x
 f x  f y
Từ  2  ,  3  suy ra f    f  1 .
 f y  f x
 
 f u  f  v
Do đó f    f  1 .
 f  v  f u
1
Do vậy, f  t   f  1  . t   0;   .
t
a
Thử lại, hàm cần tìm là f  x   x   0;   ở đó a  0 là hằng số.
x

TẠP CHÍ VÀ TƯ LIỆU TOÁN HỌC


Câu 56. Chứng minh rằng tồn tại duy nhất một hàm số f xác định trên tập các số thực
dương, nhận giá trị thực dương và thỏa mãn f  f  x    6x  f  x  .
Putnam 1988
Lời giải
Với mỗi số thực dương x 0 cố định, ta xây dựng dãy  f n n1 như sau:
f 1  x0 , f 2  f  x 0  , f n  1  f  f n  x 0   .
Khi đó, từ đẳng thức ở giả thiết ta suy ra dãy  f n n1 thỏa mãn phương trình truy hồi
f n 2  6 f n  f n1 ,
Hay f n  2  f n  1  6 f n  0.
Đến đây, giải phương trình đặc trưng của dãy  f n n1 , ta được hai nghiệm là 2 và 3 .

Do đó, f n  a  2 n  b   3  , trong đó, các hằng số a, b tìm được phụ thuộc vào f 1 , f 2 .
n

Tuy nhiên, nếu b  0 , thì tồn tại n đủ lớn sao cho f n  0 (ta có thể thấy được dễ dàng bằng
cách chọn n chẵn đủ lớn nếu b  0 , và chọn n lẻ đủ lớn nếu b  0 ).
Do vậy, b  0 . Thành ra f n  a  2 n
Suy ra f  f  x0    a.2 3 , f  x0   a.2 2 , thay hai giá trị này vào đẳng thức
f  f  x0    6 x0  f  x0 
Ta được 2a  x0 . Dẫn đến f  x0   2 x0
Và vì điều này đúng với mọi x 0 dương nên f  x   2 x , x  0 .

62 | Tạp chí và tư liệu toán học


Phương trình hàm trên tập rời rạc

Câu 57. Hàm số f :  thỏa mãn đồng thời các điều kiện sau:
 i  : f  f  n    n, n   1
 ii  : f  f  n  2   2   n, n  2
 iii  : f  0   1  3
Tìm giá trị f  1995  , f  2007 
Olympic Ukraine 1995
Lời giải
Cũng nhận xét và lý luận như các ví dụ trước, ta đưa đến f  n  phải có dạng: f  n   an  b
Khi đó điều kiện  i  trở thành: a2 n  ab  b  n , n 
 a2  1  a  1  a  1
Đồng nhất các hệ số, ta được   
 ab  b  0 b  0 b  0
a  1
Với  ta được f  n   n . Trường hợp này loại vì không thỏa mãn  ii 
CHINH PHỤC OLYMPIC TOÁN

b  0
 a  1
Với  ta được f  n   n  b
b  0
Từ điều kiện  iii  cho n  0 ta được b  1
Vậy f  n   n  1  4 
Hiển nhiên hàm số này thỏa mãn điều kiện bài toán.
Ta phải chứng minh f  n   n  1 là hàm duy nhất thỏa mãn điều kiện bài toán
Thật vậy giả sử tồn tại hàm g  n  khác f  n  cũng thỏa mãn điều kiện bài toán.
Từ  iii  suy ra f  0   g  0   1
Từ  iii  suy ra f  1   g  1   0
Sử dụng điều kiện  i  ,  ii  ta nhận được g  g  n    g  g  n  2   2  n 

   
Dno đó g g  g  n    g g g   n  2   2   n 

Hay g  n   g  n  2   2 n 
Giả sử n0 là số tự nhiên bé nhất làm cho f  n0   g  n0  5 
Do f  n  cũng thỏa mãn  4  nên ta có
g  n0  2   g  n0   2  f  n0   2  f  n0  2 
 g  n0  2   f  n0  2 
Mâu thuẫn với điều kiện n0 là số tự nhiên bé nhất thỏa mãn  5 
Vậy f  n   g  n  , n 
Chứng minh tương tự ta cũng được f  n   g  n  với mọi n nguyên âm.

Chinh phục olympic toán| 63


Bồi dưỡng học sinh giỏi

Vậy f  n   1  n là nghiệm duy nhất.


Từ đó tính được f  1995  , f  2007  .

Câu 58. Tìm f :  0, 1   thỏa mãn:


f  xyz   xf  x   yf  y   zf  z  x , y , z   0, 1 
Lời giải
 
Chọn x  y  z ta được f x 3  3xf  x 

Thay x , y , z bởi x ta được f  x   3x f  x 


2 6 2 2

Mặt khác f  x   f  x.x .x   xf  x   x f  x   x f  x 


6 2 3 2 2 3 3

Hay 3x f  x   xf  x   x f  x   3x f  x 
2 2 2 2 4

3x 3  1
 2 x 2 f  x 2   xf  x   3x 4 f  x   f  x 2   f  x  , x 

TẠP CHÍ VÀ TƯ LIỆU TOÁN HỌC


2
3x 9  1 3x 9  1
Thay x bởi x 3 ta được f  x 6   f  x 3  , x   3x 2 f  x 2   3xf  x  , x 
2 2
3x 3  1 3x 9  1
 3x 2 f x  3xf  x  , x   f  x   0, x  0
2 2
Vậy f  x   0 với mọi x   0;1  .

Câu 59. Tìm tất cả các hàm f xác định trên và thỏa mãn đồng thời các điều kiện sau:
2 f  n  f  k  n   2 f  k  n   3 f  n  f  k  , k  n

 f  1  1
Lời giải
Cho k  n  0  2 f 2  0   2 f  0   3 f 2  0   f  0   0  f  0   2
Nếu f  0   0 chọn n  0 ta được: 2 f  k   0 do đó f  k   0 với mọi k
Chọn k  1 ta được f  1   0 mâu thuẫn với giả thiết.
Vậy f  0   2
Chọn n  1 ta được phương trình 2 f  1  f  k  1   2 f  k  1   3 f  1  f  k  , k
 2 f  k  1   2 f  k  1   3 f  k  , k
Đặt xk  f  k  ta có phương trình sai phân 2 x k  1  3x k  2 x k 1  0
1
Phương trình đặc trưng là 2 2  3  2  0    2    
2
n
 1
Vậy f  n   c1 2  c 2    .
n

 2

64 | Tạp chí và tư liệu toán học


Phương trình hàm trên tập rời rạc

Ta tìm c 1 , c 2 từ điều kiện f  0   2, f  1   1 .


Dễ tìm được c 1  0, c 2  2
n
 1
Vậy f  n   2    .
 2

Câu 60. Tìm tất cả các hàm số f : *


 *
thỏa mãn đồng thời hai điều kiện sau:

 f  f n  n  2k ,  n 
 ,k
* *


 f  n  1  f  n  ,  n 
*

Lời giải
Giả sử có hàm f thoả mãn các điều kiện  1  ,  2  .
Ta chúng minh hàm f là một đơn ánh.
Thật vậy, với mọi m , n  *
và m  n , nếu có f  m   f  n  thì do  1  suy ra
CHINH PHỤC OLYMPIC TOÁN

f  f  m    f  f  n    m  2 k  n  2 k  m  n (vô lý)
Suy ra f là đơn ánh. Do  2   f  n  1   f  n  với mọi n  *
suy ra
f  n  1   f  n   1 với mọi n  *

 f  f  n  1    f  f  n   1   1  f  f  n   với mọi n  *

 f  f  n  1    1  f  f  n   với mọi n  *

 n  1  2 k  1  n  2 k với mọi n  *

 f  f  n  1    f  f  n   1  với mọi n  *

 f  n  1   f  n   1 với mọi n  *
( do f là đơn ánh)
 f  n   f  n  1   1 với mọi n  2 ; n  *

Truy hồi ta được


f  n   n  1  f  1  với mọi n  *

 f  f  n    f  n  1  f  1    n  2  2 f  1  với mọi n  *

 n  2 k  n  2  2 f  1  với mọi n  *

 f  1   k  1 với mọi n  *

Suy ra f  n   n  1  k  1 với mọi n  *


 f  n   n  k với mọi n  *

Thử lại f  n   n  k với mọi n  *


thỏa mãn các điều kiện  1  ,  2 
Vậy các hàm f cần tìm thỏa mãn đề bài là f  n   n  k với n  *
.

Chinh phục olympic toán| 65


Bồi dưỡng học sinh giỏi

Câu 61. Tìm tất cả các hàm số f :  thỏa mãn đồng thời hai điều kiện sau:
 f  2013   2016

 f  f  n    n  4, n 
Lời giải
Giả sử có hàm f thoả mãn các điều kiện  1  ,  2  .
Từ điều kiện  1  ta dễ dàng chứng minh được f là một đơn ánh.

 
Từ  1  ta suy ra f f  f  n    f  n  4  với mọi n 

 f  n   4  f  n  4  với mọi n  (3)


Với n  4 k  r với k  ; r  0, 1, 2, 3
Từ  3  ta suy ra f  4 k  r   4  4 k  f  r  với mọi k 
 Tính f  1  . Do 2013  4.503  1 nên f  2013   2012  f  1   2016  f  1   4

TẠP CHÍ VÀ TƯ LIỆU TOÁN HỌC


 Tính f  0  . Ta có f  f  0    4  f  1   f  0   1 (do f là đơn ánh)
 Tính f  2  và f  3  . Giả sử f  2   4m  r với m  và r  0, 1, 2, 3
Do  1  mà ta có f  f  2    6  2  4  f  f  2    f  4m  r   4m  f  r   6 ;
Mà f  r   0  m  0 hoặc m  1
+ Với m  0 , thì f  r   6 và f  f  2    f  r   f  2   r
+ Với m  1 , thì f  r   2 và f  2   4  r
 f  r   6
Trường hợp 1. Xét m  0  
 f  2   r  r  0, 1, 2, 3 
 f  0   6
+ Với r  0 thì  vô lý do f  0   1
 f  2   0
 f  1   6
+ Với r  1 thì  vô lý do f  1   4
 f  2   1
 f  2   6
+ Với r  2 thì  vô lý
 f  2   2
 f  2   6
+ Với r  2 thì  vô lý
 f  3   2
Vậy khi m  0 ta có f  0   1; f  1   2; f  2   3; f  3   6
 f  r   2
Trường hợp 2. Xét m  1  
 f  2   r  4  r  0, 1, 2, 3 
+ Với r  0 thì f  0   2 vô lý do f  0   1
+ Với r  1 thì f  1   2 vô lý do f  1   4

66 | Tạp chí và tư liệu toán học


Phương trình hàm trên tập rời rạc

 f  2   2
+ Với r  2 thì  vô lý
 f  2   6
 f  2   7
+ Với r  2 thì  vô lý
 f  3   2
Vậy khi m =1 ta có f  0   1; f  1   4; f  2   7; f  3   2
n  1 , n  0  mod 4

n  3 , n  1  mod 4
Suy ra f  n   
n  5 , n  2  mod 4
n  1 , n  3  mod 4

Thử lại: f  n  thỏa mãn đồng thời các điều kiện (1) và (2) nên f  n  là các hàm cần tìm.

Câu 62. Tìm tất cả các hàm số f :  *


thỏa mãn điều kiện sau:
CHINH PHỤC OLYMPIC TOÁN

f  n   f  n  1   f  n  1  . f  n  3  , n   1
Lời giải
Giả sử có hàm f thoả mãn các điều kiện  1  .
Ta lập dãy  an  với mỗi n  ta đặt an  f  n  khi đó  1  trở thành
an  an  2  an  1 an  3 , n  2
Thay n  n  2 trong phương trình  2  ta được an 2  an 4  an 3 an 5 , n   3
Trừ từng vế của  2  ,  3  ta được an 4  an  an  3  an  5  an  1  , n  .
 a4  a0  a3  a5  a1 

 a  a  a4  a6  a2 
Thay n lần lượt bởi 0, 1, 2, 3,...... ta có  5 1
 a6  a2  a5  a7  a3 
. . . . . . . . . . . . . . . .

Suy ra a4  a0  a3 a4 a5 .....an 2  an 4  an  , n  .  4 
Ta chứng minh rằng an  4  an , n   f  n  4   f  n  , n  .  5 
Hay f là hàm tuần hoàn với chu kỳ 4.
Thật vậy. Giả sử tồn tại số n0  *
mà an0  4  an0 ; n  n0

Do an  *
với n  nên an0  4  an0  1 suy ra

 
a4  a0  a3 a4 a5 .....an0  2 an0  4  an0  a3 a4 a5 .....an0 2

Do an  an  2  an  1 an  3 với n  , nên với 4 số liên tiếp an0 1 , an0 , an0  1 , an0  2 phải có ít nhất
một số lớn hơn 1.
Do đó khi n   thì a3 a4 a5 .....an0  2   suy ra an0  4  an0   (vô lý)

Chinh phục olympic toán| 67


Bồi dưỡng học sinh giỏi

Suy ra a4  a0  0  a4 n  an  0 với n   f  n  4   f  n  với r  0,1, 2, 3 trong đó f


là hàm tuần hoàn chu kỳ 4.
Hàm f được xác định khi ta tính được f  0   a0 ; f  1   a1 ; f  2   a2 ; f  3   a3 bởi vì từ
 a  a  a3 a4 a0  a2  a1 a3
2 ta có  0 2 
 a1  a3  a4 a2  a0 a2 a0 a2  a1  a3
 a0  1 a2  1  a0 a2   a0  a2   1
Mặt khác 
 a1  1 a3  1  a1 a3   a1  a3   1
Suy ra  a0  1  a2  1    a1  1  a3  1   2  6 
Do đó có các khả năng xảy ra.
 a0  1 a2  1  0  a1  2 v a1  3  a0  1 v a0  5
Khả năng 1.   
 a1  1 a3  1  2  a3  3 v a3  2  a2  1 v a 2  5
Suy ra  f  0  ; f  1  ; f  2  ; f  3     a0 ; a1 ; a2 ; a3    1; 2; 5; 3  ,  1; 3; 5; 2  ,  5; 2; 1; 3  ,  5; 3; 1; 2 

TẠP CHÍ VÀ TƯ LIỆU TOÁN HỌC


Ta tìm được 4 hàm cần tìm với f  n  với f  n  xác định bởi
f  0  , n  0  mod 4 

f  1 , n  1  mod 4 
f  n   7 
f  2  , n  2  mod 4 
f  3  , n  3  mod 4 

đều thỏa mà điều kiện 1.
 a0  1 a2  1  1
Khả năng 2.   a0  a1  a2  a3  2   a0 ; a1 ; a2 ; a3    2; 2; 2; 2 
 a1  1 a3  1  0
Suy ra f ( n)  2 với mọi n 
 a0  1 a2  1  2
Khả năng 3. 
 a1  1 a3  1  0
Lập luận tương tự như khả năng 1 ta được
 f  0 ; f  1 ; f  2  ; f  3     a ; a ; a
0 1 2 ; a3    2; 1; 3; 5  ,  3; 1; 2; 5  ,  2; 5; 3; 1  ,  3; 5; 2; 1 
Ta được 4 hàm f  n  cần tìm và cả 4 hàm đều thỏa mãn điều kiện  1  .
Với f  n  xác định bởi  7  .
Vậy có 9 hàm f thỏa mãn là nghiệm của phương trình  1  .

 
Câu 63. Tìm tất cả các hàm f :  thỏa mãn:
f  x  f  y   f  x  y   f  y 
Lời giải
Ta để ý rằng: f  a  f  b   f  c    fa  f  b   c  f  c   f  a  b  c   f  b   f c 

68 | Tạp chí và tư liệu toán học


Phương trình hàm trên tập rời rạc

Thế y bởi y  f  z  ta được:

 
f x  f  y  f  z  f  x  f  y  z   f  z   f  x  y  2z   f  y  z   f  z 

Và f  x  y  f  z    f  y  f  z    f  x  y  z   f  z   f  y  z   f  z 
Thu được: f  x  y  2 z   f  x  y  z   f  z  với mọi x , y , z  0

Với giá trị của y , ta chọn x sao cho x  y , f  y  , từ đó ta có kết quả f f  y   2 f  y   


Thế y  x vào phương trình ban đầu f  x  f  x    f  2 x   f  x 
Dễ nhận thấy f  x   x , vì vậy: f  x   f  f  x    f  2 x   f  x  , f  f  x    f  2 x 
Theo phương trình hàm Cauchy, ta được f  x   2 x với mọi x  
.

Câu 64. Tìm số nguyên dương m nhỏ nhất sao cho tồn tại hàm số f : *
 \1; 0; 1
thỏa mãn đồng thời các điều kiện sau
i) f  m   f  2015  , f  m  1   f  2016  ;
CHINH PHỤC OLYMPIC TOÁN

f n  1
ii) f  n  m   , n  1, 2,....
f n  1
Lời giải
1
Ta có f  n  2m     f  n  4m   f  n  , n  *

f  n
Với m  1 , ta có f  n  4   f  n   f  n  4 k   f  n  , k , n *

1 f  n  1
Ta có f  n  2    ; f  n  1  , n  *

f n f n  1
1
f  1  f  2015   f  4.503  3   f  3    : vô lý.
f  1
Với m  2 , ta có f  n  8   f  n   f  n  8k   f  n  , n , k  *

1 f  n  1
và f  n  4    ; f n  2  , n  *

f n f n  1
1
Ta có f  2   f  2015   f  251.8  7   f  7    ;
f  3
1
f  3   f  2016   f  251.8  8   f  8   
f  4
f 2  1
 f 2  f  4    f  2    1
2

f 2  1
Điều mâu thuẫn trên dẫn đến m  3.
Với m  3, ta xây dựng được vô số hàm f thỏa yêu cầu bài toán như sau

Chinh phục olympic toán| 69


Bồi dưỡng học sinh giỏi

1 a 1 f n  1
Cho a  \1; 0; 1 , đặt f  1   a ; f  2   ; f  3    ; và f  n  3   , n  1
1 a a f n  1
Khi đó, chứng minh quy nạp thì hàm số xác định trên *
và f  n   \1; 0; 1 , n  *

1
Hơn nữa theo chứng minh trên f  n  6    , f  n  12 k   f  n  , n , k  *

f  n
1 1  f 2 1
Khi đó f  2015   f  167.12  11   f  11        f  3
f  5 1  f  2  a
1 1  f  3 a  1
f  2016   f  167.12  12   f  12       f 4
f  6  1  f  3 a  1
Vậy hàm số thỏa mãn yêu cầu bài toán.

Câu 65. Xác định hàm số f  x  liên tục 


 
thỏa mãn đồng thời các điều kiện:
f  2 x   2 f  x  với mọi x  ,  1

TẠP CHÍ VÀ TƯ LIỆU TOÁN HỌC



   
f f 3  x  e f  x   1  x 2  e x  1  f  x  với mọi x  
, 2

 f  e  1   e  1 f  1 ,  3 
 f  k  là số nguyên dương với mọi số nguyên dương k ,  4 
Lời giải
Với a , b  
  
và f  a   f  b  , suy ra f 3  a  e    1  f 3  b  e    1 . Do đó
f a f b

f  f  a   e    1   f  f  b   e    1 
3 f a 3 f b

   
Hay a2 e a  1 f  a   b 2 e b  1 f  b  . Vì f  a   f  b   0 nên ta suy ra a 2 e a  1  b 2 e b  1 .    
Xét hàm số h  x   x 2 e x  1 trên   
 
, ta có h '  x   2 x e x  1  x 2 e x  0 với mọi x  
.

Do đó hàm số h  x   x 2 e x  1 đồng biến trên   


.

Do đó từ a 2 e a  1  b 2   e b
 1  , ta suy ra h  a   h  b  hay a  b .
Vậy f  x  là đơn ánh. Kết hợp với f  x  liên tục ta suy ra f  x  là hàm đơn điệu thực sự.
Mặt khác, theo giả thiết f  2   2 f  1  f  1 nên ta suy ra f  x  là hàm tăng thực sự trên

tập .


Từ  2  ta cho x  1 thì f f 3  1  e f  1  1   e  1  f  1  .  

Kết hợp với  3  ta suy ra f f 3  1  e f  1  1  f  e  1  .  
Vì f  x  là hàm tăng thực sự trên 
nên ta suy ra f 3  1  e  f  1

1  e1.

Xét hàm số g  x   x 3 e x  1 trên   



, ta có g '  x   3x 2 e x  1  x 3 e x  0 với mọi x   
.

Do đó hàm số g  x   x 3 e x  1 đồng biến trên   


.

70 | Tạp chí và tư liệu toán học


Phương trình hàm trên tập rời rạc

 
Do đó từ f 3  1  e f  1  1  e  1 , ta suy ra g  f  1    g  1  hay f  1   1 .

Vì f  2 x   2 f  x  với mọi x  
 
và f  1   1 nên theo quy nạp ta có f 2 n  2 n với mọi
số tự nhiên n .
Với mọi số tự nhiên n , ta có
2 n  f  2 n   f  2 n  1  f  2 n  2     f  2 n  2 n  1   f  2 n1   2 n1

     
Vì điều kiện  4  nên f 2 n  1 , f 2 n  2 ,  , f 2 n  2 n  1 đều là các số nguyên dương.

    
Do đó ta suy ra f 2 n  1  2 n  1, f 2 n  2  2 n  2, , f 2 n  2 n  1  2 n  2 n  1 . 
Vậy f  n   n với mọi số nguyên dương n .

Từ f  2 x   2 f  x  với mọi x  
 
. Ta suy ra f 2 n x  2 n f  x  với mọi x  
.
m m
Cho với mọi x  n
với mọi m , n là số nguyên dương ta suy ra f  m   2 n f  n  .
2 2 
CHINH PHỤC OLYMPIC TOÁN

m m m
Do đó m  2 n f  n  hay f  n   n mọi số nguyên dương m , n .
2  2  2
m
Với mỗi x  
tùy ý cho trước đều tồn tại dãy số q k  , q k có dạng hội tụ đến x .
2n

k  k 

Vì f  x  là hàm liên tục nên x  lim q k  lim f  q k   f lim q k  f  x 
k 

Thử lại ta thấy hàm số f  x   x thỏa mãn mọi điều kiện của bài ra.

Câu 66. Tìm tất cả các hàm f :  thỏa mãn đồng thời hai điều kiện sau:
 Với mọi cặp a, b nguyên dương không nguyên tố cùng nhau, có
f  a  . f  b   f  ab 
 Với mọi bộ a, b nguyên dương tồn tại một tam giác không suy biến có độ dài ba
cạnh là f  a  , f  b  và f  a  b  1  .
Lời giải
 f  a   f  b   f  a  b  1 ;

Từ điều kiện  2  , với mọi bộ a, b nguyên dương, ta có  f  a   f  a  b  1   f  b  ;

 f  a  b  1  f  b   f  a  ;

Nếu a  b  2 : f  4   f  2  ; 2 f  2   f  3  .
2

Nếu a  3; b  2 : f  2   f  3   f  4 

Ta có f  2   f  4   f  2   f  3   f  2   2 f  2   3 f  2   f  2   1 or f  2   2.
2

Nếu f  2   1 . Do 2 f  2   f  1   f  1   1.

Chinh phục olympic toán| 71


Bồi dưỡng học sinh giỏi

Quy nạp chứng minh f  n   1 với mọi n nguyên dương.


Cho a  n , b  2 : f  n  1   f  n   f  2   2  f  n  1   1 .
Nếu f  2   2 , bằng quy nạp chứng minh được f  2 k   f  2  . f  2 k 1   ....  f  2   2 k .
k

Do f  4   f  2   f  3   2 f  2   f  3   3
Quy nạp chứng minh f  n   n , n  2
Cho a  n  1, b  2 : f  n   f  n  1   f  2   n  1  f  n   n.
Lấy r là số nguyên lớn nhất sao cho 2 r không vượt quá n .
Nếu 2 r  n thì theo chứng minh trên có f  n   n .
Nếu n  2 r  s với 1  s  2 r .
Với a  n  2 r  s ; b  2 r  s  1. Ta có
f  2r  s  1  2 r  s  1  f  n   f  2 r  s  1  f  2 r  s  2 r  s  1  1

TẠP CHÍ VÀ TƯ LIỆU TOÁN HỌC


f  n   f  2r 1   f  2r  s  1  2 r 1   2 r  s  1  2 r  s  1  n  1  f  n   n
Vậy f  n   n , n  2 .
Do f  1   2 f  2   4 nên f  1  bằng 1, 2 hoặc 3 .
Vậy f  n   1 với mọi n nguyên dương hoặc f  n   n , n  2 ; f  1   1; 2; 3 .

Câu 67. Tìm các hàm số f :  1;    thoả mãn điều kiện:


f  x   f  y    y  x  f  xy  với mọi x , y  1  1 
Lời giải
Với mọi t > 1, thay  x ; y    t ; 2  ,  t ; 4  và  2t ; 2  vào  1  ta được:
 f  t   f  2    2  t  f  2t 

 f  t   f  4    4  t  f  4t   f  4    t  3  f  2   t  2t  5  f  4t  , t  1  2 

 f  2t   f  2    2  2t  f  4t 
5 1
Lấy t   f  4   f  2 
2 2
 5
Thay vào  2  ta được  t   f  2   t  2t  5  f  4t 
 2
5 f 2
Do đó với mọi t  1, t   f  4t  
2 2t
2 f 2 5
Từ  1  ta có f  t   f  4    4  t  f  4t   với t  1, t  .
t 2
5 5
Với t  , từ  1  thay x  , y  2 ta có:
2 2

72 | Tạp chí và tư liệu toán học


Phương trình hàm trên tập rời rạc

5 1 4 f 2 2 f 2 2 f 2


f    f  2   f  5    f t   , t  1
2 2 5 5 t
2
c
Đặt c  2 f  2   f  x   với x  1 .
x
Thử lại thỏa mãn điều kiện  1  .
c
Vậy hàm số cần tìm là f  x   .
x

Câu 68. Tìm tất cả các hàm f : *


 *
thỏa mãn đẳng thức:
ff 2
 m   2 f  n    m2  2n2 , với mọi m, n 
2 *
.
Lời giải
Nếu m1 , m2  *
sao cho f  m1   f  m2 
 f  f 2  m1   2 f 2  n    f  f 2  m2   2 f 2  n    m12  2n 2  m22  2n 2 ,
CHINH PHỤC OLYMPIC TOÁN

Suy ra m1  m2 hay f là đơn ánh.


Từ đó f  m   2 f  n   f  p   2 f  q   m2  2n 2  p 2  2q 2  1 
2 2 2 2

, n  3 ta có  n  2   2  n  1    n  2   2  n  1  2
2 2 2 2
Dế thấy với mọi n  *

Chú ý. Điều này vẫn đúng nếu ta nhân cả 2 vế với cùng một thừa số
 
Đặt f  1   a  f 3a 2  3 . Theo  1  suy ra:

f  5a  2 2
 2 f  a2   f  3a 2   2 f  3a 2   3 f  3a 2   27
2 2 2 2

Vì phương trình x 2  2 y 2  27 chỉ có nghiệm nguyên dương là  x ; y    3, 3  hoặc  5, 1 

   
nên ta có f a 2  1, f 5a 2  5 .

Cũng từ  1  ta có 2 f  4 a  2 2
 2 f  2 a2   f  5 a2   f  a 2   24 .
2 2 2

Vì phương trình x 2  y 2  12 chỉ có nghiệm nguyên dương là  x , y  là  4, 2  nên


f  4 a2   4, f  2 a2   2

Từ  1  ta có f   k  4  a2   2 f   k  3  a 2   2 f   k  1 a   f  ka 2  , suy ra từ khai triển


2 2 2 2 2

2
Vì vậy theo các kết quả trên và phép quy nạp ta suy ra f ka 2  k , với mọi k là số nguyên  
 
dương. Do đó f a3  a  f  1  mà f đơn ánh nên a 3  1  a  1 .
Vậy f  n   n với mọi n nguyên dương. Thử lại thỏa mãn bài toán.

Chinh phục olympic toán| 73


Bồi dưỡng học sinh giỏi

Câu 69. Tìm tất cả các số nguyên không âm n sao cho tồn tại một hàm f :   0;  
khác hằng thỏa mãn đồng thời 2 điều kiện sau
i) f  xy   f  x  f  y  , x , y 


ii) 2 f  x 2  y 2   f  x   f  y  x , y    0; 1; 2;...; n.
Lời giải
Với a  bất kì, bằng cách thay x  y  a k ; k  *
vào i) được

2 f  a   f  2  f  a   1  0; 1; 2;...; n  1 
k k
 
Nếu f  2   0 thì 2 f  a   0; 1; 2;...; n  f  a   0
k

 Nếu f  2   0 thì ta thấy f  a   0 hoặc f  a   1 .

Thật vậy, nếu f  a   1 thì bằng cách cho k   , ta thấy 2 f  a   f  2  f  a   1   .


k k
 
Nên  1  không thể xảy ra, còn nếu 0  f  a   1 thì với k đủ lớn ta có

TẠP CHÍ VÀ TƯ LIỆU TOÁN HỌC


2 f  a   f  2  f  a   1  0
k k
 
Nên  1  cũng không thể xảy ra.
Thành thử, ta đã chứng minh được với mọi a thì f  a   0 hoặc f  a   1 .

Từ đó suy ra  2 f  x 2  y 2   f  x   f  y    0; 1; 2 ; x , y  2


Do đó, n  2.
 
Nếu n  0, thì 2 f x 2  y 2  f  x   f  y  ; x , y  .
Vì f khác hằng nên tồn tại x0  sao cho f  x0   0 .
Khi đó f  x0   f  x0  f  1   f  1   1 .
Do f khác hằng nên tồn tại x1  sao cho f  x1   1 .
Từ i), ta có f  0   f  x1   f  0   f  0   0

 
Bây giờ, sử dụng  2  ta được 2  2 f 12  0 2  f  1   f  0   1 .
Điều vô lí này chứng tỏ n  0 không thỏa mãn.
0 if x  0
 Nếu n  1 thì hàm số f  x    .
1 if x  0
Thỏa mãn đề bài. Do đó n  1 thỏa mãn đề bài.
 Nếu n  2 thì ta thấy không thể tồn tại 2 số p , q  ;  p , q   1 sao cho f p 2  q 2  0 .  
Thật vậy, nếu trái lại, thì x , y  ta có

0  f  p2  q 2  f  x 2  y 2   f  p 2
 q 2  x 2  y 2   f  xp  yq  2
  xq  yp 
2

74 | Tạp chí và tư liệu toán học


Phương trình hàm trên tập rời rạc

Kết hợp với  2  suy ra f  xp  yq   f  xp  yq   0. Thế nhưng, do  p , q   1 nên tồn tại


x, y  để xp  yq  1. Do đó 1  f  xp  yq   0.

 
Điều vô lí này chứng tỏ f x 2  y 2  1; x , y  ;  x , y   1.

 p x
0 if 
 q x
Bây giờ, ta xét hàm số f  x    , trong đó p , q là 2 số nguyên tố phân biệt có
 p x
1 if 
 q x
dạng 4 k  3. Ta sẽ chứng minh hàm f  x  xây dựng như trên thỏa mãn 2 điều kiện:
i) f  xy   f  x  f  y  , x , y 


ii) 2 f  x 2  y 2   f  x   f  y  x , y    0; 1; 2;...; n.
 Kiểm tra điều kiện i)
CHINH PHỤC OLYMPIC TOÁN

 p xy
Nếu  thì hiển nhiên f  xy   0  f  x  f  y  .
 q xy
 pq x
Nếu  thì f  xy   1  f  x  f  y 
 pq y
 Kiểm tra điều kiện ii)


Vì f  x   0; 1 nên 2 f  x 2  y 2   f  x   f  y  x , y    0; 1; 2
2 f  1  p 2   f  1   f  p   1


Dễ thấy 2 f  p 2  q 2   f  p   f  q   2 nên 2 f  x 2  y 2   f  x   f  y  x , y 

  0; 1; 2
2 f  0   f  0   f  0   0
Vậy n  1, n  2 là tất cả các giá trị thỏa mãn đề bài.

Câu 70. Tìm tất cả các hàm số f : *  * thoả mãn điều kiện:


2 f  m2  n 2  
3
 f 2  m  . f  n   f 2  n  . f  m  , m , n  *

Lời giải
Giả sử tồn tại hàm số f thỏa mãn các yêu cầu của đề bài.
Nếu f  n   c , với c là hằng số thì hiển nhiên thỏa mãn điều kiện của bài toán.

Nếu tồn tại m, n  *


sao cho f  m   f  n  thì ta gọi a , b là 2 số thỏa mãn

f  a   f  b   min f  m   f  n  , m , n  *
 1
Giả sử f  a   f  b  . Ta có 2 f 3  b   f 2  a  . f  b   f 2  b  . f  a   2 f 3  a  .

   
Vậy f  b   f a 2  b 2  f  a   f a 2  b 2  f  b   f  a   f  b  .

Chinh phục olympic toán| 75


Bồi dưỡng học sinh giỏi

Từ đó f  a   f  b   f  a   f  b   f  a 2  b 2   f  b   f  a 2  b 2   f  b  2
Rõ ràng  2  mâu thuẫn với  1  .
Do đó f  n   c , với c là hằng số là tất cả các hàm cần tìm.

Câu 71. Tìm tất cả các hàm số f :  thoả mãn điều kiện:
 f 0  c

3 f n  1
 f  n  1  , n  *  1
3  f  n
Lời giải
1  
f n  f  n   tan f  0   tan
Từ  1  ta có f  n  1   3  6  f  1  6
1  
1 f  n  1  f  n  tan 1  f  0  tan

TẠP CHÍ VÀ TƯ LIỆU TOÁN HỌC


3 6 6
 
Do đó ta đặt f  0   c  tan  thì f  1   tan    
 6
   
f  1   tan tan      tan
6   6 6  2 
f 2   tan    
     6 
1  f  1  tan 1  tan     tan
6  6 6
 
Ta chứng minh quy nạp công thức f  n   tan     , n  2
 6
Thật vậy, với n  0,1, 2 công thức  2  đúng.
 n 
Giả sử f  n   tan    
 6 
  n  
f  n   tan tan      tan
6   6  6  
Ta có f  n  1    tan     n  1  
     6
1  f  n  tan 1  tan     tan
6  6 6
Hay  2  đúng với n  1.
 
Nghiệm của bài toán là f  n   tan     , n  .
 6
Câu 72. Tìm tất cả các hàm số f :  thỏa:
f  2 a   2 f  b   f  f  a  b   a , b 
International Mathematical Olympiad 2019
Lời giải – Trần Bá Đạt
Lần lượt thay a  0, b  0 ta được:

76 | Tạp chí và tư liệu toán học


Phương trình hàm trên tập rời rạc

f  0   2 f  b   f  f  b   , f  2 a   2 f  0   f  f  a   , với a , b 
Do đó f  2 a   2 f  a   f  0  , với mọi a 
Thay vào phương trình đề bài ta được 2 f  a   2 f  b   f  0   f  f  a  b  
Cho a  0, b  a  b trong phương trình trên ta được: f  0   2 f  a  b   f  f  a  b  
Do đó: f  a   f  b   f  a  b   f  0 
Đặt g  x   f  x   f  0  suy ra g  a   g  b   g  a  b   g  x   cx hay f  x   cx  dx  .
Thay vào phương trình ban đầu và cân bằng hệ số cho a  b ta thu được: c  d  0 hoặc
c  2.
Vậy f  0 hoặc f  x   2 x  d .

Câu 73. Có tồn tại hay không hàm số f :  sao cho


f  m  f  n    f  m   n , m , n   1
CHINH PHỤC OLYMPIC TOÁN

Lời giải
Giả sử tồn tại hàm f thoả mãn đề bài.
Từ  1  cho m  0 ta có f  f  n    f  0   n  2  .
Với n1 , n2  mà f  n1   f  n2  thì f  f  n1    f  f  n2  
Từ  2  suy ra f  0   n1  f  0   n2 , do đó n1  n2 nên f là đơn ánh.
Cho n  0 từ  1  ta có f  m  f  0    f  m   m  f  0   m
Từ đó ta được f  0   0 thay vào  2  có f  f  n    n , n   3 .
Từ  1  thay m bằng f  m  và áp dụng  3  ta được f  f  m    f  n   m  n.
Xét m , n , p , q là các số nguyên sao cho m  n  p  q , khi đó
f  f  m   f  n    m  n   p  q  f  f  p   f  q  
Theo chứng minh trên f là đơn ánh, nên suy ra f  m   f  n   f  q   f  p 
Do đó với mọi n  ta có
f  n  1  f  n  1  f  n   f  n   f  n  1  f  n   f  n   f  n  1
 f  n  1   f  n   f  n   f  n  1   ...  f  2    1   f  1   f  0 
Nên  f  n  là một cấp số cộng với số hạng đầu là U 1  f  0   0 và công sai d  f  1  suy
ra f  n   U n 1  U 1  nd  nd , n  0.
Ta xét với hai số n  0, m  0 sao cho m  nd  0 thay vào  1  được
f  m  f  n    f  m  nd   f  m   n  f  m  nd   md  n   m  nd  d  md  n
Từ đó có d 2  1 , điều này vô lý do vậy không tồn tại hàm f thoả mãn yêu cầu của đề bài.

Chinh phục olympic toán| 77


Bồi dưỡng học sinh giỏi

Câu 74. Cho hàm số f :  là hàm số thỏa mãn các điều kiện sau:
i) f  mn   f  m  f  n  , m , n 
ii)  m  n  là ước của f  m   f  n  với mọi m , n 
Chứng minh rằng tồn tại một số tự nhiên lẻ k sao cho f  n   n k , n  .
India National Olympiad 2018
Lời giải
Gọi P  x , y  là phép thế m  x , n  y vào điều kiện i)
và Q  x , y  là phép thế m  x , n  y vào điều kiện ii)
Thế P  1, 1  thì ta được f  1.1   f  1  f  1   f  1   1 vì f 
Thế Q  2, 2  thì ta được:

 2  2   f  2   f  2    2 f  2   f  2   2 k q , k  ,  2, q   1

TẠP CHÍ VÀ TƯ LIỆU TOÁN HỌC


Giả sử ta xét với q  1 thì tồn tại một số nguyên tố p sao cho p q suy ra p là một số
nguyên tố lẻ.
Từ f  2   2 k q , k  ,  2, q   1 nên ta suy ra: p f  2  .

 p1   p  1   p1
Thế P  2,  thì ta được f  2.     f  p  1  f  2  f  
 2    2   2 
Từ p f  2  nên ta suy ra: p f  p  1  .
Thế Q  1, p  1  thì ta được:
1   p  1 f  1  f  p  1  p f  1  f  p  1  p 1  f  p  1  p 1
Điều này là hoàn toàn vô lý, do đó ta phải có q  1  f  2   2 k.
Thế Q  2, 1  thì ta được:

2  1 f  2   f  1   3 2 k  1  2 k  1  0  mod 3    1   1  0  mod 3   k phải là số lẻ.


k

Từ điều kiện i) thì ta được:


f  2.2...2   f  2  f  2  ... f  2   2 k.2 k...2 k , trong đó m lần số 2

 f  2 m    2 k   2 km.
m

Từ đây, ta thế Q  n , 2 m  thì ta được:

n  2 m f  n   f  2 m   n  2 m f  n   2 km  1
Mà ta biết rằng:  x  y   x k  y k  khi k là số lẻ.

Từ đó ta suy ra:
n  2 m nk   2 m 
k
2
Từ  1  và  2  thì ta được:

78 | Tạp chí và tư liệu toán học


Phương trình hàm trên tập rời rạc

n  2 m  f  n   2 km    n k  2 km  , m   n  2 m f  n   n k , m   3
Khi đó, với m là một số tự nhiên đủ lớn thì  3  xảy ra khi:
f  n   n k , n  , k là số tự nhiên lẻ.
Vậy từ đây ta suy ra được điều phải chứng minh.

Câu 75. Tìm tất cả các hàm số f : *


 *
thỏa mãn đồng thời các điều kiện sau:
i) f  0   0, f  1   1
ii) f  0   f  1   f  2   ...
iii) f  x 2  y 2   f 2  x   f 2  y  , x , y  *

Baltic MO
Lời giải
Ta có:
CHINH PHỤC OLYMPIC TOÁN

f  2   f  12  12   f 2  1   f 2  1   2, f  5   f  12  2 2   f 2  1   f 2  2   5,..., f  xn   xn
Ở đây x0  1, xn 1  xn2  1, n 
Hiển nhiên thì ta có: lim xn  
x 

Từ đây suy ra nếu f  m   f  m  1  thì:

f   m  1 2

 12  f 2  m  1   f 2  1   1  f 2  m  1   1  f 2  m   f  m 2  1 

 f  m2  k   f  m2  1  , k  1, 2 m  2

Quy nạp lên thì ta có tồn tại vô hạn số m sao cho f  m2  k   f  m2  1  , k  1, 2m  2

Ta chọn m đủ lớn sao cho tồn tại n để an , an 1  m2  1, m2  2m  2 


Khi đó thì an  an  1 điều này hoàn toàn vô lý nên suy ra f là hàm số tăng thực sự.
Từ đó thì hiển nhiên ta có: f  n   n , n  *
, thử lại thì thấy thỏa mãn.
Vậy tất cả các hàm số thỏa mãn yêu cầu bài toán là f  n   n , n  *
.

Câu 76. Tìm tất cả các hàm số f :  thỏa mãm các điều kiện sau:
i) Nếu a b thì f  a   f  b 
ii) f  ab   f  a 2  b 2   f  a   f  b  , a , b 
Mathlinks Contest
Lời giải
Trước hết, ta có một nhận xét nhỏ sau đây.
Nếu f  x  là một nghiệm hàm thì f  x   c cũng là một nghiệm hàm thỏa mãn yêu cầu bài
toán.

Chinh phục olympic toán| 79


Bồi dưỡng học sinh giỏi

Do đó ta có thể giả sử rằng f  1   0. Chú ý rằng từ 1 n thì f  n   0, n  .


Ta sẽ giải bài toán này thông qua các bước sau đây.
 Bước 1. Từ f  1.1   f  1  1   f  1   f  1  suy ra f  2   f  1  hoặc f  2   0.
 Bước 2. Gọi n là số nguyên sao cho 1 là số chính phương modulo n. Do đó tồn tại
số a thỏa mãn: a 2  1  kn.
Suy ra:
f  a   f  a2  1   f  a   f  1   f  a 2  1   f  kn   f  1   0

Nhưng f  n   f  kn   f  a 2  1  và f  n   f  1  nên f  n   f  1   0.
Do đó nếu tồn tại u sao cho u2  1  mod n  thì f  n   0.
 Bước 3. Từ bước 2 dễ thấy f  p   0 với mọi p là số nguyên tố và p  1  mod 4  .
 Bước 4. Giả sử f  a   f  b   0 và f  ab   f  a   f  b   0 thì f  a 2  b 2   0, vô lý.
Do đó nếu f  a   f  b   0 thì f  ab   0.

TẠP CHÍ VÀ TƯ LIỆU TOÁN HỌC


 Bước 5. Gọi a , b là hai số nguyên thỏa mãn  a , b   1, khi đó gọi p là một ước của
a 2  b 2 thì ta có: a 2  b 2  0  mod p  .
Mà ta có một bổ đề quen thuộc sau. Nếu p là một số nguyên tố có dạng 4 k  3 thì với mọi
bộ số a , b thỏa mãn p a 2  b 2 thì ta sẽ có p a và p b .

Vì  a , b   1 nên nếu p a 2  b 2 thì p chỉ có dạng 4 k  1.


Từ bước 4 thì ta có:
a 2  b 2 là tích của các số nguyên tố p i thỏa mãn f  pi   0 nên f  a 2  b 2   0.

Mà từ f  ab   f  a 2  b 2   f  a   f  b  , a , b  ta có  a , b   1  f  ab   f  a   f  b  .
 Bước 6. Cho a  bc vào phương trình đã cho thì ta được:
 
f  b 2c   f b 2  c 2  1   f  bc   f  b 

 
Nhưng do f  b   f b 2  c 2  1  và f  bc   f  b 2c 

Do đó thì ta có: f  bc   f  b 2c   * 

Thế c  1 vào  *  thì được: f  b   f  b 2  .

Thế c  b vào  *  thì được: f  b 2   f  b 3  .

Từ đấy, bằng phép quy nạp thì ta được f  b k   f  b  , k  1


 Bước 7. Sử dụng bước 5 và bước 6 thì ta có:
f   p    f  p  ở đây p
ni
i i i là các số nguyên tố
Xét hàm số f  x  xác định bởi:
* f  1   0, f  2   0

80 | Tạp chí và tư liệu toán học


Phương trình hàm trên tập rời rạc

* f  p   0 với các số nguyên tố p sao cho p  1  mod 4  và p  2.


* f  p   ap  0 với mọi số nguyên tố p còn lại, ở đây a p là các số nguyên không dương.

*f   p    f  p  ở đây p
ni
i i i là các số nguyên tố.
Ta có thể chứng minh f  x  thỏa mãn các điều kiện:
Hiển nhiên nếu a b thì f  a   f  b  .
f  1.1  f  1  1   f  1   f  1   0
f  a.1  f  a2  1   f  a   f  1 
Ta có mọi ước nguyên tố p của a 2  1 đều thỏa mãn p  1  mod 4  .
Với hai số nguyên a , b  1 bất kì, ta gọi:
p i là các ước nguyên tố của a không chia hết cho b.
q i là các ước nguyên tố của b không chia hết cho a.
CHINH PHỤC OLYMPIC TOÁN

ri là các ước nguyên tố của a và b.


f  a    f  pi    f  ri 
f  b    f  qi    f  ri 
f  ab    f  pi    f  qi    f  ri 
2 2
 a   b 
f  a  b    f  ri    f  si  , ở đây si là các ước nguyên tố của A  
2 2
    .
  a, b     a, b  
Nhưng tương tự bước 5 ta có:
Các ước nguyên tố của A là các số nguyên tố thỏa mãn si  1  mod 4  và do đó f  A   0.
Suy ra f  a 2  b 2    f  ri 

Hay ta có f  ab   f  a 2  b 2   f  a   f  b  , a , b 
Và ta có nghiệm của phương trình hàm là:
Cho M là một số nguyên, hàm f được xác định như sau:
* f  1  M
* f 2  M
* f  p   M với mọi số nguyên tố p thỏa mãn p  1  mod 4  .
* f  p   M  ap với mọi số nguyên tố p còn lại, ở đây a p là các số nguyên không dương.

*f   p   M    f  p   M  ở đây p
ni
i i i là các số nguyên tố.

Chinh phục olympic toán| 81


Bồi dưỡng học sinh giỏi

Câu 77. Tồn tại hay không hàm số f : 1, 2,..., n  thỏa mãn điều kiện:
i) f là hàm đơn ánh
ii) f  ab   f  a   f  b  với mọi a , b  1, 2,..., n và ab  n
Lời giải
Ta có thể chỉ ra hàm số f như sau:
Kí hiệu các số nguyên tố bé hơn hoặc bằng n theo thứ tự tăng dần là: p1 , p2 ,..., pk .
k k
Khi đó nếu a   pii , i  , i  n  1 thì f  pi    n  1 , f  a     i  n  1
i

i 1 i 1

Ta sẽ chứng minh hàm số này thỏa mãn yêu cầu bài toán.
k k
Thật vậy, với a   pii và b   pii có ít nhất một giá trị  i và  j khác nhau.
i 1 i 1
k k
Thì f  a     i  n  1  và f  b    i  n  1 

TẠP CHÍ VÀ TƯ LIỆU TOÁN HỌC


i 1 i 1

Hiển nhiên ta chỉ có thể biểu thị f  a  , f  b  một cách duy nhất sang hệ cơ số  n  1  và vì
thế f  a   f  b  .
 k  k k k
Mặt khác ta có f  ab   f   pii i     i  i  f  pi     i f  pi   i f  pi   f  a   f  b 
 i 1  i 1 i 1 i 1

Hay ta có: f  ab   f  a   f  b  , đúng theo giả thiết đề bài.


Vậy từ đó hàm số xây dựng như trên thỏa mãn yêu cầu bài toán.

Câu 78. Giả sử Josephus có  n  1  người bạn, n người này đúng thành một vòng tròn
đánh số từ 1 đến n theo chiều kim đồng hồ, tự sát theo nguyên tắc, người thứ nhất cầm
dao đếm 1 rồi tự sát, người thứ hai đếm 2 rồi tự sát,…Quá trình dừng lại khi còn một
người. Gọi f  n  là hàm số biểu thị vị trí cùa người sống sót đó. Câu hỏi đặt ra là, hãy
tính f  n  ?
Bài toán cổ Josephus
Lời giải
Ta sẽ xét hai khả năng của n là chẵn và lẻ.
Khả năng 1. Khi số n là số chẵn, tức n  2 k. Sau vòng 1 thì còn người ở vị trí lẻ. Số người
này đánh lại thành 1, 2,..., k.
Nếu lượt trước người đó có số 2 i  1 thì sau đó mang số i. Người sống sót có số cũ là
f  2 k  sau mang số mới là f  k  . Vậy từ đây ta có f  2 k   2 f  k   1
Khả năng 2. Khi số n là số lẻ, tức là n  2 k  1. Sau vòng 1 ta ngầm hiểu rằng có 2 k  2
người bằng cách tính trùng người thứ 1 thành 2 k  2, còn lại những người số 3, 5,..., 2 k  1
đánh số lại là 1, 2,..., k.

82 | Tạp chí và tư liệu toán học


Phương trình hàm trên tập rời rạc

Nếu lượt trước người đó có số 2 i  1 thì sau đó mang số i. Người sống sót có số cũ là
f  2 k  1  sau mang số mới là f  k  . Vậy từ đây ta có f  2 k  1   2 f  k   1
Như vậy thì f  1   1, f  2 k   2 f  k   1, f  2 k  1   2 f  k   1
Ta chứng minh bằng quy nạp rằng.
Nếu biểu diễn trong cơ số 2 của n là: n   ak ak 1 ...a1 2 , ak  1, với i  k và ai  0, 1

Thì f  n    ak 1 ...a1 ak 2 . Thật vậy:


Với n  1 thì ta thấy hiển nhiên đúng.
Giả sử với mọi k  n thì mệnh đề đúng. Ta sẽ xét hai trường hợp sau đây:
Trường hợp 1. Nếu n là số chẵn, đặt n  2 m.
Khi đó nếu như: m   bk bk 1 ...b1 2 thì 2m   bk bk 1 ...b1 0 2

Và f  2m   2 f  m   1  2  bk 1 2 k  ...  b1 .2  1   1   bk 1 ...b1 01 2
Vậy trong trường hợp 1 thì mệnh đề đúng.
CHINH PHỤC OLYMPIC TOÁN

Trường hợp 2. Nếu n là số lẻ, đặt n  2 m  1.


Khi đó nếu như: m   bk bk 1 ...b1 2 thì 2m  1   bk bk 1 ...b1 1 2

Và f  2m  1   2 f  m   1  2  bk 1 2 k  ...  b1 .2  1   1   bk 1 ...b1 11 2
Vậy trong trường hợp 2 thì mệnh đề đúng.
Từ đó, theo nguyên lý quy nạp thì mệnh đề ban đầu đúng và từ đó ta suy ra điều phải
chứng minh.

Câu 79. Cho hai hàm số f , g : *


 *
là hai hàm số thỏa mãn đồng thời các điều kiện:
i) g là hàm số toàn ánh
ii) 2 f 2  n   n2  g 2  n  , n  

Nếu f  n   n  2019 n , n  
thì f có vô số điểm bất động.
Lời giải
Đầu tiên ta có định lý Dirichlet về số nguyên tố thì dãy số  pi  với p i là các số nguyên tô
có dạng 8 k  3 là một dãy vô hạn.
Từ đó với mọi n , theo công thức của kí hiệu Legendre ta có:
 2 
2
pn  1

    8  1
 1
 pn 
Sử dụng điều kiện i) thì ta tìm được dãy  xn n 1 sao cho g  xn   pn , n 
 
.
Ta có 2 f 2  xn   xn2  pn2  2 f 2  xn   xn2  mod pn 

 2   pn f  n 
Vì    1 nên suy ra: 
 pn   pn xn

Chinh phục olympic toán| 83


Bồi dưỡng học sinh giỏi

xn  an pn
Suy ra tồn tại hai dãy số nguyên dương  an  và  bn  sao cho 
 f  xn   bn pn
Từ điều kiện ii) thì ta được 2 bn2  an2  1
Cuối cùng, sử dụng giả thiết: f  n   n  2019 n , n  
thì ta có:

2019 f  xn  bn an2  1
 1   1  lim  2  lim an  1
xn xn an x  an x 

Suy ra tồn tại N 0 sao cho an  bn  1, n  N 0


Vậy từ đó f  pn   pn , n  N 0 .
Và từ đây ta suy ra điều phải chứng minh.

Câu 80. Tìm tất cả các hàm số g : *


 *
thỏa mãn điều kiện sau:
g  g  n   n   g  n  1   3  n  g  n  , n 

TẠP CHÍ VÀ TƯ LIỆU TOÁN HỌC


Doãn Quang Tiến
Lời giải
Đặt g  n   f  n   n , n  *
thì thay vào phương trình hàm ban đầu ta được:
f  f  n    f  n   f  n  1    n  1   3  n  f  n   n , n  *

 f  f  n    f  n  1   n  2, n  *
 1
Từ đây, ta đã chuyển bài toán ban đầu thành một bài toán khác có vẻ gọn đẹp hơn rất
nhiều.
Thay n  1 vào  1  thì ta được f  f  1    f  2   3
Từ đó ta suy ra: f  2   2 và f  f  1    2. Từ đó ta xét hai trường hợp sau:
Trường hợp 1. f  2   1 và f  f  1    2.
Bây giờ ta đặt f  1   k  f  k   2
Thay n  2 vào  1  thì ta được f  f  2    f  3   4
Từ đây suy ra f  3   4  f  1   4  k
Từ f  3   1 nên suy ra: k  3.
Nếu k  1 thì ta có: 2  f  f  1    f  k   f  1   1, điều này cũng mâu thuẫn.
Nếu k  2 thì ta cũng có: 2  f  f  1    f  k   f  2   1, điều này cũng mâu thuẫn.
Nếu k  3 thì ta có: 2  f  f  1    f  k   f  3   4  k  4  3  1, điều này cũng mâu thuẫn.
Vậy tóm lại không có giá trị nào của k thỏa mãn nên trường hợp 1 không xảy ra.
Trường hợp 2. f  2   2 và f  f  1    1.
Thay n  2 vào  1  thì ta được f  f  2    f  3   4
Từ đó thì ta dễ thấy f  3   2 và ta tính toán được các giá trị sau:

84 | Tạp chí và tư liệu toán học


Phương trình hàm trên tập rời rạc

f  4  5  f  f  3  5  f  2   3
f  5  6  f  f  4   6  f  3  4
f  6  7  f  f  5  7  f  4   4
Từ đấy ta dự đoán được rằng, hàm số f  n  được xác định như sau:

1 5
f  n   n   n  1, n  *
trong đó  
2
Bây giờ ta sẽ chứng minh rằng, hàm số này là hàm số thỏa mãn  1  để rồi từ đó suy ra
công thức của hàm g  n  và từ đó ta hoàn tất bài toán.
Mà trước tiên, để chứng minh nhận định đó, ta cần phải có hai bổ đề sau:
n
Bổ đề 1. Với mỗi số n  *
thì ta có   n   n  1   
n  1
Chứng minh
CHINH PHỤC OLYMPIC TOÁN

Trước hết ta có   n   n  1      n  n  1     n    1   n    n  2

Và   n   n  1      n  1  n  1   1  n    1   1  n  1

1 5 1  5
Do lưu ý rằng:    1  nên suy ra:     1   1.
2 2
Vậy từ đó ta thấy bổ đề 1 được chứng minh.
n   2 if   n   n  1    n
Bổ đề 2. Với mỗi số n  *
thì ta có   n  1      
 n   1 otherwise
Chứng minh
Hiển nhiên thì ta có   n  1   bằng  n   1 hoặc  n   2

Giả sử   n  1    n   1 thì từ đó ta có:

 
  n   n  1     n  1    n      n  1    1  n   1    n  1    1   1  n
 
Và như trên thì ta có   n   n  1      n  n  1     n    1   n    n  2

Từ đó thì ta suy ra được:   n   n  1    n  1.

Giả sử   n  1    n   2 thì từ đó ta có:

 
  n   n  1      n  1     n  1      n  1    n  1   n  1
 
Tử đó thì theo bổ đề 1 thì ta thu được:   n   n  1    n.
Vậy từ đó thì bổ đề 2 được chứng minh.
Quay trở lại với việc giải bài toán
Ta sẽ sử dụng phương pháp quy nạp để chứng minh kết quả ban đầu.
Với n  1 thì f  1   .1  1  1  1.

Chinh phục olympic toán| 85


Bồi dưỡng học sinh giỏi

Với n  2 thì f  2   .2   2  1  2.


Giả sử kết quả đúng với 1  j  n. Sử dụng  1  thì ta có:
f  n  1   n  2  f  f  n    n  2  f  n   n  1  n  2   n   n  1     n   n  1   1

Mà từ n   n  1  2n  n  1  n  1 ta có f  n  1   n   2    n   n  1  

Giả sử n thỏa mãn:   n   n  1    n thì từ đó ta có:   n  1    n   2

Và do đó ta suy ra được f  n  1     n  1    n

Nếu n không thỏa mãn   n   n  1    n thì tức là chỉ có thể xảy ra:
  n   n  1    n  1.

Và theo bổ đề 2 thì ta được:   n  1    n   1


Và từ đó ta suy ra được:
f  n  1   n   2    n   n  1    n   2   n  1   n   1  n    n  1    n

TẠP CHÍ VÀ TƯ LIỆU TOÁN HỌC


Vậy từ đó theo nguyên lý quy nạp thì mệnh đề  1  được chứng minh hoàn toàn.
Từ đây suy ra tất cả các hàm số thỏa mãn  1  là
1 5
f  n   n   n  1, n  *
trong đó   .
2
Hay từ đây ta suy ra được hàm g  n  mà chúng ta cần tìm là:
1 5
g  n   f  n   n  n   n  1  n  n   1, n  *
trong đó   .
2


Câu 81. Cho ba số thực a , b , c không âm, phân biệt sao cho tồn tại hàm f , g : 
x
thỏa mãn af  xy   bf    cf  x   g  y  với mọi số thực dương x  y .
y
Chứng minh rằng tồn tại hàm h :   sao cho:
x
f  xy   f    2 f  x   h  y  , x  y  0
y
Iran TST 2019
Lời giải
x
Đặt P  x , y  là phép thế cho phương trình: af  xy   bf    cf  x   g  y 
y
Trường hợp 1: Nếu a  b  c  0 :
g  1 x
Ta có P  x , 1   f  x   là hằng số, vậy: f  xy   f    2 f x
ab c y
Suy ra h  y   0y , thỏa mãn.

86 | Tạp chí và tư liệu toán học


Phương trình hàm trên tập rời rạc

Trường hợp 2: Nếu a  b  c  0 :


 x 
Ta có a  f  xy   f  x    b  f    f  x    g  y  và đặt phép thế là Q  x , y 
 y 
   
Lấy Q  1, y   Q  x , y  ta được
 x 1 
a  f  xy   f  x   f  y   f  1   b  f    f x  f    f  1   0  1
 
 y y 
1
Thế y  vào  1  ta được:
y
 x 1 
a  f    f  x   f    f  1   b  f  xy   f  x   f  y   f  1   0
 y 
   y 
Do a  b , a  b  c  0  a 2  b 2  0 , vì vậy:
f  xy   f  x   f  y   f  1  , x , y  0
CHINH PHỤC OLYMPIC TOÁN

x 1
f    f x  f    f  1 , x , y  0
y y
x 1
Cộng hai phương trình với nhau f  xy   f    2 f  x   f  y   f    2 f  1 
y y
1
Vậy h  y   f  y   f    2 f  1 y , thỏa mãn.
y

Câu 82. Tìm tất cả hàm số f :  thỏa mãn:


n ! f  m  ! f  n  ! f  m ! , m , n 
BMO Shortlist 2018
Lời giải
Đặt phép thế P  m , n  cho phương trình ban đầu
P  1, 1   1  f  1  ! f  1   f  1  !  f  1  ! 1 f  1   1 , từ đây hiển nhiên f  1   1
P  1, n   n ! 1 f  n  ! 1  f  n  !  n !  f  n   n
Gọi p là số nguyên tố tùy ý, ta có P  1, p  1    p  1  ! 1 f  p  1  ! 1 .
Theo định lý Wilson trong số học ta được p  p  1  ! 1 , suy ra p f  p  1  ! 1
Lưu ý rằng nếu f  p  1   p  1 thì f  p  1  ! là tích của ít nhất p thừa số nguyên dương và
là một số chia hết cho p , do đó f  p  1  ! 1  1  mod p  - mâu thuẫn chứng minh trên.
Vậy f  p  1   p  1 , lại có f  n   n nên f  p  1   p  1 với mọi số nguyên tố p .
Lại có P  m , p  1    p  1  ! f  m  ! f  m !   p  1  !   p  1  ! f  m  ! f  m !  f  m  !

Chinh phục olympic toán| 87


Bồi dưỡng học sinh giỏi

Với giá trị m bất kỳ, ta chọn p đủ lớn để thu được: f  m !  f  m  ! , sử dụng kết quả này ta
được: n ! f  m  ! f  n  ! f  m  ! tương đương n ! f  m  ! f  n  ! n !
Thay m  p  1 với p đủ lớn vào phương trình trên ra được f  n  !  n ! với mọi n .
Vậy f  n   n là hàm số cần tìm.

Câu 83. Tồn tại hay không hàm số f : *


 *
thỏa mãn điều kiện sau:
f  f  n    3n  2 f  n  , n  *

Lời giải
Giả sử tồn tại hàm số f thỏa mãn yêu cầu bài toán.
ta xây dựng dãy số như sau  an n 1 sao cho: a1  i , an 1  f  an 

Với mỗi i  *

Khi đó thì ta có an1  f  an   f  f  an1    2 f  an1   3an1  2 an  3an 1


Hay ta có an 4  4 an  1  3an  0, n  1

TẠP CHÍ VÀ TƯ LIỆU TOÁN HỌC


Do an  0, n  1 nên đẳng thức không thể xảy ra.
Nên từ đó ta kết luận rằng không tồn tại hàm số f thỏa mãn yêu cầu bài toán.

Câu 84. Tìm tất cả các hàm số tăng thực sự f : *


 *
thỏa mãn điều kiện sau:
f  n  f  n    2 f  n  , n  *

Lời giải
Do f là hàm số tăng thực sự nên ta có:
f  n  1   f  n   1 hay f  n  1   n  1  f  n   n
Suy ra: f  n   n là hàm số tăng.
Mặt khác ta đặt a0  1, an 1  an  f  an 
Từ đó ta suy ra a0  a1  ... và f  an 1   2 f  an 
Do đó f  an 1   an  1  f  an   an , n  *

Suy ra có vô hạn bộ số  m , n  sao cho: f  m   m  f  n   n


Suy ra f  n   n  c , với c là hằng số.
Vậy tất cả các hàm số thỏa mãn yêu cầu bài toán là: f  n   n  c , n  *
, c là hằng số.

Câu 85. Tìm tất cả các toàn ánh f :  sao cho với mọi m , n  thỏa mãn:
f m f n  m n
Lời giải
Kí hiệu P  là tập tất cả các số nguyên tố.
k k
Xét đơn ánh g : P  P thì nếu n   pii thì f  n    g  pi  i .

i 1 i 1

88 | Tạp chí và tư liệu toán học


Phương trình hàm trên tập rời rạc

Kí hiệu   n  là số ước nguyên dương của n.


Mà ta có nhận xét sau:   n   f    n   do f là toàn ánh.
Với mỗi số nguyên tố p thì f  p  chỉ có đúng hai ước nguyên tố nên nó cũng là số nguyên
tố.
Xác định hàm g như trên thì từ đó ta có: f  p   g  p  nên ta sẽ chứng minh g là song ánh.
Thật vậy, do f là toàn ánh nên g là toàn ánh nên g là song ánh.
Tiếp theo, ta sẽ chứng minh f  p k   g  p  với k là số nguyên dương bằng quy nạp
k

Ta thấy rằng, với k  1 thì hiển nhiên đúng.


Giả sử mệnh đề đúng với k  1.
Ta có f  p k  chia hết cho 1, g  p  , g  p  ,..., g  p 
2 k 1
và ngoài ra không chia hết cho số
nguyên dương nào khác.
 
Do đó  f  p k     p k   k  1 .
CHINH PHỤC OLYMPIC TOÁN


Nếu k  1 khi f  p k  có thêm một ước nguyên tố nữa thì  f  p k   2 k  k  1, vô lý. 
Từ đó f  p k  là lũy thừa của g  p  và nó có k  1 ước nên f  p k   g  p  .
k

Giả sử n là một số nguyên dương, p là một số nguyên tố không chia hết cho n.
Bây giờ ta sẽ đi chứng minh f  n  f  p k   f  np k  , k 
Từ  n , p k   1 nên ta có:   n    p k     np k 

Mặt khác g  p  f  np k  và g  p  f  n 
k

Do vậy mọi ước của f  n  và g  p  chia hết cho f  np k  và mọi ước của g  p  và f  n  là
k k

ước của f  np k  .

  
Lại có  f  n  f  p k     np k    f  np k  
Nếu f  np k  có ước khác với các ước của f  n  và g  p  thì
k

   
 f  n  f  p k    f  np k  , vô lý.

Vậy từ đó ta có kết quả f  np k   f  n  g  p   f  n  f  p k 


k

Từ các nhận xét trên ta có hàm f được xây dựng như trên là duy nhất.
Vậy tất cả các hàm số thỏa mãn yêu cầu bài toán là các hàm như trên.

Chinh phục olympic toán| 89


TÀI LIỆU THAM KHẢO
Dưới đây là các tài liệu mà ebook này có tham khảo và đồng thời có cả những tài liệu mà
bọn mình đề xuất cho bạn đọc

[1]. Nguyễn Văn Mậu (1997), 00 Phương trình hàm 00, NXB Giáo dục.
[2]. Nguyễn Trọng Tuấn (2004), “Bài toán hàm số qua các kì thi Olympic”, NXB Giáo
dục.
[3]. Nguyễn Tài Chung, Lê Hoành Phò (2013), “Chuyên khảo phương trình hàm” Nhà
xuất bản Đại học quốc gia Hà Nội.
[4]. J.Aczél (1966), “Lectures on functional equations and their applications”, ACADEMIC
PRESS New York San Francisco London.
[5]. Stevo Stevic (2004), “Periodic Character of a Class of Difference Equation”, Taylor &
Francis Group.
[6]. Valentine Boju, Luis Funar - The Math Problems Notebook.
[7]. Titu Andreescu, Razvan Gelca – Birkhauser Mathematical Olympiad Challenges.
[8]. Edward Lozansky , Cecil Rousseau – Winning Solutions.
[9]. The IMO Compendium. A Collection of Problems Suggested for The International
Mathematical Olympiads: 1959 – 2009 – Djukic D., Vladimir Jankovic, Ivan Matic, Nikola
Petrovic – Springer (2011).
[10]. Trần Nam Dũng, Dương Bửu Lộc – Chuyên đề Phương trình hàm trên tập số nguyên .
[11]. (Developments in Mathematics 39) Saïd Abbas, Mouffak Benchohra – Advanced.
[12]. Functional Evolution Equations and Inclusions-Springer International Publishing (2015)
[13]. Aczel – Lectures on functional equations and their applications – Academic Press (1966).
[14]. An Introduction to the Theory of Functional Equationsand Inequalities – Marek Kuczma.
[15]. Analytic Solutions of Functional Equations – Sui Sun Cheng, Wenrong Li.
[16]. Functional Analysis, Sobolev Spaces and Partial Differential Equations – Haim Brezis.
[17]. Topics in Algebra and Analysis Preparing for the Mathematical Olympiad –Radmila.
[18]. Bulajich Manfrino, José Antonio Gómez Ortega, Rogelio Valdez Delgado-Birkhäuser
Basel (2015).
[19]. Kỷ yếu gặp gỡ toán học và các kỷ yếu từ hội thảo chuyên đề các tỉnh.
[20]. The art of Mathematics.
[21]. 101 Problems in Algebra from the training of the USA IMO team – T Andreescu, Z
Feng.
[22]. Problem Primer for the Olympiad – C. R. Pranesachar, B. J. Venkatachala, C. S.
Yogananda
[23]. https://artofproblemsolving.com
TẠP CHÍ VÀ TƯ LIỆU TOÁN HỌC

HẾT

CHINH PHỤC OLYMPIC TOÁN


TẠP CHÍ VÀ TƯ LIỆU TOÁN HỌC
Thôn 6 – Thạch Hòa – Thạch Thất – Hà Nội
Điện thoại: 0343763310; Email: tuangenk@gmail.com
Fanpage: https://www.facebook.com/OlympiadMathematical/

CHỊU TRÁCH NHIỆM NỘI DUNG


DOÃN QUANG TIẾN
NGUYỄN MINH TUẤN
TÔN NGỌC MINH QUÂN

BIÊN TẬP
NGUYỄN MINH TUẤN

TRÌNH BÀY BẢN THẢO


NGUYỄN MINH TUẤN
LA THỊ ĐÔNG PHƯƠNG

PHƯƠNG TRÌNH HÀM TRÊN TẬP RỜI RẠC


Đề nghị quý bạn đọc tôn trọng bản quyền của tác giả, không sao chép bản phụ.
Mọi ý kiến thắc mắc đóng góp vui lòng gửi về địa chỉ đã cung cấp ở trên.
Phiên bản sách điện tử được phát hành vào ngày 20/8/2019.
CHINH PHỤC OLYMPIC TOÁN
MỌI Ý KIẾN THẮC MẮC XIN VUI LÒNG GỬI VỀ ĐỊA CHỈ

NGUYỄN MINH TUẤN


0343763310
tuangenk@gmail.com
Lovetoan.wordpress.com
Đại học FPT Hà Nội

PHIÊN BẢN
ĐẶC BIỆT
PHAN QUANG ĐẠT
NGUYỄN NHẤT HUY • DƯƠNG QUỲNH CHÂU

CÁC BÀI TOÁN

SỐ HỌC
TUYỂN CHỌN TỪ CÁC ĐỀ TUYỂN SINH CHUYÊN TOÁN

MATHPIAD
TUYỂN TẬP SỐ HỌC TRONG
KÌ THI CHUYÊN TOÁN 2020 − 2021

Mathpiad − Tạp chí và tư liệu toán học

Phan Quang Đạt − Nguyễn Nhất Huy − Dương Quỳnh Châu

LATEX by Mathpiad
2
LAT
EX by Mathpiad
Chương I

Một số kiến thức sử dụng trong


tài liệu
1 Các định nghĩa ngoài sách giáo khoa.

† Số chính phương là số có thể biểu diễn dưới dạng bình phương của một số tự
nhiên.
† Số lập phương là số có thể biểu diễn dưới dạng lập phương của một số nguyên.

2 Các kí hiệu, quy ước ngoài sách giáo khoa.

† Kí hiệu a | b dùng thay cho mệnh đề "a là ước của b", và đọc là "a chia hết b".
† Kí hiệu (a, b) dùng để chỉ ước chung lớn nhất của a và b. Đôi lúc, nó còn dùng
để chỉ cặp số (a, b), vì thế cần phân biệt rõ.
† Kí hiệu a ≡ b (mod m) dùng thay cho mệnh đề "a và b có cùng số dư khi chia
cho m", và đọc là "a đồng dư với b theo modulo m".

3 Các hằng đẳng thức mở rộng.

† (A + B +C)2 = A2 + B2 +C2 + 2AB + 2BC + 2CA.


† A3 + B3 +C3 − 3ABC = (A + B +C) A2 + B2 +C2 − AB − BC −CA .


4 Các tính chất về ước chung lớn nhất.

a) Với các số nguyên a, b, c khác 0 thỏa mãn c | ab và (a, c) = 1, ta có thể suy ra c | b.


b) Với các số nguyên a, b, c khác 0 thỏa mãn ab = c2 , và (a, c) = 1, ta có |a| và |b| là
hai số chính phương.
c) Với các số nguyên a, b, c khác 0 thỏa mãn ab = c3 , và (a, c) = 1, ta có a và b là hai
số lập phương.

5 Các tính chất về đồng dư thức và chia hết.

(a) Tính chia hết của tổng, tích các số nguyên liên tiếp.
† Tổng của n số nguyên liên tiếp luôn chia hết cho n.
† Tích của n số nguyên liên tiếp luôn chia hết cho n!, ở đây n! là tích của tất
cả các số tự nhiên từ 1 đến n.

3
LAT EX by Mathpiad
CHƯƠNG I. MỘT SỐ KIẾN THỨC SỬ DỤNG TRONG TÀI LIỆU

(b) Nếu a ≡ b (mod m), ta có thể suy ra


† m | (a − b).
† a + c ≡ b + c (mod m), với c là một số nguyên.
† ac ≡ bc (mod m), với c là một số nguyên.
a b m
† ≡ (mod ), với n là một ước chung nào đó của a, b, m.
n n n
† an ≡ bn (mod m).
(c) Một số chính phương bất kì chỉ có thể
† Đồng dư với 0 hoặc 1 theo modulo 3.
† Đồng dư với 0 hoặc 1 theo modulo 4.
† Đồng dư với 0, 1 hoặc 4 theo modulo 8.
(d) Định lý Fermat nhỏ. Cho p là số nguyên tố và a là số nguyên dương thỏa mãn
a không chia hết cho p, khi đó

a p−1 ≡ 1 (mod p).

6 Bổ đề kẹp.
Giữa hai lũy thừa số mũ n liên tiếp, không tồn tại một lũy thừa cơ số n nào. Hệ quả,
với mọi số nguyên a

† Không có số chính phương nào nằm giữa a2 và (a + 1)2 .


† Số chính phương duy nhất nằm giữa a2 và (a + 2)2 là (a + 1)2 .
† Có đúng k − 1 số chính phương nằm giữa a2 và (a + k)2 , đó là

(a + 1)2 , (a + 2)2 , . . . , (a + k − 1)2 .

7 Bổ đề về nghiệm nguyên của phương trình bậc hai.


Nếu phương trình bậc hai với hệ số nguyên ax2 + bx + c = 0 có hai nghiệm nguyên
(không nhất thiết phân biệt) thì ∆ = b2 − 4ac là số chính phương.

4
LAT EX by Mathpiad
Chương II

Giới thiệu một số bài toán số học


trong đề thi vào lớp 10 chuyên
Toán
Câu 1.
1 Tìm tất cả số tự nhiên n sao cho (2n + 1)3 + 1 chia hết cho 22021 .

2n + 2 4n2 + 2n + 1
2 Cho số tự nhiên n và số nguyên tố p sao cho a = và b = là các số
p p
nguyên. Chứng minh rằng a và b không đồng thời là các số chính phương.

Chuyên Phổ thông Năng khiếu - ĐHQG thành phố Hồ Chí Minh

Câu 2. Tìm tất cả các số nguyên dương m, n thỏa mãn m(m + 1)(m + 2) = n2 .
Hà Tĩnh
Câu 3.
1 Tìm tất cả các nghiệm (x; y) của phương trình x2 − 2x + 2y = 2(xy + 1).

2 Cho p là số nguyên tố sao cho tồn tại các số nguyên dương x, y thỏa mãn x3 + y3 − p =
6xy − 8. Tìm giá trị lớn nhất của p.

Lào Cai

Câu 4. Tìm tất cả bộ ba số nguyên tố (p, q, r) thỏa mãn pq = r + 1 và 2 p2 + q2 = r2 + 1.




Quảng Nam
Câu 5. Tìm tất cả số tự nhiên a và b với lớn hơn 1 sao cho (a − 1)(b − 1) | (ab − 1).
Chuyên Đại học Khoa học Huế
Câu 6. Giả sử n là số tự nhiên thỏa mãn điều kiện n(n + 1) + 7 không chia hết cho 7. Chứng
minh rằng 4n3 − 5n − 1 không là số chính phương.
Thái Bình

5
LAT EX by Mathpiad
CHƯƠNG II. GIỚI THIỆU MỘT SỐ BÀI TOÁN SỐ HỌC TRONG ĐỀ THI VÀO LỚP 10 CHUYÊN
TOÁN

Câu 7.

1 Tìm tất cả các cặp số nguyên (x, y) thỏa mãn x2 + 2y2 − 2xy − 2x − 4y + 6 = 0.

p2 − p
2 Tìm tất cả các số nguyên tố p sao cho − 1 là lập phương một số tự nhiên.
2

Thanh Hóa - Chuyên Toán

Câu 8.

1 Tìm tất cả các bộ ba số nguyên dương (x, y, z) thỏa mãn đồng thời các điều kiện

√ √ √ 1 1 1
xy + xz − yz = y, + − = 1.
x y z

2 Cho số tự nhiên n > 2 và số nguyên tố p thỏa mãn p − 1 chia hết cho n và n3 − 1 chia
hết cho p. Chứng minh rằng n + p là một số chính phương.

Thanh Hóa - Chuyên Tin

Câu 9.

1 Tìm tất cả các số nguyên dương n để n − 1989 và n − 2022 đều là các số chính phương.

2 Biết rằng phương trình x2 − ax + b + 2 = 0 (với a, b là các số nguyên) có hai nghiệm


nguyên. Chứng minh rằng 2a2 + b2 là hợp số

Quảng Trị

Câu 10. Cho hình lăng trụ đứng, đáy là tam giác vuông, chiều cao bằng 6. Số đo ba cạnh
của tam giác đáy là các số nguyên. Số đo diện tích toàn phần của lăng trụ bằng số đo thể tích
của lăng trụ. Tính số đo ba cạnh tam giác đáy của lăng trụ.

Quảng Ninh

Câu 11.

1 Tìm tất cả số nguyên x, y thỏa mãn bất đẳng thức 5x2 + 3y2 + 4xy − 2x + 8y + 8 6 0.

2 Trong 2021 số nguyên dương đầu tiên, có bao nhiêu số không chia hết cho 7 và không
chia hết cho 11?

Đồng Nai

Câu 12. Tìm tất cả các cặp số nguyên (x, y) thỏa mãn (xy − 1)2 = x2 + y2 .

Bà Rịa - Vũng Tàu

Câu 13. Giải phương trình nghiệm nguyên x2 y − xy + 2x − 1 = y2 − xy2 − 2y.

6
LAT EX by Mathpiad
MATHPIAD − TẠP CHÍ TOÁN HỌC

Bến Tre

Câu 14.

1 Tìm x, y ∈ N sao cho x3 = 1993 · 3y + 2021.


n − 23
2 Tìm số nguyên dương n để là bình phương một số hữu tỉ dương.
n + 89

Nghệ An

Câu 15.

1 Cho a là số nguyên tố lẻ và a không chia hết cho 3.


Chứng minh rằng a2 − 2021 chia hết cho 24

2 Cho các số nguyên tố p, q thỏa mãn p + q2 là số chính phương. Chứng minh rằng

a, p = 2q + 1.
b, p2 + q2021 không phải là số chính phương.

3 Cho tập hợp S gồm n số nguyên dương đôi một khác nhau (n > 3) thỏa mãn tính chất:
tổng của 3 phần tử bất kì trong S đều là số nguyên tố. Tìm giá trị lớn nhất có thể của
n.

Quảng Ngãi

Câu 16.

1 Chứng minh rằng tổng các bình phương của 6 số nguyên liên tiếp không thể là số
chính phương.

2 Tìm các nghiệm nguyên dương của phương trình x2 y + 2xy + y = 32x.

Vĩnh Long

b
Câu 17. Cho ba số nguyên a, b, c thỏa mãn a = b − c = . Chứng minh rằng a + b + c là
c
lập phương của một số nguyên.

Bình Dương

Câu 18.

1 Cho m, p, r là các số nguyên tố thỏa mãn mp + 1 = r. Chứng minh rằng m2 + r hoặc


p2 + r là số chính phương.

2 Tìm tất cả các số nguyên tố q sao cho tồn tại các số nguyên dương n để n2 + 22q là
một lũy thừa với số mũ nguyên dương của 11.

Kiên Giang

LATEX by Mathpiad 7
CHƯƠNG II. GIỚI THIỆU MỘT SỐ BÀI TOÁN SỐ HỌC TRONG ĐỀ THI VÀO LỚP 10 CHUYÊN
TOÁN

Câu 19. Hỏi có bao nhiêu số tự nhiên n không vượt quá 2021 mà n3 + 2021 chia hết cho 6.

Hòa Bình

Câu 20. Tìm tất cả các số nguyên x, y thỏa mãn y2 + 3y = x4 + x2 + 18.

Ninh Thuận

Câu 21. Tìm tất cả các số nguyên dương x, y thỏa mãn x2 − 2y · x − 421 · 9 = 0.

Thừa Thiên Huế - Chuyên Toán

Câu 22.

1 Tìm tất cả các số nguyên x, y thỏa mãn điều kiện

x2 y − y2 x − 2x2 − 3y2 + 10xy − 16x + 21y = 100.

2 Chứng minh rằng từ 1012 số nguyên bất kì, luôn tồn tại hai số mà hiệu bình phương
của chúng là một số nguyên chia hết cho 2021.

Thừa Thiên Huế - Chuyên Tin

Câu 23. Tìm nghiệm nguyên của phương trình x2 − 2y(x − y) = 2(x + 1).

Tây Ninh

Câu 24. Cho m, n là các số nguyên dương sao cho m2 + n2 + m chia hết cho mn. Chứng
minh rằng m là số chính phương.

Tiền Giang

Câu 25. Tìm tất cả các cặp số nguyên (x, y) thỏa mãn x2 + 5y2 + 4xy + 4y + 2x − 3 = 0.

Cần Thơ

Câu 26. Hai số tự nhiên khác nhau được gọi là "thân thiết" nếu tổng bình phương của chúng
chia hết cho 3. Hỏi tập X = {1; 2; 3; ...; 2021} có bao nhiêu cặp số "thân thiết" (không phân
biệt thứ tự)?

Khánh Hòa

Câu 27.

1 Cho phương trình bậc hai với tham số m x2 −2(m−2)x+m2 −3 = 0. Tìm tất cả các giá
2x1 x2
trị nguyên của m để phương trình có 2 nghiệm phân biệt x1 , x2 thỏa mãn P =
x1 + x2
nhận giá trị nguyên.

2 Tìm tất cả các nghiệm nguyên (x, y) của phương trình x2 + 3y2 + 4xy − 2x − 4y − 2 = 0.

Đắk Nông

LATEX by Mathpiad 8
MATHPIAD − TẠP CHÍ TOÁN HỌC

Câu 28.

1 Tìm nghiệm nguyên của phương trình (2x + y)(x − y) + 3(2x + y) − 5(x − y) = 22.

2 Cho hai số tự nhiên a, b thỏa mãn 2a2 + a = 3b2 + b. Chứng minh rằng 2a + 2b + 1 là
số chính phương.

Bình Phước

Câu 29.

1 Tìm tất cả các số tự nhiên n và k để n4 + 42k+1 là số nguyên tố.

2 Tìm tất cả các số nguyên dương x, y thỏa mãn

x4 − x2 + 2x2 y − 2xy + 2y2 − 2y − 36 = 0.

Đắk Lắk

Câu 30. Tìm tất cả các số nguyên dương n sao cho hai số n2 − 2n − 7 và n2 − 2n + 12 đều
là lập phương của một số nguyên dương nào đó.

Quảng Bình

Câu 31. Tìm tất cả các số nguyên dương n để A = 4n3 + 2n2 − 7n − 5 là một số chính
phương.

Thái Nguyên - Chuyên Toán

Câu 32. Tìm tất cả các số nguyên tố p, q sao cho phương trình x2 − px + q = 0 có các
nghiệm là số nguyên.

Thái Nguyên - Chuyên Tin

Câu 33. Giải phương trình x3 + y3 − x + 3z = 2021 với x, y, z là các số nguyên.

Hà Nam

Câu 34. Cho a, b, c là các số nguyên thỏa mãn a + b + 20c = c3 . Chứng minh rằng a3 +
b3 + c3 chia hết cho 6.

Lâm Đồng

Câu 35.

1 Tìm tất cả các số tự nhiên n sao cho n4 + n3 + 1 là số chính phương.

2 Cho phương trình x2 − mx + m + 2 = 0. Tìm tất cả các giá trị của m để phương trình
đã cho có các nghiệm nguyên.

Bình Định - Chuyên Toán

LATEX by Mathpiad 9
CHƯƠNG II. GIỚI THIỆU MỘT SỐ BÀI TOÁN SỐ HỌC TRONG ĐỀ THI VÀO LỚP 10 CHUYÊN
TOÁN

Câu 36. Tìm tất cả các số nguyên dương x sao cho x2 − x + 13 là số chính phương.

Bình Định - Chuyên Tin

Câu 37. Tìm tất cả các cặp số nguyên (x, y) thỏa mãn 7(x + 2y)3 (y − x) = 8y − 5x + 1.

Ninh Bình

Câu 38. Tìm các số nguyên dương x, y thỏa mãn y4 + 2y3 − 3 = x2 − 3x.

Hải Phòng

Câu 39.

1 Gọi A là số tạo nên khi viết liên tục các số tự nhiên từ 1 đến 2021, nghĩa là

A = 123 . . . 201920202021.

a, Số A gồm bao nhiêu chữ số?


b, Chữ số thứ 2021 (theo chiều từ trái qua phải) của A là chữ số nào?

2 Cho p, x, y là các số tự nhiên thỏa mãn px2 + x = (p + 1)y2 + y. Chứng minh rằng x − y
là một số chính phương.

Bình Thuận - Chuyên Toán

3
Câu 40. Cho a và b là các số nguyên dương thỏa mãn a2 − ab + b2 chia hết cho 25. Chứng
2
minh rằng cả a và b đều chia hết cho 25.

Bình Thuận - Chuyên Tin

Câu 41.

1 Tìm tất cả các cặp số nguyên (x, y) thỏa mãn x2 y − xy + y (x + y) = 3x + 1.




2 Tìm các số nguyên tố p, q đồng thời thỏa mãn hai điều kiện

i, p2 q + p chia hết cho p2 + q.


ii, pq2 + q chia hết cho q2 − p.

Phú Thọ

Câu 42.

1 Chứng minh rằng nếu n là một số nguyên tố thì n2 + 2022 không là lũy thừa của 3.

2 Tìm tất cả các số nguyên tố p sao cho p2 + 3p + 2021 cũng là một số nguyên tố.

Yên Bái

LATEX by Mathpiad 10
MATHPIAD − TẠP CHÍ TOÁN HỌC

Câu 43. Chứng minh rằng với mọi số tự nhiên n, ta luôn có

2005n + 60n − 1897n − 168n chia hết cho 2004.

Lai Châu
Câu 44.
1 Tìm tất cả các cặp số nguyên (x, y) thỏa mãn x2 + 5xy + 6y2 + x + 2y − 2 = 0.

2 Chứng minh rằng với mọi số tự nhiên n, ta luôn có n2 + n + 16 không chia hết cho 49.
2
3 Cho số thực x khác 0 thỏa mãn cả hai số x + và x3 đều là số hữu tỉ. Chứng minh rằng
x
x cũng là số hữu tỉ.

Hà Nội - Chuyên Toán

Câu 45.
1 Chứng minh rằng với mọi số nguyên n, ta luôn có n2 + 3n + 16 không chia hết cho 25
.

2 Tìm tất cả các số nguyên x, y thỏa mãn x2 − xy − 2y2 + x + y − 5 = 0.

Hà Nội - Chuyên Tin

Câu 46.
1 Tìm số nguyên dương n nhỏ nhất, biết rằng khi chia n cho 7, 9, 11, 13 ta nhận được các
số dư tương ứng là 3, 4, 5, 6.

2 Cho tập A = {1, 2, 3, . . . , 2021}. Tìm số nguyên dương k lớn nhất (k > 2) sao cho ta
có thể chọn được k số phân biệt từ tập A mà tổng của hai số phân biệt bất kì trong k số
được chọn không chia hết cho hiệu cúa chúng.

Chuyên Khoa học Tự nhiên - Vòng 1

Câu 47. Tìm x, y nguyên dương thỏa mãn 3x + 29 = 2y .


Chuyên Khoa học Tự nhiên - Vòng 2
Câu 48. Tìm tất cả các số nguyên a, b thỏa mãn a4 − 2a3 + 10a2 − 18a − 16 = 4b2 + 20b.
Cao Bằng
Câu 49.
1 Tìm tất cả các cặp số nguyên dương (x, y) thỏa mãn x2 y2 (y − x) = 5xy2 − 27.

2 Cho p1 , p2 , . . . , p12 là các số nguyên tố lớn hơn 3. Chứng minh rằng p21 + p22 + . . . + p212
chia hết cho 12.

LATEX by Mathpiad 11
CHƯƠNG II. GIỚI THIỆU MỘT SỐ BÀI TOÁN SỐ HỌC TRONG ĐỀ THI VÀO LỚP 10 CHUYÊN
TOÁN

Nam Định

Câu 50.

1 Tìm tất cả các số tự nhiên có 4 chữ số abcd thỏa mãn đồng thời các điều kiện: abcd
chia hết cho 3 và abc − bda = 650.

2 Tìm các số nguyên x, y, z thỏa mãn đồng thời các điều kiện

x2 + 4y2 + 2z2 + 2(xz + 2x + 2z) = 396, x2 + y2 = 3z.

Hải Dương

Câu 51. Cho số nguyên dương n > 2. Chứng minh rằng

1 A = n3 − 3n2 + 2n chia hết cho 6.

2 B = nA+1 − 1 chia hết cho 7.

Tuyên Quang
√ √
Câu 52. Cho a, b là hai số hữu tỉ. Chứng minh rằng nếu a 2 + b 3 là số hữu tỉ thì a = b =
0.

Chuyên Sư phạm - Vòng 1

Câu 53.

1 Tìm tất cả các số nguyên dương N sao cho N có thể biểu diễn duy nhất một cách biểu
x2 + y
diễn ở dạng với x, y là hai số nguyên dương.
xy + 1
2 Cho a, b, c là ba số nguyên dương sao cho mỗi số trong ba số đó đều biểu diễn dạng
lũy thừa của 2 với số mũ tự nhiên. Biết rằng phương trình bậc hai ax2 − bx + c = 0
có hai nghiệm đều là số nguyên. Chứng minh rằng hai nghiệm của phương trình này
bằng nhau.

Chuyên Sư phạm - Vòng 2


2
Câu 54. Tìm tất cả các cặp số nguyên (x, y) thỏa mãn x2 − y2 = 1 + 20y.

Đà Nẵng

Câu 55. 1 Cho các số nguyên x, y, z thỏa mãn x2 + y2 + z2 = 2xyz.


Chứng minh rằng xyz chia hết cho 24.

2 Tìm các bộ ba số nguyên dương (a, b, c) sao cho (a + b + c)2 − 2a + 2b là số chính


phương.

Vĩnh Phúc

LATEX by Mathpiad 12
MATHPIAD − TẠP CHÍ TOÁN HỌC

Câu 56. 1 Tìm tất cả các số nguyên (x, y) thỏa mãn x3 y − x3 − 1 = 2x2 + 2x + y.

2 Có bao nhiêu số tự nhiên gồm 6 chữ số có dạng 357abc chia hết cho 3, 5 và 7.

Kon Tum

Câu 57. Tìm tất cả các số nguyên dương n để n5 + n4 + 1 là số nguyên tố.

Sóc Trăng

Câu 58. 1 Chứng minh rằng n5 − n chia hết cho 240 với n là số tự nhiên lẻ bất kì.

2 Tìm tất cả các nghiệm nguyên của phương trình x2 − y2 x + y4 + 6y2 = 0.




Bắc Giang

Câu 59. 1 Tìm tất cả các cặp số nguyên (x, y) thỏa mãn phương trình 2x2 − xy + 9x −
3y + 4 = 0.

2 Cho f (x) là một đa thức với hệ số nguyên. Biết f (1)· f (2) = 2021, chứng minh phương
trình f (x) = 0 không có nghiệm nguyên.

3 Cho tập hợp A có các tính chất sau:

i, Tập hợp A chứa toàn bộ các số nguyên.


√ √
ii, 2 + 3 ∈ A.
iii, Với mọi x, y ∈ A thì x + y ∈ A và xy ∈ A .
1
Chứng minh rằng √ √ ∈A.
2+ 3

Lạng Sơn

LATEX by Mathpiad 13
CHƯƠNG II. GIỚI THIỆU MỘT SỐ BÀI TOÁN SỐ HỌC TRONG ĐỀ THI VÀO LỚP 10 CHUYÊN
TOÁN

LATEX by Mathpiad 14
Chương III
Lời giải tham khảo

d Câu 1

1 Tìm tất cả số tự nhiên n sao cho (2n + 1)3 + 1 chia hết cho 22021 .

2n + 2 4n2 + 2n + 1
2 Cho số tự nhiên n và số nguyên tố p sao cho a = và b =
p p
là các số nguyên. Chứng minh rằng a và b không đồng thời là các số chính
phương.

Chuyên Phổ thông Năng khiếu - ĐHQG thành phố Hồ Chí Minh

Lời giải.
1 Từ giả thiết, ta có
22021 | (2n + 2)(4n2 + 2n + 1) (*)
Ta nhận thấy 4n2 + 2n + 1 là số lẻ, do đó 4n2 + 2n + 1, 2 2021

= 1. Như vậy, (*) tương
đương với
22021 | (2n + 2) ⇔ 22020 | (n + 1).
Kết quả, tất cả các số tự nhiên n cần tìm có dạng 22020 k − 1, ở đây k nguyên dương.
2 Ta nhận thấy p là số lẻ, vậy nên
( ( (
p | (2n + 2) p | (n + 1) p | (n + 1)
2
⇒ 2
⇒ ⇒ p = 3.
p | 4n + 2n + 1 p | 4n + 2n + 1 p | [(4n − 2)(n + 1) + 3]

Ta giả sử a, b là số chính phương, khi đó ab cũng là số chính phương. Đặt ab = m2 , ta


được
2n + 2 4n2 + 2n + 1
m2 = ⇔ 9m2 = (2n + 2) 4n2 + 2n + 1

·
3 3
⇔ 9m2 = (2n + 1)3 + 1
⇔ (3m − 1)(3m + 1) = (2n + 1)3 .
Nhận xét được m chẵn chỉ ra cho ta (3m + 1, 3m − 1) = 1. Theo đó, tồn tại các số
nguyên dương x, y sao cho
3m + 1 = x3 (1)

LATEX by Mathpiad 15
CHƯƠNG III. LỜI GIẢI THAM KHẢO

3m − 1 = y3 (2)
Trừ theo vế (1) cho (2), ta được
 (
x−y = 1
(
 2 2
x + xy + y = 2 x=1
(x − y)(x2 + xy + y2 ) = 2 ⇒ 

(
 x−y = 2 ⇒ .
 y = −1
x2 + xy + y2 = 1

Các số x, y không thể âm, thế nên suy luận trên là vô lí. Giả sử phản chứng là sai, và
bài toán được chứng minh.

d Câu 2 Tìm tất cả các số nguyên dương m, n thỏa mãn m(m + 1)(m + 2) = n2 .

Hà Tĩnh

Lời giải.
Giả sử tồn tại các số nguyên m, n thỏa đề.
Hai số m + 1 và m(m + 2) không thể đồng thời bằng 0, chứng tỏ chúng tồn tại ước chung lớn
nhất, gọi là d. Ta có
( (
d | (m + 1) d | (m + 1)
⇒  ⇒ d | 1 ⇒ d = 1.
d | (m + 1)2 − 1

d | m(m + 2)

Như vậy, (m + 1, m(m + 2)) = 1. Lại do |m + 1||m(m + 2)| = n2 nên cả |m + 1| và |m(m + 2)|
đều là số chính phương.

1 Với m = −1, ta tìm được n = 0.

2 Với m 6= −1, do m là số nguyên nên m(m + 2) > 0. Lúc này, |m(m + 2)| = m(m + 2)
là số chính phương. Tiếp tục đặt m(m + 2) = x2 , với x là số nguyên dương, ta được

m(m + 2) = x2 ⇒ m2 + 2m + 1 = x2 + 1
⇒ (m + 1)2 − x2 = 1
⇒ (m + 1 − x)(m + 1 + x) = 1.

Đến đây, ta xét các trường hợp sau.

† Trường hợp 1. Với m + 1 − x = m + 1 + x = 1, ta có m = 0 và x = 0. Thử trực


tiếp, ta tìm ra n = 0.
† Trường hợp 2. Với m + 1 − x = m + 1 + x = −1, ta có m = −1 và x = 0. Thử
trực tiếp, ta tìm ra n = 0.

Kết quả, có ba cặp số nguyên (m, n) thỏa đề là (−2, 0), (−1, 0) và (0, 0). ∇

LATEX by Mathpiad 16
MATHPIAD − TẠP CHÍ TOÁN HỌC

d Câu 3

1 Tìm tất cả các nghiệm (x, y) của phương trình x2 − 2x + 2y = 2(xy + 1).

2 Cho p là số nguyên tố sao cho tồn tại các số nguyên dương x, y thỏa mãn x3 +
y3 − p = 6xy − 8. Tìm giá trị lớn nhất của p.

Lào Cai

Lời giải.
1 Phương trình đã cho tương đương
x2 − 2x − 2 = 2y(x − 1).
Dựa vào điều hiển nhiên là x 6= 1, ta suy ra x2 − 2x − 2 chia hết cho x − 1. Ta nhận thấy
x2 − 2x − 2 = (x − 1)2 − 3,
thế nên x − 1 là ước của 3. Ta lập bảng
x−1 −3 −1 1 3
x −2 0 2 4
y −1 1 −1 1

Kết quả, phương trình đã cho có 4 nghiệm nguyên là (−2, −1), (0, 1), (2, −1), (4, −1).
2 Với các số x, y, p thỏa mãn giả thiết, ta có
x3 + y3 + 23 − 3 · x · y · 2 = p ⇔ (x + y + 2) x2 + y2 + 4 − xy − 2x − 2y = p.


Do x, y nguyên dương nên ta được x + y + 2 > 1 từ lập luận trên, và như vậy
x + y + 2 = p,
x2 + y2 + 4 − xy − 2x − 2y = 1.
Ta nhận thấy
x2 + y2 + 4 − xy − 2x − 2y = 1 ⇔ (x − y)2 + (x − 2)2 + (y − 2)2 = 2.
Tổng ba bình phương trên bằng 2, chứng tỏ một bình phương phải bằng 0. Tới đây, ta
xét các trường hợp sau.
† Trường hợp 1. Với x = y, ta có p = x + y + 2 = 2x + 2. Ta được p chẵn và
p = 2.
† Trường hợp 2. Với x = 2, ta có

y=1
(2 − y)2 + (2 − 2)2 + (y − 2)2 = 2 ⇔ (y − 2)2 = 1 ⇔
y=3
Thử với từng trường hợp, ta được p = 5 và p = 7.
Tổng kết lại, p = 7 là số nguyên tố lớn nhất thỏa đề.

LATEX by Mathpiad 17
CHƯƠNG III. LỜI GIẢI THAM KHẢO

d Câu 4 Tìm tất cả bộ ba số nguyên tố (p, q, r) thỏa mãn pq = r + 1 và


2 2 2

2 p + q = r + 1.

Quảng Nam

Lời giải.
Ta chia bài toán thành các trường hợp sau.

† Trường hợp 1. Với p, q, r là ba số nguyên tố lẻ, ta có pq lẻ, còn r + 1 chẵn. Điều này
vô lí.

† Trường hợp 2. Với r = 2, ta có pq = 3. Điều này vô lí.

† Trường hợp 3. Với q = 2, ta thu được hệ phương trình


( ( (
2p = r + 1 r = 2p − 1 r = 2p − 1
2
 2
⇔ 2
 2
⇔ .
2 p +4 = r +1 2 p + 4 = (2p − 1) + 1 (p − 3)(p + 2) = 0

Do p nguyên tố, ta được p = 3 và r = 2.

† Trường hợp 4. Với p = 2, làm tương tự trường hợp trên, ta được q = 3 và r = 2.

Kết quả, có 2 bộ (p, q, r) thỏa đề là (2, 3, 2) và (3, 2, 2). ∇

d Câu 5 Tìm tất cả số tự nhiên a và b với lớn hơn 1 cho (a − 1)(b − 1) | (ab − 1).

Chuyên Đại học Khoa học Huế

Lời giải.
Từ giả thiết, ta có

(a − 1)(b − 1) | (ab − 1) ⇒ (a − 1)(b − 1) | [(a − 1)(b − 1) + a + b − 2]


⇒ (a − 1)(b − 1) | [(a − 1) + (b − 1)]

Ta được a − 1 và b − 1 chia hết cho nhau từ đây, tức là a = b. Sử dụng kết quả này cho
(a − 1)(b − 1) | (ab − 1), ta có

(a − 1)2 | a2 − 1 ⇒ (a − 1) | (a + 1) ⇒ (a − 1) | 2.


Lập luận trên chứng tỏ a = 2 hoặc a = 3.


Thử trực tiếp, ta kiểm tra được có 2 cặp (a, b) thỏa mãn là (2, 2) và (3, 3). ∇

d Câu 6 Giả sử n là số tự nhiên thỏa mãn điều kiện n(n + 1) + 7 không chia hết
cho 7. Chứng minh rằng 4n3 − 5n − 1 không là số chính phương.

Thái Bình

LATEX by Mathpiad 18
MATHPIAD − TẠP CHÍ TOÁN HỌC

Lời giải.
Từ giả thiết n(n + 1) + 7 không chia hết cho 7, ta suy ra n + 1 không chia hết cho 7. Mặt
khác, ta xét phân tích
4n3 − 5n − 1 = (n + 1) 4n2 − 4n − 1 .


Giả sử 4n3 − 5n − 1 là số chính phương. Đặt d = n + 1, 4n2 − 4n − 1 , ta có




( (
d | (n + 1) d | (n + 1)
2
⇒ ⇒ d | 7.
d | 4n − 4n − 1 d | [4n(n + 1) − 8(n + 1) + 7]

Tuy nhiên, do n + 1 không là bội của 7 nên d = 1, dẫn đến việc 4n2 − 4n − 1 là số chính
phương. Đây là điều không thể xảy ra, do

4n2 − 4n − 1 ≡ 3 (mod 4).

Giả sử phản chứng là sai, và ta có điều phải chứng minh. ∇

d Câu 7

1 Tìm tất cả các cặp số nguyên (x, y) thỏa mãn x2 + 2y2 − 2xy − 2x − 4y + 6 = 0.

p2 − p
2 Tìm tất cả các số nguyên tố p sao cho − 1 là lập phương một số tự nhiên.
2

Thanh Hóa − Chuyên Toán

Lời giải.

1 Phương trình đã cho tương đương

2y2 − (2x + 4)y + x2 − 2x + 6 = 0.




0
Coi đây là một phương trình bậc hai theo ẩn y, thế thì ∆ phải là số chính phương. Ta
tính được
0
∆ = (x + 2)2 − 2 x2 − 2x + 6 = −x2 + 8x − 8.


Ta suy ra x2 − 8x + 8 6 0. Giải bất phương trình nghiệm nguyên này, ta được 2 6 x 6 6.


Thử với từng trường hợp, ta kết luận phương trình đã cho có 4 nghiệm nguyên phân
biệt là
(2, 1), (2, 3), (6, 3), (6, 5).

p2 − p
2 Ta đặt − 1 = n3 , với n là một số tự nhiên. Phép đặt này cho ta
2
p2 − p − 2 = 2n3 ⇔ p2 − p = 2n3 + 2 ⇔ p(p − 1) = 2(n + 1) n2 − n + 1 .


Ta được p | 2(n + 1) n2 − n + 1 . Ta xét các trường hợp sau.




† Trường hợp 1. Với p = 2, bằng kiểm tra trực tiếp, ta thấy thỏa mãn đề bài.

LATEX by Mathpiad 19
CHƯƠNG III. LỜI GIẢI THAM KHẢO

† Trường hợp 2. Với p | (n + 1), ta có p 6 n + 1. Đánh giá này cho ta


2(n + 1) n2 − n + 1 = p(p − 1) 6 (n + 1)n.


Ta được 2n2 − 2n + 1 6 n. Không tồn tại số thực n nào như vậy.


† Trường hợp 3. Với p | n2 − n + 1 , ta đặt n2 −n+1 = kp, ở đây k là số nguyên


dương.
Phép đặt này cho ta
p(p − 1) = 2(n + 1) n2 − n + 1 = 2(n + 1)kp.


Từ đây, ta có p = 2kn + 2k + 1. Thế ngược lại phép đặt, ta chỉ ra


n2 − n + 1 = k(2kn + 2k + 1) ⇔ n2 − 2k2 + 1 n − 2k2 + k − 1 = 0.
 

Coi phương trình trên là một phương trình bậc hai ẩn n, lúc này
2
∆ = 2k2 + 1 + 4 2k2 + k − 1


phải là số chính phương.


2 2
Đánh giá được 2k2 + 1 < ∆ < 2k2 + 4 và ∆ lẻ, ta áp dụng bổ đề kẹp để suy
2
ra ∆ = 2k2 + 3 . Ta lần lượt tìm được k = 3, n = 20, p = 127.
Như vậy, có 2 giá trị của p thỏa đề là p = 2 và p = 127.

d Câu 8

1 Tìm tất cả các bộ ba số nguyên dương (x, y, z) thỏa mãn đồng thời các điều kiện

√ √ √ 1 1 1
xy + xz − yz = y, + − = 1.
x y z

2 Cho số tự nhiên n > 2 và số nguyên tố p thỏa mãn p − 1 chia hết cho n và n3 − 1


chia hết cho p. Chứng minh rằng n + p là một số chính phương.

Thanh Hóa − Chuyên Tin

Lời giải.
1 Điều kiện thứ nhất tương đương với
√ √ √ √ 
y+ z y − x = 0 ⇔ x = y.
Thay vào điều kiện còn lại, ta được
2 1
− = 1 ⇔ 2z − x = xz ⇔ xz + x − 2z − 2 = −2
x z
⇔ x(z + 1) − 2(z + 1) = −2 ⇔ (2 − x)(z + 1) = 2.
Từ đây, ta thu được x = 1, z = 1, và đương nhiên y = 1. Kết luận (x, y, z) = (1, 1, 1) là
bộ số duy nhất thỏa đề.

LATEX by Mathpiad 20
MATHPIAD − TẠP CHÍ TOÁN HỌC

2 Để giải bài toán này, ta xét các trường hợp sau.

† Trường hợp 1. p | (n − 1).


Ta đặt n = l p + 1, với l là số tự nhiên. Kết hợp với giả thiết n | (p − 1), phép đặt
này cho ta

(l p + 1) | (p − 1) ⇒ p − 1 > l p + 1 ⇒ (l − 1)p 6 −2 ⇒ l = 0, p = 2.

Với l = 0, p = 2, ta tìm ra n = 1, trái giả thiết n > 2.


† Trường hợp 2. p|(n2 + n + 1).
Do giả thiết n|(p − 1), ta có thể đặt p = kn + 1, thế thì

n2 + n + 1 = n2 + n − kn + kn + 1 = n(n − k + 1) + kn + 1

là bội của kn + 1, thế nhưng do (n, kn + 1) = 1 nên (kn + 1)|(n − k + 1).


Bằng dãy đánh giá

−kn − 1 6 −k − 1 < n − k + 1 6 n 6 kn < kn + 1,

ta chỉ ra k = n + 1, tức là p = n2 + n + 1.
Như vậy, n + p = n + n2 + n + 1 = (n + 1)2 là số chính phương. Chứng minh hoàn
tất.

d Câu 9

1 Tìm tất cả các số nguyên dương n để n − 1989 và n − 2022 đều là các số chính
phương.

2 Biết rằng phương trình x2 − ax + b + 2 = 0 (với a, b là các số nguyên) có hai


nghiệm nguyên. Chứng minh rằng 2a2 + b2 là hợp số

Quảng Trị

Lời giải.

1 Từ giả thiết, ta có thể đặt n − 1989 = x2 , n − 2022 = y2 , ở đây x, y là các số nguyên


dương. Lấy hiệu theo vế, ta được

(n − 1989) − (n − 2022) = x2 − y2 ⇒ 33 = (x − y)(x + y)

Vì 0 < x − y < x + y nên đến đây, ta xét các trường hợp.

† Trường hợp 1. Với x − y = 1 và x + y = 33, ta có x = 17, và vì thế n =


1989 + 172 = 2278.
† Trường hợp 2. Với x−y = 3 và x+y = 11, ta có x = 7, và vì thế n = 1989+72 =
2038.

LATEX by Mathpiad 21
CHƯƠNG III. LỜI GIẢI THAM KHẢO

Như vậy, n = 2038 và n = 2278 là tất cả các giá trị thỏa đề.

2 Gọi 2 nghiệm nguyên của phương trình đã cho là x1 và x2 . Theo định lí Viète, ta có
( (
x1 + x2 = a a = x1 + x2
⇒ .
x1 x2 = b + 2 b = x1 x2 − 2

Các hệ thức trên cho ta

2a2 + b2 = 2 (x1 + x2 )2 + (x1 x2 − 2)2 = x12 x22 + 2x12 + 2x22 + 4 = x12 + 2 x22 + 2 .
 

Do x12 + 2 > 2 và x22 + 2 > 2, ta được 2a2 + b2 là hợp số. Bài toán được chứng minh.

d Câu 10 Cho hình lăng trụ đứng, đáy là tam giác vuông, chiều cao bằng 6. Số
đo ba cạnh của tam giác đáy là các số nguyên. Số đo diện tích toàn phần của lăng trụ
bằng số đo thể tích của lăng trụ. Tính số đo ba cạnh tam giác đáy của lăng trụ.

Quảng Ninh

Lời giải.
Gọi số đo ba cạnh của tam giác đáy là a, b, c với a, b, c ∈ Z+ , c > b > a.
Tam giác đáy là tam giác vuông, nên theo định lý Pythagoras, ta có

a2 + b2 = c2 . (1)

Số đo thể tích của lăng trụ bằng 3ab, trong khi số đo diện tích của nó bằng 6(a + b + c) + ab.
Vậy nên

6(a + b + c) + ab = 3ab ⇔ 3(a + b + c) = ab ⇔ ab − 3a − 3b = 3c. (2)

Nhân đôi hai vế của (2) rồi cộng tương ứng vế với (1), ta được

a2 + b2 + 2ab − 6a − 6b = c2 + 6c ⇔ (a + b)2 − 6(a + b) = c2 + 6c


⇔ (a + b)2 − 6(a + b) + 9 = c2 + 6c + 9
⇔ (a + b − 3)2 = (c + 3)2

a+b−3 = c+3

a + b − 3 = −c − 3

c = a+b−6

a+b+c = 0

Do a, b, c nguyên dương nên ta loại trường hợp a + b + c = 0, tức là c = a + b − 6. Thế vào


(2), ta được
ab − 3a − 3b = 3a + 3b − 12 ⇔ (a − 6)(b − 6) = 6.
Giải phương trình ước số trên rồi thử lại, ta tìm ra có tổng cộng 18 bộ số đo 3 cạnh tam giác
đáy, bao gồm (7, 24, 25), (8, 15, 17)(9, 12, 15) và các hoán vị của chúng. ∇

LATEX by Mathpiad 22
MATHPIAD − TẠP CHÍ TOÁN HỌC

d Câu 11

1 Tìm tất cả số nguyên x, y thỏa mãn bất đẳng thức 5x2 +3y2 +4xy−2x +8y+8 6
0.

2 Trong 2021 số nguyên dương đầu tiên, có bao nhiêu số không chia hết cho 7 và
không chia hết cho 11?

Đồng Nai

Lời giải.

1 Bất phương trình đã cho tương đương

(2x + y)2 + (x − 1)2 + 2(y + 2)2 6 1. (*)

Tổng ba bình phương trong vế trái của (*) không vượt quá 1, chứng tỏ có ít nhất hai
bình phương bằng 0. Mặt khác, do

2x + y = 2(x − 1) + (y + 2)

nên nếu 2 trong 3 số kia bằng 0, số còn lại chắc chắn cũng bằng 0. Ta suy ra

2x + y = x − 1 = y + 2 = 0 ⇒ x = 1, y = −2.

Như vậy (x, y) = (1, −2) là nghiệm nguyên duy nhất của bất phương trình.

2 Ta kí hiệu [x] là số nguyên lớn nhất không vượt quá với x. Trong 2021 số nguyên dương
đầu tiên, ta gọi

† A là tập các số chia hết cho 7.


† B là tập các số chia hết cho 77.
† C là tập các số chia hết cho 7 và không chia hết cho 11.

Dựa vào nhận xét B ∪C = A và B ∩C = ∅, ta suy ra


   
2021 2021
|C| = |A| − |B| = − = 288 − 26 = 262.
7 77

Như vậy, trong 2021 số nguyên dương đầu tiên có tất cả 262 số chia hết cho 7 và không
chia hết cho 11.

d Câu 12 Tìm tất cả các cặp số nguyên (x, y) thỏa mãn (xy − 1)2 = x2 + y2 .

Bà Rịa - Vũng Tàu

LATEX by Mathpiad 23
CHƯƠNG III. LỜI GIẢI THAM KHẢO

Lời giải.
Phương trình đã cho tương đương
(xy)2 + 1 = x2 + y2 + 2xy ⇔ (xy)2 + 1 = (x + y)2 ⇔ 1 = (x + y − xy)(x + y + xy)
Tới đây, ta xét các trường hợp sau.
† Trường hợp 1. Với x + y − xy = x + y + xy = 1, ta có xy = 0 và x + y = 1.
Giải ra, ta được (x, y) = (0, 1) hoặc (x, y) = (1, 0).
† Trường hợp 2. Với x + y − xy = x + y + xy = −1, ta có xy = 0 và x + y = −1.
Giải ra, ta được (x, y) = (0, −1) hoặc (x, y) = (−1, 0).
Như vậy, phương trình đã cho có 4 nghiệm nguyên phân biệt, bao gồm
(−1, 0), (0, −1), (0, 1), (1, 0).

d Câu 13 Giải phương trình nghiệm nguyên x2 y − xy + 2x − 1 = y2 − xy2 − 2y.

Bến Tre

Lời giải.
Phương trình đã cho tương đương với
x2 y + xy2 − xy − y2 + 2x + 2y = 1 ⇔ xy(x + y) − y(x + y) + 2(x + y) = 1
⇔ (xy − y + 2)(x + y) = 1.
Tới đây, ta xét các trường hợp sau.
† Trường hợp 1. Với x + y = xy − y + 2 = 1, ta có
( (
x+y = 1 y = −1 − x

xy − y + 2 = 1 x(−1 − x) − (−1 − x) + 2 = −1
(
y = 1−x

x = −2, y = 3
⇔ ⇔
(x + 2)(x − 2) = 0 x = 2, y = −1

† Trường hợp 2. Với x + y = xy − y + 2 = −1, ta có


( (
x + y = −1 y = 1−x

xy − y + 2 = −1 x(1 − x) − (1 − x) + 2 = −1
(
y = 1−x

x = 0, y = 1
⇔ 2 ⇔
x − 2x − 2 = 0 x = 2, y = −1

Như vậy, phương trình đã cho có 4 nghiệm nguyên phân biệt, bao gồm
(−2, 3), (0, 1), (2, 1), (2, 3).

LATEX by Mathpiad 24
MATHPIAD − TẠP CHÍ TOÁN HỌC

d Câu 14

1 Tìm x, y ∈ N sao cho x3 = 1993 · 3y + 2021.


n − 23
2 Tìm số nguyên dương n để là bình phương một số hữu tỉ dương.
n + 89

Nghệ An

Lời giải.

1 Dựa vào tính chất đã biết x3 ≡ 0, 1, 8 (mod 9), ta có các đánh giá

1993 · 3y + 2021 ≡ 0, 1, 8 (mod 9) ⇒ 4 · 3y ≡ 3, 4, 5 (mod 9).

Ta xét các trường hợp kể trên.

† Trường hợp 1. Với 4.3y ≡ 3 (mod 9), ta được y = 1. Thay ngược lại, ta tìm ra
x = 20.
† Trường
√ hợp 2. Với 4.3y ≡ 4 (mod 9), ta được y = 0. Thay ngược lại, ta tìm ra
3
x = 4041, không là số nguyên.
† Trường hợp 3. Với 4.3y ≡ 5 (mod 9), ta không tìm được y nguyên dương thỏa
mãn.

Như vậy, cặp số (x, y) duy nhất thỏa mãn là (x, y) = (20, 1).
n − 23
2 Vì là bình phương một số hữu tỉ dương nên n > 23.
n + 89
n − 23  a 2
Lúc này, ta có thể đặt = , trong đó a, b ∈ N∗ , (a, b) = 1, a < b.
n + 89 b
Vì (a, b) = 1 nên (a2 , b2 ) = 1. Do đó, tồn tại số nguyên dương k sao cho
(
n − 23 = a2 k, (1)
n + 89 = b2 k, (2)

Trừ tương ứng vế của (1) cho (2), ta suy ra

(n + 89) − (n − 23) = (b2 − a2 )k ⇔ 112 = (b + a)(b − a)k.

Ta có các đánh giá:

i, b + a và b − a đều là ước dương của 112.


ii, b + a và b − a cùng tính chẵn lẻ.
iii, b + a > b − a > 0.

Dựa vào đây, ta lập được bảng giá trị sau

LATEX by Mathpiad 25
CHƯƠNG III. LỜI GIẢI THAM KHẢO

b+a b−a k b a n = a2 k + 23
28 4 1 16 12 loại vì (a, b) > 1
56 2 1 29 27 752
14 8 1 11 3 32
14 4 2 9 5 73
28 2 2 15 13 361
8 2 7 5 3 86
14 2 4 8 6 loại vì (a, b) > 1
4 2 14 3 1 37
7 1 16 4 3 167

Kết quả, có 7 giá trị của n thỏa mãn đề bài, gồm

n = 32, n = 37, n = 73, n = 86, n = 167, n = 361, n = 752.

d Câu 15

1 Cho a là số nguyên tố lẻ và a không chia hết cho 3. Chứng minh rằng a2 − 2021
chia hết cho 24

2 Cho các số nguyên tố p, q thỏa mãn p + q2 là số chính phương. Chứng minh


rằng

a, p = 2q + 1.
b, p2 + q2021 không phải là số chính phương.

3 Cho tập hợp S gồm n số nguyên dương đôi một khác nhau (n > 3) thỏa mãn
tính chất: tổng của 3 phần tử bất kì trong S đều là số nguyên tố. Tìm giá trị lớn
nhất có thể của n.

Quảng Ngãi

Lời giải.
1 Do a2 ≡ 0, 1, 4 (mod 8) và a lẻ, ta suy ra a2 ≡ 1 (mod 8).
Mặt khác, do a2 ≡ 0, 1 (mod 3) và a không chia hết cho 3, ta suy ra a2 ≡ 1 (mod 3).
Hai lập luận trên kết hợp với việc (3, 8) = 1 chỉ ra cho ta

a2 ≡ 1 (mod 24) ⇒ 24 | a2 − 1 .


Tuy nhiên, do 2021 − 1 = 2020 không chia hết cho 11 nên 24 không là ước của a2 −
2021. Bài toán được chứng minh.

2 a, Từ giả thiết, ta có thể đặt p + q2 = a2 , với a nguyên dương. Phép đặt trên cho ta

p = a2 − q2 ⇒ p = (a − q)(a + q).

LATEX by Mathpiad 26
MATHPIAD − TẠP CHÍ TOÁN HỌC

Do p nguyên tố và 0 < a − q < a + q, ta suy ra a − q = 1, còn a + q = p. Lấy hiệu


theo vế, ta được p = 2q + 1.
b, Giả sử (2q + 1)2 + q2021 là số chính phương. Theo đó, ta có thể đặt (2q + 1)2 +
q2021 = b2 , với b là số nguyên dương.
Phép đặt này cho ta

q2021 = b2 − (2q + 1)2 ⇒ q2021 = (b − 2q − 1)(b + 2q + 1).

Ta xét các trường hợp sau.


† Trường hợp 1. Nếu b − 2q − 1 và b + 2q + 1 có ước nguyên tố chung là r, ta
suy ra

r | (b − 2q − 1)
( (
r | (4q + 2) r|2

r | (b + 2q + 1) ⇒ ⇒ ⇒ q = r = 2.
 r | q r | q
r | q2021

Lúc này, (2q + 1)2 + q2021 = 52 + 22021 . Số này chia cho 5 dư 2, do vậy nó
không là số chính phương.
† Trường hợp 2. Nếu b − 2q − 1 và b + 2q + 1 nguyên tố cùng nhau, ta suy ra
(
b − 2q − 1 = 1
⇒ 4q + 2 = q2021 − 1 ⇒ 4q + 3 = q2021 .
b + 2q + 1 = q2021

Xét tính chia hết cho q ở cả hai vế, ta được q = 3. Thay trở lại q = 3, ta có
15 = 32021 , một điều vô lí.
Mâu thuẫn chỉ ra ở tất cả các trường hợp chứng tỏ giả sử phản chứng là sai. Bài
toán được chứng minh.

3 Ta sẽ đi chứng minh max n = 4. Thật vậy, ta giả sử trong S tồn tại 5 số nguyên dương
a, b, c, d, e thỏa mãn tổng của 3 số bất kì trong 5 số này là số nguyên tố. Gọi số dư của
chúng khi chia cho 3 lần lượt là m, n, p, q, r. Xét xâu

T = (m, n, p, q, r).

Theo đó, tổng 3 phần tử bất kì trong xâu không chia hết cho 3. Ta sẽ tìm cách phản
chứng khẳng định này.

† Nếu xâu T chứa đầy đủ các số 0, 1, 2 thì tổng của 3 phần tử đó chia hết cho 3,
mâu thuẫn.
† Nếu xâu T không chứa một số nào đó trong 0, 1, 2 (giả sử là 2) thì T chỉ có thể
chứa 0 hoặc 1. Do độ dài của T bằng 5 nên theo nguyên lí Dirichlet, tồn tại 3 số
giống nhau trong T. Tuy nhiên, tổng của 3 số này chia hết cho 3, một điều mâu
thuẫn.

LATEX by Mathpiad 27
CHƯƠNG III. LỜI GIẢI THAM KHẢO

Phản chứng ở trên cho ta n 6 4. Ta sẽ xây dựng một tập S thỏa đề. Thật vậy, tập
S = {1; 3; 7; 9} là tập thỏa đề do

1 + 3 + 7 = 11, 1 + 3 + 9 = 13,
1 + 7 + 9 = 17, 3 + 7 + 9 = 19

đều là các số nguyên tố. Kết quả, max n = 4.


d Câu 16

1 Chứng minh rằng tổng các bình phương của 6 số nguyên liên tiếp không thể là
số chính phương.

2 Tìm các nghiệm nguyên dương của phương trình x2 y + 2xy + y = 32x.

Vĩnh Long

Lời giải.
1 Xét 6 số nguyên liên tiếp n−2, n−1, n, n+1, n+2, n+3. Tổng bình phương của chúng

(n − 2)2 + (n − 1)2 + n2 + (n + 1)2 + (n + 2)2 + (n + 3)2 = 6n(n + 1) + 19.

Do n(n + 1) là tích của hai số nguyên dương liên tiếp nên n chẵn. Từ đây ta có

6n(n + 1) + 19 ≡ 3 (mod 4).

Không có số chính phương nào đồng dư 3 theo modulo 4, chứng tỏ tổng đã cho không
là số chính phương.
2 Phương trình đã cho tương đương với

y(x + 1)2 = 32x. (*)

Xét tính chia hết của (*), ta có

(x + 1) | 32x ⇒ (x + 1) | [32(x + 1) − 32] ⇒ (x + 1) | 32.

Ta được x + 1 là ước nguyên dương lớn hơn 1 của 32. Ta lập bảng giá trị sau

x+1 2 4 8 16 32
x 1 3 7 15 31
y 8 6 3, 5 1, 875 0, 96875

Căn cứ vào bảng giá trị được lập, ta kết luận phương trình đã cho có 2 nghiệm nguyên
phân biệt là (1, 8) và (3, 6).

LATEX by Mathpiad 28
MATHPIAD − TẠP CHÍ TOÁN HỌC

b
d Câu 17 Cho ba số nguyên a, b, c thỏa mãn a = b − c = . Chứng minh rằng
c
a + b + c là lập phương của một số nguyên.

Bình Dương

Lời giải.
b
Xuất phát từ b − c = , ta có
c

c(b − c) = b ⇒ bc − c2 = b ⇒ b(c − 1) = c2 . (*)

Ta được c2 chia hết cho c − 1, chứng tỏ

c − 1 | c2 − 1 + 1 = (c − 1)(c + 1) + 1.

Do c − 1 là ước của 1 và c 6= 0, ta có c = 2 hoặc c = −2. Thế vào (*), ta suy ra b = 4. Các


kết quả này cho ta
a + b + c = b − c + b + c = 2b = 8.
Ta nhận ra a + b + c là số lập phương. Bài toán được chứng minh. ∇

d Câu 18

1 Cho m, p, r là các số nguyên tố thỏa mãn mp + 1 = r. Chứng minh rằng m2 + r


hoặc p2 + r là số chính phương.

2 Tìm tất cả các số nguyên tố q sao cho tồn tại các số nguyên dương n để n2 + 22q
là một lũy thừa với số mũ nguyên dương của 11.

Kiên Giang

Lời giải.

1 Không mất tính tổng quát, ta giả sử m > p. Ta xét các trường hợp sau.

† Trường hợp 1. Nếu m, p, r cùng lẻ, ta có mp + 1 chẵn, nhưng r lẻ, vô lí.


† Trường hợp 2. Nếu r = 2, ta có mp + 1 = 2, hay là mp = 1, vô lí.
† Trường hợp 3. Nếu p = 2, ta có r = 2m + 1. Ta nhận thấy rằng

m2 + r = m2 + 2m + 1 = (m + 1)2

là một số chính phương. Bài toán được chứng minh.

2 Từ giả thiết, ta có thể đặt n2 + 22q = 11m , với m là số nguyên dương.


Ta lần lượt suy ra được

11 | 11m ⇒ 11 | n2 + 22q ⇒ 11 | n2 ⇒ 11 | n ⇒ 121 | n2 .

LATEX by Mathpiad 29
CHƯƠNG III. LỜI GIẢI THAM KHẢO

Rõ ràng m > 2, thế nên 121 | 11m . Kết hợp với 121 | n2 , ta suy ra 121 | 22q, tức q = 11.
Cuối cùng, ta sẽ chỉ ra một số nguyên dương n thỏa mãn. Với n = 33 và q = 11, ta có

n2 + 22q = 332 + 22.11 = 113 .

Kết luận, q = 11 là số nguyên tố cần tìm

d Câu 19 Hỏi có bao nhiêu số tự nhiên n không vượt quá 2021 mà n3 + 2021 chia
hết cho 6.

Hòa Bình

Lời giải.
Với số tự nhiên n thỏa mãn n3 + 2021 chia hết cho 6, ta chứng minh được

† n lẻ (thông qua việc xét tính chẵn, lẻ của n).

† n ≡ 1 (mod 3) (thông qua việc xét số dư của n khi chia cho 3).

Các khẳng định trên cho ta n ≡ 1 (mod 6). Như vậy, số các số tự nhiên n thỏa đề chính là số
các số tự nhiên chia 6 dư 1 không vượt quá 2021, và bằng

2017 − 1
+ 1 = 337.
6

d Câu 20 Tìm tất cả các số nguyên x, y thỏa mãn y2 + 3y = x4 + x2 + 18.

Ninh Thuận

Lời giải.
Phương trình đã cho tương đương với

4y2 + 12y = 4x4 + 4x2 + 72 ⇔ 4y2 + 12y + 9 = 4x4 + 4x2 + 1 + 80


2
⇔ (2y + 3)2 = 2x2 + 1 + 80
⇔ 2y − 2x2 + 2 2y + 2x2 + 4 = 80
 

⇔ y − x2 + 1 y + x2 + 2 = 20
 

Ta có các đánh giá.

† y − x2 + 1 < y + x2 + 2.

† y − x2 + 1 và y + x2 + 2 khác tính chẵn lẻ.

Dựa vào đây, ta lập được bảng giá trị sau

LATEX by Mathpiad 30
MATHPIAD − TẠP CHÍ TOÁN HỌC

y − x2 + 1 y + x2 + 2 y x
−20 −1 −12 ±3
−5 −4 −6 0
1 20 9 ±3
4 5 3 0
Kết quả, phương trình đã cho có 6 nghiệm (x, y) phân biệt, bao gồm

(−3, −12), (−3, −9), (0, −6), (0, 3), (3, 9), (3, 12).

d Câu 21 Tìm tất cả các số nguyên dương x, y thỏa mãn x2 − 2y · x − 421 · 9 = 0.

Thừa Thiên Huế − Chuyên Toán

Lời giải.
Với mỗi số nguyên dương x, luôn tồn tại các số nguyên dương z và số nguyên dương lẻ t sao
cho x = 2zt. Bằng cách đặt như vậy, phương trình đã cho trở thành

22zt 2 − 2y+zt − 9 · 421 = 0.

Phương trình kể trên tương đương với

22zt t − 2y−z = 9 · 421



(*)

Trong hai vế của (*), ta sẽ xét số mũ của lũy thừa cơ số 2. Thật vậy
† Cả t và t − 2y−21 đều lẻ, thế nên số mũ của 2 ở vế trái là 2z.
† Số mũ của 2 ở vế phải là 2 · 21 = 42.
Do vậy, z = 21. Thay z = 21 vào (*), ta được t t − 2y−21 = 9. Ta có đánh giá


0 < t − 2y−21 < t.

Đánh giá trên cho ta t = 8 và 2y−21 = 8, tức y = 24. Kết luận, (x, y) = 9 · 221 , 24 là cặp số


duy nhất thỏa đề. ∇

d Câu 22
1 Tìm tất cả các số nguyên x, y thỏa mãn điều kiện

x2 y − y2 x − 2x2 − 3y2 + 10xy − 16x + 21y = 100.

2 Chứng minh rằng từ 1012 số nguyên bất kì, luôn tồn tại hai số mà hiệu bình
phương của chúng là một số nguyên chia hết cho 2021.

Thừa Thiên Huế − Chuyên Tin

Lời giải.

LATEX by Mathpiad 31
CHƯƠNG III. LỜI GIẢI THAM KHẢO

1 Phương trình đã cho tương đương với

(x + 3)y2 − x2 + 10x + 21 y + 2x2 + 16x + 100 = 0




⇔ (x + 3)y2 − (x + 3)(x + 7)y + 2(x + 3)(x + 5) + 40 = 0


⇔ (x + 3) y2 − (x + 7)y + 2(x + 5) = −40
 

Ta được x + 3 là ước của 40. Bằng cách xét (khá nhiều) các trường hợp, ta tìm ra được
phương trình đã cho có tất cả 6 nghiệm (x, y) nguyên phân biệt, bao gồm

(−8, −5), (−8, 4), (−5, −5), (−5, 7), (4, 4), (4, 7).

p+1
2 Bổ đề. Với số nguyên tố lẻ p, các số chính phương khi chia cho p nhận tối đa
2
số dư.
Thật vậy, xét hai số chính phương a2 , b2 bất kì không cùng chia hết cho p. Ta đặt

a = pm ± i, b = pn ± j,
 
p−1
trong đó i, j thuộc tập 0; 1; . . . ; và không mất tổng quát, ta giả sử i > j.
2
Ta sẽ chứng minh a2 6≡ b2 (mod p). Ta có

a2 − b2 ≡ i2 − j2 = (i − j)(i + j) (mod p).

p−1
Với việc 0 6 j < j 6 , ta suy ra
2
0 < i − j < i + j < p − 1.

Lập luận trên chứng tỏ (i − j)(i + j) không chia hết cho p. Nói cách khác, a2 − b2
không chia hết cho p. Bổ đề được chứng minh.
Quay trở lại với bài toán. Ta có 2021 = 43 · 47. Số chính phương khi chia cho 43 chỉ
nhậntối đa 22 số dư, do vậy trong 1012 bình phương của 1012 số đã cho, tồn tại ít
1012
nhất = 46 số chính phương có cùng số dư khi chia cho 43.
22
Xét 46 số trên, khi chia cho 47 chỉ có nhiều nhất 24 số dư, do vậy tồn tại 2 số trong 46
số này có cùng số dư khi chia cho 47, giả sử là a2 và b2 . Khi đó,

43 | a2 − b2 , 47 | a2 − b2 .
 

Lại do (43, 47) = 1 nên 2021 | a2 − b2 . Bài toán được chứng minh.


d Câu 23 Tìm nghiệm nguyên của phương trình x2 − 2y(x − y) = 2(x + 1).

Tây Ninh

LATEX by Mathpiad 32
MATHPIAD − TẠP CHÍ TOÁN HỌC

Lời giải.
Phương trình đã cho tương đương với

2x2 − 4xy + 4y2 − 4x = 4 ⇔ (x − 2y)2 + (x − 2)2 = 8

Từ đây suy ra (x − 2)2 6 8 ⇒ (x − 2)2 ∈ {0, 1, 4}.

† Với (x − 2)2 = 0, thì (x − 2y)2 = 8, vô nghiệm.

† Với (x − 2)2 = 1, thì (x − 2y)2 = 7, vô nghiệm.

† Với (x−2)2 = 4, thì (x−2y)2 = 4, ta được các nghiệm (x, y) là (0, 1), (0, −1), (4, 1), (4, 3).

Vậy phương trình có các nghiệm (x, y) là (0, 1), (0, −1), (4, 1), (4, 3). ∇

d Câu 24 Cho m, n là các số nguyên dương sao cho m2 + n2 + m chia hết cho mn.
Chứng minh rằng m là số chính phương.

Tiền Giang

Lời giải.
Gọi ước chung lớn nhất của m và n là d. Ta đặt m = dx, n = dy, với (x, y) = 1. Phép đặt này
cho ta m2 + n2 − m = d 2 x2 + d 2 y2 − dx và xy = d 2 mn. Kết hợp với giả thiết, ta được

d 2 xy | d 2 x2 + d 2 y2 − dx ⇒ dxy | dx2 + dy2 − x .


 
(*)

Kết hợp (*) với việc xét tính chia hết cho x và d ở cả số bị chia và số chia, ta lần lượt suy ra
( (
x | dn2 x|d
⇒ ⇒ d = x.
d|x d|x

Ta có m = dx = d 2 là số chính phương. Bài toán được chứng minh. ∇

Bài toán này đã xuất hiện ở trong đề thi của Ba Lan.

Cho x, y là số nguyên dương sao cho x2 + y2 − x chia hết cho xy.


! Chứng minh x là số chính phương.

Polish Mathematical Olympiad 2000

d Câu 25 Tìm tất cả các cặp số nguyên (x, y) thỏa mãn x2 + 5y2 + 4xy + 4y + 2x −
3 = 0.

Cần Thơ

Lời giải.

LATEX by Mathpiad 33
CHƯƠNG III. LỜI GIẢI THAM KHẢO

Phương trình đã cho tương đương với

5y2 + (4x + 4)y + x2 + 2x − 3 = 0.




0
Coi đây là một phương trình bậc hai theo ẩn y, thế thì ∆ phải là số chính phương. Ta tính
được
0
∆ = (2x + 2)2 − 5 x2 + 2x − 3 = −x2 − 2x + 19.


Ta suy ra −x2 − 2x + 19 > 0. Giải bất phương trình nghiệm nguyên này, ta được −5 6 x 6 3.
Thử với từng trường hợp, ta kết luận phương trình đã cho có 4 nghiệm nguyên phân biệt là

(−5, 2), (−3, 0), (1, 0), (3, −2).

d Câu 26 Hai số tự nhiên khác nhau được gọi là "thân thiết" nếu tổng bình phương
của chúng chia hết cho 3. Hỏi tập X = {1; 2; 3; ...; 2021} có bao nhiêu cặp số "thân
thiết" (không phân biệt thứ tự)?

Khánh Hòa

Lời giải.
Trước hết, ta chứng minh bổ đề: Với mọi số nguyên a, b, ta phát biểu mệnh đề
(
3|a
⇔ 3 | a2 + b2 .

3|b

Thật vậy, ta chỉ cần chứng minh chiều đảo của mệnh đề trên. Ta đã biết, một số chính phương
khi chia cho 3 chỉ có thể dư 0 hoặc 1.
† Nếu a2 ≡ 1 (mod 3), ta có b2 ≡ 1 (mod 3), một điều vô lí.
† Nếu a2 ≡ 0 (mod 3), ta có b2 ≡ 0 (mod 3). Lúc này, cả a và b đều chia hết cho 3.
Mệnh đề trên được chứng minh, thế nên ta suy ra một cặp số "thân thiết" chính là một cặp
hai số cùng chia hết cho 3. Trong X, có tổng cộng 673 số tự nhiên chia hết cho 3, vậy nên số
cặp "thân thiết" là
673 · 672
= 226128.
2

Khi thay 3 bằng một số nguyên tố bất kì có dạng 4k + 3, dựa vào việc chứng minh bổ
đề (
p|a
⇔ p | a2 + b2 ,

p|b
!
bài toán cũng có hướng giải quyết tương tự.
Một trong những cách chứng minh bổ đề này chính là sử dụng định lí Fermat nhỏ. Về
chi tiết, mời bạn đọc tự tìm hiểu.

LATEX by Mathpiad 34
MATHPIAD − TẠP CHÍ TOÁN HỌC

d Câu 27

1 Cho phương trình bậc hai x2 − 2(m − 2)x + m2 − 3 = 0, với tham số m. Tìm tất
cả các giá trị nguyên của m để phương trình có 2 nghiệm phân biệt x1 , x2 thỏa
2x1 x2
mãn P = nhận giá trị nguyên.
x1 + x2
2 Tìm tất cả các nghiệm nguyên (x, y) của phương trình

x2 + 3y2 + 4xy − 2x − 4y − 2 = 0.

Đắk Nông

Lời giải.

1 Điều kiện để phương trình đã cho có hai nghiệm phân biệt x1 , x2 là

0 7
∆ > 0 ⇔ (m − 2)2 − m2 − 3 > 0 ⇔ −4m + 7 > 0 ⇔ m < .

4
Với điều kiện như vậy, áp dụng định lí Viète, ta có

x1 + x2 = 2(m − 2), x1 x2 = m2 − 3.

Như vậy, P là số nguyên khi và chỉ khi

2 m2 − 3

2x1 x2 m2 − 4 + 1 1
= = = m+2+ .
x1 + x2 2(m − 2) m−2 m−2

là số nguyên, tức là m − 2 là ước của 1. Ta tìm được m = 3, m = 1. Đối chiều với điều
7
kiện m < , ta chỉ ra m = 1 là số nguyên duy nhất thỏa mãn.
4
2 Phương trình đã cho tương đương với

(x + y)(x + 3y) − (x + y) − (x + 3y) + 1 = 3 ⇔ (x + y − 1)(x + 3y − 1) = 3.

Đến đây, ta lập bảng giá trị

x+y−1 −3 −1 1 3
x + 3y − 1 −1 −3 3 1
x −3 1 1 5
y 1 −1 1 −1

Kết quả, phương trình đã cho có 4 nghiệm nguyên phân biệt (−3, 1), (1, −1), (1, 1) và
(5, −1).

LATEX by Mathpiad 35
CHƯƠNG III. LỜI GIẢI THAM KHẢO

d Câu 28

1 Tìm nghiệm nguyên của phương trình (2x + y)(x − y) + 3(2x + y) − 5(x − y) =
22.

2 Cho hai số tự nhiên a, b thỏa mãn 2a2 + a = 3b2 + b. Chứng minh rằng 2a +
2b + 1 là số chính phương.

Bình Phước

Lời giải.

1 Phương trình đã cho tương đương với

(2x + y)(x − y) + 3(2x + y) − 5(x − y) − 15 = 7 ⇔ (2x + y − 5)(x − y + 3) = 7

Đến đây, ta lập bảng giá trị

2x + y − 5 −7 −1 1 7
x−y+3 −1 −7 7 1
x −2 −2 6∈ Z 6∈ Z
y 2 8 6∈ Z 6∈ Z

Kết quả, phương trình đã cho có 2 nghiệm nguyên phân biệt (−2, 2) và (−2, 8).

2 Với các số a, b thỏa mãn giả thiết, ta có

2a2 + a = 3b2 + b ⇒ 2a2 − 2b2 + a − b = b2


⇒ 2(a − b)(a + b) + (a − b) = b2
⇒ (a − b)(2a + 2b + 1) = b2 .

Ta đặt d = (a − b, 2a + 2b + 1). Do b2 = (a − b)(2a + 2b + 1) nên b2 chia hết cho d 2 ,


tức b chia hết cho d. Khi đó
  
d | (a − b)
 d | a
 d | a

d | (2a + 2b + 1) ⇒ d | (2a + 2b + 1) ⇒ d | 1 ⇒ d = 1.
  
d|b d|b d|b
  

Do vậy, ta được a − b và 2a + 2b + 1 đều là các số chính phương. Chứng minh hoàn


tất.

d Câu 29

1 Tìm tất cả các số tự nhiên n và k để n4 + 42k+1 là số nguyên tố.

LATEX by Mathpiad 36
MATHPIAD − TẠP CHÍ TOÁN HỌC

2 Tìm tất cả các số nguyên dương x, y thỏa mãn

x4 − x2 + 2x2 y − 2xy + 2y2 − 2y − 36 = 0.

Đắk Lắk

Lời giải.
1 Ta đặt 2k = m, khi đó 42k+1 = m4 . Ta nhận thấy rằng

n4 + 4m4 = n4 + 4n2 m2 + 4m4 − 4m2 n2


2
= n2 + 2m2 − (2mn)2
= n2 − 2mn + 2n2 n2 + 2mn + 2n2
 

Do 0 < n2 − 2mn + 2n2 < n2 + 2mn + 2n2 nên n4 + m4 là số nguyên tố chỉ khi
n2 − 2mn + 2n2 = 1. (*)
Theo chứng minh ở (*), ta có
" (
m = 0, |m − n| = 1 m=1
m2 + (m − n)2 = 1 ⇒ ⇒ ⇒ m = n = 1.
m = 1, |m − n| = 0 |m − n| = 0

Với m = n = 1, ta tìm được k = 0. Thử trên tiếp, ta thấy đúng. Như vậy, cặp (n, k) duy
nhất thỏa mãn yêu cầu đề bài là (1, 0).
2 Phương trình đã cho tương đương với
2
x2 + y − 1 + (x − y)2 = 37.
Có duy nhất một cách biểu diễn 37 thành tổng hai bình phương, đó là 372 = 1 + 62 .
Đồng thời, ta chỉ ra được x2 + y − 1 > x − y. Ta xét các trường hợp sau.
† Trường hợp 1. Với x2 + y − 1 = 6 và x − y = 1, ta có
x2 + (x − 1) − 1 = 6 ⇔ x2 + x − 8 = 0.
Ta không tìm được x nguyên dương ở đây.
† Trường hợp 2. Với x2 + y − 1 = 6 và x − y = −1, ta có
x2 + (x + 1) − 1 = 6 ⇔ x2 + x − 6 = 0 ⇔ (x + 3)(x − 2) = 0.
Do x nguyên dương, ta nhận được x = 2 và y = 3.
† Trường hợp 3. Với x2 + y − 1 = 1 và x − y = −6, ta có
x2 + (x + 6) − 1 = 1 ⇔ x2 + 4 = 0.
Ta không tìm được x nguyên dương ở đây.
Như vậy, cặp (x, y) duy nhất thỏa mãn yêu cầu đề bài là (2, 3).

LATEX by Mathpiad 37
CHƯƠNG III. LỜI GIẢI THAM KHẢO

d Câu 30 Tìm tất cả các số nguyên dương n sao cho hai số n2 − 2n − 7 và n2 −


2n + 12 đều là lập phương của một số nguyên dương nào đó.

Quảng Bình

Lời giải.
Từ giả thiết ta có thể đặt

n2 − 2n + 12 = a3 , n2 − 2n − 7 = b3 ,

ở đây a, b là các số nguyên dương, đồng thời a > b.


Lấy hiệu theo vế, ta được

19 = a3 − b3 ⇒ (a − b) a2 + ab + b2 = 19.


Do 0 < a − b < a2 + ab + b2 và 19 là số nguyên tố, ta nhận được a − b = 1 và a2 + ab + b2 =


19.
Thế b = a − 1 vào a2 + ab + b2 = 19, ta được

a2 + a(a − 1) + (a − 1)2 = 19 ⇒ 3a2 − 3a − 18 = 0 ⇒ 3(a − 3)(a + 2) = 0 ⇒ a = 3.

Thay trở lại a = 3, ta tìm ra n = 5. Đây là số nguyên dương duy nhất thỏa mãn yêu cầu. ∇

d Câu 31 Tìm các số nguyên dương n để A = 4n3 + 2n2 − 7n − 5 là số chính


phương.

Thái Nguyên − Chuyên Toán

Lời giải.
Giả sử A là số chính phương. Xét phân tích

A = (n + 1) 4n2 − 2n − 5 .


Ta đặt d = n + 1, 4n2 − 2n − 5 . Phép đặt này cho ta




( (
d | (n + 1) d | (n + 1)
⇒ ⇒ d = 1.
d | 4n2 − 2n − 5

d | [2(n + 1)(2n − 3) + 1]

Tích của hai số nguyên tố cùng nhau là một số chính phương, vậy nên cả 2 số này chính
phương. Tới đây, ta đánh giá

(2n − 2)2 < 4n2 − 2n − 5 < (2n)2 .

Do 4n2 − 2n − 5 chính phương, đánh giá trên cho ta

4n2 − 2n − 5 = (2n − 1)2 ⇔ 4n2 − 2n − 5 = 4n2 − 4n + 1 ⇔ n = 3.

Thử với n = 3, ta được A chính phương. Đây là giá trị duy nhất của n thỏa đề. ∇

LATEX by Mathpiad 38
MATHPIAD − TẠP CHÍ TOÁN HỌC

d Câu 32 Tìm tất cả các số nguyên tố p, q sao cho phương trình x2 − px + q = 0


có các nghiệm là số nguyên.

Thái Nguyên − Chuyên Tin

Lời giải.
Phương trình đã cho có nghiệm nguyên, thế nên ∆ = p2 − 4q phải là số chính phương.
Ta đặt p2 − 4q = a2 với a ∈ N, khi đó 4q = (p − a)(p + a).
Do p − a ≡ p + a (mod 2) và 0 < p − a < p + a nên ta có bảng sau

p−a 2 4 q
p+a 2q q 4
q+4 q+4
p q+1
2 2
Từ bảng trên, kết hợp với tính nguyên tố của p và q, ta tìm được p = 3, q = 2.
Thử lại, với p = 3, q = 2 phương trình có 2 nghiệm là x = 1 và x = 2, do vậy (p, q) = (3, 2)
là cặp duy nhất thỏa mãn yêu cầu đề bài. ∇

d Câu 33 Giải phương trình x3 + y2 − x + 3z = 2021 với x, y, z là các số nguyên.

Hà Nam

Lời giải.
Ta đã biết x3 − x = x(x + 1)(x − 1) chia hết cho 3 do đây là tích ba số nguyên liên tiếp. Trong
phương trình ban đầu, lấy đồng dư theo modulo 3 hai vế, ta được

y2 ≡ 2 (mod 3).

Đây là điều không thể xảy ra. Phương trình đã cho vô nghiệm. ∇

d Câu 34 Cho a, b, c là các số nguyên thỏa mãn a + b + 20c = c3 . Chứng minh


rằng a3 + b3 + c3 chia hết cho 6.

Lâm Đồng

Lời giải.
Ta nhận thấy

a3 + b3 + c3 = a3 + b3 + 2c3 − (a + b + 20c)
= (a3 − a) + (b3 − b) + 2(c3 − c) − 18c
= a(a − 1)(a + 1) + b(b − 1)(b + 1) + 2c(c − 1)(c + 1) − 18c.

Các tích dạng x3 − x = x(x − 1)(x + 1) chia hết cho 6 do đây là tích ba số nguyên liên tiếp,
và 18c cũng chia hết cho 6. Từ đây, ta suy ra điều phải chứng minh. ∇

LATEX by Mathpiad 39
CHƯƠNG III. LỜI GIẢI THAM KHẢO

d Câu 35

1 Tìm tất cả các số tự nhiên n sao cho n4 + n3 + 1 là số chính phương.

2 Cho phương trình x2 − mx + m + 2 = 0. Tìm tất cả các giá trị của m để phương
trình đã cho có các nghiệm nguyên.

Bình Định − Chuyên Toán

Lời giải.
1 Từ giả thiết, ta suy ra 4n4 + 4n3 + 4 cũng là số chính phương. Với mọi số nguyên n, ta

2
4n4 + 4n3 + 4 − 2n2 + n − 1 = 3n2 + 2n + 3 > 0,
2
2n2 + n + 3 − 4n4 + 4n3 + 4 = 13n2 + 6n + 5 > 0.


Các đánh giá trên cho ta


2 2
2n2 + n − 1 < 4n4 + 4n3 + 4 < 2n2 + n + 3 .

Do 4n4 + 4n3 + 4 là số chính phương nên theo bổ đề kẹp, ta xét các trường hợp sau.
2
† Trường hợp 1. Với 4n4 + 4n3 + 4 = 2n2 + n , ta có

4n4 + 4n3 + 4 = 4n4 + 4n3 + n2 ⇔ n2 = 4 ⇔ n = ±2.


2
† Trường hợp 2. Với 4n4 + 4n3 + 4 = 2n2 + n + 1 , ta có

4n4 + 4n3 + 4 = 4n4 + 4n3 + 5n2 + 2n + 1 ⇔ 5n2 + 2n − 3 = 0


⇔ (n + 1)(5n − 3) = 0.

Không tồn tại n tự nhiên trong trường hợp này.


2
† Trường hợp 3. Với 4n4 + 4n3 + 4 = 2n2 + n + 2 , ta có

4n4 + 4n3 + 4 = 4n4 + 4n3 + 9n2 + 4n + 4 ⇔ 9n2 = 0 ⇔ n = 0.

Như vậy, có 2 giá trị của n thỏa đề là n = 0 và n = 2.

2 Giả sử phương trình đã cho có 2 nghiệm nguyên x1 , x2 , áp dụng định lý Viete, ta có


x1 + x2 = m, suy ra m là số nguyên.
Do phương trình đã cho có nghiệm nguyên nên ∆ = m2 − 4(m + 2) = (m − 2)2 − 12
phải là số chính phương. Đặt (m − 2)2 − 12 = a2 với a ∈ N. Phép đặt này cho ta

(m − 2 − a)(m − 2 + a) = 12.

Với chú ý m − 2 − a < m − 2 + a và m − 2 − a ≡ m − 2 + a (mod 2), ta có bảng sau

LATEX by Mathpiad 40
MATHPIAD − TẠP CHÍ TOÁN HỌC

m−2−a −6 2
m−2+a −2 6
m −2 6

† Với m = −2, phương trình x2 + 2x = 0 có 2 nghiệm là 0 và −2, thỏa mãn.


† Với m = 6, phương trình x2 − 6x + 8 = 0 có 2 nghiệm là 2 và 4, thỏa mãn.

Vậy các giá trị của m thỏa mãn yêu cầu đề bài là m = 2, m = 4.

d Câu 36 Tìm tất cả các số nguyên dương x sao cho x2 − x + 13 là số chính


phương.

Bình Định − Chuyên Tin

Lời giải.
Từ giả thiết, ta có thể đặt x2 − x + 13 = t 2 , với t nguyên dương. Phép đặt này cho ta
4x2 − 4x + 52 = 4t 2 ⇒ (2x − 1)2 + 51 = (2t)2 ⇒ (2t − 2x + 1)(2t + 2x − 1) = 51.
Do 0 < 2t − 2x + 1 < 2t + 2x − 1, ta lập được bảng giá trị sau
2t − 2x + 1 1 3
2t + 2x − 1 51 17
x 13 4
Căn cứ vào bảng, ta kết luận x = 4 và x = 13 là tất cả các giá trị nguyên dương thỏa mãn yêu
cầu bài toán. ∇

d Câu 37 Tìm tất cả các cặp số nguyên (x, y) thỏa mãn 7(x + 2y)3 (y − x) = 8y −
5x + 1.

Ninh Bình

Lời giải.
Ta đặt A = x + 2y, B = y − x. Bằng biểu diễn 8y − 5x = A + 6B, phương trình đã cho trở thành
7A3 B = A + 6B + 1 ⇔ 7A3 − 6 B = A + 1

(*)
Ta được 7A3 − 6 | (A + 1) . Ta lần lượt suy ra


7A3 − 6 | 7 (A + 1) A2 − A + 1 ⇒ 7A3 − 6 | 7A3 + 7 ⇒ 7A3 − 6 | 13.


    

Do 13 là số nguyên tố, ta có 7A3 − 6 ∈ {±1; ±13}. Kết hợp với điều kiện A nguyên, ta tìm
ra A = 1 hoặc A = −1. Ta lập bảng giá trị
A B x y
−1 0 6∈ Z 6∈ Z
−1 2 −1 1
Căn cứ vào bảng, ta nhận thấy (x, y) = (−1, 1) là nghiệm nguyên duy nhất của phương trình
đã cho. ∇

LATEX by Mathpiad 41
CHƯƠNG III. LỜI GIẢI THAM KHẢO

d Câu 38 Tìm các số nguyên dương x, y thỏa mãn y4 + 2y3 − 3 = x2 − 3x.

Hải Phòng

Lời giải.
Phương trình đã cho tương đương với

4y4 + 8y3 − 12 = 4x2 − 12x ⇔ 4y4 + 8y3 − 3 = (2x − 3)2 .

Tới đây, ta xét các hiệu


2
4y4 + 8y3 − 3 − 2y2 + 2y − 1 = 4(y − 1) > 0,

2
2y2 + 2y − 4y4 + 8y3 − 3 = −4y3 − 3 < 0.


Các đánh giá theo hiệu bên trên cho ta biết


2 2
2y2 + 2y − 1 6 4y4 + 8y3 − 3 < 2y2 + 2y .
2
Do 4y4 + 8y3 − 3 là số chính phương, ta bắt buộc phải có 4y4 + 8y3 − 3 = 2y2 + 2y − 1 ,
tức y = 1. Ta tìm ra x = 3 từ đây. Kết luận, (x, y) = (1, 3) là cặp số duy nhất thỏa yêu cầu. ∇

d Câu 39

1 Gọi A là số tạo nên khi viết liên tục các số tự nhiên từ 1 đến 2021, nghĩa là

A = 123 . . . 201920202021.

a, Số A gồm bao nhiêu chữ số?


b, Chữ số thứ 2021 (theo chiều từ trái qua phải) của A là chữ số nào?

2 Cho p, x, y là các số tự nhiên thỏa mãn px2 + x = (p + 1)y2 + y. Chứng minh


rằng x − y là một số chính phương.

Bình Thuận − Chuyên Toán

Lời giải.

1 a, Số chữ số của A là tổng số chữ số của các số tự nhiên thuộc tập S = {1; 2; . . . ; 2021}.
Trong tập S, ta có
† Tổng số chữ số của các số có một chữ số là 1 · 9 = 9.
† Tổng số chữ số của các số có hai chữ số là 2 · 90 = 180.
† Tổng số chữ số của các số có ba chữ số là 3 · 900 = 2700.
† Tổng số chữ số của các số có bốn chữ số là 4 · 1022 = 4088.
Như vậy, số chữ số của A là 9 + 180 + 2700 + 4088 = 6977.

LATEX by Mathpiad 42
MATHPIAD − TẠP CHÍ TOÁN HỌC

b, Do 9 + 180 < 2021 < 9 + 180 + 2700, chữ số thứ 2021 của A phải là chữ số trong
một số có ba chữ số tương ứng.
Dựa vào nhận xét
2021 = 9 + 180 + 610 · 3 + 2,

ta suy ra chữ số cần tìm là chữ số thứ hai trong số 610 + 100 − 1 = 709, tức là chữ
số 0.

2 Với các số x, y thỏa mãn giả thiết, ta có

px2 − py2 + x − y = y2 ⇒ p(x − y)(x + y) + (x − y) = y2


⇒ (x − y)(px + py + 1) = y2 .

Ta đặt d = (x − y, px + py + 1). Do y2 = (x − y)(px + py + 1) nên y2 chia hết cho d 2 ,


tức y chia hết cho d. Khi đó
  
d | (x − y)
 d | x
 d | x

d | (px + py + 1) ⇒ d | (px + py + 1) ⇒ d | 1 ⇒ d = 1.
  
d|y d|y d|y
  

Do vậy, ta được x − y là số chính phương. Chứng minh hoàn tất.


Trường hợp đặc biệt của ý thứ hai cũng đã xuất hiện ở trong đề tỉnh Bình Phước
! "Cho hai số tự nhiên a, b thỏa mãn 2a2 + a = 3b2 + b. Chứng minh rằng
2a + 2b + 1 là số chính phương."

3
d Câu 40 Cho a và b là các số nguyên dương thỏa mãn a2 − ab + b2 chia hết
2
cho 25. Chứng minh rằng cả a và b đều chia hết cho 5.

Bình Thuận − Chuyên Tin

Lời giải.
Với các số nguyên dương a, b thỏa mãn giả thiết, ta có
 
3 2
25 | 4 a − ab + b = (2a − b)2 + 5b2 .
2
2

Ta được 2a − b chia hết cho 25, tức là 2a − b chia hết cho 5.


Tiếp tục sử dụng 25 | (2a − b)2 + 5b2 , ta nhân thấy 25 cũng là ước của 5b2 , thế nên b chia
hết cho 5. Kết hợp với 5 | (2a − b), ta có điều phải chứng minh là a và b cùng chia hết cho 5.

LATEX by Mathpiad 43
CHƯƠNG III. LỜI GIẢI THAM KHẢO

d Câu 41

1 Tìm tất cả các cặp số nguyên (x, y) thỏa mãn x2 y − xy + y (x + y) = 3x + 1.




2 Tìm các số nguyên tố p, q đồng thời thỏa mãn hai điều kiện

i, p2 q + p chia hết cho p2 + q.


ii, pq2 + q chia hết cho q2 − p.

Phú Thọ

Lời giải.
1 Phương trình đã cho tương đương với

y x2 − x + 1 (x + y) = 3x − 1.


Ta nhận được x2 − x + 1 | (3x − 1), thế nên




x2 − x + 1 | (3x − 1)(3x − 2) ⇒ x2 − x + 1 | 9x2 − 9x + 9 − 7


  

⇒ x2 − x + 1 | 7.


Lần lượt cho x2 − x + 1 nhận các giá trị là −7, −1, 1, 7 ta kết luận rằng phương trình
đã cho có ba nghiệm nguyên là (−2, 1), (1, −2), (1, 1).
2 Giả sử tồn tại các số nguyên tố p, q thỏa đề. Điều kiện i cho ta

p2 + q | q(p2 + q) + p − q2 ⇒ p2 + q | p − q2 .
   

Đồng thời, điều kiện ii cho ta

q2 − p | p q2 − p + q + p2 ⇒ q2 − p | q + p2
    

Dựa vào hai nhận xét trên, ta suy ra

p2 + q = q2 − p ,

† Với q2 > p, ta có

p2 + q = q2 − p ⇒ (p + q)(p − q + 1) = 0 ⇒ q = p + 1.

Ta được p, q là hai số nguyên tố liên tiếp, và bắt buộc p = 2, q = 3.


† Với p > q2 , ta có

p2 + q = p − q2 ⇒ p2 + q2 = p − q,

điều này là không thể xảy ra do p2 > p và q2 > −q.


Kết luận (p, q) = (2, 3) là cặp số nguyên tố duy nhất thỏa mãn.

LATEX by Mathpiad 44
MATHPIAD − TẠP CHÍ TOÁN HỌC

d Câu 42

1 Chứng minh rằng nếu n là một số nguyên tố thì n2 + 2022 không là lũy thừa của
3.

2 Tìm tất cả các số nguyên tố p sao cho p2 + 3p + 2021 cũng là một số nguyên tố.

Yên Bái

Lời giải.
1 Giả sử n2 + 2022 là lũy thừa của 3. Rõ ràng, do 2022 > 9 nên 9 | n2 + 2022 , và ta có


n2 + 2022 ≡ 0 (mod 9) ⇒ n2 ≡ 3 (mod 9).

Ta suy ra n2 chia hết cho 3, nhưng không chia hết cho 9. Đây là một điều vô lí. Giả sử
sai, và ta có điều phải chứng minh.

2 Với p = 3, ta có p2 + 3p + 2021 = 2039 là một số nguyên tố. Với p > 3, ta có

p2 + 3p + 2021 ≡ 1 + 0 + 2021 ≡ 0 (mod 3).

Nói cách khác, p2 + 3p + 2021 chia hết cho 3 khi p 6= 3, chứng tỏ nó là hợp số. Như
vậy, chỉ có kết quả p = 3 thỏa yêu cầu.

d Câu 43 Chứng minh rằng với mọi số tự nhiên n, ta luôn có 2005n + 60n −
1897n − 168n chia hết cho 2004.

Lai Châu

Lời giải.
Đặt A = 2005n + 60n − 1897n − 168n . Ta sẽ chứng minh A chia hết cho 3, 4 và 167. Thật vậy
† A = (3 · 668 + 1)n + (3 · 20)n − (3 · 632 + 1)n − (3 · 56)n ≡ 1 + 0 − 1 − 0 ≡ 0 (mod 3).

† A = (4 · 501 + 1)n + (4 · 15)n − (4 · 474 + 1)n − (4 · 42)n ≡ 1 + 0 − 1 − 0 ≡ 0 (mod 4).

† A = (12 · 167 + 1)n + 60n − 11 · 167 + 60)n − (167 + 1)n ≡ 1 − 60n + 60n − 1 ≡ 0
(mod 167).
Dựa vào các nhận xét trên, ta suy ra A chia hết cho các số 3, 4, , 167. Các số này đôi một
nguyên tố cùng nhau và có tích bằng 2004, thế nên A chia hết cho 2004. Bài toán được
chứng minh. ∇

d Câu 44
1 Tìm tất cả các cặp số nguyên (x, y) thỏa mãn x2 + 5xy + 6y2 + x + 2y − 2 = 0.

LATEX by Mathpiad 45
CHƯƠNG III. LỜI GIẢI THAM KHẢO

2 Chứng minh rằng với mọi số tự nhiên n, ta có n2 + n + 16 không chia hết cho
49.
2
3 Cho số thực x khác 0 thỏa mãn cả hai số x + và x3 đều là số hữu tỉ. Chứng
x
minh rằng x cũng là số hữu tỉ.

Hà Nội − Chuyên Toán

Lời giải.
1 Phương trình đã cho tương đương với

(x + 2y)(x + 3y) + x + 2y − 2 = 0 ⇔ (x + 2y)(x + 3y + 1) = 2.

Ta lập bảng giá trị

x + 2y −2 −1 1 2
x + 3y + 1 −1 −2 2 1
x −2 3 1 6
y 0 −2 0 −2

Như vậy, có tổng cộng bốn cặp (x, y) thỏa đề, bao gồm (−2, 0), (1, 0), (3, −2) và
(6, −2).
2 Giả sử tồn tại số tự nhiên n thỏa mãn 49 | n2 + n + 16 . Giả sử này cho ta


49 | 4 n2 + n + 16 ⇒ 49 | (2n + 1)2 + 63 ⇒ 7 | (2n + 1)2 + 63


    

⇒ 7 | (2n + 1)2 ⇒ 7 | (2n + 1) ⇒ 49 | (2n + 1)2 .

Tiếp tục kết hợp điều này với 49 | (2n + 1)2 + 63 , ta có 63 chia hết cho 49, một điều
 

vô lí. Như vậy, giả sử phản chứng là sai, và ta có điều phải chứng minh.
2
3 Từ giả thiết thứ nhất, ta có thế đặt x + = a, với a là số hữu tỉ. Phép đặt này cho ta
x

2
 a 2 a2 − 8 a ± a2 − 8
x − ax + 2 = 0 ⇔ x − = ⇔x= .
2 4 2
Kết hợp biến đổi vừa rồi giả thiết x3 hữu tỉ, ta suy ra
√ !3
a ± a 2 −8 1h 3 p i
x3 = a − 6a + a2 − 2

= a2 − 8 ∈ Q.
2 2
√ √
Nếu như a2 − 8 là số vô tỉ, ta bắt buộc phải có a2 − 2 =√0, tức là a = ± 2. Điều này
2
√ với điều kiện phép đặt là a hữu tỉ. Như vậy, a − 8 là số hữu tỉ. Dựa vào
mâu thuẫn
a± a −82
x= , ta thu được điều phải chứng minh.
2

LATEX by Mathpiad 46
MATHPIAD − TẠP CHÍ TOÁN HỌC

d Câu 45

1 Chứng minh rằng với mọi số nguyên n, ta có n2 + 3n + 16 không chia hết cho
25.

2 Tìm tất cả các số nguyên x, y thỏa mãn x2 − xy − 2y2 + x + y − 5 = 0.

Hà Nội − Chuyên Tin

Lời giải.

1 Giả sử tồn tại số tự nhiên n thỏa mãn 25 | n2 + 3n + 16 . Giả sử này cho ta




25 | 4 n2 + 3n + 16 ⇒ 25 | (2n + 3)2 + 55 ⇒ 5 | (2n + 3)2 + 55


    

⇒ 5 | (2n + 3)2 ⇒ 5 | (2n + 3) ⇒ 25 | (2n + 3)2 .

Tiếp tục kết hợp điều này với 25 | (2n + 3)2 + 55 , ta có 55 chia hết cho 25, một điều
 

vô lí. Như vậy, giả sử phản chứng là sai, và ta có điều phải chứng minh.

2 Phương trình đã cho tương đương với

(x + y)(x − 2y) + (x + y) − 5 = 0 ⇔ (x + y)(x − 2y + 1) = 5.

Tới đây, ta lập bảng được giá trị

x+y −5 −1 1 5
x − 2y + 1 −1 −5 5 1
x −4 6∈ Z 2 6∈ Z
y −1 6∈ Z −1 6∈ Z

Như vậy, có tổng cộng hai cặp (x, y) thỏa đề, bao gồm (−4, −1) và (2, −1).

d Câu 46

1 Tìm số nguyên dương n nhỏ nhất, biết rằng khi chia n cho 7, 9, 11, 13 ta nhận
được các số dư tương ứng là 3, 4, 5, 6.

2 Cho tập A = {1, 2, 3, . . . , 2021}. Tìm số nguyên dương k lớn nhất (k > 2) sao
cho ta có thể chọn được k số phân biệt từ tập A mà tổng của hai số phân biệt bất
kì trong k số được chọn không chia hết cho hiệu cúa chúng.

Chuyên Khoa học Tự nhiên − Vòng 1

Lời giải.

LATEX by Mathpiad 47
CHƯƠNG III. LỜI GIẢI THAM KHẢO

1 Giả sử tồn tại số nguyên dương n thỏa mãn yêu cầu. Ta có


  

 n≡3 (mod 7) 
 2n ≡ 6 (mod 7) 
 2n + 1 ≡ 0 (mod 7)
n≡4 (mod 9) 2n ≡ 8 (mod 9) 2n + 1 ≡ 0 (mod 9)
  
⇒ ⇒

 n≡5 (mod 11) 
 2n ≡ 10 (mod 11) 
 2n + 1 ≡ 0 (mod 11)
n≡6 (mod 13) 2n ≡ 12 (mod 13) 2n + 1 ≡ 0 (mod 13)
  

Dựa vào nhận xét trên, ta suy ra 2n + 1 là số nguyên dương nhỏ nhất chia hết cho
7, 9, 11, 13. Bốn số đó đôi một nguyên tố cùng nhau, thế nên

2n + 1 = 7 · 9 · 11 · 13 = 9009.

Từ đây, ta thu được n = 4504.

2 Ta dự đoán rằng k = 674. Ta sẽ chứng minh kết quả trên.

a, Điều kiện cần. Nếu k > 675, ta xét 675 số bất kì trong A. Ta phân hoạch A
thành các tập con như sau

{1; 2; 3}, {4; 5; 6}, ..., {2017; 2018; 2019}, {2020; 2021}.

Theo nguyên lí Dirichlet, tồn tại hai số a, b nằm trong cùng một tập con kể trên.
Tuy nhiên, khi đó a + b lại chia hết cho a − b, mâu thuẫn với giả thiết đã cho.
b, Điều kiện đủ. Nếu k = 674, ta xét các số chia 3 dư 1 trong A, đó là

1, 4, 7, · · · , 2020

Tổng 2 số bất kì trong 674 số đó chia 3 dư 2, trong khi hiệu giữa chúng chia hết
cho 3, thế nên nó không là ước của tổng.

Kết quả của bài toán chính là k = 674.

d Câu 47 Tìm x, y nguyên dương thỏa mãn 3x + 29 = 2y .

Chuyên Khoa học Tự nhiên − Vòng 2

Lời giải.
Giả sử tồn tại các số nguyên dương x, y thỏa mãn yêu cầu bài toán.
Với x = 1, kiểm tra trực tiếp, ta nhận được y = 5. Với x > 2, ta có

3x + 29 ≡ 2 (mod 9) ⇒ 2y ≡ 2 (mod 9).

Xét các số dư khi chia cho 6 của y, ta nhận được y ≡ 1 (mod 6). Bằng cách đặt y = 6z + 1
(trong đó z là số tự nhiên), ta chỉ ra

2y = 26z+1 = 2 · 64z ≡ 2 (mod 7).

LATEX by Mathpiad 48
MATHPIAD − TẠP CHÍ TOÁN HỌC

Ta tiếp tục suy ra


3x + 29 ≡ 2 (mod 7) ⇒ 3x ≡ 1 (mod 7).
Xét các số dư khi chia cho 6 của x, ta nhận được x ≡ 0 (mod 6). Tiếp tục đặt x = 6t (trong
đó t là số tự nhiên), ta chỉ ra

3x + 29 = 36y + 29 = 729t + 29 ≡ 6 (mod 8).

Ta được 2y ≡ 6 (mod 8) từ đây, là điều không thể xảy ra.


Kết luận, (x, y) = (1, 5) là cặp số nguyên dương duy nhất thỏa yêu cầu. ∇

Cơ sở của việc chọn modulo 6 để xét cho x và y khi ta tìm ra 2y ≡ 2 (mod 9) và 3x ≡ 1


(mod 7) là nhờ vào hai định lí sau:

1 Cho số nguyên dương a và số nguyên tố p, khi đó nếu (a, p) = 1 thì

a p−1 ≡ 1
! (mod p).

2 Cho các số nguyên dương a, b nguyên tố cùng nhau. Khi đó, tồn tại số nguyên
dương n sao cho an ≡ 1 (mod b).

Số nguyên dương n nhỏ nhất ở trong định lí thứ hai chính là modulo ta cần xét.

d Câu 48 Tìm tất cả các số nguyên a, b thỏa mãn a4 − 2a3 + 10a2 − 18a − 16 =
4b2 + 20b.

Cao Bằng

Lời giải.
Phương trình đã cho tương đương với

a4 − 2a3 + 10a2 − 18a + 9 = 4b2 + 20b + 25 ⇔ a4 − 2a3 + a2 + 9a2 − 18a + 9 = (2b + 5)2
⇔ a2 (a − 1)2 + 9(a − 1)2 = (2b + 5)2
⇔ a2 + 9 (a − 1)2 = (2b + 5)2


5
Nếu như a = 1, ta suy ra b = − , mâu thuẫn giả thiết a, b là số nguyên. Lập luận này chứng
2
tỏ (a − 1)2 > 0, và như vậy, a2 + 9 là số chính phương.
Ta đặt a2 + 9 = c2 , với c là số nguyên dương. Phép đặt này cho ta

(c − a)(c + a) = 9

Do c > |a|, ta suy ra c − a > 0 và c + a > 0. Dựa vào đây, ta lập được bảng giá trị sau
c+a 9 3 1
c−a 1 3 9
a 4 0 −4

LATEX by Mathpiad 49
CHƯƠNG III. LỜI GIẢI THAM KHẢO

Lần lượt thay a = −4, 0, 4 trở lại phương trình ban đầu, ta tìm được tất cả 6 cặp (a, b) thỏa
yêu cầu, đó là
(−4, −15), (−4, 10), (0, −4), (0, −1), (4, −1), (4, 5).

d Câu 49

1 Tìm tất cả các cặp số nguyên dương (x, y) thỏa mãn x2 y2 (y − x) = 5xy2 − 27.

2 Cho p1 , p2 , . . . , p12 là các số nguyên tố lớn hơn 3. Chứng minh rằng p21 + p22 +
. . . + p212 chia hết cho 12.

Nam Định

Lời giải.

1 Giả sử tồn tại cặp số (x, y) thỏa đề. Theo đó:

xy 5y − xy2 + x2 y = 27.

(*)

Biến đổi trên chứng tỏ xy là ước nguyên dương của 27. Ta lần lượt xét các trường hợp
sau.

† Trường hợp 1. xy = 1.
Trong trường hợp này, rõ ràng x = y = 1. Đối chiếu với (*), ta thấy không thỏa.
† Trường hợp 2. xy = 3. Thay trở lại (*), ta được

5y − 3y + 3x = 9 ⇔ 3x + 2y = 9.

Lần lượt kiểm tra với (x, y) = (1, 3), (3, 1) ta thấy chỉ có (x, y) = (3, 1) thỏa mãn.
Trường hợp 3. xy = 9. Thay trở lại (*), ta được

5y − 9y + 9x = 3 ⇔ 9x − 4y = 3.

Lần lượt kiểm tra với (x, y) = (1, 9), (3, 3), (9, 1), ta thấy chúng đều không thỏa
mãn.
† Trường hợp 4. xy = 27. Thay trở lại (*), ta được

5y − 27y + 27x = 1 ⇔ 27x − 22y = 3.

Lần lượt kiểm tra với (x, y) = (1, 27), (3, 9), (9, 3), (27, 1), ta thấy chúng đều
không thỏa mãn.

Kết luận, (x, y) = (3, 1) là cặp số duy nhất thỏa đề.

LATEX by Mathpiad 50
MATHPIAD − TẠP CHÍ TOÁN HỌC

2 Tất cả các số p21 , p22 , . . . p212 đều không chia hết cho 3, thế nên chúng chia cho 3 được
dư là 1. Lập luận này cho ta

p21 + p22 + . . . + p212 ≡ 12 (mod 3).

Ta được p21 + p22 + . . . + p212 chia hết cho 3. Đây chính là điều phải chứng minh.

d Câu 50

1 Tìm tất cả các số tự nhiên có 4 chữ số abcd thỏa mãn đồng thời các điều kiện:
abcd chia hết cho 3 và abc − bda = 650.

2 Tìm các số nguyên x, y, z thỏa mãn đồng thời các điều kiện

x2 + 4y2 + 2z2 + 2(xz + 2x + 2z) = 396, x2 + y2 = 3z.

Hải Dương

Lời giải.

1 Giả sử tồn tại số tự nhiên abcd thỏa đề. Rõ ràng, c = a. Đồng thời, từ giả thiết, ta nhận
thấy a > 7. Ta xét các trường hợp sau.

† Trường hợp 1. a = c = 7. Trong trường hợp này, ta có

650 = abc − bda = 7b7 − bd7 = 700 − 90b − 10d 6 700 − 90 · 1 = 630,

một điều mâu thuẫn.


† Trường hợp 2. a = c = 8. Trong trường hợp này, ta có

650 = abc − bda = 8b8 − bd8 = 800 − 90b − 10d.

Từ đây, ta suy ra 9b + d = 15, tức b = 1 và c = 6. Kiểm tra trực tiếp, ta thấy số


abcd = 8186 không chia hết cho 3, mâu thuẫn.
† Trường hợp 3. a = c = 9. Trong trường hợp này, ta có

650 = abc − bda = 9b9 − bd9 = 900 − 90b − 10d.

Từ đây, ta suy ra 9b+d = 25. Do b, d là các chữ số, chỉ có trường hợp b = 2, d = 7
xảy ra. Kiểm tra trực tiếp, ta thấy số abcd = 9297 chia hết cho 3, và đây là số tự
nhiên duy nhất thỏa mãn yêu cầu.

2 Từ phương trình thứ hai, ta chỉ ra cả x, y, z đều chia hết cho 3, đồng thời z dương. Ngoài
ra, phương trình thứ nhất trong hệ tương đương với

(x + z + 2)2 + (2y)2 + z2 = 400.

LATEX by Mathpiad 51
CHƯƠNG III. LỜI GIẢI THAM KHẢO

Có đúng hai cách để viết 400 thành tổng ba số chính phương, đó là

400 = 02 + 02 + 202 = 02 + 122 + 162 .

Dựa vào các nhận xét kể trên, ta xét các trường hợp sau.

† Trường hợp 1. Nếu z = 0, phương trình thứ hai trở thành

x2 + y2 = 0.

Ta có x = y = 0. Thế ngược lại phương trình thứ nhất, ta thấy không thỏa.
† Trường hợp 2. Nếu z = 12, một trong hai số x + y + 2, 2y phải bằng 0, vậy nên
"
 x+y+2 = 0
 "
x = 6, y = 0
2y = 0 ⇒ .

 2 x = −6, y = 0
2
x + y = 36

Thế ngược lại phương trình thứ nhất, ta thấy không thỏa.

Như vậy, không tồn tại các số nguyên x, y, z thỏa yêu cầu.

d Câu 51 Cho số nguyên dương n > 2. Chứng minh rằng

1 A = n3 − 3n2 + 2n chia hết cho 6.

2 B = nA+1 − 1 chia hết cho 7.

Tuyên Quang

Lời giải.
1 Ta viết lại biểu thức A như sau:

A = n n2 − 3n + 2 = n(n − 1)(n − 2).




Cách viết trên kết hợp giả thiết n > 2 cho ta A là tích ba số nguyên dương liên tiếp, và
vì thế, A chia hết cho 6.

2 Theo như kết quả trên, ta có thể đặt A = 6m, với m là số nguyên dương. Ta xét các
trường hợp sau:

† Trường hợp 1. Nếu n chia hết cho 7, hiển nhiên B cũng chia hết cho 7.
† Trường hợp 2. Nếu n không chia hết cho 7, ta suy ra n3 ≡ 1, −1 (mod 7), kéo
theo n6 ≡ 1 (mod 7). Kết quả này cho ta

n6 ≡ 1 (mod 7) ⇒ n6m ≡ 1 (mod 7)


⇒ n6m+1 ≡ n (mod 7) ⇒ 7 | nA+1 − n .


LATEX by Mathpiad 52
MATHPIAD − TẠP CHÍ TOÁN HỌC

Như vậy, bài toán được chứng minh trong mọi trường hợp.


Nhận xét. Cách xét modulo 7 của lũy thừa thông qua xét modulo 6 của số mũ là ý tưởng đã
xuất hiện trong đề thi của chuyên Khoa học Tự nhiên vào cùng năm: "Tìm x, y nguyên dương
thỏa mãn 3x + 29 = 2y ."

d Câu 52 Cho a, b là hai số hữu tỉ.


√ √
Chứng minh rằng nếu a 2 + b 3 là số hữu tỉ thì a = b = 0.

Chuyên Sư phạm − Vòng 1

Lời giải. √ √
Từ giả thiết, ta có thể đặt a 2 + b 3 = c, với c là số hữu tỉ. Bình phương hai vế, ta được

2a2 + 3b2 + 2ab 6 = c2 ,

hay là √
2a2 + 3b2 − c2 = 2ab 6.
Trong trường hợp ab 6= 0, ta suy ra

√ 2a2 + 3b2 − c2
6= .
2ab

Lập luận trên chứng tỏ 6 là số hữu tỉ, mâu thuẫn. Như vậy, ta thu được ab = 0.

† Nếu a = 0, ta suy ra b 3 là số hữu tỉ, thế nên b = 0.

† Nếu b = 0, ta suy ra a 2 là số hữu tỉ, thế nên a = 0.

Bài toán được chứng minh. ∇

d Câu 53

1 Tìm tất cả các số nguyên dương N sao cho N có thể biểu diễn duy nhất một cách
x2 + y
biểu diễn ở dạng với x, y là hai số nguyên dương.
xy + 1
2 Cho a, b, c là ba số nguyên dương sao cho mỗi số trong ba số đó đều biểu
diễn dạng lũy thừa của 2 với số mũ tự nhiên. Biết rằng phương trình bậc hai
ax2 − bx + c = 0 có hai nghiệm đều là số nguyên. Chứng minh rằng hai nghiệm
của phương trình này bằng nhau.

Chuyên Sư phạm − Vòng 2

Lời giải.

LATEX by Mathpiad 53
CHƯƠNG III. LỜI GIẢI THAM KHẢO

1 Giả sử tồn tại số nguyên dương N thỏa mãn, tức là tồn tại duy nhất một cặp (x, y) sao
cho
x2 − (Ny) x + y − N = 0. (*)
Tính tồn tại của x chứng tỏ N 2 y2 − 4y + 4N là số chính phương. Ta nhận thấy rằng

(Ny + 2)2 − N 2 y2 − 4y + 4N = 4Ny + 4y − 4N + 4 = 4(N + 1)(y − 1) + 8 > 0,




N 2 y2 − 4y + 4N − (Ny − 2)2 = 4Ny − 4y + 4N − 4 = 4(N − 1)(y + 1) > 0,




Các đánh giá trên cho ta

(Ny − 2)2 6 N 2 y2 − 4y + 4N < (Ny + 2)2

Đến đây, ta xét các trường hợp sau

† Trường hợp 1. N 2 y2 − 4y + 4N = (Ny − 2)2 .


Trường hợp này cho ta

N 2 y2 − 4y + 4N = N 2 y2 − 4Ny + 4 ⇔ 4Ny + 4N − 4y − 4 = 0
⇔ 4(N − 1)(y + 1) = 0.

Ta tìm ra N = 1. Thay trở lại (*), ta có

x2 − xy + y − 1 = 0 ⇔ (x − 1)(x − y + 1) = 0.

Bằng phân tích như vậy, ta chỉ ra số 1 có vô hạn dạng biểu diễn thỏa mãn, đó là

x2 + (x + 1)
1= ,
x(x + 1) + 1

với x là một số nguyên dương bất kì.


† Trường hợp 2. N 2 y2 − 4y + 4N = (Ny − 1)2 .
Trường hợp này cho ta

N 2 y2 − 4y + 4N = N 2 y2 − 2Ny + 1 ⇔ 2Ny − 2y + 4N = 1.

So sánh tính chẵn lẻ của hai vế, ta thấy mâu thuẫn.


† Trường hợp 3. N 2 y2 − 4y + 4N = (Ny)2 .
Trường hợp này cho ta y = N. Thay trở lại (*), ta tìm được x = N 2 .
† Trường hợp 4. N 2 y2 − 4y + 4N = (Ny + 1)2 .
Trường hợp này cho ta

N 2 y2 − 4y + 4N = N 2 y2 + 2Ny + 1 ⇔ −2Ny − 2y + 4N = 1.

So sánh tính chẵn lẻ của hai vế, ta thấy mâu thuẫn.

LATEX by Mathpiad 54
MATHPIAD − TẠP CHÍ TOÁN HỌC

Kết luận, tất cả các số nguyên dương N > 1 đều thỏa mãn yêu cầu bài toán, và cách
biểu diễn của mỗi số N này là
2
N2 + N
N= 2 .
N ·N +1

2 Từ giả thiết, ta có thể đặt a = 2m , b = 2n , c = 2 p , với m, n, p nguyên dương. Phương


trình đã cho trở thành
2m x2 − 2n x + 2 p = 0.
Phương trình này có hai nghiệm nguyên khi và chỉ khi

∆ = 4n − 2m+p = 22n − 2m+p

là số chính phương, và hiển nhiên 2n > m + p.


Ta đặt 22n − 2m+p = q2 , với q nguyên dương. Theo đó,

(2n − q) (2n + q) = 2m+p .

Rõ ràng, 2n −q và 2n +q đều là các lũy thừa cơ số 2. Ta tiếp tục đặt 2n −q = 2k , 2n +q =


2l , với k, l là các số tự nhiên và k 6 l. Lấy tổng theo vế, ta được
 
2n+1 = 2k + 2l = 2k 2l−k + 1 .

So sánh số mũ của 2 ở hai vế, ta tìm được k = l, kéo theo q = 0 và ∆ = 0. Nói cách
khác, phương trình bậc hai đã cho có nghiệm kép. Bài toán được chứng minh.

d Câu 54 Tìm tất cả các cặp số nguyên (x, y) thỏa mãn đẳng thức
2
x2 − y2 = 1 + 20y.

Đà Nẵng

Lời giải.
Giả sử tồn tại các cặp số nguyên (x, y) thỏa mãn. Rõ ràng y > 0, đồng thời khi thay x thành
−x, đẳng thức vẫn đúng, thế nên không mất tổng quát, giả sử x > 0.
Ta nhận thấy x = y không thỏa mãn. Trong trường hợp x 6= y, ta suy ra (x − y)2 > 1, vì thế
2
1 + 20y = x2 − y2 = (x − y)2 (x + y)2 > (x + y)2 > y2 .

Dựa vào đánh giá trên, ta có


√ √
y2 6 20y + 1 ⇒ (y − 10)2 6 101 ⇒ 10 − 101 6 y 6 10 + 101.

Do y là số tự nhiên, ta chọn y = 0, 1, 2, . . . , 20. Trong các số này, 20y + 1 chỉ nhận giá trị là
số chính phương với y = 0, y = 4, y = 6 và y = 18.

LATEX by Mathpiad 55
CHƯƠNG III. LỜI GIẢI THAM KHẢO

1 Với y = 0, ta có x4 = 1. Do x > 0, ta chọn x = 1.


" 2
2 x =7
2 Với y = 4, ta có x2 − 16 = 81 ⇔ x2 − 16 = ±9 ⇔ ⇔ x = ±5.
x2 = 25
Do x > 0, ta chọn x = 5.
" 2
2 x = 25
3 Với y = 6, ta có x2 − 36 = 121 ⇔ x2 − 36 = ±11 ⇔ 2

⇔ x = ±5.
x = 47
Do x > 0, ta chọn x = 5.
" 2
2 x = 305
4 Với y = 18, ta có x2 − 324 = 381 ⇔ x2 − 324 = ±19 ⇔ 2

, mâu thuẫn.
x = 343

Kết quả, có tất cả 6 cặp (x, y) thỏa đề, bao gồm

(1, 0), (−1, 0), (5, 4), (−5, 4), (5, 6), (−5, 6).

d Câu 55

1 Cho các số nguyên x, y, z thỏa mãn x2 + y2 + z2 = 2xyz.


Chứng minh rằng xyz chia hết cho 24.

2 Tìm các bộ ba số nguyên dương (a, b, c) sao cho (a + b + c)2 − 2a + 2b là số


chính phương.

Vĩnh Phúc

Lời giải.

1 Ta thực hiện các bước làm sau đối với bài toán này.

a, Ta chứng minh xyz chia hết cho 8.


Từ giả thiết, ta suy ra x2 + y2 + z2 là số chẵn, thế nên trong x, y, z có 1 hoặc 3 số
chẵn. Không mất tính tổng quát, ta xét các trường hợp sau.
† Trường hợp 1. x là số chẵn, y, z là số lẻ.
Trong trường hợp này, ta có

xyz ≡ 0 (mod 4). (1)

Ta cũng biết rằng một số chẵn dạng a2 khi chia cho 8 được dư là 0 hoặc 4,
trong khi số dư của một số lẻ dạng a2 là 1. Lập luận này chỉ ra cho ta

x2 + y2 + z2 ≡ 2, 6 (mod 8). (2)

Đối chiếu (1) và (2), ta nhận thấy điều mâu thuẫn. Trường hợp này không
thỏa.

LATEX by Mathpiad 56
MATHPIAD − TẠP CHÍ TOÁN HỌC

† Trường hợp 2. x, y, z đều là số chẵn. Trong trường hợp này, hiển nhiên xyz
chia hết cho 8.
b, Ta chứng minh xyz chia hết cho 3.
Giả sử phản chứng rằng x, y, z không chia hết cho 3, lúc này

x2 + y2 + z2 ≡ 1 + 1 + 1 ≡ 0 (mod 3).

Đánh giá trên chứng tỏ xyz chia hết cho 3, tức một trong ba số x, y, z chia hết cho
3, mâu thuẫn với giả sử. Mâu thuẫn này chứng tỏ một trong ba số x, y, z chia hết
cho 3.

Như vậy, xyz chia hết cho [3, 8] = 24. Bài toán được chứng minh.

2 Ta xét các hiệu sau

(a + b + c + 1)2 − (a + b + c)2 − 2a + 2b = 2a + 2b + 2c + 1 + 2a − 2b
 

= 4a + 2c + 1 > 0,
2 2
 
(a + b + c) − 2a + 2b − (a + b + c − 1) = 2a + 2b + 2c − 1 − 2a + 2b
= 4b + 2c − 1 > 0.

Các đánh giá trên cho ta

(a + b + c − 1)2 < (a + b + c)2 − 2a + 2b < (a + b + c + 1)2 .

Như vậy, (a + b + c)2 − 2a + 2b là số chính phương khi và chỉ khi

(a + b + c)2 − 2a + 2b = (a + b + c)2 ,

tức là a = b. Nói cách khác, tất cả các bộ số (a, b, c) thỏa đề là (k, k,t), với k,t là các
số nguyên dương.


Đây là dạng toán sử dụng bổ đề kẹp trong trường hợp nhiều biến. Dạng toán này cũng
xuất hiện trong bài đề thi chuyên Sư phạm vào cùng năm, và đề thi vòng 2 của chuyên
Khoa học Tự nhiên vào năm 2014 - 2015:

1 Cho x, y là những số nguyên lớn hơn 1 sao cho 4x2 y2 − 7x + 7y là số


chính phương. Chứng minh rằng x = y.
! Chuyên Khoa học Tự nhiên 2014 - 2015 vòng 2
2 Tìm tất cả các số nguyên dương N sao cho N có thể biểu diễn duy
x2 + y
nhất một cách biểu diễn ở dạng với x, y là hai số nguyên
xy + 1
dương.
Chuyên Sư phạm 2021 - 2022 vòng 2

LATEX by Mathpiad 57
CHƯƠNG III. LỜI GIẢI THAM KHẢO

d Câu 56

1 Tìm tất cả các số nguyên (x, y) thỏa mãn x3 y − x3 − 1 = 2x2 + 2x + y.

2 Có bao nhiêu số tự nhiên gồm 6 chữ số có dạng 357abc, biết rằng số này chia
hết cho 3, 5 và 7.

Kon Tum

Lời giải.

1 Phương trình đã cho tương đương với

y(x − 1)(x2 + x + 1) = (x + 1)(x2 + x + 1)

1 2 3
 
Do x2 + x + 1 = x+ + > 0, ∀x ∈ Z nên phương trình trên tiếp tục tương đương
2 4

y(x − 1) = x + 1 ⇔ y(x − 1) = x − 1 + 2 ⇔ (y − 1)(x − 1) = 2.

Tới đây, ta lập được bảng giá trị

x−1 1 2 −1 −2
y−1 2 1 −2 −1
x 2 3 0 −1
y 3 2 −1 0

Như vậy, phương trình đã cho có 4 nghiệm nguyên là (2, 3), (0, −1), (3, 2), (−1, 0).

2 Từ giả thiết, ta có số

357abc = 357000 + abc = 3 · 5 · 7 · 3400 + abc

chia hết cho cả 3, 5 và 7, vậy nên số abc cũng có tính chất tương tự. Trong các cụm
abc từ 000 đến 999, có tất cả  
999
+ 1 = 10
105
số chia hết cho cả 3, 5, 7, do đó có đúng 6 số thỏa mãn yêu cầu đề bài.

d Câu 57 Tìm tất cả các số nguyên dương n để n5 + n4 + 1 là số nguyên tố.

Sóc Trăng

Lời giải.

LATEX by Mathpiad 58
MATHPIAD − TẠP CHÍ TOÁN HỌC

Ta nhận thấy rằng

n5 + n4 + 1 = n5 − n2 + n4 − n + n2 + n + 1
= n2 (n − 1) n2 + n + 1 + n (n − 1) n2 + n + 1
 

= n2 + n + 1 n3 − n2 + n2 − n + 1
 

= n2 + n + 1 n3 − n + 1
 

Do n5 + n4 + 1 nên hoặc n2 + n + 1 = 1, hoặc n3 − n + 1 = 1. Kiểm tra trực tiếp từng trường


hợp, ta nhận thấy n = 1 là số nguyên dương duy nhất thỏa yêu cầu. ∇

d Câu 58

1 Chứng minh rằng n5 − n chia hết cho 240 với n là số tự nhiên lẻ bất kì.

2 Tìm tất cả các nghiệm nguyên của phương trình x2 − y2 x + y4 + 6y2 = 0.




Bắc Giang

Lời giải.
1 Ta chia bài toán kể trên thành 3 bước làm

† Ta chứng minh n5 − n chia hết cho 16.


Ta đã biết rằng n4 ≡ 0, 1 (mod 16). Tuy nhiên, do n là số lẻ nên n4 ≡ 1 (mod 16),
và vì thế
16 | n n4 − 1 = n5 − n.


† Ta chứng minh n5 − n chia hết cho 3.


Ta nhận xét

n5 − n = n n4 − 1 = n n2 − 1 n2 + 1 = n(n − 1)(n + 1) n2 + 1 .
   

Tích n(n − 1)(n + 1) là tích ba số nguyên liên tiếp, và tích này chia hết cho 3.
† Ta chứng minh n5 − n chia hết cho 5.
Ta đã biết n2 ≡ 0, 1, −1 (mod 5). Dựa trên biến đổi

n5 − n = n n2 − 1 n2 + 1 ,
 

mệnh đề trong ý này được chứng minh.

Thông qua các kết quả trên, ta chỉ ra số n5 − n chia hết cho [16, 3, 5] = 240. Toàn bộ
bài toán được chứng minh.

2 Phương trình đã cho tương đương với


2
x2 − xy2 = y6 + 6y4 ⇔ 4x2 − 4xy2 + y4 = 4y6 + 25y4 ⇔ 2x − y2 = y4 4y2 + 25 .


Tới đây, ta xét các trường hợp sau.

LATEX by Mathpiad 59
CHƯƠNG III. LỜI GIẢI THAM KHẢO

† Trường hợp 1. Nếu 2x = y2 , ta tìm được x = y = 0.


† Trường hợp 2. Nếu 2x 6= y2 , ta có 4y2 + 25 là số chính phương. Đặt 4y2 + 25 =
z2 , trong đó z là số nguyên dương. Phép đặt này cho ta

z2 − 4y2 = 25 ⇒ (z − 2y)(z + 2y) = 25.

Do z − 2y 6 z + 2y nên ta lập được bảng giá trị


z − 2y z + 2y y x
−25 −1 −6 252 hoặc −216
−5 −5 0 0
5 5 0 0
1 25 6 252 hoặc −216

Kiểm tra trực tiếp từng trường hợp, ta nhận thấy rằng phương trình đã cho có tất cả 5
nghiệm nguyên, đó là

(−216, −6), (−216, 6), (0, 0), (252, −6), (252, 6).

d Câu 59

1 Tìm tất cả các cặp số nguyên (x, y) thỏa mãn 2x2 − xy + 9x − 3y + 4 = 0.

2 Cho f (x) là một đa thức với hệ số nguyên. Biết f (1) f (2) = 2021, chứng minh
phương trình f (x) = 0 không có nghiệm nguyên.

3 Cho tập hợp A có các tính chất sau:

i, Tập hợp A chứa toàn bộ các số nguyên.


√ √
ii, 2 + 3 ∈ A.
iii, Với mọi x, y ∈ A thì x + y ∈ A và xy ∈ A .
1
Chứng minh rằng √ √ ∈A.
2+ 3

Lạng Sơn

Lời giải.

1 Phương trình đã cho tương đương với

2x2 + 9x + 4 = (x + 3)y ⇔ (x + 3)(2x + 3) − 5 = (x + 3)y


⇔ (x + 3)(2x − y + 3) = 5.

Căn cứ vào biến đổi kể trên, ta lập được bảng giá trị

LATEX by Mathpiad 60
MATHPIAD − TẠP CHÍ TOÁN HỌC

x+3 2x − y + 3 x y
−5 −1 −8 −12
−1 −5 −4 0
1 5 −2 −6
5 1 2 6

Kết luận, có tất cả 4 cặp (x, y) thỏa yêu cầu, bao gồm

(−8, −12), (−4, 0), (−2, −6), (2, 6).

2 Giả sử đa thức f (x) có nghiệm nguyên a. Theo như định lí Bezout, đa thức f (x) chia
hết cho đa thức x − a, thế nên ta viết được f (x) = (x − a)g(x), trong đó g(x) là đa thức
có các hệ số nguyên. Kết hợp với giả thiết, ta có

f (1) = (1 − a)g(1)
f (2) = (2 − a)g(2)

Lấy tích theo vế, ta được

2021 = (1 − a)(2 − a)g(1)g(2)

Tuy nhiên, đẳng thức trên không xảy ra vì vế trái là số lẻ, còn vế phải chia hết cho số
chẵn (a − 1)(a − 2). Giả sử phản chứng là sai. Bài toán được chứng minh.

3 Ta sẽ chứng minh kết quả trên bằng biến đổi hệ quả liên tục. Ta có
√ √ √ √ 2 √ √
2+ 3 ∈ A ⇒ 2 + 3 ∈ A ⇒ 5 + 2 6 ∈ A ⇒ −5 + 2 6 ∈ A
√ √ √ 2 √ √  √ √ 2
⇒ 5−2 6 ∈ A ⇒ 3− 2 ∈ A ⇒ 2+ 3 3− 2 ∈ A
√ √ 1
⇒ 3− 2 ∈ A ⇒ √ √ ∈A.
2+ 3
Bài toán đã cho được chứng minh.

LATEX by Mathpiad 61
ĐOÀN QUANG ĐĂNG

PHƯƠNG TRÌNH

HÀM
QUA CÁC CUỘC THI
TRÊN THẾ GIỚI

2022

f(x+y) = f(x) + f(y)

Happy New Year


2023

MATHPIAD
ĐOÀN QUANG ĐĂNG
Email: dangdoanquang8@gmail.com

PHƯƠNG TRÌNH HÀM QUA CÁC


CUỘC THI
TRÊN THẾ GIỚI NĂM 2022

Mathpiad
Thành phố Hồ Chí Minh, ngày 16 tháng 1 năm 2023
Đoàn Quang Đăng Mathpiad

§1 Đề bài
§1.1 Phương trình hàm trên tập số thực
Bài toán 1 (Việt Nam TST 2022). Cho số thực α và xét hàm số φ(x) = x2 eαx với mọi x ∈ R.
Tìm tất cả hàm số f : R → R thỏa mãn

f (φ(x) + f (y)) = y + φ(f (x))

với mọi số thực x, y.


Bài toán 2 (District Olympiad 2022). Tìm tất cả hàm số f : R → R thỏa mãn

f (f (y − x) − xf (y)) + f (x) = y (1 − f (x))

với mọi số thực x, y.


Bài toán 3 (Iran MO Second Round 2022). Tìm tất cả hàm số f : R → R thỏa mãn

f (xf (y) + f (x) + y) = xy + f (x) + f (y)

với mọi số thực x, y.


Bài toán 4 (Kosovo MO 2022). Tìm tất cả hàm số f : R → R thỏa mãn

f (f (x − y) − yf (x)) = xf (y)

với mọi số thực x, y.


Bài toán 5 (Kosovo TST 2022). Tìm tất cả hàm số f : R → R thỏa mãn

f x2 + 2f (xy) = xf (x + y) + yf (x)


với mọi số thực x, y.


Bài toán 6 (IMO 2022 Malaysian Training Camp). Tìm tất cả hàm số f : R → R thỏa mãn

f (xf (x) + 2y) = f (x)2 + x + 2f (y)

với mọi số thực x, y.


Bài toán 7 (IMO 2022 Malaysian Training Camp). Tìm tất cả hàm số f : R → R thỏa mãn

f x2 + f (x + y) = y + xf (x + 1)


với mọi số thực x, y.


Bài toán 8 (MEMO 2022). Tìm tất cả hàm số f : R → R thỏa mãn

f (x + f (x + y)) = x + f (f (x) + y))

với mọi số thực x, y.


Bài toán 9 (DAMO). Tìm tất cả hàm số f : R → R thỏa mãn

f (2xy + f (x)) = xf (y) + f (yf (x) + x)

với mọi số thực x, y.

2
Đoàn Quang Đăng Mathpiad

Bài toán 10 (Nordic 2022). Tìm tất cả hàm số f : R → R thỏa mãn đồng thời

i) f (f (x)f (1 − x)) = f (x) ∀x ∈ R;


ii) f (f (x)) = 1 − f (x) ∀x ∈ R.
Bài toán 11 (Romania EGMO TST 2022). Tìm tất cả hàm số f : R → R thỏa mãn

f (f (x) + y) = f (x2 − y) + 4yf (x)

với mọi số thực x, y.


Bài toán 12 (HMMT 2022). Tìm tất cả hàm số f : R\{0} → R thỏa mãn

f (x)2 − f (y)f (z) = x(x + y + z) (f (x) + f (y) + f (z))

với mọi số thực x, y, z thỏa mãn xyz = 1.


Bài toán 13 (China TST 2022). Tìm tất cả hàm số f : R → R thỏa mãn

{f (xf (y) + 1) , f (yf (x) − 1)} = {xf (f (y)) + 1, yf (f (x)) − 1}



a = c, b = d
với mọi x, y ∈ R, trong đó {a, b} = {c, d} ⇐⇒ .
a = d, b = c

§1.2 Phương trình hàm trên tập số thực dương


Bài toán 14 (VMO 2022). Tìm tất cả hàm số f : R+ → R+ thỏa mãn
 
f (x)
f + y = 1 + f (y)
x

với mọi số thực dương x, y.


Bài toán 15 (Balkan MO 2022). Tìm tất cả hàm số f : R+ → R+ thỏa mãn

f yf (x)3 + x = x3 f (y) + f (x)




với mọi số thực dương x, y.


Bài toán 16 (PAMO 2022). Tìm tất cả hàm số f, g : R+ → R+ thỏa mãn

i) (f (x) + y − 1) (g(y) + x − 1) = (x + y)2 ∀x, y > 0;


ii) (−f (x) + y) (g(y) + x) = (x + y + 1)(y − x − 1) ∀x, y > 0.
Bài toán 17 ( Iran MO Third Round 2022). Tìm tất cả hàm số f : R+ → R+ thỏa mãn

f (x + f (y) + f (f (z))) = z + f (y + f (x))

với mọi số thực dương x, y, z.


Bài toán 18 (Czech-Polish-Slovak Match 2022). Tìm tất cả hàm số f : R+ → R+ thỏa mãn
 
y+1 1
f f (x) + = +x+1
f (y) f (y)

với mọi số thực dương x, y.

3
Đoàn Quang Đăng Mathpiad

Bài toán 19 (Switzerland TST 2022). Tìm tất cả hàm số f : R+ → R+ thỏa mãn
x + f (yf (x) + 1) = xf (x + y) + yf (yf (x))
với mọi số thực dương x, y.
Bài toán 20 (Taiwan TST 2022, Round 2). Tìm tất cả hàm số f : R+ → R+ thỏa mãn
f x + y 2 f (y) = f (1 + yf (x))f (x)


với mọi số thực dương x, y.


Bài toán 21 (USAMO 2022). Tìm tất cả hàm số f : R+ → R+ thỏa mãn
f (x) = f (f (f (x)) + y) + f (xf (y)) f (x + y)
với mọi số thực dương x, y.

§1.3 Phương trình hàm trên tập rời rạc


Bài toán 22 (British MO 2022). Tìm tất cả hàm số f : Z+ → Z+ thỏa mãn
2bf f a2 + a) = f (a + 1)f (2ab)
 

với mọi số nguyên dương a, b.


Bài toán 23 (District Olympiad 2022). Tìm tất cả hàm số f : Z+ → Z+ thỏa mãn
x3 + 3x2 f (y) y 3 + 3y 2 f (x) (x + y)3
+ =
x + f (y) y + f (x) f (x + y)
với mọi số nguyên dương x, y.
Bài toán 24 (Francophone MO 2022). Tìm tất cả hàm số f : Z → Z thỏa mãn
f (m + n) + f (m)f (n) = n2 (f (m) + 1) + m2 (f (n) + 1) + mn(2 − mn)
với mọi số nguyên m, n.
Bài toán 25 (Japan MO Final 2022). Tìm tất cả hàm số f : Z+ → Z+ thỏa mãn
f f (n) (m) + mn = f (m)f (n)
với mọi số nguyên dương m, n, trong đó f k (n) = f (f (. . . f ( n) . . .)).
| {z }
k

Bài toán 26 (Taiwan TST 2022, Round 1). Tìm tất cả hàm số f : Z → Z thỏa mãn
   
f (x) + f (y) f (x) + f (y)
f + f (x) = f (f (y)) +
2 2
với mọi số nguyên x, y, trong đó ký hiệu ⌊x⌋ chỉ số nguyên lớn nhất không vượt quá x.
Bài toán 27 (USA TSTST 2022). Tìm tất cả hàm số f : Z+ → Z thỏa mãn
 
f (mn)
= f (m)
n
với mọi số nguyên dương m, n.
Bài toán 28 (MEMO 2022). Tìm tất cả hàm số f : Z+ → Z+ sao cho f (1) ≤ f (2) ≤ f (3) ≤
f (4) ≤ . . . và
f (n) + n + 1, f (f (n)) − f (n)
đều là số chính phương với mọi số nguyên dương n.

4
Đoàn Quang Đăng Mathpiad

§1.4 Bất phương trình hàm


Bài toán 29 (IMO 2022). Tìm tất cả hàm số f : R+ → R+ sao cho với mỗi số thực dương x,
tồn tại duy nhất số thực dương y thỏa mãn

xf (y) + yf (x) ≤ 2.

Bài toán 30 (AUS 2022). Tìm tất cả hàm số f : [1; +∞) → [0; +∞) thỏa mãn

f (2f (x) + y) ≥ f x3 + f y 2 f (x) + x2 + xy


 

với mọi số thực x, y ≥ 1.

Bài toán 31 (Abel Competition Final 2021-2022). Tìm tất cả hàm số f : R+ → R+ thỏa mãn
q
f (x)f x1
  
1
f ≥1− ≥ x2 f (x)
x x

với mọi số thực dương x.

Bài toán 32 (Indonesia TST 2022). Tìm tất cả hàm số f : R → R thỏa mãn

f x2 − f y 2 ≤ (f (x) + y)(x − f (y))


 

với mọi số thực x, y.

Bài toán 33 (Philippine MO 2022). Tìm tất cả hàm số f : R → R thỏa mãn

f (a − b)f (c − d) + f (a − d)f (b − c) ≤ (a − c)f (b − d)

với mọi số thực a, b, c và d.

5
Đoàn Quang Đăng Mathpiad

§2 Lời giải
§2.1 Phương trình hàm trên tập số thực
Bài toán 1 (Việt Nam TST 2022)
Cho số thực α và xét hàm số φ(x) = x2 eαx với mọi x ∈ R. Tìm tất cả hàm số f : R → R
thỏa mãn
f (φ(x) + f (y)) = y + φ(f (x))
với mọi số thực x, y.

Lời giải. Giả sử tồn tại hàm số thỏa mãn yêu cầu bài toán.
Ký hiệu P (x, y) chỉ mệnh đề f (φ(x) + f (y)) = y + φ(f (x)) ∀x, y ∈ R.
Đặt φ(f (0)) = c. Từ P (0, y) thu được
f (f (y)) = y + c ∀y ∈ R.
Do đó f là một song ánh. Thay y bởi f (y) vào đẳng thức trên ta suy ra
f (y + c) = f (y) + c ∀x ∈ R.
Vì f là song ánh nên tồn tại duy nhất d ∈ R sao cho f (d) = 0. Từ P (d, y + c) thu được
f (φ(d) + f (y + c)) = y + c ∀y ∈ R.
Chú ý rằng f (f (y)) = y + c nên từ đẳng thức trên, kết hợp với f là đơn ánh ta thu được
φ(d) + f (y + c) = φ(d) + f (y) + c = f (y)
hay
φ(d) + c = 0.
Chú ý rằng φ(x) ≥ 0 ∀x ∈ R và đẳng thức xảy ra khi và chỉ khi x = 0 nên từ đẳng thức trên
ta suy ra
f (0) = d = 0.
Khi đó f (f (y)) = y ∀y ∈ R và từ P (x, 0) suy ra
f (φ(x)) = φ(f (x)) ∀x ∈ R.
Chú ý rằng φ(x) nhận mọi giá trị trên tập [0; +∞) nên ta suy ra f (x) ≥ 0 ∀x ≥ 0. Từ
P (x, f (y)) ta suy ra
f (φ(x) + y) = f (y) + f (φ(x)) ∀x, y ∈ R
hay
f (x + y) = f (x) + f (y) ∀x ≥ 0, y ∈ R.
Với hai số thực x, y, ta chọn z đủ lớn sao cho z > max{−y, 0}, khi đó
f (x + y) + f (z) = f (x + y + z) = f (x) + f (y + z) = f (x) + f (y) + f (z)
hay
f (x + y) = f (x) + f (y) ∀x ∈ R.
Kết hợp với f (x) ≥ 0 ∀x ≥ 0 ta dễ dàng thu được f (x) = kx ∀x ∈ R. Thay lại vào phương
trình ban đầu ta tìm được k = 1. Vậy tất cả hàm số cần tìm là
f (x) = x ∀x ∈ R.

6
Đoàn Quang Đăng Mathpiad

Bài toán 2 (District Olympiad 2022)


Tìm tất cả hàm số f : R → R thỏa mãn

f (f (y − x) − xf (y)) + f (x) = y (1 − f (x))

với mọi số thực x, y.

Lời giải. Dễ dàng kiểm ta được hàm f (x) ≡ 1 không thỏa mãn yêu cầu bài toán, do đó tồn tại
số thực c sao cho f (c) ̸= 1. Từ P (c, y) ta suy ra

f (f (y − c) − cf (y)) = y (1 − f (c)) − f (c) ∀y ∈ R,

chú ý rằng 1 − f (c) ̸= 0 nên dễ dàng suy ra f là toàn ánh.


Từ P (0, y) suy ra
f (f (x)) + f (0) = x (1 − f (0)) ∀x, y ∈ R.
Nếu f (0) = 1 thì ta suy ra f (f (x)) = −1 ∀x ∈ R. kết hợp với f là toàn ánh thì được
f (x) ≡ −1, thử lại ta thấy hàm số này không thỏa mãn yêu cầu bài toán. Xét trường hợp
f (0) ̸= 1, với hai số thực a, b sao cho f (a) = f (b), từ P (0, a), P (0, b) ta suy ra a = b, hay f là
đơn ánh.
Từ P (x, −1) ta suy ra f (f (−x − 1) − xf (−1)) = 1 ∀x ∈ R. Do f là toàn ánh nên tồn tại số
thực k sao cho f (k) = 1 hay

f (f (−x − 1) − xf (−1)) = 1 = f (k) =⇒ f (−x − 1) − xf (−1) = k ∀x ∈ R.

Từ đây thay x bởi −x − 1 thì được

f (x) − (−x − 1)f (−1) = k

hay f (x) = ax + b ∀x ∈ R. Thay lại vào phương trình ban đầu tìm được f (x) ≡ x, thử lại
thấy hàm số này thỏa mãn.

7
Đoàn Quang Đăng Mathpiad

Bài toán 3 (Iran MO Second Round 2022)


Tìm tất cả hàm số f : R → R thỏa mãn

f (xf (y) + f (x) + y) = xy + f (x) + f (y)

với mọi số thực x, y.

Lời giải. Giả sử tồn tại hàm số thỏa mãn yêu cầu bài toán.
Ký hiệu P (x, y) chỉ mệnh đề f (xf (y) + f (x) + y) = xy + f (x) + f (y) ∀x, y ∈ R.
Từ P (0, −f (0)) thu được f (−f (0)) = 0. Do đó tồn tại số thực c sao cho f (c) = 0. Từ P (c, c)
suy ra c2 = 0 hay c = 0. Như vậy f (x) = 0 ⇐⇒ x = 0.
 
Từ P (x, 0) ta suy ra f (f (x)) = f (x) ∀x ∈ R. Với mọi y ̸= 0, từ P f−y
(y)
, y ta suy ra

y2
    
y y
f f − =− +f − + f (y)
f (y) f (y) f (y)

y2
=⇒ f (y) = =⇒ f (y)2 = y 2 .
f (y)
Từ P (−1, −1) ta được f (−1) = −1. Khi đó từ P (−1, y) suy ra

f (y − f (y) − 1) = f (y) − y − 1 ∀y ∈ R.

Nếu tồn tại số thực a ̸= 0 sao cho f (a) = −a thì từ đẳng thức trên ta suy ra f (2a−1) = −2a−1.
Điều này dẫn đến −2a − 1 = 2a − 1 hoặc −2a − 1 = −2a + 1, giải cả hai trường hợp ta đều thu
được điều vô lý. Như vậy f (x) ̸= x ∀x ̸= 0. Khi đó từ f (x)2 = x2 ta suy ra f (x) = x ∀x ∈ R
(chú ý rằng f (0) = 0). Thử lại thấy hàm số này thỏa mãn.

8
Đoàn Quang Đăng Mathpiad

Bài toán 4 (Kosovo MO 2022)


Tìm tất cả hàm số f : R → R thỏa mãn

f (f (x − y) − yf (x)) = xf (y)

với mọi số thực x, y.

Lời giải. Giả sử tồn tại hàm số thỏa mãn yêu cầu bài toán.
Ký hiệu P (x, y) chỉ mệnh đề f (f (x − y) − yf (x)) = xf (y) ∀x, y ∈ R.
Từ P (0, 0) thu được f (f (0)) = 0. Từ P (x, 0) suy ra

f (f (x)) = xf (0) ∀x ∈ R.

Thay x bởi f (0) vào đẳng thức trên ta suy ra f (0)2 = f (0) hay f (0) = 0 hoặc f (0) = 1.
Trường hợp 1. f (0) = 0.

Khi đó f (f (x)) = 0 ∀x ∈ R. Nếu tồn tại số k ̸= 0 sao cho f (k) = 0. Khi đó từ P (k, y) ta suy
ra
kf (y) = f (f (k − y)) = 0 =⇒ f (y) = 0 ∀y ∈ R.
Thử lại ta thấy hàm số này thỏa mãn. Nếu f (x) = 0 ⇐⇒ x = 0 thì từ f (f (x)) = 0 ta suy ra
f (x) = 0 ∀x ∈ R, vô ly
Trường hợp 2. f (0) = 1.

Khi đó f (f (x)) = x ∀x ∈ R. Từ đây ta dễ dàng chứng minh f lả song ánh. Do đó, tồn tại số
thực a ̸= 0 sao cho f (a) = 0. Từ P (a, y) ta suy ra
a−y
a − y = f (f (a − y)) = af (y) =⇒ f (y) = ∀y ∈ R.
a
Thay lại vào phương trình ban đầu ta dễ dàng tìm được f (x) = x + 1 ∀x ∈ R hoặc f (x) =
1 − x ∀x ∈ R. Thử lại ta thấy chỉ có hàm số f (x) = 1 − x ∀x ∈ R thỏa mãn. Vậy tất cả
hàm số cần tìm là
f (x) = 0 ∀x ∈ R, f (x) = 1 − x ∀x ∈ R.

9
Đoàn Quang Đăng Mathpiad

Bài toán 5 (Kosovo TST 2022)


Tìm tất cả hàm số f : R → R thỏa mãn

f x2 + 2f (xy) = xf (x + y) + yf (x)


với mọi số thực x, y.

Lời giải. Giả sử tồn tại hàm số thỏa mãn yêu cầu bài toán.
Ký hiệu P (x, y) chỉ mệnh đề f (x2 ) + 2f (xy) = xf (x + y) + yf (x) ∀x, y ∈ R.
Từ P (0, 0) thu được f (0) = 0. Khi đó từ P (x, 0) ta suy ra f (x2 ) = xf (x) ∀x ∈ R. Từ đây
thay x bởi −x ta dễ dàng chứng minh được f là hàm lẻ.
Từ P (x, −y) ta suy ra

f x2 − 2f (xy) = xf (x − y) − yf (x) ∀x, y ∈ R.




Kết hợp với P (x, y) thì được

xf (x + y) + xf (x − y) = 2f x2 = 2xf (x)


hay
f (x + y) + f (x − y) = 2f (x) ∀x ̸= 0, y ∈ R.
Chú ý rằng với y = 0 thì đẳng thức vẫn đúng do f là hàm lẻ. Do đó

f (x + y) + f (x − y) = 2f (x) ∀x ∈ R.

Cho y = 0 thì được f (2x) = 2f (x) ∀x ∈ R. Do đó

f (x + y) + f (x − y) = f (2x) ∀x, y ∈ R.

Từ đây dễ dàng suy ra f (x + y) = f (x) + f (y) ∀x, y ∈ R. Ta tính f ((x + 1)2 ) bằng hai cách

f ((x + 1)2 ) = f (x2 + 2x + 1) = xf (x) + 2f (x) + f (1)


f ((x + 1)2 ) = (x + 1)f (x + 1) = (x + 1)(f (x) + f (1)) = xf (x) + xf (1) + f (x) + f (1)

Từ hai đẳng thức trên ta suy ra f (x) = xf (1) ∀x ∈ R. Thử lại ta thấy hàm số này thỏa mãn
yêu cầu bài toán.

10
Đoàn Quang Đăng Mathpiad

Bài toán 6 (IMO 2022 Malaysian Training Camp)


Tìm tất cả hàm số f : R → R thỏa mãn

f (xf (x) + 2y) = f (x)2 + x + 2f (y)

với mọi số thực x, y.

Lời giải. Giả sử tồn tại hàm số thỏa mãn yêu cầu bài toán.
Ký hiệu P (x, y) chỉ khẳng định f (xf (x) + 2y) = f (x)2 + x + 2f (y) ∀x, y ∈ R.
Từ P (0, 0) ta suy ra f (0) = f (0)2 + 2f (0) =⇒ f (0) ∈ {−1; 0}.
Trường hợp 1. f (0) = 0.

Ta xét các phép thế


P (0, x) =⇒ f (2x) = 2f (x) ∀x ∈ R;
P (x, 0) =⇒ f (xf (x)) = f (x)2 + x ∀x ∈ R.
Từ P (2x, 2yf (y)) ta suy ra
f (4xf (x) + 4yf (y)) = 4f (x)2 + 2x + 2f (2yf (y)) = 4f (x)2 + 2x + 4f (yf (y)) ∀x, y ∈ R.
Suy ra
x
f (xf (x) + yf (y)) = f (x)2 +
+ f (yf (y)) ∀x, y ∈ R.
2
Thay đổi vai trò x, y trong đẳng thức trên và đối chiếu với chính nó thì được

x y
f (x)2 + + f (yf (y)) = f (y)2 + + f (xf (x)) ∀x, y ∈ R
2 2
x y
=⇒ f (x) + + f (y) + y = f (y) + + f (x)2 + x =⇒ x = y ∀x, y ∈ R
2 2 2
2 2
điều này không thể xảy ra. Do đó không tồn tại hàm số thỏa mãn trong trường hợp này.
Trường hợp 2. f (0) = −1.

Ta đặt g(x) = f (x) + 1 ∀x ∈ R. Khi đó g(0) = 0 và P (x, y) viết lại thành


g(xg(x) − x + 2y) = g(x)2 − 2g(x) + x + 2g(y) ∀x, y ∈ R.
Từ P (0, y)ta suy ra g(2y) = 2g(y) ∀y ∈ R.
Từ P x, x2 ta thu được
g(xg(x)) = g(x)2 + x − g(x) ∀x ∈ R.
Từ P (2x, x) suy ra
g(4xg(x)) = 4g(x)2 − 2g(x) + 2x ∀x ∈ R
g(x) x
=⇒ g(xg(x)) = g(x)2 − + ∀x ∈ R.
2 2
Đối chiếu các kết quả thu được, suy ra
g(x) = x =⇒ f (x) = x − 1 ∀x ∈ R.
Thử lại ta thấy hàm số này thỏa mãn yêu cầu bài toán. Vậy tất cả hàm số cần tìm là
f (x) = x − 1 ∀x ∈ R.

11
Đoàn Quang Đăng Mathpiad

Bài toán 7 (IMO 2022 Malaysian Training Camp)


Tìm tất cả hàm số f : R → R thỏa mãn

f x2 + f (x + y) = y + xf (x + 1)


với mọi số thực x, y.

Lời giải. Giả sử tồn tại hàm số thỏa mãn yêu cầu bài toán.
Ký hiệu P (x, y) chỉ khẳng định f (x2 + f (x + y)) = y + xf (x + 1) ∀x, y ∈ R.
Đặt a = f (0). Từ P (0, x) ta thu được f (f (x)) = x ∀x ∈ R, suy ra f là song ánh.
Từ P (x, f (y) − x) ta suy ra

f x2 + y = f (y) + xf (x + 1) − x ∀x, y ∈ R.


Ký hiệu mệnh đề trên là Q(x, y). Khi đó từ Q(x, 0) ta được f (x2 ) = xf (x + 1) − x + a ∀x ∈ R.


Thay lại vào Q(x, y) thì được

f x2 + y = f x2 + f (y) − a ∀x, y ∈ R.
 

Đặt g(x) = f (x) − a, khi đó g(0) = 0 và g(x + y) = g(x) + g(y) ∀x ≥ 0, y ∈ R.


Nhận xét 1. g(x) + g(y) = g(x + y) ∀x ∈ R.

Chứng minh. Từ đây y = −x thì được g(x) = −g(−x) ∀x ≥ 0 hay g là hàm lẻ. Khi đó với
x < 0, y ∈ R ta có

g(x + y) = −g(−x − y) = −g(−x) + −g(−y) = g(x) + g(y).

Như vậy g(x + y) = g(x) + g(y) ∀x, y ∈ R. Chứng minh hoàn tất.
Nhận xét 2. g(g(x)) = x ∀x ∈ R.

Chứng minh. Ta thay f (x) = g(x) + a vào đẳng thức f (f (x)) = x thì được

g(g(x) + a) + a = x =⇒ g(g(x)) = x − g(a) − a ∀x ∈ R.

Từ đây thay x = 0 và chú ý g(0) = 0 ta suy ra g(a) + a = 0 hay g(g(x)) = x ∀x ∈ R. Chứng


minh hoàn tất.
Nhận xét 3. g (x2 ) = xg(x) ∀x ∈ R.

Chứng minh. Thay f (x) = g(x) + a vào P (x, 0) ta thu được

g x2 + g(x) + a + a = xg(x + 1) + ax ∀x ∈ R


=⇒ g x2 + g(g(x)) + g(a) + a = xg(x) + x(g(1) + a) ∀x ∈ R.




Ta thay x ∈ Q, x ̸= 0 vào đẳng thức trên và chú ý g (x2 ) = xg(x) ∀x ∈ Q ta suy ra g(1)+a = 1.
Điều này dẫn đến g (x2 ) = xg(x) ∀x ∈ R. Chứng minh hoàn tất.

12
Đoàn Quang Đăng Mathpiad

Từ các khẳng định trên, ta đi tính g ((x + 1)2 ) bằng hai cách. Ta có

g (x + 1)2 = (x + 1)g(x + 1) = xg(x) + xg(1) + g(x) + g(1)





g (x + 1)2 = g x2 + 2x + 1 = g x2 + g(2x) + g(1) = xg(x) + 2g(x) + g(1).
  

So sánh hai đẳng thức trên ta suy ra g(x) = g(1)x ∀x ∈ R hay f (x) = cx + a, ∀x ∈ R. Thay
lại vào phương trình ban đầu ta tìm được f (x) ≡ x và f (x) ≡ 2 − x. Thử lại ta thấy các hàm
số này thỏa mãn.
Vậy tất cả hàm số cần tìm là

f (x) = x ∀x ∈ R, f (x) = 2 − x ∀x ∈ R.

13
Đoàn Quang Đăng Mathpiad

Bài toán 9 (DAMO)


Tìm tất cả hàm số f : R → R thỏa mãn

f (2xy + f (x)) = xf (y) + f (yf (x) + x)

với mọi số thực x, y.

Lời giải. Giả sử tồn tại hàm số thỏa mãn yêu cầu bài toán.
Với x, y ∈ R, ký hiệu P (x, y) chỉ mệnh đề f (2xy + f (x)) = xf (y) + f (yf (x) + x).
Nếu f (x) ≡ c thì thay vào P (x, y) ta suy ra f (x) ≡ 0. Xét trường hợp f không là hàm hằng.
Nhận xét 1. f (0) = 0.

Chứng minh. Từ P (x, 0) ta được f (f (x)) = xf (0) + f (x) ∀x ∈ R. Nếu f (0) ̸= 0 thì từ đây
suy ra f đơn ánh trên R. Khi đó từ P (0, 0) thu được f (f (0)) = f (0) hay f (0) = 0, vô lý. Như
vậy f (0) = 0.
Nhận xét 2. f (x) = 0 ⇐⇒ x = 0.

Chứng minh. Giả sử tồn tại a ̸= 0 sao cho f (a) = 0. Khi đó từ P a, 21 ta được


     
1 1 1
af = f 2a. = f (a) = 0 =⇒ f = 0.
2 2 2

Từ P 12 , y ta suy ra f (y) = 0 ∀y ∈ R, mâu thuẫn do ta xét f khác hằng.




Nhận xét 3. Với mọi x ∈ R thì f (x) = x hoặc f (x) = 2x.

Chứng minh. Với x ̸= 0, giả sử f (x) ̸= 2x. Khi đó từ


     
x − f (x) f (x) − x f (x) − x
P x, =⇒ xf = 0 =⇒ f = 0.
2x − f (x) f (x) − 2x f (x) − 2x

Kết hợp với Nhận xét 2 ta suy ra f (x) = x.

Giả sử tồn tại b ̸= 0 sao cho f (b) = 2b. Từ P (b, y) suy ra

f (2by + 2b) = bf (y) + f (2by + b) ∀y ∈ R.

Thay y = − 21 vào đẳng thức trên, suy ra


   
1 1
2 = f (b) = bf − =⇒ f − = 2,
2 2

vô lý do f − 12 = − 12 hoặc f − 12 = −1. Như vậy f (x) ̸= 2x ∀x ̸= 0. Suy ra f (x) ≡ x. Thử


 

lại ta thấy hàm số này thỏa mãn. Vậy tất cả hàm số cần tìm là

f (x) ≡ 0, f (x) ≡ x.

16
Đoàn Quang Đăng Mathpiad

Bài toán 11 (Romania EGMO TST 2022)


Tìm tất cả hàm số f : R → R thỏa mãn

f (f (x) + y) = f (x2 − y) + 4yf (x)

với mọi số thực x, y.

x2 −f (x)
Lời giải. Ta thay y bởi 2
vào phương trình ban đầu thì được

(x2 − f (x))f (x) = 0 ∀x ∈ R.

Do đó f (x) = 0 hoặc f (x) = x2 với mọi x ∈ R. Chú ý rằng ta cũng có f (0) = 0 và từ P (0, y)
thu được f (y) = f (−y) hay f là hàm chẵn.
Dễ dàng kiểm tra được hàm số f (x) = x2 ∀x ∈ R và f (x) = 0 ∀x ∈ R thỏa mãn yêu cầu
bài toán. Ta sẽ chứng minh không tồn tại hàm thứ ba thỏa mãn. Giả sử tồn tại a ̸= 0 sao cho
f (a) = 0 và b ̸= 0 sao cho f (b) = b2 . Do f là hàm chẵn nên ta giả sử b > 0 (vì nếu không thì
ta xét f (−b) = b2 ). Từ P (a, −b) ta suy ra

f (−b) = f (a2 + b) =⇒ b2 = f (b) = f (a2 + b).

Nếu f (a2 + b) = 0 thì b = 0, vô lý. Nếu f (a2 + b) = (a2 + b)2 thì ta cũng thu được điều vô lý do

0 < b < a2 + b =⇒ b2 < (a2 + b)2 .

Như vậy tất cả hàm số cần tìm là f (x) ≡ x2 và f (x) ≡ 0.

18
Đoàn Quang Đăng Mathpiad

Bài toán 12 (HMMT 2022)


Tìm tất cả hàm số f : R\{0} → R thỏa mãn

f (x)2 − f (y)f (z) = x(x + y + z) (f (x) + f (y) + f (z))

với mọi số thực x, y, z thỏa mãn xyz = 1.

Lời giải. Giả sử tồn tại hàm số thỏa mãn. Ký hiệu P (x, y, z) chỉ phương trình ban đầu.
Nhận xét 1. f (x) ∈ 0, x2 − x1

∀x ̸= 0.

Chứng minh. Từ P (1, 1, 1) suy ra f (1) = 0. Với mọi x ̸= 0, ta có


     
1 2 1 1
P x, 1, =⇒ f (x) = x x + + 1 f (x) + f
x x x
       
1 1 1 1
P 1, x, =⇒ −f (x)f = x+ +1 f (x) + f .
x x x x

Điều này dẫn đến f (x)2 = −xf (x)f x1 hay f (x) = 0 hoặc f x1 = − f (x)
 
x
với mọi x ̸= 0. Thay
vào phương trình thứ nhất thì được
  
2 1 f (x)
f (x) = x x + + 1 f (x) −
x x
1
hay f (x) = 0 hoặc f (x) = x2 − x
với mọi x ̸= 0.

Nhận xét 2. Nếu tồn tại số thực t ̸= 1 sao cho f (t) = 0 thì f (x) = 0 ∀x ̸= 0.

Chứng minh. Với mọi x ̸= 0, ta có


     
1 2 1 1
P x, t, =⇒ f (x) = x x + +t f (x) + f
xt xt xt
       
1 1 1 1
P t, x, =⇒ −f (x)f =t x+ +t f (x) + f .
xt xt xt xt
Điều này dẫn đến tf (x)2 = −xf (x)f xt 1 1
= − xt f (x).
 
hay f (x) = 0 hoặc f xt
1
= − xt f (x), thay vào đẳng thức thứ nhất thì được

Xét các số thực x ̸= 0 thỏa mãn f xt
  
2 1 tf (x)
f (x) = x x + +t f (x) −
xt x
(
f (x) = 0
=⇒ 1
 .
+ t 1 − xt = x2 − x1 − t2 − 1t
 
f (x) = x x + xt
Chú ý rằng với t ̸= 1 thì x2 − x1 − t2 − 1t ̸= x2 − x1 nên f (x) = 0. Tóm lại f (x) = 0 ∀x ̸= 0.


1
Nếu f (x) ̸= 0 ∀x ̸= 1 thì ta suy ra f (x) = x2 − x
∀x ̸= 0, thử lại ta thấy hàm số này thỏa.
Vậy tất cả hàm số cần tìm là
1
f (x) = 0 ∀x ̸= 0 f (x) = x2 − ∀x ̸= 0.
x

19
Đoàn Quang Đăng Mathpiad

Bài toán 13 (China TST 2022)


Tìm tất cả hàm số f : R → R thỏa mãn

{f (xf (y) + 1) , f (yf (x) − 1)} = {xf (f (y)) + 1, yf (f (x)) − 1}



a = c, b = d
với mọi x, y ∈ R, trong đó {a, b} = {c, d} ⇐⇒ .
a = d, b = c

Lời giải. Giả sử tồn tại hàm số thỏa mãn yêu cầu bài toán. Ký hiệu P (x, y) chỉ khẳng định
{f (xf (y) + 1) , f (yf (x) − 1)} = {xf (f (y)) + 1, yf (f (x)) − 1} ∀x, y ∈ R.
Từ P (0, 0) ta suy ra {f (1), f (−1)} = {1, −1} . Ta xét trường hợp f (1) = 1 và f (−1) = −1,
trường hợp còn lại làm tương tự.
Nhận xét 1. Hàm f là toàn ánh.

Chứng minh. Từ P (x, 1) ta suy ra


{f (x + 1), f (f (x) − 1)} = {x + 1, f (f (x)) − 1} ∀x ∈ R.
Từ đây ta dễ dàng suy ra f là toàn ánh. Chứng minh hoàn tất.
Nhận xét 2. f (0) = 0.

Chứng minh. Do f là toàn ánh nên tồn tại số thực a sao cho f (a) = 0. Khi đó, từ P (a, a) ta
suy ra
{f (af (a) + 1) , f (af (a) − 1)} = {af (f (a)) + 1, af (f (a)) − 1}
=⇒ {af (f (a)) + 1, af (f (a)) − 1} = {1, −1} .
Giải hai trường hợp ta đều thu được

a=0
af (f (a)) = 0 =⇒ af (0) = 0 =⇒ .
f (0) = 0

Như vậy f (0) = 0 trong mọi trường hợp. Chứng minh hoàn tất.

Dễ dàng kiểm tra được hàm số f (x) = x ∀x ∈ R thỏa mãn yêu cầu bài toán. Ta xét trường
hợp tồn tại số thực x0 ̸= 0 sao cho f (x0 ) ̸= x0 .
Từ P (x0 − 1, 1) ta suy ra
{f (x0 ), f (f (x0 − 1) − 1)} = {x0 , f (f (x0 − 1)) − 1} .
Do f (x0 ) ̸= x0 nên từ đây suy ra
f (x0 ) = f (f (x0 − 1)) − 1, x0 = f (f (x0 − 1) − 1) . (1)

Từ P (1, x0 + 1) thu được


{f (f (x0 + 1) + 1) , f (x0 )} = { f (f (x0 + 1) + 1, x0 } .
Tương tự, ta cũng suy ra
f (x0 ) = f (f (x0 + 1)) + 1, x0 = f (f (x0 + 1) + 1) . (2)

20
Đoàn Quang Đăng Mathpiad

+ +
Tìm tất cả hàm số f : → thỏa mãn

2bf f a2 + a) = f (a + 1)f (2ab)


 

với mọi số nguyên dương a, b.

Giả sử tồn tại hàm số thỏa mãn yêu cầu bài toán.
Ký hiệu P (a, b) chỉ khẳng định 2bf (f (a2 ) + a)) = f (a + 1)f (2ab) ∀a, b ∈ +
.’
Từ P (1, b) ta suy ra
+
2bf (f (1) + 1) = f (2)f (2b) ⇒ f (2b) = 2bc ∀b ∈

với c = f (ff(1)+1)
(2)
là hằng số.
Với số nguyên dương k, từ P (2k, 1) ta thu được

2f f 4k 2 + 2k = f (2k + 1)f (4k) ⇒ f (2k + 1) = 2kc + 1.


 

Với a lẻ, b chẵn, từ P (a, b) ta suy ra (a − 1)c + 1 = ac hay c = 1. Do đó f (x) = x với mọi số
nguyên x ≥ 2.
Chú ý rằng ta có f (f (1) + 1) = f (2) suy ra f (1) + 1 = 2 hay f (1) = 1. Từ đó, ta kết luận được
f (x) = x ∀x ∈ + . Thử lại ta thấy hàm số này thỏa mãn yêu cầu bài toán.
Vậy tất cả hàm số cần tìm là
f (x) = x ∀x ∈ + .

38
Đoàn Quang Đăng Mathpiad

Bài toán 25 (Japan MO Final 2022)


Tìm tất cả hàm số f : Z+ → Z+ thỏa mãn

f f (n) (m) + mn = f (m)f (n)

với mọi số nguyên dương m, n, trong đó f k (n) = f (f (. . . f ( n) . . .)).


| {z }
k

Lời giải. Giả sử tồn tại hàm số thỏa mãn yêu cầu bài toán.
Ký hiệu P (m, n) chỉ khẳng định f f (n) (m) + mn = f (m)f (n) ∀m, n ∈ Z+ .
Nhận xét 1. f là đơn ánh.

Chứng minh. Giả sử a, b là hai số nguyên dương sao cho f (a) = f (b), khi đó, từ P (m, a) và
P (m, b) ta suy ra
ma = mb ⇒ a = b.
Do đó, f là đơn ánh.
Nhận xét 2. f (n) > 1 ∀n ∈ Z+ .

Chứng minh. Giả sử phản chứng, khi đó tồn tại n0 ∈ Z+ sao cho f (n0 ) = 1. Cố định m, từ
P (m, n0 ) suy ra
f (m) + mn0 = f (m).1 ⇒ mn0 = 0,
điều này không thể xảy ra. Như vậy f (n) ̸= 1 ∀n ∈ Z+ .
Nhận xét 3. Nếu a, b là hai số nguyên dương sao cho f a (n) = f b (n) ∀n ∈ Z+ thì a = b.

Chứng minh. Giả sử a > b, đặt c = a − b > 0. Khi đó chú ý rằng f là đơn ánh nên từ

f a (n) = f b (n) ⇒ f c (n) = n ∀n ∈ Z+ .

Do đó tồn tại n1 sao cho f (n1 ) = 1, điều này mâu thuẫn với Nhận xét 2. Do đó, ta phải có
a = b. Chứng minh hoàn tất.

Trở lại bài toán, từ P (m, n) và P (n, m) ta suy ra

f f (n) (m) = f f (m) (n) ∀n ∈ Z+ .

Ta thay m bởi f (n) vào phương trình trên thì được


2 (n)
ff (n) = f f (n)+1 (n) ∀n ∈ Z+ . (1)

Áp dụng Nhận xét 3, suy ra

f 2 (n) = f (n) + 1 ∀n ∈ Z+ .

Khi đó, với m, n nguyên dương, ta có

f m+1 (n) = f m (n) + 1 = f m−1 (n) + 2 = ... = f (n) + m. (2)

Thay m bởi f (n) − 1 vào phương trình (2) thì được

f f (n) (n) = 2f (n) − 1 ∀n ∈ Z+ . (3)

41
Đoàn Quang Đăng Mathpiad

Khi đó, từ (1) và (3) ta suy ra

f (n)2 − 2f (n) + 1 = n2 ⇒ f (n) = n + 1 ∀n ∈ Z+ .

Thử lại ta thấy hàm số này thỏa mãn yêu cầu bài toán. Vậy tất cả hàm số cần tìm là

f (n) = n + 1 ∀n ∈ Z+ .

42
Đoàn Quang Đăng Mathpiad

Bài toán 26 (Taiwan TST 2022, Round 1)


Tìm tất cả hàm số f : Z → Z thỏa mãn
   
f (x) + f (y) f (x) + f (y)
f + f (x) = f (f (y)) +
2 2

với mọi số nguyên x, y, trong đó ký hiệu ⌊x⌋ chỉ số nguyên lớn nhất không vượt quá x.

Lời giải. Giả sử tồn tại hàm số thỏa mãn yêu cầu bài toán.
Ký hiệu P (u, v) chỉ việc thay bộ (x, y) bởi bộ (u, v) vào phương trình ban đầu.
Ký hiệu Im(f ) = {f (x) | x ∈ Z}. Từ P (x, y) và P (y, x) ta suy ra
f (f (x)) + f (x) = f (f (y)) + f (y) ∀x, y ∈ Z.
Do đó, tồn tại hằng số N ∈ Z sao cho f (f (x)) + f (x) = N ∀x ∈ Z hay
f (x) + x = N ∀x ∈ Im(f ).
Khi đó, phương trình ban đầu viết lại thành
   
a+b a+b
f − = N − a − b ∀a, b ∈ Im(f ). (*)
2 2
Từ phương trình trên, ta thấy rằng với a, b ∈ Im(f ) thì
   
a+b a+b
N −a−b+ =N− ∈ Im(f ).
2 2
Mặt khác,từ f (x) = N − x ∀x ∈ Im(f ) ta suy ra với x ∈ Im(f ) thì N − x ∈ Im(f ). Điều này
dẫn đến với a, b ∈ Im(f ) thì
    
a+b a+b
N− N− = ∈ Im(f ).
2 2
Ta chứng minh Im(f ) có duy nhất một phần tử. Giả sử phản chứng, khi đó tồn tại u > v sao
cho m, n ∈ Im(f ). Đặt u = v + k với k ∈ Z+ . Nếu k ≥ 2 thì theo nhận xét trên ta suy ra
   
u+v k
=v+ ∈ Im(f ).
2 2
Tiến hành tương tự, ta suy ra v + kn ∈ Im(f ) với mọi n ∈ Z+ , trong đó dãy (kn ) xác định bởi
 
kn
k1 = k, kn+1 = ∀n ≥ 1.
2
Dễ thầy (kn ) là dãy giảm và bị chặn dưới nên  tụ, do vậy, tồn tại số nguyên m đủ lớn sao
 hội
km
cho km = km+1 = . . . Chú ý rằng với km = ta suy ra km = 1. Như vậy v, v + 1 ∈ Im(f ).
2
Thay a = v + 1 và b = u vào phương trình (*) thu được
   
u+u+1 u+u+1
N − 2u = f (u) − u = f − = N − 2u − 1,
2 2
vô lý. Như vậy Im(f ) chỉ chứa một phần tử hay f là hàm hằng. Thử lại ta thấy hàm hằng thỏa
mãn yêu cầu bài toán.

43
Đoàn Quang Đăng Mathpiad

Tìm tất cả hàm số f : Z+ → Z+ sao cho f (1) ≤ f (2) ≤ f (3) ≤ f (4) ≤ . . . và

f (n) + n + 1, f (f (n)) − f (n)

đều là số chính phương với mọi số nguyên dương n.

Giả sử tồn tại hàm số thỏa mãn yêu cầu bài toán.
p
Đặt b = f (n) + n + 1 ∀n ∈ Z+ . Vì

f (n) + n + 1 > f (n − 1) + n − 1 + 1 ∀n > 1

nên dãy (b ) tăng thực sự, chú ý rằng b1 ≥ 2 nên ta suy ra

b ≥b 1 + 1 ≥ . . . ≥ b1 + n − 1 ≥ n + 1 ∀n ∈ Z+ .

Hay nói cách khác f (n) ≥ n2 + n ∀n ∈ Z+ .


Với số nguyên dương n bất kỳ, theo giả thiết, tồn tại số nguyên dương j, k sao cho

f (f (n)) + f (n) + 1 = j 2 , f (f (n)) − f (n) = k 2 .

Dễ thấy j > k và

2f (n) + 1 = (f (f (n)) + f (n) + 1) − (f (f (n)) − f (n)) = j 2 − k 2 = (j − k)(j + k).

Vì j > k nên j ≥ k + 1 và j + k ≥ 2k + 1, khi đó

2f (n) + 1 ≥ 2k + 1 =⇒ f (n) ≥ k

hay
f (f (n)) − f (n) ≥ f (n)2 .
Từ các kết quả trên ta suy ra

f (f (n)) = f (n)2 + f (n) ∀n ∈ Z+ .

Khi đó
p p
b ( ) = f (f (n)) + f (n) + 1 = f (n)2 + 2f (n) + 1 = f (n) + 1 ∀n ∈ Z+ .

Với số nguyên dương n0 bất kỳ, ta chọn n đủ lớn sao cho f (n) > n0 (hiển nhiên chọn được do
f (n) → +∞ khi n → +∞). Ta có

f (n) + 1 = b ( ) ≥b ( ) 1 + 1 ≥ f (n) + 1 =⇒ b ( ) 1 = f (n).

Tiến hành tương tự ta thu được b = i + 1 ∀i < f (n) hay b = n0 + 1.


Như vậy b = n + 1 ∀n ∈ Z+ . Thay vào ta suy ra f (n) = n2 + n ∀n ∈ Z+ . Thử lại thấy hàm
số này thỏa mãn. Vậy tất cả hàm số cần tìm là

f (n) = n2 + n ∀n ∈ Z+ .

46
DOÃN QUANG TIẾN
HUỲNH KIM LINH
TÔN NGỌC MINH QUÂN
NGUYỄN MINH TUẤN

CHINH PHỤC
OLYMPIC
TOÁN

x y z
n n n

Chuyên đề
ĐA THỨC VÀ SỐ HỌC

TẠP CHÍ VÀ TƯ LIỆU TOÁN HỌC


TẠP CHÍ VÀ TƯ LIỆU TOÁN HỌC

Copyright © 2019 by Tap chi va tu lieu toan hoc.

All rights reserved. No part of this book may be reproduced or distributed in any form
or by anymeans, or stored in data base or a retrieval system, without the prior written
the permission of the author.
LỜI GIỚI THIỆU

S
ố học hay đa thức đều là các chủ đề thường xuyên xuất hiện trong các đề thi học
sinh giỏi cấp quốc gia, các kì thi khu vực cũng như quốc tế với các bài toán khó
tới rất khó được các nước cũng như các thầy cô phát triển rất nhiều. Đa thức là
mảng mà chứa đựng trong nó các yếu tố về đại số, giải tích, hình học và cả các tính
chất về số học. Chính vì thế ta có thể xem đa thức có thể xem như là các bài toán tổ
hợp giữa các mảng khác của Toán học cũng như đóng vai trò liên kết các mảng đó lại
với nhau thành một thể thống nhất. Và chúng ta cũng biết rằng số học không phải tự
nhiên rất nhiều nhà toán học, những người làm toán gọi nó với cái tên mỹ miều là Bà
chúa của toán học. Thế giới các con số rất quen thuộc với chúng ta trong cuộc sống
thường ngày, là một thế giới hết sức kì lạ đầy bí ẩn: loài người đã phát hiện trong đó
bao nhiêu tính chất rất hay, bao nhiêu quy luật rất đẹp và bất ngờ đồng thời cũng đang
chịu "bó tay" trước nhiều sự kiện, nhiều dự đoán. Điều lí thú là nhiều mệnh đề khó
nhất của số học được phát biểu rất đơn giản, ai cũng hiểu được ; nhiều bài toán khó
nhưng có thể giải rất sáng tạo với những kiến thức số học phổ thông đơn giản. Không ở
đâu như trong số học,chúng ta lại có thể lần theo được dấu vết của những bài toán cổ
xưa để đến được với những vấn đề mới đang còn chờ đợi người giải – Trích từ cuốn
sách Số học – Bà chúa của toán học – Hoàng Chúng. Chính vì thế sự kết hợp của 2
mảng kiến thức này sẽ mang tới cho chúng ta những bài toán đẹp nhưng vẻ đẹp thì
không bao giờ là dễ để chúng ta chinh phục cả, nó luôn ẩn chứa những điều khó khăn
và “nguy hiểm”. Trong chủ đề của bài viết này, chúng ta sẽ đi khám phá cũng như
chinh phục phần nào vẻ đẹp của sự kết hợp đó.

Nhóm tác giả


ĐA THỨC VÀ SỐ HỌC
TẠP CHÍ VÀ TƯ LIỆU TOÁN HỌC

CHINH PHỤC OLYMPIC TOÁN


Đa thức và số học

CÂU CHUYỆN TOÁN HỌC


Joseph – Louis Lagrange

N
ếu nhạc sĩ người Áo Wolgang Amadeus Mozart (1756 – 1791) đã để lại cho đời sau
những bản nhạc tuyệt vời thì hơn hai trăm năm sau, trong những năm đầu tiên
của thế kỷ 21, với lòng tôn sùng một bậc tài danh, những người yêu âm nhạc cổ
điển chỉ còn biết lắng nghe để thưởng thức âm điệu mà thôi. Nhưng cùng thời với ông, ở
Âu châu còn có một thiên tài khác cũng lừng danh, nhưng tiếng tăm không vang ra ngoài
nhân thế vì ở trong bộ môn hạn hẹp là toán học. Tuy vậy công trình của ông để lại, không
những được người đời sau ghi chú học hỏi, mà còn được áp dụng trong nhiều bộ môn
khoa học thực dụng cho đời sống hàng ngày, và cả trong những chương trình thám hiểm
CHINH PHỤC OLYMPIC TOÁN

không gian và vũ trụ để tìm hiểu về nguồn gốc đời sống của con người và tương lai về
sau. Người được nhắc đến trong bài này là Joseph – Louis Lagrange (1736 – 1813), một nhà
toán học lỗi lạc nhất, mà cũng là người thật khiêm tốn, đã được nhiều bậc vương giả Âu
châu trọng vọng vào cuối thế kỷ 18 và đầu thế kỷ 19. Để phê bình về danh nhân này, Đại
đế Napoléonđã từng nói rằng: "Lagrange thật là một kim tự tháp cao vời trong bộ môn toán
học". Lời nói của Hoàng đế thường đi đôi với việc làm và ông đã phong cho Lagrange làm
Bá tước, cử ông làm Thượng Nghị sĩ và còn vinh tặng ông Đệ Nhất Đẳng Bắc Đẩu Bội Tinh.
Nhiều bậc vương giả khác ở Âu châu như Quốc vương xứ Sardinia và Hoàng
đế Frederick của Đức quốc cũng đã hết mực tôn vinh Lagrange.

Joseph – Louis Lagrange (1736 – 1813)

Chinh phục olympic toán| 1


Bồi dưỡng học sinh giỏi

Lagrange sinh ngày 25 – 1 – 1736 tại Turin (Italia), mất ngày 10 – 4 – 1813 tại Paris (Pháp).
Ông được xem là một trong những thiên tài toán học lớn nhất trong lịch sử toán học, đồng
thời cũng là một nhân vật đặc sắc trong thời đại ông – một thời đại đầy xáo động về mọi
mặt: chính trị, văn hóa, xã hội.

Ông là người Pháp, nhưng có pha dòng máu Ý. Tổ phụ của Lagrange là một đại úy kỵ binh
Pháp, đã tới phục vụ dưới trướng của Quốc vương đảo Sardinia là Charles Emmanuel
II. Sau đó vị sĩ quan kỵ binh điển trai và anh dũng này tới định cư ở tỉnh Turin và được
nhận vào làm rể của dòng họ quyền quý Conti của nước Ý. Thân phụ của Lagrange cũng
được hưởng cái may mắn trong hôn ước như thế và đã kết duyên cùng cô Marie – Thérèse
Gros là ái nữ độc nhất của một bác sĩ giàu có ở tỉnh Cambiano. Cặp tài tử và giai nhân này
sống vui hạnh phúc và hai ông bà có đến mười người con nhưng tất cả đều mệnh yểu khi
còn tuổi ấu thơ và chỉ về sau mới may mắn được thêm cậu út là Joseph – Louis ra đời ngày

TẠP CHÍ VÀ TƯ LIỆU TOÁN HỌC


25 tháng Giêng năm 1736 để rồi lớn lên và trở thành một nhà bác học danh tiếng lẫy lừng.
Thân phụ của Lagrange cũng là người có tài trí, và đã có thời làm Tổng Giám Đốc ngân
sách binh bị cho đảo quốc Sardinia. Ông xây dựng nên một tài sản khá lớn, lại cộng thêm
với của hồi môn của bà vợ nên gia đình được vào hạng giàu có lớn trong tỉnh. Nhưng ông
lại ham mê đầu tư nên theo với nền kinh tế đương thời ở Châu Âu, tài sản của gia đình bị
giảm sút dần dần đến khánh kiệt khi Lagrange bước vào tuổi trưởng thành. Cậu con út
được cưng chiều nay lại không được thừa kế chút di sản nào của cha mẹ, vì thật ra không
còn gì đáng giá để lại. Trong cuộc đời sau này của Lagrange, ông thường cho sự phá sản đó
lại là một điều may cho mình và đã nói rằng: "Nếu tôi được hưởng một gia tài lớn thì chắc
tôi đã không dựa vào Toán Học để xây dựng đời mình".

Sự Nghiệp Toán Học

Vào đầu thế kỷ 18, nền khoa học nói chung, và toán học nói riêng, chưa phải là một môn
học chính cho sĩ tử, nên lúc mới đầu Lagrange theo về văn học cổ điển. Nhưng trong khi
nghiên cứu về văn hoá Hy Lạp, chàng thanh niên được biết đến những công trình về Hình
Học của những vĩ nhân toán học đời trước như Euclid (330 – 275 tr. CN) và Archimedes (287
– 212 tr. CN). Tuy vậy chàng cũng không chú ý lắm về những môn này. Nhưng sau
đó Lagrange được đọc một bài tham luận của nhà thiên văn học Edmund Halley (1656 –
1742) ca tụng môn Giải Tích Học mới được xây dựng và hoàn bị bởi nhà bác học Isaac
Newton (1642 – 1727) và cho rằng môn toán học này vượt trội hơn môn Hình Học. Bài này
gợi trí tò mò của chàng thanh niên và anh đã dồn hết tâm trí vào để trong một thời gian
ngắn học được hết những gì đã được công bố trên sách vở về những phép tính vi phân và
tích phân trong môn giải tích học. Sự hiểu biết về toán học cao cấp này đã làm

2 | Tạp chí và tư liệu toán học


Đa thức và số học

cho Lagrange được bổ nhiệm làm giáo sư toán học tại Trường Pháo binh Hoàng gia ở tỉnh
Turin khi chàng mới 16 tuổi. Nơi đây, hàng ngày Lagrange giảng bài cho lớp sinh viên mà
người nào cũng lớn tuổi hơn mình. Tuy vậy chàng cũng thừa uy tín để chinh phục được
mọi người và có nhiều năng lực để tổ chức được một Hội Nghiên cứu Khoa học là khởi thủy
của một Trung tâm để sau này trở thành Viện Hàn lâm Khoa học Turin. Chỉ mấy năm sau,
vào năm 1759, khi Lagrange mới 23 tuổi, mà Hội Nghiên cứu do chàng sáng lập đã xuất bản
được Tập san đầu tiên. Nhưng ta phải nói rằng với một tâm địa tốt, luôn luôn nâng đỡ các
bạn đồng nghiệp mà nhiều bài khảo cứu toán học đăng trên những số đầu tiên của tập san
nghiên cứu, tuy ký tên những tác giả khác, mà thực ra là công trình của Lagrange vì đã
được chàng sửa chữa và viết lại hoàn toàn. Trong những trường hợp này, có một tác giả
của một bài viết thật đặc sắc – sau khi đã được Lagrange sửa lại – được mọi người chú ý và
ngợi khen, và khi chuyện tới tai quốc vương Sardinia, tác giả được vời tới và giao cho
giữ Bộ Hải quân là một chức vụ thật quan trọng vì Sardinia là một đảo quốc. Chỉ có một
CHINH PHỤC OLYMPIC TOÁN

điều là trong lịch sử môn toán học, người ta thấy ông này chỉ viết ra được một bài độc
nhất là bài mà do sự nâng đỡ của Lagrange đã giúp cho ông được địa vị trong triều. Cũng
trong thời gian sáng tác phong phú này mà Lagrange đã tạo dựng nên lý thuyết cho
môn Cơ học Giải tích.

Một bài toán được biết từ thời thượng cổ là bài toán đẳng chu (isoperimetric problem) khi
người ta tìm một hình phẳng có môt diện tích cực đại cho một chu vi cho sẵn. Lời giải tất
nhiên là hình tròn nhưng phải đợi đến thế kỷ 17 mọi người mới chú ý đến những bài toán
cực đại hay cực tiểu khi hai anh em toán gia Bernouilli, người Thụy Sĩ, ông anh tên
là James (1654 – 1706) và người em là John (1667 – 1748) thách thức nhau giải bài toán sau
đây:

"Từ một điểm khởi đầu O, thả trôi một cái vòng theo một đường giây nhẵn thín nằm trong mặt
phẳng thẳng đứng, để cho tuột xuống một điểm A ở dưới. Phải uốn đường giây theo hình nào để
cho thời gian tuột được ngắn nhất."

Dĩ nhiên hai anh em nhà Bernouilli không những đưa ra nhiều lời giải, nhưng lại còn đề ra
nhiều bài toán khác nữa thuộc loại này. Những bài viết của anh em nhà Bernouilli đã gây
phấn khởi cho một thiên tài toán học khác người Thụy Sĩ là Leonhard Euler (1707 – 1783) là
học trò của John Bernouilli, và Euler đã đưa ra phương pháp tổng quát để giải những bài
toán mà James Bernouilli đã đề nghị khi xưa. Ông cũng đặt tên cho phép tính này là Phép
tính biến thiên (Calculus of Variations). Nhưng ngưòi thực sự đã đưa phép giải những bài
toán để tìm ra những trường hợp tối ưu lại là Lagrange, lúc đó vẫn chỉ còn là một giáo sư ở
Turin. Tuy chàng thanh niên, mới ở tuổi 19 và ở thế hệ sau, chỉ nghiên cứu bài toán đẳng

Chinh phục olympic toán| 3


Bồi dưỡng học sinh giỏi

chu sau những bậc tiền bối danh tiếng vang lừng, nhưng Lagrange đã có những nhận xét
tân kỳ để giải bài toán, và đã có can đảm viết một bức thư cho Euler, đang là Chủ tịch Ủy
ban Toán học của Viện Hàn lâm Khoa học Vương quốc Phổ ở Berlin, để đưa ra một lời giải mà
chàng cho là có tính cách tổng quát. Cũng may là Euler tuy là một thiên tài toán học thời
ấy, danh tiếng vang lừng, nhưng cũng là người rộng lượng, ông nhận ngay ra rằng
phương pháp của Lagrange đã giải toả được một vài thắc mắc của chính ông khi tìm
phương pháp giải bài toán và Euler đã nhường cho Lagrange công bố kết quả ra trước. Hơn
hai trăm năm sau, những khoa học gia không gian, khi tìm những qũy đạo tối ưu để đưa
những vệ tinh thám sát lên những hành tinh xa vời trong Thái dương hệ, đều phải viết
những phương trình có tên chung là phương trình Euler – Lagrange. Không mấy người, dù
chỉ trong một khoảnh khắc, đã nghĩ đến tài trí siêu việt của Lagrange và đức tính cao
thượng của Euler, là những người đầu tiên đã khai phá ra môn toán học này. Trong những
năm đầu tiên của một cuộc đời nghiên cứu và sáng tác toán học của Lagrange, những bài

TẠP CHÍ VÀ TƯ LIỆU TOÁN HỌC


viết đều được đăng trong những tập san đề là Miscellanea Taurinensia tất cả tổng cộng có 5
Tập. Những bài viết này dù là để tên những học sinh hay những người cộng sự đều là do
chàng giáo sư tuổi mới ngoài hai mươi đưa ra ý kiến và duyệt xét cùng sửa đổi lại. Tuy là
ở một thị thành hẻo lánh nơi có hội toán học mà Lagrange sáng lập mà sau này trở
thành Viện Hàn lâm Khoa học Turin, nhưng những tập san toán học phát xuất từ nơi đây,
mà ngay ở số đầu tiên đã nói về Phép tính biến thiên, đã được toàn thế giới khoa học ở Âu
châu chú ý tới và làm cho Lagrange đương nhiên trở thành một toán gia hàng đầu được
mọi người ngưỡng mộ.

Ngoài toán gia Euler, Lagrange còn được một trưởng bối người Pháp là D’Alembert (1717 –
1783) nhiệt tình ủng hộ. Những người bạn tốt này đều nghĩ rằng chỉ khi nào chàng tới một
thủ đô văn học và tiếp xúc với những toán gia hàng đầu của thế kỷ thì tài năng
của Lagrange mới được nảy nở toàn diện. Trước đó Lagrange đã được mời tới London,
nhưng đi được nửa đường khi vừa tới Paris thì bị ốm. Nơi đây ông được tiếp đón trọng
vọng và vì sức khoẻ chưa hồi phục được nên đành phải trở về Turin một thời gian để chờ
cơ hội khác. Mấy năm sau thì dịp may đó tới khi đại toán gia Euler nhận lời mời của Viện
Hàn lâm Khoa học St Petersburg để chuyển cư tới đó. Do đề nghị của 2 nhà toán học
D’Alembert và Euler, Hoàng đế Frederick của Phổ Quốc đã viết cho Lagrange một bức thư
đại để nói là Hoàng đế Frederick vĩ đại nhất châu Âu muốn được toán gia lừng danh nhất
của thế kỷ tới vương triều để hàng ngày cùng nhau bàn luận. Lagrange đã nhận lời để tới
Berlin thế vào chỗ trống của Euler và trong khoảng 20 năm khi cư ngụ ỏ Phổ Quốc ông đã
viết hơn một trăm bài khảo luận toán học để đăng trên các tập san ở Turin và ở Berlin.
Cũng trong thời gian này mà Lagrange hoàn tất tác phẩm vĩ đại nhất của đời ông về
môn Cơ học Giải tích.

4 | Tạp chí và tư liệu toán học


Đa thức và số học

Khi mất ông được chôn cất trong điện Panthéon tại Paris.

Nguồn nội dung: Diễn đàn toán học Việt Nam – VMF
Nguồn ảnh: Wikipedia
CHINH PHỤC OLYMPIC TOÁN

Chinh phục olympic toán| 5


Bồi dưỡng học sinh giỏi

Chuyên đề
ĐA THỨC VÀ SỐ HỌC
Tạp chí và tư liệu toán học

Trong chủ đề này, thay vì việc phân chia các dạng toán cụ thể kèm lời phân tích chi tiết
từng dạng thì mình sẽ mang tới cho bạn đọc một tuyển tập các bài toán hay và khó để ôn
tập và nâng cao kiến thức chuẩn bị cho kì thi học sinh giỏi cũng như các kì thi khác mà các
bạn tham gia. Nào chúng ta cùng bắt đầu nhé!

CÁC KIẾN THỨC CƠ BẢN

TẠP CHÍ VÀ TƯ LIỆU TOÁN HỌC


1. ĐA THỨC.

Đơn thức theo biến x là biểu thức có dạng m.x n trong đó m là hằng số và n là số nguyên
không âm.
Đa thức là tổng hữu hạn của nhiều đơn thức hay đa thức là biểu thức có dạng
P  x   ak x k  ak 1 x k 1  ...  a1 x  a0  ak  0  .
Khi đó
 ai được gọi là các hệ số của đa thức.
Nếu ai  , i thì ta gọi đa thức P  x tức tập các đa thức hệ số nguyên.
 n được gọi là bậc của đa thức, ký hiệu là deg P  n .

2. MỘT SỐ TÍNH CHẤT CẦN NẮM.

Tính chất 1. Với hai số nguyên a , b trong đó b  0 , nếu tồn tại số nguyên c sao cho a  bc
thì ta gọi a chia hết cho b hoặc b chia hết a hoặc b là ước của a hoặc cũng hay gọi a là
bội của b .
Ký hiệu. a b hoặc b|a .
Tính chất 2. Với P  x và a , b là hai số nguyên khác nhau, ta luôn có
P  a  P b  a  b .
Chứng minh. Giả sử P  x   an x n  an1 x n1  ...  a1 x  a0  an  0  .
 
Sử dụng hằng đẳng thức a k  b k   a  b  a k 1  a k 2 b  ...  b k 1 với k  1 là số nguyên.

Khi đó P  a   P  b    a  b   an  an1  an2 b  ...  b n1   an1  a n2  a n3b  ...  b n2   ...  a1  .

6 | Tạp chí và tư liệu toán học


Đa thức và số học

Từ đó bài toán được chứng minh.


Tính chất 3. Cho đa thức P  x  với hệ số nguyên.
Khi đó không tồn tại ba số phân biệt a , b , c sao cho P  a   b , P  b   c.P  c   a.

3. NHỮNG ĐỊNH LÝ QUAN TRỌNG.

Hai đa thức f  x  , g  x  được gọi là nguyên tố cùng nhau nếu ước chung lớn nhất của hai
đa thức đó là một hằng số.
Nhưng vì ước chung lớn nhất của hai đa thức chỉ khác nhau hằng số nên nếu hai đa thức
nguyên tố cùng nhau thì ta có thể xem ước chung lớn nhất nhất của chúng là 1. Nên ta ký
hiệu  f  x  , g  x    1.
Định lý Bézout. Cho hai đa thức P  x  , Q  x   x. Gọi d  x  là ước chung lớn nhất của
CHINH PHỤC OLYMPIC TOÁN

hai đa thức P  x  , Q  x  .
 Khi đó tồn tai hai đa thức U  x  ,V  x  sao cho d  x   U  x  .P  x   V  x  .Q  x  .
 Nếu  P  x  , Q  x    1 thì tồn tại các đa thức U  x  ,V  x  sao cho
U  x  .P  x   V  x  .Q  x   1.
Định lý Schur. Cho P  x   x là đa thức khác đa thức hằng. Khi đó có vô hạn các số
nguyên tố p thỏa mãn tính chất: Ứng với số nguyên tố p tồn tại số nguyên m sao cho p P  m  .
Chứng minh.
Ta xét các trường hợp sau
Trường hợp 1. Hệ số tự do bằng 0. Khi đó p P  p  với mọi số nguyên tố p.
Trường hợp 2. Hệ số tự do bằng 1. Tức là P  0   1 .
Ta giả sử tập các số nguyên tố thỏa mãn bài toán là hữu hạn. Gọi p là số nguyên tố lớn
nhất trong các số đó. Ta xét P  p !  1  mod p ! . Ta gọi q  1 là số nguyên tố khác mà thỏa
mãn p P  p ! thì q  p vì nếu q  p thì do q  p ! nên từ q P  p ! ta suy ra q 1 là vô lí.
Nhưng q  p thì lại mâu thuẫn với chuyện p là số nguyên tố lớn nhất.
Trường hợp nếu P  0   a  1 .
1
Ta xét đa thức Q  x   P  ax  thì Q  x    x  và Q  0   1 . Theo trên tồn tại vô hạn các số
a
nguyên tố p sao cho ứng với mỗi số p thì luôn tồn tại số nguyên m để cho p Q  m  .
Nhưng nếu p Q  x   p P  ax  . Định lí được chứng minh.
Định lý Dirichlet về số nguyên tố. Cho a,b là các số tự nhiên với a  0,  a , b   1. Khi đó tập
hợp an  b , n   chứa vô hạn số nguyên tố.

Chinh phục olympic toán| 7


Bồi dưỡng học sinh giỏi

Định lý về dãy tuần hoàn. Cho f , g là hai đa thức hệ số nguyên và nguyên tố cùng nhau.
Đặt an  gcd  f  n  , g  n   , n  1, 2, 3... . Khi đó dãy  an  tuần hoàn.
Chứng minh – Nguyễn Hữu Điển.
Do f , g là hai đa thức hệ số nguyên và nguyên tố cùng nhau nên tồn tại hai đa thức F , G
và số nguyên dương a sao cho f .F  g.G  a . Khi đó do chia hết cho cả f  n  , g  n  nên ta có
f  n  a , n. . Ta sẽ chứng minh an tuần hoàn theo chu kì a.
Ta chứng minh rằng an an  a .
Thật vậy ta có f  n   f  n  a   mod a  mà an a , an f  n   an f  n  a  .
Tương tự ta có an g  n  a  . Như vậy ta có an an  a .
Lập luận tương tự ta có an an  a . Vậy an  an  a .
Chú ý. Hai đa thức nguyên tố cùng nhau khi ước chung lớn nhất của chúng là một đa thức hằng.

TẠP CHÍ VÀ TƯ LIỆU TOÁN HỌC


Bổ đề Hensel. Cho đa thức f  x  hệ số nguyên và số nguyên tố p . Nếu phương trình đồng
dư f  x   0  mod p  có đúng r nghiệm phân biệt x1  , x2  ,  , xr thuộc đoạn  1; p  sao cho
1 1 1

    0  mod p  , i  1, r thì phương trình đồng dư


f ' xi
1
f  x   0  mod p k  có đúng r nghiệm

nguyên phân biệt thuộc đoạn  1, p k  .


Chứng minh.
Với k  1 hiển nhiên đúng.
Giả sử khẳng định đúng với k  1 . Điều đó có nghĩa là trên đoạn  1, p k  , phương trình

f  x   0  mod p k  có đúng r nghiệm x1 k  , x2 k  ,  , xr k  và f ' xi     0  mod p  , i  1, r .


k

 
Ta cần chứng minh f  x   0 mod p k  1 có đúng r nghiệm thuộc  1, p k  1  .
Gọi x 0 là một nghiệm của phương trình f  x   0  mod p   1 .
Xét số x1  x0  p k t , t   0; p  1 với t là nghiệm duy nhất của phương trình
f  x0 
 f '  x0  t  0  mod p 
pk
Ta sẽ chứng minh x 1 là nghiệm của f  x   0 mod p k  1   2 .
f '  x0  f ''  x0  f  x0 
n

Ta có f  x1   f  x0    x1  x0    x1  x0   x1  x0 
2 n

1! 2! n!
f '  x0  f '  x0  f  n
 x0 
p t p t
2 n
f  x1   f  x0   pkt  k
 k

1! 2! n!
f    x0 
i


Suy ra f  x1   f  x0   f '  x0  p k t mod p k 1 vì  i!

8 | Tạp chí và tư liệu toán học


Đa thức và số học

 f x  
 f  x1   p k  k 0  f '  x0  t   mod p k 1   0  mod p k 1 
 p 
Vậy phương trình  2  có ít nhất r nghiệm.
Thật vậy, giả sử x là nghiệm của  2  , gọi x 0 là nghiệm của  1  . Ta có
f  x   0  mod p k  1   f  x   f  x0   mod p k   x  x0  mod p k   x  x0  p k t
f  x0 
Theo chứng minh trên thì t là nghiệm của phương trình  f '  x0  t  0  mod p  .
pk
 
Vậy phương trình f  x   0 mod p k  1 có đúng r nghiệm.
Từ cách chứng minh trên ta rút ra được nhận xét sau.
  
Nhận xét. f x0  p k t  f  x0   f '  x0  p kt mod p k  1 
Công thức nội suy Lagrange.
Cho đa thức P  x  có bậc nhỏ hơn  n  1  và  n  1  số thực phân biệt xi , i  1, n  1 .
CHINH PHỤC OLYMPIC TOÁN

n1 n1 x  xj
Khi đó P  x  được xác định duy nhất như sau: P  x    P  xi    .
i 1 j 1 xi  x j
ji

Chinh phục olympic toán| 9


Bồi dưỡng học sinh giỏi

ĐỀ BÀI

Câu 1. Tìm các đa thức P  x  có hệ số nguyên, không âm, bậc không lớn hơn 6 thỏa
P  7   102013 .
Câu 2. Cho a , b , c  thỏa mãn các đa thức f  x   ax 2  bx  c và
g  x    a  b  x 2  c  a  x  a  b có nghiệm chung. Chứng minh rằng a  b  2c 3 .
Câu 3. Tồn tại hay không đa thức f  x   x 2  ax  b với a , b nguyên thỏa mãn a 2  4b  0 và
nhận giá trị chính phương tại 2010 điểm phân biệt.
Câu 4. Chứng minh rằng không tồn tại đa thức P với hệ số nguyên, khác đa thức hằng có
bậc không quá 4 thỏa mãn: tồn tại 5 số nguyên x1 , x2 ,..., x5 khác nhau sao cho
P  x1  P  x2  P  x3  P  x 4  P  x5   1.
Câu 5. Chứng minh x n  x  1 bất khả quy trên Z  x  , n  2

TẠP CHÍ VÀ TƯ LIỆU TOÁN HỌC


Câu 6. Cho đa thức P là đa thức hệ số và tồn tại số nguyên dương m sao cho
P  1  ; P  2  ;...; P  m  không chia hết cho m . Chứng minh rằng P  a   0 với mọi a  .
Câu 7. Cho P  x   an x n  an1 x n1  a1x  a0 là đa thức với a0 , a1 ,  , an là các số nguyên
dương. Đặt P1  x   P  x  và Pk  x   P  Pk 1  x   với k  1 . Tồn tại hay không M  0 sao cho
với m  M ta có m|PP m  m 
Câu 8. Cho đa thức P  x  là đa thức hệ số nguyên với bậc n  2 . Chứng minh rằng đa thức
P  P  x    x có nhiều nhất n nghiệm.
Câu 9. Tìm tất cả các đa thức P hệ số nguyên sao cho P  n |2 n  1 với mọi số nguyên
dương n .
Câu 10. Gọi x0 , x1 ,  , xn là các số nguyên thỏa mãn x0  x1   xn . Chứng minh rằng một
n!
trong các số P  x0  , P  x1  , P  xn  không nhỏ hơn với P  x  là đa thức bậc n .
2n
Câu 11. Giả sử các đa thức P  x  , Q  x  , R  x  và S  x  thỏa mãn:

P  x 5   xQ  x 5   x 2 R  x 5    x 4  x 3  x 2  x  1  S  x 
Chứng minh rằng khi đó đa thức P  x  chia hết cho x  1 .
Câu 12. Tìm tất cả các đa thức P có hệ số nguyên sao cho với mọi số nguyên tố p và mọi
cặp số tự nhiên  u , v  thỏa mãn P| uv  1  thì ta có P  P  u  P  v   1  .
Câu 13. Tìm tất cả các đa thức P hệ số nguyên sao cho với mọi số tự nhiên a , b , c ta luôn
có a  b  c P  a   P  b   P  c  .
Câu 14. Cho m , n là các số nguyên dương. Chứng minh rằng n m khi và chỉ khi
P  x  Q  x  , trong đó P  x  ; Q  x  là các đa thức hệ số nguyên được xác định:

10 | Tạp chí và tư liệu toán học


Đa thức và số học

P  x   x n1  Cn1 x n2  ...  Cnn1


Q  x   x m1  Cm1 x m2  ...  Cmm1
Câu 15. Tìm tất cả các đa thức P hệ số nguyên thỏa mãn tính chất: Với m , n là hai số
nguyên tố cùng nhau thì hai số P  m  ; P  n  cũng là hai số nguyên tố cùng nhau.
Câu 16. Cho P  x  , Q  x    x  là hai đa thức monic bất khả quy thỏa mãn với n đủ lớn thì
P  n  , Q  n  có cùng tập ước nguyên tố. Chứng minh rằng P  Q .
Câu 17. Gọi đa thức P  x   x  khác đa thức hằng và gọi n , k là hai số nguyên dương.
Chứng minh rằng tồn tại số nguyên dương a sao cho mỗi f  a  , f  a  1  ,..., f  a  n  1  có
ít nhất k ước nguyên tố phân biệt.
Câu 18. Cho đa thức P  x  , Q  x  là hai đa thức có hệ số nguyên nguyên tố cùng nhau. Đặt
an   P  n  , Q  n   . Chứng minh dãy an  tuần hoàn.
Câu 19. Với hai đa thức có hệ số nguyên p  x  , q  x  ta viết p  x   q  x  mod 2  nếu
CHINH PHỤC OLYMPIC TOÁN

p  x   q  x  có tất cả các hệ số đều chia hết cho 2. Cho dãy đa thức pn  x  thỏa mãn
p1  x   p2  x   1 và pn 2  x   pn 1  x   xpn  x  1  , với mọi n là số tự nhiên.
Chứng minh rằng p2n  x   1  mod 2  .
Câu 20. Giả sử p2n  x   1 là hai số tự nhiên lớn hơn 1 và pn  x   pn .
Dãy số vô hạn n  1, 2, 3... được xác định như sau
pn 4  pn 3  xpn 2   pn 2  xpn 1   xpn 2   x  1  pn 2  xpn 1
Chứng minh rằng trong dãy số nói trên chứa vô hạn số đôi một nguyên tố cùng nhau.
Câu 21. Với mỗi số tự nhiên n , ta kí hiệu f  n  là tổng các chữ số của nó biểu diễn trong
hệ thập phân. Ta xây dựng dãy số như sau

u1  n   f  n  ; u2  n   f  f  n   ;...; uk  n   f f ... f  n  ... 
k

Chứng minh rằng với mọi số tự nhiên n , tồn tại số tự nhiên d sao cho
uk  n   ud  n  , k  d  1  .
Câu 22. Cho abc là một số nguyên tố.
Chứng minh rằng phương trình ax 2  bx  c  0  1  không có nghiệm hữu tỉ.

   
Câu 23. Cho đa thức P  x   x  2 x  3 x  6 . Chứng minh rằng với mọi số nguyên tố
2 2 2

p đều tìm được số nguyên dương n sao cho P  n  p .


Câu 24. Chứng minh rằng với mọi số nguyên p ta có thể tìm được số nguyên dương f  x 
sao cho n không phải là số chính phương.
Câu 25. Cho đa thức P  x   x 2017  x 1000  1 . Tồn tại hay không các số tự nhiên a1 , a2 ,...

 
, a2018 sao cho tích P  ai  .P a j ai a j với mọi i  j .

Chinh phục olympic toán| 11


Bồi dưỡng học sinh giỏi

Câu 26. Với mọi số tự nhiên m , n , chứng minh rằng  n ! chia hết cho m khi và chỉ khi tồn
n
tại đa thức hệ số nguyên f  x    ak x k thỏa mãn  a0 , a1 ,..., an , m   1, m f  j  với mọi j
k 0

nguyên dương.
Câu 27. Cho đa thức f  x   2009x 5  x 4  x 3  x 2  2006x  1. Chứng minh rằng với n là số
nguyên tùy ý thì các số f  n  , f  f  n   ,..., f ... f  n  ...  đôi một nguyên tố cùng nhau.
1
 n  1
Câu 28. Gọi P  x  là đa thức bậc n thỏa mãn với k  0,1, , n thì P  k    
 k 
Định đa thức P  n  1  .
u  1990, u2  1989, u3  2000
Câu 29. Cho dãy un  được xác định như sau  1 .
u
 n 3  19 un 2  9 un 1  un  1991, n  1, 2...
a) Với mọi n gọi rn là số dư của phép chia un cho 1992. Chứng minh rằng dãy rn  là một

TẠP CHÍ VÀ TƯ LIỆU TOÁN HỌC


dãy tuần hoàn.
b) Chứng minh rằng tồn tại vô số số x của dãy un  sao cho
5x 1992  5x 1994  4x 1975  8x 1945  2 x 1990  11x 2  48
chia hết cho 1992.
Câu 30. Chứng minh rằng tồn tại hằng số dương c sao cho với mọi số nguyên dương n
và các số thực a1 , a2 ,  , an , nếu P  x    x  a1  x  a2   x  an  thì
max P  x   c n max P  x 
x0,2  x0,1

Câu 31. Tìm tất cả các đa thức P  x   x  và m  


sao cho m  2 n P  n  là số chính
phương với mọi số nguyên dương n .
Câu 32. Cho P  x    x  , p là số nguyên tố và x  a  mod p  . Chứng minh rằng

P  x   P  a    x  a  P '  a   mod p 2 
Câu 33. Với p là số nguyên tố, đặt h  x  là đa thức có hệ số nguyên sao cho

h  0  , h  1  ,  , h  p 2  1  là một hệ thặng dư đầy đủ modulo p 2 . Chứng minh rằng

h  0  , h  1  ,  , h  p 3  1  là một hệ thặng dư đầy đủ modulo p 3 .


Câu 34. Với số nguyên n  3 , đặt f  x  , g  x  là đa thức với hệ số thực sao cho các điểm

 f  1  , g  1   ,  f  2  , g  2   , ,  f  n  , g  n   trong tập 2
là các đỉnh của đa giác n cạnh
theo thứ tự ngược chiều kim đồng hồ. Chứng minh rằng ít nhất một trong các đa thức f , g
có bậc không nhỏ hơn n  1 .

12 | Tạp chí và tư liệu toán học


Đa thức và số học

p1
Câu 35. Cho p là một số nguyên tố lẻ. Chứng minh rằng có ít nhất giá trị
2
p 1
n  0, 1, 2  , p  1 sao cho  k !n
k 0
k
không chia hết cho p .

Câu 36. Có tồn tại hay không một dãy số thực và khác 0 là a1 ; a2 ;  , an thỏa với mỗi n 
thì đa thức a0  a1 x  an x n có đúng n nghiệm trên .
Câu 37. Cho P  x   x thỏa mãn P  x  là số chính phương với mọi x nguyên thì
P  x   Q 2  x  với Q  x   x  .
Câu 38. Tìm tất cả các đa thức số hệ số nguyên thỏa mãn a  b là số chính phương thì
P  a   P  b  cũng là số chính phương, trong đó a , b là các số tự nhiên.
Câu 39. Giả sử m , p là các số nguyên tố khác nhau. Chứng minh rằng nếu có một số tự

nhiên x nào đó mà p là ước của P  x   x  x  ...  1 thì ta có p  1  mod m  .
m1 m2

Câu 40. Cho đa thức P  x  có bậc n và có n nghiệm phân biệt x1 , x2 ,  , xn .
CHINH PHỤC OLYMPIC TOÁN

Chứng minh rằng:


P ''  x1  P ''  x2  P ''  xn 
a)   0
P '  x1  P '  x2  P '  xn 
1 1 1
b)   0
P '  x1  P '  x 2  P '  xn 
Câu 41. Cho f là một đa thức có hệ số hữu tỉ và bậc không nhỏ hơn 2, và dãy  an  chỉ
gồm các số hữu tỉ thỏa mãn f  an 1   an với mọi số nguyên dương n. Chứng minh rằng
dãy  an  tuần hoàn.
P '  1 n
Câu 42. Cho P  x  là đa thức bậc n với hệ số thực sao cho P  1  khác 0 và  
P  1 2
Chứng minh rằng P  x  luôn có ít nhất một nghiệm x 0 sao cho x0  1 .
Câu 43. Giả sử tồn tại đa thức hệ phức P , Q , R thỏa mãn P a  Qb  Rc  a , b , c  .
1 1 1
Chứng minh rằng   1
a b c
Câu 44. Cho đa thức P  x  và Q  x  với số thực k bất kì thỏa mãn Pk  z  P  z   k và
Qk  z  |Q  z   k . Chứng minh rằng P0  Q0 và P1  Q1 suy ra được P  x   Q  x  .
Câu 45. Cho đa thức P  x    x  ,deg P  2 . Chứng minh rằng tồn tại m 
để P  m ! là
hợp số.
Câu 46. Cho f  x  là đa thức với hệ số hữu tỉ bậc lớn hơn hoặc bằng 2. Xét dãy an  các số
hữu tỉ thỏa mãn điều kiện f  an 1   an , n  1 . Chứng minh rằng tồn tại k  1 để
an k  an  n  1  .

Chinh phục olympic toán| 13


Bồi dưỡng học sinh giỏi

Câu 47. Tìm tất cả đa thức   x  sao cho  p  2 p  p , p là số nguyên tố


Câu 48. Xét đa thức T  x   x 3  17 x 2  1239x  2001 . Đặt
T1  x   T  x  , T2  x   T T1  x   , , Tn1  x   T Tn  x   với mọi n  1, 2, 3...
Chứng minh rằng tồn tại số nguyên n  1 sao cho Tn  x   x chia hết cho 2003 với mọi số
nguyên x .
Câu 49. Tìm tất cả các cặp số nguyên a , b sao cho tồn tại đa thức P  x   x  sao cho tích

x 2
 ax  b   P  x  là đa thức được viết dưới dạng:
x n  cn1 x n1   c 1 x  c 0 với c 0 , c 1 ,  , c n 1 bằng 1 hoặc 1
Câu 50. Cho hai đa thức hệ số nguyên
P  x   an x n  an1 x n1  a1x  a0
Q  x   bn x n  bn1 x n1  b1 x  b0

TẠP CHÍ VÀ TƯ LIỆU TOÁN HỌC


Biết rằng an  bn là một số nguyên tố và an 1  bn 1 . Gọi m là là một nghiệm hữu tỷ chung
của hai đa thức. Chứng minh rằng m là một số nguyên.
Câu 51. Hỏi có tất cả bao nhiêu đa thức Pn  x  bậc n chẵn thỏa mãn các điều kiện
 Các hệ số của Pn  x  thuộc tập M  0; 1; 1 và Pn  0   0 .

 Tồn tại đa thức Q  x  có các hệ số thuộc M sao cho Pn  x   x  1 .Q  x  .


2
 
Câu 52. Cho dãy số nguyên  an n 1 thỏa mãn m  n|am  an với mọi số tự nhiên m,n phân

biệt. Giả sử tồn tại đa thức P  x  sao cho an  P  n  , n . Chứng minh rằng tồn tại đa thức
Q  x  sao cho an  Q  n  , n .
Câu 53. Cho n là số nguyên dương và a1 , a2 ,..., an là các số thực dương.
Ta đặt g  x    x  a1  x  a2  ...  x  an  .
Gọi a0 là một số thực bất kì và đặt f  x    x  a0  g  x   x n1  b1x n  ...  bn x  bn 1 .
Chứng minh rằng b1 , b2 ,..., bn  1 đều là số âm khi và chỉ khi a0  a1  a2  ...  an .
Câu 54. Cho F là tập các đa thức  có hệ số nguyên và phương trình   x   1 có nghiệm
nguyên. Cho trước một số nguyên dương k , tìm giá trị nhỏ nhất của m  1 theo k thỏa
mãn tồn tại   F sao cho   x   m có đúng k nghiệm nguyên phân biệt.
Câu 55. Cho hai đa thức P  x  , Q  x   x  nguyên tố cùng nhau và khác đa thức hằng.
Chứng minh rằng không có quá ba số thực  thỏa mãn P  x   Q  x  là bình phương của
một đa thức.
Câu 56. Chứng minh rằng nếu đa thức f  x    x  có bậc n và nhận giá trị nguyên tại
n  1 giá trị nguyên liên tiếp từ a  a  n , a  thì f  x   , x  .

14 | Tạp chí và tư liệu toán học


Đa thức và số học

Câu 57. Tìm tất cả các đa thức P  x   x  sao cho với mọi a , b mà a không là nghiệm

của P  x  thì P  a  P  a  b   P  b  .
Câu 58. Xét đa thức P  x   x n  an1 x n1  ...  a1 x  a0 với n  2, n  . Giả sử P  x  có n
nghiệm là x1 , x2 ,..., xn . Kí hiệu max  xi  là số lớn nhất trong các số x1 , x2 ,..., xn . Chứng
n
1
minh rằng P  x      2n2 n1 , x  max  xi  với   0 .
i 1 x  xi
Câu 59. Giả sử R là nghiệm của phương trình x n  a1 x n1  ...  an1 x  an  0 và đặt
n n
A   a j , B   ja j . Khi đó thì ta có A A  R B .
j 1 j 1

Câu 60. Cho đa thức P  x  là đa thức monic bậc n  1 có n nghiệm thực là x1 , x2 ,..., xn
phân biệt và khác 0 . Chứng minh rằng:
 1
n1
1 1 1
CHINH PHỤC OLYMPIC TOÁN

  ...  
x 1 P  x 1  x 2 P   x 2  xn P  xn  x1x2 ...xn
n
xk
Câu 61. Cho n  2 và đa thức P  x  xác định bởi P  x    . Chứng minh rằng phương
k 0 k !

trình P  x   0 không có nghiệm hữu tỉ.


Câu 62. Cho p là một số nguyên tố. Tìm tất cả các đa thức f  x  với hệ số nguyên sao cho
với mọi số nguyên dương n , f  x  là ước của p n  1.
Câu 63. Cho số nguyên dương n và số nguyên tố p lớn hơn n  1 . Chứng minh rằng đa
x x2 xp
thức P  x   1    không có nghiệm nguyên.
n  1 2n  1 pn  1
Câu 64. Cho đa thức P  x   x 3  11x 2  87 x  m  m   . Chứng minh rằng với mọi m tồn tại
số nguyên n sao cho P  n  191 .
Câu 65. Cho m là số nguyên dương, tìm số nghiệm của phương trình x 2  x  mod m  .
Câu 66. Cho p là số nguyên tố  p  3  . Xét đa thức
f  x    p  1  x p  2   p  2  x p  3  3x 2  2 x  1 .


Biết rằng hệ A  a1 , a2 , , ap  là một hệ thặng dư đầy đủ modulo p. Chứng minh rằng khi
đó hệ B   f  a  , f  a  , , f  a  cũng là một hệ thặng dư đầy đủ modulo p.
1 2 p

Câu 67. Xét đa thức P  x   x 3  14x 2  2 x  1 . Chứng minh rằng tồn tại số nguyên dương n
sao cho với mọi số nguyên x ta có 101 P  P  P  x    x .

Chinh phục olympic toán| 15


Bồi dưỡng học sinh giỏi

Câu 68. Cho tập S  p1 , p2 ,  , pk  là tập hợp k số nguyên tố phân biệt và P  x  là đa thức
với hệ số nguyên sao cho với mọi số nguyên dương n đều tồn tại p i trong S sao cho
pi P  n  . Chứng minh rằng tồn tại i sao cho pi P  n  ,n  *
.
Câu 69. Cho đa thức P  x   x 3  153x 2  111x  38 .
a) Chứng minh rằng trong đoạn  0; 3 2000  tồn tại ít nhất một số nguyên dương a sao cho
P  a  32000 .
b) Hỏi trong đoạn  0; 3 2000  có tất cả bao nhiêu số nguyên dương a sao cho P  a  chia hết
cho 32000 .
Câu 70. Tìm tất cả các đa thức f với hệ số nguyên sao cho f  n  f  m   n m .
Câu 71. Cho a , b , c , d , e , f là các số nguyên dương. Giả sử rằng S  a  b  c  d  e  f là ước
của các số abc  def và ab  bc  ca  de  ef  fd . Chứng minh rằng S là hợp số.

TẠP CHÍ VÀ TƯ LIỆU TOÁN HỌC


Câu 72. Tìm tất cả các đa thức P với hệ số nguyên thỏa mãn
P  n |2557 n  213.2014, n  *

Câu 73. Cho P là đa thức hệ số nguyên, có bậc n  1 và k là số nguyên dương bất kỳ. Xét
đa thức Q  x   P k  x  với P được tác động k lần. Chứng minh rằng có nhiều nhất n số
nguyên t sao cho Q  t   t .
Câu 74. Cho A là tập vô hạn các số nguyên dương. Tìm tất cả các số nguyên dương n
thỏa mãn với mọi a là phần tử của A thì
1  a  a2  ...  an 1  a1!  a2!  ...  an!
Câu 75. Cho P , Q là hai đa thức hệ số nguyên không âm, khác đa thức hằng. Xét dãy số
 Q  n  , n  1 . Chứng minh rằng tồn tại vô hạn số nguyên tố p thỏa mãn: ứng
P n
xn  2016
với mỗi p , tồn tại số nguyên dương m sao cho p xm .
Câu 76. Tìm tất cả các đa thức P hệ số nguyên thỏa mãn P  p  2 p  p , với mọi số nguyên
tố p .
Câu 77. Cho P  x  , Q  x  là các đa thức hệ số nguyên khác đa thức hằng. Giả sử rằng đa
thức P  x  .Q  x   2009 có ít nhất 25 nghiệm nguyên phân biệt. Chứng minh rằng bậc của
mỗi đa thức P  x  , Q  x  đều không nhỏ hơn 3.
Câu 78. Gọi d  n  là ước nguyên tố nhỏ nhất của số nguyên n , với n  1, 0, 1 và ta kí
hiệu d  1   d  0  , d  1   0.
Tìm tất cả các đa thức P  x  với hệ số nguyên thỏa mãn P  n  d  n    n  d  P  n   .

16 | Tạp chí và tư liệu toán học


Đa thức và số học

Câu 79. Tìm tất cả các số nguyên dương k thỏa mãn tồn tại đa thức f  x  với các hệ số
đều nguyên, có bậc lớn hơn 1 sao cho với mọi số nguyên tố p và mọi số tự nhiên a , b mà
p ab  k thì p f  a  f  b   k .
Câu 80. Cho P là đa thức hệ số nguyên thỏa mãn P  0   0 và  P  0  , P  1  ,...  1 . Chứng
minh rằng có vô hạn số n sao cho  P  n   P  0  , P  n  1   P  1  ,   n .
Câu 81. Cho p là số nguyên tố và P  x  là các đa thức bậc d hệ số nguyên thỏa mãn
 P  0   0, P  1   1
 Với mọi số nguyên dương n thì số dư trong phép chia P  n  cho p là 0 hoặc 1 .
Chứng minh rằng d  p  1 .
Câu 82. Chứng minh rằng không tồn tại đa thức P hệ số thực deg P  n  1 sao cho P  m 
là số nguyên tố với mọi số nguyên dương m .
CHINH PHỤC OLYMPIC TOÁN

Câu 83. Cho n  , n  3 và đa thức f  x   x n  an1 x n1  a1 x  a0  x  thỏa mãn a0


chẵn, ak  an  k chẵn với mọi k  1, n  1 .
Giả sử thêm rằng tồn tại hai đa thức g  x  , h  x   x thỏa mãn deg g  deg h , mọi hệ số
của h  x  đều lẻ và f  x   g  x  .h  x  , x .
Chứng minh rằng f  x  có ít nhất một nghiệm nguyên.
Câu 84. Cho đa thức f  x  monic, hệ số nguyên, bất khả quy và f  0  không phải là số

 
chính phương. Chứng minh rằng g  x   f x 2 cũng là đa thức bất khả quy.
Câu 85. Cho đa thức hệ số nguyên f  x   an x n  an1 x n1  a1x  a0 thỏa mãn điều kiện
a0  a1  a2  an và a0 là số nguyên tố thì f  x  bất khả quy.
Câu 86. Tìm tất cả các đa thức P  x  , Q  x  hệ số nguyên thỏa mãn với dãy số  xn  xác
định bởi x0  2014, x2 n1  P  x2 n  , x2 n 2  Q x2 n 1  , n  thì mỗi số nguyên dương m là
ước của một số hạng khác 0 nào đó của  xn  .
Câu 87. Chứng minh rằng không tồn tại đa thức
P  x   an x n  an1 x n1  ...  a1 x  a0  x  bậc n  1
sao cho P  0  , P  1  ,... đều là số nguyên tố.
 x  với a0 chẵn và an k  ak
Câu 88. Cho đa thức P  x   x n  an1 x n 1  ...  a1 x  a0 , P  x  
chẵn, với mọi k  1, n  1. Giả sử P  x   Q  x  R  x  , với Q  x  , R  x  là các đa thức hệ số
nguyên khác hằng, deg  Q  x    deg  R  x   và tất cả các hệ số của R  x  đều lẻ. Chứng
minh rằng, đa thức P  x  có nghiệm nguyên.

Chinh phục olympic toán| 17


Bồi dưỡng học sinh giỏi

Câu 89. Chứng minh rằng với mọi số nguyên dương n thì đa thức P  x    x 2  x   1 là
2n

đa thức bất khả quy trên .


Câu 90. Giả sử n là số tự nhiên lớn hơn hoặc bằng 2 và P  x   x n  an1 x n1  ...  a1 x  1 là
đa thức hệ số nguyên dương. Giả sử ak  an  k với mọi k  1, n  1. Chứng minh rằng tồn tại
 y P  x 
vô hạn cặp số nguyên dương  x , y  sao cho:  * .
 x P  y 
Câu 91. Chứng minh rằng, với mỗi số nguyên dương n tồn tại đa thức P  x   x  bậc n
sao cho P  0  , P  1  ,..., P  n  phân biệt và tất cả các số đó đều có dạng

2.2019 k  3, k  .
Câu 92. Chứng minh rằng tồn tại tập vô hạn các điểm
..., P3 , P2 , P1 , P0 , P1 , P2 , P3 ,...

TẠP CHÍ VÀ TƯ LIỆU TOÁN HỌC


trong mặt phẳng thỏa mãn tính chất: Với ba số nguyên a , b , c phân biệt thì các điểm
Pa , Pb , Pc thẳng hàng khi và chỉ khi a  b  c  2014.
Câu 93. Cho x1  x2  ...  xn là n , n  3 số thực thỏa mãn:
x2  x1  x3  x2  x 4  x3  ...  xn  xn1
Giả sử đa thức P  x  có n nghiệm thực là các giá trị x1 , x2 ,..., xn . Chứng minh rằng giá trị
lớn nhất của P  x  đạt được tại một điểm x0   xn1 , xn  .
Câu 94. Cho P  x   an x n  an1 x n1  ...  a1 x  a0 là đa thức hệ số nguyên thỏa mãn điều kiện
P  r   P  s   0 trong đó r , s là các số nguyên thỏa mãn điều kiện 0  r  s. Chứng minh
rằng tồn tại k  0,1, 2,..., n sao cho ak  s.
Câu 95. Cho P  x  , Q  x  là các đa thức hệ số nguyên. Đặt an  n ! n. Chứng minh rằng nếu
P  an  P n
 , n thì  , n và Q  n   0 .
Q  an  Q n
Câu 96. Cho P  x  là đa thức bậc n  5 với hệ số nguyên và
P  x   an x n  an1 x n1  ...  a1 x  a0
Giả sử P  x  có n nghiệm nguyên phân biệt là 0,  2 ,...,  n . Tìm tất cả các số nguyên k sao
cho thỏa mãn P  P  k    0.
Câu 97. Tìm số nguyên dương n nhỏ nhất sao cho tồn tại đa thức P  x  bậc n có hệ số
nguyên thỏa mãn: P  0   0, P  1   1 và với mọi   *
thì
 P     2   P     1 P    là bội của p với p là số nguyên tố
Câu 98. Chứng minh rằng với mọi m  tồn tại đa thức Pm  x  có hệ số hữu tỉ thỏa mãn
với mọi n  *
thì 12 m 1  2 2 m1  ...  n2 m1  Pm  n  n  1  

18 | Tạp chí và tư liệu toán học


Đa thức và số học

Câu 99. Cho P  x  , Q  x  , R  x  là ba đa thức hệ số thực thỏa mãn:


P  Q  x    P  R  x    c , x  với c  const 
Chứng minh rằng: P  x  là hằng số hoặc  Q  x   R  x   là hằng số.
Câu 100. Chứng minh rằng tồn tại các đa thức hệ số nguyên S1 , S2 ,  tương tứng với các
biến x1 , x2 ,  , y1 , y 2 ,  thỏa mãn với mọi số nguyên n  1
n
 nd n

 d  Sd
d
  d  x
 d  y d
d  *
d|n d|n  
Với hàm tổng chạy qua các ước nguyên dương d của n .
Chú ý. Lưu ý rằng ta chỉ xét đến các đa thức trong trường  x  . Ví dụ, xét hàm S1  x1  y1
và S2  x2  y 2  x1 y 1 trong trường hợp n  2 ta được
S12  2S2   x12  y12   2   x2  y 2  là hàm  *  thỏa mãn yêu cầu bài toán.
CHINH PHỤC OLYMPIC TOÁN

Chinh phục olympic toán| 19


Bồi dưỡng học sinh giỏi

HƯỚNG DẪN GIẢI

Câu 1. Tìm các đa thức P  x  có hệ số nguyên, không âm, bậc không lớn hơn 6 thỏa
P  7   102013 .
Đề chọn đội tuyển DakLak 2014
Lời giải
Ta có P  x   an x  an1 x
n n1
 a1 x1  a0
 P  7   an 7 n  an1 7 n1  a1 7  a0  P  7   an an1  a0 7 

Lại có 102013  6.7 5  3.7 3  2.7 2  6.7  2.7 0  603262 7   P  x   6x 5  3x 3  2 x 2  6x  2 .


Nhận xét. Ý tưởng các dạng bài toán này là ta đưa về việc xử lý dữ kiện đề bài dưới dạng
chuyển đổi linh hoạt các hệ cơ số để đơn giản hóa cách làm cho bài toán.

TẠP CHÍ VÀ TƯ LIỆU TOÁN HỌC


Câu 2. Cho a , b , c  thỏa mãn các đa thức f  x   ax 2  bx  c và
g  x    a  b  x 2  c  a  x  a  b có nghiệm chung. Chứng minh rằng a  b  2c 3 .
Lời giải
 
Ta có f  x   g  x    a  b  c  x  x  1 . Giả sử x 0 là nghiệm chung của 2 phương trình
2

f  x   0 và g  x   0 . Khi đó
 Nếu a  b  c  0 thì do a  b  2c  a  b  c  mod 3  nên a  b  2c 3
 Nếu a  b  c  0 , thì do x 0 là nghiệm chung của f  x  và g  x  nên x 0 là nghiệm

 
của phương trình x 2  x  1  0 . Theo định lý về phép chia với số dư, ta có
f  x   a  x 2  x  1  r  x   * 
trong đó r   x  ,deg r  2 . Trong  *  , thay x  x0 ta được
1  5
0  f  x0   a  x02  x0  1  r  x0   x0  , r  x0   0
2
Từ đó, do r   x  ,deg r  1, r  x0   0 và x0  nên r  x   0 x  suy ra
f  x   a  x 2  x  1
Và do đó b  a , c   a suy ra a  b  2c  0 3 .

Câu 3. Tồn tại hay không đa thức f  x   x 2  ax  b với a , b nguyên thỏa mãn a 2  4b  0
và nhận giá trị chính phương tại 2010 điểm phân biệt.
Lời giải
Tồn tại đa thức bậc hai có tính chất như vậy. Thật vậy:
Xét f  x   x 2  ax  b , ta có 4 f  x   4x 2  4 ax  4b   2 x  a   4b  a 2
2

20 | Tạp chí và tư liệu toán học


Đa thức và số học

 
Giả sử tồn tại x1 , x2 ,..., x2010; y1 , y2 ,..., y2010 , xi  x j , 1  i  j  2010 là các số nguyên thỏa

 4 f  x1    2 xi  a 2  4b  a 2
mãn  i  1, 2010
 4 f  x i   4 y 2
i

Suy ra 4b  a 2   2 y i  2 xi  a  2 y i  2 xi  a  i  1, 2...2010
 a 4
Chọn a , b thỏa mãn  ( Pi là các số nguyên tố phân biệt)
 4 b  a 2
 16 P1 .P2 ...P2010

 2 y i  2 xi  a  4 Pi

Xét hệ phương trình  4 P1 P2 ...P2010 , i  1.2...2010
 2 y i  2 xi  a  Pi

 P1 P2 ...P2010
 y i  Pi   
 Pi
Giải hệ, thu được  , thỏa mãn.
 x  P1 P2 ...P2010  P  a 
CHINH PHỤC OLYMPIC TOÁN

 1 Pi
i
2
Rõ ràng xi  x j  1  i  j 
Vậy tồn tại đa thức thỏa mãn yêu cầu.

Câu 4. Chứng minh rằng không tồn tại đa thức P với hệ số nguyên, khác đa thức hằng
có bậc không quá 4 thỏa mãn: tồn tại 5 số nguyên x1 , x2 ,..., x5 khác nhau sao cho
P  x1  P  x2  P  x3  P  x 4  P  x5   1.
Lời giải
Không mất tính tổng quát, ta chỉ xét ba trường hợp sau:
 Trường hợp 1. Với P  x1   P  x 2   P x 3   P x 4   P x 5    1 .
Khi đó đa thức P  x   1 có bậc không quá 4 mà có đến 5 nghiệm nguyên khác
nhau. Điều đó dẫn đến: P  x   1 (loại vì đa thức P cần tìm khác đa thức hằng).
 Trường hợp 2. Với P  x1   P  x 2   P x 3    1; P x 4   P x 5   1 . Khi đó:
2  P  x4   P  x1   P  x4   P  x2   P  x 4   P  x3 

2  P  x5   P  x1   P  x5   P  x2   P  x5   P  x3 
Ta có 2 là bội của các số x4  x1 ; x4  x2 ; x4  x3 ; x5  x1 ; x5  x2 ; x5  x3 .
Điều này là không thể.
 Trường hợp 3. Với P  x1   P  x 2   P x 3   P x 4   1; P x 5    1 .
Khi đó đa thức P  x   1 có bốn nghiệm là x1 ; x2 ; x3 ; x 4 phân biệt.
Do đó P  x   1  K .  x  x1  x  x2  x  x3  x  x 4  với K là một hằng số nguyên.
Điều này đẫn đến 2  P  x5   1  K.  x5  x1  x5  x2  x5  x 3  x 5  x 4  .
Điều này lại không thể. Như vậy không tồn tại đa thức thỏa yêu cầu bài toán.

Chinh phục olympic toán| 21


Bồi dưỡng học sinh giỏi

Câu 5. Chứng minh x n  x  1 bất khả quy trên Z  x  , n  2


Lời giải
Bổ đề. Với mọi nghiệm phức z của đa thức P  x   x n  x  1 chúng ta có bất đẳng thức
sau.
 1 1
2 Re  z    2  1
 z z

Chứng minh. Ta đặt z  r .e it , bất đẳng thức tương đương  1  2r cos t  r 2  1  0 và do nó  


là nghiệm của P  x  nên r 2 n  z
2n 2
 z  1  1  2r cos t  r 2  1  2r cos t  r 2 n  r 2


Bất đẳng thức trở thành r 2 n  r 2 r 2  1  0 (đúng)  
Giả sử P  x   f  x  g  x  trong đó deg f  1 và f , g  Z  x  , gọi các nghiệm phức của P  x 
k
 1 k 1
là z1 , z2 , .zn , sử dụng bổ đề, ta có 2   zi     2  k

TẠP CHÍ VÀ TƯ LIỆU TOÁN HỌC


i 1  zi  i 1 zi
k
Trong đó ta đã giả sử f  x     x  xi  .
i 1
k
Lại theo Viete ta có z
i 1
i  f 0  1 .
k
1
Theo bất đẳng thức AM  GM ta có z
i 1
2
 k  0.
i

k
 1
Từ đó kéo theo  Re  z  z i   0.
i 1  i 

k
 1
Mặc khác f monic và hệ số nguyên nên  Re  z  z i   1.
i 1  i 

 n  1 
Lý luận tương tự với g , cộng lại thu được Re    zi     2.
 i 1 zi  
 
n
 1
Theo Viete, ta lại có  z  z i 1
i 1  i 

Từ đây suy ra mâu thuẫn, vậy ta có P bất khả quy trên x .
Câu 6. Cho đa thức P là đa thức hệ số và tồn tại số nguyên dương m sao cho
P  1  ; P  2  ;...; P  m  không chia hết cho m . Chứng minh rằng P  a   0 với mọi a  .
Beijing 1967
Lời giải
Giả sử tồn tại a  sao cho P  a   0 .
Ta viết a dưới dạng a  qm  n với q , n là các số nguyên và 0  n  m .

22 | Tạp chí và tư liệu toán học


Đa thức và số học

 Trường hợp 1. Với n  0 . Khi đó qm  P  m   P  a   P  m   qm  m  m .


Điều này dẫn đến P  m  chia hết cho m (mâu thuẫn).
 Trường hợp 2. Với 1  n  m . Hiển nhiên a  1; 2;...; m nên q  0 .
Khi đó P  n   P  a  qm   P  a  qm .
Điều này dẫn đến P  n  m (mâu thuẫn giả thiết).
Vậy bài toán được chứng minh.

Câu 7. Cho P  x   an x n  an1 x n1  a1x  a0 là đa thức với a0 , a1 ,  , an là các số nguyên
dương. Đặt P1  x   P  x  và Pk  x   P  Pk 1  x   với k  1 . Tồn tại hay không M  0 sao
cho với m  M ta có m|PP m  m 
S.T.E.M.S Cat A/B P4
CHINH PHỤC OLYMPIC TOÁN

Lời giải – AoPS


Bổ đề. Nếu Q  x  là đa thức với hệ số thực thì khi đó:
x  y  mod m   Q  x   Q  y   mod m 
Ta sẽ chứng minh M không tồn tại. Giả sử điều ngược lại rằng M tồn tại.
Do m  0  mod m  , áp dụng bổ đề trên ta được PP m  m   PP m   0  mod m  .
Vì vậy m  M , m|PP m  0  .
Gọi q là số nguyên tố lớn hơn max{ M , a0 } .
Đặt t q là số nguyên dương nhỏ nhất sao cho q|Ptq  0  , như vậy t q tồn tại và tq  P  q  vì

các điều kiện ở trên.


Yêu cầu 1. Nếu q|Pr  0  thì tq |r
Chứng minh. Ta dùng chung ý tưởng trong việc chứng minh ordn  x |  n 
Chứng minh phản chứng. Đặt r  tq x  y với 0  y  tq , khi đó:

 
0  Pr  0   Pr tq Ptq  0   Pr tq  0   mod q 

Lặp lại các thao tác trên, ta thu được Py  0   0  mod q  , mâu thuẫn với điều tối thiểu của t q
Yêu cầu 2. tq |P  1 
Chứng minh. Từ các điều kiện đã cho và từ yêu cầu 1, tq |P  q  suy ra được q tq .
Mặt khác q|a0 – vô lý vì q  a0  0 .

 
Vậy gcd tq , q  1 dẫn đến tồn tại số tự nhiên k sao cho qk  1 mod tq  
Từ những điều kiện trên ta có q|qk|PP qk   0 

Từ yêu cầu 1 suy ra được rằng tq |P  qk  , nhưng theo bổ đề ta có P  1   P  qk  mod tq  

Chinh phục olympic toán| 23


Bồi dưỡng học sinh giỏi

Vậy tq |P  1  .
Quay trở lại bài toán
Xét các số P1  0  , P2  0  ,  PP 1  0  . Ta chọn số nguyên tố p đủ lớn sao cho p không chia
hết cho tất cả các số trên và p  max M , P  1 
Khi đó, theo cách chọn p , t p  P  1  .
Nhưng theo yêu cầu 2 thì tq |P  1  , điều này dẫn tới P  1   0 , vô lý vì tất cả các hệ số đều
dương.
Vậy M không tồn tại.

Câu 8. Cho đa thức P  x  là đa thức hệ số nguyên với bậc n  2 . Chứng minh rằng đa
thức P  P  x    x có nhiều nhất n nghiệm.

TẠP CHÍ VÀ TƯ LIỆU TOÁN HỌC


Romani TST – Gh. Eckstein
Lời giải
Ta chứng minh bằng phản chứng.
Giả sử tồn tại các số nguyên a1  a2  ...  an 1 sao cho P  P  ai    ai với mọi i .
Theo tính chất trên ta có:
      P  P  a   a
a j  ai | P a j  P  ai  | P P a j i j  ai ; 1  i  j  n  1

Điều đó kéo theo P  a   P  a   a  a . Từ đó ta được:


j i j i

n n n

  P  a   P  a   P  a   P  a   a
i 1
i 1 i n1 1 n  1  a1    ai  1  ai    P  ai  1   P  ai 
i 1 i 1

Như vậy tất cả các giá trị P  ai  1   P  ai  có cùng dấu, nên tồn tại A , B sao cho:
P  ai   ai  A; i  1, 2,..., n  1 hoặc P  ai   ai  B; i  1, 2,..., n  1 .
Tuy nhiên nhiên điều trên là không thể vì deg  P  x   x   n không thể nhận cùng một giá
trị nào đó với n  1 giá trị khác nhau của x . Bài toán được chứng minh.
Nhận xét. Với P  x  là đa thức có bậc là n  1 .

  
Chứng minh rằng phương trình P P ...  P  x    x có nhiều nhất n nghiệm.
k

IMO – 2006
Câu 9. Tìm tất cả các đa thức P hệ số nguyên sao cho P  n |2  1 với mọi số nguyên n

dương n .
Polish Olympiad
Lời giải
Dễ dàng kiểm tra được rằng nếu P là đa thức hằng thì P  1; 1 .

24 | Tạp chí và tư liệu toán học


Đa thức và số học

Ta giả sử P khác đa thức hằng. Không mất tính tổng quát, ta có thể giả sử hệ số cao nhất
của đa thức P  x  là số nguyên dương.
Xét n sao cho P  n   1 và xét p là số nguyên tố trong phân tích của P  n  .
Khi đó p P  n  2 n  1 và theo tính chất bên trên, ta được:

p    n  p   n  P  n  p   P  n   p P  n  p  2 n p  1

 
Điều đó ta có p 2 n.2 p  2 p  2 p  1   2 p  2 n  1   2 p  1  p 2 p  1

Điều trên là trái với định lý Fermat.

Câu 10. Gọi x0 , x1 ,  , xn là các số nguyên thỏa mãn x0  x1   xn . Chứng minh rằng
n!
một trong các số P  x0  , P  x1  , P  xn  không nhỏ hơn với P  x  là đa thức bậc n .
2n
Đề xuất 1977 IMO – Vietnam
CHINH PHỤC OLYMPIC TOÁN

Lời giải
Định f 0 , f 1 ,  , f n bằng x0 , x1 ,  , xn .
n
fi  x 
Theo công thức nội suy Lagrange ta có P  x    P  xi 
i 0 f i  xi 
Vì cả hai vế là đa thức bậc n và bằng nhau tại các giá trị x0 , x1 ,  , xn .
n
P  xi 
So sánh các hệ số của x n ta được 1  
i 0 f i  xi 
Do x0 , x1 ,  , xn là các số nguyên, giảm nghiêm ngặt, ta có
i 1 n
1 n
f i  xi    x j  xi x j  xi  i !  n  i  !   
j 0 j i  1 n!  i 
Đặt giá trị lớn nhất của P  x0  , P  x1  , P  xn  là P  x k  .
n P  xi  P  xk  n
 n  2 P  xk 
n

Theo bất đẳng thức tam giác ta được 1      


i  0 || f i  xi | n! i 0  i  n!
n!
Vậy P  xk   .
2n

Câu 11. Giả sử các đa thức P  x  , Q  x  , R  x  và S  x  thỏa mãn:


P  x 5   xQ  x 5   x 2 R  x 5    x 4  x 3  x 2  x  1  S  x 
Chứng minh rằng khi đó đa thức P  x  chia hết cho x  1 .
USAMO – 1976
Lời giải
Giả sử S  x   sn x  sn1 x
n n1
 ...  s1 x  s0 . Khi đó:

Chinh phục olympic toán| 25


Bồi dưỡng học sinh giỏi

 x  1 P  x 5   x  x  1 Q  x 5   x 2  x  1 R  x 5    x 5  1 S  x 
  x  1  P  x 5   x  x  1  Q  x 5   x 2  x  1  R  x 5    x 5  1  S1  x   S2  x  

n
với S1  x   s0  s5 x 5  ...  s5m x 5m ; m    và S2  x   S  x   S1  x  . Khi đó ta được:
5
xP  x 5   x  x  1  Q  x 5   x 2  x  1  R  x 5    x 5  1  S2  x    x 5  1  S1  x   P  x 5 
Vì vế phải của đẳng thức trên là đa thức mũ bội 5 trong khi vế trái thì không.
Điều này xảy ra khi và chỉ khi cả hai vế là hai đa thức có tất cả các hệ số là 0 .
   
Điều đó dẫn đến P x 5   x 5  1 S1  x   P  1   0 . Do đó P  x    x  1  .G  x  với G  x  là
một đa thức nào đó.

Câu 12. Tìm tất cả các đa thức P có hệ số nguyên sao cho với mọi số nguyên tố p và
mọi cặp số tự nhiên  u , v  thỏa mãn P| uv  1  thì ta có P  P  u  P  v   1  .

TẠP CHÍ VÀ TƯ LIỆU TOÁN HỌC


Iran TST 2009
Lời giải
Ta xét các trường hợp sau.
 Trường hợp 1. Với P là đa thức hằng. Dễ dàng kiểm tra được P  x   1; P  x   1 .
1
 Trường hợp 2. Gọi n  deg P  1 . Ta đặt G  x   x n P   thì G  x  cũng là đa thức hệ
x
số nguyên.
 
Từ giả thiết bài toán, ta có được p P  u  P  u 1   1 với mọi số nguyên dương u sao

cho  u , p   1 và trong đó u1 là chỉ số nguyên dương v sao cho uv  1  mod p  .

 
Từ đó ta được p un P  u  P  u1   un hay nói cách khác p P  u  G  u   un với mọi số

nguyên dương u sao cho  u , p   1 .


Ta cố định u và lấy p đủ lớn, thì tính chất trên chỉ đúng khi và chỉ khi
P  u  G  u   un . Hơn nữa G  x  P  x  là các đa thức nên điều đó kéo theo
G  x  H  x   x n . Mà deg P  n nên G là đa thức hằng. Tức là P  x   a x n , bằng cách
thử lại ta được a  1; a  1 .
Vậy ta có bốn hàm đa thức thỏa mãn P  x   1; P  x   1; P  x   x n ; P  x   x n .

Câu 13. Tìm tất cả các đa thức P hệ số nguyên sao cho với mọi số tự nhiên a , b , c ta luôn
có a  b  c P  a   P  b   P  c  .
Lời giải
Giả sử P  x   an x  an1 x
n n 1
 ...  a1 x  a0 với ai  .

26 | Tạp chí và tư liệu toán học


Đa thức và số học

Với ba số tự nhiên a , b , c  1 tùy ý, ta có 2 P  0   P  c   3a0  mod c 


Tức c 3 a0 , mà c là số tự nhiên từ ý nên điều trên đúng khi và chỉ khi a0  0 . Từ đó bằng
cách cho c  0 ta được tính chất a  b|P  a   P  b  đúng với mọi a , b  .
Dễ dàng chứng minh được tất cả các hệ số của đơn thức bậc chẵn đều bằng 0 . Do đó:
 a  b  c | P  c   P  a  b 
Kết hợp với giả thiết của bài toán, ta được P  a   P  b   P  a  b   0  mod a  b  c 
Mà P là đa thức và a , b , c là các số tự nhiên tùy ý nên P  a   P  b   P  a  b  .
Tức P cộng tính, một lần nữa vì P là đa thức nên ta được P  x   k.x . Thử lại ta được kết
luận cho bài toán.

Câu 14. Cho m , n là các số nguyên dương. Chứng minh rằng n m khi và chỉ khi
P  x  Q  x  , trong đó P  x  ; Q  x  là các đa thức hệ số nguyên được xác định:
CHINH PHỤC OLYMPIC TOÁN

P  x   x n1  Cn1 x n2  ...  Cnn1


Q  x   x m1  Cm1 x m2  ...  Cmm1
Lời giải
Với các định nghĩa của hai đa thức trên, ta được:
xP  x    x  1   1; xQ  x    x  1   1
n m

Do đó P  x  Q  x    x  1  1  x  1   1 

n
 
m
   x  1 n m

 1,  x  1   1   x  1   1
m

 m ,n 
  x  1  1   x  1   1   m, n   m  n m
m

Bài toán được chứng minh.


Câu 15. Tìm tất cả các đa thức P hệ số nguyên thỏa mãn tính chất: Với m , n là hai số
nguyên tố cùng nhau thì hai số P  m  ; P  n  cũng là hai số nguyên tố cùng nhau.
Iran TST
Lời giải
Dễ thấy P không thể là đa thức hằng.
Giả sử với p là số nguyên tố đủ lớn sao cho p không là ước của P  p  .
Khi đó  p , p  P  p    1 do đó P  p  và P  p  P  p   là hai số nguyên tố cùng nhau.

Tuy nhiên ta thấy P  p    P  p   p  p | P  p  P  p    P  p    P  p |P  p  P  p  

Như vậy, để đúng với giả sử bên trên ta cần P  p   1 hoặc P  p   0 .


Điều này là không thể vì P là đa thức khác đa thức hằng và p là số nguyên tố đủ lớn.
Tóm lại, với p là số nguyên tố đủ lớn thì p|P  p  , theo tính chất bên trên, ta lại có:
p  p  0|P  p   P  0   p|P  0 

Chinh phục olympic toán| 27


Bồi dưỡng học sinh giỏi

Mặt khác p là số nguyên tố đủ lớn tùy ý nên P  0   0 .


Từ đó ta được P  x   x.G  x  với G  x  là đa thức hệ số nguyên thỏa mãn tính chất giống
như P  x  nhưng có bậc nhỏ hơn.
Thực hiện tương tự như vậy kết hợp với phương pháp quy nạp theo bậc của đa thức ta
được: P  x    x n với n  *
.

Câu 16. Cho P  x  , Q  x    x  là hai đa thức monic bất khả quy thỏa mãn với n đủ lớn
thì P  n  , Q  n  có cùng tập ước nguyên tố. Chứng minh rằng P  Q .
Lời giải
Ta chứng minh bằng phản chứng bằng cách giả sử P  Q . Khi đó P,Q là hai đa thức
nguyên tố cùng nhau do chúng đều bất khả quy và là đa thức monic.
Theo định lí Bézout thì tồn tại hai đa thức f , g có hệ số nguyên và số nguyên dương a sao

TẠP CHÍ VÀ TƯ LIỆU TOÁN HỌC


cho P  x  . f  x   Q  x  .g  x   a . Lại theo định lí Schur thì tồn tại vô hạn số nguyên tố p sao
cho ứng với mỗi số p đang xét thì ta có tồn tại số nguyên n sao cho p P  n  . Nhưng vì
P  n  , Q  n  có cùng tập ước nguyên tố nên ta có p a . Như vậy số a có chứa vô hạn các
ước nguyên tố của nó là điều vô lý. Vậy Q  P .

Câu 17. Gọi đa thức P  x   x khác đa thức hằng và gọi n , k là hai số nguyên dương.
Chứng minh rằng tồn tại số nguyên dương a sao cho mỗi f  a  , f  a  1  ,..., f  a  n  1 
có ít nhất k ước nguyên tố phân biệt.
Bulgaria Olympiad
Lời giải
Bài này gợi ta nghĩ đến định lí thặng dư Trung hoa.
Theo định lí Schur tồn tại vô hạn số nguyên tố p sao cho ứng với mỗi p sẽ tồn tại số
nguyên m sao cho p P  m  . Ta gọi A là tập số nguyên tố đó thì A   .
Gọi pij  1  i  n,1  j  k  là các số nguyên tố thuộc tập A.

Ta cố định mỗi i . Theo định lí Schur thì tồn tại số xij     


sao cho P xij  0 mod pij .

Theo định lí thặng dư Trung Hoa thì cứ mỗi i cố định sẽ tồn tại số a sao cho
 
a  i  1  xij mod pij với j  1; k. Khi đó với mỗi i cố định thì ta đều có

   
f  a  i  1   f xij  0 mod pij với j  1, k.

Nên f  a  i  1  chia hết cho tất cả các số nguyên tố pij  j  1, k  với mọi số i  1; n.

28 | Tạp chí và tư liệu toán học


Đa thức và số học

Câu 18. Cho đa thức P  x  , Q  x  là hai đa thức có hệ số nguyên nguyên tố cùng nhau.
Đặt an   P  n  , Q  n   . Chứng minh dãy an  tuần hoàn.
Lời giải
Theo đính lý Bézuot tồn tại hai đa thức R  x  , S  x  và hằng số a nguyên thỏa mãn
P  x  .R  x   Q  x  .S  x   a , x  . Suy ra P  n  .R  n   Q  n  .S  n   a , n  .
Từ giả thiết an   P  n  , Q  n   , ta suy ra an a , n . Do tập các ước là hữu hạn nên các số an
lặp lại. Ta cần chứng minh là an  a  an .
Ta có P  an  a   P  an  mod a  . Mà an P  n  , an a  an P  n  a  .
Hoàn toàn tương tự ta có an Q  n  a   an an  a .
Mặt khác ta lại có P  an  a   P  an  mod a  . Mà an a P  n  a  , an a a  an a P  n  .
Lập luận tương tự ta có an a Q  n  . Do đó an  a an . Từ đó ta có an  a  an .
CHINH PHỤC OLYMPIC TOÁN

Câu 19. Với hai đa thức có hệ số nguyên p  x  , q  x  ta viết p  x   q  x  mod 2  nếu


p  x   q  x  có tất cả các hệ số đều chia hết cho 2. Cho dãy đa thức pn  x  thỏa mãn
p1  x   p2  x   1 và pn 2  x   pn 1  x   xpn  x  1  , với mọi n là số tự nhiên.
Chứng minh rằng p2n  x   1  mod 2  .
Lời giải
Ta quy ước viết pn  x   pn , n  1, 2, 3
Trước hết ta xét pn 4  pn 3  xpn 2   pn 2  xpn1   xpn 2   x  1  pn 2  xpn1
  x  1  pn 2  x  pn 2  xpn    2 x  1  pn 2  x 2 pn  pn 2  x 2 pn  mod 2   * 
Phần tiếp theo ta chứng minh p2 n  pn2  mod 2  với mọi n .
Thật vậy với n  1 ta có p2  p1  1 nên khẳng định đúng.
Giả sử khẳng định đúng với n  k  1 .
Khi đó kết hợp với  *  ta có p2 k  2  p2 k  x 2 p2 k 2  mod 2 

 pk2  x 2 pk21  mod 2   pk2  x 2 pk21  2 pk xpk 1  mod 2    pk  xpk   pk21  mod 2 
2

Như vậy p2n  p222n1  p22n1  p24n2  p12  1  mod 2  .


n

Câu 20. Giả sử p2n  x   1 là hai số tự nhiên lớn hơn 1 và pn  x   pn .


Dãy số vô hạn n  1, 2, 3... được xác định như sau
pn 4  pn 3  xpn 2   pn 2  xpn 1   xpn 2   x  1  pn 2  xpn 1
Chứng minh rằng trong dãy số nói trên chứa vô hạn số đôi một nguyên tố cùng nhau.
Lời giải

Chinh phục olympic toán| 29


Bồi dưỡng học sinh giỏi

Để ý là a  1 nên  a , a  1   d  1.
Giả sử từ dãy  x  1  pn 2  x  pn  2  xpn  ta trích ra dãy con hữu hạn
 2 x  1 pn 2  x 2 pn  pn 2  x 2 pn  mod2 
Mà hai số bất kì trong dãy này thì nguyên tố cùng nhau, do dãy ban đầu vô hạn nên luôn
trích được như vậy.
Đặt p2n  pn2 . Xét n số sau n  1 . Khi đó luôn tồn tại hai số đồng dư modulo q.
Tức là tồn tại hai số n  k  1 sao cho p2 k  2  p2 k  x 2 p2 k  2 . Do đó  pk2  x 2 pk21 .
Do  pk2  x 2 pk21  2 pk xpk 1 nên ta suy ra   pk  xpk   pk2 1 , nên ta cũng suy ra
2

 ax , q   1  3 
t

Từ  1  ,  3  ta có x s t  1  mod q   x s t  lq  1, l  .
Xét số uik1  a  lq  1   a  1  alq  1  4  .

TẠP CHÍ VÀ TƯ LIỆU TOÁN HỌC


 
k
Vì uik1  alq  1  al. ui j  1 nên uik1 , ui j  1, j  1, 2,..., k .
j 1

Hệ thức  5  chứng tỏ ta có thể bổ sung vào dãy ui1 , ui2 ,..., uik các bộ số mới mà bộ số này
vẫn thỏa mãn hai số bất kì nguyên tố cùng nhau. Chứng tỏ có vô hạn số như vậy.

Câu 21. Với mỗi số tự nhiên n , ta kí hiệu f  n  là tổng các chữ số của nó biểu diễn trong
hệ thập phân. Ta xây dựng dãy số như sau

u1  n   f  n  ; u2  n   f  f  n   ;...; uk  n   f f ... f  n  ... 
k

Chứng minh rằng với mọi số tự nhiên n , tồn tại số tự nhiên d sao cho
uk  n   ud  n  , k  d  1  .
Lời giải
a) Giả sử rằng n  a1 a2 ...an là số được biểu diễn trong hệ thập phân với a1  0 .
Khi đó ta có biểu diễn n  a1 .10 p 1  a2 .10 p  2  ...  ap 1 .10  ap .
Ta luôn có bất đẳng thức n  a1  a2  ...  an  f  n  .
Dấu đẳng thức xảy ra khi và chỉ khi số có 1 chữ số.
Ta thấy rằng dãy un  là dãy giảm thực sự cho đến khi ta thu được số có 1 chữ số thì dãy
trở thành dãy dừng bởi vì f  n   f  f  n    ...  f  f ... f  n   . Gọi d là số có một chữ số ấy.
Khi đó ta có uk  n   ud  n  , k  d  1  .
b) Kí hiệu Sk  x : f  x   k là tập hợp các số x mà trong biểu diễn thập phân thì tập
Sk   vì chẳng hạn số 11...1 thuộc S k .Với mỗi số tự nhiên k thì tập S k là hữu hạn nên
k  so 1

trong mỗi tập S k đều tồn tại số bé nhất.

30 | Tạp chí và tư liệu toán học


Đa thức và số học

Câu 22. Cho abc là một số nguyên tố.


Chứng minh rằng phương trình ax 2  bx  c  0  1  không có nghiệm hữu tỉ.
Lời giải
Do abc là nguyên tố nên a  0, c  0 . Ta chứng minh bằng phản chứng.
Giả sử phương trình có nghiệm hữu tỉ nên   b 2  4 ac  d 2 , d  .
Do đó b 2  d 2  4 ac  0  b  d .
Mặt khác ta có 4 a.abc  4 a  100 a  10b  c    20 a  b    b 2  4 ac    20 a  b   d 2
2 2

  20 a  b  d  20 a  b  d  .
Vì abc là số nguyên tố nên suy ra abc là ước của  20a  b  d  hoặc  20a  b  d  .
Tuy nhiên ta có  20 a  b  d   100 a  b  d  100 a  10b  d  abc .
Tương tự  20 a  b  d   100a  b  d  100a  10b  d  abc .
CHINH PHỤC OLYMPIC TOÁN

Như vậy ta suy ra điều vô lí.


Vậy phương trình không thể có nghiệm hữu tỉ.

   
Câu 23. Cho đa thức P  x   x 2  2 x 2  3 x 2  6 . Chứng minh rằng với mọi số nguyên
tố p đều tìm được số nguyên dương n sao cho P  n  p .
Lời giải
Rõ ràng với p  2 thì ta tìm được n  2 thỏa mãn bài toán.
Với p  3 ta tìm được n  3 cũng thỏa mãn.
Bây giờ ta xét các số nguyên tố p  3 .
p 1
Trước hết ta có nhận xét là nếu  a , p   1 và n2  a  mod p  , n. thì a 2
 1  mod p  .
Chứng minh.
Với mỗi k  1, 2,..., p  1 thì tồn tại duy nhất một số k '  1, 2,..., p  1 sao cho
k.k '  a  mod p  .
p1
Vì n2  a  mod p  , n nên k  k ' . Ta để ý là p  1 là số chẵn nên ta sẽ có được cặp
2
tương ứng giữa k với k ' .
p 1
Từ đó ta suy ra  p  1 !   1.1' 2.2' ...  k.k '  a 2
 mod p  . Ở đây ta hiểu 1 ' là số tương
ứng với 1 để 1.1'  a  mod p  .
p 1
Mặt khác theo định lí Wilson ta có  p  1  !  1  mod p  . Vậy nên ta có a 2
 1  mod p  .
Nhận xét được chứng minh.
Trở lại với bài toán. Giả sử P  n  không chia hết cho p với mọi n .

Chinh phục olympic toán| 31


Bồi dưỡng học sinh giỏi

   
Vì P  n   n2  2 n2  3 n2  6 nên n 2 không chia hết cho 2; 3; 6 .
p 1 p 1 p 1
Theo nhận xét ở trên ta suy ra 2 2
 1  mod p  ; 3 2
 1  mod p  ; 6 2
 1  mod p  .
p 1 p 1 p 1
Nhưng vì 2 2
 1  mod p  ; 3 2
 1  mod p   6 2
 1  mod p  là điều mâu thuẫn.
Chứng tỏ rằng giả thiết phản chứng là sai.

Câu 24. Chứng minh rằng với mọi số nguyên p ta có thể tìm được số nguyên dương
f  x  sao cho n không phải là số chính phương.
Lời giải
Đặt f  n  . Nếu p  1 là số chính phương thì p  1 hoặc f  x   b , b  A .
n

Giả sử tồn tại bộ số nguyên n  N * , pn  1    p  1 b sao cho với mọi số nguyên dương

TẠP CHÍ VÀ TƯ LIỆU TOÁN HỌC


f  x  thì n  N * đều là số chính phương. Khi đó q đều là các số chính phương. Ta có
f n ; f n  q   f n n  q  n  q  f n q  1
Mặt khác do tính chất của số chính phương nên ta suy ra f  n  q  q .
Kết hợp với  1  ta suy ra f  2   f  4   0  mod 4  .


Suy ra p  q , pq  1 q  2 .
Bây giờ ta lại xét pq  p  mod q  .
Lập luận như trên ta suy ra pq  1  pq  p  p  1 q  3  .  
Kết hợp  2  ,  3  ta suy ra n  *
là điều vô lí.
Chứng tỏ giả thiết phản chứng là sai.

Câu 25. Cho đa thức P  x   x 2017  x 1000  1 . Tồn tại hay không các số tự nhiên a1 , a2 ,...

 
, a2018 sao cho tích P  ai  .P a j ai a j với mọi i  j .

Lời giải
Giả sử tồn tại các số a1 , a2 ,..., a2018 sao cho với mọi i  j ta có P  ai  .P a j ai a j .  
Dễ thấy rằng  ai , P  ai    1 . Từ đó suy ra P  ai  a j và với mọi i  j .

  
Ta suy ra ai2017  ai1000  1 a j , từ đó suy ra ai , a j  1 với mọi i  j. 
Không mất tính tổng quát ta có thể giả sử rằng a1  a2  ...  a2018 .
Từ P  ai  a j ta suy ra rằng P  a1  a2 .a3 ...a2018  a12017 .
Nhưng từ P  x  ta suy ra P  a1   a12017 là điều mâu thuẫn.

32 | Tạp chí và tư liệu toán học


Đa thức và số học

Câu 26. Với mọi số tự nhiên m , n , chứng minh rằng  n ! chia hết cho m khi và chỉ khi
n
tồn tại đa thức hệ số nguyên f  x    ak x k thỏa mãn  a0 , a1 ,..., an , m   1, m f  j  với
k 0

mọi j nguyên dương.


Lời giải
Ta chứng minh tính hai chiều
Giả sử ta có m  n ! . Ta xét đa thức sau f  x    x  1  x  2  ...  x  n  thì f là đa thức với hệ
số nguyên. Ta cũng có f  j   n !C nn j . Do m  n ! nên ta có f  j  m . Mặt khác do hệ số bậc
cao nhất của f là 1 nên ta có điều kiện  a0 , a1 ,..., an , m   1 được thỏa mãn. Như vậy tồn tại
đa thức f  x    x  1  x  2  ...  x  n  .
n
Ngược lại giả sử ta có tồn tại đa thức f  x    ak x k hệ số nguyên thỏa mãn điều kiện là
k 0
CHINH PHỤC OLYMPIC TOÁN

 a0 , a1 ,..., an , m   1, m f  j  .
Ta xét dạng của f là f  x   b0  b1 x  b2 x  x  1   ...  bn x  x  1  x  2  ...  x  n  1 
n
 b0   bk x  x  1 ...  x  k  1  .
k 1

Với mỗi đa thức f hoàn toàn xác định thì các số b k xác định duy nhất với mọi số k  0, n .
Tiếp theo ta có nhận xét là  b0 , b1 ,..., bn    a0 , a1 ,..., an  .
Thật vậy. Ta giả p là một ước của  b0 , b1 ,..., bn  , thì khi đó p là ước của f  j  với mọi số
nguyên j . Do đó khi và chỉ khi p là một ước của  a0 , a1 ,..., an  . Do đó ta có
 b0 , b1 ,..., bn    a0 , a1 ,..., an  .
Do m f  j  nên f  j   0  mod m  với mọi j  0, 1, 2,..., n . Nên đặc biệt ta có
b0  f  0   0  mod m  ; b0  b1  f  1   0  mod m   b1  0  mod m  .
Tương tự f  2   b0  2b1  2b2  0  mod m   2b2  0  mod m  .
Bằng cách tương tự ta có khẳng định là j !.b j  0  mod m  .
Do  b0 , b1 ,..., bn , m    a0 , a1 ,..., an , m   1 nên tồn tại các số nguyên c 0 , c 1 ,..., c n , D sao cho
c0 b0  c1b1  ...  c nbn  Dm  1  n !  c 0 b0  c 1b1  ...  c nbn   n ! Dm  n !  1  .
Nhưng do j !.b j  0  mod m  nên từ  1  ta suy ra n ! m (ở bên vế phải).

Câu 27. Cho đa thức f  x   2009x 5  x 4  x 3  x 2  2006x  1. Chứng minh rằng với n là
số nguyên tùy ý thì các số f  n  , f  f  n   ,..., f ... f  n  ...  đôi một nguyên tố cùng nhau.
Lời giải
Nhận xét. Nếu a  b  mod q  thì h  a   h  b   mod q  .

Chinh phục olympic toán| 33


Bồi dưỡng học sinh giỏi

Đặt f k  x   f  f ... f  x  ... với f có mặt k lần với k  1.


Gọi p k là một ước nguyên tố tùy ý của  Pn  x   thì ta có f k  n   0  mod pk 
Ta có  Pn  x   ; f k  2  n   f  f k  1  n    f  1   1  mod p 
Bằng quy nạp ta chứng minh được f m  n   1  mod p  với mọi  Pn  x   , tức là f k  n  và
f m  n  nguyên tố cùng nhau với mọi số m  k. Vì nếu không như thế thì  f k  n  , f m  n    d.
Gọi p là một ước nguyên tố của d thì p là ước chung của cả f k  n  , f m  n  là mâu thuẫn
với chứng minh nêu trên. Ta hiểu là là khi mà k thay đổi thì số nguyên tố p được điều chỉnh
2015
thay đổi theo chỉ số k . Do k là số tùy nên ta có P  x     x  xi   1 đôi một nguyên tố cùng
i 1

nhau.

1

TẠP CHÍ VÀ TƯ LIỆU TOÁN HỌC


 n  1
Câu 28. Gọi P  x  là đa thức bậc n thỏa mãn với k  0, 1,  , n thì P  k    
 k 
Định đa thức P  n  1  .
Đề xuất 1981 IMO – Romanian
Lời giải
k ! n  1  k !
1
 n  1
Với k  0, 1, , n , đặt wk  k và c k    
 k   n  1 !
Định các giá trị cho f 0 , f 1 ,  , f n , ta được f k  k    1 
n k
k !  n  k  ! và f k  n  1 
 n  1 !
(n  1  k )
n
f k  n  1 n
Theo công thức nội suy Lagrange ta có P  n  1   c k    1
n k

k 0 fk  k  k 0

Bằng 0 nếu n lẻ và bằng 1 nếu n chẵn.


Nhận xét. Trong một số bài toán về đa thức, ta đôi lúc sẽ bắt gặp một số công thức như nội
suy Lagrange, khai triển Taylor,... việc hiểu và vận dụng linh hoạt các công thức trên sẽ
giúp bài toán trở nên gọn gàng và ít rườm rà hơn.

u  1990, u2  1989, u3  2000


Câu 29. Cho dãy un  được xác định như sau  1 .
un 3  19un 2  9un 1  un  1991, n  1, 2...
a) Với mọi n gọi rn là số dư của phép chia un cho 1992. Chứng minh rằng dãy rn  là
một dãy tuần hoàn.
b) Chứng minh rằng tồn tại vô số số x của dãy un  sao cho
5x 1992  5x 1994  4x 1975  8x 1945  2 x 1990  11x 2  48
chia hết cho 1992.
Lời giải

34 | Tạp chí và tư liệu toán học


Đa thức và số học

a) Đây là dãy truy hồi tuyến tính cấp ba. Gọi rn là số dư khi chia un cho 1992. Ta thấy chỉ
3
có tối đa 1992 số dư khác nhau. Số bộ ba các số dư có kể thứ tự khác nhau có thể là A1992 .
Xét các bộ ba số dư của phép chia un cho 1992 là ( vì ta cần truy hồi tới ba số hạng phía
trước nên ta xét các bộ ba số  r1 , r2 , r3  ;  r2 , r3 , r4  ; , , , ;  ri , ri  1 , ri  2  ;  ri  1 , ri  2 , ri  3  ; , , , số các bộ
số lập theo cách trên là vô hạn. Tuy nhiên chỉ có hữu hạn số dư ( tối đa là 1992 số) trong
3
phép chia un cho 1992 nên chỉ có hữu hạn các bộ ba khác nhau. Tối đa là A1992 bộ. Do đó
luôn tồn tại các số nguyên dương m, s sao cho
rm  rm s
 rm , rm1 , rm 2    rms , rms1 , rms 2  , tức là rm1  rms1  1 .
r  r
 m 2 ms 2
Như vậy có nghĩa là tồn tại k  m , m  1, m  2 sao cho ta có rk  rk  s  2  , k  1
Ta sẽ chứng minh bằng quy nạp rằng rk  rk  s  2  , k  1 .
CHINH PHỤC OLYMPIC TOÁN

Ta đã có rk  rk  s với k  m , k  m  1, k  m  2.
Giả sử ta có  2  đúng với m  k  p trong đó p  m  2 . Ta xét k  p  1 .

    
Ta có up  1 s  up  1  19 up s  up  9 up s 1  up 1  up s  2  up  2 
 19  r ps  r   9 r
p ps1  rp 1   r
ps2 
 rp 2  0  mod 1992  .

Như vậy ta có rp  s  1  rp  1 .
Vậy ta có rk  rk  s , k  m
Cuối cùng ta phải chứng minh khẳng định rk  rk  s , k  1 .
Nếu m  1 thì ta có r1  r1 s .
Nếu m  1 , ta phải chứng minh rk  rk  s , 1  k  m  1 .
Từ cách xác định dãy ta truy ngược có un 3  19un 2  9un 1  1991  un .
Cho nên ta có um1 s  um1   um 2  s  um 2   19  um 1 s  um 1   9  um s  um 
 rm 2 s  rm 2   19  rm1s  rm1   9  rms  rm   0 .
Cứ thế sau m  1 bước lùi chỉ số ta thu được rk  rk s , 1  k  m  1 .
Kết hợp lại ta có rk  rk  s  2  , k  1 .
Như vậy dãy rn  là dãy tuần hoàn với chu kì s  1 ( tại vì r1  2  r2  3 )
b) Đặt P  x   5x 1992  5x 1994  4 x 1975  8x 1945  2 x 1990  11x 2  48 .
Ta thấy dãy un  là dãy tăng khi n  2 .
Do đó ta cũng suy ra với chu kì s  1 ở trên thì ta có ( chu kỳ s  1 vì r2  r1 ).
us  u2 s  ...  uks  u k  1s  ...
Đây là dãy tăng thực sự nên có vô hạn phần tử.

Chinh phục olympic toán| 35


Bồi dưỡng học sinh giỏi

Ta chỉ cần chứng minh P  uks  1992, k  1 .


Ta có từ cách xác định dãy ta suy ra p  x  q  x  .
Do đó p  x   q  x  ( do p  x   q  x  ) mà pn  x  , lại do p1  x   p2  x   1 nên suy ra
pn  2  x   pn  1  x   xpn  x  .
Vậy ta có điều phải chứng minh!

Câu 30. Chứng minh rằng tồn tại hằng số dương c sao cho với mọi số nguyên dương n
và các số thực a1 , a2 ,  , an , nếu P  x    x  a1  x  a2   x  an  thì
max P  x   c n max P  x 
x0,2  x0,1

Lời giải – Lee Kai Seng


i
Đặt S là giá trị lớn nhất của |P  x |x   0, 1 . Với i  0, 1, 2, , n , đặt bi  và

TẠP CHÍ VÀ TƯ LIỆU TOÁN HỌC


n
f i  x    x  b0   x  bi 1  x  bi 1   x  bn 
n
fi  x 
Bằng công thức nội suy Lagrange, với mọi số thực x ta có P  x    P  bi 
i 0 f i  bi 
Với các giá trị w  [0, 2], w  bk  2  bk k  0, 1, 2,  , n nên
n
 i  2n  2n  1  2n  2   n  1    2n  !
fi  w  fi  2     2   
i 0  n nn n ! nn
i ! n  i  !
Lại có P  bi   S và f i  bi   .
nn
n fi  w n
 2n  2n  i 
Theo bất đẳng thức tam giác ta có P  w    P  bi   S   
i 0 f i  bi  i  0  i  n 
n
 2n  2n  i  n  2n  2n  2n 
2n 
Suy ra  
i  0  i 
       2    2
n  i 0  i  n 
4n

n
Vậy max P  w   2 4 n S  16n max P  x  .
w0 ,2  x 0 ,1

Câu 31. Tìm tất cả các đa thức P  x   x và m  


sao cho m  2 n P  n  là số chính
phương với mọi số nguyên dương n .
Nguyễn Song Minh
Lời giải
Lời giải sau đây của thầy Nguyễn Song Minh – Admin Mathscope.
Giả sử P  x  và m là là đa thức và số nguyên dương thỏa mãn, ta có 2 nhận xét sau:
Nhận xét 1. Nếu p là ước nguyên tố lẻ của m  2 n P  n  thì p|P   n 

 
Chứng minh. Ta có vp m  2 n P  n   2 , theo bổ đề Fermat bé thì:

36 | Tạp chí và tư liệu toán học


Đa thức và số học

P  n  p  p  1    mod p 
n  p p  1
m  2n P  n   m  2

Vì thế ta lại có vp m  2  n  p p  1

P  n  p  p  1    2 , theo bổ đề tiếp tuyến và định lý Euler ta

P  n  p  p  1    m  2 n P  n   2 n p  p  1  P '  n   mod p 2  .
n  p p  1
có 0  m  2
Từ đây có p P '  n  .
Nhận xét 2. Nếu p là ước nguyên tố lẻ của m  2 K P  n  với K  
thì p|P   n  .
Chứng minh. Theo định lý thặng dư Trung Hoa, sẽ tồn tại số nguyên dương N sao cho
N  K  mod p  1  và N  n  mod p  , từ đây có p| m  2 N P  N  .  
Do đó theo nhận xét 1 thì p|P '  N  nhưng P '  x   x nên từ N  n  mod p  ta có:
p P 'n

Quay lại bài toán, với mỗi n  đủ lớn, sẽ có vố số số nguyên tố p có dạng 8 k  3 đồng
2
thời p  max m, P  n  . Lúc này ta để ý rằng    1 nên 2 là căn nguyên thủy modulo p
CHINH PHỤC OLYMPIC TOÁN

 p
 
khi đó sẽ tồn tại K sao cho p m  2 K P  n  (ta chọn K  ind 2  mP  với P là nghịch đảo
của P  n  theo mod p , theo nhận xét 2 thì p P '  n  .
Do đó có vô số số nguyên tố p như thế, và giá trị của chúng có thể lớn tùy ý nên P '  n   0
Từ đây ta thấy rằng phương trình P '  x   0 có vô số nghiệm, nên P  x   C (với C là hằng
số), từ đây ta dễ dàng chứng minh được P  x   0 còn m là số chính phương.
Nhận xét. Bài toán trên yêu cầu ta phải vận dụng linh hoạt các tính chất số học của đa
thức, ngoài ra có một số bổ đề nhỏ ta cần phải quan tâm như bổ đề tiếp tuyến được phát
biểu và chứng minh ngắn gọn như sau.

Câu 32. Cho P  x    x  , p là số nguyên tố và x  a  mod p  . Chứng minh rằng


P  x   P  a    x  a  P '  a   mod p 2 
Bổ đề tiếp tuyến
Lời giải
Với mọi đa thức P  x  và a  b nguyên thì a  b P  a   P  b 
P
k
 x0  
Nếu P  x    x  và k  
x0  , do tích của k số nguyên liên tiếp chia
thì
k!
hết cho k ! , mà đạo hàm cấp k thì chứa các tích đó.
Khai triển Taylor của đa thức tại x  x0 :
P   x0  P  x0  P  x0 
n

P  x   P  x0    x  x0    x  x0   x  x0 
2 n
 
1! 2! n!
Từ những điều trên, xét x  a  pt với t  thì dễ dàng có:

Chinh phục olympic toán| 37


Bồi dưỡng học sinh giỏi

P  x   P  x0   P '  x0  pt  P ''  x0   pt    P  x0   P '  x0  pt  mod p 2 


2

p2

Nhận xét. Bổ đề trên là cơ sở cho bổ đề Hensel (Hensel lifting lemma) dùng để chứng minh
tồn tại hoặc đếm số nghiệm của phương trình đồng dư với modulo là lũy thừa của số
nguyên tố.

Câu 33. Với p là số nguyên tố, đặt h  x  là đa thức có hệ số nguyên sao cho
h  0  , h  1  ,  , h  p 2  1  là một hệ thặng dư đầy đủ modulo p 2 . Chứng minh rằng

h  0  , h  1  ,  , h  p 3  1  là một hệ thặng dư đầy đủ modulo p 3 .


Putnam 2008 B4
Lời giải
Dưới đây xin giới thiệu với bạn đọc lời giải chính thức của cuộc thi.

TẠP CHÍ VÀ TƯ LIỆU TOÁN HỌC


deg  h 
h   x 
i

Ta xác định đa thức với khai triển Taylor h  x  y   i 0 i!


yi

Trong khai triển này,


h i
x là đa thức theo biến x với hệ số nguyên.
i!
Với x  0,  , p  1 , ta suy ra được h  x  p   h  x   ph '  x  mod p 2  
Điều này có thể suy ra được trực tiếp bằng cách sử dụng khai triển Nhị thức Newton
Vì thế ta giả sử h  x  và h  x  p  phân biệt với modulo p 2 , ta kết luận được
h '  x   0  mod p  . Do h ' là đa thức với hệ số nguyên, ta có h '  x   h '  x  mp  mod p  với
mọi số nguyên m , vì thế h '  x   0  mod p  với mọi số nguyên x .
Với x  0, , p 2  1 và y  0,  , p  1 , ta viết được đa thức h dưới dạng:
h  x  yp 2   h  x   p 2 yh '  x   mod p 3 

   
Do đó h  x  , h x  p 2 , , h x   p  1  p 2 chạy qua tất cả các lớp giá trị dư của modulo p 3 ,

đồng dạng với h  x  theo modulo p 2 .


Vì đa thức h  x  đã chạy qua các lớp giá trị dưtheo modulo p 2 , chứng minh này đã dẫn tới
h  0  , h  1  , , h  p 3  1  phân biệt với modulo p 3 .
Cách 2.
Trước hết ta chứng minh P '  x  không chia hết cho p với mọi x nguyên.
Thật vậy giả sử tồn tại x mà P '  x   0  mod p  , khi đó ta chọn y sao 0  y  p và
y  x  mod p  . Theo cách chứng minh định lí Hensel ta có
P '  y   P '  x   mod p   P  x  p   P  y   p.P '  y  mod p   P  y   mod p 2  .

38 | Tạp chí và tư liệu toán học


Đa thức và số học


Mà ta lại có 0  y  y  p  p 2  1 . Điều này mâu thuẩn với hệ P  0  , P  1  ,  , P  p 2  1  là 
hệ thặng dư đầy đủ modulo p.
 
Phần tiếp theo ta chứng minh hệ P  0  , P  1  ,  , P  p 3  1  là một hệ thặng dư đầy đủ
modulo p 3 .
Giả sử tồn tại hai số 0  a  b  p 3 và P  a   P  b  mod p 3 .  

Ta đặt a  a0  p 2 a1 ; b  b0  p 2 b1 với 0  a0 , b0 , a1 , b1  p 2 ;0  a, b  p . 
Ta cần chứng minh khi đó a  b .
Từ cách đặt và giả sử ta có các điều kiện ta suy ra
P  a   P  a0   mod p 2  , P  b   P  b0   mod p 2  ; P  a   P  b   mod p 2 

 
Ta suy ra P  a0   P  b0  mod p 2 . Từ đó suy ra a0  b0 .
Lại theo phần chứng minh trong định lí Hensel ta có
0  P  a   P  b   P  a0   p 2 a1 P '  a0    P  b0   p 2 b1 P '  b0    p 2 P '  a0  a1  b1   mod p 3 
CHINH PHỤC OLYMPIC TOÁN

 a1  b1  0  mod p  , do ta đã chứng minh P '  x  p .

 
Từ đó ta có a  b  P  a   P  b  mod p 3  a  b . Ta có điều cần chứng minh.
Nhận xét.
 Một cách tổng quát hơn, bằng cách chứng minh tương tự như trên ta có thể chỉ ra
rằng với số nguyên d , e  1 bất kỳ, đa thức h hoán vị cho các lớp giá trị dư modulo
p d khi và chỉ khi nó hoán vị cho các lớp giá trị dư modulo p e . Lập luận trên được
thể hiện quen thuộc trong kết quả chứng minh của bổ đề Hensel.
 Khái niệm về lớp giá trị dư – Residue class – AoPS: Trong các bài toán số học về
modulo, phần còn lại của một số nguyên a theo modulo n là giá trị duy nhất của
0  r  n  1 , vậy a  kn  r . Lớp giá trị dư là tập các số nguyên đồng dạng theo
modulo n với một vài giá trị nguyên dương n . Trong modulo n , có chính xác n
lớp giá trị dư phân biệt, tương ứng với n giá trị còn lại 0,1, 2, 3, n  2, n  1  . Mỗi
lớp giá trị dư chứa tất cả các số nguyên dạng kn  r với r là phần dư tương ứng.

Câu 34. Với số nguyên n  3 , đặt f  x  , g  x  là đa thức với hệ số thực sao cho các điểm
 f  1 , g  1  ,  f  2  , g  2   ,,  f  n  , g  n   trong tập 2
là các đỉnh của đa giác n cạnh
theo thứ tự ngược chiều kim đồng hồ. Chứng minh rằng ít nhất một trong các đa thức
f , g có bậc không nhỏ hơn n  1 .
Putnam 2008 A5
Lời giải

Chinh phục olympic toán| 39


Bồi dưỡng học sinh giỏi

Ta biểu diễn các điểm trên dưới dạng đa thức hệ số phức P  z   f  z   ig  z  . Không khó
để chứng minh rằng deg P  n  1 , khi đó một trong các hàm f , g phải có bậc không nhỏ
hơn n  1 .
Thay P  z  bằng aP  z   b với các số a , b  thích hợp, ta biến đổi các đỉnh của đa giác n
 2 i 
cạnh trên thành dạng  n ,  n2 ,  ,  nn với  n  exp   . Dễ kiểm tra rằng không tồn tại đa
 n 
thức P  z  có bậc không vượt quá n  2 sao cho P  i    in với i  1,  , n .
Ta chứng minh rằng với mọi số phức t  0, 1 , với mọi số nguyên m  1 , mọi đa thức
Q  z  sao cho Q  i   t i với i  1,  , m có bậc không bé hơn m  1 .
Nếu deg Q  d và có hệ số cao nhất là c, khi đó R  z   Q  z  1   tQ  z  có deg R  d và có
hệ số cao nhất là  1  t  c . Tuy nhiên, theo giả thiết trên, R  z  có các nghiệm phân biệt

TẠP CHÍ VÀ TƯ LIỆU TOÁN HỌC


1, 2  , m  1 nên d  m  1 .
Nhận xét. Ta có thể giải bài toán trên bằng cách sử dụng ma trận Vandermonde hoặc công
thức nội suy Lagrange để tính toán hệ số cao nhất của Q .

p1
Câu 35. Cho p là một số nguyên tố lẻ. Chứng minh rằng có ít nhất giá trị
2
p 1
n  0, 1, 2  , p  1 sao cho  k !n
k 0
k
không chia hết cho p .

Putnam 2011
Lời giải
p 1
Đặt f  x    k ! x k  Fp  x  là tập hợp các đa thức bậc p có hệ số thuộc trường Fp .
k 0

Ta phân hoạch Fp  Z   
Z ' , trong đó Z  a  Fp ; f  a   0 ; Z '  a  Fp ; f  a   0 
Ta thấy f  0   1  0  Z '  Z  1 . Đặt Z  p  r , Z '  r  1
Xét đa thức s  x     x  a   deg s  x   r .
aZ '

Dưa vào cách ta định nghĩa thì đa thức s  x  f  x  triệt tiêu ( luôn bằng 0 ) tại mọi điểm của
trường Fp .

 
Do đó s  x  f  x   x p  x h  x  trong đó deg h  r  1 và thuộc Fp [ x ] , lưu ý đến bậc của hai
vế + định lý Fermat nhỏ. Trong những tính toán tiếp theo có sử dụng đến đạo hàm hình
thức. Ta có
d d p 1 p2 p 1
x  xf  x    x  k ! x  x   k  1 ! x   k ! x k  f  x   1
k 1 k

dx dx k 0 k 0 k 1

 x 2 f '  x   xf  x   f  x   1  x 2 f '  x    x  1  f  x   1  0

40 | Tạp chí và tư liệu toán học


Đa thức và số học


Lấy đạo hàm hình thức s  x  f  x   x p  x h  x  ta được: 
 
s ' x  f  x   s  x  f ' x   x p  x h ' x   h  x 
Chú ý. Loại bỏ các số hạng triệt tiêu trên trường Fp
Từ các kết luận trên ta thấy
 
x2s ' x  f  x   x p  x x2h ' x   x2h  x   s  x  x 2 f ' x 

 
 x p  x x 2 h '  x   x 2 h  x   s  x    x  1 f  x   1

   
 x p  x x 2 h '  x   x 2 h  x    x  1 x p  x h  x   s  x 

  x  x   x h '  x    x  1 h  x    x h  x   s  x 
p 2 2

Ta thấy vế trái triệt tiêu trên Z 0, x p  x triệt tiêu trên Fp nên đa thức x 2 h  x   s  x 
triệt tiêu trên z  {0} .
p1
CHINH PHỤC OLYMPIC TOÁN

Ta thấy r  1  deg  x 2 h  x   s  x    Z  1  p  r  1  r 
.
2
Câu 36. Có tồn tại hay không một dãy số thực và khác 0 là a1 ; a2 ;  , an thỏa với mỗi
n thì đa thức a0  a1 x  an x n có đúng n nghiệm trên .
Lời giải – Dark Templar – VMF. Diễn đàn toán học Việt Nam
Ta chứng minh dãy trên tồn tại bằng cách xây dựng nó bằng quy nạp.
Chọn a0  1, a1  1 sao cho thỏa mãn tính chất trên với n  1 .
n
Đặt Pn  x    ak x k với n nghiệm phân biệt là r1  r2    rn .
k 0

Chọn x0  r1 . Cho x k   rk ; rk  1  ; k   1; n  1 là giá trị mà Pn  x  đạt giá trị lớn nhất.


Cho xn  rn , có Pn  xk   0; k   0; n và ta có thể chọn a  0 sao cho

 Pn  ( xk  ; k   0; n .
n1
a xk
Nếu Pn  xn   0, đặt an  1   a
Nếu Pn  xn   0, đặt an  1  a , có a xk  Pn  xk  chứng tỏ rằng Pn 1  xk   Pn  x k   an 1 x n 1
n1

luôn khác không và có cùng dấu với Pn  x k  và lim Pn 1  x  có dấu khác Pn  1  xn  .
x 

Do đó Pn  1  x  có 1 nghiệm thuộc  xk ; xk  1  ; k  0; n  1 và 1 nghiệm lớn hơn x n

Chinh phục olympic toán| 41


Bồi dưỡng học sinh giỏi

Câu 37. Cho P  x    x  thỏa mãn P  x  là số chính phương với mọi x nguyên thì
P  x   Q 2  x  với Q  x    x  .
Lời giải
Ta chứng minh bằng phản chứng. Giả sử P  x   Q  x   p1  x   p2  x   pk  x  với pi  x  là
2

các đa thức bất khả quy bậc lớn hơn 1


Do pi  x  bất khả quy nên p1  x  và p1 '  x  .p2  x   pk  x  nguyên tố cùng nhau nên theo
định lý Bézout thì tồn tại hai đa thức R  x  , S  x  với hệ số nguyên sao cho

R  x   p1  x   S  x   p1  x   p2  x  pk  x   n ( n  )

Chọn p  n nguyên tố sao cho tồn tại a để p1  a  chia hết cho p t hì p1  a  p2  a   pk  a 
không chia hết cho p , luôn tồn tại p theo Schur.
Nếu p1  a  không chia hết cho p 2 ta suy ra vô lý vì khi đó vp  P  a   lẻ.

TẠP CHÍ VÀ TƯ LIỆU TOÁN HỌC


Nếu p1  a  chia hết cho p 2 ta xét p1  a  p   p1  a   p  p1 '  a   ( mod p 2  thì p1  a  p  không
chia hết cho p 2 .
Chứng minh tương tự như trên ta suy ra mâu thuẫn
Vậy P  x   Q 2  x 
Nhận xét.
 Bài toán có thể tổng quát cho trường hợp P  x  là lũy thừa bậc n của một số
nguyên với cách chứng minh hoàn toàn tương tự.
 Bằng bài toán nhỏ trên ta có thể giải bài toán dưới đây với lời giải ngắn gọn hơn.

Câu 38. ìm tất cả các đa thức số hệ số nguyên thỏa m n a  b là số chính phương thì
P  a   P  b  cũng là số chính phương, trong đó a , b là các số tự nhiên.
Lời giải – Nguyễn Tiến Khải, Phạm Khoa Bằng – THPT chuyên KHTN
Bổ đề. Nếu P  x   x  là số chính phương với mọi x thì nó là bình phương của một đa
thức khác
Quay trở lại bài toán nhận xét rằng a  b chính phương thì  a  b  x 2 chính phương.

   
Cố định a , b , ta xét đa thức Q  x   P ( ax  P (bx là số chính phương với vô số x nên là
2 2

bình phương của đa thức G  x  nào đó.


Đặt G  x   gm x m  và P  x   pn x n  ,
n n 2
 
Đồng nhất hệ số bậc cao nhất ta suy ra pn ( a  b  g nghĩa là với mọi a  b chính
phương thì ta có điều trên

42 | Tạp chí và tư liệu toán học


Đa thức và số học

Cặp  1, 0  có 1  0 chính phương suy ra với mọi a  b số chính phương thì a n  b n là số


chính phương với n  1
Với n  0 cho ta P  x   2 k 2 .
Với  2, 2  ta suy ra 2.2 n  2 n 1 suy ra n lẻ. Chọn  a , b    3, 1  ta có
3n  1  u 2 hay 3n   u  1  u  1 
Ta có  3, 2   1 và gcd  u  1, u  1 |2 , mà 3 nguyên tố nên u  1  1 hay u  2  n  1
Đặt P  x   kx ta cần có k chính phương
Vậy các đa thức cần tìm: P  x   k 2 x ; P  x   2 k 2 .

Câu 39. Giả sử m , p là các số nguyên tố khác nhau. Chứng minh rằng nếu có một số tự

 
nhiên x nào đó mà p là ước của P  x   x  x  ...  1 thì ta có p  1  mod m  .
m1 m2
CHINH PHỤC OLYMPIC TOÁN

Lời giải
Giả sử p  mk  r  0  r  m  1  , ta cần chỉ ra là r  1 thì khi đó p  1  mk .

  
Giả sử tồn tại số tự nhiên x sao cho x  x  ...  1 p  p x  1  x m  1  mod p   1 
m1 m2 m

Do đó x không chia hết cho p nên  x , p   1.
Ta sẽ chứng minh x r 1  1  mod p  .
 Nếu p  m  r  p là số nguyên tố nên x r 1  1  mod p   2  đúng do định lí Fermat.
pr
 Nếu p  m . Ta có k  . Trong  1  ta nâng lũy bậc k hai vế , ta được
m
x p r  1  mod p   * 
Nhưng theo định lí Fermat thì x p 1  1  mod p  .

Do đó trừ vế theo vế ta suy ra x


p r
 
x r 1  1  0  mod p   x r 1  0  mod p  do ta đ có  *  .
Cuối cùng ta chỉ ra r  1 . Ngược lại nếu r  1 . Khi đó  m , r  1   1 vì m là số nguyên tố.

 r 1
Do đó x  1, x  1  x
m 
 m , r  1
 1  x  1 . Nhưng từ  1  ,  2  ta có p x m  1 và p x r 1  1 , do
đó p x  1  x  1  mod p  , điều này do ước chung lớn nhất thì chia hết cho mọi ước. Dẫn
đến P  x   m  mod p  , điều này trái với giả thiết là p P  x  . Vậy r  1 nên p  1  mod m  .

Chinh phục olympic toán| 43


Bồi dưỡng học sinh giỏi

Câu 40. Cho đa thức P  x  có bậc n và có n nghiệm phân biệt x1 , x2 ,  , xn .


Chứng minh rằng:
P ''  x1  P ''  x2  P ''  xn 
a)   0
P '  x1  P '  x2  P '  xn 
1 1 1
b)   0
P '  x1  P '  x 2  P '  xn 
Lời giải
a) Ta viết đa thức P dưới dạng P  x    x  x1  x  x2   x  xn  , giả sử rằng
x1  x 2    xn .
 1 1 1  n
1
Ta có P '  x   P  x       P  x    1
 x  x1 x  x 2 x  xn  i  1 x  xi

Do P  xi   0, i  1, n nên theo định lý Rolle tồn tại

TẠP CHÍ VÀ TƯ LIỆU TOÁN HỌC


c 1 , c 2 ,  , c n  1 ; x1  c 1  x 2  c 2    c n  1  x n
Sao cho P '  c i   0, i  1, n  1 2
 1 1 1  n1
1
Lại có P ''  x   P '  x       P '  x    3
 x  c1 x  c 2 x  cn1  i 1 x  c i

Từ  1  ,  2  suy ra:
  1 1 1  n
1
 P '  c1   P  c1       P  c 1 
 0
  c 1  x1 c 1  x 2 c 1  xn  i 1 c 1  xi
  
 P '  c 2   P  c 2   1  1  1   P  c 2   1  0
n

  c 2  x1 c 2  x 2 c 2  xn  i 1 c 2  xi


  1 1 1  n
1
  n1 
P ' c  P  n1  
c     P  n1  
c 0
  c n  1  x1 c n  1  x 2 c n 1  xn  i 1 c n 1  xi

Do P  ci   0, i  1, n  1 nên ta có:
 1 1 1 n
1
c  x c  x    
c 1  xn i 1 c 1  xi
0
 1 1 1 2

 1 1 1 n
1
 
 c 2  x1 c 2  x 2
  
c 2  xn i 1 c 2  xi
0
...................

 1 1 1 n
1
 c  x  c  x  c  x   c  x  0
 n1 1 n1 2 n1 n i 1 n1 i

n
P ''  xi  n
1 n
1 n
1
Suy ra  P ' x 
i 1

i  1 c 1  xi

i 1 c 2  xi
 ...  
i 1 c n 1  xi
0
i

44 | Tạp chí và tư liệu toán học


Đa thức và số học

n
b) Xét phân tích P  x    x  x1  x  x2  x  xn     x  xi  .
i 1
n n
Đặt Pi  x    x  x   P x   P x .
j  1, j  i
j

i 1
i

 
Ngoài ra Pi x j  0, j  i ; Pi  xi   0, i  1, n  P  xi   Pi  xi  , i  1, n .
n
Pi  x 
Xét đa thức Q  x     1 có bậc không vượt quá n  1 .
i 1 P   xi 
Ta có i  1, n  Q  xi   0 .
Suy ra đa thức này có n nghiệm thực, tức Q  x   0 và hệ số cao nhất của Q cũng bằng 0.
1 1 1
Do đó   0.
P '  x1  P '  x 2  P '  xn 
CHINH PHỤC OLYMPIC TOÁN

Câu 41. Cho f là một đa thức có hệ số hữu tỉ và bậc không nhỏ hơn 2, và d y  an  chỉ
gồm các số hữu tỉ thỏa mãn f  an 1   an với mọi số nguyên dương n. Chứng minh rằng
dãy  an  tuần hoàn.
Lời giải
f x
Để ý vì P  x  có bậc lớn hơn 2 nên lim  .
x
 Ta sẽ chỉ ra dãy  an  là bị chặn.

Vì f có bậc không nhỏ hơn 2 nên f có tính chất là khi x đủ lớn thì f  x   .
f x
Điều đó có nghĩa là tồn tại số M  0 mà nếu x  M và đồng thời lim   thì ta có
x
f  x   x  M. Ta chọn được số M như vậy mà M  a1 vì do a1  .
Ta sẽ chỉ ra là an  M , n  *
.
Thật vậy. Giả sử phản chứng tồn tại số n để an  M .

Khi đó f  an   an  an1  an  M. Tiếp tục như vậy ta đi đến a1  M là mâu thuẫn với
cách chọn ban đầu. Như vậy dãy  an  bị chặn.
 Tiếp theo ta cần chỉ ra dãy  an  chỉ nhận hữu hạn giá trị.
Để làm việc đó ta cần chỉ ra là tồn tại một số nguyên dương N để N .an là số nguyên với
mọi n. Như thế thì đi đến dãy  Nan  chỉ có hữu hạn phần tử. Và đi đến dãy  an  cũng chỉ
có hữu hạn phần tử.
Thật vậy. Gọi k là số thỏa mãn g  x   k. f  x  là một đa thức có hệ số nguyên.
Đặt g  x   k. f  x   bs x s  bs 1x s 1  ...  b1x  b0 thì bs là các số nguyên.

Chinh phục olympic toán| 45


Bồi dưỡng học sinh giỏi

Gọi N là số nguyên dương để cho N .a1  .


Khi đó ta thấy nếu Nan  thì Nan 1  *.
Chứng minh  *  .
x
Ta thấy Nan  1 là nghiệm của đa thức f    an . Ta tạo ra đa thức monic bằng cách xét đa
N
k.N s  x 
thức h  x   f    an  thì đa thức này nhận Nan  1 làm nghiệm nên nghiệm này
b s  N 
phải là nghiệm nguyên. Vì vậy Nan 1  . Từ đó theo nguyên lý quy nạp ta có  Nan 
nguyên với mọi số số nguyên dương n. Lại vì  an  là dãy bị chặn nên  Nan  cũng bị
chặn. Vì thế dãy  Nan  chỉ có hữu hạn phần tử.
 Cuối cùng ta cần chỉ ra sự tồn tại của chu kỳ.
Giả sử dãy trên có tất cả m giá trị phân biệt.

TẠP CHÍ VÀ TƯ LIỆU TOÁN HỌC


Xét các bộ gồm m  1 số của dãy là ai , ai  1 ,.., ai  m .
Vì bộ này có tính thứ tự nên số các bộ khác nhau tối đa là mm  1 .
Điều này có nghĩa là sẽ có một loại bộ xuất hiện vô hạn lần trong dãy.
Trong mỗi bộ m  1 số này luôn có hai số bằng nhau theo nguyên lý Dirichlet .
Ta giả sử hai số đó là a j  a j  k . Như thế luôn tồn tại k nguyên dương sao cho với j đủ lớn
sao cho a j  a j  k . Từ a j  a j  k ta suy ra ai  ai  k với mọi i  j do cấu trúc của hàm số cho
ban đầu. Mặt khác vì ta luôn chọn được j đủ lớn để có được điều trên nên ta suy ra
ai  ai  k , i  .

P '  1 n
Câu 42. Cho P  x  là đa thức bậc n với hệ số thực sao cho P  1  khác 0 và  
P  1 2
Chứng minh rằng P  x  luôn có ít nhất một nghiệm x 0 sao cho x0  1 .
Lời giải
Giả sử xi , i  1, n là các nghiệm phức của đa thức P , khi đó ta xét với a  0 :
n
P ' x  n
1 P '  1 n 1
P  x   a  x  xi     
i 1 P x i 1 x  xi P  1 i 1 1  xi
P '  1 n n n 1 n
1 1  1 n  xi  1 
Suy ra          
P  1 2 2 i  1 1  xi i  1  2 1  xi  2 i  1  xi  1 
xi  1  xi  1  xi  1
  Re 
 2

 xi  1  xi  1
Theo tính chất của số phức  , i  1, n
xi  1 xi  1
2
 xi  1  xi  1
2

46 | Tạp chí và tư liệu toán học


Đa thức và số học

P '  1  P '  1  n 1 n  xi  1 
2
n
  
2
Mặt khác  R    0  xi  1  xi  1
P  1  2 P  1  2 2 i 1  xi  1 
2

Vậy ta có điều phải chứng minh.

Câu 43. Giả sử tồn tại đa thức hệ phức P , Q , R thỏa mãn P a  Qb  Rc  a , b , c  .


1 1 1
Chứng minh rằng   1
a b c
The IMO Compendium
Lời giải
Bổ đề. Nếu A , B, C là các cặp đa thức “nguyên tố cùng nhau” (coprime) theo dạng
A  B  C , khi đó bậc của mỗi đa thức nhỏ hơn số lượng các đa thức khác 0 của đa thức
ABC .
k l m
CHINH PHỤC OLYMPIC TOÁN

Chứng minh. a đặt A  x     x  pi  , B  x     x  qi  , C  x     x  ri  i .


i a i b c

i 1 i 1 i 1

Viết lại điều kiện A  B  C dưới dạng A  x  C  x   B  x  C  x   1 , đạo hàm hai vế biểu
1 1

thức theo biến x ta được:


1  a c  1  b c 
k m l m
A  x  C  x    i   i   B  x  C  x    i   i 
 i 1 x  pi i 1 x  ri   i 1 x  qi i 1 x  ri 
Ax
Ta thấy rằng có thể viết được dưới dạng thương của hai đa thức có bậc không vượt
Bx
quá k  l  m  1 , từ đây ta suy được rằng A và B “nguyên tố cùng nhau”
Áp dụng bổ đề trên với các đa thức P a , Q b , Rc có các bậc của chúng là
a deg P , b deg Q , c deg R lần lượt nhỏ hơn deg P  deg Q  deg R , từ đó suy ra được rằng:
1 deg P
 
a deg P  deg Q  deg R
Cộng lần lượt vế theo vế các bất đẳng thức, ta có điều phải chứng minh.
Nhận xét.
 Trong bài toán trên ta bắt gặp một khái niệm mới về “cặp đa thức nguyên tố cùng
nhau” (coprime polynomials) được phát biểu tương tự như đối với cặp số nguyên
tố cùng nhau, là khi các đa thức p  t  , q  t  thỏa mãn gcd  p  t  , q  t    1
 Bài toán ta vừa thực hiện là mở rộng cho định lý lớn Fermat cho đa thức.

Câu 44. Cho đa thức P  x  và Q  x  với số thực k bất kì thỏa mãn Pk  z  P  z   k


và Qk  z  |Q  z   k . Chứng minh rằng P0  Q0 và P1  Q1 suy ra được P  x   Q  x 
Lời giải

Chinh phục olympic toán| 47


Bồi dưỡng học sinh giỏi

Không mất tính tổng quát, giả sử n  deg P  deg Q .


Đặt P0  z1 , z2 , , zk  và P1  zk  1 , zk  2 ,  , zk m  .
Hai đa thức P và Q trùng nhau tại k  m điểm z1 , z2 ,  , zk  m .
Ta cần chứng minh k  m  n
Ta có P  x    x  z1   x  zk    x  zk  1   x  zk  m 
1 k  k 1  k m
1
Với  1 ,  ,  k  m là các số tự nhiên.

Xét P '  x  , ta biết đa thức này chia hết cho  x  zi 


i  1
với i  1, 2, , k  m .
k m

x  z  |P '  x 
i  1
Suy ra i
i 1
k m
Vì vậy 2n  k  m  deg   x  zi 
i  1
 deg P '  n  1, hay k  m  n  1 .
i 1

TẠP CHÍ VÀ TƯ LIỆU TOÁN HỌC


Vậy ta có điều phải chứng minh.

Câu 45. Cho đa thức P  x    x  ,deg P  2 . Chứng minh rằng tồn tại m 
để P  m !
là hợp số.
IMO Shortlist 2005
Lời giải
Xét số nguyên tố p và số tự nhiên chẵn k  p . heo định lý Wilson ta có
 p  k  !  k  1   1 k  1   p  1 !  1  mod p 
n n
Do đó  k  1 ! P   p  k  !   ai  k  1  !   p  k  !  k  1  !   ai  k  1  !  mod p 
ni i ni

i 0 i 0

Suy ra  k  1  ! P   p  k  !  S   k  1  !  mod p  với S  x   an  an1 x  a0 x n

Từ đó suy ra p P   p  k  !  S   k  1  ! p
Chú ý rằng S   k  1  ! là đa thức phụ thuộc vào k .

Xét k  2 an  1 ta được s 
 k  1 ! là số nguyên dương chia hết cho mọi số nguyên tố nhỏ
an
hơn k .
Ta có S   k  1  !  an bk , bk  1  mod s  .
Suy ra b k chỉ chia hết cho các số nguyên tố lớn hơn k .
Cho k càng lớn thì S   k  1  ! càng lớn. Với k đủ lớn thì bk  1 .

Do đó tồn tại ước nguyên tố p của b k mà p P   p  k  ! .

Ta cần chọn k sao cho đủ lớn để P   p  k  !  p .

Xét số nguyên tố q đủ lớn mà k   q  1  !

48 | Tạp chí và tư liệu toán học


Đa thức và số học

Nhận xét rằng k  i là hợp số với i  1,  , q  1


Khi đó p là số nguyên tố lớn hơn k mà P   p  k  ! : p nên p  k  q  1 .
Suy ra p  k  q  r , r  0
Với số nguyên tố q đủ lớn, do deg P  2 nên
P   p  k  !  P   q  r  !   q  r  !   q  1  ! q  r  p

Vậy ta có p P   p  k  ! và p  P   p  k  ! nên ta có điều phải chứng minh.

Câu 46. Cho f  x  là đa thức với hệ số hữu tỉ bậc lớn hơn hoặc bằng 2. Xét dãy an  các
số hữu tỉ thỏa m n điều kiện f  an 1   an , n  1 . Chứng minh rằng tồn tại k  1 để
an k  an  n  1 
Lời giải
CHINH PHỤC OLYMPIC TOÁN

Phần đầu ta chứng minh dãy an  là giới nội (bị chặn), nghĩa là tồn tại số M sao cho
an  M , n  .
f x
Thật vậy, do deg f  2 nên lim  .
x  x
Do đó tồn tại số M sao cho khi x  M  f  x   x , chẳng hạn có thể lấy M  a1 .
Với M chọn như thế ta chỉ ra an  M , n  .

Giả sử tồn tại số n để cho an  M . Khi đó an1  f  an   an  M .

ương tự an 2  f  an1   an1  M ,…, a1  M .


Bằng cách tương tự ta có a1  a2  ...  an  M , trái với cách chọn M  a1 như trên.
Vậy an  M , n  nên dãy an  là dãy bị chặn.
Phần tiếp theo ta chứng minh rằng tồn tại k  1 để an k  an  n  1  .
bd x d  ...  b0
Vì f  x  là đa thức với hệ số hữu tỉ nên ta luôn có thể viết f  x   , trong đó
c
b0 ,..., bd , c là các số nguyên.
r
Lại do dãy an  gồm toàn các số hữu tỉ nên ta giả sử a1  với r , s là các số nguyên.
s
Ta cần chọn ra số tự nhiên N để cho ta có N .an là số nguyên với mọi n .
a đặt N  s.bd , ta chứng minh bằng quy nạp N .an là số nguyên. Thật vậy
r
Ta có N .a1  sbd .  r.bd nguyên.
s
Giả sử là N .an cũng là số nguyên với n  1 . Ta xét số N .an  1 .

Chinh phục olympic toán| 49


Bồi dưỡng học sinh giỏi

d
x
bd    ...b0
x  N cN d  x 
Ta có f    . Nên ta xét đa thức P  x    f  N   an  thì đa thức
N c bd    
P  x  là hệ số nguyên với hệ số bậc cao nhất là 1.
cN d
Mà ta có P  N .an1    f  an1   an   0 ( do f  an 1   an , n  1 ).
bd 
Nên N .an  1 là nghiệm hữu tỉ của đa thức P  x  nên nó cũng là nghiệm nguyên.
Vậy N .an  1 là số nguyên, do đó N .an là số nguyên với mọi n .
Dãy an  là dãy bị chặn và do N .an  1 là các số nguyên nên mỗi phần tử đề là một bội số
1
nào đó của .
N
Do đó d y chỉ nhận hữu hạn giá trị khác nhau với vô hạn số hạng của nó.

TẠP CHÍ VÀ TƯ LIỆU TOÁN HỌC


Chẳng hạn ta giả sử dãy chỉ nhận m giá trị khác nhau.
Ta xét m  1 số hạng của dãy là a1 , a2 ,..., am 1 . Khi đó tồn tại hai số nguyên dương i1 , k1 sao
cho 1  i1  i1  k1  m  1 và ai1  ai1  k1 . ương tự tồn tại các số k j cho mỗi đoạn  m  1  số
hạng tiếp theo. Do các k j chỉ nhận hữu hạn giá trị nên tồn tại số k  k j với vô hạn giá trị
khác nhau j .
Ta sẽ chứng minh k chính là chu kì của dãy an  , tức là an  k  an , n .
Ngược lại, giả sử có chỉ số n0 nào đó mà an0  k  an0 .

Do dãy an  chỉ nhận hữu hạn phần tử nên ta có thể chọn n j đủ lớn n j  n0 để có thể xảy  
   
ra an j  k  an j . Khi đó theo giả thiết ta có an j  k 1  f an j  k  f an j  an j 1 . Tiếp tục quá trình

trên sau nhiều lần ta sẽ lùi chỉ số n j về tới n0 và ta có an0  k  an0 là vô lí.
Vậy an  k  an , n .
Bài toán được chứng minh.

Câu 47. Tìm tất cả đa thức   x  sao cho  p  2 p  p , p là số nguyên tố


IMOTC 2016 – Senior Batch
Lời giải
Ta có  2  2,  5  27 và 3  5   2  nên  2    5  1 hoặc  2    5   1
Không mất tính tổng quát, giả sử  2   1 , ta thấy  0   1 bởi nếu tồn tại 1 ước nguyên
tố lẻ cũa 0 là q thì q| q  , hay q|2 q (vô lý)
Xét p nguyên tố bất kỳ, gọi q là 1 ước nguyên tố bất kỳ của  p  thì q|2 p  p
Gọi m  ord 2  q  , giả sử m  2 có  m, q   1 nên theo định lý thặng dư Trung Hoa thì:

50 | Tạp chí và tư liệu toán học


Đa thức và số học

 x  p  mod q 

x  p  v  mod m 
với v là 1 số thỏa mãn v không chia hết cho m và  p  v ; m   1 , hoàn toàn chọn được, do
m  2 nên   m   2 ), có 1 nghiệm x  mod qm 
Dễ thấy do  0   1 nên  p ; q   1 và  p  v ; m   1 nên  x ; qm   1
Theo định lý Dirichlet, tồn tại 1 số nguyên tố t để t  x  mod qm  .

Ta có q|t  p nên q  t    q   q  t   q 2t  t  q 2 p  2t  p  1  p  t  q|2t  p  1 , vô


lý do t  p không phải là bội của m .
Suy ra m  2 hay q  3 , vậy  p  có dạng 3 t với mọi p nguyên tố  1 
Cố định p  3 và t , theo định lí Dirichlet thì tồn tại vô hạn số nguyên tố q thỏa mãn
q  p  mod 3t 1  .
CHINH PHỤC OLYMPIC TOÁN

Có 3t  1 |q  p nên 3t  1 | q    p .
Kết hợp với  1  ta có  q   3t có vô số nghiệm nên đa thức là đa thức hằng.
Thử lại thấy x  1 hoặc  x   1 thỏa mãn.

Câu 48. Xét đa thức T  x   x 3  17 x 2  1239x  2001 . Đặt


T1  x   T  x  , T2  x   T T1  x   , , Tn1  x   T Tn  x   với mọi n  1, 2, 3...
Chứng minh rằng tồn tại số nguyên n  1 sao cho Tn  x   x chia hết cho 2003 với mọi số
nguyên x .
Lê Quang Nẫm – TPHCM
Lời giải
rước hết ta có các nhận xét sau:
Nhận xét 1. x  y  mod 2003   x 3  y 3  mod 2003  .
Để thuận tiện trong quá trình viết, ta kí hiệu là x  y thay cho x  y  mod 2003  .
Hiển nhiên ta có x  y  x 3  y 3 (do 2003 là số nguyên tố)
Ta cần chứng minh x 3  y 3  x  y .
Nếu x  0 thì y  0 do x 3  y 3 mà 2003 là số nguyên tố.
Nếu x không chia hết cho 2003 thì suy ra x 3 không chia hết cho 2003, suy ra y 3 không
chia hết cho 2003. Suy ra y không chia hết cho 2003.
heo định lí Fermat nhỏ thì x 2003  x , y 2003  y  x 2002  y 2002  1 .
Do x 3  y 3  x 2001  y 2001  xy 2001  x 2002  y 2002  y.y 2001  x  y .
Nhận xét 2. T  x   T  y   x  y  mod 2003 

Chinh phục olympic toán| 51


Bồi dưỡng học sinh giỏi

Ta phân tích T  x    x  662   2003  x 2  657 x   2001  662 3


3

Do đó T  x   T  y    x  662    y  662   x  662  y  662  x  y .


3 3

Trở lại với bài toán. a đặt A  0; 1; 2;...; 2002 . Khi đó với mỗi x  A ,ta xét d y các đa
thức T1  x  , T2  x  ,..., T2004  x  . Theo nguyên lí Dirichlet sẽ tồn tại hai số 1  a  b  2004 mà
Ta  x   Tb  x  , vì khi chia cho 2003 thì có tối đa 2003 số dư. Ở đây các Tk  x  này giờ là các
số)  Ta  x   Ta Tb a  x    Tb a  x   x , theo nhận xét 2.
Như vậy với mỗi x  A thì tồn tại một nx  *
sao cho Tnx  x , vì theo trên ta có thể xét một
dãy 2004 đa thức khác, tức là một vòng mới mà có chỉ số tăng lên. Gọi n  1 là một bội chung
của n0 , n1 , n2 ,..., n 2002 . Ta sẽ chứng minh n thỏa mãn bài toán.
 Nếu x  A thì ta có Tnx  x   x  T2 nx  x   Tnx  x   x (vì T2 nx  x   Tnx Tnx  x  .  
ương tự ta có Tknx  x   Tnx  x   x , k  *

TẠP CHÍ VÀ TƯ LIỆU TOÁN HỌC


.
Do n nx tức là n là bội của n x nên ta suy ra Tn  x   x  Tn  x   x  2003 .
 Nếu x  A thì tồn tại y  A sao cho x  y . Do đó Tn  x   Tn  y   y  x .
Vậy Tn  x   x  2003 .

Câu 49. Tìm tất cả các cặp số nguyên a , b sao cho tồn tại đa thức P  x   x sao cho

 
tích x  ax  b  P  x  là đa thức được viết dưới dạng:
2

x n  cn1 x n1   c 1 x  c 0 với c 0 , c 1 ,  , c n 1 bằng 1 hoặc 1


Polish MO 2006
Lời giải
Bổ đề. Cho đa thức P  x   c n x  c n1 x n n1
  c 0 , nếu x 0 là nghiệm của P  x  thì khi đó:

ci
x0  1  max
i  0, n  1 c
n

Chứng minh. Rõ ràng chúng ta chỉ cần phải xem xét trường hợp x0  1 :

n c0  c1 x0   cn1 x0n1
cn x0n  c0  c1 x0   cn1 x0n1  x0 
cn
n
n1
c n1
c c n1
c x c
  i x0i   i x0  max i x
i i
 max i  0  x0  1  max i
i 0 cn i 0 cn
i  0,n  1 c
n i 0
0
i  0, n  1 c
n x0  1 i  0, n  1 c
n

Quay trở lại bài toán ta có b  P  0   c 0  b  1, 1 .


Hơn thế nữa, nếu x 0 là nghiệm của x n  c n1 x n1   c 1 x  c 0 thì khi ấy ta có
ci
x0  1  max 2
i  0, n  1 cn

52 | Tạp chí và tư liệu toán học


Đa thức và số học

Như vậy, với x 0 là nghiệm của phương trình x 2  ax  b , ta có:


  a  a 2  4b  a  a 2  4b 
x0   ,  và x0  2
 2 2 
Kết hợp với dữ kiện b  1, 1 , ta có thể tìm được giá trị của a với a  2, 1,0,1, 2  .

Câu 50. Cho hai đa thức hệ số nguyên


P  x   an x n  an1 x n1  a1x  a0
Q  x   bn x n  bn1 x n1  b1 x  b0
Biết rằng an  bn là một số nguyên tố và an 1  bn 1 . Gọi m là là một nghiệm hữu tỷ
chung của hai đa thức. Chứng minh rằng m là một số nguyên.
Lời giải
r
 r , s  ,gcd  r , s   1 .
CHINH PHỤC OLYMPIC TOÁN

Đặt m 
s
r
Ta có P    0  r a0 , s an . ương tự r b0 , s bn .
s
Từ đó ta có s|an  bn , mà an  bn là một số nguyên tố nên s  1 hoặc s  an  bn
r
Nếu s  1 thì m   r  , ta có điều phải chứng minh!
s
r r
Nếu s  an  bn thì ta có P    Q    0
s s
n n2
r r r
  an  bn      an2  bn2      a1  b1     a0  b0   0
s s s
  an  bn   r n   an 2  bn 2  r n 2   a1  b1  r  s n1   a0  b0  s n  0
 s  r n   an 2  bn 2  r n  2  s 2   a1  b1  r  s n 1   a0  b0  s n  0  s|r n
Lại có s nguyên tố nên s r , mâu thuẫn với gcd  r , s   1 .
Vậy ta có điều phải chứng minh!

Câu 51. Hỏi có tất cả bao nhiêu đa thức Pn  x  bậc n chẵn thỏa m n các điều kiện
 Các hệ số của Pn  x  thuộc tập M  0; 1; 1 và Pn  0   0 .

 Tồn tại đa thức Q  x  có các hệ số thuộc M sao cho Pn  x   x  1 .Q  x  .


2
 
Nguyễn Viết Long – THPT Chuyên Lam Sơn – Thanh Hóa
Lời giải
Đặt deg Pn  x   n  2m . Xét đa thức Q  x   b0 x n 2  b1 x n 3  ...  bn 3 x  bn 2 .

 
Từ giả thiết ta có Pn  x   x  1 .Q  x  nên suy ra
2

Chinh phục olympic toán| 53


Bồi dưỡng học sinh giỏi

Pn  x   b0 x n  b1 x n1   b2  b0  x n 2   b3  b1  x n 3  ...   bn 2  bn 4  x 2  bn 3x  bn 2 .


Từ đó suy ra số các đa thức Pn  x  bằng số dãy số  b0 , b1 ,..., bn  2  trong đó các số hạng của
dãy bn  thỏa m n điều kiện là bk  1; 0; 1 với mọi 0  k  n  2 và bk  2  bk  1; 0; 1 với
mọi 0  k  n  4 đồng thời bn  2  0 .
 Để ý rằng nếu bk  1 thì bk  2  0; 1 .
 Để ý rằng nếu bk  0 thì bk  2  1; 0; 1 .
 Để ý rằng nếu bk  1 thì bk  2  0; 1 .
Gọi x k là số các dãy chỉ số chẵn  b0 ; b2 ;...; b2 k  thỏa mãn b2 i  M , 0  i  k và
bi  2  bi  M , 0  i  2 k  2
Gọi y k là số các dãy chỉ số lẻ  b1 ; b3 ;...; b2 k 1  thỏa mãn b2 i 1  M , 0  i  k và
bi  2  bi  M , 0  i  2 k  3

TẠP CHÍ VÀ TƯ LIỆU TOÁN HỌC


Ta có x0  2; x1  4 và xk  1  xk  2 xk 1 vì nếu b2 k  0 thì b2 k  2  0; b2 k  và nếu b2 k  0 thì có
ba cách lấy b2 k  2  M , số dãy  b0 ; b2 ;...; b2 k  mà b2 k  0 bằng x k 1 .

1  2   1  2 
k k

Bằng cách sử dụng phương trình đặc trưng ta tìm được x k  .


2
ương tự ta có y0  3, y1  7, y k  1  2 y k  y k 1 , k  1 .
Do đó số các dãy số  b0 , b1 ,..., bn  2  bằng

 xm  1  xm  2  y m  2  1  2   1  2   21 1  2   1  2  
m1 m1 m m

1  2   2  1  2    1  2    1 
2 m1 2 m1 n1 n1
 2  1 
m1
 1 n
1
  2 .
2 2

Câu 52. Cho dãy số nguyên  an n 1 thỏa mãn m  n|am  an với mọi số tự nhiên m,n

phân biệt. Giả sử tồn tại đa thức P  x  sao cho an  P  n  , n . Chứng minh rằng tồn tại
đa thức Q  x  sao cho an  Q  n  , n .
Lời giải
Đặt degP  d . Tồn tại đa thức Q duy nhất có bậc cao nhất là d sao cho Q  k   ak với
k  1, 2, , d  1 . Ta chứng minh Q  n   an với mọi n .
Đặt n  d  1 . Đa thức Q có thể không phải là đa thức hệ số nguyên nên ta không thể trực
tiếp suy ra n  m|Q  n   Q  m  , nhưng chắc chắn rằng Q là đa thức có hệ số hữu tỉ, khi đó
tồn tại số tự nhiên M sao cho R  x   MQ  x  là đa thức hệ số nguyên. Từ điều kiện đề bài
ta có M  an  Q  n    M  an  ak    R  n   R  k   chia hết cho n  k với k  1, 2, , d  1 . Vậy

54 | Tạp chí và tư liệu toán học


Đa thức và số học

với mỗi số n ta có an  Q  n  hoặc Ln  lcm  n  1, n  2,  , n  d  1   M  an  Q  n    Cn d


với C là hằng số không phụ thuộc vào n .
Giả sử rằng an  Q  n  ở một vài giá trị n .
Lưu ý rằng Ln không nhỏ hơn tích  n  1   n  d  1 được chia bởi tích P của số
gcd  n  i , n  j  trên tất cả các cặp  i , j  phân biệt lấy từ tập {1, 2,  , d  1}.
Do gcd  n  i , n  j   i  j , ta có P  1d 2 d 1  d . Từ đó ta được:
 n  1 n  2   n  d  1  PLn  CPnd
Điều này sai với n đủ lớn vì vế trái có bậc là d  1 .
Vậy an  Q  n  cho mỗi giá trị n đủ lớn, ta gọi tạm là n  N .
Nếu n  N , từ điều kiện đề bài M  an  Q  n    M  an  ak    R  n   R  k   chia hết cho
m  n với mọi giá trị m  N , vậy chúng phải bằng 0.
Vì vậy an  Q  n  với mọi n .
CHINH PHỤC OLYMPIC TOÁN

Câu 53. Cho n là số nguyên dương và a1 , a2 ,..., an là các số thực dương.


a đặt g  x    x  a1  x  a2  ...  x  an  .
Gọi a0 là một số thực bất kì và đặt f  x    x  a0  g  x   x n1  b1 x n  ...  bn x  bn 1 .
Chứng minh rằng b1 , b2 ,..., bn  1 đều là số âm khi và chỉ khi a0  a1  a2  ...  an .
VMF – Diễn đàn Toán học
Lời giải
Chiều suy ra.
n
Thì theo định lý Viete ta có b1  a0   ak , từ đó thì ta có điều phải chứng minh.
k 1

Chiều ngược lại.


n
Với x  0 thì ta có  a0  ak  bn1 nên suy ra bn  1  0.
k 1

Ta giả sử rằng  a1   a2  ...   an  a0


Mà ta có g  x    x  a0  g '  x    n  1  x n  ...  bn1x  bn .
Lại theo định lý Lagrange thì ta thấy tồn tại các số dương y i , i  0, n  1 sao cho
 a1   y 1   a2   y 2  ...   an  y 0  a0 và f '  yi   0, i  0, n  1
n
1
Do đó ta có f '  x    x  y0    x  yi  .
n1 i 1
n1
n  n

Đồng nhất hệ số của x n 1 thì ta thu được y0   yi   a0   ai   0 .
i 1 n1 i 1 
Từ ý tưởng trên ta sẽ dùng phương pháp quy nạp để chỉ ra các hệ số bi đều là số âm.

Chinh phục olympic toán| 55


Bồi dưỡng học sinh giỏi

Câu 54. Cho F là tập các đa thức  có hệ số nguyên và phương trình   x   1 có


nghiệm nguyên. Cho trước một số nguyên dương k , tìm giá trị nhỏ nhất của m  1 theo
k thỏa mãn tồn tại   F sao cho   x   m có đúng k nghiệm nguyên phân biệt.
VMF – Diễn đàn Toán học
Lời giải
Do   x   m có k nghiệm nguyên phân biệt nên ta có:
  x    x  x1  x  x2  ...  x  x k  Q  x   m
Với các số nguyên khác nhau x1 , x2 ,..., x k và Q  x   x
Gọi t là nghiệm của phương trình   x   1 khi đó ta có:
 t  x1  t  x2  ...  t  xk  Q  t   1  m
 1  m  t  x1 t  x2 ... t  x k Q  t 

TẠP CHÍ VÀ TƯ LIỆU TOÁN HỌC


 1  m  1.2.3...k.1  m  1  k !  m  k ! 1
Vậy từ đây ta suy ra gái trị nhỏ nhất của m là k ! 1.
Và nó xảy ra khi và chỉ khi   x    x  1  x  2  ...  x  k  1  x  k   k ! 1.

Câu 55. Cho hai đa thức P  x  , Q  x   x nguyên tố cùng nhau và khác đa thức hằng.
Chứng minh rằng không có quá ba số thực  thỏa mãn P  x   Q  x  là bình phương
của một đa thức.
USA December Team Selection Test 2016
Lời giải
Kí hiệu  A  x  , B  x   là ước đa thức monic có bậc lớn nhất của A  x  , B  x  .
rước hết, ta phát biểu một bổ đề sau đây:
 
Bổ đề. Nếu ab  cd  max deg  aP  x   cQ  x   ,deg bP  x   dQ x    max  P x  , Q x  .

Chứng minh
Đặt n  max  P  x  , Q  x   thì ta có vế trái rõ ràng không lớn hơn n.
Xét hệ số của x n trong hai đa thức aP  x   cQ  x  , bP  x   dQ  x  sẽ có một đa thức có hệ số
khác không và dễ dàng suy ra vế trái bằng n nên ta có điều phải chứng minh.
Quay lại với việc chứng minh bài toán
Giả sử tồn tại ba số thực phân biệt  1 ,  2 ,  3 , P  x    iQ  x   Ri  x 
2

Theo bổ đề trên thì tồn tại đa thức Ri  x  có bậc bằng max  P  x  , Q  x   giả sử là R1  x 
2 2

1  2
a đặt   , A  x   R1  x   R2  x  , B  x 
1  3
 R1  x   R2  x  , C  x   R1  x   R3  x  , D  x   R1  x   R3  x 

56 | Tạp chí và tư liệu toán học


Đa thức và số học

Khi đó thì ta có A  x   B  x   C  x   D  x   2 R1  x  , A  x  B  x 
 C  x  D  x  ,   1,  A  x  , B  x    C  x  , D  x    1
Và A  x  B  x  có bậc bằng max  P  x  , Q  x   .
Bây giờ ta sẽ đặt PA x C  x   x    A  x  , C  x   , PA x D x   x    A  x  , D  x   ,
PB x C  x   x    B  x  , C  x   , PB x D x   x    B  x  , D  x  
hì ta có các đa thức trên nguyên tố cùng nhau. Giả sử x 0 là nghiệm bội n của A  x  thì
nó cũng là nghiệm bội n của duy nhất một trong hai đa thức PA x C  x   x  , PAx Dx   x  hay
cũng là nghiệm bội n của đa thức PA x C  x   x  .PAx Dx   x  .
ương tự thì ta cũng có điều ngược lại.
Vậy A  x   aPA x C  x   x  .PA x D x  x  với a là hằng số nào đó.
ương tự thì ta cũng được:
CHINH PHỤC OLYMPIC TOÁN

B  x   bPB x D x   x  .PB x C  x   x  , C  x   cPA x C  x   x  .PB x C  x   x  , D  x   dPA x D x   x  .PB x D x   x 


Mà ta có A  x   C  x   D  x   B  x  .
Giả sử x 0 là nghiệm bội n của PA x C  x   x  .PB x D x   x  thì nó cũng là nghiệm bội n của một
trong hai đa thức PA x C  x   x  , PB x D x   x  nên là nghiệm bội ít nhất n của đa thức
A  x  C  x.
Giả sử x 0 là nghiệm bội n của A  x   C  x  thì nó là nghiệm của nhiều nhất một trong hai
đa thức PA x C  x   x  , PB x D x   x  , giả sử PB x D x   x  không có nghiệm x 0 và x 0 nghiệm bội m
có thể bằng không của PA x C  x   x  .
Giả sử m  n thì ta đạo hàm A  x   C  x   D  x   B  x  từ 0 đến m  1 lần.
Thay x  x0 thì ta có
A   x   C    x   0, 0  i  m, A  x   C m1  x   0, B  x   D  x   0.
i i m  1

Đạo hàm A  x  B  x   C  x  D  x   m  1  lần, thay x  x0 vào thì ta có


A
m  1
 x  B  x   C m1  x  D  x  , dễ thấy điều vô lý.
Vậy ta phải có m  n nên từ đây ta có
A  x   C  x   D  x   B  x   PA x C  x   x  .PB x D x   x 
 aPA x D x   x   bPB x C  x   PB x D x  , dPA x D x   x   bPB x C  x   PA x C  x 

Thay PA x C  x   x  , PB x D x   x  thì ta có

Ax Bx 
ab
P
 2 A x D x 
 
 x  .PC  x D x   x  aPAx Dx   x   cPBx C x   x  dPAx Dx   x   bPBx C x   x  
2 R1  x  
1


adPA x D x   x   bcPB x C  x   x  
Chinh phục olympic toán| 57
Bồi dưỡng học sinh giỏi

Áp dụng bổ đề trên thì ta được

    
max deg PA x D x   x  ,deg PB x C  x   x   2 deg  R1  x  

 deg  Q  x    deg  A  x  B  x  

       
 deg PA x D x   x   deg PB x C  x   x   max deg PA x D x   x  ,deg PB x C  x   x  
  
Từ đây ta suy ra deg PA x D x   x   deg PB x C  x   x   0. 
Từ đây thì ta có R1  x  , Q  x  là các đa thức hằng nên P  x  cũng là đa thức hằng, điều này
là vô lý do giả thiết bài toán nói đa thức P  x  khác đa thức hằng.
Vậy từ đây suy ra tồn tại nhiều nhất hai số thỏa m n điều kiện đề bài, nên từ đó ta suy ra
điều phải chứng minh.

Câu 56. Chứng minh rằng nếu đa thức f  x    x  có bậc n và nhận giá trị nguyên tại

TẠP CHÍ VÀ TƯ LIỆU TOÁN HỌC


n  1 giá trị nguyên liên tiếp từ a  a  n , a  thì f  x   , x  .
Lời giải
rước hết, ta cần có công thức nội suy Abel – Newton được phát biểu như sau:
Cho f  x    x  có bậc n và n số thực a1 , a2 ,..., an khi đó tồn tại duy nhất n  1 số thực
b0 , b1 ,..., bn sao cho:
f  x   b0  x  a1  x  a2  ...  x  an   b1  x  a1  x  a2  ...  x  an1   ...  bn1  x  a1   bn
Công thức này có thể chứng minh rất là đơn giản bằng phương pháp quy nạp Toán học.
Quay trở lại bài toán
Ta áp dụng công thức nội suy Abel – Newton cho n số thực ai  a  i  , i  1, n.
f  x   b0  x  a  1 x  a  2  ...  x  a  n   b1  x  a  1  x  a  2 
...  x  a  n  1  ...  bn1  x  a  1  bn
 f  a  1   bn 

 f  a  2   bn 1 .1! bn   bn 1 
 f  a  3   b .2! b .1! b   b .2! 
 n2 n1 n n2
Từ đây suy ra  .
 ........
 f  a  n    b .  n  1 ! 
 1

 f  a    b0 .n ! 
n
 x  a
Ta sẽ viết lại đa thức f  x  như sau f  x    k ! bn k  .
k 0  k 
 x  a   x  a  1 x  a  2  ...  x  a  k 
rong đó thì    , x  và bn k k !  , chứng minh
 k  k!
trên.

58 | Tạp chí và tư liệu toán học


Đa thức và số học

Vậy từ đó ta được f  x   , x  .

Câu 57. Tìm tất cả các đa thức P  x   x sao cho với mọi a , b mà a không là nghiệm

của P  x  thì P  a  P  a  b   P  b  .
MYTS 2016
Lời giải
Ta sẽ làm việc trên các số nguyên lớn hơn nghiệm thực lớn nhất của P  x  , nghĩa là ta
không cần quan tâm đến việc P     0.
Đế ý rằng P  x   c  const là một đa thức thỏa mãn yêu cầu đề bài, xét deg  P  x    1.
Ta nhận xét thấy.
Nếu P  x  là một đa thức thỏa mãn yêu cầu bài toán thì  P  x  cũng thỏa mãn yêu cầu đề bài
Không mất tính tổng quát, ta có thể giả sử hệ số cao nhất của P  x  là dương.
CHINH PHỤC OLYMPIC TOÁN

Ta có lim P( x )   nghĩa là từ một lúc nào đó, đa thức của ta chỉ nhận giá trị dương, lớn
x 

hơn hẳn P  1  .
P  x  1  P  1  P  x  1  P  x   P  1
heo đề bài thì ta có    .
P x P x
Đặt Q  x   P  x  1   P  x 
Trường hợp 1. deg  P  x    2 khi đó thì deg  Q  x    deg  P  x    1  1.
Q  x   P  1 Q  x   P  1
Khi đó thì  . Mặt khác, ta lại có lim  0.
P x x  P x
Hay từ một chỉ số x 0 nào đó trở đi thì
Q  x   P  1 Q  x   P  1
1  0, x  x0  Q  x   P  1  0, x  x0
P x P x
Nghĩa là một đa thức có deg Q  x   P  1   deg P  x   1  1 và có vô số nghiệm, vô lý.
Trường hợp 2. Ta có deg  P  x    1 thì ta đặt P  x   ax  b , a  0.
ax
Khi đó   b  0 hay P  x   ax , a  0 thử lại thấy thỏa mãn yêu cầu đề bài.
ax  b
Vậy tất cả các hàm số thỏa mãn yêu cầu đề bài là P  x   c  const và P  x   ax , a  0.

Chinh phục olympic toán| 59


Bồi dưỡng học sinh giỏi

Câu 58. Xét đa thức P  x   x n  an1 x n1  ...  a1 x  a0 với n  2, n  . Giả sử P  x  có n


nghiệm là x1 , x2 ,..., xn . Kí hiệu max  xi  là số lớn nhất trong các số x1 , x2 ,..., xn . Chứng
n
1
minh rằng P  x      2n2 n1 , x  max  xi  với   0 .
i 1 x  xi
VMF – Diễn đàn Toán học
Lời giải
n
Do P  x  bậc n và có n nghiệm là x1 , x2 ,..., xn nên P  x     x  xi 
i 1

Với x  max  xi  , i  1, n thì x  xi  0, i  1, n


Từ đó ta áp dụng bất đẳng thức Cauchy – Schwarz thì ta được:

 x  xi   
n
n n
1 n
1
P  x     P  x    .n n   nn 
i  1 x  xi i  1 x  xi i 1 x  xi

TẠP CHÍ VÀ TƯ LIỆU TOÁN HỌC


Do vậy ta cần chứng minh rằng

 x  xi     x  xi   
n n
n n

    2n  
n n  1 
2 n1 n
n n  2n 
i 1 x  xi i 1 x  xi
Ta sẽ chứng minh  x  xi     2nn1  x  xi  , i  1, n .
n

Tổng quát, với mọi số thực không âm x , y và n  2, n  .


n 2
n  n  1  n2 2
x  y   C nk x n  k y k   C nk x n k y k  x n  nx n1 y 
n
x y
k 0 k 0 2
n  n  1 n n2 2
2 x .x y  nx n 1 y  2 nx n 1 y
2
n  n  1
 n, n  2 nên từ đây ta suy ra được  x  xi     2nn1  x  xi  , i  1, n.
n
Do 2
2
Hay bất đẳng thức ban đầu được chứng minh.

Câu 59. Giả sử R là nghiệm của phương trình x n  a1 x n1  ...  an1 x  an  0 và đặt
n n
A   a j , B   ja j . Khi đó thì ta có A A  R B .
j 1 j 1

VMF – Diễn đàn Toán học


Lời giải
aj n
Đặt c j 
A
suy ra c j  0 và c
j 1
j  1.

Do hàm số y   ln  x  là hàm lõm trong khoảng từ  0,   nên theo bất đẳng thức Jensen
thì

60 | Tạp chí và tư liệu toán học


Đa thức và số học

  A  n   n A  n aj 
  ln  j   j
 c    ln  j jc    ln   j    ln f  R    ln 1  0
  R    j 1   j 1 R   j 1 R 
 n   n 
 c 
n
Suy ra j ln R j
 
c j ln  A   0 và  ln  A     c j    ln  R     jc j 
j 1  j 1   j 1 
1 n 1 n aj
Hay  a j  ln  A     ja j  ln  R   , do c j  , A  0.
A j 1 A j 1 A
Vậy nên ta được: ln A  ln R  A  R
A B A B
   
Hay từ đây ta được điều phải chứng minh.

Câu 60. Cho đa thức P  x  là đa thức monic bậc n  1 có n nghiệm thực là x1 , x2 ,..., xn
phân biệt và khác 0 . Chứng minh rằng:
 1
n1
CHINH PHỤC OLYMPIC TOÁN

1 1 1
  ...  
x 1 P  x 1  x 2 P   x 2  xn P  xn  x1x2 ...xn
VMF – Diễn đàn Toán học
Lời giải
Từ giả thiết bài toán thì ta suy ra P  x    x  x1  x  x2  ...  x  xn 
Ta bổ sung x0  0 và với mọi j  0 thì ta có:
n n n n
P  x    
j  1 i  1;i  j
 x  xi   x j P x j  x j    x
i  1;i  j
j  xi    x
i  0;i  j
j  xi 
Q x j  1, j  1, n
 n  
a xét đa thức Q  x   1    x  xi    n

        xi
n
i 1 Q 0  1  P 0  1  1
 i 1

Ta dễ thấy deg  Q  x    n nên ta áp dụng công thức nội suy Lagrange cho Q  x  tại n  1
n

n  x  x  i

điểm x i thì ta được Q  x    Q x j   i  0 ;i  j


n
.
j 0
 x
i  0 ;i  j
j  xi 
n n n

n   x  xi    x  x   1  Q  0  P  x 
x  x  i n i n

  Q 0 
i  0 ;i  j i  0 ;i  j
 i 1

x P  x  x 
n n
x P
 1   x x
j 1 n j 1
j j j j
i i
i 1 i 1

So sánh hệ số của x n ở hai vế thì ta được:

Chinh phục olympic toán| 61


Bồi dưỡng học sinh giỏi

 n

 1   
n1 n
 1   1 xi 
 1  Q  0   n    1 
n n1
n
1 1 
1    1 i 1

 
j  1 x j P x j
n

 xi  
j 1 x j P x j
n

 i x
n

 xi
i 1 i 1 i 1

Và từ đây ta có điều phải chứng minh.

n
xk
Câu 61. Cho n  2 và đa thức P  x  xác định bởi P  x    . Chứng minh rằng
k 0 k !

phương trình P  x   0 không có nghiệm hữu tỉ.


VMF – Diễn đàn Toán học
Lời giải
Nếu   là nghiệm của P  x  thì  cũng là nghiệm của phương trình:
n! k

TẠP CHÍ VÀ TƯ LIỆU TOÁN HỌC


x n  nx n1  ... 
x  ...  n !  0
k!
n!
  n  n n1  ...   k  ...  n !  0  1 
k!
Vì P  x  là đa thức hệ số nguyên có hệ số cao nhất là 1 và có nghiệm   thì  là số
nguyên.
Gọi p là một ước nguyên tố bất kì của n
k  k  k
Với k  1, n ta đặt rk  vp  k !  rk      2   ...   s  .
p p  p 
rong đó, s là số tự nhiên thỏa mãn p s  k  p s  1
k k k 1 p
 rk   2  ...  s  k.  k  rn  rk  rn  k
p p p 1  ps
n ! rn  k  1
 rn  rk  rn  k  1 
p
k!
n!
Lại có, do p n nên từ  1  p   p k  k  prn 1  k , k  1, n
k!
Do  1  nên p rn  1 n ! , mâu thuẫn do rn  vp (n !) nên n ! không chia hết cho p rn  1 .
Vậy điều giả sử là sai hay đa thức P  x  không có nghiệm hữu tỉ.

Câu 62. Cho p là một số nguyên tố. Tìm tất cả các đa thức f  x  với hệ số nguyên sao
cho với mọi số nguyên dương n , f  x  là ước của p n  1.
THTT Tháng 12 – 2014
Lời giải

62 | Tạp chí và tư liệu toán học


Đa thức và số học

Ký hiệu A là tập các ước số nguyên của p  1 . Khi đó tất cả các đa thức hằng
f  x   b , b  A đều thỏa mãn bài toán, vì n  *
,  pn  1  p  1 b .
Ta sẽ chứng minh đó cũng là tất cả các đa thức thỏa mãn bài toán.
Thật vậy. Phản chứng giả sử tồn tại đa thức f  x  mà deg f  x   1 thỏa mãn bài toán.
Với n  *
và giả sử q là ước số nguyên tố bất kỳ của f  n  .
Ta có f  n  q   f  n   n  q  n   q . Mà f  n  q nên từ đó ta có f  n  q  q .
Như vậy ta có pn  1 f  n  q , p n q  1 f  n  q  q .
Nên suy ra pn q  1   pn  1   p n  pq  1  q .


Từ các sự kiện trên ta suy ra p  q vì q f  n  , f  n  pn  1  q p n  1 và p q  1 q . 
Theo định lý Fermat ta lại có p q  p  mod q  , suy ra pq  1   pq  p   p  1 q .
Bây giờ ta xét các số tự nhiên n  *
mà n  1 p  1 . Ta biết số các số n thỏa mãn là vô hạn,
CHINH PHỤC OLYMPIC TOÁN

trong khi tập A ta xét là hữu hạn. Suy ra tồn tại số n mà f  n   A .


Mặt khác pn  1   p  1  B với
B  pn1  pn 2  p  1  n  mod p  1   1  mod p  1    p  1, B   1 .
Do f  n   A suy ra có ước số nguyên tố q của f  n  mà B q  p  1  q .
Mâu thuẫn với gỉa thiết quy nạp.

Câu 63. Cho số nguyên dương n và số nguyên tố p lớn hơn n  1 . Chứng minh rằng đa
x x2 xp
thức P  x   1    không có nghiệm nguyên.
n  1 2n  1 pn  1
Lời giải

Ta viết lại P  x   ap x p  ap 1 x p 1  a2 x 2  a1 x  a0 trong đó ak 


 n  1 2n  1 pn  1
kn  1
là các số nguyên ( chú ý là ta quy đồng hết nên trên tử chứa cả nhân tử dưới mẫu).
Do p là số nguyên tố lớn hơn n  1 nên  p , n   1 .
Vì thế tập A  1.n  1; 2n  1;  ; pn  1 là hệ thặng dư đầy đủ modulo p.
Do đó tồn tại duy nhất k  1, 2, , p sao cho kn  1  0  mod p  .
Ta lại có k  1, 0  kn  1  p 2 nên kn  1 p 2 . Như vậy với số k đó thì ak p ( vì nhân tử
kn  1 đ bị rút gọn), đồng thời các hệ số còn lại đều chia hết cho p kế cả các hệ số a0 , a1
nhưng không chia hết cho p 2 .
Bây giờ ta sử dụng lập luận đó để chứng minh phương trình trên không có nghiệm
nguyên.
Ngược lại giả sử rằng phương trình có nghiệ nguyên là c thì khi đó ta có

Chinh phục olympic toán| 63


Bồi dưỡng học sinh giỏi

P  c   0  apc p  ap 1c p 1  a2c 2  a1c  a0  0


Theo lập luận trên thì ta có ai p , i  0, 1, 2,  , p , i  k , k  0; 1 .
Do đó dẫn đến ak c k p ( do p là số nguyên tố).
Suy ra c k p  c p  ai c i p 2 với mọi i  2, 3,  p .
a cũng có a1c p 2 vì cả a1 , c đều chia hết cho p.
Vì P  c   0 p 2  a0 p 2 là điều vô lí.

Câu 64. Cho đa thức P  x   x 3  11x 2  87 x  m  m   . Chứng minh rằng với mọi m tồn
tại số nguyên n sao cho P  n  191 .
Lời giải
Ta có P  n   n  11n  87 n  m .
3 2

TẠP CHÍ VÀ TƯ LIỆU TOÁN HỌC


Đối với bài này ta sẽ chứng tỏ với n  A  1, 2,  ,191 là hệ đầy đủ modulo 191 thì ta có hệ
A*  P  1  , P  2  ,..., P  191  cũng là hệ đầy đủ modulo 191.
Như thế thì khi đó tồn tại n  A để P  n   0  mod 191  . Để chứng minh được A* là hệ đầy
đủ modulo 191 thì ta cần chỉ ra n1 , n2  A và n1  n2  mod 191  thì
P  n1   P  n2   mod 191  .
Thật vậy.
Giả sử ta có P  n1   P  n2   mod 191   27 P  n1   27 P  n2   mod 191  , vì  27, 191   1 .
  3n1  11   18.191n1  113  27 m   3n2  11   18.191n2  113  27 m  mod 191 
3 3

  3n1  11    3n2  11   n1  n2  mod 191 


3 3

Để hoàn thành được bước cuối, ta cần có bổ đề sau


Bổ đề. Giả sử p là SNT và p  2  mod 3  thì x 3  y 3  mod p   x  y  mod p  .
Chứng minh bổ đề
 Nếu x  0  mod p   x 3  0  y 3  0  y  0  mod p  . Vậy x  y  mod p  .
 Nếu x  0  mod p  thì dễ thấy x 3  0  mod p   y 3  0  mod p   y  0  mod p  .
Do nên theo định lí Fermat ta có
x p 1  1  mod p   x 3 k 1  1  mod p  , y p 1  y 3 k 1  1  mod p  .

Từ đó suy ra x 3 k 1  y 3 k 1  mod p   x   x 3   y   x 3   mod p   x  y  mod p  do  x , p   1


k k

Bổ đề được chứng minh nên ta cũng kết thúc bài toán.

64 | Tạp chí và tư liệu toán học


Đa thức và số học

Câu 65. Cho m là số nguyên dương, tìm số nghiệm của phương trình x 2  x  mod m  .
Lời giải
Ta giả sử phân tích m ra số nguyên tố là m  p11  p22  pkk trong đó p i là các số nguyên tố.
Khi đó phương trình x 2  x  mod m   x 2  x  mod pii  , i  1; k .
Ta có x 2  x  mod pii   x 2  x  0  mod pii   x  x  1   0  mod pii  .

Vì  x , x  1   1 nên phương trình tương đương x  0  mod pii  hoặc x  1  mod pii  .
heo định lí thặng dư rung hoa thì với mỗi bộ a1 , a2 ,  , ak thì hệ phương trình


x  ai mod pii  luôn có nghiệm duy nhất theo modulo m  p11  p22 pkk .

i  1; k
Do mỗi phương trình x 2  x  0  mod pii  luôn có hai nghiệm theo modulo pii nên

phương trình đ cho có tất cả 2 k nghiệm.


CHINH PHỤC OLYMPIC TOÁN

Câu 66. Cho p là số nguyên tố  p  3  . Xét đa thức


f  x    p  1  x p  2   p  2  x p  3  3x 2  2 x  1 .


Biết rằng hệ A  a1 , a2 , , ap  là một hệ thặng dư đầy đủ modulo p. Chứng minh rằng


khi đó hệ B  f  a1  , f  a2  , , f ap   cũng là một hệ thặng dư đầy đủ modulo p.
Lời giải
p  p  1
Xét x  1  f  1   p  1    p  2   2  1  p.
2
1 
Nếu x  1 thì ta có f  x  
x1     
 p  1 x p1  x p2   p  2  x p2  x p3  x  1 .
Do đó  x  1  f  x    p  1   x p 1  x p  2    p  2   x p  2  x p  3   2  x 2  x   x  1
Suy ra  x  1  f  x    p  1  x p 1   x p  2  x p  3  1  .
Áp dụng công thức tổng của cấp số nhân và quy đồng ta suy ra
 x  1 f  x    p  1  x  1 x p1   x p1  1  1  x  mod p   1 .
2

Ta cần chỉ ra bằng phản chứng


Giả sử B không phải là hệ thặng dư đầy đủ.
 
Khi đó tồn tại ai , a j  A  1  i  j  p  sao cho f  ai   f a j  m  mod p  .

m  ai  1 2  1  ai  mod p 

Từ  1  ta suy ra   m  0; p  1 .
 
2
m a j  1  1  a j  mod p 
Nếu m  0 thì từ hệ ta suy ra ai  a j  1  mod p  là điều vô lí.

Chinh phục olympic toán| 65


Bồi dưỡng học sinh giỏi

 
Nếu m  0  f  ai   f a j  0 . Nhưng f  1  p  ai  1, a j  1 .

 
Do ai , a j  A nên 1  ai , 1  a j  1 . Nên nhân phương trình trên và phương trình dưới của

hệ lần lượt với 1  a j và 1  ai ta thu được

   
2
m  ai  1 1  a j  m a j  1  1  ai   mod p   1  ai  1  a j  mod p   ai  a j ( vô lí).
2

Câu 67. Xét đa thức P  x   x 3  14x 2  2 x  1 . Chứng minh rằng tồn tại số nguyên dương
n sao cho với mọi số nguyên x ta có 101 P  P  P  x    x .
Thặng dư bình phương- Nguyễn Duy Liên
Lời giải
Ta sẽ chứng minh rằng x  y  mod 101   P  x   P  y  mod 101  .
Ta có P  x   P  y    x  y   x 2  xy  y 2  14x  14 y  2 

TẠP CHÍ VÀ TƯ LIỆU TOÁN HỌC


Do đó 4  P  x   P  y     x  y   2 x  y  14   3  y  29  
2 2

 
 x  y  mod 101 
Vì  4; 101   1 nên P  x   P  y  mod 4    .
  2 x  y  14   3  y  29    0  mod 101 
2 2

 
Xét  2 x  y  14   3  y  29    0  mod 101  .
2 2

 
 3 
Nếu  y  29, 101   1 thì khi đó     1 nên 101  1  mod 6  , vô lí.
 101 
Nếu 101  y  29   101  2 x  y  14   x  y  29 mod 101  .
Như vậy cả hai trường hợp ta có x  y  mod 101  . Nhận xét được chứng minh.

Xét 102 đa thức như sau P  x  , P  P  x   ,..., P P  ...P  x  ... . 


Theo nguyên lý Dirichlet phải tồn tại hai số m , n  1; 2;  ; 102 , m  n sao cho

   
P P  ...P  x  ...  P P ...P  x  ...  mod 101  .
m n

 
Từ nhận xét trên ta suy ra P P  ...P  x  ...  x  mod 101  với mọi số nguyên x.
mn

Câu 68. Cho tập S   p1 , p2 ,  , pk  là tập hợp k số nguyên tố phân biệt và P  x  là đa


thức với hệ số nguyên sao cho với mọi số nguyên dương n đều tồn tại p i trong S sao cho
pi P  n  . Chứng minh rằng tồn tại i sao cho pi P  n  ,n  *
.
Lời giải
Ta chứng minh phản chứng bằng cách sử dụng định lí thặng dư Trung Hoa.

66 | Tạp chí và tư liệu toán học


Đa thức và số học

Giả sử không tồn tại i sao cho pi P  n  ,n  *


.
Suy ra với mọi i  1, k , luôn tồn tại ai sao cho pi P  ai  .
x  ai  mod pi 

heo định lí thặng dư rung Hoa luôn tồn tại x sao cho  .

 i  1; k

 P  x   P  ai   mod pi 

Do đó ta có  .

 i  1; k
Hay ta có pi P  x  , i  1; k . Điều này mâu thuẫn với giả thiết.

Câu 69. Cho đa thức P  x   x 3  153x 2  111x  38 .


a) Chứng minh rằng trong đoạn  0; 3 2000  tồn tại ít nhất một số nguyên dương a sao cho
P  a  32000 .
CHINH PHỤC OLYMPIC TOÁN

b) Hỏi trong đoạn  0; 3 2000  có tất cả bao nhiêu số nguyên dương a sao cho P  a  chia hết
cho 32000 .
Lời giải
a xét phương trình đồng dư P  x   0  mod 32000  .

a để ý rằng P  x   0  mod 3   x 3  153x 2  111x  38  0  mod 3  .


 x 3  38  0  mod 3   x  1  mod 3  .
Đặt x  3y  1  0  y  31999  1  .
Vậy P  x   P  3 y  1  27  y 3  52 y 2  22 y  3 mod 32000  .

 y  3t
 Q  y   y 3  52 y 2  22 y  3  0  mod 31997  , suy ra   
, 1  t  31998  1 .
 y  3t  1
Nếu y  3t  1 thì phương trình y 3  52 y 2  22 y  3 không chia hết cho 9, nên không chia
hết cho 31997 .
Nếu y  3t thì y 3  52 y 2  22 y  3  3  9t 3  156t 2  22t  1  .
Nên suy ra P  x   0  mod 32000   G  t   9t 3  156t 2  22t  1  0  mod 31996  .

Bây giờ ta xét đa thức G  t   9t 3  156t 2  22t  1 .


Ta nhận thấy G  t   0  mod 3   22t  1  0  mod 3 .
rong đoạn  0; 3  phương trình 22t  1  0  mod 3  có nghiệm duy nhất t  2 , và có thêm
điều kiện là G '  2   22  1  mod 3  suy ra G '  2   0  mod 3  nên áp dụng định lí Hensel
thì phương trình G  t   0  mod 31996  có nghiệm duy nhất t0  0; 31996  .

Chinh phục olympic toán| 67


Bồi dưỡng học sinh giỏi

Với t  , t  1; 3 2000  , G  t   0 mod 3 1998  khi và chỉ khi tồn tại h  ,0  h  8 sao cho
t  t0  31996  h . Vậy phương trình đồng dư G  t   0  mod 31998  có đúng 9 nghiệm nguyên

thuộc đoạn 1; 31998  1 . Từ đó suy ra trong đoạn  1; 3 2000  có đúng 9 số a sao cho P  a 
chia hết cho 32000 .

Câu 70. Tìm tất cả các đa thức f với hệ số nguyên sao cho f  n  f  m   n m .
Iran TST
Lời giải
Ta dự đoán rằng đa thức thỏa mãn sẽ là ax k . Nên ta đặt f  x   x k . g  x  với g  0   0 .
Ta sẽ chứng minh g  x  là đa thức hằng.
Từ giả thiết f  n  f  m   n m , ta suy ra n k g  n  m k g  m   n m . Nên để phản chứng g  x 

TẠP CHÍ VÀ TƯ LIỆU TOÁN HỌC


không phải đa thức hằng thì ta có thể chọn n k g  n  m k g  m  nhưng không suy ra được n m
Từ đó có mâu thuẫn. Vấn đề ta chọn thế nào ?
Nếu chọn cả n k m k và g  n  m k thì dẫn đến n m , việc chọn như vậy không có ý nghĩa gì.
Vậy liệu ta có thể chọn n k m k và g  n  g  m  . Ta sẽ theo dõi lập luận sau.

Xét đa thức h  x    x  , h  0   0 . Ta sẽ chứng minh với mọi k tồn tại m, p  h  0  sao cho
p k h  m  . Thật vậy, giả sử h  x   h1  x  .h2  x  ...hn  x  , trong đó hi  x  bất khả quy và

hi  0   0 . Do đó thay vì chọn h  x  bất kì ta có thể chọn luôn h  x  bất khả quy.


Khi h '  x  nguyên tố cùng nhau với h  x  thì tho định lí Bézout ta có tồn tại hai đa thức
P , Q sao cho tồn tại số nguyên a ta có h.P  h ' Q  a . Khi đó theo định lí Schur luôn tồn tại
số nguyên tố p đủ lớn sao cho p  a , p h  n  , p không là ước của h '  n  .
Áp dụng định lí Hensel ta thấy p k h  m  .

Đặt f  x   x k g  x  , k  0, g  0   0 . Giả sử rằng g  x  khác đa thức hằng. Theo chứng minh


trên thì tồn tại số nguyên tố p sao cho p  g  0  và p k g  m  với m nguyên dương.

Dễ thấy p không là ước của g  p  nên theo định lí thặng dư Trung Hoa thì tồn tại n sao cho

n  m  mod p 
 k


n  p  mod g  p  

Khi đó p k g  n  và g  p  g  n  nên dẫn đến f  p  f  n  .

Do đó p n mà p g  n  nên p g  0  là vô lí.
Do đó g  x  là đa thức hằng. Vậy f  x   a.x k .

68 | Tạp chí và tư liệu toán học


Đa thức và số học

Câu 71. Cho a , b , c , d , e , f là các số nguyên dương. Giả sử rằng S  a  b  c  d  e  f là


ước của các số abc  def và ab  bc  ca  de  ef  fd . Chứng minh rằng S là hợp số.
IMO Shortlist 2005
Lời giải
Xét đa thức P( x )   x  a  x  b  x  c    x  d  x  e   x  f   Sx 2  Qx  R .
rong đó S  a  b  c  d  e  f ; Q  ab  bc  ca  de  ef  fd ; R  abc  def .
Vì theo giả thiết S Q ; S R nên S P  x  ,x  .
Ta có S P  d    d  a  d  b  d  c  .
Nếu S là số nguyên tố thì một trong 3 số d  a , d  b , d  c chia hết cho S . Nhưng điều đó là
vô lý vì S  max d  a , d  b , d  c .
Do đó S là hợp số.
CHINH PHỤC OLYMPIC TOÁN

Câu 72. Tìm tất cả các đa thức P với hệ số nguyên thỏa mãn
P  n |2557 n  213.2014, n  *

Thailand 2014
Lời giải
Dễ thấy P  n   1, n  *
và P  n   1, n  *
là những đa thức thỏa m n điều kiện bài
toán. Giả sử P là đa thức thỏa mãn bài toán và n0  *
: P  n0   2 .
Gọi p là ước nguyên tố của P  n0  .
Ta có: P  n0 |2557 n0  213.2014 và P  n0  p |2557 0  213.2014 .
n p

Do đó p P  n0  p   P  n0  2557 n0  2557 p  1  .
Mặt khác, vì p P  n0  2557 n0  213.2014 nên p  2, 3, 19, 53,71, 2557 .
Do đó p| 2557 p  1  . Hơn nữa, theo định lý Fermat nhỏ ta có p| 2557 p  2557  nên
p|2556 . Suy ra p  2, 3,71 , vô lý.
Vậy chỉ có hai đa thức P  n   1, n  *
và P  n   1, n  *
thỏa mãn bài toán.

Câu 73. Cho P là đa thức hệ số nguyên, có bậc n  1 và k là số nguyên dương bất kỳ.
Xét đa thức Q  x   P k  x  với P được tác động k lần. Chứng minh rằng có nhiều nhất
n số nguyên t sao cho Q  t   t .
IMO Shortlist 2006
Lời giải
Bổ đề. Nếu t là số nguyên thỏa mãn Q  t   t thì P 2  t   t .
Thật vậy. Ta có P  t   t P 2  t   P  t   P k  t   P k 1  t  P k 1  t   P k  t  .

Chinh phục olympic toán| 69


Bồi dưỡng học sinh giỏi

Mà P k  1  t   P k  t   P  t   t nên P  t   t  P 2  t   P  t     P k  t   P k 1  t 
Đặt d  P  t   t . Nếu d  0  P  t   t  P 2  t   P t   t .
Nếu d  0 . Giả sử i là chỉ số nhỏ nhất mà d    P i  t   P i 1  t   , 2  i  k khi đó
P i 1  t   P i  2  t   P i 1  t   P i  t 
Suy ra P i  t   P i  2  t  nên P 2  t   t .
Ngược lại nếu d  P  t   t  P 2  t   P  t     P k  t   P k 1  t  thì P k  t   t  kd  t , điều này
mâu thuẫn.
Quay lại bài toán, giả sử rằng có  n  1  số nguyên t1  t2    tn  tn  1 thỏa mãn
Q  ti   ti , 1  i  n  1
Khi đó theo bổ đề trên, ta có P 2  ti   ti , 1  i  n  1 .

 
Với mọi i , j thỏa mãn 1  i  j  n  1 ta có ti  t j P  ti   P t j P 2  ti   P 2 t j .  

TẠP CHÍ VÀ TƯ LIỆU TOÁN HỌC


 
Nên P  ti   P t j  t j  ti . Theo bất đẳng thức về giá trị tuyệt đối, ta có:

tn 1  t1  P  tn 1   P  t1   P  tn 1   P  tn   P  tn   P  tn 1   P  t2   P  t1   tn 1  t1


Do đó, tất cả các hiệu P  ti  1   P  ti  đều cùng dấu.
Giả sử tất cả các hiệu P  ti  1   P  ti  đều cùng dấu dương, khi đó
P  ti  1   P  ti   ti  1  ti , 1  i  n
Suy ra P  ti  1   ti  1  P  ti   ti , 1  i  n .
Do đó, đa thức P  x   x   P  t1   t1  có  n  1  nghiệm phân biệt t1  t2    tn  tn  1 , điều
này là vô lý vì P  x   x   P  t1   t1  là một đa thức bậc n.
ương tự cho trường hợp tất cả các hiệu P  ti  1   P  ti  đều cùng dấu âm.
Vậy ta có điều phải chứng minh.

Câu 74. Cho A là tập vô hạn các số nguyên dương. ìm tất cả các số nguyên dương n
thỏa mãn với mọi a là phần tử của A thì
1  a  a2  ...  an 1  a1!  a2!  ...  an!
Serbia MO 2010
Lời giải
Xét một số n thỏa mãn yêu cầu bài toán.
a đặt: P  x   1  x  x 2  ...  x n , Q  x   1  x 1!  x 2!  ...  x n!
Theo kết quả trên, từ giả thiết P  a  Q  a  với vô hạn a ta suy ra P  x  Q  x  .
Ta thấy rằng x n 1  1  mod P  x  
Do vậy, gọi t i là số dư trong phép chia i ! cho n  1 với i  n thì

70 | Tạp chí và tư liệu toán học


Đa thức và số học

Q  x   x 0  x t1  ...  x tn  S  x   mod P  x  
Vì các t i đều không vượt quá n  1 và cả P  x  , S  x  đều là tổng của n  1 đơn thức nên để
P  x  Q  x  thì P  x   S  x 
Như vậy, ta cần có 0, 1!,..., n ! lập thành một hệ thặng dư đầy đủ theo mod n  1
rước tiên, trong n  1 số trên chỉ có một số chia hết cho n  1 là 0 nên ta suy ra n ! không
là bội của n  1, điều này chỉ xảy ra khi n  1 là số nguyên tố.
Với n  2 thì n !  1  n  mod n  1  theo định lý Wilson
Suy ra  n  1  !  1  mod n  1  nên hệ 0, 1!,..., n ! không lập thành một hệ thặng dư đầy đủ
mod n  1.
Và như vậy ta phải có n  2, kiểm tra trực tiếp ta thấy n  1 và n  2 đều thỏa mãn.
Vậy tất cả các giá trị của n thỏa mãn yêu cầu bài toán là n  1 và n  2.
CHINH PHỤC OLYMPIC TOÁN

Câu 75. Cho P , Q là hai đa thức hệ số nguyên không âm, khác đa thức hằng. Xét dãy số
 Q  n  , n  1 . Chứng minh rằng tồn tại vô hạn số nguyên tố p thỏa mãn:
P n
xn  2016
ứng với mỗi p , tồn tại số nguyên dương m sao cho p xm .
Ukraine 2016
Lời giải
 Q  n   2016  Q  n   2016  R n
P n P n  P  1 P  1 P n P  1
Ta có xn  2016  2016  2016  2016
rong đó R  Q  2016 P 1 là một đa thức hệ số nguyên, khác đa thức hằng và R  0   0 .
Theo định lý Schur, tồn tại vô hạn số nguyên tố p , p  max R  0  , 2016 thỏa mãn ứng với
mỗi p , tồn tại số nguyên dương n sao cho p R  n  .
Nếu p n thì p|R  n   R  0  nên p|R  0  , vô lý nên  p , n   1 .
Mặt khác  p , p  1   1 nên theo định lý thặng dư Trung Hoa, tồn tại số nguyên m sao cho
m  n  mod p  ; m  1  mod p  1 
Vì p|R  n  và m  n  mod p  nên p|R  m  .
Hơn nữa m  1  mod p  1  nên P  m   P  1   mod  p  1   do đó theo định lý Fermat nhỏ ta
P m P 1
có p|2016  2016 . Suy ra p xm .
Vậy ta có điều phải chứng minh!

Câu 76. Tìm tất cả các đa thức P hệ số nguyên thỏa mãn P  p  2 p  p , với mọi số nguyên
tố p .

Lời giải
Dễ thấy nếu P là đa thức hằng thì P  1 thỏa m n điều kiện bài toán.

Chinh phục olympic toán| 71


Bồi dưỡng học sinh giỏi

Xét P không phải là đa thức hằng. Theo định lý Schur, tồn tại vô hạn số nguyên tố p thỏa
mãn: Ứng với mỗi p tồn tại số nguyên n sao cho p P  n  .
Nếu p n thì p P  n   P  p  nên p P  p  2 p  p , vô lý nên  p , n   1 .
heo định lý Dirichlet về số nguyên tố, ta có thể chọn số nguyên k sao cho q  n  kp là số
nguyên tố. Khi đó p|P  q   P  n  nên p|P  q |2 q  q  2 n kp   n  kp  . Kết hợp định lý
Fermat nhỏ suy ra p|2 n k  n . Đặt h  ordp  2  .
Nếu có các số nguyên k1 , k2 thỏa mãn p|2 n k1  n và p|2 n k2  n thì k2  k1  mod h  .
Do đó, ta chỉ có thể chọn k theo một lớp thặng dư nào đó đối với modul h .
Nhưng, theo định lý Dirichlet, ta có thể chọn k sao cho số nguyên tố q  n  kp nguyên
tố cùng nhau với h. Khi đó, vì h| p  1 nên  n  k , h   1 , nghĩa là ta có   h  cách chọn lớp
thặng dư đối với modul h cho k. Vô lý .
Vậy chỉ có P  1 là những đa thức thỏa mãn bài toán.

TẠP CHÍ VÀ TƯ LIỆU TOÁN HỌC


Câu 77. Cho P  x  , Q  x  là các đa thức hệ số nguyên khác đa thức hằng. Giả sử rằng đa
thức P  x  .Q  x   2009 có ít nhất 25 nghiệm nguyên phân biệt. Chứng minh rằng bậc
của mỗi đa thức P  x  , Q  x  đều không nhỏ hơn 3.
Belarus 2009
Lời giải
Giả sử T  ai , 1  i  25, i   là tập hợp gồm 25 nghiệm nguyên của đa thức
P  x  .Q  x   2009 .
Khi đó P  ai  Q  ai   2009, 1  i  25 .
Suy ra P  ai  2009; Q  ai  2009, 1  i  25 .
Vì 2009  7 2  41 nên 2009 có tất cả 12 ước số nguyên.
Do đó, mỗi số P  a1  , P  a2  ,  , P  a25  sẽ bằng với 1 trong 12 ước số nguyên của 2009, và
theo nguyên lý Dirichlet, phải có ít nhất 3 trong số 25 số P  a1  , P  a2  ,  , P  a25  có giá trị
bằng nhau. Không mất tổng quát, giả sử P  a1   P  a2   P  a3   m .
Khi đó, đa thức P  x   m có ít nhất 3 nghiệm phân biệt nên có bậc không nhỏ hơn 3, và do
đó, đa thức P  x  có bậc không nhỏ hơn 3.
ương tự, đa thức Q  x  cũng có bậc không nhỏ hơn 3. a có điều phải chứng minh.

72 | Tạp chí và tư liệu toán học


Đa thức và số học

Câu 78. Gọi d  n  là ước nguyên tố nhỏ nhất của số nguyên n , với n  1, 0, 1 và ta kí
hiệu d  1   d  0  , d  1   0. Tìm tất cả các đa thức P  x  với hệ số nguyên thỏa mãn
P n  d n  n  d  P n
Turkey MO 2014
Lời giải
rước tiên, ta có một đánh giá chung cho hai vế của đẳng thức ở đề bài.
a đ biết d  n  là ước nguyên tố nhỏ nhất của n thì d  n   n hoặc d  n   n.
Do vậy ta dự đoán, nếu P  x  có bậc lớn hơn 1 thì chọn n đủ lớn sẽ có
P n  d n  n  d  P n
Thật vậy, giả sử P  x  có bậc không nhỏ hơn 2.
Chọn n  q nguyên tố thì d  n   q thay vào phương trình ban đầu ta được
P  2q   q  d  P  q    q  d  P  q    q  P  q 
CHINH PHỤC OLYMPIC TOÁN

P  2q  q
Suy ra  1
P q  P q 
deg  P  x  
Cho q tiến ra vô cùng, thì rõ ràng vế trái tiến về 2 , còn vế phải tiến đến 1 nên điều
này là vô lý.
Dễ thấy, P  x  là hằng số thì không thỏa mãn yêu cầu bài toán.
Cuối cùng, ta xét P  x  có bậc bằng 1 thì ta viết P  x   bx  c ; b , c  , b  0.
Thay n  q là số nguyên tố một lần nữa, thì ta được:
2bq  c  q  d  bq  c 
Ta thấy rằng q  d  bq  c   q nên chọn q đủ lớn thì ta suy ra phải có b là số dương, hay
b  1.
Mặt khác ta lại có  2b  1  q  c  d  bq  c   bq  c nên b  1.
Từ đấy ta phải có b  1, thay lại vào phương trình đề bài thì ta được
n  d n  c  n  d n  c 
Suy ra d  n  c   d  n   c , n  
.
Từ đây ta suy ra với mọi n đủ lớn nếu n  c là số nguyên tố thì n cũng là số nguyên tố và
ngược lại. Ta chứng minh chỉ có c  0 thỏa mãn tính chất này.
Thật vậy, giả sử c  0 thì ta chọn được A  2  c
Xét r là số nguyên tố nhỏ nhất lớn hơn A ! 2
Khi đó r  A  c  2 do tất cả các số từ A  2 đến A  c  2 đều là hợp số.
Ta cần có r  c và r  c đều là số nguyên tố, điều này không thể xảy ra vì một trong hai số
này là r  c và là hợp số theo cách chọn r .
Vậy chỉ có c  0 thỏa m n, khi đó thì P  x   x , thử lại thấy thỏa mãn yêu cầu bài toán.

Chinh phục olympic toán| 73


Bồi dưỡng học sinh giỏi

Vậy tất cả các đa thức thỏa m n điều kiện bài toán là P  x   x.

Câu 79. Tìm tất cả các số nguyên dương k thỏa mãn tồn tại đa thức f  x  với các hệ số
đều nguyên, có bậc lớn hơn 1 sao cho với mọi số nguyên tố p và mọi số tự nhiên a , b
mà p ab  k thì p f  a  f  b   k .
Lời giải
Với k  1 thì không khó để thấy rằng ta chọn được f  x   x n với n  1 là đa thức thỏa
mãn.
Ta chứng minh các giá trị còn lại của k không thỏa mãn yêu cầu bài toán.
Giả sử có k  1 thỏa mãn yêu cầu bài toán.
Xét đa thức f  x  bậc n bất kì, giả sử f  x   an x n  an1 x n 1  ...  a1x  a0
k
Ta lập đa thức g  x   x n f    a0 k 0 x n  a1 k 1x n1  ...  an k n

TẠP CHÍ VÀ TƯ LIỆU TOÁN HỌC


x
Với mỗi số nguyên dương a và số nguyên tố p mà  a , p   1 thì ta chọn được b
sao cho ab  k  mod p  .
Khi đó thì ta thấy g  a   an f  b   mod p 
Như vậy, ta có với mọi số nguyên tố p mà  a , p   1 thì
g  a  f  a   an f  b  f  a   kan  mod p 
Vì tập các số nguyên tố là vô hạn nên ta có thể cố định chọn p đủ lớn để

 
p  max g  a  f  a   kan  3n , a  1, 3n

Khi đó thì ta suy ra g  a  f  a   kan  0, a  1, 3n


Mặt khác, đa thức f  x  g  x   kx n chỉ có bậc 2n mà lại nhận đến 3n giá trị phân biệt là
nghiệm nên đồng nhất với đa thức không.
Kết hợp với f  x  có bậc n ta suy ra g  x  phải là hằng số.
Từ công thức của g  x  ta suy ra g  x   an k n .
Ta cần có g  1  f  1   k nên g  1  k tuy nhiên điều này không thể xảy ra khi n  2 hay vô
lý.
Vậy k  1 là số nguyên dương duy nhất thỏa mãn yêu cầu bài toán.

Câu 80. Cho P là đa thức hệ số nguyên thỏa mãn P  0   0 và  P  0  , P  1 ,...  1 .


Chứng minh rằng có vô hạn số n sao cho  P  n   P  0  , P  n  1   P  1  ,   n .
USA TST 2010
Lời giải
Từ điều kiện P  0   0,  P  0  , P  1  ,...   1 suy ra P khác đa thức hằng.

74 | Tạp chí và tư liệu toán học


Đa thức và số học

Không mất tính tổng quát, giả sử P '  1   0 .


Ta chứng minh rằng nếu p là một số nguyên tố bất kỳ sao cho p không là ước của P '  1  thì
n  p k thỏa điều kiện bài toán.


Vì p k |P  p k  i   P  i  với mọi i nên p k P  p k   P  0  , P  p k  1   P  1  , . 
Mặt khác theo nhận xét trong chứng minh bổ đề Hensel, ta có
P  p k  1  P  1   P '  1  p k  mod p k  1 
Mà P '  1  không chia hết cho p nên P  p k  1   P  1 không chia hết cho p k  1 .
Cuối cùng ta chỉ ra rằng, không có số nguyên tố p  q là ước số của
P  p k   P  0  , P  p k  1  P  1 ,
Giả sử ngược lại với mọi i, ta có q|P  p k  i   P  i  .
Mặt khác ta cũng có q|P  q  i   P  i  suy ra P  i  ap k  bq   P  i   mod q  với mọi số
CHINH PHỤC OLYMPIC TOÁN

nguyên a , b . Vì  p k , q   1 nên tồn tại các số nguyên a, b sao cho ap k  bq  1 .


Do đó q|P  i  1   P  i  .
Mà P  0   0 nên q P  i  với mọi i , mâu thuẫn với giả thiết.

 
Vậy p k  P  p k   P  0  , P  p k  1  P  1 , . a có điều phải chứng minh!

Câu 81. Cho p là số nguyên tố và P  x  là các đa thức bậc d hệ số nguyên thỏa mãn
 P  0   0, P  1   1
 Với mọi số nguyên dương n thì số dư trong phép chia P  n  cho p là 0 hoặc 1 .
Chứng minh rằng d  p  1 .
Italian Proposal to 1997 IMO
Lời giải
Giả sử d  p  2 . Áp dụng công thức nội suy Lagrange với  p  1  mốc nội suy
xi  i , i  0, p  2 ta được
p2 p2 p2
x j
P x   P i   P  p  1    P  i    1  C pi 1
p i

i 0 j 0 i j i 0
j i

Do p nguyên tố nên C pi 1   1   mod p  , i  0, p  2


i

p2 p 1
 P  p  1   P  i   mod p    P  i   0  mod p 
i 0 i 0
p 1
Nhưng từ 2 điều kiện ở giả thiết ta lại có  P  i   k  mod p  với k  1, 2, , p  1 .
i 0

Điều này mâu thuẫn, suy ra d  p  1 .

Chinh phục olympic toán| 75


Bồi dưỡng học sinh giỏi

Câu 82. Chứng minh rằng không tồn tại đa thức P hệ số thực deg P  n  1 sao cho
P  m  là số nguyên tố với mọi số nguyên dương m .
Lời giải
Giả sử tồn tại đa thức P thỏa mãn điều kiện đề bài.
Trước hết, ta chứng minh n ! P  x   x .
n n
x j
Thật vậy, theo công thức nội suy Lagrange, ta có: P  x    P  i   .
i 0 j 0 i j
ji

P  i  Cni  1
ni
n
Nên n ! P  x   x  x  1 x  n    x .
i 0 xi
Chọn 2 số nguyên tố p , q , p , q  n sao cho P  a   p , P  b   q , a , b  *
.
Theo định lý thặng dư Trung Hoa thì tồn tại c  *
sao cho

TẠP CHÍ VÀ TƯ LIỆU TOÁN HỌC


c  a  mod q  n ! P  c   n ! P  a   0  mod p 
   n ! P  c   0  mod pq 
c  b  mod q  n ! P  c   n ! P  b   0  mod q 
Điều này vô lý vì P  c  là số nguyên tố và  n !, pq   1 .
Vậy ta có điều phải chứng minh.

Câu 83. Cho n  , n  3 và đa thức f  x   x n  an1 x n1  a1 x  a0  x  thỏa mãn a0


chẵn, ak  an  k chẵn với mọi k  1, n  1 .
Giả sử thêm rằng tồn tại hai đa thức g  x  , h  x   x thỏa mãn deg g  deg h , mọi hệ
số của h  x  đều lẻ và f  x   g  x  .h  x  , x .
Chứng minh rằng f  x  có ít nhất một nghiệm nguyên.
Romani TST 2007
Lời giải
Giả sử deg g  j ,deg h  k , 1  j  k , j  k  n và
g  x   b0  b1 x  b j x j , h  x   c 0  c 1x  c k x k

 
Ta có f  x   b0  b1 x  b j x j  c 0  c 1 x  c k x k  . Đồng nhất hệ số, ta có
b0  b1  b j 1  a j 1 và b1  b j  ak  1
Giả sử j  1  0 khi đó theo giả thiết a j 1  ak  1 chẵn nên b0  b j  1  mod 2  .
Mà a0 chẵn nên c 0 chẵn (mâu thuẫn giả thiết). Do đó j  1 mà b j  1 nên g  x  có
nghiệm nguyên. Suy ra f  x  cũng có nghiệm nguyên.
Vậy ta có điều phải chứng minh.

76 | Tạp chí và tư liệu toán học


Đa thức và số học

Câu 84. Cho đa thức f  x  monic, hệ số nguyên, bất khả quy và f  0  không phải là số
chính phương. Chứng minh rằng g  x   f  x 2  cũng là đa thức bất khả quy.
Romani TST 2003
Lời giải
Giả sử ta có phân tích g  x   f  x 2   p  x  .q  x  với p  x  , q  x  là 2 đa thức monic có hệ số
nguyên và bậc không nhỏ hơn 1.
Gọi  là một nghiệm ( thực hoặc phức ) của f  x  thì p    .q     f     0 . Không mất
tổng quát, giả sử p    0 .
k k
Đặt p  x    ai x i , ai  thì a  i
i
0.
i 0 i 0

Do đó, tồn tại các đa thức hệ số nguyên t , u thỏa mãn t       u     0 .


CHINH PHỤC OLYMPIC TOÁN

Do f là đa thức bất khả quy và deg u  deg f nên theo định lý Bézout, tồn tại số nguyên
m và hai đa thức hệ số nguyên s, r sao cho s  x  u  x   r  x  f  x   m , x  .

s    u    
2
s  u 
Suy ra s    u     m nên      .
m m2
Đặt  1 ,  2 ,  ,  n là các nghiệm của đa thức f thì
s   1  t   1  s   2  t   2  s   n  t   n 
2 2 2 2 2 2

1 2  n 
m2 n
là bình phương của một số hữu tỉ.
Mặt khác f  0    1 2  n là số nguyên nên f  0  là số chính phương, trái giả thiết.
Vậy từ đó suy ra điều phải chứng minh!

Câu 85. Cho đa thức hệ số nguyên f  x   an x n  an1x n1  a1x  a0 thỏa mãn điều kiện
a0  a1  a2  an và a0 là số nguyên tố thì f  x  bất khả quy.
Tiêu chuẩn Perron
Lời giải
Giả sử f  x  có nghiệm z thỏa z  1 thì a0  an zn  an1 zn1  a1 z  a1  an , mâu
thuẫn. Do đó tất cả các nghiệm của f đều có module lớn hơn 1.
Giả sử f  x   P  x  .Q  x  với P , Q là hai đa thức hệ số nguyên có bậc không nhỏ hơn 1.
Vì a0  f  0   P  0  Q  0  mà a0 là số nguyên tố nên P  0   1 hoặc Q  0   1 .

Không mất tính tổng quát ta giả sử P  0   1 .


Gọi b là hệ số cao nhất của P và z1 , z2 ,  , zk là tất cả các nghiệm của P .

Chinh phục olympic toán| 77


Bồi dưỡng học sinh giỏi

1
Khi đó z1 z2  zk   1 , mâu thuẫn vì tất cả các nghiệm của P cũng là nghiệm của f .
b
Vậy đa thức f  x  bất khả quy.

Câu 86. Tìm tất cả các đa thức P  x  , Q  x  hệ số nguyên thỏa mãn với dãy số  xn  xác
định bởi x0  2014, x2 n 1  P  x2 n  , x2 n 2  Q  x2 n1  , n  thì mỗi số nguyên dương m là
ước của một số hạng khác 0 nào đó của  xn  .
Việt Nam TST 2014
Lời giải
Ta sẽ chứng minh rằng nếu các đa thức P  x  , Q  x  thỏa mãn đề bài thì chúng đều có bậc
bằng 1. Thật vậy:
Xét trường hợp một trong hai đa thức P  x  , Q  x  là đa thức hằng.

TẠP CHÍ VÀ TƯ LIỆU TOÁN HỌC


 1 Nếu P  x   a , x  thì  xn  có dạng:
x0  2014, x1  a , x2  Q  a  , x3  a ,...
Và dễ thấy mọi số hạng của dãy chỉ nhận một trong ba giá trị 2014, a , Q  a  .
2 Nếu Q  x   a , x  thì  xn  có dạng:
x0  2014, x1  P  2014  , x2  a , x3  P  a  ,...
Và dễ thấy mọi số hạng của dãy chỉ nhận một trong bốn giá trị 2014, P  2014  , a , P  a  .
Cả hai điều này đều không thỏa mãn điều kiện đề bài do mỗi số nguyên dương m phải là
ước của một số hạng khác 0 nào đó của dãy số  xn  .
Tiếp theo, nếu một trong hai đa thức P  x  , Q  x  có bậc lớn hơn 1.
Không mất tính tổng quát, ta giả sử đó là Q  x  thì rõ ràng khi đó Q  P  x   cũng có bậc
lớn hơn 1. Ta thấy nếu R  x  là đa thức có bậc lớn hơn 1 thì với mọi k  0 lớn tùy ý, tồn
tại x có giá trị tuyệt đối đủ lớn sau cho R  x   k x , điều này là dễ thấy do khi x   thì
R x
ta có giới hạn lim  
x  x
Ta sẽ chứng minh rằng tồn tại N đủ lớn sao cho Q  P  x    P  x   x , x  N .
Ta chỉ cần xét hai trường hợp. Nếu P  x  là bậc 1 thì dễ thấy tồn tại k đủ lớn sao cho
k P  x   x , với mọi x  .
Suy ra tồn tại N đủ lớn sao cho:
Q  P  x     k  1  P  x   P  x   k P  x   P  x   x với mọi x  N
Nhận xét được chứng minh.

78 | Tạp chí và tư liệu toán học


Đa thức và số học

Theo giả thiết thì trong dãy số đã cho, phải tồn tại số hạng xi lớn tùy ý và rõ ràng ta cũng
phải có j  0 sao cho x2 j  N  1 và x2 j có giá trị tuyệt đối lớn nhất trong 2 j số hạng đầu
tiên của dãy  xn  .
Thật vậy, ta thấy rằng, tồn tại vô số số hạng x2 j thỏa mãn điều kiện

x2 j  max  xk  , k  0, 2 j
Gọi T là tập hợp các chỉ số thỏa mãn. Nếu như trong các số hạng như thế, không có số
hạng nào thỏa mãn x2 j  N  1 thì với mọi t  T
Ta có xi  xt  N  1 với mọi i  t  T .
Tuy nhiên, do T vô hạn nên điều giả sử ở trên là vô lý và nhận xét được chứng minh.
Với x2 j là số hạng thỏa mãn điều kiện trên, chọn m  x2 j  2  x 2 j thì ta thấy

    x
m  Q P x2 j     x
 Q P x2 j  
 P x2 j  x2 j và x2 n  2  Q  x2 n  1   x2 n  1
CHINH PHỤC OLYMPIC TOÁN

2j 2j

Do đó, trong 2 j  1 số hạng đầu tiên của dãy, không có số hạng nào chia hết cho m.
Mặt khác ta có:

x2 k  2  x2 k  Q  P  x2 k    Q  P  x2 k  2     P  x   P  x   x
2k 2 k 2 2k  x2 k 2   m

Và tượng tự thì x2 k  3  x2 k 1   P  x2 k  2   P  x2 k   x  x   m
2j 2 j 2

Từ đây suy ra với k  j thì x2 k  2  x2 k và x2 k  3  x2 k  1 đều chia hết cho m , tuy nhiên x2 j  1 và
x2 j  2 đều không chia hết cho m nên x k không chia hết cho m với k  2 j  2.
Do đó, trong dãy đã cho không có số hạng nào chia hết cho m.
Điều mâu thuẫn này cho ta thấy nhận xét ban đầu là đúng và deg  P  x    deg Q  x    1.
Đặt P  x   ax  b với a  1 và Q  x   cx  d với a , b , c , d  và ab  0 thì ta có:
x2 n1  ax2 n  b
 , n  0
x2 n 2  cx2 n1  d
Suy ra x2 n 2  cax2 n  bc  d và x2 n  3  cax2 n  1  ad  b với mọi n  0.
Cả hai dãy này đều có công thức truy hồi dạng y n  1  ky n  h với k  ac , h  .
 kn  1 
Giả sử k  1 thì công thức tổng quát của dãy này là yn  k y0  h   với mọi n.
n

 k 1 
Rõ ràng nếu k  1 thì dãy số tương ứng không thỏa mãn, ta xét k  1.
 Nếu h  0 thì ta có yn  k n y0 , rõ ràng không thỏa mãn điều kiện.
 kn  1 
 Nếu h  0 thì do  k ,   1 với mọi n nên giả sử t là số mũ lớn nhất mà k h
t

 k 1 
thì các số nguyên dương có dạng k s với s  t đều không là ước của bất cứ số hạng
nào của dãy, không thỏa mãn. Từ đây, suy ra k  1 hay ac  1.

Chinh phục olympic toán| 79


Bồi dưỡng học sinh giỏi

Cuối cùng, ta chỉ cần xét hai trường hợp:


Trường hợp 1. Nếu P  x   x  a và Q  x   x  b với a , b  thì bằng quy nạp, ta chứng
minh được x2 k  2014  k  a  b  và x2 k  1  2014  a  k  a  b 
Dễ thấy rằng nếu a  b  0 thì dãy này không thỏa mãn.
Nếu như a  b  0 thì gọi S , T , R lần lượt là tập hợp các ước nguyên của a  b , 2014 và
2014  a.
Ta xét các trường hợp sau:
Với m  S thì giả sử a  b chia m dư t với t  0 do đó khi k chạy qua một hệ thặng dư
đầy đủ mod m .thì tồn tại một số hạng của dãy chia hết cho m và số lượng các số hạng
như thế là vô hạn. Rõ ràng số các số hạng bằng 0 của dãy là hữu hạn, không quá hai nên
tồn tại số hạng khác 0 của dãy chia hết cho m.
Với m  S và m  T , R thì dãy số tương ứng không thỏa mãn.

TẠP CHÍ VÀ TƯ LIỆU TOÁN HỌC


Với m  S van m  T hoặc m  R thì tương ứng, mọi số hạng có chỉ số chẵn hoặc mọi số
hạng có chỉ số lẻ của dãy đều chia hết cho m và dễ thấy, tồn tại số hạng khác 0 của dãy
chia chết cho m. Do đó, các số a , b phải thỏa mãn  S \ T    S \ R    hay mỗi ước của
a  b phải là ước của 2014 hoặc là ước của 2014  a.
Trường hợp 2. Nếu P  x   x  a và Q  x   x  b thì cũng có lập luận tương tự vì các số
hạng của dãy tương ứng khi đó là x2 k  2014  k  a  b  và x2 k  1  2014  a  k  a  b  .
Điều kiện của a , b là a  b  0 và mỗi ước của a  b phải là ước của 2014 hoặc là ước của
a  2014.
Vậy tất cả các đa thức P  x  , Q  x  cần tìm là:
 P  x   x  a , Q  x   x  b trong đó a , b  thỏa mãn a  b  0 và mỗi ước của a  b
phải là ước của 2014 hoặc là ước của a  2014.
 P  x   x  a , Q  x   x  b trong đó a , b  thỏa mãn a  b  0 và mỗi ước của
a  b phải là ước của 2014 hoặc là ước của a  2014.

Câu 87. Chứng minh rằng không tồn tại đa thức


P  x   an x n  an1 x n1  ...  a1 x  a0  x  bậc n  1
sao cho P  0  , P  1  ,... đều là số nguyên tố.
Lời giải
Giả sử tồn tại đa thức thỏa mãn yêu cầu bài toán.
Khi đó, a0  P  0  là số nguyên tố.
Mặt khác, ta có a0 P  ka0  , k  0, .
Nhưng vì P  ka0  là số nguyên tố nên P  ka0   a0 , k  0.

80 | Tạp chí và tư liệu toán học


Đa thức và số học

Điều này suy ra đa thức Q  x   P  a0 x   a0 có vô hạn nghiệm.


Dẫn đến Q  x   0 hay P  a0 x   a0 , mâu thuẫn với việc đa thức P  x  có bậc ít nhất là 1.
Vậy giả sử phản chứng là sai nên ta suy ra không tồn tại đa thức thỏa mãn yêu cầu bài
toán.

Câu 88. Cho đa thức P  x   x n  an1 x n 1  ...  a1 x  a0 , P  x   x  với a0 chẵn và an  k  ak


chẵn, với mọi k  1, n  1. Giả sử P  x   Q  x  R  x  , với Q  x  , R  x  là các đa thức hệ số
nguyên khác hằng, deg  Q  x    deg  R  x   và tất cả các hệ số của R  x  đều lẻ. Chứng
minh rằng, đa thức P  x  có nghiệm nguyên.
Lời giải
Định nghĩa: P  x   x n  an1 x n1  ...  a1 x  a0 với ai  ai  mod 2  , i  1, n.

Khi đó thì ta có P  x   Q  x .R  x 


CHINH PHỤC OLYMPIC TOÁN

Đặt deg  Q  x    r ,deg  R  x    s , r  s và Q  x   x r  ...  b1 x  b0 .


Khi đó nếu r  1 thì
 
x n  an1 x n1  ...  a1 x  a0  x r  br 1 x r 1  ...  b1 x  b0  x s  x s 1  ...  x  1 

 ar  1  br  br 1  ...  b1  b0
Đồng nhất hệ số của x r  1 và x s 1 thì ta được 
 as  1  br  br 1  ...  b1  1
Vì ar  1  as 1 nên b0  1 hay Q  0  là số lẻ, vì R  0  là số lẻ nên dẫn đến P  0  là số lẻ, mà
P  0   a0 là số chẵn, điều này dẫn đến vô lý. Vậy r  1 chứng tỏ P  x  có nghiệm nguyên.

Câu 89. Chứng minh rằng với mọi số nguyên dương n thì đa thức P  x    x 2  x   1
2n

là đa thức bất khả quy trên .


Romanian Team Selection Test 1998
Lời giải
Với n  0 dễ dàng kiểm tra là đúng. Do đó ta giả sử n  1. Ta liên kết mỗi đa thức
G  x   an x n  an1 x n1  ...  a1x  a0  x 
Với một đa thức G  x   an x n  an1 x n1  ...  a1 x  a0  2 x
Với các hệ số lấy trong trường 2 modulo 2.
2n1 2 n 2

    x 2  x   1   x 2  x   1  ...   x 2  x   1


2n 2 2
2 2n
Mà do x  x  1
2
   

 
2n
Nên suy ra P  x   x 2  x  1

Bây giờ giả sử P  x  khả quy, tức là có thể phân tích được P  x   G  x  H  x  với

Chinh phục olympic toán| 81


Bồi dưỡng học sinh giỏi

G x , H x  x.


Từ đây ta suy ra P  x   G  x .H  x  , lại vì x 2  x  1 là đa thức bất khả quy trên 2 nên suy

   
p 2n  p
ra G  x   x 2  x  1 , H  x   x 2  x  1 , 1  p  2n  1

G  x    x 2  x  1  p  2U  x 

Điều này dẫn đến 
H  x    x 2  x  1
2n  p

  2V  x 

Với U  x  ,V  x  là các đa thức hệ số nguyên. Gọi  là một nghiệm của phương trình
x 2  x  1.
Thay x bởi  trong đẳng thức:

P  x    x 2  x   1   x 2  x  1  2U  x    x 2  x  1   2V  x  
n n
2 p 2 p

  
1

TẠP CHÍ VÀ TƯ LIỆU TOÁN HỌC


Thì ta nhận được 2  2U    .2V     U   V    
2
Nhưng ta nhận thấy rằng U  x  .V  x  là đa thức hệ số nguyên và  2    1 nên
U    V    là một số phức có dạng a  b , a , b  .
1
Do đó đẳng thức U    V     không thể xảy ra.
2
Vậy điều giả sử là sai và từ đó ta có điều phải chứng minh.

Câu 90. Giả sử n là số tự nhiên lớn hơn hoặc bằng 2 và P  x   x n  an1 x n1  ...  a1 x  1
là đa thức hệ số nguyên dương. Giả sử ak  an  k với mọi k  1, n  1. Chứng minh rằng
 y P  x 
tồn tại vô hạn cặp số nguyên dương  x , y  sao cho:  * .
 x P  y 
Lời giải
Trước tiên, ta nhận thấy rằng  1, P  1   là một cặp số thỏa mãn  *  .
Giả sử có một số hữu hạn các cặp số nguyên dương  x , y  thỏa mãn yêu cầu bài toán, ta
chọn cặp  x , y  với x  y và y có giá trị lớn nhất.
 Py 
Ta chứng minh cặp  y ,  cũng thỏa mãn  * 
 x 
Py Py
Do  x , y  thỏa mãn  *  nên có  
hiển nhiên Py
x x
 Py 
Ta cần chứng minh y P    1
 x 
Do y P  x    x , y   1 suy ra tồn tại z sao cho xz  1  mod y  .

82 | Tạp chí và tư liệu toán học


Đa thức và số học

Py  Py 
Theo tính chất a  b P  a   P  b  thì ta có  zP  y  P    P  zP  y    2 
z  x 
Py P  y   xzP  y 
Nhưng do  zP  y  
x x
Vì xz  1  mod y  nên y P  y   xzP  y  . Ngoài ra x P  y   xzP  y  do x P  y 

P  y   xzP  y 
Mà  x , y   1 nên ta có y
x
 Py 
Từ đó theo  2  thì ta được P    P  zP  y    P  z   mod y  , do P  y   1  mod y 
 x 
1 1
Vì ak  an  k nên ta suy ra x n P    P  x   y x n P    y P  z 
x x
 Py   Py 
CHINH PHỤC OLYMPIC TOÁN

Hay y P   . Vậy cặp  y ,  thỏa mãn yêu cầu bài toán.


 x   x 
Py
Ngoài ra thì ta có P  y   y n  1  y 2  yx  y
x
Mâu thuẫn với cách chọn y là lớn nhất.
Vậy điều giả sử là sai và từ đó ta có điều phải chứng minh.

Câu 91. Chứng minh rằng, với mỗi số nguyên dương n tồn tại đa thức P  x   x bậc
n sao cho P  0  , P  1  ,..., P  n  phân biệt và tất cả các số đó đều có dạng

2.2019 k  3, k  .
Lời giải
Bài toán chứng minh tồn tại đa thức, do đó phải chỉ ra đa thức thỏa mãn. Hãy để ý đến
yêu cầu P  0  , P  1  ,..., P  n  nhận giá trị đặc biệt, do đó bài toán này mang tư tưởng của
nội suy Lagrange. Tuy nhiên không hoàn toàn là nội suy Lagrange, vì các giá trị của chúng
chỉ có dạng chứ không có giá trị cụ thể. Do đó ta sẽ lấy đa thức đơn giản nhất của nội suy
x x x
là P  x   a0  a1    a2    ...  an   , ai  , i  1, n
1 2  n
Lưu ý rằng đa thức P  x  trên nhận giá trị nguyên, nhưng hệ số của chúng là hữu tỉ. Rõ
ràng muốn kết quả của chúng có dạng 2.2019 k  3, đặt a  2019 thì dạng 2 a k  3. Do đó
việc đầu tiên là cho các số P  i  có dạng a k đã. Dẫn đến các hệ số a0 , a1 ,..., an là hằng số
được không? Nếu chúng là hằng số thì P  x   k.2 x lai không được dạng mong muốn.
Nhưng ý trên cho ta suy nghĩ, để các hệ số ai cùng tham gia vào khai triển Newton. Do đó

ta sẽ chọn ai   a k  1  .
i

Chinh phục olympic toán| 83


Bồi dưỡng học sinh giỏi

Khi đó, với x  0, n thì ta được:


1 x n x
P  x    a k  1   a k  1    ...   a k  1      a k  1   1  a kn
0 n

1 n
Đến đây thì ta gần thu được kết quả mong muốn. Vì khi đó ta chỉ cần chọn đa thức
Q  x   2 P  x   3 thì sẽ có được đa thức thỏa mãn bài toán. Tuy nhiên đã đúng chưa? Hãy
lưu ý rằng đa thức P  x  hệ số hữu tỉ nên đa thức Q  x  cũng có hệ số hữu tỉ. vậy ta phải
cải tiến để đa thức Q  x  có hệ số nguyên. Lưu ý rằng đa thức P  x  hệ số hữu tỉ, với các
hệ số có mẫu lớn nhất là n1 . Do đó ta sẽ phải nhân vào, nhưng không làm thay đổi một
lượng. Lưu ý các hệ số của P  x  đều có nhân thêm  a k  1  . Vậy phải chọn k như thế

nào? Chọn làm sao để  a k  1  chia hết cho n !. Việc này quá to tát, không thể được? Tuy

nhiên ta sẽ chọn được k để  a k  1  chia hết cho một phần tử nào đó của n !. Liên quan

TẠP CHÍ VÀ TƯ LIỆU TOÁN HỌC


đến lũy thừa ta nghĩ ngay đến định lý Euler. Phân tích n !  n1 .n2 trong đó n1 là ước
nguyên tố lớn nhất của n ! mà nguyên tố cùng nhau với a , khi đó tất nhiên n 2 và a cũng
có chung ước nguyên tố.
Khi đó thì ta có a  1   1  mod n1 
n

Do đó ta chọn k    n1  thì n1 a k  1.
Tiếp đến ta phân tích n2  p11 .p22 ...pii
Đặt s  max  1 ,  2 ,...,  i  thì n 2 và a có chung ước nguyên tố p1 , p2 ,..., pi nên n2 s.

Từ đây dẫn đến n ! as  a k  1 

Đến đây thì bài toán được sang tỏ, bằng cách lấy Q  x   2 as P  x   3
n
x
Với P  x       a k  1 là đa thức chúng ta cần tìm.
i

i 0  i 

Câu 92. Chứng minh rằng tồn tại tập vô hạn các điểm
..., P3 , P2 , P1 , P0 , P1 , P2 , P3 ,...
trong mặt phẳng thỏa mãn tính chất: Với ba số nguyên a , b , c phân biệt thì các điểm
Pa , Pb , Pc thẳng hàng khi và chỉ khi a  b  c  2014.
USA MO 2014 Problem 3
Lời giải
Trước hết, ta có thể đặt a1  a  671, b1  b  671, c1  c  671 ta có thể đưa điều kiện bài toán
về thành a  b  c  1.

84 | Tạp chí và tư liệu toán học


Đa thức và số học

Một điểm Pi  f  i  , g  i   gồm hai thành phần hoành độ và tung độ. Để chứng minh tồn tại
dãy vô hạn, ý tưởng tự nhiên là đi tìm biểu diễn tường minh cho hai hàm f  i  và g  i  .
Và tìm sự biểu diễn hợp lý nhất, đơn giản nhất chính là tìm trong lớp hàm đa thức.
Trước tiên, ta sẽ tìm điều kiện để Pa , Pb , Pc thẳng hang.
Ta có Pa  f  a  , g  a   , Pb  f  b  , g  b   , Pc  f  c  , g  c  

 Pa Pb   f  b   f  a  , g  b   g  a  

Khi đó thì ta được 
 Pa Pc   f  c   f  a  , g  c   g  a  

f c   f  a g c   g  a 
Khi đó ba điểm Pa , Pb , Pc thẳng hàng khi và chỉ khi 
f b  f  a g b  g  a
Dẫn đến f  b  g  c   f  b  g  c   f  a  g  c   f  a  g  a 
 f c  g  b   f c  g  a  f  a  g b   f  a  g  a 
CHINH PHỤC OLYMPIC TOÁN

Đến đây ta đặt:


F  a , b , c   f  a  g  b   f  b  g  c   f  c  g  a    f  a  g c   f c  g  b   f  b  g  a  
Khi đó F  a , b , c  là đa thức ba biến và F  a , a , c   F  a , b , b   F  a , b , a  dẫn đến
F  a , b , c    a  b  b  c  c  a  G  a , b , c 
Mặt khác chúng ta muốn ba điểm Pa , Pb , Pc thẳng hang khi và chỉ khi a  b  c  1 hay ta
dịch qua ngôn ngữ của đa thức là F  a , b , c   0 khi và chỉ khi a  b  c  1.
Điều này cho ta thêm một nhân tử của F  a , b , c  khi đó thì
F  a , b , c    a  b  c  1  a  b  b  c  c  a  G  a , b , c 
Để đa thức này đơn giản nhất thì ta chọn G  a , b , c   1, khi đó thì
F  a , b , c    a  b  c  1  a  b  b  c  c  a 
Khai triển đa thức này, dẫn ta đến tìm đa thức f  x  và g  x  sao cho
f  a  g  b   f  b  g  a   ab 3  a 3b  a 2 b  ab 2  a  b 3  b 2   b  a 3  a 2 
Đồng nhất thì ta chọn f  x   x , g  x   x 3  x 2 , khi đó thì các điểm Pi của đề bài ban đầu có


tọa độ là Pi  i  671,  i  671    i  671 
3 2

Từ đây ta dễ dàng suy ra rằng, tồn tại tập vô hạn điểm thỏa mãn đề bài.

Chinh phục olympic toán| 85


Bồi dưỡng học sinh giỏi

Câu 93. Cho x1  x2  ...  xn là n , n  3 số thực thỏa mãn:


x2  x1  x3  x2  x 4  x3  ...  xn  xn1
Giả sử đa thức P  x  có n nghiệm thực là các giá trị x1 , x2 ,..., xn . Chứng minh rằng giá
trị lớn nhất của P  x  đạt được tại một điểm x0   xn1 , xn  .
Russia All – Russian Olympiad 2010
Lời giải
Vì đa thức P  x  có n nghiệm là x1 , x2 ,..., xn nên P  x   a  x  x1  x  x 2  ...  x  xn 
Do đó bài toán quy về tìm giá trị lớn nhất của A  x  x1 x  x 2 ... x  xn , x   x 1 , xn 
Giả sử phản chứng A đạt max tại x0   xi 1 , xi  với i  2, n  1. Khi đó ta tịnh tiến giá trị
này về   xn1  xi  x0 thì
   xn  1 vì xi  x0  0 do x0   xi 1 , xi  .

TẠP CHÍ VÀ TƯ LIỆU TOÁN HỌC


   xn vì xn1  xi  x0  xn  xi  x0  xn  xn 1 điều này đúng do giả thiết thì
xi  x0  xi  xi  1  xn  xn  1
Ta có x0  xk    xk , k  1, i  2.
Vì x0  x k  xi  x k  x0  xi     x k  xi  xn1   
Ta có x0  xn1  xn1  x0    xi vì   xn1  xi  x0 .
Tương tự thì ta có x0  xn2  xn2  x0    xi 1
Vì   xn 2  xn1  xn 2  xi  x0  xi 1  x0  xn 1  xn  2  xi  1  xi đúng theo giả thiết.
Tiếp tục như vậy thì ta có x0  xn 3  xn 3  x0    xi  3 ,...
Tổng quát luôn thì ta được   xk  xn1 i  k  x0  0, k  i , n  1
Cuối cùng thì  xn      xi 1    xn  x0  x0  xi 1   * 
Thật vậy  *  khai triển ra thì được:
 *   xn  xn xi 1   2  xi 1  xn x0  xn xi 1  x02  x0 xi 1
 xn    x0    2  x02  xi 1    x0 
 xn    x0  xi 1 do   x0  0
 xn  xn  1  xi  xi  x 0  xi  1
 xn  xn1  xi  xi 1 True 
Kết hợp tất cả các điều trên thì ta được:
   x1  ...    xi 2  .    xn1  ...    xi  .    xi 1  ...    xn 
    x1  ...    xi  2    xn1  ...    xi    xi 1  xn   
  x0  x1  ...  x0  xi  2  xn1  x0  ...  xi  x0  x0  xi 1  xn  x0 
  x0  x1  ...  x0  xi  2  .  x0  xn1  ...  x0  xi  .  x0  xi 1  ...  x0  xn 
Điều này mâu thuẫn với giả sử A đạt giá trị lớn nhất tại x 0

86 | Tạp chí và tư liệu toán học


Đa thức và số học

Vậy điều giả sử là sai hay từ đó ta suy ra được điều phải chứng minh.

Câu 94. Cho P  x   an x n  an1 x n1  ...  a1 x  a0 là đa thức hệ số nguyên thỏa mãn điều
kiện P  r   P  s   0 trong đó r , s là các số nguyên thỏa mãn điều kiện 0  r  s. Chứng
minh rằng tồn tại k  0, 1, 2,..., n sao cho ak  s.
Lời giải
Do P  s   0 nên P  x    x  s  Q  x  , Q  x    x  ,deg Q  x    n  1
Đặt Q  x   xt R  x  , với R  0   0, t  , R  x   x  ,deg  R x    n  t  1 với biểu thức
tường minh của R  x  là R  x   bnt 1 x nt 1  ...  b1 x  b0
Khi đó thì 0  P  r    r  s  r t R  r   R  r   0
Chứng tỏ R  x  có nghiệm nguyên dương r khi đó bắt buộc các hệ số của R  x  phải đổi
CHINH PHỤC OLYMPIC TOÁN

dấu.
 Trường hợp 1. Nếu b0  0 thì khi đó P  x    x  s  x t  bnt 1 x nt 1  ...  b1 x  b0 
Đến đây thì at  sb0  s , tức at chính là hệ số cần tìm.
 Trường hợp 2. Nếu b0  0 do các hệ số của R  x  phải đổi dấu nên sẽ tồn tại
i  0, 1,..., n  t  2 sao cho: bi  0  bi  1
Ta viết lại như sau: P  x    x  s  x t  bnt 1 x nt 1  ...  bi  1 x i  1  bi x i  ...  b1x  b0 
Khi đó thì ta được at  i  1  bi  sbi  1  s hay at  i  1 chính là hệ số cần tìm
Vậy trong mọi trường hợp đều tồn tại k  0,1, 2,..., n sao cho ak  s.

Câu 95. Cho P  x  , Q  x  là các đa thức hệ số nguyên. Đặt an  n ! n. Chứng minh rằng
P  an  P n
nếu  , n thì  , n và Q  n   0 .
Q  an  Q n
Canadian Mathematical Olympiad 2010
Lời giải
P x R x 
Chia đa thức P  x  cho Q  x  thì ta được  Ax 
Q x Q x
Trong đó: A  x  , R  x  là các đa thức hệ số hữu tỉ và R  x   0 hoặc
deg  R  x    deg  Q  x   .
Bx
Quy đồng tất cả các hệ số của đa thức A  x  thì ta có thể viết A  x  ,
b
Với B  x  là đa thức hệ số nguyên, còn b là bội chung nhỏ nhất của tất cả các mẫu số của
các hệ số trong A  x  , b  0.

Chinh phục olympic toán| 87


Bồi dưỡng học sinh giỏi

Nếu R  x  không đồng nhất 0 thì, với chú ý nếu k nguyên thì hoặc A  k   0 hoặc
1
Ak  .
b
Rk 1
Nhưng với k đủ lớn thì 0  
Qk b
P  an 
Dẫn đến với n đủ lớn thì không thể là số nguyên nên ta phải có R  x   0.
Q  an 
P x Bx
Khi đó thì  biểu diễn này với những giá trị x mà Q  x   0.
Q x b
Bây giờ với n0 là số nguyên cho trước, khi đó tồn tại vô hạn số nguyên dương k sao cho:
ak  n  mod b 

Có thể lấy các giá trị k  tn0 b  n0 , t  , với t nguyên dương đủ lớn thì k là số nguyên

TẠP CHÍ VÀ TƯ LIỆU TOÁN HỌC


dương.
B  ak 
Khi đó thì là số nguyên hay b B  ak 
b
P n
Từ đó suy ra b B  n  dẫn đến là một số nguyên.
Q n
Vậy từ đây ta có điều phải chứng minh.

Câu 96. Cho P  x  là đa thức bậc n  5 với hệ số nguyên và


P  x   an x n  an1 x n1  ...  a1 x  a0
Giả sử P  x  có n nghiệm nguyên phân biệt là 0,  2 ,...,  n . Tìm tất cả các số nguyên k
sao cho thỏa mãn P  P  k    0.
French Team Selection Test 2005
Lời giải
Nếu số nguyên k mà P  P  k    0 thì chứng tỏ P  k  là một nghiệm của P  x  . Nhưng các
nghiệm nguyên của P  x  đã cho là 0,  2 ,...,  n . Do đó phải có P  k   0,  2 ,...,  n  . Ta phân
tích P  k  dưới dạng:
P  x   an x  x   2  ...  x   n 
 Nếu P  k   0 chứng tỏ k là một nghiệm của P  k  khi đó k  0,  2 ,...,  n  .
 Nếu P  k    2 khi đó thì an k  k   2  ...  k   n    2  *  .
Từ  *  thì k  0, k   2 ,..., k   n ta cũng có k   i  k   j với mọi i  j
Mặt khác vì k  5 nên  2   n . k . k   2 . k   3 ... k   n
 k . k   2 . k   3 . k   4 . k   5  4  1.1.2. 1 . 2

88 | Tạp chí và tư liệu toán học


Đa thức và số học

Vì k   2  k   3  k   4  k   5 và k có thể bằng với một trong số k   i với i  1, 5


  2 k
Từ đó  2  4 mặt khác từ  *  thì ta được  2 k . k   2  
  2 k   2
Mặt khác k   2 và k   2   2 đẳng thức nếu có xảy ra thì k  2  2 tuy nhiên lại mâu
thuẫn với  *  do đó ta phải có 2 k   2 , 2 k   2   2
 22 2
Nhưng từ đây ta suy ra k . k   2   2 .   2 do  2  4
4 4
2
Nhưng từ đây theo  *  thì an  k   3  k   4  ...  k   n   1
k  k  2 
Nhưng cũng theo  *  thì an  k   3  k   4  ...  k   n   k   3 . k   4 . k   5
 2  1.1.2  1. 1 . 2  1. 1 .2  1.1. 2
CHINH PHỤC OLYMPIC TOÁN

Hai kết quả trên là mâu thuẫn với nhau.


Chứng tỏ không tồn tại số nguyên k để P  k    2 . Hoàn toàn tượng tự, ta cũng chứng
minh được không tồn tại số nguyên k để P  k    i , i  2, n.
Từ đó suy ra các số nguyên k cần tìm là k  0,  2 ,...,  n  .

Câu 97. Tìm số nguyên dương n nhỏ nhất sao cho tồn tại đa thức P  x  bậc n có hệ số
nguyên thỏa mãn: P  0   0, P  1   1 và với mọi   *
thì
 P     2   P     1 P    là bội của p với p là số nguyên tố
Doãn Quang Tiến
Lời giải
Ta sẽ chứng minh n  p  1 thỏa mãn yêu cầu bài toán.
Ta sẽ giả sử phản chứng rằng 1  n  p  2 và tồn tại đa thức P  x  thỏa mãn bài toán.
Từ đây, ta sử dụng công thức nội suy Lagrange thì ta được:
p2 p2
xi
P x   P k 
k 0 i  0,i  k k  i
p2 p2
p  1  i p2
Từ đây ta suy ra P  p  1   P  k      1 P  k  C pk1  1 
k 1

k 0 i  0,i  k ki k 0

Bây giờ, ta sẽ đi chứng minh một kết quả sau C pk1   1   mod p  , k  0, p  2
k

Ta sẽ chứng minh một cách nhanh chóng bằng phương pháp quy nạp như sau:
Với k  0 thì C p01  1   1   1  mod p  điều này là hiển nhiên.
0

Giả sử mệnh đề đúng với k tức là ta có C pk1   1   mod p  , ta sẽ chứng minh mệnh đề
k

cũng đúng với k  1.

Chinh phục olympic toán| 89


Bồi dưỡng học sinh giỏi

Mà ta có kết quả cơ bản của hệ số nhị thức như sau: C pk11  C pk  1  C pk1 nên từ đó ta suy ra:

C pk11  C pk  1  C pk1  0   1    1  .  1    1   mod p 


k k k 1

Vậy từ đó theo nguyên lý quy nạp toán học thì kết quả trên được chứng minh.
Từ đây, kết hợp với kết quả của  1  thì ta được:
p2 p2
P  p  1     1  P  k  C pk 1    1  P  k  1 
k 1 k 1 k

k 0 k 0
p2 p2 p 1
   1  P  k     P  k   mod p    P  k   0  mod p   2 
2 k 1

k 0 k 0 k 0
p 1 p 1 p 1
Nhưng mà ta có  P  k   P  0   P  1  P  k   1  P  k 
k 0 k 2 k 2

do giả thiết cho P  0   0, P  1   1.


Và từ giả thiết bài toán là  P     2   P     1  P    là bội của p nên ta suy ra được:

TẠP CHÍ VÀ TƯ LIỆU TOÁN HỌC


P  k   0, 1, 2  mod p  , k  2, p  1
p 1 p 1
Do đó ta suy ra được  P  k  1   P  k   1, p  1, 2 p  3  0  mod p 
k 0 k 2

Ta thấy điều này là hoàn toàn mâu thuẫn với  2 


Vậy giả sử phản chứng là sai nên ta phải có n  p  1.
Với n  p  1 ta xét P  x   x p 1 thỏa mãn yêu cầu bài toán.
Do đó giá trị nhỏ nhất của n là p  1 và ta hoàn tất bài toán.

Câu 98. Chứng minh rằng với mọi m  tồn tại đa thức Pm  x  có hệ số hữu tỉ thỏa mãn
với mọi n  *
thì 12 m 1  2 2 m1  ...  n2 m1  Pm  n  n  1  
Đề chọn đội tuyển VMO Đà Nẵng 2017
Lời giải
x
 Với m  0 thì ta chọn được đa thức P0  x  
2
x2
 Với m  1 thì ta chọn được đa thức P1  x  
4
 Giả sử khẳng định đúng đến m  1 thì ta xét đa thức:
  x   x  x 2  12  x 2  2 2  ...  x 2  m 2   x 2 m  1  am 1 x 2 m 1  am  2 x 2 m  3  ...  a0 x
   x   am 1 x 2 m 1  am  2 x 2 m  3  ...  a0 x  x 2 m  1
Thay x lần lượt bởi 1, 2,..., n thì ta được:
n m1 n

   i    ai Pi  n  n  1    i 2 m1  * 
i 1 i 0 i 1

Mà ta có   x   x  x  1  x  1  ...  x  m  x  m    x  m  x  m  1  ...  x  m  1  x  m 

90 | Tạp chí và tư liệu toán học


Đa thức và số học

1
Lại có   x     x  x  m  1    x  x  m  1 
2m  2 
Bây giờ ta đặt:
d  x  m    x  m  1    x    x  m  x  m  1  ...  x  m  1 
 d  x  m  1    x  m  1  x  m  ...  x  m    x  m  1    x 
1
 x   d  x  m   d  x  m  1  
2m  2 
n
1
Do đó thì ta được    i    d  n  m   d  m  
i 1 2m  2 
1 1
 d  n  m   n  m  n  m  1 ...  n  m  1
2m  2 2m  2
Mà ta có  n  i  n  i  1   n 2  i 2  n  i  n  n  1   i  i  1  , i  0, m
m
n
1
Từ đây suy ra    i   2m  2  n  n  1  i  i  1 Q  n  n  1  .
m
CHINH PHỤC OLYMPIC TOÁN

i 1 i 0

Khi đó Qm là đa thức có hệ số hữu tỉ và từ  *  ta chọn:


Pm  x   Qm  x   am1 Pm1  x   am 2 Pm 2  x   ...  a0 P0  x 
Và từ đây ta có điều phải chứng minh.

Câu 99. Cho P  x  , Q  x  , R  x  là ba đa thức hệ số thực thỏa mãn:


P  Q  x    P  R  x    c , x  với c  const 
Chứng minh rằng: P  x  là hằng số hoặc  Q  x   R  x   là hằng số.
Đề chọn đội tuyển VMO Hà Nam 2017
Lời giải
Bỏ qua trường hợp tầm thường P  x  là hằng số.
Bây giờ ta xét
P  x   an x n  an1 x n1  ...  a1 x  a0 , an  0, n  1
Q  x   bm x m  bm1 x m1  ...  b1 x  b0
R  x   c k x k  c k 1 x k 1  ...  c 1 x  c 0
Giả sử ta có m  k thì từ:
P  Q  x    P  R  x    c , x  với c  const  *
Đồng nhất hệ số bậc cao nhất của  *  thì ta được: an bmn  0.
Do đó hoặc bm  0 suy ra deg  R  x    deg  Q  x    0 hoặc m  k.
Khi m  k thì an  bmn  cmn   0  bm  cm với n là số lẻ.
Khi đó thì ta được:
Q  x   bm x m  bm1 x m1  ...  b1 x  b0 , bm  0
R  x   bm x m  cm1x m1  ...  c1x  c 0 , bm  0

Chinh phục olympic toán| 91


Bồi dưỡng học sinh giỏi

Bây giờ xét đa thức H  x   an  Qn  x   R n  x    an Q  x   R  x   S  x  , trong đó


S  x   Qn1  x   Q n 2  x  R  x   Q n 3  x  R 2  x   ...  Q  x  R n 2  x   R n1  x 
m n  1 
Xét hệ số của x trong S  x  thì ta được:

b n1  b n 2  b   ...  b  b    b 
n2 n1
 nb n1  0
Do đó deg  S  x    m  n  1  .
Ta sẽ giả sử phản chứng rằng deg  Q  x   R  x    1.
Khi đó thì ta có deg  H  x    m  n  1   1
n1
Ta xét đa thức T  x   P Q  x    P  R  x    H  x    ai Q i  x   R i  x  
i 0

Nhận thấy rằng: deg T  x    m  n  1  rõ ràng điều này là vô lý.


Từ đó ta phải có deg  Q  x   R  x    0 hay  Q  x   R  x   là hằng số.

TẠP CHÍ VÀ TƯ LIỆU TOÁN HỌC


Vậy từ đây ta suy ra điều phải chứng minh.

Câu 100. Chứng minh rằng tồn tại các đa thức hệ số nguyên S1 , S2 ,  tương tứng với
các biến x1 , x2 ,  , y1 , y 2 ,  thỏa mãn với mọi số nguyên n  1
n
 nd n

 d  S   d   xd  y dd 
d
d *
d|n d|n  
Với hàm tổng chạy qua các ước nguyên dương d của n .
Chú ý. Lưu ý rằng ta chỉ xét đến các đa thức trong trường  x  . Ví dụ, xét hàm
S1  x1  y 1 và S2  x2  y 2  x1 y 1 trong trường hợp n  2 ta được
S12  2S2   x12  y12   2   x2  y 2  là hàm  *  thỏa mãn yêu cầu bài toán.
2018 Brazil 4th TST Day 2
Lời giải
Ta chứng minh rằng Sn có hệ số nguyên bằng phương pháp quy nạp
Với n  1 ta được hiển nhiên đúng
Giả sử điều ta cần chứng minh vẫn đúng đến n  1 , tức các đa thức S1 , S2 ,  , Sn 1 đều là đa
thức có hệ số nguyên
Gọi p là số nguyên tố thỏa mãn vp  n   k  0 , từ đây ta có thể nhận thấy rằng:
 nd n
 n

 d  x
 d 
pky d
d    d.Sd
d

d|n   d|n ,d n
Đặt n  pm , suy ra với mỗi giá trị d không phải là ước số của m sẽ bị triệt tiêu trong tổng
trên khi lấy modulo p k , do đó đủ để ta thấy được điều sau:

92 | Tạp chí và tư liệu toán học


Đa thức và số học

 nd n
 n
pk  d  x
 d  y d
d    dSd
d

d|m   d|m
 
Đặt Ti  x1 , x2 , , y 1 , y 2 ,  Si x1p , x 2p , , y 1p , y 2p , , i
Áp dụng giả thiết bài toán cho Ti , xip , y ip , tổng trước đó trở thành:
 pm pm
 pm
 md pm

 d   x d
d
 y d
d
   dSd
d
  d  Td  S d
d 
d|m   d|m d|m  
Ta sẽ chứng minh p k có thể chia cho mỗi phần trong tổng này.
m l l 1
Thật vậy, đặt  p l q với p q , dễ dàng thấy được p l  1 Tdp q  Sdp q
d
Theo giả thiết quy nạp: A  Sdpq là đa thức hệ số nguyên, theo tính chất của tự đồng cấu
Frobenius (Frobenius Endomorphism), ta có được Tdq  A  Bp với hàm B có hệ số nguyên,
 pl 
 A  Bp    Bp  A
l1 pl pl i pl  i
A 
CHINH PHỤC OLYMPIC TOÁN

vậy ta có được điều sau p  


1 i  pl i 
Lại có p l  1 chia hết cho mỗi phần trong tổng này vì
 i  pl    i pl  pl  1  
vp  p     vp  p  
      i  l  vp  i   l  1  i  vp  i   1 ,
  i   i  i  1  
Điều này đúng với i  1 , ta có điều phải chứng minh.
Nhận xét. Đây là một bài toán khó với việc sử dụng lý thuyết về hàm tự đồng cấu rất ít
gặp trong các bài toán dự thi Olympic, tự đồng cấu Forbenius sẽ được tìm hiểu rõ hơn
trong chương trình đại học.

Chinh phục olympic toán| 93


TÀI LIỆU THAM KHẢO

Dưới đây là các tài liệu mà ebook này có tham khảo và đồng thời có cả những tài liệu mà
bọn mình đề xuất cho bạn đọc

[1]. A comprehensive course in number theory – Alan Baker – Cambridge University Press
(2012).
[2]. Problem – Solving and Selected Topics in Number Theory_ In the Spirit of the
Mathematical Olympiads – Michael Th. Rassias-Springer – Verlag New York (2011).
[3]. Tính chất số học trong các bài toán về đa thức – Phạm Viết Huy – THPT Chuyên Lê
Khiết – Quảng Ngãi.
[4] Chuyên đề đa thức và số học – Nguyễn Thành Nhân – THPT Chuyên Biên Hòa – Đồng
Nai.
[5]. Number Theory A Historical Approach – John J.Watkins.
[6]. Number theory and polynomials (London Mathematical Society Lecture Note Series) –
James McKee, Chris Smyth – CUP (2008).
[7]. An Introduction to Theory of Functional Equations – Marek Kuczma, Attila Gilányi
and Inequalities.
[8]. The IMO Compendium. A Collection of Problems Suggested for The International
Mathematical Olympiads: 1959 – 2009 – Djukic D., Vladimir Jankovic, Ivan Matic, Nikola
Petrovic – Springer (2011).
[9]. Problem – Solving and Selected Topics in Number Theory – Michael Th. Rassias.
[10]. Number Theory Structures, Examples and Problems – Titu Andreescu, Dorin Andrica.
[11]. Elementary Methods in Number Theory – Nathanson M.B.
[12]. Elementary Number Theory Primes Congruences and Secrets A Computational
Approach – William Stein.
[13]. http://maths.vn/tag/so-hoc/?fbclid=IwAR34ppP14Xm45l9dK8H_R6YKpjKPdSeCY-
AT1eRjLwQATzvluVo4raqrVH4.
[14]. diendantoanhoc.net
[15]. artofproblemsolving.com
[16]. Đột phá đỉnh cao bồi dưỡng học sinh giỏi chuyên đề số học – Văn Phú Quốc.
[17]. A Computational Introduction To Number Theory And Algebra – Victor Shoups.
[18]. Tuyển chọn các bài toán trong kì thi chọn đội tuyển của các tỉnh, thành phố năm học
2016 – 2017 – Toán học cho mọi người.
[19]. Định hướng bồi dưỡng học sinh năng khiếu toán – Nhà xuất bản Giáo dục Việt Nam
[20]. 104 Number Theory Problems: From the Training of the USA IMO Team – Titu
Andreescu, Dorin Andrica, Zuming Feng
TẠP CHÍ VÀ TƯ LIỆU TOÁN HỌC

CHINH PHỤC OLYMPIC TOÁN


TẠP CHÍ VÀ TƯ LIỆU TOÁN HỌC
Thôn 6 – Thạch Hòa – Thạch Thất – Hà Nội
Điện thoại: 0343763310; Email: tuangenk@gmail.com
Fanpage: https://www.facebook.com/OlympiadMathematical/

CHỊU TRÁCH NHIỆM NỘI DUNG


DOÃN QUANG TIẾN
NGUYỄN MINH TUẤN
TÔN NGỌC MINH QUÂN
HUỲNH KIM LINH

BIÊN TẬP
NGUYỄN MINH TUẤN

TRÌNH BÀY BẢN THẢO


NGUYỄN MINH TUẤN

ĐA THỨC VÀ SỐ HỌC
Đề nghị quý bạn đọc tôn trọng bản quyền của tác giả, không sao chép bản phụ.
Mọi ý kiến thắc mắc đóng góp vui lòng gửi về địa chỉ đã cung cấp ở trên.
Phiên bản sách điện tử được phát hành vào ngày 2/9/2019.
CHINH PHỤC OLYMPIC TOÁN
MỌI Ý KIẾN THẮC MẮC XIN VUI LÒNG GỬI VỀ ĐỊA CHỈ
DOÃN QUANG TIẾN
0817431404
doanquangtien1442001@gmail.com
https://www.facebook.com/OlympiadMathematical/
Đại học KHTN – Đại học Quốc Gia TP.HCM

LIMITED
EDITION
ỨNG DỤNG ĐỊNH LÝ VIÈTE
TRONG CÁC BÀI TOÁN SỐ HỌC

Tạp chí và tư liệu toán học sưu tầm và tổng hợp


Hướng tới VMO 2020

Ngày 14 tháng 12 năm 2019

Tóm tắt nội dung


Trong chương trình toán lớp 9, chúng ta đã được tìm hiểu đến một định lý cực kì nối tiếng đó là
định lý Viète, tuy nhiên ứng dụng của nó không chỉ có là biểu diễn mối quan hệ của các nghiệm
trong phương trình đa thức, mà còn ứng dụng trong nhiều mảng khác như số học, đa thức,... Ở bài
viết này, chúng tôi sẽ giới thiệu cho bạn đọc một số các bài toán số học có sử dụng định lý Viète và
nâng cao hơn nữa là phương pháp bước nhảy Viète - Vieta Jumping - để giải quyết các bài toán số
học hay và khó. Trong bài viết có tham khảo các tư liệu trong và ngoài nước, các bạn xem ở mục
tài liệu tham khảo ở cuối bài viết. Mọi ý kiến thắc mắc, đóng góp vui lòng gửi về địa chỉ.
1. Doãn Quang Tiến. fb.com/profile.php?id=100016406718327
2. Nguyễn Minh Tuấn. fb.com/tuankhmt.fpt

1 Nhà toán học Francois Viète.


Francois Viète (1540-1603) là nhà toán học Pháp vĩ đại. Ông là người đầu tiên đưa ra các kí hiệu bằng
chữ, do thế, người ta gọi ông là người cha của môn Đại số. Tên tuổi của ông gắn liền với một định lí về
nghiệm số của phương trình mà học sinh lớp 9 đều biết đó là định lí Viète, nhưng công lao của ông to
lớn hơn nhiều.

Ông vốn là một trạng sư, từng làm “cố vấn cơ mật” cho các triều vua Henry III và Henri IV. Giữa
những bộn rộn của công việc ở cung đình, hễ có ít phút rảnh rỗi là ông lại giải trí bằng cách... nghiên
cứu Toán học! Trong cuộc chiến tranh Pháp... Tây Ban Nha thời ấy, quân Tây Ban Nha thường liên
lạc với những kẻ nội phản trong nước Pháp bằng các mật thư. Vì được viết bằng các mật mã gồm
toàn các chữ số, nên các mật thư ấy hầu như không thể khám phá được.

Biết vị “cố vấn” Viète thích toán, vua Henry III đã nhờ ông thử dò tìm “chìa khóa” các mật thư
này. Nhận lời, suốt hai tuần lễ, ông làm việc quên ăn quên ngủ. Cuối cùng, chính Viète đã xé tung
tấm màn bí mật: ông đã tìm ra quy luật thay thế các chữ và số trong cách viết mật thư. Đọc được
các mật thư, quân Pháp đã làm thất bại hoàn toàn những mưu đồ của Tây Ban Nha. Về phía địch,
chúng gắng dò tìm nguyên nhân: cuối cùng chúng biết được những kí hiệu đã bị phơi trần, dù nhiều
lần thay đổi mật mã, và kẻ tìm ra bí mật là Francois Viète! Quân Tây Ban Nha tuyên bố Viète là
kẻ tử thù và đã xử án hỏa thiêu vắng mặt ông, nhưng bản án dã man đó không bao giờ thực hiện
được. Không chỉ quan tâm sâu sắc đến Đại số; nghiên cứu các phương trình, Viète còn nghiên cứu
cả Hình học và Lượng giác. Ông cũng đã khảo cứu kĩ lưỡng nhiều công trình của các nhà toán học thời cổ.

Phần lớn cuộc đời của Viète bị các công việc pháp lí của nghề trạng sư chiếm mất nên khó có thể
tưởng tượng ông đã lấy đâu ra thời gian để làm nên những công trình toán học của mình. Bí quyết
của ông chính là khả năng tập trung cao độ khi làm việc. Người ta còn kể lại, lúc gặp đươc một vấn đề
thú vị, ông có thể ngồi ở bàn làm việc suốt ba ngày đêm liền.

1
Hướng tới VMO 2020 Tạp chí và tư liệu toán học

2 Định lý Viète.
Định lý Viète được trình bày trong sách giáo khoa toán 9 - tập 2, cho ta mối quan hệ giữa các nghiệm
của phương trình bậc hai và các hệ số của nó. Sau đây ta sẽ nhắc lại nó.

Định lý Viète. Nếu phương trình bậc hai ax2 + bx + c = 0 (a 6= 0) có hai nghiệm x1 và x2 thì
tổng và tích của chúng là
 S = x1 + x2 = −b

a
 P = x1 .x2 = c
a
Ngược lại nếu có hai số x1 và x2 thỏa mãn S = x1 + x2 và P = x1 .x2 thì x1 và x2 là hai nghiệm
của phương trình t2 − St + P = 0.

Chú ý rằng trong khi giải toán, đôi khi ta không quan tâm tới giá trị của x1 và x2 mà chỉ cần quan
tâm đến 2 giá trị tổng và tích của chúng, từ đó ta có những đánh giá cần thiết. Ngoài ra cũng từ định
lí Viète ta nhận thấy nếu một phương trình bậc hai ax2 + bx + c = 0 có một nghiệm x1 thì nó sẽ có
thêm một nghiệm x2 nữa. Ngoài ra ta có thể mở rộng định lý cho phương trình đa thức bậc n bất kì.
Cho phương trình a0 + a1 x + a2 x2 + ... + an xn = 0, an 6= 0. Gọi x1 , x2 , . . . , xn là n nghiệm của phương
trình trên, khi đó thì

a0 + a1 x + a2 x2 + ... + an xn = a(x − x1 )(x − x2 )...(x − xn )

Nhân toàn bộ vế phải ra, chúng ta sẽ có công thức Viète, được phát biểu như sau



 a = an




 −a(x1 + x2 + ... + xn ) = an−1

. . .


 ...
(−1)n−1 a(x1 x2 ...xn−1 + x1 x2 ...xn−2 xn + ... + x2 x3 ...xn ) = a1




(−1)n a(x x ...x ) = a

1 2 n 0

và trong hàng k bất kỳ, vế phải của đẳng thức là an−k an−k còn vế trái được tính theo công thức
(−1)k a nhân với tổng của các tích từng cụm (n − k) các nghiệm của phương trình trên. ∇

3 Các bài toán cơ bản.


Sau đây ta sẽ đi tìm hiểu một vài ví dụ trước khi đi tìm hiểu về phương pháp bước nhảy Viète.

Bài toán 1. Tìm tất cả các giá trị của m để phương trình x2 − mx + m + 2 = 0 có các nghiệm
đều nguyên.

Lời giải
Lời giải 1. Điều kiện để phương trình có nghiệm là ∆ = m2 − 4 (m + 2) > 0. Phương trình đầu có
nghiệm nguyên thì ∆ phải là số chính phương, tức là tồn tại số nguyên k sao cho m2 − 4 (m + 2) = k 2 .
Ta có
m2 − 4 (m + 2) = k 2 ⇔ (m − 2 + k) (m − 2 − k) = 12
Đến đây ta có một chú ý rằng, theo định lí Viète, ta có x1 + x2 = m, do vậy mà m sẽ là số nguyên. Từ
đây do m và k là các số nguyên nên ta tìm được m = −2 hoặc m = 6. Ta xét các trường hợp.
1. Với m = −2 thì ta được phương trình x2 + 2x = 0, khi đó tìm được hai nghiệm nguyên là x1 = 0
và x2 = −2.

2
Chinh phục olympic toán Ứng dụng định lí Viète trong các bài toán số học

2. Với m = 6 thì ta được phương trình x2 − 6x + 8 = 0, khi đó ta được hai nghiệm nguyên là x1 = 2
và x2 = 8.
Như vậy đến đây bài toán đã được giải quyết. 
Lời giải 2. Theo định lý Viète với hai nghiệm x1 ; x2 thì ta có

x1 + x2 = m
(1)
x1 .x2 = m + 2

Do đó nếu ta tìm được các nghiệm nguyên của phương trình thì ta sẽ tìm được giá trị của m. Điều này
làm ta có ý tưởng giải phương trình nghiệm nguyên 2 ẩn x1 và x2 . Từ (1) ta được

x1 .x2 − (x1 + x2 ) = 2 ⇔ (x1 − 1) (x2 − 1) = 3

Khi đó x1 − 1 và x2 − 1 là các ước của 3, lại có 3 = 1.3 = −1. (−3). Đến đây việc tìm hai nghiệm x1 ; x2
hoàn toàn đơn giản và qua đó ta tìm được các giá trị m là m = −2 và m = 6. 

xét. Ở lời giải 1 sẽ có một số học sinh mắc sai lầm là chưa chỉ ra được m là số nguyên vì
! Nhận
đề bài không có đề cập. Do đó ta phải sử dụng định lí Viète để chỉ ra điều này.


xy + yz + zx = 8
Bài toán 2. Tìm nghiệm nguyên của hệ phương trình
x+y+z =5

Lời giải
Trước tiên ta sẽ đưa bài toán về việc giải phương trình nghiệm nguyên. Ta biến đổi
  
xy + (y + x) z = 8 xy + (5 − z) z = 8 xy = 8 − (5 − z) z
⇔ ⇔
x+y =5−z x+y =5−z x+y =5−z

Theo định lí Viète thì x và y là hai nghiệm của phương trình bậc hai t2 − (5 − z) t + 8 − (5 − z) z = 0,
trong đó z là tham số. Phương trình có nghiệm thì ∆ > 0 hay ta được
7
3z 2 − 10z + 7 6 0 ⇔ 1 6 z 6
3
Do z nguyên nên z = 1 hoặc z = 2. Ta xét các trường hợp sau.
• Với z = 1, khi đó phương trình trở thành t2 − 4t + 4 = 0, đến đây ta tìm được x = y = 2 thỏa
mãn.
• Với z = 2 khi đó phương trình trở thành t2 − 3t + 2 = 0, đến đây ta tìm được x = 2; y = 1 hoặc
x = 1; y = 2 thỏa mãn.
Vậy hệ phương trình có các nghiệm nguyên là (x; y; z) = (2; 2; 1) , (1; 2; 2) , (2; 1; 2). 

Bài toán 3. Giải phương trình x2 − mx + n = 0, biết phương trình có hai nghiệm nguyên dương
phân biệt và m, n là hai số nguyên tố.

Lời giải
Với bài toán này nếu dùng công thức nghiệm để xác định nghiệm của phương trình thì sẽ gây cho ta
nhiều khó khăn do phương trình có đến hai tham số. Do vậy ta sẽ sử dụng định lý Viète. Gọi x1 ; x2 là
các nghiệm nguyên dương của phương trình đã cho (x1 < x2 ). Ta biết rằng một số nguyên tố khi viết
thành tích hai số thì một thừa số là 1 và một thừa số là chính nó. Để ý ta lại thấy theo định lí Viète
thì x1 x2 = n. Do đó rất tự nhiên ta nghĩ đến sử dụng định lí Viète giả quyết bài toán. Thật vậy theo
định lí Viète ta được 
x1 + x2 = m
x1 x2 = n

3
Hướng tới VMO 2020 Tạp chí và tư liệu toán học

Do n là một số nguyên tố nên ta được x1 = 1; x2 = n, suy ra m = n + 1, do vậy m và n là hai số tự


nhiên liên tiếp nên ta được n = 2; m = 3, thử lại ta thấy tỏa mãn yêu cầu bài toán. 

Bài toán 4. Cho phương trình 2x2 + mx + 2n + 8 = 0, trong đó m và n là các tham số nguyên.
Giả sử phương trình có các nghiệm đều là số nguyên. Chứng minh rằng m2 + n2 là hợp số.

Lời giải
Để chứng minh được m2 + n2 là hợp số thì một suy nghĩ hết sức tự nhiên đó là xây dựng biểu thức
m2 + n2 theo các nghiệm của phương trình đề rồi từ đó phân tích biểu thức nghiệm thành nhân tử. Có
hai ý tưởng để xây dựng biểu thức m2 + n2 đó là áp dụng công thức nghiệm để tìm các nghiệm của
phương trình rồi từ đó tính m2 + n2 hoặc áp dụng định lí Viète. Rõ ràng trong hai ý tưởng đó việc áp
dụng định lí Viète giúp ta xây dựng biểu thức nghiệm mà không chứa các căn bậc hai.
Gọi x1 , x2 là hai nghiệm của phương trình trên, theo định lí Viète ta được
( m
x1 + x2 = −
2
x1 .x2 = n + 4

Khi đó ta có

m2 + n2 = (2x1 + 2x2 )2 + (x1 x2 − 4)2 = 4x21 + 4x22 x21 + x22 x21 + 16 = x21 + 4 x22 + 4
 

Do x1 ; x2 là các số nguyên nên x21 + 4; x22 + 4 là các số nguyên dương lớn hơn 1.
Từ đó ta được m2 + n2 là hợp số. 

a2 + b2
Nhận xét. Xét 2m = a + b; 2n = a − b, khi đó từ m2 + n2 = , ta có bài toán sau.
2
Bài toán. Cho phương trình 4x2 + (a + b) x + 2 (a − b) + 16 = 0, trong đó a và b là các tham số
a2 + b2
! nguyên. Giả sử phương trình có các nghiệm đều là số nguyên, chứng minh rằng
2
là hợp số.
Bài toán tương tự. Giả sử phương trình bậc hai x2 + ax + b + 1 = 0, trong đó a, b là các tham
số nguyên, đồng thời b 6= −1 có hai nghiệm đều là số nguyên khác 0, chứng minh rằng a2 + b2 là
hợp số.

Bài toán 5. Tìm các số nguyên dương a và b, trong đó (a > b) sao cho phương trình bậc hai
x2 − abx + a + b = 0 có các nghiệm đều là số nguyên.

Lời giải
Điều kiện để phương trình có nghiệm là ∆ = (ab)2 − 4 (a + b) > 0, giả sử phương trình có hai nghiệm
nguyên x1 ; x2 (x1 6 x2 ), khi đó theo định lí Viète ta được

x1 + x2 = ab
x1 .x2 = a + b

Do a và b là các số nguyên dương nên suy ra các nghiệm x1 ; x2 cũng là các số nguyên dương. Ta chú ý
rằng với hai số lớn hơn 2 thì tích của chúng bao giờ cũng lớn hơn tổng của chúng. Do đó ta nghĩ đến
chứng minh một trong bốn số dương trên không vượt quá 2.
Thật vậy, nếu a > 2; b > 2 thì ta có ab > 2a; ab > 2b nên 2ab > 2 (a + b) hay ab > a + b.
Nếu cả bốn số dương x1 ; x2 ; a; b đều lớn hơn 2 thì x1 .x2 > x1 + x2 và ab > a + b, khi đó định lí Viète
trên không thể xảy ra. Như vậy trong bốn số dương x1 ; x2 ; a; b tồn tại ít nhất một số không vượt quá
2. Theo giả thiết và theo cách chọn hai nghiệm x1 ; x2 thì trong hai số x1 và b có ít nhất một số không
lớn hơn 2. Do vai trò của hai số x1 và b là như nhau nên không mất tính tổng quát ta giả sử rằng
0 < x1 6 2, đến đây ta xét từng trường hợp của x1 .

4
Chinh phục olympic toán Ứng dụng định lí Viète trong các bài toán số học

 
x1 + x2 = ab 1 + x2 = ab
1. Nếu x1 = 1, khi đó từ ta suy ra , từ đó ta được ab − a − b = 1,
x1 .x2 = a + b x2 = a + b
suy ra (a − 1) (b − 1) = 2. Chú ý a > b nên từ phương trình trên ta được a = 3; b = 2, khi đó ta
tìm được x2 = 3.
2. Nếu x1 = 2, tương tự như trường hợp trên ta tìm được các cặp số nguyên dương (a; b) thỏa mãn
là (a; b) = (5; 1) , (2; 2).
Bài toán được giải quyết hoàn toàn. 

Bài toán 6. Tìm tất cả các số nguyên tố p, q sao cho tồn tại số tự nhiên m thỏa mãn

pq m2 + 1
=
p+q m+1

Lời giải
pq m2 + 1
Quan sát hệ thức = thì hai đại lượng pq và p + q làm ta liên tưởng đến hệ thức Viète.
p+q m+1
m2 + 1
Nếu là phân số tối giản thì từ hệ thức của bài toán ta được
m+1

p+q =m+1
pq = m2 + 1

m2 + 1
Còn nếu chưa tối giản thì chỉ cần rút gọn ta cũng được một hệ điều kiện tương tự. Vấn đề là
m+1
m2 + 1
ta cần kiểm tra xem phân số có rút gọn được hay không.
m+1
pq m2 + 1
• Nếu p = q thì từ = ta được
p+q m+1
2 m2 + 1

4
p= = 2m − 2 +
m+1 m+1
Do m ∈ N và p là số nguyên tố nên (m + 1) | 4 ⇒ m = 0; m = 1; m = 3. Từ đó ta tìm được
p = 2; p = 5 thỏa mãn yêu cầu bài toán.
• Nếu p 6= q thì pq và p + q là nguyên tố cùng nhau vì pq chỉ chia hết cho các ước nguyên tố là p
và q còn p + q thì không chia hết cho p và không chia hết cho q. Gọi r là một ước chung của
m2 + 1 và m + 1. Khi đó ta có
r | [(m + 1) (m − 1)] ⇒ r | m2 − 1


Do đó r | m2 + 1 − m2 − 1 ⇒ r | 2 suy ra r = 1 hoặc r = 2.
  

p+q =m+1
1. Với r = 1 suy ra , khi đó p và q là hai nghiệm của phương trình
pq = m2 + 1

x2 − (m + 1) x + m2 + 1 = 0
Ta có ∆ = −3m2 + 2m − 3 = −(m − 1)2 − 2m2 + 2 < 0 nên phương trình trên vô nghiệm.


2pq = m2 + 1

2. Với r = 2 suy ra , khi đó p và q là hai nghiệm của phương trình
2 (p + q) = m + 1

2x2 − (m + 1) x + m2 + 1 = 0
Ta có ∆ = −7m2 + 2m − 7 = −(m − 1)2 − 6m2 + 6 < 0 nên phương trình vô nghiệm.


Vậy bộ các số nguyên tố cần tìm là (p; q) = (2; 2) , (5; 5). 

5
Hướng tới VMO 2020 Tạp chí và tư liệu toán học

Bài toán 7. Tìm số nguyên tố p để p2 − p + 1 là lập phương của một số nguyên tố khác.

Lời giải
Do p là số nguyên tố nên ta được p | q − 1 hoặc p | q 2 + q + 1. Đến đây ta sẽ chứng minh p | q − 1
không xẩy ra. Thật vậy. nếu p | q − 1 thì ta được q − 1 = kp, k ∈ N, do đó q = kp + 1. Khi đó từ
p2 − p + 1 = q 3 ta được
p2 − p + 1 = (kp + 1)3 ⇔ p2 − p + 1 = k 3 p3 + 3k 2 p2 + 3kp + 1
Nhận thấy với k > 2 thì hiển nhiên
p2 − p + 1 < k 3 p3 + 3k 2 p2 + 3kp + 1
Từ đó suy ra được k 6 1.
1. Với k = 0, khi đó ta được p2 − p + 1 = 1 ⇔ p (p − 1) = 0, điều này vô lí do p là số nguyên tố.
2. Với k = 2, khi đó ta được
p3 − 2p2 + 4p = 0 ⇔ p2 − 2p + 4 = 0
không tồn tại p thỏa mãn.
Vậy với p | q − 1 thì không tồn tai số nguyên tố p thỏa mãn yêu cầu bài toán.
Như vậy ta phải có p | q 2 + q + 1, khi đó (p, q − 1) = 1 và có (q − 1) | p (p − 1) nên q − 1 | p − 1. Đặt
q2 + q + 1
p = (q − 1) k + 1 với k ∈ N, khi đó từ p | q 2 + q + 1 ta suy ra được là số nguyên dương
(q − 1) k + 1
hay ta được
q 2 + qk + k 3k − q − 2
=q+2+
qk − k + 1 qk − k + 1
là số nguyên dương. Từ đó ta phải có |3k − q − 2| > qk − k + 1. Ta xét các trường hợp sau
• Nếu 3k − q − 2 > qk − k + 1, khi đó ta được k (4 − q) > q + 3. Từ đó nếu q > 4 thì ta được
k (4 − q) < q + 3, điều này mâu thuẫn. Do đó ta suy ra được q < 4, mà ta lại có q > 1 nên q = 2
hoặc q = 3.
1. Khi q = 2 thì từ p2 − p + 1 = q 3 ta được p (p − 1) = 7, phương trình vô nghiệm.
2. Khi q = 3 thì từ p2 − p + 1 = q 3 ta được p (p − 1) = 26, phương trình vô nghiệm.
• Nếu 3k − k − 2 6 − (qk − k + 1) thì ta suy ra được
2 + q − 3k > qk − k + 1 ⇒ k (q + 2) 6 q + 1
Điều này vô lí.
• Nếu 3k−q−2 = 0 ⇒ q = 3k−2, khi đó từ p = (q − 1) k+1 ta được p = 3k (k − 1)+1 = 3k 2 −3k+1
và đồng thời có
q 2 + q + 1 = (3k − 2)2 + (3k − 2) + 1 = 9k 2 − 9k + 3
Từ đó suy ra
q2 + q + 1 9k 2 − 9k + 3
= 2 =3
p 3k − 3k + 1
Do đó từ p (p − 1) = (q − 1) q 2 + q + 1 ta được p − 1 = 3 (q − 1) nên suy ra ra được


(k − 1) (k − 3) = 0. Từ đó ta được k = 1 hoặc k = 3.
1. Với k = 1 thì p = q, khi đó ta được p2 − p + 1 = p3 ⇔ p p2 − p + 1 = 1, điều này vô lí.


2. Với k = 3 thì p = 3q − 2, khi đó ta được 9q 2 − 15q + 7 = q 3 ⇔ (q − 1) (q − 7) = 0 nên q = 1


hoặc q = 7. Thử trực tiếp ta được q = 7 thỏa mãn yêu cầu bài toán. Khi đó ta được p = 19.
Vậy p = 19 là số nguyên tố duy nhất thỏa mãn yêu cầu bài toán. 

6
Chinh phục olympic toán Ứng dụng định lí Viète trong các bài toán số học

Bài toán 8. Cho x = a + b − c; y = c + a − b; z = b + c − a với a, b, c là các số nguyên tố. Giả sử


√ √
rằng x2 = y và z − y là bình phương của một số nguyên tố. Tìm giá trị của biểu thức

T = (a + 2) (b − 10) (c + 2)

Lời giải
Biến đổi giả thiết tương đương

 2a = x + x2
 
 2a = x + y
2b = x + z ⇒ 2b = x + z
2c = y + z 2c = x2 + z
 

Xét phương trình bậc 2 là x2 + x = 2a ⇔ x2 + x − 2a = 0. Dễ thấy ∆ = 8a + 1 > 0 nên nên phương


trình có hai nghiệm phân biệt. Gọi hai nghiệm của phương trình là x1 và x2 . Như vậy nếu một trong
hai nghiệm là số nguyên thì nghiệm còn lại cũng nguyên. Chú ý rằng a là số nguyên tố nên ta nghĩ
đến sử dụng định lí Viète để xác định các nghiệm. Theo định lí Viète ta có

x1 + x2 = −1
x1 .x2 = −2a

Do 2 và a là số nguyên tố nên từ x1 .x2 = −2a ta được x1 ∈ {−2; −a; 2; a}. Ta xét các trường hợp sau.
• Nếu x1 = −2, khi đó ta tìm được a = 1 không phải là số nguyên tố.
• Nếu x1 = −a, khi đó ta được a2 − 3a = 0, do a là số nguyên tố nên a = 3. Từ đó ta tìm được hai
nghiệm của phương trình là x1 = −3 và x2 = 2.
• Nếu x1 = 2, khi đó ta tìm được a = 3, từ đó ta tìm được hai nghiệm là x1 = 2 và x2 = −3.
• Nếu x = a, khi đó ta được a2 − a = 0 nên a = 0 và a = 1, loại vì không phải là số nguyên tố.
Vậy phương trình trên có hai nghiệm nguyên là x = 2 và x = −3, đồng thời ta có a = 3. Bây giờ ta sẽ
xác định các số nguyên tố b, c ứng với mỗi trường hợp.

1. Với x = 2 khi đó ta được y = 4. Do đó z − 2 = p2 với p là số nguyên tố. Do x là số chẵn và
2b = x + z nên z là số chẵn. Khi đó p là số chẵn, dẫn đến p = 2. Khi đó ta được z = 36, suy ra
c = 20, loại do c không phải là số nguyên tố.

2. Với x = −3, khi đó ta được y = 9. Do đó z − 3 = p2 với p là số nguyên tố. Do x là số lẻ và
2b = x + z nên z là số chẵn. Khi đó p là số chẵn, dẫn đến p = 2. Khi đó ta được z = 49, suy ra
c = 29 và b = 23 là các số nguyên tố.
Như vậy ta tính được T = (a + 2) (b − 10) (c + 2) = (3 + 2) (23 − 10) (29 + 2) = 2015. 

Bài toán 9. Tìm các cặp số nguyên (a; b) sao cho hai số a2 + 4b và b2 + 4a đều là số chính
phương.

Lời giải
Ta sẽ chứng minh các cặp số sau thỏa mãn yêu cầu bài toán

(a; b) = 0; k 2 , k 2 ; 0 , (−4; −4) , (−5; −6) , (−6 − 5) , (k; 1 − k) , (1 − k, k) , k ∈ Z


 

Thật vậy, do vai trò của a và b như nhau nên không mất tính tổng quát ta có thể giả sử |a| > |b|.
• Nếu b = 0, khi đó để a2 + 4b và b2 + 4a đều là số chính phương thì a = k 2 với k là số nguyên.
• Nếu b 6= 0, khi đó biểu thức a2 + 4b là ta liên tưởng đến biệt thức ∆ của phương trình
x2 + ax − b = 0.

7
Hướng tới VMO 2020 Tạp chí và tư liệu toán học

Do ∆ = a2 + 4b là số chính phương nên phương trình trên sẽ có hai nghiệm nguyên là x1 và x2 . Theo
định lí Viète ta được
1 1 1 1 x1 + x2 a
+ > + = = >1
|x1 | |x2 | x1 x2 x1 x2 b

Từ đó suy ra một trong hai nghiệm nguyên của phương trình trên, chẳng hạn x1 thỏa mãn |x1 | 6 2.
Từ đó ta được x1 ∈ {−2; −1; 1; 2}. Đến đây ta xét các trường hợp sau.

1. Nếu x1 = 2, khi đó từ phương trình x2 + ax − b = 0 ta được b = 2a + 4. Suy ra

b2 + 4a = (2a + 4)2 + 4a = 4a2 + 20a + 16 = (2a + 5)2 − 9

là số chính phương. Đặt (2a + 5)2 − 9 = y 2 với y ∈ N, từ đó ta được (2a + 5 − y) (2a + 5 + y) = 9.


Lúc này tìm được a = −4 và a = −1.

• Với a = −4, khi đó a = −4. Từ đó ta được (a; b) = (−4; −4) thỏa mãn yêu cầu bài toán.

• Với a = −1, khi đó b = 2. Trường hợp này loại do không thỏa mãn |a| > |b|.

2. Nếu x1 = −2, khi đó từ phương trình x2 + ax − b = 0 ta được b = 4 − 2a. Suy ra

b2 + 4a = (4 − 2a)2 + 4a = 4a2 − 12a + 16 = (2a − 3)2 + 7

là số chính phương. Đặt (2a − 3)2 + 7 = y 2 với y ∈ N, từ đó ta được (y − 2a − 3) (y − 2a + 3) = 7.


Giải phương trình trên ta được (a; b) = (3; −2) , (0; 4), trong đó nghiệm (0; 4) bị loại do không
thỏa mãn |a| > |b|. Chú ý là (3; −2) có dạng (k; 1 − k).

3. Nếu x1 = 1, khi đó từ phương trình x2 + ax − b = 0 ta được b = a + 1. Suy ra

b2 + 4a = (a + 1)2 + 4a = a2 + 6a + 1 = (a + 3)2 − 8

là số chính phương. Đặt (a + 3)2 − 8 = y 2 với y ∈ N, từ đó ta được (a + 3 − y) (a + 3 + y) = 8.


Giải phương trình trên ta được (a; b) = (−6; −5) , (0; 1), trong đó nghiệm (0; 1) bị loại do không
thỏa mãn |a| > |b|.

4. Nếu x1 = −1, khi đó từ phương trình x2 + ax − b = 0 ta được b = 1 − a. Suy ra

b2 + 4a = (1 − a)2 + 4a = a2 + 2a + 1 = (a + 1)2

là số chính phương và

a2 + 4b = a2 + 4 (1 − a) = a2 − 4a + 4 = (a − 2)2

cũng là số chính phương. Do đó (a; b) = (k; 1 − k) với k là số nguyên thỏa mãn yêu cầu bài toán.

Chú ý với (a; b) thỏa mãn yêu cầu bài toán thì (b; a) cũng thỏa mãn yêu cầu bài toán. Do vậy kết hợp
các trường hợp lại ta được các cặp số nguyên (a; b) như trên thỏa mãn yêu cầu bài toán. 

Bài toán 10. Cho a, b, c, d là các số thực thỏa mãn

a2 − 1 b2 − 1 c2 − 1 d2 − 1
= = = =p
5a 5b 4c 4d
trong đó p là số nguyên dương. Chứng minh rằng (a − c) (b − c) (a + d) (b + d) là một số chính
phương.

Lời giải

8
Chinh phục olympic toán Ứng dụng định lí Viète trong các bài toán số học

Biến đổi giả thiết tương đương

a2 + 5pa − 1 = b2 + 5pb − 1 = 0; c2 + 4pc − 1 = d2 + 4pd − 1 = 0

Xét hai phương trình bậc hai ẩn x là x2 + 5px − 1 = 0 và x2 + 4px − 1 = 0. Khi đó ta thấy a, b là hai
nghiệm của phương trình x2 + 5px − 1 = 0 và c, d là hai nghiệm của phương trình x2 + 4px − 1 = 0.
Theo định lý Viète ta được  
a + b = −5p c + d = −4p

ab = −1 cd = −1

Ta có (a − c) (b − c) (a + d) (b + d) = ab − (a + b) c + c2 ab + (a + b) d + d2 . Áp dụng các hệ thức


  

Viète trên ta được

ab − (a + b) c + c2 ab + (a + b) d + d2 = c2 + 5pc − 1 d2 − 9pd − 1
    

Chú ý rằng c2 + 5pc − 1 d2 − 9pd − 1 = c2 + 4pc − 1 + pc d2 + 4pd − 1 − 9pd , và đồng thời kết
   

với c2 + 4pc − 1 = d2 + 4pd − 1 = 0 ta được

(a − c) (b − c) (a + d) (b + d) = −9p2 cd = 9p2 = (3p)2

Vậy (a − c) (b − c) (a + d) (b + d) là một số chính phương. 

4 Phương pháp bước nhảy Viète - Vieta Jumping.


Đây là một phương pháp mạnh để xử lý lớp phương trình Diophantine bậc hai trở lên. Sau đây ta sẽ
tìm hiểu về phương pháp giải của nó.

Phương pháp. Ta tiến hành qua 2 bước sau.


1. Bước 1. Cố định một giá trị nguyên mà đề bài cho, rồi giả sử tồn tại một cặp nghiệm
thỏa mãn một vài điều kiện mà không làm mất tính tổng quát của bài toán.
2. Bước 2. Dựa vào định lý Viète để tìm các mối quan hệ và sự mâu thuẫn, từ đó tìm được
kết luận của bài toán.

Một trong các bài toán nổi tiếng nhất để minh họa cho phương pháp này và luôn xuất hiện trong bất
kì các tài liệu nói về vấn đề này, mà mỗi khi nhắc tới học sinh chuyên toán không thể không biết đó
chính là bài toán trong kì thi IMO 1988. ∇

Bài toán 1 [IMO 1988]. Cho a, b là các số nguyên dương thỏa mãn ab + 1 | a2 + b2 . Chứng
a2 + b2
minh rằng là số chính phương.
ab + 1

Lời giải
Bàn luận. Đây chính là bài toán khó nhất kì thi năm đó, và chỉ có mười một học sinh cho lời giải
hoàn chỉnh của bài toán. Trong số 11 học sinh giải được bài toán đó, Việt Nam chúng ta có một đại
diện chính là Giáo sư Ngô Bảo Châu. Sau đây là lời giải cho bài toán này.
a2 + b2
Lời giải. Đặt k = , khi đó theo phương pháp đã đề cập tới ở trên, ta cố định k, sau đó xét tất
ab + 1
cả các cặp (a, b) nguyên dương thỏa mãn phương trình

a2 + b2
k=
ab + 1
a2 + b2
 
∗ ∗
Hay có nghĩa là ta xét tập S = (a, b) ∈ N × N | k = . Vì S là tập các cặp số nguyên dương
ab + 1
nên luôn tồn tại một cặp (a0 , b0 ) trong S mà a0 + b0 thỏa mãn a0 > b0 đạt giá trị nhỏ nhất.

9
Hướng tới VMO 2020 Tạp chí và tư liệu toán học

Xét phương trình


x2 + b20
= k ⇔ x2 − kx.b0 + b20 − k = 0
xb0 + 1
là một phương trình bậc hai ẩn x. Ta đã biết rằng phương trình trên có một nghiệm là a0 . Như vậy
theo định lý Viète thì tồn tại nghiệm a1 thỏa mãn phương trình bậc hai với ẩn x trên và
b20 − k
a1 = kb0 − a0 =
a0
Từ đây ta có a1 cũng là số nguyên. Ta chứng minh a1 không âm. Thật vậy, nếu a1 < 0 thì

a21 − kb0 a1 + b20 − k > a21 + k + b20 − k > 0

điều này mâu thuẫn. Do đó ta có a1 > 0. Đến đây ta xét a1 > 0 thì (a1 , b0 ) là một cặp thuộc S.
Theo định nghĩa của (a0 , b0 ) ta có

a0 + b0 6 a1 + b0 ⇒ a0 6 a1

Mặt khác cũng theo định lí Viète thì

a20 6 a0 a1 = b20 − k < b20 ⇒ a0 < b0

điều này trái với giả thiết ban đầu. Do đó a1 = 0, vì vậy suy ra k = b20 là một số chính phương, ta có
điều cần chứng minh. 

Nhận xét. Trong bài toán này, ta đã sử dụng tới nguyên lí cực hạn: Trong tập hợp các số nguyên
dương thì luôn tồn tại số nguyên dương nhỏ nhất. Mệnh đề trên không những hữu dụng trong
! các lớp bài toán này mà còn trong nhiều bài toán tổ hợp, tổ hợp số học và số học. Bài toán các
bạn sẽ tìm hiểu sau đây cũng là một kết quả rất nối tiếng.

Bài toán 2 [Phương trình Markov]. Giải phương trình nghiệm nguyên

x2 + y 2 + z 2 = 3xyz.

Lời giải
Bàn luận. Đây là một phương trình cực kì nổi tiếng, xuất hiện trong luận án tiến sĩ tại trường Đại học
Saint Petersburg với chủ đề “Dạng toàn phương xác định dương” của nhà toán học Andrei Andreevich
Markov (1856 - 1922) - nhà toán học nổi tiếng người Nga. Luận án tiến sĩ của Markov đã giải quyết
được một số vấn đề khó trong “Lý thuyết số” và mở ra một hướng nghiên cứu trong toán học, đó là “Lý
thuyết xấp xỉ Diophant”. Phương trình Markov - một phương trình Diophant bậc hai đặc biệt đóng vai
trò chủ đạo trong các nghiên cứu của Markov về các dạng toàn phương.
Lời giải. Ta thấy rằng phương trình Markov có một nghiệm (1, 1, 1). Đặt

S = {(x, y, z); x, y, z ∈ Z+ | x2 + y 2 + z 2 = 3xyz}

là tập hợp tất cả các nghiệm nguyên dương của phương trình Markov thì S 6= ∅. Do vai trò của x, y, z
trong phương trình là như nhau, không mất tính tổng quát ta có thể giả sử rằng x 6 y 6 z. Với mỗi
cặp (x, y, z) ∈ S; (x0 , y 0 , z 0 ) ∈ S ta định nghĩa (x, y, z) > (x0 , y 0 , z 0 ) nếu x + y + z > x0 + y 0 + z 0 . Markov
đã dùng ý tưởng “thông minh”sau để chứng minh có vô hạn bộ ba số nguyên dương (x, y, z) thỏa mãn
phương trình trên. Với mỗi nghiệm (xn , yn , zn ) ∈ S ta xây dựng bộ nghiệm mới như sau: Ta coi xn là
ẩn và các biến còn lại là các tham số thì rõ ràng phương trình bậc hai

x2 − 3yn zn x + yn2 + zn2 = 0

có một nghiệm là xn , nên nó có nghiệm thứ hai là x0 . Theo định lý Viète, ta có

xn + x0 = 3yn zn và xn x0 = yn2 + zn2 (1)

10
Chinh phục olympic toán Ứng dụng định lí Viète trong các bài toán số học

Từ đây ta được x0 là một số nguyên dương, kết hợp với giả thiết xn 6 yn 6 zn và (1) ta được

yn2 + zn2 2x2


x0 = > n = 2xn > xn .
xn xn

Đặt (xn+1 , yn+1 , zn+1 ) = (x0 , yn , zn ) thì (xn+1 , yn+1 , zn+1 ) là một nghiệm của phương trình Markov.
Cách xây dựng này cho ta một dãy vô hạn các nghiệm của phương trình Markov vì các nghiệm tiếp
theo lớn hơn các nghiệm trước theo định nghĩa thứ tự ở trên. Do đó phương trình Markov có vô số
nghiệm. Ta thấy ý tưởng của Markov trong chứng minh trên là coi một biến là nghiệm của tam thức
bậc hai khi cố định các nghiệm còn lại để từ đó xây dựng nghiệm mới từ một nghiệm đã biết bằng các
định lí Viète. Cụ thể ta xét phương trình Diophant là phương trình bậc hai đối với một biến nào đó,
chẳng hạn x21 + G(x2 , x2 , . . . , xn ) 6= 0 là phương trình bậc 2 ẩn x1 . Nếu phương trình này có nghiệm
(x1 , x2 , . . . , xn ) = (a1 , a2 , . . . , an ) thì rõ ràng a1 là nghiệm phương trình X 2 + G(a2 , a3 , . . . , an ) = 0.
Phương trình trên phải còn một nghiệm nữa là a01 . Kết hợp với định lý Viète và dữ kiện của đầu bài ta
sẽ xây dựng bộ (a01 , a2 , . . . , an ) là nghiệm của phương trình trên. 

Nhận xét. Thông qua 2 bài toán đầu tiên, ta đã phần nào hiểu được ý tưởng của phương pháp
này, bài toán thứ 3 sau đây là bài toán tổng quát của bài toán này, nó sẽ trả lời cho ta câu
! hỏi “Nếu tổng các bình phương S ba số nguyên dương chia hết cho tích P của chúng thì khi đó
S
thương số bằng bao nhiêu?”.
P

Bài toán 3. Tìm tất cả các số nguyên dương k để phương trình x2 + y 2 + z 2 = kxyz có nghiệm
nguyên dương.

Lời giải
Lời giải 1. Trước tiên ta thấy rằng với k = 1 phương trình đã cho có nghiệm (3; 3; 3) và với k = 3
thì phương trình có nghiệm (1; 1; 1). Như vậy với k = 1 hoặc k = 3 thì phương trình luôn có nghiệm
nguyên dương. Bây giờ ta cần kiểm tra xem với k 6= 1 và k 6= 3 thì phương trình có nghiệm nguyên
dương không. Giả sử với k 6= 1 và k 6= 3 phương trình đã cho có nghiệm nguyên dương là (x0 ; y0 ; z0 ).
Không mất tính tổng quát ta giả sử x0 6 y0 6 z0 và x0 + y0 + z0 có giá trị bé nhất. Ta xét các trường
hợp.
1. Nếu y0 < z0 , ta xét phương trình bậc hai z 2 − kx0 y0 z + x20 + y02 = 0. Khi đó phương trình có một
nghiệm là z0 . Theo định lí Viète thì phương trình có một nghiệm nữa là z1 . Như vậy thì

z0 + z1 = kx0 y0
,
z0 z1 = x20 + y02

từ đó suy ra
x20 + y02
z1 = kx0 y0 − z0 =
z0
Ta thấy z1 nhận giá trị nguyên dương nên (x0 ; y0 ; z1 ) là một nghiệm nguyên dương của phương
trình ban đầu. Từ điều giả sử ta có

x0 + y0 + z0 6 x0 + y0 + z1

nên z0 6 z1 . Do đó ta được

x20 + y02 − kx0 y0 = z0 z1 − z1 − z0 = (z1 − 1) (z0 − 1) − 1 > y02 − 1

Suy ra 1 > x0 (ky0 − x0 ) > x0 (kx0 − x0 ) > x0 . Do x0 là số nguyên dương nên ta được x0 = 1.
Từ đây ta đưa phương trình ban đầu về thành y 2 + z 2 + 1 = kyz. Đến đây ta cần chỉ ra rằng
phương trình y 2 + z 2 + 1 = kyz có nghiệm nguyên dương khi và chỉ khi k = 3, tuy nhiên điều này
đơn giản nếu ta sử dụng phương pháp bước nhảy Viète. Do đó điều này mâu thuẫn với k 6= 3.

11
Hướng tới VMO 2020 Tạp chí và tư liệu toán học

2. Nếu y0 = z0 thì ta có

2y02 − kx20 y02 + x20 = 0 ⇒ x20 = y02 (kx0 − 2) > x20 (kx0 − 2)

Từ đó dẫn đến 3 > kx0 , mà ta lai có kx0 > 2 nên kx0 = 3, suy ra k = 1 hoặc k = 3, điều này
trái với k 6= 1 và k 6= 3.
Vậy với k 6= 1 và k 6= 3 thì phương trình đã cho không có nghiệm nguyên dương. Như vậy với k = 1
hoặc k = 3 thì phương trình đã cho có nghiệm nguyên dương. 
+
Lời giải 2. Với x, y, z ∈ Z , ta viết phương trình đã cho dưới dạng

x2 − kxyz + y 2 + z 2 = 0. (1)

Giả sử k là số nguyên dương sao cho phương trình (1) có nghiệm nguyên dương. Cố định k và xét tập
hợp
S = {(x, y, z); x, y, z ∈ Z+ | x2 − kxyz + y 2 + z 2 = 0}.
Theo điều giả sử ở trên thì S 6= ∅, khi đó theo nguyên lý sắp thự tự tốt tồn tại (x0 , y0 , z0 ) ∈ S sao
cho x0 + y0 + z0 là nhỏ nhất. Ta thấy rằng, nếu (x0 , y0 , z0 ) ∈ S thì các hoán vị của nó cũng thuộc S,
không mất tính tổng quát ta có thể giả sử x0 > y0 > z0 . Phương trình

f (x) = x2 − xky0 z0 + y02 + z02 = 0

hiển nhiên có một nghiệm x0 . Gọi nghiệm còn lại là x1 , theo định lý Viète, ta có

x0 + x1 = ky0 z0 ; x0 x1 = y02 + z02 .

Từ đây, ta được x1 không âm, do đó (x1 , y0 , z0 ) ∈ S, theo cách xác định của bộ (x0 , y0 , z0 ) thì ta thu
được x1 + y0 + z0 > x0 + y0 + z0 hay x1 > x0 . Do đó ta có

x 1 > x 0 > y0 > z 0 . (2)

Theo định lý về dấu của tam thức bậc hai và từ (2) ta được

0 6 f (y0 ) 6 y02 − ky02 z0 + 2y02 = y02 (3 − kz0 ).

Suy ra kz0 6 3 ⇒ k 6 kz0 6 3 mà k ∈ Z+ nên k ∈ {1, 2, 3}.


1. Nếu k = 1, phương trình (1) có nghiệm nguyên dương x = y = z = 3.
2. Nếu k = 2, thì từ kz0 6 3 ta được z0 = 1 khi đó ta có (x0 − y0 )2 + 1 = 0, mâu thuẫn.
3. Nếu k = 3, phương trình (1) có nghiệm nguyên dương x = y = z = 1.
Vậy với k ∈ {1, 3} thì phương trình có nghiệm nguyên dương. 

Nhận xét. Trong lời giải 1 ta có đề cập tới một kết quả đó là Phương trình y 2 + z 2 + 1 = kyz có
! nghiệm nguyên dương khi và chỉ khi k = 3, đây là bài toán trong đề thi HSG toán 9 Tỉnh Thanh
Hóa 2015 – 2016. Sau đây ta sẽ cùng xét tới bài toán này.

Bài toán 4. Tìm các số nguyên dương m để phương trình x2 − mxy + y 2 + 1 = 0 có nghiệm
nguyên dương.

Lời giải
Với các nghiệm nguyên dương (x; y) thỏa mãn phương trình, giả sử (x0 ; y0 ) là một nghiệm thỏa mãn
x0 + y0 nhỏ nhất. Do vai trò của x và y trong phương trình là như nhau nên không mất tính tổng quát

12
Chinh phục olympic toán Ứng dụng định lí Viète trong các bài toán số học

ta có thể giả sử x0 6 y0 .
Xét phương trình bậc hai có ẩn y là
y 2 − mx0 y + x20 + 1 = 0
La có y0 là một nghiệm của phương trình trên. Ta gọi nghiệm còn lại là y1 . Khi đó theo định lí Viète
ta có 
y0 + y1 = mx0
y0 .y1 = x20 + 1
Dễ dàng nhận thấy y1 có giá trị nguyên và từ cách chọn (x0 ; y0 ) ta suy ra được y0 6 y1 . Đến đây ta
xét các trường hợp sau.
1
1. Nếu x0 = y0 thì từ phương trình ban đầu ta được m = 2 + . Nên để m và x0 có giá trị nguyên
x20
thì x0 = 1 và m = 3.
Với m = 3 ta thấy (x; y) = (1; 1) là một nghiệm nguyên dương của phương trình đã cho.
2. Nếu y0 = y1 thì từ y0 .y1 = x20 + 1 hay
(y0 − x0 ) (y0 + x0 ) = 1
Từ đó ta suy ra được  
y0 − x 0 = 1 y0 = 1

y0 + x 0 = 1 x0 = 0
Trường hợp này loại vì (x0 ; y0 ) nguyên dương.
3. Nếu x0 < y0 < y1 khi đó ta được
y0 > x0 + 1; y1 > x0 + 2
Kết hợp với y0 .y1 = x20 + 1 ta được x20 + 1 > x20 + 3x0 + 2 ⇒ 3x0 + 1 6 0, điều này vô lý vì x0 > 0.
Như vậy để phương trình đã cho có nghiệm nguyên dương thì m = 3 và khi đó phương trình có nghiệm
nguyên dương là (x; y) = (1; 1). 

Bài toán 5. Tìm tất cả các số nguyên dương k sao cho phương trình x2 + y 2 + x + y = kxy có
nghiệm nguyên dương.

Lời giải
Gọi (x0 , y0 ) là bộ nghiệm nguyên dương của phương trình thỏa mãn x0 + y0 nhỏ nhất. Không mất
tính tổng quát, ta giả sử x0 > y0 > 1. Xét phương trình bậc hai ẩn x
x2 + y02 + x + y0 = kxy0 ⇔ x2 + x(1 − ky0 ) + y02 + y0 = 0
Phương trình bậc hai này hiển nhiên có một nghiệm x0 , gọi nghiệm còn lại là x1 . Theo định lí Viète ta
có 
x0 + x1 = ky0 − 1 (1)
x0 x1 = y02 + y0 (2)
Từ (1) ta có x1 nguyên, từ (2) ta có x1 dương. Như vậy (x1 , y0 ) cũng là một nghiệm thỏa mãn phương
trình, mặt khác, do tính nhỏ nhất của tổng x0 + y0 mà ta có x1 > x0 . Do đó từ (1), ta có
2x0 1
ky0 − 1 > 2x0 ⇒ + 6k
y0 y0
Ta có
 
x0 y0 1 1 x0 1 y0 1 1 k 1
k= + + + = + + + + 6 +1+1+
y0 x0 x0 y0 y0 2y0 x0 x0 2y0 2 2
⇒ k 6 5 ⇒ k ∈ {1; 2; 3; 4; 5}

13
Hướng tới VMO 2020 Tạp chí và tư liệu toán học

1. Với k = 1, ta có x2 + y 2 + x + y = xy, phương trình này vô nghiệm nguyên dương vì

x2 + y 2 + x + y > 2xy + x + y > xy

2. Với k = 2, tương tự như trên, ta cũng lập luận được phương trình này vô nghiệm nguyên dương.
3. Với k = 3, phương trình có nghiệm nguyên dương (2; 2).
4. Với k = 4 thì phương trình có nghiệm (1; 1).
5. Với k = 5, dấu bằng phải đồng thời xảy ra ở các điểm

x 1 k 5 1 1 y
+ = = , = 1, = 1, = 1
y 2y 2 2 x 2y x

Dễ thấy không tồn tại các số nguyên dương x, y thỏa mãn tất cả các điều trên. Trường hợp này
bị loại.
Vậy các giá trị của k thỏa mãn là k ∈ {3; 4}. 

Bài toán 6 [IMO 2007]. Cho trước a, b là hai số nguyên dương. Chứng minh rằng nếu 4ab − 1
là ước số của (4a2 − 1)2 thì a = b.

Lời giải
Theo giả thiết thì 4ab − 1 | (4a2 − 1)2 nên ta có 4ab − 1 là ước của

b2 (4a2 − 1)2 − (4ab − 1)(4a3 b − 2ab + a2 ) = (a − b)2 .

(a − b)2
Đến đây ta đặt k = thì k ∈ Z+ . Cố định k và xét tập hợp
4ab − 1

S = (a, b); a, b ∈ Z+ ; a 6= b | a2 − 2ab(2k + 1) + b2 + k = 0




Giả sử S 6= ∅, khi đó theo nguyên lý sắp thứ tự tốt tồn tại cặp số (a0 , b0 ) ∈ S sao cho a0 = 6 b0 và
a0 + b0 nhỏ nhất. Chú ý rằng, nếu (a0 , b0 ) ∈ S thì (b0 , a0 ) ∈ S, do vậy không mất tính tổng quát ta có
thể giả sử a0 > b0 . Phương trình T 2 − 2T b0 (2k + 1) + b20 + k = 0 có một nghiệm hiển nhiên là a0 . Gọi
nghiệm còn lại là a1 , theo định lý Viète ta có

a0 + a1 = 2b0 (2k + 1)
(1)
a0 a1 = b20 + k.

Suy ra a1 là số nguyên không âm, do đó (a1 , b0 ) ∈ S, theo cách xác định (a0 , b0 ) thì

a1 + b0 > a0 + b0 ⇔ a1 > a0 .

Kết hợp với (1) ta được


b20 + k b2 + k
a0 = 6 0 ⇔ k > a20 − b20 .
a1 a0
Như vậy ta suy ra
(a0 − b0 )2
= k > a20 − b20 = (a0 − b0 )(a0 + b0 ).
4a0 b0 − 1
Mặt khác lại do a0 > b0 nên a0 − b0 > 1, vì vậy

a0 − b0 > (a0 + b0 )(4a0 b0 − 1) > a0 + b0 ,

điều này là mâu thuẫn, do đó điều giả sử là sai hay S 6= ∅. 

14
Chinh phục olympic toán Ứng dụng định lí Viète trong các bài toán số học

a2 + b2
Bài toán 7. Chứng minh rằng nếu a, b là các số nguyên dương sao cho k = là số nguyên
ab − 1
thì k = 5.

Lời giải
Lời giải 1. Đẳng thức đề bài tương đương với a2 − kab + b2 + k = 0. Không mất tính tổng quát giả sử
a > b. Do a, b là các số nguyên dương và ab 6= 1 nên ta xét a = 2; b = 1 thì được k = 5.
Như vậy ta cần chứng minh là với a + b > 3 thì k = 5, giả sử cặp số dương (a0 ; b0 ) có tổng nhỏ nhất
thỏa mãn bài toán. Khi đó ta được

a20 − ka0 b0 + b20 + k = 0

Xét phương trình bậc 2 ẩn x là


a2 − kab + b2 + k = 0
Ta thấy rằng a0 là một nghiệm của phương trình. Như vậy theo định lí Viète thì phương trình trên còn
có nghiệm là a1 , khi đó ta có a = kb0 − a0 hay ta có cặp số (kb0 − a0 ; b0 ) thỏa mãn yêu cầu bài toán.
a0 k a2 + b2
Theo điều giả sử ta có a0 6 kb0 − a0 hay 6 . Từ k = ta suy ra
b0 2 ab − 1
a0 b0 k
+ + = k.
b0 a0 a0 b0
a0 k k k
Do 6 và a0 b0 > 3 nên k 6 + 1 + hay k 6 6. Mặt khác ta lại có
b0 2 2 3
a0 b0
+ >2
b0 a0
nên k > 3. Như vậy ta được 3 6 k 6 6.
1. Với a = 3; b = 1 ta tìm được k = 5.
2. Với a = b = 2 hay a = 4; b = 1 thì không tìm được giá trị của k.
3. Với ab > 5, lại dùng đánh giá tương tự như trên ta có k 6 3. Xét k = 3 thì a2 + 3ab + b2 = 3, ta
thấy không có cặp số dương (a; b) thỏa mãn.
Do đó suy ra ab > 6. Thử với a = 6; b = 1 hoặc a = 3; b = 2 đều không thỏa nên ta lại được ab > 7.
14
Lại dùng đánh giá như trên ta được suy ra k 6 , điều này mâu thuẫn với k nguyên và lớn hơn 2.
5
Vậy chỉ có k = 5 thỏa mãn bài toán. 
a2 + b2
Lời giải 2. Đặt k = thì k là số nguyên dương. Theo bất đẳng thức AM − GM thì
ab − 1
a2 + b2 2ab 2
k= > =2+ >2
ab − 1 ab − 1 ab − 1
2
hay k > 3. Nếu a = b thì ta được k = 2 + < 3, mâu thuẫn.
a2
−1
Ta sẽ chứng minh k = 5. Thật vậy, cố định k và xét tập hợp
a2 + b2
 
+
S = (a, b); a, b ∈ Z | k =
ab − 1

Theo giả thiết S 6= ∅, khi đó theo nguyên lý sắp thứ tự tốt tồn tại cặp số (a0 , b0 ) ∈ S sao cho a0 6= b0
và a0 + b0 nhỏ nhất. Ta thấy rằng nếu (a0 , b0 ) ∈ S thì (b0 , a0 ) ∈ S, không mất tính tổng quát ta có thể
giả sử a0 > b0 . Nhận thấy rằng phương trình
T 2 + b20
= k ⇔ T 2 − kT b0 + b20 + k = 0
T b0 − 1

15
Hướng tới VMO 2020 Tạp chí và tư liệu toán học

có một nghiệm hiển nhiên là a0 . Gọi nghiệm còn lại là a1 , theo định lý Viète ta có

a0 + a1 = kb0 ; a0 a1 = b20 + k.

Từ đây, ta được a1 ∈ Z+ , do đó (a1 , b0 ) ∈ S, theo cách xác định (a0 , b0 ) thì a1 + b0 > a0 + b0 hay
a1 > a0 . Vì a0 > b0 nên a0 > b0 + 1, từ đó ta thu được:

b20 + k − kb0 = a0 a1 − a0 − a1 = (a0 − 1)(a1 − 1) − 1 > b20 − 1.

Do đó k(b0 − 1) 6 1. Nếu b0 6= 1 theo chứng minh trên thì k(b0 − 1) > 3 > 1, vì vậy ta phải có b0 = 1.
Khi đó a0 + a1 = k và a0 a1 = k + 1, suy ra

a0 a1 − a0 − a1 − 1 = 0 ⇔ (a0 − 1)(a1 − 1) = 2

mà a1 > a0 nên a0 = 2, a1 = 3, từ đây ta được

k = a0 + a1 = 5.

Như vậy, ta có điều phải chứng minh. 

Bài toán 8. Tìm tất cả các giá trị k sao cho phương trình (x + y + z)2 = kxyz có nghiệm
nguyên dương.

Lời giải
Ta gọi k là giá trị cần tìm và (x0 ; y0 ; z0 ) nghiện nguyên dương của phương trình (x + y + z)2 = kxyz
có x0 + y0 + z0 nhỏ nhất. Khi đó không mất tính tổng quát, ta có thể giả sử rằng x0 > y0 > z0 . Phương
trình đã cho được viết dưới dạng

x2 − (kyz − 2y − 2z) x + (y + z)2 = 0

Theo định lí Viète ta có


(y0 + z0 )2
x1 = ky0 z0 − 2y0 − 2z0 − x0 =
x0
cũng là nghiệm của phương trình trên, suy ra (x1 ; y0 ; z0 ) là nghiệm của phương trình đầu. Ngoài ra ta
cũng suy ra được x1 nguyên dương, hay nói cách khác (x1 ; y0 ; z0 ) là nghiệm nguyên dương của phương
trình đầu. Từ tính nhỏ nhất của x0 + y0 + z0 ta có được x1 > x0 , suy ra

(y0 + z0 )2
ky0 z0 − 2y0 − 2z0 − x0 > x0 và > x0
x0

Từ bất đẳng thức thứ 2 ta có y0 + z0 > x0 , áp dụng bất đẳng thức thứ nhất ta được ky0 z0 > 4x0 . Chia
2 vế của
x20 + y02 + z02 + 2x0 y0 + 2y0 z0 + 2z0 x0 = kx0 y0 z0
cho x0 y0 z0 ta thu được
x0 y0 z0 2 2 2
+ + + + + =k
y0 z0 x0 z0 x0 y0 z0 x0 y0
k 32
Như vậy ta được + 1 + 1 + 2 + 2 + 2 > k hay k 6 , suy ra k 6 10. Chú ý rằng nếu x0 = 1 thì
4 3
k 1
y0 = z0 = 1 suy ra k = 9. Nếu k 6= 9 thì x0 > 2 và đánh giá ở trên trở thành + 1 + + 2 + 1 + 2 > k,
4 2
26
như thế thì ta suy ra được k 6 nên k 6 8. Giá trị k = 10 bị loại. Ta xét các trường hợp sau.
3
1. Với k = 1 phương trình có nghiệm, chẳng hạn (9; 9; 9).
2. Với k = 2 phương trình có nghiệm, chẳng hạn (4; 4; 8).

16
Chinh phục olympic toán Ứng dụng định lí Viète trong các bài toán số học

3. Với k = 3 phương trình có nghiệm, chẳng hạn (3; 3; 3).


4. Với k = 4 phương trình có nghiệm, chẳng hạn (2; 2; 4).
5. Với k = 5 phương trình có nghiệm, chẳng hạn (1; 4; 5).
6. Với k = 6 phương trình có nghiệm, chẳng hạn (1; 2; 3).
7. Với k = 8 phương trình có nghiệm, chẳng hạn (1; 1; 2).
8. Với k = 9 phương trình có nghiệm, chẳng hạn (1; 1; 1).
Bây giờ ta cần chứng chứng minh được rằng trường hợp k = 7 phương trình không có nghiệm nguyên
dương. Thật vậy, giả sử với k = 7 thì phương trình đã cho có nghiệm nguyên dương (x0 ; y0 ; z0 ) có các
tính chất như trên. Khi đó ta có
(x0 + y0 + z0 )2 x0 y0 z0 2 2 2
7= = + + + + +
x0 y0 z0 y0 z0 z0 x0 x0 y0 x0 y0 z0
1 1 y0 + z 0 2 2 2 10
< + + + + + 6
y0 z 0 y0 z0 x0 y0 z0 z0
10
Do đó ta có z0 < nên z0 = 1, khi đó ta có
7
3 3 4 1 1 2
7< + + ⇒1< + 6 ⇒ y0 < 2
x 0 y0 1 x 0 y0 y0

Từ đó tại được y0 = 1. Như vậy ta có

(x0 + y0 + z0 )2
r
(x0 + 2) 4 4
7= = = + x0 + 4 > 2. .x0 + 4 = 8
x0 y0 z0 x0 x0 x0

Như vậy khi k = 7 thì phương trình không có nghiệm nguyên dương.
Vậy các giá trị k cần tìm là k ∈ {1; 2; 3; 4; 5; 6; 8; 9}. 

Bài toán tương tự.


1. Cho các số nguyên dương x, y, z thỏa mãn điều kiện (x + y + z)2 chia hết cho xyz. Tính
! các giá trị của A =
(x + y + z)2
xyz
.
2. Chứng minh rằng phương trình (x + y + z)2 = 7xyz không có nghiệm nguyên dương.

Bài toán 9. Chứng minh rằng phương trình (x + y + z)2 = 7xyz không có nghiệm nguyên
dương.

Lời giải
Gọi (x0 ; y 0 ; z 0 ) là một nghiệm thỏa mãn phương trình với z 0 là số nhỏ nhất. Không mất tính tổng quát,
ta giả sử x0 6 y 0 6 z 0 , khi đó ta có

z 0 | (x0 + y 0 + z 0 )2 ⇒ z | (x0 + y 0 )2 + 2z 0 (x0 + y 0 ) + z 02 ⇒ z 0 | (x0 + y 0 )2

Ta xét phương trình bậc hai ẩn z là z 2 − (7x0 y 0 − 2x0 − 2y 0 )z + (x0 + y 0 )2 = 0, hiển nhiên phương trình
(x0 + y 0 )2
này có một nghiệm z 0 , nên theo định lí Viète thì nghiệm còn lại của nó là ∈ Z.
z0
0 0 2
 
(x + y )
Như vậy x0 ; y 0 ; cũng là một bộ số thỏa mãn phương trình. Nếu giả sử
z0

(x0 + y 0 )2
x0 + y 0 < z 0 ⇒ < z0
z0

17
Hướng tới VMO 2020 Tạp chí và tư liệu toán học

thì vô lí vì (x0 ; y 0 ; z 0 ) cũng là một bộ số thỏa mãn phương trình và vì tính nhỏ nhất của z 0 . Do đó phải
có z 0 6 x0 + y 0 . Khai triển phương trình ban đầu và chia hai vế của nó cho x0 y 0 z 0 ta được
x0 y0 z0 2 2 2 1 1 x0 + y 0 2 2 2
76 0 0
+ 0 0
+ 0 0
+ 0
+ 0
+ 0
6 0
+ 0
+ 0 0
+ 0+ 0+ 0
yz zx xy x y z z x xy x y z
4 3 3 10 10
= 0 + 0 + 0 6 0 ⇒ x0 6 ⇒ x0 = 1
x y z x 7
Khi đó ta được y 0 6 z 0 6 y 0 + 1 ⇒ z 0 = y ∨ z 0 = y 0 + 1. Xét các trường hợp
1. Nếu z 0 = y 0 thì ta có phương trình

0 2 02 02 02± 7 0
(1 + 2z ) = 7z ⇔ 3z − 4z − 1 = 0 ⇔ z =
3
trường hợp này loại.
2. Nếu z 0 = y 0 + 1 thì ta có phương trình
 
0 2 0 0 02 0 4
(2 + 2z ) = 7z (z + 1) ⇔ 3z − z − 4 = 0 ⇔ z ∈ −1;
3

Như vậy không tồn tại nghiệm nguyên dương của phương trình đã cho. 

Bài toán 10 [VMO 2012]. Xét các số tự nhiên lẻ a, b thỏa mãn a | b2 + 2 và b | a2 + 2. Chứng
minh rằng a, b là các số hạng của dãy số tự nhiên (vn ) được xác định bởi công thức

v1 = v2 = 1
vn = 4vn−1 − vn−2 , ∀n > 2

Lời giải
Đầu tiên ta sẽ đi chứng minh rằng b | a2 + 2 ∧ a | b2 + 2 ⇔ ab | a2 + b2 + 2. Thật vậy, ta có

b | a2 + 2 ∧ a | b2 + 2 ⇒ ab | a2 + 2 b2 + 2 ⇒ ab | 2 a2 + b2 + 2
  

Do a, b lẻ nên ab | a2 + b2 + 2. Ngược lại nếu có ab | a2 + b2 + 2 thì dễ dàng suy ra ngay được b | a2 + 2


và a | b2 + 2. Từ đó giả thiết đề bài tương đương với việc tồn tại số nguyên dương k sao cho

a2 + b2 + 2 = kab.

Sử dụng phương pháp bước nhảy Viète ta sẽ đi chứng minh k = 4. Cố định k và xét tập
a2 + b2 + 2
 
2
S = a, b ∈ Z+ | k = ∈ N∗
ab

Trong S ta chọn ra cặp (A, B) sao cho tổng A + B là nhỏ nhất. Không mất tính tổng quát, ta giả
sử A > B. Xét phương trình bậc hai ẩn t, t2 − ktB + B 2 + 2 = 0. Dễ thấy phương trình này có một
nghiệm là A, gọi nghiệm còn lại là t0 . Theo định lí Viète ta có

t0 + A = kB
t0 A = B 2 + 2

Từ đây suy ra được t0 nguyên dương. Chú ý vì A + B là nhỏ nhất nên ta được t0 > A.
A k
Suy ra t0 + A = kB > 2A hay 6 .
B 2
Nếu có một trong hai số a, b bằng 1, giả sử b = 1 thì ka = a2 + 3, dễ suy ra k = 4.
Nếu cả hai số a, b > 2. Ta có A > B > 2. Thì
A B 2 k 2
k= + + 6 +1+ ⇔k63
B A AB 2 2.2

18
Chinh phục olympic toán Ứng dụng định lí Viète trong các bài toán số học

Theo bất đẳng thức AM − GM ta có

kab = a2 + b2 + 2 > 2(ab + 1) ⇒ k > 3

lúc này ta được k = 3. Khi đó A2 + B 2 + 2 = 3AB. Từ đẳng thức này dễ dàng suy ra phải có một
trong hai số chia hết cho 3, giả sử 3 | A thì A > 3. Nếu B = 1 ta gặp mâu thuẫn, do đó B > 2. Tức
AB > 6. Tuy nhiên
A B 2 3 2
3= + + 6 +1+
B A AB 2 6
Điều này vô lí. Vậy k = 4 là giá trị duy nhất cần tìm. Ta chứng minh xong việc các số a, b thỏa giả
thiết thì cũng phải thỏa mãn phương trình

a2 + b2 + 2 = 4ab (1)

Bài toán sẽ hoàn tất nếu ta chỉ rằng nếu cặp (a, b) bất kỳ thỏa mãn (1) thì sẽ luôn tồn tại số tự nhiên
n sao cho a = xn , b = xn+1 . Giả sử (u0 , u1 ) là một cặp số nguyên dương bất kỳ thỏa (1). Ta hoàn
toàn có quyền giả sử u0 > u1 . Nếu u0 = 1 thì u1 = 1, tức tồn tại n = 1 để u0 = v1 , u1 = v2 . Tương tự
khi xét u1 = 1. Do đó ta chỉ cần xét u0 , u1 > 1. Khi đó ta chọn cặp (u1 , u2 ) = (u1 , 4u1 − u0 ), dễ thấy
u1 , u2 nguyên dương và (u1 , u2 ) cũng thỏa mãn (1). Lúc này ta chú ý 4u1 − u0 < u0 vì

u21 + 2 2 − (u0 − u1 )(u0 + u1 )


4u1 − u0 = − u0 = 6 0.
u0 u0

Suy ra u1 + u2 = u1 + (4u1 − u0 ) < u1 + u0 . Tương tự ta cũng chọn được cặp (u2 , u3 ) = (u2 , 4u2 − u1 )
cũng thỏa u2 , u3 nguyên dương, cũng thỏa (1) và u2 + u3 < u1 + u2 < u1 + u0 . Cứ tiếp tục quá trình
này, ta được
... < ui + ui+1 < ... < u1 + u2 < u1 + u0
Thế nhưng u1 + u0 > 2 nên phải tồn tại k sao cho uk + uk+1 = 2, suy ra uk = uk+1 = 1. Tức là ta có
uk = v2 , uk+1 = v1 . Ta có thể thấy được cách xác định un là như sau

un+2 = 4un+1 − un hay un = 4un+1 − un+2 .

Từ đó ta có
uk−1 = 4uk − uk+1 = 4v2 − v1 = v3
uk−2 = 4uk−1 − uk = 4v3 − v2 = v4
...
u1 = 4u2 − u3 = 4vk − vk−1 = vk+1
Như vậy tồn tại n = k + 1 để với cặp (u0 , u1 ) bất kỳ thỏa (1) thì ta có (u1 , u0 ) = (vk+1 , vk+2 ). 

Bài toán tương tự [Canada MO 1998]. Cho m là một số nguyên dương. Dãy số (un ) với
n > 0 được xác định như sau 
u0 = 0, u1 = m
un+1 = m2 un − un−1
! với mọi n > 1. Chứng minh rằng, các cặp số (a, b) với a, b ∈ Z , a > b, là nghiệm của phương
+

a2+ b2
trình = m2 khi và chỉ khi (a, b) = (un , un+1 ) với mọi số tự nhiên n.
ab + 1

Bài toán 11 [Vietnam TST 1992]. Tìm tất cả các nghiệm nguyên dương (x, y) của phương
trình
x2 − 5xy + y 2 + 5 = 0. (1)

Lời giải

19
Hướng tới VMO 2020 Tạp chí và tư liệu toán học

Lời giải và bình luận bài toán này chúng tôi xin được trích từ chuyên đề bước nhảy Viète của thầy Hà
Tuấn Dũng - Khoa Toán - ĐH Sư Phạm Hà Nội 2.
Đầu tiên, chúng ta chứng minh bổ đề.
Bổ đề. Xét hai dãy số (un ) và (vn ) được xác định như sau

u0 = 1, u1 = 2, un+2 = 5un+1 − un , ∀n = 0, 1, . . . ;
v0 = 1, v1 = 3, vn+2 = 5vn+1 − vn , ∀n = 0, 1, . . .

Khi đó, với mọi n ∈ N các cặp số (un , un+1 ) và (vn , vn+1 ) là nghiệm nguyên dương của (1).
Chứng minh. Ta sẽ chứng minh mệnh đề sau bằng phương pháp quy nạp toán học.
Với n = 0, ta có u21 + u20 − 5u0 u1 = −5. Do đó (u0 , u1 ) là nghiệm của phương trình (1). Như vậy, mệnh
đề đúng với n = 0.
Giả sử mệnh đề đúng với n = k > 0, tức là

u2k + u2k+1 − 5uk uk+1 + 5 = 0.

Khi đó

u2k+1 + u2k+2 − 5uk+1 uk+2 = uk+2 (uk+2 − 5uk+1 ) + u2k+1 = u2k+1 + u2k − 5uk uk+1 .

Từ giả thiết quy nạp, ta được

u2k+1 + u2k+2 − 5uk+1 uk+2 + 5 = 0.

Do đó (uk+1 , uk+2 ) cũng là nghiệm của phương trình (1). Theo nguyên lý quy nạp toán học thì mệnh
đề đúng với mọi n ∈ N. Chứng minh tương tự, ta cũng thu được với mọi n ∈ N thì (vn , vn+1 ) là nghiệm
của phương trình (1). Từ công thức xác định số hạng tổng quát của hai dãy số (un ) và (vn ) ta được
các số hạng của hai dãy đều là các số nguyên dương. Do đó, các cặp số (un , un+1 ) và (vn , vn+1 ) là
nghiệm nguyên dương của phương trình (1). Như vậy bổ đề được chứng minh. ∇
Quay lại bài toán. Xét tập hợp

S = {(a, b); a, b ∈ Z+ | a2 − 5ab + b2 + 5 = 0}.

Với (a, b) ∈ S nếu a = b thì ta có


5
3a2 − 5 = 0 ⇒ a2 = ,
3
điều này mâu thuẫn. Do đó a 6= b. Ta thấy rằng (a, b) ∈ S thì (b, a) ∈ S, không giảm tính tổng quát ta
có thể giả sử rằng với mọi (a, b) ∈ S thì a < b. Với (a, b) là một phần tử bất kì thuộc S. Xét dãy số
(an ) được xác định như sau

a0 = b, a1 = a, an+2 = 5an+1 − an , ∀n ∈ N.

Ta có b(5a − b) = a2 + 5 > 0 ⇒ 5a > b. Từ công thức xác định số hạng tổng quát của dãy số (an ) ta
được an ∈ Z+ với mọi n ∈ N. Ta có (a0 , a1 ) = (a, b) ∈ S, giả sử (ak , ak+1 ) ∈ S với mọi k > 1, khi đó

a2k+1 + a2k+2 − 5ak+1 ak+2 = ak+2 (ak+2 − 5ak+1 ) + a2k+1 = a2k+1 + a2k − 5ak ak+1 .

Từ đây ta được (ak+1 , ak+2 ) ∈ S, theo nguyên lý quy nạp toán học thì (an , an+1 ) ∈ S, ∀n ∈ N. Nếu
a = 1 thì từ (1) ta được b2 − 5b + 6 = 0 ⇔ b ∈ {2, 3}..
1. Nếu b = 2 ta có (a, b) = (u0 , u1 ).
2. Nếu b = 3 thì (a, b) = (v0 , v1 ).
Ta xét trường hợp a > 1, khi đó (4a − b)(a − b) = 3a2 − 5 > 0, mà a < b nên 4a < b, mà a0 > a1 nên từ
đây ta được an > an+1 với mọi n ∈ N. Như vậy với a > 1 thì dãy (an ) là một dãy giảm ngặt, nên phải
tồn tại một chỉ số k sao cho a0 > a1 > · · · > ak+1 = 1. Do (ak , ak+1 ) là một nghiệm của phương trình
(1) nên ta có ak ∈ {2, 3}. Với ak = 2 thì ta có ak+1 = u0 , ak = u1 , khi đó ak−1 = 5ak − ak+1 = u2 , từ
đó ta được ai = uk+1−i . Tương tự với ak = 3 thì (a, b) là các số hạng liên tiếp trên dãy (vn ). Như vậy,

20
Chinh phục olympic toán Ứng dụng định lí Viète trong các bài toán số học

các bộ (un , un+1 ) và (vn , vn+1 ) (với mọi n ∈ N) là tập tất cả các nghiệm nguyên dương của phương
trình (1). 

Nhận xét. Ta thiết lập quan hệ thứ tự trên S như sau nếu (x, y) ∈ S, (x0 , y 0 ) ∈ S thì

(x, y) > (x0 , y 0 ) ⇔ x > x0 và y > y 0

Từ một nghiệm bất kì của phương trình (1) bằng phương pháp bước nhảy Viète ta thiết lập được
mới nhỏ hơn nghiệm (a, b) theo quan hệ thứ tự nói trên. Từ nghiệm mới vừa thu được này ta lại
xây dựng nghiệm mới nhỏ hơn, cứ tiếp tục quá trình như vậy đến khi không thể xây dựng được
nữa. Khi đó, ta thu được nghiệm nhỏ nhất. Dãy (an ) đã mô tả các nghiệm của phương trình (1)
được xây dựng từ quá trình trên và được xây dựng dựa vào các tính chất: a, b là hai số hạng đầu
tiên của dãy: (ai , ai+1 ) là một nghiệm của phương trình (1). Để xác định được công thức truy

! hồi của dãy (an ) ta đã sử dụng phương pháp bước nhảy Viète. Xét phương trình

T 2 − 5T an+1 + a2n+1 + 5 = 0

có một nghiệm là an , gọi nghiệm còn lại là an+2 thì theo hệ thức Viète ta có

an + an+2 = 5an+1
(2)
an an+2 = a2n+1 + 5

Từ đây, ta có an+2 là số nguyên dương, do đó (an , an+2 ) cũng là một nghiệm của phương trình,
và từ (2) ta được an+2 = 5an+1 − an . Sau khi thu được nghiệm nhỏ nhất, ta xây dựng các nghiệm
của phương trình từ nghiệm nhỏ nhất đó thông qua hai dãy (un ) và (vn ).

Bài toán 12 [VMO 2002]. Tìm tất cả các giá trị nguyên dương k sao cho phương trình

(x + y + z + t)2 = k 2 xyzt

có nghiệm nguyên dương.

Lời giải
Biến đổi phương trình ban đầu về dạng

x2 + (2y + 2z + 2t − k 2 yzt)x + (y + z + t)2 = 0

Trong các nghiệm nguyên dương của phương trình, ta chọn ra bộ nghiệm (x0 , y0 , z0 , t0 ) có tổng
x0 + y0 + z0 + t0 nhỏ nhất. Khi đó dễ thấy x0 là một nghiệm của phương trình bậc hai

x2 + (2y0 + 2z0 + 2t0 − k 2 y0 z0 t0 )x + (y0 + z0 + t0 )2 = 0 (*)

Gọi nghiệm còn lại của (∗) là x1 , theo định lí Viète ta có

x0 + x1 = k 2 y0 z0 t0 − 2y0 − 2z0 − 2t0 (1)




x0 .x1 = (y0 + z0 + t0 )2 (2)

Từ (1) ta có x1 nguyên và từ (2) ta có x1 dương. Như vậy (x1 , y0 , z0 , t0 ) cũng là một bộ số thỏa (∗),
nhưng vì tính nhỏ nhất của tổng x0 + y0 + z0 + t0 mà ta có x1 > x0 . Do đó từ (2) ta suy ra

(y0 + z0 + t0 )2
x1 = > x0 ⇒ y0 + z0 + t0 > x0
x0
Kết hợp với (1) ta được

k 2 y0 z0 t0 − 2y0 − 2z0 − 2t0 − x0 > x0 ⇒ k 2 y0 z0 t0 > 2x0 + 2(y0 + z0 + t0 ) > 4x0 .

21
Hướng tới VMO 2020 Tạp chí và tư liệu toán học

Chia hai vế của đẳng thức x20 + y02 + z02 + t20 + 2x0 y0 + 2x0 z0 + 2x0 t0 + 2y0 z0 + 2y0 t0 + 2z0 t0 = k 2 x0 y0 z0 t0
cho x0 y0 z0 t0 , ta được
x0 y0 z0 t0 2 2 2 2 2 2
+ + + + + + + + + = k2
y0 z0 t0 x0 z0 t0 x0 y0 t0 x0 y0 z0 z0 t0 y0 t0 y0 z0 x0 t0 x0 z0 x0 y0
Không mất tính tổng quát ta giả sử rằng x0 > y0 > z0 > t0 > 1. Khi đó suy ra
x0 k2 y0 1 z0 1 t0 1
6 , 6 61 , 6 6 1, 6 61
y0 z0 t0 4 z0 t0 x0 z0 t0 x 0 y0 t 0 x0 t 0 x 0 y0 z0 x 0 y0
Như vậy ta được
k2 k2
k2 6 +1+1+1+2+2+2+2+2+2= + 15 ⇒ k 2 6 20 ⇒ k ∈ {1, 2, 3, 4}
4 4
1. Nếu k = 1, phương trình có nghiệm (4, 4, 4, 4).
2. Nếu k = 2, phương trình có nghiệm (2, 2, 2, 2).
3. Nếu k = 3, phương trình có nghiệm (1, 1, 2, 2).
4. Nếu k = 4, phương trình có nghiệm (1, 1, 1, 1).
Như vậy để phương trình có nghiệm nguyên dương thì tập hợp tất cả các giá trị nguyên dương của k
là k ∈ {1, 2, 3, 4}. 

a2
Bài toán 13 [IMO 2003]. Hãy tìm tất cả các cặp số nguyên dương (a; b) sao cho
2ab2 − b3 + 1
là một số nguyên dương.

Lời giải
a2
Giả sử tồn tại cặp số nguyên dương (a, b) thỏa mãn điều kiện bài toán. Đặt k = thì k là
2ab2 − b3 + 1
một số nguyên dương. Cố định k và xét tập hợp
S = {(a, b); a, b ∈ Z+ | a2 − 2akb2 + k(b3 − 1) = 0}.
Như vậy ta ta có S 6= ∅. Do k ∈ Z+ nên với (a, b) ∈ S ta có 2ab2 − b3 + 1 > 0 suy ra
b2 (2a − b) > −1 ⇒ b2 (2a − b) > 0.
Do đó 2a = b hoặc 2a > b. Nếu 2a > b thì do k > 1 nên ta được
a2 > 2ab2 − b3 + 1 > b2 (2a − b) > b2 .
Từ đó suy ra nếu (a, b) ∈ S thì 2a = b hoặc a > b. Gọi (a0 , b0 ) là một phần tử bất kì thuộc S. Xét
phương trình T 2 − 2T kb20 + k(b30 − 1) = 0 là phương trình bậc hai ẩn T có một nghiệm là a0 . Gọi
nghiệm còn lại a1 , theo định lí Viète ta có
a0 + a1 = 2kb20 

(1)
a0 a1 = k b30 − 1
Như vậy ta được a1 ∈ Z và a1 > 0. Nếu a1 = 0, thì từ (1) ta có b0 = 1 và a0 = 2k, như thế thì (2k, 1)
là một cặp số thỏa mãn điều kiện bài toán. Nếu a1 ∈ Z+ thì (a1 , b0 ) ∈ S. Không giảm tính tổng quát
ta có thể giả sử a1 > a0 . Chú ý rằng, theo nhận xét ở trên thì 2a0 = b0 hoặc a0 > b0 . Nếu a0 > b0 thì
ta có ngay a1 > a0 > b0 , kết hợp với (1) ta thu được
k(b30 − 1) k(b30 − 1)
kb20 6 a1 = 6 < kb20 .
a0 b0
Điều này mâu thuẫn. Với 2a0 = b0 thì ta được (k, 2k) là một cặp số thỏa mãn điều kiện bài toán. Từ
hệ thức a0 a1 = k(b30 − 1) ta thu được (8k 3 − 1, 2k) là một cặp số cần tìm. Vậy các cặp số (a, b) thỏa
mãn điều kiện bài toán (2k, 1), (k, 2k) và (8k 3 − 1, 2k) với k là số nguyên không âm.

22
Chinh phục olympic toán Ứng dụng định lí Viète trong các bài toán số học

Bài toán 14. Cho phương trình x2 + y 2 + z 2 + t2 − N xyzt − N = 0 trong đó N là một số nguyên
dương cho trước.
a) Chứng tỏ rằng, có vô số giá trị nguyên dương N để phương trình trên có nghiệm nguyên
dương (nghĩa là mỗi nghiệm gồm 4 số nguyên dương x, y, z, t).
b) Cho N = 4k (8m + 7) với k, m là các số nguyên không âm. Chứng minh rằng, khi đó phương
trình trên không có nghiệm nguyên dương.

Lời giải
a) Biến đổi phương trình tương đương
x2 + y 2 + z 2 + t2 − N xyzt − N = 0 (1)
⇔ t(t − N xyzt) = N − x2 + y 2 + z 2

(2)
Với ba số nguyên dương bất kỳ a, b, c và N = a2 + b2 + c2 thì dễ thấy phương trình (2) có nghiệm
x0 = a, y0 = b, z0 = c, t0 = N abc = a2 + b2 + c2 abc

(*)
Chú ý rằng khi hoán vị bốn số a, b, c, N abc ta lại được nghiệm (x1 , y1 , z1 , t1 ) của phương trình (1).
b) Giả sử phương trình (1) có nghiệm nguyên dương, chọn (x0 , y0 , z0 , t0 ) là nghiệm nguyên dương của
(1) sao cho tổng x0 + y0 + z0 + t0 là số nguyên dương nhỏ nhất. Không làm mất tính chất tổng quát,
giả sử rằng x0 6 y0 6 z0 6 t0 . Ta sẽ chứng minh rằng với N > 7 thì nghiệm nguyên dương của phương
trình (1) với x0 6 y0 6 z0 6 t0 nếu có phải có dạng (∗) như trên.
Theo giả thiết t0 là nghiệm của phương trình bậc hai
t2 − N x0 y0 z0 t + x20 + y02 + z02 − N = 0 (3)
Phương trình (3) có nghiệm thứ hai t1 thoả mãn

t1 + t0 = N (x0 y0 z0 ) (4)
t1 · t0 = x20 + y02 + z02 − N (5)

Từ (4) suy ra t1 ∈ Z. Lại theo giả thiết ta có


N (1 + x0 y0 z0 t1 ) = t21 + x20 + y02 + z02 > 0
1
nên ta được t1 > − , vì t1 ∈ Z nên t1 > 0. Giả sử t1 > 0 khi đó (x0 , y0 , z0 , t1 ) là nghiệm nguyên
x0 y0 z0
dương của (1). Do cách chọn (x0 , y0 , z0 , t0 ) thì
x0 + y0 + z0 + t1 > x0 + y0 + z0 + t0 ⇒ t1 > t0
Từ đó theo (5) ta có
t20 6 t1 t0 = x20 + y02 + z02 − N < x20 + y02 + z02 6 3z02
Ta có
N 1 + x0 y0 z02 6 N (1 + x0 y0 z0 t0 ) = x20 + y02 + z02 + t20 6 z02 + z02 + z02 + 3z02 = 6z02


Từ đó, do N > 7, nên ta suy ra được


N 1 + x0 y0 z02 6 6z02 < N z02 ⇒ 1 + x0 y0 z02 < z02


Điều vô lý này chứng tỏ t1 > 0 là sai, suy ra t1 = 0. Từ (4), (5) suy ra


N = x20 + y02 + z02


t0 = N x0 y0 z0

là nghiệm (∗) của phương trình (1). Với N = 4k (8m + 7) > 7, áp dụng kết quả trên thì N = x2 + y 2 + z 2 .
Do đó nếu chứng minh được phương trình x2 + y 2 + z 2 = 4k (8m + 7) không có nghiệm nguyên dương
thì phương trình (1) cũng không có nghiệm nguyên dương. Ta xét các trường hợp sau.

23
Hướng tới VMO 2020 Tạp chí và tư liệu toán học

1. Khi k = 0 ta có x2 + y 2 + z 2 = 8m + 7 hay x2 + y 2 + z 2 ≡ 7(mod8). Trong ba số x, y, z phải có


một số lẻ hoặc cả ba số lẻ. Nếu số a lẻ thì a2 ≡ 1 (mod 8), do đó x2 + y 2 + z 2 6= 7(mod8).
2. Khi k > 0 ta có
x2 + y 2 + z 2 = 4k (8m + 7) (**)
hay x2 + y 2 + z 2 ≡ 0(mod4). Trong ba số x, y, z phải có một số chẵn hoặc ba số chẵn. Nếu có
một số chẵn, còn hai số a, b lẻ thì a2 + b2 ≡ 2(mod4), suy ra x2 + y 2 + z 2 =
6 7(mod8). Nếu x, y, z
đều chẵn, đặt x = 2x1 , y = 2y1 , z = 2z1 thì (∗∗) tương đương với

x2 + y 2 + z 2 = 4k−1 (8m + 7)

Sau k lần biến đổi như thế ta có x2 + y 2 + z 2 = 8m + 7, nhưng phương trình này vô nghiệm
nguyên dương như khi xét k = 0.
Bài toán được giải quyết. 

5 Các bài toán tổng hợp.


5.1 Đề bài
a2 + ab + b2
Câu 1. Cho a, b, k là các số nguyên dương thỏa mãn k = . Chứng minh rằng k là một số
ab + 1
chính phương.
ab(5a2 + 5b2 − 2)
Câu 2. Cho các số nguyên dương a, b thỏa mãn ∈ Z. Chứng minh rằng a = b.
5ab − 1
x + y 2 + 30
2
Câu 3. Cho các số nguyên dương x, y, A thỏa mãn hệ thức A = . Chứng minh rằng A là
xy
lũy thừa bậc năm của một số nguyên.
x2 + y 2 + z 2
Câu 4. Cho các số nguyên dương x, y, z thỏa mãn nhận giá trị nguyên dương. Chứng
xyz + 1
x2 + y 2 + z 2
minh rằng có thể biểu diễn được thành tổng của hai số chính phương.
xyz + 1
Câu 5. Cho a, b, c là các số nguyên dương thỏa mãn 0 < a2 +b2 −abc 6 c. Chứng minh rằng a2 +b2 −abc
là số chính phương.
Câu 6. Cho m > n là các số nguyên dương lẻ và n2 − 1 chia hết cho m2 − n2 + 1. Chứng minh rằng
m2 − n2 + 1 là một số chính phương.
Câu 7. Cho x và y là các số nguyên dương thỏa mãn điều kiện x2 + y 2 + 1 chia hết cho 2xy + 1. Chứng
minh rằng x = y.
Câu 8. Cho a, b là các số nguyên dương lẻ thỏa mãn a2 + 2 chia hết cho b và b2 + 2 chia hết cho a.
a2 + b2 + 2
Chứng minh rằng là số chính phương.
ab
Câu 9. Cho các số nguyên dương a, b, c, d thỏa mãn b2 + 1 = ac và c2 + 1 = bd. Chứng minh rằng
a + c = 3b và b + d = 3c.
Câu 10. Giả sử phương trình x2 + y 2 + x + y + 1 = xyz có nghiệm nguyên dương. Tìm tất cả các giá
trị của z.
Câu 11. Tìm các số nguyên dương x và y sao cho x + 1 chia hết cho y và y + 1 chia hết cho x.
Câu 12. Tìm các số nguyên dương x, y để x2 + 2 chia hết cho xy + 1.
Câu 13. Tìm tất cả các số có ba chữ số chia hết cho 11 sao cho thương số của phép chia số đó cho 11
bằng tổng bình phương của các chữ số của số đó.
Câu 14 [Kiran Kedlaya]. Cho các số nguyên dương a, b, c là thỏa mãn (ab + 1) (bc + 1) (ca + 1) là
số chính phương. Chứng minh rằng ba số ab + 1; bc + 1; ca + 1 đều là số chính phương.
Câu 15. Tồn tại hay không năm số nguyên dương a1 ; a2 ; a3 ; a4 ; a5 thỏa mãn hệ điều kiện
 2
 a2 + 1 = (a1 + 1) (a3 + 1)
a2 + 1 = (a2 + 1) (a4 + 1)
 32
a4 + 1 = (a3 + 1) (a5 + 1)

24
Chinh phục olympic toán Ứng dụng định lí Viète trong các bài toán số học

5.2 Hướng dẫn giải - Lời giải

a2 + ab + b2
Câu 1. Cho a, b, k là các số nguyên dương thỏa mãn k = . Chứng minh rằng k là
ab + 1
một số chính phương.

Lời giải. Trước tiên ta sẽ cố định k và xét tập


a2 + ab + b2
 
S = (a, b) ∈ N × N | k =
ab + 1
Giả sử phản chứng k không là số chính phương, khi đó trong các phần tử của S ta chọn ra cặp (A, B)
thỏa mãn điều kiện A + B là nhỏ nhất. Không mất tính tổng quát, ta giả sử rằng A > B > 0. Xét
phương trình bậc hai ẩn x
x2 + xB + B 2
k= ⇔ x2 + (B − kB)x + B 2 − k = 0
xB + 1
Phương trình này hiển nhiên có hai nghiệm là A và x0 , khi đó theo định lí Viète ta có

x0 + A = kB − B (1)
x0 .A = B 2 − k (2)
Từ (1) ta suy ra được x0 là số nguyên.
1. Nếu x0 < 0 thì x0 6 −1 ⇒ x2 − (Bk − B)x + B 2 − k > x2 + (Bk − B) + B 2 − k > 0, điều này là
mâu thuẫn.
2. Nếu x0 = 0 thì k = B 2 là một số chính phương, trường hợp này loại.
3. Nếu x0 > 0 thì (x0 , B) ∈ S.
Như vậy ta được
B2 − k B2 A2
x0 + B = +B < +B < +B =A+B
A A A
điều này mâu thuẫn với tính nhỏ nhất của tổng A + B. Do đó giả thiết phản chứng là sai, từ đó ta có
k phải là một số chính phương. 

ab(5a2 + 5b2 − 2)
Câu 2. Cho các số nguyên dương a, b thỏa mãn ∈ Z. Chứng minh rằng a = b.
5ab − 1

5a2 + 5b2 − 2 5a2 + 5b2 − 2


Lời giải. Vì gcd(ab, 5ab − 1) = 1 nên ta có ∈ Z. Đặt = k ∈ Z, ta dễ dàng
5ab − 1 5ab − 1
có được
5a2 + 5b2 − 2
5(a2 + b2 ) − 2 > 10ab − 2 > 5ab − 1 ⇒ k = > 1 ⇒ k > 2.
5ab − 1
5a2 + 5b2 − 2
 
Xét tập S = (a, b) ∈ Z+ × Z+ | k = ∈ Z . Cố định k và trong các phần tử của S, ta
5ab − 1
chọn ra cặp số (A, B) nguyên dương thỏa mãn tổng A + B nhỏ nhất. Gỉa sử A 6= B, không mất tính
tổng quát, xét A > B. Xét phương trình bậc hai ẩn x
5x2 + 5B 2 − 2
= k ⇔ 5x2 − 5xBk + 5B 2 + k − 2 = 0
5xB − 1
Dễ thấy phương trình này có một nghiệm là A, gọi nghiệm còn lại là x0 . Theo định lí Viète, ta có

 A + x0 = Bk (1)
2
5B + k − 2
Ax0 = (2)
5
Từ (1) ta có x0 nguyên.

25
Hướng tới VMO 2020 Tạp chí và tư liệu toán học

1. Nếu x0 < 0 thì x0 6 −1 ⇒ 5x20 − 5x0 Bk + 5B 2 + k − 2 > 5 + 5Bk + 5B 2 + k − 2 > 0, điều này là
mâu thuẫn.
2. Nếu x0 > 0 thì (x0 , B) ∈ S. Khi đó do tính nhỏ nhất của tổng A + B mà ta có
5B 2 + k − 2 5A2 + 5B 2 − 2
x0 > A ⇒ >A⇔ − 2 > 5(A − B)(A + B)
5A 5AB − 1
5(A − B)2
⇔ > 5(A − B)(A + B)
5AB − 1
⇔ A − B > (A + B)(5AB − 1).
Rõ ràng điều này vô lí.
Như vậy phải có x0 = 0, suy ra 5B 2 = 2 − k > 0, lại có k > 2, do đó k = 2. Suy ra
5a2 + 5b2 − 2
= 2 ⇔ (a − b)2 = 0 ⇔ a = b
5ab − 1
Bài toán được giải quyết. 

x2 + y 2 + 30
Câu 3. Cho các số nguyên dương x, y, A thỏa mãn hệ thức A = . Chứng minh rằng
xy
A là lũy thừa bậc năm của một số nguyên.

Lời giải. Gọi (x0 ; y0 ) là cặp số thỏa mãn đề bài và có tổng x0 + y0 nhỏ nhất. Ta giả sử x0 6 y0 . Xét
phương trình bậc hai ẩn y
y 2 − A.x0 .y + x20 + 30 = 0 (*)
Vì (x0 ; y0 ) thỏa mãn đề bài nên y0 là một nghiệm của phương trình (∗). Gọi nghiệm còn lại là y1 . Theo
định lí Viète ta có 
y0 + y1 = Ax0 (1)
y0 y1 = x20 + 30 (2)
Ta có x0 , y0 , A ∈ Z nên từ (1) suy ra y1 ∈ Z. Các cặp (x0 ; y0 ) ; (x0 ; y1 ) đều thỏa mãn (∗) mà x0 + y0
nhỏ nhất nên ta được
x0 + y0 6 x0 + y1 ⇔ y0 6 y1 .
Như vậy x0 6 y0 6 y1 . Ta xét các trường hợp sau.
• Trường hợp 1. Nếu x0 = y0 khi đó ta thay vào A thì ta được
30
A = 2 + 2 ∈ Z ⇒ x0 = 1 ⇒ A = 32
x0

• Trường hợp 2. Nếu y0 = y1 thì từ (2) ta được


x20 + 30 = y02 ⇔ (y0 + x0 ) (y0 − x0 ) = 30
Dễ thấy y0 + x0 ; y0 − x0 cùng tính chẵn lẻ mà 30 = 1.30 = 2.15 = 5.6 = 3.10. Trường hợp này
không xảy ra.
• Trường hợp 3. Nếu x0 < y0 < y1 , thì ta suy ra

y0 > x 0 + 1
y1 > x 0 + 2
Do đó từ (2) suy ra
x20 + 30 > (x0 + 1) (x0 + 2) ⇔ x0 6 9
Khi x0 = 9 thì từ (2) suy ra y0 y1 = 92 + 30 = 111, vì y0 < y1 ⇒ (y0 ; y1 ) = (1; 111); (3; 37). Điều
này là vô lí vì phải có x0 < y0 . Tương tự khi xét x = 1; 2; 3; 4; 5; 6; 7; 8. Tất cả đều dẫn đến vô lí.
Trường hợp này loại.
Do đó ta luôn có A = 32 = 25 là lũy thừa bậc năm của một số nguyên. 

26
Chinh phục olympic toán Ứng dụng định lí Viète trong các bài toán số học

x2 + y 2 + z 2
Câu 4. Cho các số nguyên dương x, y, z thỏa mãn nhận giá trị nguyên dương.
xyz + 1
x2 + y 2 + z 2
Chứng minh rằng có thể biểu diễn được thành tổng của hai số chính phương.
xyz + 1

x2 + y 2 + z 2
Lời giải. Ta đặt n = , khi đó ta sẽ chứng minh n là tổng của hai số chính phương. Viết
xyz + 1
lại đẳng thức trên thành x2 + y 2 + z 2 = n (xyz + 1). Giả sử (x0 ; y0 ; z0 ) là một bộ số nguyên dương
thỏa mãn yêu cầu bài toán, điều đó có nghĩa là

x20 + y02 + z02 = n (x0 y0 z0 + 1)

Hay ta viết lại được


x20 − nx0 y0 z0 + y02 + z02 − n = 0
Xét phương trình bậc hai x2 − nxy0 z0 + y02 + z02 − n = 0, khi đó ta thấy x0 là một nghiệm của phương
trình. Theo định lí Viète thì ngoài nghiệm x0 phương trình còn có một nghiệm nữa, ta gọi nghiệm đó
là x1 . Như vậy theo định lí Viète ta có

x1 + x0 = ny0 z0
x1 x0 = y02 + z02 − n

Từ hệ thức trên ta suy ra được x1 nhận giá trị nguyên. Không mất tính tổng quát ta chọn x0 + y0 + z0
bé nhất và x0 > y0 > z0 . Ta xét các trường hợp sau
• Trường hợp 1. Nếu y02 + z02 < n, khi đó x1 là số nguyên âm. Từ đó suy ra

0 = x21 − ny0 z0 x1 + y02 + z02 − n > x21 + n + y02 + z02 − n = x21 + y02 + z02 > 0

điều này vô lí.


• Trường hợp 2. Nếu y02 + z02 > n, khi đó x1 là số nguyên dương. Khi đó (x1 ; y0 ; z0 ) là một bộ số
nguyên dương thỏa mãn yêu cầu bài toán. Theo cách chọn (x0 ; y0 ; z0 ) ta suy ra được x0 6 x1 .
Khi đó từ định lí Viète ta có

y02 + z02 − n − ny0 z0 = (x0 − 1) (x1 − 1) − 1 > (x0 − 1)2 − 1

Ta lại xét 2 khả năng sau.


1. Nếu x0 > y0 thì ta được (x0 − 1)2 − 1 > y02 − 1. Do đó

y02 + z02 − n − ny0 z0 > y02 − 1

Từ đó suy ra z02 + 1 > ny0 z0 + n > n z02 + 1 , như vậy ta có n = 1.




2. Nếu x0 = y0 , khi đó ta được 2y02 + z02 = n y02 z0 + 1 . Do đó




z02 = y02 (nz0 − 2) + n > z02 (nz0 − 2) + n > z02 (nz0 − 2)

Từ đây ta suy ra được nz0 < 3 nên n = 1 hoặc n = 2, chú ý rằng n = 1 = 02 + 12 và


n = 2 = 1 2 + 12 .
• Trường hợp 3. Nếu y02 + z02 = n thì có nghĩa là n viết được thành tổng của hai số chính phương.
Vậy bài toán được chứng minh xong. 

Câu 5. Cho a, b, c là các số nguyên dương thỏa mãn 0 < a2 + b2 − abc 6 c. Chứng minh rằng
a2 + b2 − abc là số chính phương.

27
Hướng tới VMO 2020 Tạp chí và tư liệu toán học

Lời giải. Giả sử tồn tại các số nguyên dương a, b, c thỏa mãn 0 < a2 + b2 − abc 6 c mà k = a2 + b2 − abc
không phải là số chính phương. Khi đó ta có 0 < k 6 c. Không mất tính tổng quát ta giả sử a > b. Xét
phương trình bậc hai ẩn x là
x2 − bcx + b2 − k = 0
Khi đó a là một nghiệm của phương, khi đó theo định lí Viète thì phương trình còn có một nghiệm
nữa là x = a1 . Từ đó ta được 
a + a1 = bc
a.a1 = b2 − k

Như vậy ta suy ra a1 là số nguyên.


1. Nếu a1 = 0, khi đó từ hệ thức a.a1 = b2 − k ta được k = b2 là số chính phương, điều này mâu
thuẫn với giả sử ở trên.
2. Nếu a1 < 0, khi đó k = a21 + b2 − a1 bc > a21 + b2 + bc > c, mâu thuẫn do 0 < k 6 c.
Như vậy ta được a1 là số nguyên dương. Cũng theo định lí Viète ta có
2

 a1 = b − k

2
a ⇒ a1 < a
 b −k <a

a

Ta thấy cặp số (a1 ; b) cũng là một nghiệm. Khi đó ta có a1 + b < a + b, điều này sẽ vô lí khi ta chọn
cặp số (a; b) với a + b bé nhất. Như vậy không thể tồn tại k để k = a2 + b2 − abc không phải là số
chính phương. Như vậy a2 + b2 − abc phải là số chính phương. 

Câu 6 [Taiwan MO 1998]. Cho m > n là các số nguyên dương lẻ và n2 − 1 chia hết cho
m2 − n2 + 1. Chứng minh rằng m2 − n2 + 1 là một số chính phương.

Lời giải. Theo giả thiết ta có n2 − 1 chia hết cho m2 − n2 + 1 nên ta được m2 − m2 − n2 + 1 chia


hết cho m2 − n2 + 1. Từ đó suy ra


m2 − n2 + 1 | m2


Từ điều trên ta suy ra tồn tại k để


m2 = k m2 − n2 + 1


m+n m−n 2
 
2 m+n m−n
Ở đây ta chú ý rằng m = + và m2 −n2 +1 = 4. . −1. Do m và n là các số
2 2 2 2
m+n m−n m+n m−n
nguyên dương lẻ, lại có m > n nên ; là các số nguyên dương, đặt x = ;y = ,
2 2 2 2
khi đó ta được
(x + y)2 = k (4xy + 1)
Bây giờ ta sẽ đi chứng minh rằng 4xy + 1 là số chính phương, tuy nhiên trước tiên ta cần phải chứng
minh k là số chính phương. Thật vậy, giả sử cặp số nguyên dương (x0 ; y0 ) với x0 + y0 nhỏ nhất thỏa
mãn thỏa đẳng thức trên, khi đó ta có

(x0 + y0 )2 = k (4x0 y0 + 1)

Xét phương trình bậc hai ẩn x là x2 − (4k − 2) y0 x + y02 − k = 0. Khi đó x0 là một nghiệm của phương
trình trên. Như vậy theo định lí Viète thì phương trình còn có một nghiệm nữa là x1 , lúc này ta có

x0 + x1 = (4k − 2) y0
x0 .x1 = y02 − k

Từ hệ thức thứ nhất x0 + x1 = (4k − 2) y0 ta suy ra được x1 là số nguyên. Ta xét các trường hợp sau

28
Chinh phục olympic toán Ứng dụng định lí Viète trong các bài toán số học

• Nếu x1 < 0 thì từ hệ thức thứ hai x0 .x1 = y02 − k ta được y02 − k < 0 ⇒ y02 < k, ta suy ra

x21 − (4k − 2) y0 x1 + y02 − k = (x1 + y0 )2 + k (−4x1 − 1) > 0

điều này mâu thuẫn vì x1 là nghiệm của phương trình.


• Nếu x1 = 0, khi đó từ x0 .x1 = y02 − k ta được y02 − k = 0 ⇒ k = y02 là số chính phương.
• Nếu x1 > 0 thì ta được y02 − k > 0 ⇒ k > y02 . Khi đó (x1 ; y0 ) là một nghiệm của phương trình
(x + y)2 = k (4xy + 1). Theo cách chọn cặp số (x0 ; y0 ) ta có

x0 + y0 6 x1 + y0 ⇒ y0 6 x0 6 x1

Kéo theo y02 − (4k − 2) y02 + y02 − k = (4 − 4k) y02 − k > 0, điều này vô lí vì k là số nguyên dương.
Vậy ta được k là số chính phương nên dẫn đến m2 − n2 + 1 là số chính phương. 

Câu 7. Cho x và y là các số nguyên dương thỏa mãn điều kiện x2 + y 2 + 1 chia hết cho 2xy + 1.
Chứng minh rằng x = y.

x2 + y 2 + 1
Lời giải. Do 2xy + 1 | x2 + y 2 + 1 nên ta đặt k = với k là số nguyên dương. Nhận thấy
2xy + 1
khi x = y thì k = 1 và ngược lại vẫn đúng. Do đó ta đi chứng minh k = 1. Giả sử cặp số nguyên dương
(a0 ; b0 ) với a0 + b0 bé nhất thỏa mãn yêu cầu bài toán. Không mất tính tổng quát ta giả sử a0 > b0 .
Xét phương trình
x2 + b20 + 1
k= ⇔ x2 − 2kb0 x + b20 + 1 − k = 0
2xb0 + 1
Khi đó ta có
∆0 = b20 k 2 − b20 − 1 + k = (k − 1) b20 (k + 1) + 1 > 0
 

Do đó phương trình luôn có nghiệm. Dễ thấy a0 là một nghiệm của phương trình nên theo định lí
Viète thì phương trình còn có thêm một nghiệm nữa là a1 . Khi đó ta có

a0 + a1 = 2kb0
a0 .a1 = b20 − k + 1

Từ hệ thức thứ nhất ta được a1 là số nguyên. Giả sử a1 < 0, khi đó

a21 − 2kb0 a1 + b20 + 1 − k > a21 + 2kb0 + b20 + 1 − k > 0

điều này vô lí vì a1 là một nghiệm của phương trình x2 − 2kb0 x + b20 + 1 − k = 0, suy ra a1 > 0. Ta xét
các trường hợp sau.
• Trường hợp 1. Nếu a1 = 0, khi đó ta được a1 b0 = 0, điều này dẫn đến xy = 0.
• Trường hợp 2. Nếu a1 > 0, khi đó nếu a0 > b0 thì ta được

b20 − k + 1 a2 − 1 + 1
a1 + b0 = + b0 < 0 + b0 = a0 + b0
a0 a0

Đều này mâu thuẫn với các chọn (a0 ; b0 ) với a0 + b0 bé nhất.
2a20 + 1
Như vậy a0 = b0 , suy ra k = = 1. Do đó ta được x = y. 
2a20 + 1

Câu 8. Cho a, b là các số nguyên dương lẻ thỏa mãn a2 + 2 chia hết cho b và b2 + 2 chia hết cho
a2 + b2 + 2
a. Chứng minh rằng là số chính phương.
ab

29
Hướng tới VMO 2020 Tạp chí và tư liệu toán học

Lời giải. Ta thấy rằng a, b là 2 số nguyên tố cùng nhau. Thật vậy nếu d = (a, b) thì do b | a2 + 2 nên
d | a2 + 2, lại có d | a nên d | 2. Mặt khác a, b là số lẻ nên suy ra d = 1. Do b | a2 + 2 và a | b2 + 2 nên

ab | a2 + 2 b2 + 2 ⇔ ab | 2a2 + 2b2 + 4
 

Do a và b là số lẻ nên suy ra ab | a2 + b2 + 2. Khi đó tồn tại số nguyên dương k sao cho a2 + b2 + 2 = kab.
Bây giờ ta sẽ đi chứng minh k là số chính phương. Trước tiên thử một vài giá trị đặc biệt ta nhận thấy
được k = 4. Đến đây thì tư tưởng giống như các ví dụ trên đó là sử dụng định lý Viète đển chứng
minh k = 4 thỏa mãn bài toán. Giả sử cặp số dương (a0 ; b0 ) với a0 + b0 nhỏ nhất thỏa mãn yêu cầu
bài toán, tức là ta sẽ có
a20 + b20 + 2 − ka0 b0 = 0
Không mất tính tổng quát ta giả sử a0 > b0 . Xét phương trình bậc hai ẩn a

a2 − kb0 a + b20 + 2 = 0

Khi đó ta thấy a0 là một nghiệm của phương trình trên. Khi đó theo định lí Viète thì phương trình
trên còn có một nghiệm nữa, ta gọi là a1 , từ đó ta có

a0 + a1 = kb0
a0 + a1 = b20 + 2

Từ hệ thức trên ta thu được a1 là số nguyên dương. Như vậy cặp số (a1 ; b0 ) cũng thỏa mãn yêu cầu
bài toán. Theo cách chọn cặp số (a0 ; b0 ) ta được

a0 + b0 6 a1 + b0 ⇒ a0 6 a1

Từ đó ta có
a0 k
a0 = kb0 − a1 6 kb0 − a0 ⇒ 6
b0 2
Mặt khác từ điều kiện a20 + b20 + 2 − ka0 b0 = 0 ta lại được
a0 b0 2
+ + =k
b0 a0 a0 b0
a0 k k
Do 6 nên k 6 + 2 + 1 hay k 6 6. Theo bất đẳng thức AM − GM ta có
b0 2 2
a20 + b20 > 2a0 b0 ⇒ k > 2

Đến đây ta xét các trường hợp sau


1. Nếu k = 3, khi đó ta được a20 + b20 + 2 = 3a0 b0 . Từ đó suy ra 3 | a20 + b20 + 2 Do số chính phương
chia 3 dư 1 nên trong hai số chính phương a20 và b20 phải có một số chia hết cho 3 và một số
không chia hết cho 3. Từ đó suy ra vế phải chia hết cho 9 còn vế trái không chia hết cho 9. Điều
này dẫn đến mâu thuẫn.
a0 b0 2
2. Nếu k 6= 4, khi đó từ + + = k ta được (a0 ; b0 ) 6= (1; 1).
b0 a0 a0 b0
k
Như vậy ta được k 6 + 1 + 1 hay k 6 4, do đó k = 5 và k = 6 không thỏa mãn bà toán. Vậy ta suy
2
ra được chỉ có k = 4 thỏa mãn yêu cầu bài toán. 

Câu 9. Cho các số nguyên dương a, b, c, d thỏa mãn b2 + 1 = ac và c2 + 1 = bd. Chứng minh
rằng a + c = 3b và b + d = 3c.

Lời giải. Từ giả thiết ta thấy (b, c) = 1. Thật vậy, gọi (b, c) = k khi đó ta có

b = b0 k kac0 | k 2 b0 2 + 1
 

c = c0 k kdb0 | k 2 c0 2 + 1

30
Chinh phục olympic toán Ứng dụng định lí Viète trong các bài toán số học

 0  0 
Từ đó ta được k 2 adb0 c0 | k 2 b 2 + 1 k 2 c 2 + 1 , suy ra
 0  0  0 0 0 0
k2 | k2 b 2 + 1 k2 c 2 + 1 ⇒ k2 | k4 b 2 c 2 + k2 b 2 + k2 c 2 + 1 ⇒ k2 | 1 ⇒ k = 1

b2 + 1 = ac b | c2 + 1
 
Hơn nữa từ giả thiết ta lại có do vậy ta được .
c2 + 1 = bd c | b2 + 1
Như vậy ab | b2 + c2 + 1 hay b2 + c2 + 1 = mbc với số nguyên dương m nào đó. Đến đây sử dụng kết
quả của bài toán 4 trong phần bước nhảy Viète ta có được m = 3, khi đó thì b2 + c2 + 1 = 3bc. Từ
đây suy ra  2  2
b + c2 + 1 = c2 + ac ac + c2 = 3bc

b + 1 = ac
⇒ ⇒
c2 + 1 = bd b2 + c2 + 1 = b2 + bc b2 + bd = 3bc

a + c = 3b
Hay ta được , vậy bài toán được chứng minh. 
b + d = 3c

Câu 10. Giả sử phương trình x2 + y 2 + x + y + 1 = xyz có nghiệm nguyên dương. Tìm tất cả
các giá trị của z.

Lời giải. Trước tiên ta giả sử cặp số nguyên dương (x0 ; y0 ) với x0 + y0 nhỏ nhất thỏa mãn thỏa đẳng
thức trên. Không mất tính tổng quát ta giả sử x0 > y0 , khi đó ta được

x20 + y02 + x0 + y0 + 1 = zx0 y0

Xét phương trình bậc hai ẩn x là

y 2 + (1 − zx0 ) y + x20 + x0 + 1 = 0

Khi đó y0 là một nghiệm của phương trình trên, mà theo hệ thức Viète thì phương trình còn có một
nghiệm nữa là y1 . Do đó ta được 
y0 + y1 = zx0 − 1
y0 .y1 = x20 + x0 + 1
Từ các hệ thức trên ta suy ra được y1 có giá trị nguyên dương. Khi đó xặp số nguyên dương (y1 ; x0 ) là
một nghiệm của phương trình đã cho. Theo cách chọn cặp số (x0 ; y0 ) ta suy ra được y0 6 y1 . Ta xét
các trường hợp sau.
• Nếu x0 = y0 thì từ x20 + y02 + x0 + y0 + 1 = zx0 y0 ta suy ra

2x20 + 2x0 + 1 = zx20

Từ đó ta được x0 = y0 = 1 và z = 5.
• Nếu x0 > y0 thì ta được y0 < x0 < y1 , khi đó ta được
2 1
x20 + (1 − zx0 ) x0 + x20 + x0 + 1 < 0 ⇒ x < 2 + + 2
x0 x0

Đến đây ta xét các giá trị x0 > 2 đều không tìm được z thỏa mãn.
Như vậy z = 5 là giá trị thỏa mãn yêu cầu đề bài. 

Câu 11. Tìm các số nguyên dương x và y sao cho x + 1 chia hết cho y và y + 1 chia hết cho x.

Lời giải. Do x + 1 chia hết cho y và y + 1 chia hết cho x nên ta được (x + 1) (y + 1) chia hết cho xy.
Từ đó suy ra
xy | (x + y + 1) ⇒ xy | (x + y + 1)2

31
Hướng tới VMO 2020 Tạp chí và tư liệu toán học

Đến đây biến đổi tiếp ta được xy | x2 + y 2 + 2x + 2y + 1 . Khi đó tồn tại số nguyên dương k sao cho


x2 + y 2 + 2x + 2y + 1 = kxy. Giả sử cặp số nguyên dương (x0 ; y0 ) với x0 + y0 nhỏ nhất thỏa mãn yêu
cầu bài toán. Khi đó ta được

x20 + y02 + 2x0 + 2y0 + 1 − kx0 y0 = 0

Không mất tính tổng quát ta giả sử x0 > y0 . Xét phương trình bậc hai ẩn x là

x2 + (2 − ky0 ) x + (y0 + 1)2 = 0

Khi đó x0 là một nghiệm của phương trình. Như vậy theo định lí Viète thì phương trình trên còn có
một nghiệm khác là x1 . Do đó ta được

x0 + x1 = ky0 − 2
x0 .x1 = (y0 + 1)2

Từ hệ thức x0 + x1 = ky0 − 2 ta suy ra được x1 là số nguyên. Từ hệ thức x0 .x1 = (y0 + 1)2 ta suy ra
được x1 là số nguyên dương. Chú ý rằng lúc này cặp số nguyên dương (x1 ; y0 ) thỏa mãn bài toán. Ta

x0 + y0 6 x1 + y0 ⇒ x0 6 x1
do vậy ta được y0 6 x0 6 x1 . Đến đây ta xét các trường hợp sau
1. Nếu x0 = y0 , khi đó ta được
1 4
2x20 + 4x0 + 1 = kx20 ⇒ k = 2 + 2 +
x0 x0

Do k là số nguyên dương nên ta suy ra được x0 = 1 thỏa mãn, do đó suy ra x0 = y0 = 1 thỏa


mãn bài toán.
2. Nếu x0 = x1 > y0 > 1, khi đó từ x0 .x1 = (y0 + 1)2 ta được x0 = y0 + 1. Thay vào hệ thức
x0 + x1 = ky0 − 2 ta được
4
2x0 = k (x0 − 1) − 2 ⇔ k = 2 +
x0 − 1
Do k và x0 là các số nguyên dương nên ta tìm được x0 = 2, x0 = 3 và x0 = 5, tương ứng thì
y0 = 1, y0 = 2 và y0 = 4. Tuy nhiên khi thử vào bài toán thì ta thấy có hai cặp số nguyên dương
thỏa mãn đó là (2; 1) , (3; 2).
3. Nếu x1 > x0 > y0 > 1, khi đó ta được x1 > x0 + 1 > y0 + 2. Từ đó ta được

x0 .x1 > (y0 + 2) (y0 + 1) > (y0 + 1)2

Do đó trường hợp này không thỏa mãn.


Vậy các cặp số nguyên dương thỏa mãn bài toán là (x; y) = (1; 1) , (1; 2) , (2; 1) , (2; 3) , (3; 2). 

Câu 12. Tìm các số nguyên dương x, y để x2 + 2 chia hết cho xy + 1.

Lời giải. Ta biến đổi giả thiết thành x2 − mxy + 2 − m = 0, khi đó giả sử cặp số nguyên dương (x0 ; y0 )
với x0 + y0 bé nhất thỏa mãn yêu cầu bài toán, tức là ta có x20 + 2 = m (x0 y0 + 1). Xét phương trình
bậc hai ẩn x là
x2 − mxy0 − m + 2 = 0
Khi đó x0 là một nghiệm của phương trình trên. Theo định lí Viète thì phương trình trên còn có một
nghiệm nữa, gọi nghiệm đó là x1 . Khi đó ta có

x0 + x1 = my0
x0 .x1 = 2 − m

32
Chinh phục olympic toán Ứng dụng định lí Viète trong các bài toán số học

Từ hệ thức x0 + x1 = my0 ta suy ra x1 là số nguyên. Mặt khác nếu x1 < 0, khi đó ta được x1 6 −1.
Do đó
x21 − mx1 y0 − m + 2 > x21 + my0 − m + 2 > 0
Điều này mâu thuẫn với x1 là nghiệm của phương trình trên. Như vậy x1 > 0 hay x1 là số nguyên
không âm. Do đó từ hệ thức x0 .x1 = 2 − m ta được

2 − m > 0 ⇒ m ∈ {1; 2}

Ta xét các trường hợp sau


1. Nếu m = 1, khi đó ta được x (x − y) = −1 nên ta được x = 1; y = 2 thỏa mãn yêu cầu bài toán.
2. Nếu m = 2, khi đó x (x − 2y) = 0. Lại do x 6= 0 nên suy ra x − 2y = 0 ⇔ x = 2y. Do đó
x = 2k; y = k với k là số nguyên dương thỏa mãn yêu cầu bài toán.
Vậy cặp số nguyên dương thỏa mãn là (x; y) = (1; 2) , (2k; k) với k là số nguyên dương. 

Câu 13. Tìm tất cả các số có ba chữ số chia hết cho 11 sao cho thương số của phép chia số đó
cho 11 bằng tổng bình phương của các chữ số của số đó.

Lời giải. Gọi số có ba chữ số thỏa mãn yêu cầu bài toán là A = abc. Trong đó các chữ số thỏa mãn

a ∈ {1; 2; ...9} ; b, c ∈ {0; 1; 2; ...9}

Do A chia hết cho 11 nên ta được a−b+c chia hết cho 11. Kết hợp với a ∈ {1; 2; ...9} ; b, c ∈ {0; 1; 2; ...9}
ta suy ra được a − b + c = 0 hoặc a − b + c = 11. Như vậy ta đi xét hai trường hợp sau
• Với a−b+c = 0, khi đó ta được b = a+c. Ta có A = 100a+10b+c = 99a+10b+a+c = 99a+11b.
Khi A chia 11 thì thương số của phép chia bằng tổng bình phương các chữ số của A nên ta được

A
= a2 + b2 + c2 ⇔ 9a + b = a2 + b2 + c2
11

Kết hợp với b = a + c ta được

9a + (a + c) = a2 + (a + c)2 + c2 ⇔ 10a + c = 2a2 + 2ac + 2c2

Do a > 1 nên suy ra

25
10a + c > 2a2 + 2c + 2c2 ⇒ 2c2 + c 6 10a − 2a2 6
2

Do vậy mà 2c2 + c 6 12 ⇒ c 6 2. Cũng từ 10a + c = 2a2 + 2ac + 2c2 ta suy ra được c là số chẵn.
Từ đó ta được c = 0 hoặc c = 2. Ta xét 2 khả năng.
1. Với c = 0, khi đó ta được a = b nên số cần tìm có dạng A = aa0. Do đó

A
= 50 = 2a2 ⇒ a = 5 ⇒ a = b = 5
11

Từ đó ta tìm được A = 550.


2. Với c = 2, khi đó từ 10a + c = 2a2 + 2ac + 2c2 ta được

10a + 2 = 2a2 + 4ac + 8 ⇔ a2 − 3a + 3 = 0

Nhận thấy phương trình trên không có nghiệm nguyên dương nên không tồn tại số A thỏa
mãn bài toán.

33
Hướng tới VMO 2020 Tạp chí và tư liệu toán học

• Với a − b + c = 11, khi đó ta được b + 11 = a + c. Do a, b, c là các chữ số nên từ b + 11 = a + c ta


suy ra được a > 2. Ta có

A = 100a + 10b + c = 99a + 10b + a + c = 99a + 11b + 11

Ta xét các khả năng sau.


1. Xét a = 2, khi đó c = 9; b = 0. Ta được A = 209 không thỏa mãn bài toán.
2. Xét a = 3, khi đó ta được c = 8; b = 0 hoặc c = 9; b = 1. Ta được A = 308 hoặc A = 319
không thỏa mãn.
3. Xét a > 4, khi đó A chia 11 thì thương số của phép chia bằng tổng bình phương các chữ số
của A nên ta được
A
= a2 + b2 + c2 ⇔ 9a + b + 1 = a2 + b2 + c2
11
Kết hợp với b = a + c − 11 ta được

9a + (a + c − 11) + 1 = a2 + (a + c − 11)2 + c2
⇔ 10a + c − 10 = 2a2 + 2ac + 2c2 − 22 (a + c) + 121
⇔ 32a + 23c − 131 = 2a2 + 2ac + 2c2

Do a > 4 nên suy ra

32a + 23c − 131 > 2a2 + 8c + 2c2 ⇒ 2c2 − 15c 6 32a − 2a2 − 131 6 −3

Do đó suy ra 2c2 − 15c 6 −3 ⇒ c 6 7. Từ 32a + 23c − 131 = 2a2 + 2ac + 2c2 ta suy ra
được c là số lẻ. Do đó ta được c = 1; 3; 5; 7. Đến đây xét các trường hợp của c thì được
b = 0; a = 8 thỏa mãn.
Do đó số cần tìm là A = 803.
Vậy các số thỏa mãn yêu cầu bài toán là 550 và 803. 

Câu 14 [Kiran Kedlaya]. Cho các số nguyên dương a, b, c là thỏa mãn (ab + 1) (bc + 1) (ca + 1)
là số chính phương. Chứng minh rằng ba số ab + 1; bc + 1; ca + 1 đều là số chính phương.

Lời giải. Từ giả thiết ta nhận thấy để chứng minh ab + 1; bc + 1; ca + 1 đều là số chính ta cần chỉ ta
được ba số ab + 1; bc + 1; ca + 1 nguyên tố với nhau theo từng đôi một. Tuy nhiên với lượng thông
tin hạn chế từ giả thiết ta không thể chứng minh được nhận định trên. Với ý tưởng sử dụng định lí
Viète, ta cần tạo ra một phương trình bậc hai có nghiệm nguyên và có biệt thức ∆ có chứa biểu thức
(ab + 1) (bc + 1) (ca + 1). Ngoài ra do ta cần chứng minh ab + 1; bc + 1; ca + 1 đều là số chính phương
nên khi biến đổi phương trình bậc hai thì cần chứa các đại lượng ab + 1; bc + 1; ca + 1. Xét phương
trình bậc hai ẩn t sau

t2 + a2 + b2 + c2 − 2 (ab + bc + ca + ta + tb + tc) − 4abct − 4 = 0


⇔ t2 − 2t (a + b + c + 2abc) + a2 + b2 + c2 − 2 (ab + bc + ca) − 4 = 0

Ta nhận thấy phương trình bậc hai trên tương đương với ba phương trình sau

(a + b − c − t)2 = 4 (ab + 1) (ct + 1)


(a + c − b − t)2 = 4 (ac + 1) (bt + 1)
(b + c − a − t)2 = 4 (bc + 1) (at + 1)

Giải phương trình bậc hai trên ta được


 p
t1 = a + b + c + 2abc + 2p(ab + 1) (bc + 1) (ca + 1)
t2 = a + b + c + 2abc − 2 (ab + 1) (bc + 1) (ca + 1)

34
Chinh phục olympic toán Ứng dụng định lí Viète trong các bài toán số học

Do (ab + 1) (bc + 1) (ca + 1) là số chính phương nên t nhận các giá trị nguyên. Từ ba phương trình
trên ta được
43 (ab + 1) (ac + 1) (bc + 1) (ct + 1) (bt + 1) (at + 1)
là số chính phương, do đó (ct + 1) (bt + 1) (at + 1) là số chính phương, ta lại có

at + 1 > 0; bt + 1 > 0; ct + 1 > 0

nên at + 1 > 0; bt + 1 > 0; ct + 1 > 0. Trong các bộ số nguyên dương (a; b; c) thỏa mãn bài toán ta xét
bộ số (a; b; c) sao cho a + b + c nhỏ nhất. Không mất tính tổng quát ta chọn c = max {a; b; c}. Dễ thấy
−1
a = b = c = 1 không thỏa mãn bài toán nên c = max {a; b; c} > 1, do đó t > > −1. Ta
max {a; b; c}
xét các trường hợp sau
1. Nếu t = 0, khi đó từ phương trình bậc hai ta được

(a + b + c)2 = 4 (ab + bc + ca) + 4 ⇔ (a + b − c)2 = 4 (ab + 1)

Suy ra ab + 1 là số chính phương. Chứng minh hoàn toàn tương tự ta được bc + 1; ca + 1 là các
số chính phương.
2. Nếu t > 0, khi đó do a + b + c có giá trị nhỏ nhất nên ta được t > c và t chỉ nhận một trong hai
giá trị  p
t1 = a + b + c + 2abc + 2p(ab + 1) (bc + 1) (ca + 1)
t2 = a + b + c + 2abc − 2 (ab + 1) (bc + 1) (ca + 1)
Mà theo định lí Viète ta có

t1 t2 = a2 + b2 + c2 − 2 (ab + bc + ca) − 4 6 c2 − a (2c − a) − b (2c − b) < c2

Điều này dẫn đến mâu thuẫn. Nên trường hợp này không có giá trị t thỏa mãn.
Như vậy bài toán đã được giải quyết hoàn toàn. 

Câu 15. Tồn tại hay không năm số nguyên dương a1 ; a2 ; a3 ; a4 ; a5 thỏa mãn hệ điều kiện
 2
 a2 + 1 = (a1 + 1) (a3 + 1)
a2 + 1 = (a2 + 1) (a4 + 1)
 32
a4 + 1 = (a3 + 1) (a5 + 1)

Lời giải. Trước hết ta kiểm tra tính chẵn lẻ của các số đã cho trước khi có các đánh giá hợp lí. Giả
sử a1 là số lẻ, khi đó a2 là số lẻ nên a22 + 1 chia 4 dư 2, từ đó suy ra a3 + 1 là số lẻ, dẫn đến a3 là số
chẵn. Điều này vô lý vì a2 + 1 là số chẵn nên không thể là ước của số lẻ a23 + 1. Do vậy a1 phải là số
chẵn. Lập luận hoàn toàn tương tự ta được a2 ; a3 ; a4 ; a5 cùng là số chẵn. Đặt a2 = x; a3 = y. Khi đó
. .
từ hệ điều kiện trên ta được y 2 + 1 .. (x + 1) và x2 + 1 .. (y + 1). Ta sẽ chứng minh rằng không tồn
tại cặp số chẵn x và y thỏa mãn điều trên.
. .
Giả sử tồn tại cặp số nguyên dương (x, y) thỏa mãn y 2 + 1 .. (x + 1) và x2 + 1 .. (y + 1), khi đó ta
được
. .
y 2 + 1 + x2 − 1 .. (x + 1) ⇒ x2 + y 2 .. (x + 1)
.
Tương tự ta cũng có x2 + y 2 .. (y + 1). Gọi d = (x + 1, y + 1), khi đó d là ước của x2 + 1; y 2 + 1; x2 + y 2 .
Do đó
 . .
x + 1 + y 2 + 1 − x2 + y 2 ..d ⇒ 2..d
 2

Như vậy thì d = 1 hoặc d = 2. Mà x + 1; y + 1 là các số lẻ, do đó ta được d = 1 hay x + 1; y + 1 nguyên


tố cùng nhau. Từ đó dẫn đến
.
x2 + y 2 .. (x + 1) (y + 1)

35
Hướng tới VMO 2020 Tạp chí và tư liệu toán học

Khi đó tồn tại số nguyên dương k sao cho


x2 + y 2 = k (x + 1) (y + 1)
Trong các cặp số nguyên dương (x; y) thỏa mãn ta chọn cặp số nguyên dương (x0 ; y0 ) với x0 + y0 bé
nhất. Không mất tính tổng quát ta giả sử x0 > y0 . Xét phương trình bậc hai ẩn x là
x2 − k (y0 + 1) x + y02 − k (y0 + 1) = 0
Khi đó x0 là một nghiệm của phương trình. Theo định lí Viète thì còn có một nghiệm nữa là x1 . Khi
đó ta có 
x0 + x1 = k (y0 + 1)
x0 x1 = y02 − k (y0 + 1)
.
Nếu x1 = 0, khi đó từ x0 x1 = y02 − k (y0 + 1) ta được y02 = k (y0 + 1) nên suy ra y02 .. (y0 + 1), điều này
vô lí vì y0 và y0 + 1 nguyên tố cùng nhau. Do đó x1 6= 0. Mặt khác ta có
(x0 + 1) (x1 + 1) = x0 x1 + x0 + x1 + 1 = y02 + 1
nên x1 + 1 là số lẻ. Do đó x1 > 0 là số chẵn. Đồng thời ta cũng có
y02 + 1 y2 + 1
x1 + 1 = 6 0 6 y0 6 x 0
x0 + 1 y0 + 1
Như vậy cặp số (y0 ; x1 ) là một nghiệm của
x2 + y 2 = k (x + 1) (y + 1)
Cặp số (y0 ; x1 ) thỏa mãn y0 + x1 6 y0 + x0 , điều này vô lí vì ta đã chọn x0 + y0 bé nhất. Vậy điều
ta giả sử là sai hay không tồn tại các số chẵn x, y thỏa mãn. Như vậy không tồn tại năm số nguyên
dương thỏa mãn yêu cầu bài toán. 

Nhận xét. Từ bài toán này, ta có thể giải được các bài toán tương tự sau.
1. Cho các số nguyên dương x, y thỏa mãn x2 + 1 chia hết cho y + 1 và y 2 + 1 chia hết cho
x + 1. Chứng minh rằng x và y là các số lẻ.
2. Tìm số nguyên dương k để phương trình x2 + y 2 = k (x + 1) (y + 1) có nghiệm nguyên

! dương.
3. Tìm số tự nhiên n bé nhất sao cho tồn tại số nguyên dương a1 ; a2 ; ...; an thỏa mãn

a2k = (ak−1 + 1) (ak+1 + 1)

với các số 2 6 k 6 n − 1.

6 Bài tập tự luyện.


Câu 1. Tìm tất cả các số tự nhiên a, b, c sao cho tồn tại số nguyên dương n, m, k thỏa mãn các điều
kiện sau
m2 + b n2 + c k2 + a
a= ;b = ;c =
2m 2n 2k
Câu 2. Tìm nghiệm nguyên của phương trình sau x + x y + xy 2 + y 3 = 8 x2 + xy + y 2 + 1 .
3 2


Câu 3. Tìm nghiệm nguyên của phương trình 3 x2 − xy + y 2 = 7 (x + y).




Câu 4 [Putnam 1998]. Chứng minh rằng với mỗi số thực N thì phương trình
x21 + x22 + x23 + x24 = x1 x2 x3 + x1 x2 x4 + x1 x3 x4 + x2 x3 x4
có nghiệm (a1 , a2 , a3 , a4 ) với a1 , a2 , a3 , a4 là các số nguyên lớn hơn N .
Câu 5. Giả sử bốn số nguyên a, b, c, d đôi một khác nhau và thoả mãn hệ điều kiện sau
 2
a − 2ac − 5d = b2 − 2bc − 5d = 0
c2 − 2ca − 5b = d2 − 2bd − 5b = 0

36
Chinh phục olympic toán Ứng dụng định lí Viète trong các bài toán số học

Chứng minh rằng a + b + c + d là một hợp số.


√ √ √
Câu 6. Tìm các số nguyên dương x, y sao cho x + y − x − y + 2 = 0.
Câu 7 [Turkey National Olympiad 2015]. Với m, n là các số nguyên dương sao cho

(m + n)2
k=
4m(m − n)2 + 4

cũng là số nguyên. Chứng minh rằng k là số chính phương.


Câu 8. Cho p là một số nguyên dương. Giả sử phương trình x2 + px + 1 = 0 có hai nghiệm là a1 ; a2
và phương trình x2 + qx + 1 = 0 có hai nghiệm b1 ; b2 . Chứng minh rằng

(a1 − b1 ) (a2 − b1 ) (a1 + b2 ) (a2 + b2 )

là hiệu của hai số chính phương.


Câu 9. Tìm các cặp số nguyên (a; b) sao cho hai số a2 + 4b và b2 +√4a đều là số chính phương.
Câu 10. Tìm các chữ số a, b, c, d, e thỏa mãn điều kiện ab + cde = abcde.
Câu 11. Cho a, b là các số nguyên dương thỏa mãn a2 + b2 chia hết cho ab. Tính giá trị của biểu thức

a2 + b2
A=
ab

Câu 12 [Đề thi trường Đông phía Bắc 2015]. Tìm tất cả các số nguyên dương k sao cho phương
trình x2 − (k 2 − 4)y 2 + 24 = 0 có nghiệm nguyên dương.
Câu 13. Chứng minh rằng tất cả các nghiệm nguyên dương của phương trình x2 + y 2 + 1 = 3xy là
(x, y) = (F2k−1 , F2k+1 ) với Fn là số Fibonacci.
Câu 14. Tìm tất cả các số nguyên dương n sao cho phương trình sau có nghiệm nguyên dương

x2 + y 2 = n(x + 1)(y + 1).

Câu 15. Giả sử a, b là các số nguyên dương thỏa mãn b + 1 | a2 + 1, a + 1 | b2 + 1. Chứng minh rằng a,
b đều là các số lẻ.
a2 + b2 + 6
Câu 16. Chứng minh rằng nếu a, b là các số nguyên dương sao cho k = nguyên thì k = 8.
ab
a+1 b+1
Câu 17. Chứng minh rằng có vô số cặp số nguyên dương (a; b) thỏa mãn + = 4.
b a

CHÚC CÁC BẠN THI TỐT!

37
Hướng tới VMO 2020 Tạp chí và tư liệu toán học

Tài liệu
[1] Bước nhảy Viète - Hà Tuấn Dũng, Đại học Sư phạm Hà Nội 2.
[2] Bước nhảy Viète - Phạm Huy Hoàng, Chuyên đề số học Mathscope.
[3] Đặng Hùng Thắng, Nguyễn Văn Ngọc, Vũ Kim Thủy - Bài giảng số học. NXB Giáo dục 1996.
[4] Vận dụng định lí Viète giải các bài toán số học - Nguyễn Công Lợi.
[5] Lời giải và bình luận VMO 2012 - Trần Nam Dũng. Diễn đàn Mathscope, 2012.
[6] The Method of Vieta Jumping - Yimin Ge, Mathematical Reflections 5 (2007).
[7] A Rational Function Whose Integral Values Are Sums of Two Squares - Sam Vandervelde.
[8] Diễn đàn AoPS Online, https://artofproblemsolving.com/community.
[9] Diễn đàn toán học Việt Nam - VMF, https://diendantoanhoc.net/.

38
`

CHINH PHỤC
OLYMPIC TOÁN

Các bài toán


VẬN DỤNG CAO
DÃY SỐ
HAPPY NEW YEAR 2019

TẠP CHÍ VÀ TƯ LIỆU


TOÁN HỌC
LỜI GIỚI THIỆU

Nhân dịp năm mới 2019 thay mặt nhóm quản trị viên Tạp chí và tƣ liệu toán học ,
lời đầu tiên xin gửi tới các bạn đọc , các thầy cô theo dõi fanpage một lời chúc sức
khỏe, mong rằng sang năm mới các thầy cô sẽ đạt đƣợc nhiều thành công hơn
trong công việc, các bạn học sinh sẽ thực hiện ƣớc mơ nguyện vọng vào các
trƣờng Đại học của mình. Chuyên đề “CÁC BÀI TOÁN VẬN DỤNG CAO DÃY SỐ”
đƣợc 2 thành viên trong nhóm Chinh Phục Olympic Toán sƣu tầm và biên soạn với
mục đích chào xuân năm mới cũng nhƣ là một món quà với các bạn theo dõi page
trong suốt 1 năm vừa qua và đồng thời ủng hộ bọn mình phát triển tới nay, xin
gửi lời cảm ơn tới tất cả mọi ngƣời. Nhƣ các bạn đã biết, trƣớc kia thì dãy số tuy
không phải là một phần quan trọng trong kì thi THPT Quốc Gia, kì thi đại học
nhƣng trong 2 năm gần đây vấn đề này đã đƣợc các trƣờng kết nối với các mảng
khác nhƣ hàm số, mũ – logarit, tích phân... và cũng gây ra không ít những bỡ
ngỡ, những sự lúng túng cho các bạn lần đầu gặp những bài nhƣ thế. Vì vậy trong
chủ đề này, chúng mình và các bạn sẽ cùng tìm hiểu các bài toán liên quan tới
chúng, hy vọng phần nào sẽ giúp mọi ngƣời có kinh nghiệm và hƣớng giải quyết
khi gặp các bài toán nhƣ thế này. Để hoàn thành đƣợc chuyên đề này bọn mình
cũng đã sƣu tầm và tham khảo, đồng thời cũng nhận đƣợc sự giúp đỡ của các
thầy cô, xin gửi lời cảm ơn tới

 NHÓM STRONG TEAM TOÁN VD – VDC.


 ANH PHẠM MINH TUẤN – ADMIN NHÓM PI
 CÁC THÀNH VIÊN TRONG NHÓM CHINH PHỤC OLYMPIC TOÁN
Mặc dù chuyên đề đƣợc biên soạn cẩn thận tuy nhiên sẽ không thể tránh khỏi
những thiếu sót, mọi ý kiến thắc mắc vui lòng gửi về 1 trong 2 địa chỉ sau

NGUYỄN MINH TUẤN


Sinh viên K14 – Đại học FPT
Email: tuangenk@gmail.com
Facebook: https://www.facebook.com/tuankhmt.fpt

NGUYỄN NHẬT LINH


Chuyên Thái Bình
Email: linhnhatnhatlinhnguyen@gmail.com
Facebook: https://www.facebook.com/profile.php?id=100009880805520

MỘT LẦN NỮA, XIN GỬI LỜI CẢM ƠN MỌI NGƢỜI ĐÃ THEO DÕI FANPAGE TRONG
SUỐT THỜI GIAN QUA, HY VỌNG CÁC BẠN SẼ TIẾP TỤC ỦNG HỘ BỌN MÌNH PHÁT
TRIỂN HƠN NỮA

THANK YOU! HAPPY NEW YEAR!


TUYỂN TẬP MỘT SỐ NHÓM CÂU HỎI VẬN DỤNG CAO MÔN TOÁN

CÁC BÀI TOÁN VẬN DỤNG CAO DÃY SỐ


Nguyễn Minh Tuấn – Nguyễn Nhật Linh
CÂU CHUYỆN MỞ ĐẦU
Trước khi cùng nhau đi vào tìm hiểu các bài toán dãy số của chuyên đề này, bọn mình
muốn gửi tới các bạn một bài viết rất hay về nhà bác học Newton để phần nào làm giảm
bớt độ nhạt nhẽo của chuyên đề, bài viết mang tên “ 10 phát minh nổi tiếng của Newton”
Mời các bạn cùng thưởng thức!

Nhắc tới nhà phát minh vĩ đại Isaac Newton, chắc chắn ai cũng nghĩ tới câu chuyện "quả
táo rơi vào đầu" đã làm nên thuyết vạn vật hấp dẫn. Không chỉ vậy, ông còn sở hữu nhiều
phát minh vĩ đại giúp thay đổi thế giới: ba định luật chuyển động, vi phân, tích phân, giả
thuật kim...
Tại nhà thờ Westminster Abbey, một
dòng chữ bằng tiếng Latin đã được khắc
lên trên bia mộ của Newton "Hic
depositum est, quod mortale fult Isaac
Newtoni" với ý nghĩa là "Một con người đã
từng tồn tại và trang hoàng cho sự phát triển
của nhân loại". Lời ca tụng trên không hề
quá mức đối với những di sản mà thiên
tài Newton đã để lại cho loài người.
Cùng điểm lại 10 phát minh quan trọng
và nổi tiếng nhưng cũng hết sức thú vị
Của Isaac Newton trong suốt sự nghiệp sáng tạo của ông mà có thể chúng ta ít khi chú ý
đến.

I. Ý TƯỞNG CỦA NEWTON KHẨU PHÁO BẮN VÀO QUỸ ĐẠO.

Đối với một số ý kiến xuyên tạc sẽ cho rằng làm sao một người đàn ông đang ngáy ngủ và
một quả táo vô tình rơi xuống lại làm nên một phát minh vĩ đại đến như vậy? Kết quả của
quá trình "chờ sung rụng" chăng? Không hề, điều đó chỉ đến với một bộ óc thiên tài luôn
suy nghĩ về các quy luật vật lý mà cụ thể là lực hấp dẫn. Không chỉ dừng lại ở trọng lực
mà Newton còn đưa ra nhiều ý tưởng khác đi trước thời đại. Trong định luật hấp dẫn phổ
quát, Newton đã diễn tả đến một ngọn núi khổng lồ mà đỉnh của nó là khoảng trên bầu
khí quyển của Trái Đất, trên đỉnh có đặt một khẩu pháo vô cùng lớn có thể bắn một viên
đạn theo chiều ngang ra ngoài không gian.

Tinh hoa của toán học nằm ở tự do của nó – Georg Cantor Chinh phục olympic toán | 1
CÁC BÀI TOÁN VẬN DỤNG CAO DÃY SỐ

Ý tưởng của Newton khẩu pháo bắn vào quỹ đạo


Newton không hề có ý định tạo ra một loại siêu vũ khí nhằm bắn những kẻ xâm lược
ngoài hành tinh! Khẩu pháo của ông là một ý tưởng thí nghiệm nhằm giải thích làm thế
nào để đưa một vật thể vào một quỹ đạo quay quanh Trái Đất.

Nếu lực hấp dẫn tác động lên quá pháo, nó sẽ bay theo đường tùy thuộc vào vận tốc ban
đầu của nó . Tốc độ thấp, nó chỉ đơn giản là sẽ rơi trở lại trên Trái đất. Nếu tốc độ là tốc độ
quỹ đạo, nó sẽ đi lòng vòng xung quanh Trái đất theo một quỹ đạo tròn cố định giống như
mặt trăng. Tốc độ cao hơn so với vận tốc quỹ đạo, nhưng không đủ lớn để rời khỏi trái đất
hoàn toàn (thấp hơn vận tốc thoát) nó sẽ tiếp tục xoay quanh Trái đất dọc theo một quỹ
đạo hình elip. Tốc độ rất cao, nó thực sự sẽ rời khỏi quỹ đạo và bay ra ngoài vũ trụ.

Thí nghiệm trên đã được trình bày trong Principia Mathematica vào năm 1687, theo đó, tất
cả mọi hạt đều gây ra một lực hấp dẫn và bị hấp dẫn bởi những vật thể khác. Lực tương
tác này phụ thuộc vào trọng lượng và khoảng cách của hạt hay vật thể đó. Quy tắc này chi
phối tất cả các hiện tượng từ mưa rơi cho đến quỹ đạo của các hành tinh. Đây chính là tác
phẩm nổi tiếng với nhiều đóng góp quan trọng cho vật lý học cổ điển và cung cấp cơ sở lý
thuyết cho du hành không gian cũng như sự phát triển của tên lửa sau này. Sau đó,
Einstein cùng các nhà vật lý thế kỷ 16, 17 đã tiếp tục củng cố học thuyết của Newton để
cho chúng ta những hiểu biết về lực hấp dẫn như ngày nay.

II. CÁNH CỬA DÀNH CHO CHÓ MÈO.

Không chỉ có tầm nhìn mang tính vĩ mô như khẩu pháo không gian và phát hiện ra mối
liên hệ giữa vạn vật trong vũ trụ, Newton cũng dùng trí tuệ tuyệt vời của mình để giải
quyết những vấn đề thường thức trong đời sống hàng ngày. Điển hình là phương pháp

2 | Chinh phục olympic toán Điều ta biết là giọt nước, điều ta chưa biết là đại dương - Newton
TUYỂN TẬP MỘT SỐ NHÓM CÂU HỎI VẬN DỤNG CAO MÔN TOÁN

giúp các mèo không cần cào cấu vào cánh cửa nhờ vào tạo ra một lối đi dành riêng cho
chúng.
Như chúng ta đã biết, Newton
không kết hôn và cũng có ít
các mối quan hệ bạn bè, đổi lại
ông chọn mèo và chó làm bầu
bạn trong căn phòng của của
mình. Hiện nay, có nhiều giả
thuyết và lập luận cho rằng
ông dành nhiều mối quan tâm
đến những "người bạn" bé nhỏ
của mình. Một số sử gia
đương đại cho rằng Newton là
một người rất yêu động vật.
Một số còn chỉ ra rằng ông đặt tên cho một con chó của mình là Diamond (kim cương). Dù
vậy, một số nhà sử học vẫn nghi ngờ về giả thuyết trên.

Một câu chuyện kể rằng trong quá trình nghiên cứu của Newton tại Đại học Cambridge,
các thí nghiệm của ông liên tục bị gián đoạn bởi một con mèo của ông luôn cào vào cánh
cửa phòng thí nghiệm gây ra những âm thanh phiều toái. Để giải quyết vấn đề, ông đã
mời một thợ mộc tại Cambridge để khoét 2 cái lỗ trên cửa ra vào phòng thí nghiệm: 1 lỗ
lớn dành cho mèo mẹ và 1 lỗ nhỏ dành cho mèo con!

Dù câu chuyện trên là đúng hay sai thì theo các ghi chép đương thời sau khi Newton qua
đời thì có một sự thật hiển nhiên rằng người ta đã tìm thấy 1 cánh cửa với 2 cái lỗ tương
ứng với kích thước của mèo mẹ và mèo con. Cho tới ngày nay vẫn còn nhiều tranh cãi
xung quanh câu chuyện trên. Tuy nhiên, nhiều ý kiến vẫn cho rằng chính Newton mới là
tác giả của cánh cửa dành cho chó mèo vẫn còn được sử dụng ngày nay.

III. BA ĐỊNH LUẬT CHUYỂN ĐỘNG CỦA NEWTON.

Trong khi các sử gia vẫn còn tranh cãi về những cánh cửa dành cho thú cưng có phải là
của Newton hay không thì không một ai có thể phủ nhận đóng góp của Newton cho hiểu
biết của con người trong vật lý học ngày nay. Tầm quan trọng tương đương với việc phát
hiện ra định luật vạn vật hấp dẫn, 3 định luật về chuyển động được Newton giới thiệu vào
năm 1687 trong tác phẩm Philosophiae Naturalis Principia Mathematica (Các nguyên lý
toán học trong triết học tự nhiên). 3 định luật của ông đã đặt nền móng vững chắc cho sự
phát triển của cơ học cổ điển (còn gọi là cơ học Newton) trong thời gian sau này.

Tinh hoa của toán học nằm ở tự do của nó – Georg Cantor Chinh phục olympic toán | 3
CÁC BÀI TOÁN VẬN DỤNG CAO DÃY SỐ

3 định luật của ông được miêu tả ngắn


gọn như sau:
1. Nếu một vật không chịu tác dụng
của lực nào hoặc chịu tác dụng của các lực
có hợp lực bằng không thì nó giữ nguyên
trạng thái đứng yên hoặc chuyển động
thẳng đều.
2. Gia tốc của 1 vật cùng hướng với
lực tác dụng lên vật. Độ lớn của gia tốc tỷ
lệ thuận với độ lớn của lực và tỉ lệ nghịch
với khối lượng của vật.
Ba định luật chuyển động của Newton
3. Trong mọi trường hợp, khi vật A tác dụng lên vật B một lực, thì vật B cũng tác
dụng lại vật A một lực. Hai lực này có cùng giá, cùng độ lớn nhưng ngược chiều.

Ngày nay, chúng ta có thể dễ dàng phát


biểu và hiểu về 3 định luật nổi tiếng trên.
Tuy nhiên, các học giả trong lịch sử đã phải
vật lộn với những khái niệm cơ bản về
chuyển động trong suốt nhiều thế kỷ. Nhà
triết học Hy Lạp Aristotle từng nghĩ rằng sở
dĩ khói có thể bay lên trên không là vì khói
chứa nhiều không khí. Trước đó, các học giả
khác lại nghĩ rằng khói bay lên trời để tụ
hợp cùng với những đám khói "bạn bè" của
chúng. Nhà triết học Pháp René Descartes
đã từng nghĩ tới những lý thuyết về chuyển
động tương tự như Newton nhưng cuối
cùng, ông vẫn cho rằng Thiên Chúa mới
chính là động lực của các chuyển động.

Bìa quyển sách Philosophiae Naturalis Principia 3 định luật Newton như một vẻ đẹp đến từ
Mathematica (Các nguyên lý toán học trong sự tối giản trong khoa học. Dù đơn giản
triết học tự nhiên) xuất bản năm 1687 như thế, nhưng đây chính là căn cứ để các
nhà khoa học có thể hiểu được tất cả mọi thứ chuyển động từ của các hạt electron cho tới
chuyển động xoắn ốc của cả thiên hà.

IV. HÒN ĐÁ PHÙ THỦY CỦA “ NHÀ GIẢ KIM THUẬT “ NEWTON.

4 | Chinh phục olympic toán Điều ta biết là giọt nước, điều ta chưa biết là đại dương - Newton
TUYỂN TẬP MỘT SỐ NHÓM CÂU HỎI VẬN DỤNG CAO MÔN TOÁN

Trong một bức vẽ về một nhà giả kim thuật, chúng ta thấy các biểu tượng hành tinh diễn
tả các kim loại trong một quyển sách đang mở ra dưới sàn nhà. Đây được cho là các biểu
tượng mà Newton đã sử dụng trong các ghi chép của ông.
Newton đã cống hiến rất nhiều cho
nhân loại với những khám phá khoa
học của ông. Bên cạnh đó, người ta
cũng nhắc đến ông như 1 trong những
nhà giả kim học lỗi lạc nhất: huyền
thoại giả kim thuật với hòn đá phù
thủy. Các văn bản ghi chép lại còn được
lưu trữ đến ngày nay đã có nhiều mô tả
khác nhau về hòn đá này: từ khả năng
tạo nên người từ đá cho tới khả năng
chuyển hóa từ chì thành vàng. Thậm
chí, những người bấy giờ còn cho rằng Hòn đá phù thủ của “nhà giả kim thuật” Newton
hòn đá của ông có thể chữa bệnh hoặc có thể biến một con bò không đầu thành một bầy
ong

Có lẽ các bạn sẽ thắc mắc tại sao một biểu tượng của khoa học lại trở thành một nhà giả
kim thuật? Để trả lời câu hỏi đó, hãy nghĩ đến bối cảnh bấy giờ, cuộc cách mạng khoa học
chỉ mới đạt được động cơ hơi nước vào những năm 1600. Các nhà giả kim thuật bấy giờ
vẫn còn tồn tại cùng với những thủ thuật lỗi thời của họ cùng với các học thuyết và triết
học huyền bí nhằm mê hoặc một số người. Dù vậy, các ghi chép giả kim thuật vẫn được
cho là những thí nghiệm hóa học.

Bút tích còn lưu lại của Newton về nghiên cứu giả kim
Tuy nhiên, những ghi chép trong suốt 30 năm làm thí nghiệm của Newton đã tiết lộ rằng
ông cũng hy vọng về một cái gì đó hơn là những phản ứng hóa học bình thường, thậm chí

Tinh hoa của toán học nằm ở tự do của nó – Georg Cantor Chinh phục olympic toán | 5
CÁC BÀI TOÁN VẬN DỤNG CAO DÃY SỐ

là hứa hẹn về việc biến các nguyên tố khác thành vàng. Theo sử gia William Newman, ông
cho rằng Newton muốn tìm kiếm những "quyền lực siêu hạn trong tự nhiên."

Đây chính là những căn cứ cho lập luận rằng Newton cũng đã có những nghiên cứu và để
lại ghi chép về giả kim mà người đương thời gọi là "hòn đá phù thủy." Các ghi chép cho
thấy ông đã tìm cách tạo nên những loại nguyên tố bí ẩn lúc bấy giờ. Trên thực tế, Newton
đã có những nỗ lực nhằm tạo ra một loại hợp kim đồng màu tím. Dù vậy, nghiên cứu của
ông đã thất bại.

Đây có thể không phải là một sáng chế của Newton, nhưng nó cũng cho chúng ta một cái
nhìn về những suy nghĩ cũng như thời gian mà ông dành cho các nghiên cứu khoa học.
Vào năm 2005, nhà sử học Newman cũng đã tạo nên một "hòn đá phù thủy" dựa trên các
ghi chép 300 năm trước của Newton và dĩ nhiên, không có sự chuyển hóa tạo thành vàng
xảy ra.

V. CHA ĐẺ CỦA CÁC PHÉP TÍNH VI PHÂN.

Nếu bạn đã hoặc đang đau đầu với môn toán học mà đặc biệt là tích phân và vi phân đã
cày nát bộ não của bạn, bạn có thể đổ một phần lỗi cho Newton! Trên thực tế, hệ thống
toán học chính là một công cụ để chúng ra có thể tìm hiểu được mọi thứ trong vũ trụ này.
Giống như nhiều nhà khoa học cùng thời, Newton cũng đã nhận thấy rằng các lý thuyết
đại số và hình học trước đó không đủ cho yêu cầu nghiên cứu khoa học của ông. Hệ thống
toán học đương thời không đủ để phục vụ ông.

Bút tích của Newton còn lưu giữ đến ngày nay
Các nhà toán học lúc bấy giờ có thể tính toán được vận tốc của một con tàu nhưng họ vẫn
không thể tính toán được mối liên hệ với gia tốc của nó cũng như tỷ lệ của lực tác động.

6 | Chinh phục olympic toán Điều ta biết là giọt nước, điều ta chưa biết là đại dương - Newton
TUYỂN TẬP MỘT SỐ NHÓM CÂU HỎI VẬN DỤNG CAO MÔN TOÁN

Họ vẫn chưa thể tính toán được góc bắn là bao nhiêu để viên đạn pháo bay đi xa nhất. Các
nhà toán học đương thời vẫn cần một phương pháp để tính toán các hàm có nhiều biến.

Một sự kiện đã xảy đến trong quá trình nghiên cứu của Newton, một đợt bùng phát bệnh
dịch hạch đã khiến hàng loạt người chết trên khắp các đường phố tại Cambridge. Tất cả
các cửa hàng đều đóng cửa và dĩ nhiên, Newton cũng phải hạn chế đi ra ngoài. Đó là
khoảng thời gian 18 tháng nghiên cứu của Newton để rồi ông xây dựng nên một mô hình
toán học và đặt tên là "khoa học của sự liên tục".

Ngày nay, chúng ta biết đó chính là các phép tính vi-tích phân. Một công cụ quan trọng
trong vật lý, kinh tế học và các môn khoa học xác suất. Vào những năm 1960, chính các
hàm số vi-tích phân này đã cung cấp công cụ cho phép các kỹ sư phi thuyền Apollp có thể
tính toán được các số liệu trong sứ mạng đặt chân lên Mặt Trăng.

Dĩ nhiên, một mình Newton không tạo nên phép toán mà chúng ta sử dụng ngày nay.
Ngoài Newton, nhà toán học người Đức Gottfried Leibniz (1646-1716) cũng đã độc lập
phát triển mô hình phép tính vi - tích phân trong cùng thời gian với Newton. Dù vậy,
chúng ta vẫn phải công nhận tầm quan trọng của Newton trong sự phát triển toán học
hiện đại với các đóng góp không nhỏ của ông.

VI. SINH SỰ VỚI CẦU VỒNG.

Cầu vồng? Cầu vồng là


gì? Bạn nghĩ rằng
Newton để yên cho
những bí mật bên trong
cầu vồng? Không hề!
Thiên tài của chúng ta đã
quyết tâm giải mã những
điều ẩn chứa bên trong
hiện tượng thiên nhiên
này. Vào năm 1704, ông
đã viết một quyển sách Thí nghiệm của Newton
về vấn đề khúc xạ ánh sáng với tiêu đề "Opticks". Quyển sách đã góp một phần không
nhỏ trong việc thay đổi cách nghĩ của chúng ta về ánh sáng và màu sắc.

Các nhà khoa học bấy giờ đều biết rằng cầu vồng được hình thành khi ánh sáng bị khúc xạ
và phản xạ trong những hạt nước mưa trong không khí. Dù vậy, họ vẫn chưa thể lý giải rõ

Tinh hoa của toán học nằm ở tự do của nó – Georg Cantor Chinh phục olympic toán | 7
CÁC BÀI TOÁN VẬN DỤNG CAO DÃY SỐ

ràng được tại sao cầu vồng lại chứa nhiều màu sắc như vậy. Khi Newton bắt đầu nghiên
cứu tại Cambridge, các lý thuyết phổ biến trước đó vẫn cho rằng các hạt nước bằng cách
nào đó đã nhuộm nhiều màu sắc khác nhau lên tia sáng Mặt Trời.

Bằng cách sử dụng một lăng kính và một chiếc đèn, Newton đã thực hiện thí nghiệm bằng
cách cho ánh sáng chiếu qua lăng kính. Và kết quả như tất cả chúng ra đều biết, ánh sáng
bị tách ra thành các màu như cầu vồng.

VII. KÍNH VIỄN VỌNG PHẢN XẠ.

Newton được sinh ra trong thời kỳ mà sự hiện diện của kính viễn vọng vẫn còn khá mờ
nhạt. Mặc dù vậy, các nhà khoa học đã có thể chế tạo nên các mô hình sử dụng một tập
hợp các thấu kính thủy tinh để phóng to hình ảnh. Trong thí nghiệm với các màu sắc của
Newton, ông đã biết được các màu sắc khác nhau sẽ khúc xạ với các góc độ khác nhau, từ
đó tạo nên một hình ảnh lờ mờ cho người xem.

Để cải tiến chất lượng hình ảnh,


Newton đã đề xuất sử dụng
một gương khúc xạthay cho các
thấu kính khúc xạ trước đó. Một
tấm gương lớn sẽ bắt lấy hình
ảnh, sau đó một gương nhỏ hơn
sẽ phản xạ hình ảnh bắt được tới
mắt của người ngắm. Phương
pháp này không chỉ tạo nên
hình ảnh rõ ràng hơn mà con
cho phép tạo nên một kính viễn
vọng với kích thước nhỏ hơn.
Một số ý kiến cho rằng, nhà toán
Một bản sao của chiếc kính viễn vọng phản xạ do Newton
học người Scotland James
chế tạo và đã trình bày trước Hội đồng hoàng gia vào năm
Gregory là người đầu tiên đề
1672
xuất ý tưởng chế tạo kính viễn vọng phản xạ vào năm 1663 dù mô hình này vẫn chưa thể
hoạt động hoàn chỉnh. Tuy nhiên, dựa trên các ghi chép còn lưu trữ lại, các nhà sử học cho
rằng Newton mới là người đầu tiên có thể chế tạo một chiếc kính viễn vọng phản xạ dựa
trên lý thuyết do ông đề xuất.

Trên thực tế, Newton đã tự mài các tấm gương, lắp ráp một mẫu thử nghiệm và trình bày
nó với Hội đồng hoàng gia vào năm 1672. Đó chỉ đơn thuần là 1 thiết bị dài 15 cm, có khả

8 | Chinh phục olympic toán Điều ta biết là giọt nước, điều ta chưa biết là đại dương - Newton
TUYỂN TẬP MỘT SỐ NHÓM CÂU HỎI VẬN DỤNG CAO MÔN TOÁN

năng loại bỏ sự khúc xạ và có độ phóng đại lên tới 40 lần. Đến ngày nay, gần như tất cả
các đài thiên văn học đều sử dụng các biến thể của thiết kế ban đầu nói trên của Newton.

VIII. ĐỒNG XU HOÀN HẢO.

Vào những cuối những năm 1600, hệ thống tài chính tại Anh lâm vào tình trạng khủng
hoảng nghiêm trọng. Bấy giờ, toàn bộ hệ thống tiền tệ trong cả nước Anh đều sử dụng
các đồng xu bạc và dĩ nhiên, bản thân bạc có giá trị cao hơn so với giá trị định danh được
in trên mỗi đồng xu. Lúc đó nảy sinh ra một vấn đề, có người sẽ cắt xén bớt hàm lượng
bạc và thêm vào các kim loại khác trong quá trình nấu và đúc tiền. Lượng bạc cắt xén
được sẽ bị "chảy máu" sang Pháp thông qua đường biên giới để bán được giá cao hơn.

Những đồng 2 pound tại Anh với các khía 2 xung quanh cạnh

Thậm chí, bấy giờ còn là cuộc khủng hoảng của việc tranh giành nhau nhận thầu đúc tiền.
Do đó, lòng tin của người dân vào hệ thống tài chính suy giảm nghiêm trọng. Đồng thời,
các tổ chức tội phạm làm tiền giả cũng mặc sức lan tràn do đã không còn một đồng tiền
chuẩn đáng tin tưởng nào đang lưu thông. Mặt khác, sự gian lận cũng diễn ra ngay trong
quá trình đúc tiền. Sau khi đúc mỗi mẻ tiền xu, người ta sẽ cân mỗi đồng xu lấy ra và xem
nó lệch so với tiêu chuẩn là bao nhiêu. Nếu giá trị bạc dư ra lớn hơn so với giá trị in trên
nó, những kẻ đầu cơ sẽ mua chúng, nấu chảy ra và tiếp tục bán lại cho chính xưởng đúc
tiền để kiếm lời.

Trước tình hình đó, vào năm 1696, chính phủ Anh đã kêu gọi Newton giúp tìm ra giải
pháp tìm ra giải pháp chống nạn sao chép và cắt xén đồng xu bạc. Newton đã có một bước
Tinh hoa của toán học nằm ở tự do của nó – Georg Cantor Chinh phục olympic toán | 9
CÁC BÀI TOÁN VẬN DỤNG CAO DÃY SỐ

đi hết sức táo bạo là thu hồi toàn bộ tiền xu trên khắp đất nước, tiến hành nấu lại và đúc
theo một thiết kế mới của ông. Bước đi này đã khiến cho toàn bộ nước Anh không có tiền
trong lưu thông trong suốt 1 năm.

Bấy giờ, Newton đã làm việc cật lực trong suốt 18 giờ mỗi ngày để rồi cuối cùng, thiết kế
tiền xu mới cũng được ra đời. Những đồng tiền mới được đúc ra với chất lượng bạc cao
hơn, đồng thời rìa mỗi đồng xu đều được khía các cạnh theo một công thức đặc biệt. Nếu
không có các cỗ máy khía cạnh chuyên dụng thì sẽ không thể nào tạo ra được các đồng xu
mang đặc trưng như do Hoàng gia đúc ra.

IX. SỰ MẤT NHIỆT.

Trong các nghiên cứu của mình, Newton cũng đã dành nhiều thời gian để tìm hiểu khía
cạnh vật lý của hiện tượng lạnh đi của các chất. Vào cuối những năm 1700, ông đã tiến
hành các thí nghiệm với quả cầu sắt nung đỏ. Ông đã lưu ý trong các ghi chép rằng có sự
khác biệt giữa nhiệt độ của quả bóng sắt và không khí xung quanh. Cụ thể, nhiệt độ chênh
lệch lên tới 10 độ C. Và ông cũng nhận ra rằng tốc độ mất nhiệt tỷ lệ thuận với sự khác biệt
về nhiệt độ.

Từ đó, Newton hình thành nên định luật về trạng thái làm mát. Theo đó, tốc độ mất nhiệt
của cơ thể tỷ lệ thuận với sự khác biệt về nhiệt độ giữa môi trường xung quanh so với
nhiệt độ cơ thể. Sau này, nhà hóa học người Pháp Piere Dulong và nhà vật lý Alexis Prtot
đã hoàn thiện định luật trên vào năm 1817 dựa trên nền tảng từ nghiên cứu của Newton.
Nguyên tắc của Newton đã đặt nền móng cho nhiều nghiên cứu khác của vật lý hiện đại
từ lò phản ứng hạt nhân an toàn cho tới việc thám hiểm không gian.

X. DỰ ĐOÁN CỦA NEWTON VỀ NGÀY TẬN THẾ.

Ngày tận thế luôn là nỗi ám ảnh của con người. Dù vậy, Newton không phải là dạng
người có thể dễ dàng chấp nhận nỗi sợ hãi về ngày tận thế qua những câu chuyện hay
những truyền thuyết. Bản thân Newton là một người thực tế và luôn tìm cách kiểm định,
đưa ra các quan điểm của mình trong quá trình nghiên cứu Kinh Thánh.

Trong quá trình nghiên cứu, Newton đã không đặt nặng khía cạnh Thần học mà dùng các
kiến thức của mình nhằm cố lý giải vấn đề. Theo các ghi chép cách đây 300 năm còn được
lưu trữ đến ngày nay cho thấy Newton đã nghiên cứu Book of Daniel. Để phục vụ nghiên
cứu, ông đã tự học tiếng Do Thái, tập trung nghiên cứu triết học Do Thái bí truyền.

10 | Chinh phục olympic toán Điều ta biết là giọt nước, điều ta chưa biết là đại dương - Newton
TUYỂN TẬP MỘT SỐ NHÓM CÂU HỎI VẬN DỤNG CAO MÔN TOÁN

Hình vẽ 4 loài thú dữ xuất hiện vào ngày tận thế mô tả trong Book of Daniel

Qua nghiên cứu, ông dự đoán ngày tận cùng của thế giới là vào năm 2060 hoặc có thể là
sau đó nhưng không thể sớm hơn. Dù sao đi nữa, đó vẫn là những gì mà ông tuyên bố với
mọi người vào thế kỷ 18. Dĩ nhiên, ngày nay, các nhà khoa học đã có một lời giải đáp hoặc
dự đoán tốt hơn cho hiện tượng tận thế nói chung. Qua đó, chúng ta phần nào hiểu được
thêm về quan điểm của 1 nhà khoa học vào thế kỷ 18 về ngày tàn của nhân loại.

Tinh hoa của toán học nằm ở tự do của nó – Georg Cantor Chinh phục olympic toán | 11
CÁC BÀI TOÁN VẬN DỤNG CAO DÃY SỐ

A. ĐỀ BÀI.
Câu 1. Cho hàm số y  x 3  2009 x có đồ thị là C  . M1 là điểm trên C  có hoành độ
x1  1 . Tiếp tuyến của C  tại M1 cắt C  tại điểm M 2 khác M1 , tiếp tuyến của C  tại
M 2 cắt C  tại điểm M 3 khác M 2 , <, tiếp tuyến của C  tại M n 1 cắt C  tại Mn khác
M n 1  n  4; 5;... , gọi  xn ; y n  là tọa độ điểm Mn . Tìm n để: 2009xn  yn  2 2013  0 .
A. n  685 B. n  679 C. n  672 D. n  675
Câu 2. Một hình vuông ABCD có cạnh AB  2 , diện tích S1 . Nối 4 trung điểm A1 , B1 ,
50

C 1 , D1 theo thứ tự của 4 cạnh AB , BC , CD , DA ta được hình vuông thứ hai là A1 B1C 1D1
có diện tích S 2 . Tiếp tục như thế ta được hình vuông thứ ba A2 B2C 2 D2 có diện tích S 3 và cứ
tiếp tục như thế, ta được diện tích S4 , S5 ,... Tính S  S1  S2  S3  ...  S100
A. S  2 101  2 B. S  2 101  1 C. S  2 100  2. D. S  2 100  1
Câu 3. Khối tứ diện ABCD có thể tích V , khối tứ diện A1 B1C 1D1 có thể tích V1 , các đỉnh
A1 , B1 , C 1 , D1 lần lượt là trọng tâm các tam giác BCD , CDA , DAB , ABC . Khối tứ diện
A2 B2C 2 D2 có thể tích V2 , các đỉnh A2 , B2 , C 2 , D2 lần lượt là trọng tâm các tam giác
B1C 1D1 , C 1 D1 A1 , D1 A1 B1 , A1 B1C 1 . Cứ tiếp tục như thế ta được khối tứ diện An BnC n Dn có
thể tích Vn , các đỉnh An , Bn , C n , Dn lần lượt là trọng tâm các tam giác Bn1C n1Dn1 ,
C n 1Dn 1 An 1 , Dn 1 An 1 Bn 1 , An 1 Bn 1C n 1 . Tính S  V1  V2  ...  V2018 .

A. S 
3 2018
 1V
B. S 
 27 2019
 1V
C. S 
 27 2018
 1V
D. S 
3 2019
 1V
2.32018 26.27 2019 26.27 2018 2.32019
Câu 4. Tam giác mà ba đỉnh của nó là ba trung điểm ba cạnh của tam giác ABC được gọi
là tam giác trung bình của tam giác ABC . Ta xây dựng dãy các tam giác
A1 B1C 1 , A2 B2C 2 , A3 B3C 3 ,... sao cho A1 B1C 1 là một tam giác đều cạnh bằng 3 và với mỗi số
nguyên dương n  2 , tam giác An BnC n là tam giác trung bình của tam giác An 1 Bn 1C n 1 .
Với mỗi số nguyên dương n , kí hiệu Sn tương ứng là diện tích hình tròn ngoại tiếp tam
giác An BnC n . Tính tổng S  S1  S2  ...  Sn  ... ?
15 9
A. S  . B. S  4 . C. S  . D. S  5.
4 2
 
Câu 5. Cho dãy số  un  có số hạng tổng quát un  cos  2n  1   . Tổng 2018 số hạng đầu
 6
tiên của dãy số  un  bằng bao nhiêu?
3 3 1
A. 0 B.  C. D.
2 2 2

12 | Chinh phục olympic toán Điều ta biết là giọt nước, điều ta chưa biết là đại dương - Newton
TUYỂN TẬP MỘT SỐ NHÓM CÂU HỎI VẬN DỤNG CAO MÔN TOÁN

u1  3

Câu 6. Cho dãy số  un  thỏa mãn  un  2  1 , n  *
.
u 


n  1
 
1  2  1 un

Khi đó u2019  a  b 3 , a , b  . Tính tổng S  a  b .


A. S  3 B. S  4 C. S  9 D. S  2
Câu 7. Cho tam giác ABC có độ dài các cạnh là a , b , c theo thứ tự lập thành một cấp số
A C x x
cộng. Biết tan tan  với x , y  và tối giản. Tính giá trị của x  y .
2 2 y y
A. 4 B. 1 C. 2 D. 3
 u1  11
Câu 8. Cho dãy số  un  xác định  . Tính giá trị của u2018 ?
un 1  10un  1  9n, n  1
A. u2018  10 2018 B. u2018  20182018 C. u2018  2018 D. u2018  10 2018  2018
1 un u u u u
Câu 9. Cho dãy số (un ) thỏa mãn u1  ; un1  , n  1 . Đặt Sn  1  2  3  ...  n .
2 un  1 1 2 3 n
2019
Tìm giá trị nhỏ nhất của n để Sn  ?
2020
A. 2019 B. 2020 C. 2018 D. 2021
u0  2 2018

Câu 10. Cho dãy  un  :  2 un  1 . Tìm phần nguyên của S   ui .
un  1  u  2 i 1
 n

A. 2020 B. 2017
C. 2019 D. 2018 .
u1  2019

Câu 11. Cho dãy số  un  được xác định bởi:  2019 .
u
 n    u1  u2  u3  ...  un1  , n  1
n
Tính giá trị của biểu thức A  2.u1  2 2 u2  ...  2 2019.u2019 .
A. 32019 B. 2019 C. 3 D. 2
 x1  2

Câu 12. Cho dãy số  xn  xác định bởi  xn 1 2 n  1  3x n *
 x  1  n 2  3x , n 
 n n

20182 8144648 8144648 8144648


A. B. C. D.
2019 12105 12107 12103
Câu 13. Cho dãy số  un  thỏa mãn u1  1, un1  aun2  1, n  1 , a  1 . Biết rằng
lim  u12  u22  ...  un2  2n   b . Giá trị của biểu thức T  ab ?
A. 1 B. 2 C. 1 D. 2

Tinh hoa của toán học nằm ở tự do của nó – Georg Cantor Chinh phục olympic toán | 13
CÁC BÀI TOÁN VẬN DỤNG CAO DÃY SỐ

2 un
Câu 14. Cho dãy số (un ) được xác định bởi u1  và un1  , n  *
 . Tính
3 2  2n  1  un  1
tổng 2019 số hạng đầu tiên của dãy số đó ?
4036 4035 4038 4038
A. B. C. D.
4035 4034 4037 4039
u1  1
Câu 15. Cho dãy số  un  xác định như sau:  , với n  1, 2, 3,...
 un  1  un
2020
 2018un
2019
 un

 u12019 u22019 u32019 un2019 


Tính lim     ...  .
 u2  2018 u3  2018 u4  2018 un  1  2018 
4 3 2 1
A. . B. . C. . D. .
2019 2019 2019 2019
Câu 16. Xét dãy số nguyên x1  34, x2  334, x3  3334, , xn  33...34 (có n số 3). Hỏi có bao
3
nhiêu chữ số 3 trong số 9x2018 ?
A. 6054 B. 6055 C. 6056 D. 6057
un 1
Câu 17. Cho dãy số  un  xác định bởi u1  1 và un1   với n nguyên dương.
2018 2019n1
Tính giới hạn A  lim un
x 

2019 2018
A. B. 2018 C. D. 0
2018 2019
un 1
Câu 18. Cho dãy số (u n ) xác định bởi u1  1 và un1   với n nguyên dương.
2018 2019n1
Tính giới hạn A  lim  u1  u2  un 
x 

2018 2017 2017 2019


A. B. C. D.
2019 2019 2018 2017
x1  1 n
1
Câu 19. Cho dãy số ( xn ) có  ; n  * . Đặt yn   .
xn  1  xn  xn  1  xn  2  xn  3   1 i  1 xi  2

a a
Biết lim yn  với là phân số tối giản và a, b nguyên dương. Khi đó tọa độ M  a ; b 
b b
nằm trên đường tròn nào?
A.  x  1    y  2   4 B.  x  1    y  1   4
2 2 2 2

C.  x  1    y  1   10 D.  x  1   y 2  10
2 2 2

 3
u1 
Câu 20. Cho dãy số  un  xác định bởi  16
u  9u  4 1  3u  4, n  
 n1 n n

Tìm số nguyên dương n nhỏ nhất thỏa mãn un  10 8.


A. 9. B. 10. C. 12. D. 13.

14 | Chinh phục olympic toán Điều ta biết là giọt nước, điều ta chưa biết là đại dương - Newton
TUYỂN TẬP MỘT SỐ NHÓM CÂU HỎI VẬN DỤNG CAO MÔN TOÁN

Câu 21. Xét các cấp số nhân có 2 n  1 số hạng dương ( n là số nguyên dương) thỏa tổng tất
cả các số hạng của nó bằng 400 và tổng tất cả các nghịch đảo của các số hạng của nó bằng
4 . Giá trị lớn nhất của n là?
A. 17 B. 18 C. 19 D. 20
u0  2018
 u
Câu 22. Cho dãy số (un ) được xác định bởi u1  2019 . Hãy tính lim nn .
u  4u  3u ; n  1 3
 n1 n n 1

1 1
A. B. C. 32018 D. 32019
2 3
n3
Câu 23. Cho dãy số  un  xác định bởi u1  1; un1  2un  2 , n  * . Hỏi u2018
n  3n  2
thuộc khoảng nào sau đây?
A.  2 2015 ; 2 2016  B.  2 2016 ; 2 2017  C.  2 2017 ; 2 2018  D.  2 2018 ; 2 2019 

 2
u1  3 u u un 
Câu 24. Cho dãy số  un  xác định  . Tính L  lim  1  22   
u  2nun ; n  *
n 
 2 2 2n 
 n1
n3
1 3 3
A. L  B. L  C. L  1 D. L 
2 4 2
(3xn  1)2019
Câu 25. Cho dãy số  xn  được xác định bởi: x1  1; xn1   xn với n là số
2019
 3x 1  1   3x2  1  3x3  1  3xn  1
2018 2018 2018 2018

nguyên dương. Đặt un     ...  . Tính


3x 2  1 3x 3  1 3x 4  1 3 xn  1  1
lim un
2019 2019 673 673
A. B. C. D.
4 3 3 4
u1  2019

Câu 26. Cho dãy số thực  un  tăng xác định bởi  2
un  2018un  2020un1  1  0, n  1  1

1 1 1
Đặt Sn    ...  . Tính lim Sn
u1  2019 u2  2019 un  2019
1 1
A. 2018 B. C. 2019 D.
2018 2019
u1  1

Câu 27. Cho dãy số:  un   un 1 . Tìm lim u n .
un  1  5n.u , n  2
 n1

A. k  1616 B. k  808 C. k  404 D. k  1212


u1  1, u2  3 un
Câu 28. Cho dãy số  un  được xác định  . Tính lim
un 2  2un1  un  1, n 
* n  n2

Tinh hoa của toán học nằm ở tự do của nó – Georg Cantor Chinh phục olympic toán | 15
CÁC BÀI TOÁN VẬN DỤNG CAO DÃY SỐ

1 1 1
A. 1 B. C. D.
6 3 2
u1  4

Câu 29. Cho dãy số (un ) xác định như sau:  un 2 . Giả sử giới hạn
un 1  un  , n *
 2018
u u u  a
lim  1  2  ...  n    a , b  *
 và ba tối giản. Tính a  3b.
 u2 u3 un1  b
A. 1012 B. 1021 C. 1015 D. 1018
2 xn
Câu 30. Cho dãy số  xn  được xác định như sau x1  ; xn  1  ,  n  1, 2....
3 2  2 n  1  xn  1
Hỏi tổng của 2018 số hạng đầu tiên là bao nhiêu?
4035 2017 2018 4036
A. B. C. D.
4036 2018 2019 4037
u  1; u2  2
Câu 31. Cho dãy số  un   1 . Tổng S  1  2  ...  2017  u2018  u2019
un1  2un  un1  1; n  2
có giá trị bằng bao nhiêu?
A. 2039190 B. 2035153 C. 2037171 D. 2033136
 4
u1  3
Câu 32. Cho dãy số  un  xác định bởi  , n  1 . Tìm lim un .
 n  2 2 u   n  1  u u  n 2 u
 n n n1 n1

3
A. lim un  2 B. lim un  4 C. lim un  D. lim un  3
4
2 n  5n
Câu 33. Cho dãy  un  với un  n . Giả sử ta có tổng sau
2n  5
100
a
1 1 1 1   c
b
S    ....  
u1  1 u2  1 u3  1 u100  1 ba
Trong đó a, b c là các số nguyên dương và a, b là hai số dương nguyên tố cùng nhau . Khi
đó S  a  c  ?
A. 151 B. 153 C. 152 D. 154
u1  9
Câu 34. Cho dãy số  un  được xác định bởi  n1 n1 n1
.
un  n un  1  3.2  2.3 , n  2; 3....
Tính giá trị của u2018 ?

A. u2018  2018 3.2 2018  2.32018 B. u2018  2018 9  3.2 2018  2.32018

C. u2018  2018 3.2 2017  2.32017 D. u2018  2018 3.2 2018  32018 .

16 | Chinh phục olympic toán Điều ta biết là giọt nước, điều ta chưa biết là đại dương - Newton
TUYỂN TẬP MỘT SỐ NHÓM CÂU HỎI VẬN DỤNG CAO MÔN TOÁN

 a1  2008
Câu 35. Cho dãy số thực a1 ; a2 ;...; an được xác định bởi  . Tính
 a1  a2  ...  an  n .an , n  1
2

giá trị của a2008 .


1 2 1 2
A. B. C. D.
2009 2007 2007 2009
 u1  1

Câu 36. Cho dãy số  un  xác định bởi  1
n
. Có bao nhiêu số nguyên
u
 n1  un    , n 
*

 2
1999
dương n sao cho un  .
1000
A. 11 B. 10 C. 15 D. Vô số
1
Câu 37. Cho dãy số  xn  xác định bởi x1  . Biết rằng
4
x1  4x2  9x3  ...   n  1 xn1
2

xn  , n  2, 3....
n2  n  1 
Tính lim  30n2  12 n  2018  xn
15 15
A. 15 B. 30 C. D.
4 2
u1  2
Câu 38. Cho dãy số  un  được xác định bởi công thức  . Tìm
2019un1  un  2018un , n  1
2

u1 u un
giới hạn của dãy số  Sn  xác định bởi công thức Sn   2   .
u2  1 u3  1 un 1  1
2018
A. lim Sn  2018 B. lim Sn  2019 C. lim Sn  D. lim Sn  1
2019
un
Câu 39. Cho dãy số  un  được xác định bởi: u1  1, un1  , n  1, 2, 3,...
un  1
2018  u1  1  u2  1  ...  un  1 
Tính lim .
2019n
2018
A. lim Sn  2018 B. lim Sn  2019 D. lim Sn  1
C. lim Sn 
2019
Câu 40. Cho các số a1 , a2 , a3 , a4 , a5  0 lập thành cấp số cộng với công sai d và
b1 , b2 , b3 , b4 , b5  0 lập thành cấp số nhân với công bội q . Biết rằng a1  b1 và a5  b5 . Hỏi
có bao nhiêu khẳng định luôn đúng trong các khẳng định sau?
i) a2  b2 ii) a3  b3 iii) a4  b4 iv) d  q
A. 1 B. 2 C. 3 D. 4
3  n  1
Câu 41. Cho dãy số  un  biết : u1  1 , un1  un  2n3  3n  1  n  *  . Giá trị nhỏ
n
nhất của n để un  n3  n.32018 là bao nhiêu?

Tinh hoa của toán học nằm ở tự do của nó – Georg Cantor Chinh phục olympic toán | 17
CÁC BÀI TOÁN VẬN DỤNG CAO DÃY SỐ

A. n  2019 B. n  2018 C. n  2017 D. n  2020


Câu 42. Cho dãy số không âm  un  ,  n  *
 được xác định bởi công thức sau
u2  1

 2 1 2  m, n  , m  n 
 um  n  1  um n  1  2  u2 m 1  u2 n  1 
2 2

Khi đó tổng của 2019 số hạng đầu tiên của dãy khi viết dưới dạng thập phân có chữ số ở
hàng đơn vị bằng bao nhiêu?
A. 1 B. 2 C. 3 D. 4
Câu 43. Cho dãy số  xn  được xác định bởi x1  2019, xn 1  x  xn  1, n  1, 2, 3,... . Với
2
n

 1 1 1
mỗi số nguyên dương n , đặt yn  2019    ...   . Khi đó lim y n bằng?
 x1 x 2 xn 
2018 2019
A. B. C. 2018 D. 2019
2019 2018
u1  2020
Câu 44. Cho dãy số (un) được xác định bởi  .
 4n  16n  un  1   n  6n  5  un , n  1
2 2

 4n 
Gọi k  lim  2 .un  thì k có giá trị là?
n 
A. k  1616 B. k  808 C. k  404 D. k  1212
u1  1

Câu 45. Cho dãy  un  được xác định bởi  1  un21  1 , đặt
u
 n  ; n  2, n 
 un1
Sn  u1  u2  ...  un . Hãy chọn mệnh đề sai trong các mệnh đề sau?

 1 
n 1

A.  un  là dãy bị chặn. B. Sn  1  1    
4   2  
C.  un  là dãy giảm D. Sn  n , n  
.
u1  1

Câu 46. Cho dãy số  un  thỏa mãn  2  n  1  un 1 2n
un   , n  *
.

 
2
n n 2
 n  1  1

Tìm giới hạn của dãy số  sn  với sn  n3 un , n  *
.
A. lim  sn    B. lim  sn   0

C. lim  sn   1 1
D. lim  sn   . .
2
un  1  4un2  4un  0

Câu 47. Cho các dãy  un  thỏa:  1  n  *
 . Khi đó u 1 có thể nhận tất cả
u2018 
 2
bao nhiêu giá trị?

18 | Chinh phục olympic toán Điều ta biết là giọt nước, điều ta chưa biết là đại dương - Newton
TUYỂN TẬP MỘT SỐ NHÓM CÂU HỎI VẬN DỤNG CAO MÔN TOÁN

A. 2 2017 B. 2 2018
C. 2 2019 D. 2 2018  1 .
2 2
Câu 48 . Cho dãy số  un  thỏa mãn: u1  1 ; un1  un  a , n  *
.
3
Biết rằng lim  u12  u22  ...  un2  2n   b . Giá trị của biểu thức T  ab là?
A. 2 B. 1 C. 1 D. 2
Câu 49. Cho 2 dãy cấp số cộng un  u1 ; u2 ;...un có công sai d 1 và vn  v1 ; v2 ;...vn có công sai
d2 . Gọi tổng của n số hạng đầu của mỗi cấp số theo thứ tự là
u11
Sn  u1  u2  ...  un  7 n  1 và Tn  v1  v2  ...  vn  14n  27 . Tính tỉ số của
v11
5 4 9 5
A. B. C. D.
3 3 4 4
Câu 50. Cho dãy số  an  xác định bởi a1  5, an 1  q.an  3 với mọi n  1 , trong đó q là
hằng số, q  0 , q  1 . Biết công thức số hạng tổng quát của dãy số viết được dưới dạng
1  q n1
an  .q n1   . Tính   2 ?
1q

A. 13 B. 9 C. 11 D. 16

Câu 51. Cho cấp số nhân u1 , u2 , u3 ,.., un ; trong đó ui  0, i  1, 2,..., n . Biết rằng
1 1 1 1 1
Sn  u1  u2  u3  ...  un  2018 , Tn     ...   2019 và P  u1 .u2 .u3 ....un  .
u1 u2 u3 un 100
Hỏi số tự nhiên nhỏ nhất thỏa mãn P là?

A. 9295 B. 9296 C. 18592 D. 18591


4
Câu 52. Gọi q là công bội của một cấp số nhân , biết tổng ba số hạng đầu bằng 16 , đồng
9
thời theo thứ tự , chúng là số hạng thứ nhất , thứ tư và thứ tám của một cấp số cộng . Hỏi
q thuộc khoảng nào sau đây?
A. q   3; 4  B. q   1; 2  C. q   2; 3  D. q   0; 1 
n
Câu 53. Cho dãy số  un  như sau: un  , n  1 , 2 , ... Tính giới hạn của tổng
1  n2  n4
lim  u1  u2  ...  un  .
x 

1 1 1
A. B. 1. C. D.
4 2 3

 x  10 khi x  2018
Câu 54. Cho hàm số f  x    . Tính giá trị f  1   f  2018  .
 f  f  x  11 
 khi x  2018
A. 1999 B. 2009 C. 4018 D. 4036
Câu 55. Cho dãy un  thỏa mãn 25.2 2 u5  1  15.2 u1  u5  2  5.2 u5  15.2 u1  4  0 và un  1  un  8.

Tinh hoa của toán học nằm ở tự do của nó – Georg Cantor Chinh phục olympic toán | 19
CÁC BÀI TOÁN VẬN DỤNG CAO DÃY SỐ

Giá trị nhỏ nhất của n để un  2019.


A. 512. B. 258. C. 511. D. 257.

Câu 56. Cho một cấp số cộng : u1 , u2 , u3 , u4 thỏa u1u4  u2 u3  6 . Tìm tập xác định D của

hàm f  x    x  u1  x  u2  x  u3  x  u4   9
A. D    ; 6  B. D   6;   C. D  D. D   6; 6 
2 2 2
 1  1   1 
Câu 57. Biết tổng Sn   2     2 2  2   ...   2 n  n  . Giá trị nhỏ nhất của n để
 2  2   2 
399  2n4n
Sn  , n *

4n
A. 41 B. 40 C. 51 D. 50
Câu 58. Cho dãy ( xn ) thỏa mãn x1  5, xn 1  x  2, n  1 . Tính giá trị của
2
n

 1 1 1 
M  lim    ........  
 x1 x1 x 2 x1x2 ...xn 
5  21 5  21 3  31 3  15
A. M  B. M  C. M  D. M 
2 2 3 3
 1 
Câu 59. Cho hàm số y  f  x   ln  1  2  . Biết rằng :
 x 
f  2   f  3   ...  f  2018   ln a  ln b  ln c  ln d
trong đó a , c , d là các số nguyên tố và a  b  c  d . Tính P  a  b  c  d
A. 1986 B. 1698 C. 1689 D. 1989
Câu 60. Cho dãy số  un  thỏa mãn log u1  2  log u1  2 log u10  2 log u10 và un 1  2un với
mọi n  1 . Giá trị nhỏ nhất để un  5100 bằng

A. 247 B. 248 C. 229 D. 290

Câu 61. Cho dãy số  un  thỏa mãn ln 2 u6  ln u8  ln u4  1 và un 1  un .en  1 . Tìm u1

A. e B. e 2 C. e 3 D. e 4

Câu 62. Cho dãy số  un  thỏa mãn e u18  5 e u18  e 4 u1  e 4 u1 và un  1  un  3 với mọi n  1 .
Giá trị lớn nhất của n để log 3 un  ln 2018 bằng?

A. 1419 B. 1418 C. 1420 D. 1417


3
Câu 63. Cho dãy số  an  thỏa mãn a1  1 và 5an1  an  1  , với mọi n  1 . Tìm số
3n  2
nguyên dương n  1 nhỏ nhất để an là một số nguyên.

A. n  123 B. n  41 C. n  39 D. n  49

20 | Chinh phục olympic toán Điều ta biết là giọt nước, điều ta chưa biết là đại dương - Newton
TUYỂN TẬP MỘT SỐ NHÓM CÂU HỎI VẬN DỤNG CAO MÔN TOÁN

 4 e 2 u9  2 eu9  4 eu1 u9  eu1  e 2 u1  3



Câu 64. Cho dãy số  un  thỏa mãn  . Giá trị nhỏ nhất của
 u
 n1  un  3,  n  *

số n để un  1 ?

A. 725 B. 682 C. 681 D. 754

Câu 65. Cho dãy số  un  có số hạng đầu tiên u1  1 thỏa mãn đẳng thức sau :
log 22  5u1   log 22  7 u1   log 22 5  log 22 7 và un 1  7 un với mọi n  1 . Giá trị nhỏ nhất của n
để un  1111111 bằng?

A. 11 B. 8 C. 9 D. 10

Câu 66. Có bao nhiêu giá trị nguyên của tham số a thuộc đoạn  0; 2018  sao cho ba số
a
5x 1  51 x ; ; 25x  25 x theo thứ tự đó, lập thành một cấp số cộng?
2
A. 2008 B. 2006 C. 2018 D. 2007
8
Câu 67. Cho dãy số  un  thỏa mãn 2 2 u1  1  2 3u2  và un 1  2un với
1 
log 3  u32  4u1  4 
4 
mọi n  1 . Giá trị nhỏ nhất của n để Sn  u1  u2  ...  un  5100 bằng

A. 230 B. 231 C. 233 D. 234

Câu 68. Cho dãy số  un  thỏa mãn log 3  2u5  63   2 log 4  un  8n  8  , n  *


.
un .S2 n 148
Đặt Sn  u1  u2  ...  un . Tìm số nguyên dương lớn nhất n thỏa mãn  .
u2 n .Sn 75

A. 18 B. 17 C. 16 D. 19
1 1
1  m
x2  x  1 2
Câu 69. Cho hàm số f  x   e . Biết f  1  . f  2  . f  3  ... f  2017   e n  m, n   với
m
là phân số tối giản. Tính P  m  n 2 .
n
A. 2018 B. 2018 C. 1 D. 1

Câu 70. Cho cấp số cộng  un  có tất cả các số hạng đều dương thoả mãn điều kiện
u1  u2  ...  u2018  4  u1  u2  ...  u1009  . Tìm giá trị nhỏ nhất của biểu thức
P  log 23 u2  log 23 u5  log 23 u14 .
A. 3 B. 1 C. 2 D. 4
Câu 71. Cho cấp số cộng  an  , cấp số nhân  bn  thỏa mãn a2  a1  0 và b2  b1  1 ; và hàm
số f  x   x 3  3x sao cho f  a2   2  f  a1  và f  log 2 b2   2  f  log 2 b1  . Số nguyên
dương n nhỏ nhất và lớn hơn 1 sao cho bn  2018 an là

Tinh hoa của toán học nằm ở tự do của nó – Georg Cantor Chinh phục olympic toán | 21
CÁC BÀI TOÁN VẬN DỤNG CAO DÃY SỐ

A. 16 B. 15 C. 17 D. 18
Câu 72. Cho cấp số nhân  bn  thỏa mãn b2  b1  1 và hàm số f  x   x 3  3x sao cho
f  log 2  b2    2  f  log 2  b1   . Giá trị nhỏ nhất của n để bn  5100 bằng

A. 234 B. 229 C. 333 D. 292

  2 1 1  4 u2 7 6 u1 6
log 1  u1  u3  4 u1  8   e e 3
  
Câu 73. Cho dãy số  un  thỏa mãn  3

u  3  u  n  4  , n  *
 n  1 2  n n 2  3n  2 

3   n  1 2 2018
Giá trị lớn nhất của số n để un 
n1
A. 3472 B. 3245 C. 3665 D. 3453

f  1 . f  3  ... f  2n  1
Câu 74. Cho f  n    n2  n  1   1 n  N * . Đặt un 
2
.
f  2  . f  4  ... f  2n 
10239
Tìm số n nguyên dương nhỏ nhất sao cho un thỏa mãn điều kiện log 2 un  un  .
1024
A. n  23 B. n  29 C. n  21 D. n  33

  
Câu 75. Cho biểu thức A  log 2017  log 2016  log 2015  log ... log 3 log 2 ... 
Biểu thức A có giá trị thuộc khoảng nào trong các khoảng dưới đây?
A.  log 2017; log 2018  B.  log 2019; log 2020 
C.  log 2018; log 2019  D.  log 2020; log 2021 
Câu 76. Cho dãy số  un  xác định bởi un  ln  2n2  1   ln  n2  n  1  , n  1 . Tìm số
2
nguyên n lớn nhất sao cho un   un   . Biết  a kí hiệu phần nguyên của số a là số tự
3
nhiên nhỏ nhất không vượt quá a.

A. 37 B. 36 C. 38 D. 40

Câu 77. Cho dãy số  un  có tất cả số hạng đều dương thỏa mãn un 1  2un và đồng thời

4
u12  u22  ...  un2  un2 1  un2  1  , n  1 . Số tự nhiên n nhỏ nhất để un  5100 là?
3
A. 232 B. 233 C. 234 D. 235

Câu 78. Cho dãy số  un  thỏa mãn ln  u12  u22  10   ln  2u1  6u2  và đồng thời
un  2  un  2un  1  1, n  1 . Giá trị nhỏ nhất của n để un  5050

A. 100 B. 99 C. 101 D. 102

22 | Chinh phục olympic toán Điều ta biết là giọt nước, điều ta chưa biết là đại dương - Newton
TUYỂN TẬP MỘT SỐ NHÓM CÂU HỎI VẬN DỤNG CAO MÔN TOÁN

  391  1 39 
log  u2    log  u1    2
  40  4 4 
Câu 79. Cho dãy số  un  thỏa mãn  .
u  2  n  1  un  1  2n
, n  *

  
n 2
n n 2
 n  1  1

5100  n2  1
Giá trị nhỏ nhất của n để un  .
5100  n3  n 

A. 235 B. 255 C. 233 D. 241


4
Câu 80. Gọi q là công bội của một cấp số nhân , biết tổng ba số hạng đầu bằng 16 , đồng
9
thời theo thứ tự , chúng là số hạng thứ nhất , thứ tư và thứ tám của một cấp số cộng . Hỏi
q thuộc khoảng nào sau đây?

A. q   3; 4  B. q   1; 2  C. q   2; 3  D. q   0; 1 

I n 2
Câu 81. Cho I n   2 sin n xdx với n nguyên dương. Tính lim .
0 In
A. 1 B. 1 C. 2 D. 
1
I n1
Câu 82. Với mỗi số nguyên dương n ta kí hiệu I n   x 2  1  x 2  dx . Tính lim
n
.
n  I
0 n

A. 1 B. 2 C. 3 D. 5

Câu 83. Đặt I n   n 1 


  x  12 n 2 x 2  1
 n
2x2  1 

I
dx. Tính lim n .
0 
 x  1  x  1 
2 n 1 n 1
 2 2 I n1

1 3
A. 1 B. C. 1 D.
2 2
n
x  xn
Câu 84. Ta đặt Fn  x    dx . Biết Fn  1   0 n . Tính lim Fn  2  .
x n1 n 

A. 1 B.  C. 1 D. 
1
e  nx
Câu 85. Cho tích phân I n   dx với n  .
0
1  e x
Đặt un  1.  I 1  I 2   2  I 2  I 3   3  I 3  I 4   ...  n  In  In 1   n . Biết lim un  L . Mệnh đề nào
sau đây là đúng?
A. L   1; 0  B. L   2; 1  C. L   0; 1  D. L   1; 2 
Câu 86. Có bao nhiêu giá trị nguyên dương n thỏa mãn tích phân
2

 1  n  2 x  3x 2  4x 3  ...  nx n1  dx  2
2

A. 1 B. 2 C. 0 D. 3

Tinh hoa của toán học nằm ở tự do của nó – Georg Cantor Chinh phục olympic toán | 23
CÁC BÀI TOÁN VẬN DỤNG CAO DÃY SỐ

Câu 87. Cho hàm số f  x  có đạo hàm liên tục trên đoạn  0; 1 thỏa mãn điều kiện
1
f  2018x  2017   2018 f  x  ,x   f  x   dx ?
2
. Tính tích phân 0 

4 5 7 8
 f  1    f  1    f  1    f  1  
2 2 2 2
A. B. C. D.
3 3 3 3
Câu 88. Cho I n   tan n xdx với n  . Khi đó I 0  I 1  2  I 2  I 3  ...  I 8   I 9  I 10 bằng?

 tan x   tan x   tan x   tan x 


9 r 9 r 1 10 r 10 r 1

A. 
r 1 r
C B. 
r 1 r 1
C C. 
r 1 r
C D. 
r 1 r 1
C

 6 2
U  U
Câu 89. Cho dãy số xác định bởi  1 4 , n  1, n  N * . S= lim n có giá trị là ?
U  2.U 2  1 n
 n1 n

1 1
A. 1 B. C. 0 D.
2 4
 1
U 1  2
Câu 90. Cho dãy số Un xác định bởi  ,n  1
U  U n  n U n  1   n
2 2

 n  1 n
 1 1 1 1 
Khi đó S  lim      thuộc khoảng nào sau đây?
 U1 U 2 U 3 Un 
A.  3; 1  B.  1; 2  C.  1; 2  D.  1; 1 
Câu 91. Trong dịp hội trại hè 2017, bạn Anh thả một quả bóng cao su từ độ cao 6  m  so
với mặt đất, mỗi lần chạm đất quả bóng lại nảy lên một độ cao bằng ba phần tư độ cao lần
rơi trước. Biết rằng quả bóng luôn chuyển động vuông góc với mặt đất. Tổng quãng
đường quả bóng đã bay (từ lúc thả bóng cho đến lúc bóng không nảy nữa) khoảng ?
A. 44  m  B. 45  m  C. 42  m  D. 43  m 
Câu 92. Có hai cơ sở khoan giếng A và B. Cơ sở A giá mét khoan đầu tiên là 8000 (đồng)
và kể từ mét khoan thứ hai, giá của mỗi mét sau tăng thêm 500 (đồng) so với giá của mét
khoan ngay trước đó. Cơ sở B: Giá của mét khoan đầu tiên là 6000 (đồng) và kể từ mét
khoan thứ hai, giá của mỗi mét khoan sau tăng thêm 7% giá của mét khoan ngay trước
đó. Một công ty giống cây trồng muốn thuê khoan hai giếng với độ sâu lần lượt là 20  m 
và 25  m  để phục vụ sản xuất. Giả thiết chất lượng và thời gian khoan giếng của hai cơ
sở là như nhau. Công ty ấy nên chọn cơ sở nào để tiết kiệm chi phí nhất?
A. luôn chọn A.
B. luôn chọn B.
C. giếng 20  m  chọn A còn giếng 25  m  chọn B.
D. giếng 20  m  chọn B còn giếng 25  m  chọn B.

24 | Chinh phục olympic toán Điều ta biết là giọt nước, điều ta chưa biết là đại dương - Newton
TUYỂN TẬP MỘT SỐ NHÓM CÂU HỎI VẬN DỤNG CAO MÔN TOÁN

Câu 93. Cho cấp số cộng  un  có các số hạng đều dương, số hạng đầu u1  1 và tổng của
100 số hạng đầu tiên bằng 14950 . Tính giá trị của tổng sau?
1 1 1
S   ... 
u2 u1  u1 u2 u3 u2  u2 u3 u2018 u2017  u2017 u2018
1 1  1
A.  1  B. 1  C. 2018 D. 1
3 6052  6052
Câu 94. Giá trị của tổng 4  44  444  ...  44...4 (tổng đó có 2018 số hạng) bằng?
40
A.
9
 10 2018  1   2018 . B. 
4  10 2019  10 
 2018  .
9 9 
4
C.
4  10 2019  10


 2018  . D.
9
 10 2018  1  .
9 9 
Câu 95. Cho dãy số  un  thỏa mãn un  un 1  6 , n  2 và log 2 u5  log 2
u9  8  11 . Đặt
Sn  u1  u2  ...  un . Tìm số tự nhiên n nhỏ nhất thỏa mãn Sn  20172018 .
A. 2587 B. 2590 C. 2593 D. 2584
Câu 96. Cho hai cấp số cộng  an  : a1  4 ; a2  7 ;...; a100 và  bn  : b1  1 ; b2  6 ;...; b100 . Hỏi có
bao nhiêu số có mặt đồng thời trong cả hai dãy số trên?
A. 32 B. 20 C. 33 D. 53
Câu 97. Cho tam giác ABC cân tại A . Biết rằng độ dài cạnh BC , trung tuyến AM và độ
dài cạnh AB theo thứ tự đó lập thành một cấp số nhân có công bội q . Tìm công bội q của
cấp số nhân đó?
1 2 22 2 1  2 2  2 2
A. q  B. q  C. q  D. q 
2 2 2 2
Câu 98. Cho hàm số f  x    x 2  3x  2 
cos 2017 x 
và dãy số  un  được xác định bởi công thức
tổng quát un  log f  1   log f  2   log f  n  Tìm tổng tất cả các giá trị của n thỏa mãn
điều kiện un2018  1
A. 21 B. 18 C. 3 D. 2018
un  u4 n  u42 n  u42018 n a 2019  b
Câu 99. Biết rằng L  lim  Trong đó  un  xác định
un  u2 n  u2 2 n  u22018 n c

bởi u1  0; un 1  un  4n  3 và a b c , , là các số nguyên dương và b  2019 . Tính S  a  b  c


A. 1 B. 0 C. 2017 D. 2018
Câu 100. Cho ba số dương a , b , c theo thứ tự lập thành cấp số cộng. Giá trị lớn nhất của
a 2  8bc  3
biểu thức P  có dạng x y  x , y   Hỏi x  y bằng bao nhiêu?
 2a  c 
2
1
A. 9 B. 11 C. 13 D. 7

Tinh hoa của toán học nằm ở tự do của nó – Georg Cantor Chinh phục olympic toán | 25
CÁC BÀI TOÁN VẬN DỤNG CAO DÃY SỐ

Câu 101. Cho các số hạng dương a, b, c là số hạng thứ m, n, p của một cấp số cộng và một
cấp số nhân. Tính giá trị của biểu thức log 2 ab c  bc  a  c a b
A. 0 B. 2 C. 1 D. 4

Câu 102. Cho a  b  c  và cot a , cot b , cot c Tạo thành cấp số cộng. Giá trị của cot a.cot c
2
bằng?
A. 1 B. 2 C. 3 D. 4

26 | Chinh phục olympic toán Điều ta biết là giọt nước, điều ta chưa biết là đại dương - Newton
TUYỂN TẬP MỘT SỐ NHÓM CÂU HỎI VẬN DỤNG CAO MÔN TOÁN

B. LỜI GIẢI.

Câu 1. Cho hàm số y  x 3  2009 x có đồ thị là C  . M1 là điểm trên C  có hoành độ


x1  1 . Tiếp tuyến của C  tại M1 cắt C  tại điểm M 2 khác M1 , tiếp tuyến của C  tại
M 2 cắt C  tại điểm M 3 khác M 2 , <, tiếp tuyến của C  tại M n 1 cắt C  tại Mn khác
M n 1  n  4; 5;... , gọi  xn ; y n  là tọa độ điểm Mn . Tìm n để: 2009xn  yn  2 2013  0 .
A. n  685 B. n  679 C. n  672 D. n  675
Lời giải
Phương trình hoành độ giao điểm của C  và tiếp tuyến là
x 3  2009x   3x1 2  2009   x  x1   x1 3  2009x1  1  .
Phương trình  1  có một nghiệm kép x1  1 và một nghiệm x 2 .
Ta có  1   x 3  3x  2  0 .
 2 x1  x 2  0

Áp dụng định lí Viét cho phương trình bậc ba, ta có x1 2  2 x1 x2  3  x2  2 x1 .
x 2 .x  2
 1 2
Suy ra x1  1 , x 2  2 , x3  4 , <, xn   2 
n1
.
Ta có: 2009xn  yn  2 2013  0  2009xn  xn 3  2009xn  2 2013  0
  2 
3n3
 2 2013  3n  3  2013  n  672 .
Chọn ý C.
Câu 2. Một hình vuông ABCD có cạnh AB  250 , diện tích S1 . Nối 4 trung điểm A1 , B1 ,
C 1 , D1 theo thứ tự của 4 cạnh AB , BC , CD , DA ta được hình vuông thứ hai là A1 B1C 1D1
có diện tích S 2 . Tiếp tục như thế ta được hình vuông thứ ba A2 B2C 2 D2 có diện tích S 3 và cứ
tiếp tục như thế, ta được diện tích S4 , S5 ,... Tính S  S1  S2  S3  ...  S100
A. S  2 101  2 B. S  2 101  1 C. S  2 100  2. D. S  2 100  1
Lời giải
Dễ thấy S1  2 100
; S2  2 ; S3  2 ;  ; S100  2
99 98

1
Như vậy S1 , S2 , S3 ,..., S100 là cấp số nhân với công bội q  .
2
 1 
2 100.  1  100 
 2 
Khi đó ta có S  S1  S2  ...  S100  2 100  2 99  2 98  ...  2   2 101  2
1
1
2
Chọn ý B.

Tinh hoa của toán học nằm ở tự do của nó – Georg Cantor Chinh phục olympic toán | 27
CÁC BÀI TOÁN VẬN DỤNG CAO DÃY SỐ

Câu 3. Khối tứ diện ABCD có thể tích V , khối tứ diện A1 B1C 1D1 có thể tích V1 , các đỉnh
A1 , B1 , C 1 , D1 lần lượt là trọng tâm các tam giác BCD , CDA , DAB , ABC . Khối tứ diện
A2 B2C 2 D2 có thể tích V2 , các đỉnh A2 , B2 , C 2 , D2 lần lượt là trọng tâm các tam giác
B1C 1D1 , C 1 D1 A1 , D1 A1 B1 , A1 B1C 1 . Cứ tiếp tục như thế ta được khối tứ diện An BnC n Dn có
thể tích Vn , các đỉnh An , Bn , C n , Dn lần lượt là trọng tâm các tam giác Bn1C n1Dn1 ,
C n 1Dn 1 An 1 , Dn 1 An 1 Bn 1 , An 1 Bn 1C n 1 . Tính S  V1  V2  ...  V2018 .

A. S 
3 2018
 1V
B. S 
 27 2019
 1V
C. S 
 27 2018
 1V
D. S 
3 2019
 1V
2.32018 26.27 2019 26.27 2018 2.32019
Lời giải
A

C1

D1 B1
B D

A1

C
1
Ta có  B1C 1D1  //  BCD  nên d  A1 ,  B1C1D1    d  D1 ,  BCD    d  A,  BCD   .
3
1 1
Lại có B1C 1D1 BCD với tỉ số đồng dạng k  nên SB1C1D1  SBCD .
3 9
1 1 1 1 1 1
Do đó V1  V . Tương tự ta có V2  V1  2 V , V3  V2  3 V , <, V2018  2018 V .
27 27 27 27 27 27
1
 1 1 1 
 S    2  ...  2018  V 
1
.
1  2018
27 V 
 27 2018  1  V
.
 27 27 27  27 1  1 26.27 2018
27
Câu 4. Tam giác mà ba đỉnh của nó là ba trung điểm ba cạnh của tam giác ABC được gọi
là tam giác trung bình của tam giác ABC .Ta xây dựng dãy các tam giác
A1 B1C 1 , A2 B2C 2 , A3 B3C 3 ,... sao cho A1 B1C 1 là một tam giác đều cạnh bằng 3 và với mỗi số
nguyên dương n  2 , tam giác An BnC n là tam giác trung bình của tam giác An 1 Bn 1C n 1 .
Với mỗi số nguyên dương n , kí hiệu Sn tương ứng là diện tích hình tròn ngoại tiếp tam
giác An BnC n . Tính tổng S  S1  S2  ...  Sn  ... ?
15 9
A. S  . B. S  4 . C. S  . D. S  5.
4 2
Lời giải
28 | Chinh phục olympic toán Điều ta biết là giọt nước, điều ta chưa biết là đại dương - Newton
TUYỂN TẬP MỘT SỐ NHÓM CÂU HỎI VẬN DỤNG CAO MÔN TOÁN

Vì dãy các tam giác A1 B1C 1 , A2 B2C 2 , A3 B3C 3 ,... là các tam giác đều nên bán kính đường
3
tròn ngoại tiếp các tam giác bằng cạnh .
3
Với n  1 thì tam giác đều A1 B1C 1 có cạnh bằng 3 nên đường tròn ngoại tiếp tam giác
2
3  3
A1 B1C 1 có bán kính R1  3.  S1    3.  .
3  3 
3
Với n  2 thì tam giác đều A2 B2C 2 có cạnh bằng nên đường tròn ngoại tiếp tam giác
2
2
1 3  1 3
A2 B2C 2 có bán kính R2  3. .  S2    3. .  .
2 3  2 3 
3
Với n  3 thì tam giác đều A3 B3C 3 có cạnh bằng nên đường tròn ngoại tiếp tam giác
4
2
1 3  1 3
A2 B2C 2 có bán kính R3  3. .  S3    3. .  .
4 3  4 3 
n1
1
Như vậy tam giác đều An BnC n có cạnh bằng 3.   nên đường tròn ngoại tiếp tam giác
2
2
1
n1
3   1 n1 3 
An BnC n có bán kính Rn  3.   .  Sn    3.   .  .
2 3  2 3
 
Khi đó ta được dãy S1 , S 2 , ...Sn ... là một cấp số nhân lùi vô hạn với số hạng đầu
1
u1  S1  3 và công bội q  .
4
u1
Do đó tổng S  S1  S2  ...  Sn  ...   4 .
1q
 
Câu 5. Cho dãy số  un  có số hạng tổng quát un  cos  2n  1   . Tổng 2018 số hạng đầu
 6
tiên của dãy số  un  bằng bao nhiêu?
3 3 1
A. 0 B.  C. D.
2 2 2
Lời giải
      
Ta có un6  cos  2n  11   cos  2n  1  2   cos  2n  1   un , n  *
.
 6  6   6
u1  u7  u13  ...  u2011  u2017
u  u8  u14  ...  u2012  u2018
 2
u3  u9  u15  ...  u2013

u4  u10  u16  ...  u2014
 u5  u11  u17  ...  u2015

u6  u12  u18  ...  u2016

Tinh hoa của toán học nằm ở tự do của nó – Georg Cantor Chinh phục olympic toán | 29
CÁC BÀI TOÁN VẬN DỤNG CAO DÃY SỐ

 u1  u2  ...  u6  u7  u8  ...  u12  ...  u2011  u2012  ...  u2016 .


 S2018   u1  u2  ...  u6    u7  u8  ...  u12   ...   u2011  u2012  ...  u2016    u2017  u2018 
 336.  u1  u2  ...  u6    u1  u2 
 3  3  3 3  3  3
 336.   0          0      0   .
 2  2   2  2   2  2

u1  3

Câu 6. Cho dãy số  un  thỏa mãn  un  2  1 , n  *
.
un1  1  2  1 u
  n 
Khi đó u2019  a  b 3 , a , b  . Tính tổng S  a  b .
A. S  3 B. S  4 C. S  9 D. S  2
Lời giải

2 tan
 8  tan 2   2 tan   1  0  tan   2  1 vì tan   0
Ta có 1  tan 
4 1  tan 2  8 8 8 8
8
 
tan  tan
u1  2  1 3 8  tan     
Do đó u2    
 
1  2  1 u1 1  tan  tan 
3 8
3 8

  
tan     tan
u2  2  1 3 8 8  
u3    tan   2. 
 
1  2  1 u2 1  tan      tan 
 
3 8
3 8 8
 
Bằng phương pháp quy nạp ta chứng minh được un  tan    n  1   , n  *

3 8
 
Do đó u2019  tan   2018.   2  3  S  3 .
3 8
Câu 7. Cho tam giác ABC có độ dài các cạnh là a , b , c theo thứ tự lập thành một cấp số
A C x x
cộng. Biết tan tan  với x , y  và tối giản. Tính giá trị của x  y .
2 2 y y
A. 4 B. 1 C. 2 D. 3
Lời giải
Theo giả thiết ta có
AC A C B B
a  c  2b  sin A  sin C  2 sin B  2 sin .cos  4 sin .cos
2 2 2 2
AC A C A C A C
 2 sin .cos  4 sin .cos
2 2 2 2
A C AC A C A C A c A C
 cos  2 cos  cos cos  sin sin  2 cos cos  2 sin sin
2 2 2 2 2 2 2 2 2 2

30 | Chinh phục olympic toán Điều ta biết là giọt nước, điều ta chưa biết là đại dương - Newton
TUYỂN TẬP MỘT SỐ NHÓM CÂU HỎI VẬN DỤNG CAO MÔN TOÁN

A C A C A C A C 1
 3 sin sin  cos cos  3 tan tan  1  tan tan 
2 2 2 2 2 2 2 2 3.
Do đó x  y  1  3  4 .
 u1  11
Câu 8. Cho dãy số  un  xác định  . Tính giá trị của u2018 ?
un 1  10un  1  9n, n  1
A. u2018  10 2018 B. u2018  20182018 C. u2018  2018 D. u2018  10 2018  2018
Lời giải
Cách 1.
u1  11  10  1

Ta nhận thấy u2  10u1  1  9  10.11  8  102  10 2  2
u  10u  1  18  10.102  17  1003  10 3  3
 3 2

Nên dự đoán un  10 n  n
Chứng minh bằng quy nạp. Ta có u1  11  10  1 . Giả sử đúng với n  k  1  uk  10 k  k
khi đó uk  1  10.uk  1  9.k  10.  10 k  k   1  9 k  10 k  1  k  1 .
Vậy un  10n  n nên u2018  10 2018  2018 .
Cách 2. Ta có un1  10un  1  9n  un1   n  1   10  un  n 
Đặt vn  un  n  vn 1  un 1   n  1  và v1  u1  1  10
 v  10
Ta có dãy số  1 ,  vn  là một cấp số nhân có công bội q  10 và v1  10 .
 vn1  10 vn , n  1
Ta có công thức tổng quát vn  v1q n1  vn  10.10 n1  10 n  un  n  10 n  un  10 n  n
Do đó u2018  10 2018  2018
1 un u u u u
Câu 9. Cho dãy số (un ) thỏa mãn u1  ; un1  , n  1 . Đặt Sn  1  2  3  ...  n .
2 un  1 1 2 3 n
2019
Tìm giá trị nhỏ nhất của n để Sn  ?
2020
A. 2019 B. 2020 C. 2018 D. 2021
Lời giải
Từ hệ thức truy hồi ta có un  0, n  1 .
un 1 1  1  1
Ta có un1     1 . Do đó   là cấp số cộng có  2 và công sai d  1 ,
un  1 un1 un u
 n u1

1
Từ đó suy ra  2   n  1   n  1 , n  1 .
un
1 u 1 1 1
Do đó un  , n  1  n    .
n1 n n  n  1 n n  1
1 1 1 1 1 1 1 1 1
Ta có Sn        ...    1 .
1 2 2 3 3 4 n n1 n1

Tinh hoa của toán học nằm ở tự do của nó – Georg Cantor Chinh phục olympic toán | 31
CÁC BÀI TOÁN VẬN DỤNG CAO DÃY SỐ

2019 1 2019
Khi đó Sn   1   n  2019 . Do đó n  2020 .
2020 n  1 2020
u0  2 2018

Câu 10. Cho dãy  un  :  2 un  1 . Tìm phần nguyên của S   ui .
un  1  u  2 i 1
 n

A. 2020 B. 2017
C. 2019 D. 2018 .
Lời giải
un  1 1 3
Ta có un1  1    1 .
un  2 un 1  1 un  1
1 3n  1  1 2
Đặt an   a0  1 và an  1  3 an  1  an   un  1  n1 .
un  1 2 3 1
2018 2018
1 2 2018 1
 S   ui  2018  2  i 1  2018  S  2018   i  2019
i 1 i 1 3 1 3 i 1 3
Phần nguyên của S . bằng 2018 .
u1  2019

Câu 11. Cho dãy số  un  được xác định bởi:  2019 .
un   n  u1  u2  u3  ...  un1  , n  1

Tính giá trị của biểu thức A  2.u1  2 2 u2  ...  2 2019.u2019 .


A. 32019 B. 2019 C. 3 D. 2
Lời giải

Ta có đẳng thức sau


1
Cnk 
1
.
n!

1
.
 n  1 ! 
1
.Cnk11
k1 k  1 k !.  n  k  ! n  1  k  1 !  n  1   k  1   ! n  1
1 1 k 1 k 1
Suy ra S  k
C 2018 .u1  C 2019 .2019  C 2019
k1 2019
Từ giả thiết ta có nun  2019  u1  u2  u3  ...  un2   2019un1
n  2020
  n  1  un1  2019un1   n  2020  un1  un  un1 .
n
 2018 1 1 1 1
u2   2 u1   2 C 2018 u1   2 C 2018 .2019  C 2019
2


u  2017.2018 u  1 C 2 .u  C 3
 2 2.3
1
3
2018 1 2019

 2016.2017.2018 1 3
 u3     C 2018 u1  C 2019
4

 2.3.4 4
 ...

u2019  1 C 20182018
u1  C 2019
2019

 2019

  2  1
2019
 S  2.C 2019
1
 2 2 C 2019
2
 2 3 C 2019
3
 2 4 C 2019
4
 ...  2 2019 C 2019
2019
1 2.

32 | Chinh phục olympic toán Điều ta biết là giọt nước, điều ta chưa biết là đại dương - Newton
TUYỂN TẬP MỘT SỐ NHÓM CÂU HỎI VẬN DỤNG CAO MÔN TOÁN

 x1  2

Câu 12. Cho dãy số  xn  xác định bởi  xn 1 2 n  1  3x n *
 x  1  n 2  3x , n 
 n n

20182 8144648 8144648 8144648


A. B. C. D.
2019 12105 12107 12103
Lời giải
xn
 n  1  xn
2
xn  1 2 n  1  3xn xn  1 xn 2
Ta có 1  xn  1  2   2  n
n  3x n n  3x n  n  1 n  3xn 1  3 xn2
2 2
xn
n
x yn 1 1
Đặt dãy số yn  n2 , n  *  yn1     3 và y 1  2
n 1  3yn yn1 yn
1
Đặt un  , n  *  un1  un  3
yn
1
Suy ra  un  là cấp số cộng với u1  và công sai d  3
2
2n2 8144648
5  xn  6n  5 , n   x2018 
*
1
 un    n  1 3  3n  12103
2 2
Câu 13. Cho dãy số  un  thỏa mãn u1  1, un1  aun2  1, n  1 , a  1 . Biết rằng
lim  u12  u22  ...  un2  2n   b . Giá trị của biểu thức T  ab ?
A. 1 B. 2 C. 1 D. 2
Lời giải
1  1 
Theo giả thiết ta có un21  aun2  1  un21   a  un2  
1 a  1 a 
1
Đặt vn  un2   vn1  avn   vn  là cấp số nhân với công bội q  a
1 a
 1  n1 a a 1
Suy ra vn  v1 an1   u12  n1
a  a .  un2  an1 . 
 1 a  a1 a1 1 a
 2 a 1
u1  a  1  1  a

u 2  a. a  1 a 1
Ta có  2 a  1 1  a  u1  u2  ...  un 
2 2 2

a1
 1  a  ...  an 1  
1 a
.n
.............................

u 2  a n  1 . a  1

n
a1 1 a
1 a 1  an
 u12  u22  ...  un2  .n  .
1 a a1 1 a
Thực hiện phép đồng nhất ta được

Tinh hoa của toán học nằm ở tự do của nó – Georg Cantor Chinh phục olympic toán | 33
CÁC BÀI TOÁN VẬN DỤNG CAO DÃY SỐ

 1
 a
 1 2

1  a  2    1 n 

   1      T  1
 a 1  a n
  2 
b  lim  .  b  lim  1   2

  a1 1 a  
  2 1 
  
2 un
Câu 14. Cho dãy số (un ) được xác định bởi u1  và un1  , n  *
 . Tính
3 2  2n  1  un  1
tổng 2019 số hạng đầu tiên của dãy số đó ?
4036 4035 4038 4038
A. B. C. D.
4035 4034 4037 4039
Lời giải
Theo giả thiết ta có
1 1  1  1
  4n  2    4  n  1  2   4n  2    4.1  2    4.2  2   ...   4n  2 
un1 un  un1  u1
3 4n2  8n  3 2 2
  2 n 2  4n   un1  2 
2 2 4n  8n  3  2n  1 2n  3 
2 1 1
 un   
 2n  1 2n  1 2n  1 2n  1
 1 1 1  1 1  1
Từ đó suy ra Sn  u1  u2  ....  un   1        ...      1
 3 3 5  2n  1 2n  1  2n  1
1 4038
 S2019  1   .
2.2019  1 4039

u1  1
Câu 15. Cho dãy số  un  xác định như sau:  , với n  1, 2, 3,...
un1  un  2018un  un
2020 2019

 u12019 u22019 u32019 un2019 


Tính lim     ...  .
 u2  2018 u3  2018 u4  2018 un1  2018 
4 3 2 1
A. . B. . C. . D. .
2019 2019 2019 2019
Lời giải
Ta dễ ràng thấy rằng un  1 với mọi n  1, 2, 3,...
Xét un 1  un  un2020  2018un2019  0 với mọi n  1, 2, 3,... , nên dãy  un  tăng.
Giả sử dãy  un  bị chặn trên, khi đó  un  có giới hạn. Giả sử lim un  a  1 .
Từ hệ thức un  1  un2020  2018un2019  un chuyển qua giới hạn có
a  a 2020  2018a 2019  a  a  0  a  2018 - Điều này vô lý
Vậy, dãy  un  không bị chặn trên. Suy ra lim un   .
Ta có

34 | Chinh phục olympic toán Điều ta biết là giọt nước, điều ta chưa biết là đại dương - Newton
TUYỂN TẬP MỘT SỐ NHÓM CÂU HỎI VẬN DỤNG CAO MÔN TOÁN

uk2019 uk2019  uk  2018  uk 1  uk 1 1


    .
uk 1  2018  uk 1  2018  uk  2018   uk 1  2018  uk  2018  uk  2018 uk 1  2018
u12019 u22019 u32019 un2019 1 1
    ...   
u2  2018 u3  2018 u4  2018 un1  2018 u1  2018 un1  2018
 u12019 u22019 u32019 un2019   1 1 
Vậy lim     ...    lim   
 u2  2018 u3  2018 u4  2018 un1  2018   u1  2018 un1  2018 
1 1
  .
u1  2018 2019

Câu 16. Xét dãy số nguyên x1  34, x2  334, x3  3334, , xn  33...34 (có n số 3). Hỏi có bao
3
nhiêu chữ số 3 trong số 9x2018 ?
A. 6054 B. 6055 C. 6056 D. 6057
Lời giải
Ta đặt un  3xn  2 . Khi đó un  10n 1

 10  2   103n3  1  2.102n2  4.10n1  3


n1 3
10n1  2
 xn   9xn3 
3 3 3
Lại có 10 3 n 3  1   10  1   10 3 n 2  10 3 n1   10  1 

103 n 3  1
  3  103n 2  103 n1   10  1
3
 9xn3  3  10 3 n 2  10 3 n 1   10  1   2.10 2 n 2  4.10 n 1  3
Để ý rằng 10 3 n 2  10 3 n 1   10  1  111...111 (có 3n +2 số 1)
2 n 2
2.10  2000...00 (có 2n +2 số 0) và 4.10 n  1  400...00 (có n+1 số 0)
 9 xn3  33...33533...33733...336
(trước 5 có n số 3, giữa 5 và 7 có n số 3, giữa 7 và 6 có n số 3)
 9 x có 3n số 3.
3
n

un 1
Câu 17. Cho dãy số  un  xác định bởi u1  1 và un1   với n nguyên dương.
2018 2019n1
Tính giới hạn A  lim un
x 

2019 2018
A. B. 2018 C. D. 0
2018 2019
Lời giải
n1
un 1  2018 
Do un1   n1
 2018n1 un1  2018n un    .
2018 2019  2019 
n1
 2018 
Đặt vn  2018 un ta được v1  2018 và vn 1  vn  
n
 với n nguyên dương.
 2019 
Suy ra vn  ( vn  vn 1 )  ( vn 1  vn  2 )   ( v2  v1 )  v1

Tinh hoa của toán học nằm ở tự do của nó – Georg Cantor Chinh phục olympic toán | 35
CÁC BÀI TOÁN VẬN DỤNG CAO DÃY SỐ

  2018  
n

n k  1    
 2018  2018   2019    2018
    2018 
k  1  2019  2019  1  2018 
 2019 

  2018  
n
vn 2018   2018  
n
1   2018  
n

 2018  2      un   2    n1 
2   
  2019   2018n 2018n   2019   2018   2019  

 1   2018   
n

Vì un  0, n  mà lim un  lim 
*
 2       0  xlim un  0
x   2018 n  1  2019
x 
    


u 1
Câu 18. Cho dãy số (u n ) xác định bởi u1  1 và un1  n  với n nguyên dương.
2018 2019n1
Tính giới hạn A  lim  u1  u2  un 
x 

2018 2017 2017 2019


A. B. C. D.
2019 2019 2018 2017
Lời giải
n1
u 1  2018 
Do un1  n  n1
 2018n1 un1  2018n un    .
2018 2019  2019 
n1
 2018 
Đặt vn  2018 un ta được v1  2018 và vn 1
n
 vn    với n nguyên dương.
 2019 
Suy ra vn   vn  vn1    vn1  vn 2     v 2  v 1   v1
  2018  
n

n k  1    
 2018  2018   2019    2018
    2018 
k  1  2019  2019  1  2018 
 2019 

  2018  
n
vn 2018   2018  
n
1   2018  
n

 2018  2      un   n 
2     n1 
2
  2019  2018 n
2018   2019  2018   2019  
     
1   2018  
n n k n n
1 1
Do đó  uk   k 1 
2     4036  2018
k  1 2018
  2019   k  1 2018
k
k  1 2019
k
k 1 
Áp dụng công thức tính tổng cấp số nhân lùi vô hạn ta được
1 1
4036 2019
A  lim  u1  u2  un   4036  2018  2018  2019  1
x  1 1 2017 2017
1 1
2018 2019

36 | Chinh phục olympic toán Điều ta biết là giọt nước, điều ta chưa biết là đại dương - Newton
TUYỂN TẬP MỘT SỐ NHÓM CÂU HỎI VẬN DỤNG CAO MÔN TOÁN

x1  1 n
1
Câu 19. Cho dãy số ( xn ) có  ; n  * . Đặt yn   .
x
 n  1  x n  x n  1  x n  2  x n  3   1 i  1 x i  2
a a
Biết lim yn  với là phân số tối giản và a, b nguyên dương. Khi đó tọa độ M  a ; b 
b b
nằm trên đường tròn nào?
A.  x  1    y  2   4 B.  x  1    y  1   4
2 2 2 2

C.  x  1    y  1   10 D.  x  1   y 2  10
2 2 2

Lời giải

x  3xn  xn2  3xn  2   1  x  3xn  1  xn2  3xn  1


2
Từ giả thiết xn1  2
n
2
n

Lại có xn 1  xn  xn 2  2 xn  1   xn  1   0; n 
2 *
.
Suy ra  xn  là một dãy số tăng. Giả sử  xn  là dãy bị chặn trên
 lim xn  a  a 2  3a  1  a  a  1 . Vô lý. Vậy lim xn   .
Mặt khác xn 1  1  xn 2  3xn  2  xn 1  1   xn  1  xn  2 
1 1 1 1 1 1 1 1
        .
 xn  2  xn  1 xn  1  1 xn  1 xn  2 xn  1  1 xn  2 xn  1 xn  1  1
1 1 1
Vậy yn    lim yn   M  2; 1 .
2 xn  1  1 2
 3
u1 
Câu 20. Cho dãy số  un  xác định bởi  16
u  9u  4 1  3u  4, n  
 n1 n n

Tìm số nguyên dương n nhỏ nhất thỏa mãn un  10 8.


A. 9. B. 10. C. 12. D. 13.
Lời giải

Đặt dãy số xn  1  3un , n 

 xn2  1
Ta có xn  0 và xn2  1  3un , n   un 
3
xn21  1 x2  1
 4xn  4  xn2 1   3xn  2 
2
Thay vào giả thiết ta được 9 n
3 3
Suy ra xn  1  3xn  2 n  . Giả sử xn 1    3  xn    thì   1.

9
Xét dãy  yn  xác định bởi y n  xn  1 . Khi đó  yn  là cấp số nhân với y1  x1  1  , công
4
2
 9 n1 
 .3  1   1
9 9 4
bội q  3  y n  .3n1  xn  .3n1  1  un   
4 4 3
2
9  9
Có un  108   .3n1  1   3.108  1  .3n1  1  3.108  1
4  4

Tinh hoa của toán học nằm ở tự do của nó – Georg Cantor Chinh phục olympic toán | 37
CÁC BÀI TOÁN VẬN DỤNG CAO DÃY SỐ


 n  log 3 

 4 
3.108  1  1 .    1  n  9, 14.
9 
Vậy n nhỏ nhất bằng 10
Câu 21. Xét các cấp số nhân có 2 n  1 số hạng dương ( n là số nguyên dương) thỏa tổng tất
cả các số hạng của nó bằng 400 và tổng tất cả các nghịch đảo của các số hạng của nó bằng
4 . Giá trị lớn nhất của n là?
A. 17 B. 18 C. 19 D. 20
Lời giải
a a a a
Đặt các số hạng của cấp số nhân là n , n1 , n2 ,..., , a , aq ,..., aq n1 , aq n với a , q là các
q q q q
số dương.
a a n1
  1 1 n1 n

 q n q n1  ...  a  ...  aq  aq n
 400  a    ...  1  ...  q  q   400
  q
n
q n1 
Ta có  n n1

 q  q  ...  1  ...  1  1  4  1  q n  q n1  ...  1  ...  1  1   4
 a a a aq n1
aq n
  q n1 q n 

a 
  1 1 n 1 1 1
 n  q  n1  q  ...   q  1  40  * 
n1 n1
 a  n  n1  ...  1  ...  q  q   400
n

 q q   q q q
 2  a  10
 a  100 
Muốn tồn tại cấp số nhân thì điều kiện cần và đủ là phương trình  *  phải có nghiệm
1 1 1
dương. Xét hàm số f  x   n
 x n  n1  x n1  ...   x  1 liên tục trên D   0;   .
x x x
Theo bất đẳng thức AM – GM ta có
1 1 1
f  x   n  x n  n1  x n 1  ...   x  1  2  2  ...  2  1  2 n  1
x x x

Dấu bằng xảy ra khi x  1 . Mặt khác lim f  x    , lim f  x   


x 0  x 

Suy ra tập giá trị của hàm số f trên D là  2n  1;   .


Phương trình  *  có nghiệm dương khi và chỉ khi 40  2 n  1  n  19, 5 .
Vậy giá trị lớn nhất của n là 19 .
u0  2018
 u
Câu 22. Cho dãy số (un ) được xác định bởi u1  2019 . Hãy tính lim nn .
u  4u  3u ; n  1 3
 n1 n n 1

1 1
A. B. C. 32018 D. 32019
2 3
Lời giải
Ta có un1  4un  3un 1  un 1  un  3  un  un 1  .

38 | Chinh phục olympic toán Điều ta biết là giọt nước, điều ta chưa biết là đại dương - Newton
TUYỂN TẬP MỘT SỐ NHÓM CÂU HỎI VẬN DỤNG CAO MÔN TOÁN

v  1
Đặt vn  un  un 1 ta có  1 . Suy ra vn  3n1 ; n  1.
v
 n  3 vn1 ; n  2
Ta được un   un  un 1    un 1  un 2     u2  u1   u1
3n  1 3n  1
 3n  1  3n  2   3  2019   2018   2018
31 2
un  3n  1 2018  1
Suy ra lim  lim   n  .
3n  2.3 n
3  2
n3
Câu 23. Cho dãy số  un  xác định bởi u1  1; un1  2un  , n  *
. Hỏi u2018
n  3n  2
2

thuộc khoảng nào sau đây?


A.  2 2015 ; 2 2016  B.  2 2016 ; 2 2017  C.  2 2017 ; 2 2018  D.  2 2018 ; 2 2019 
Lời giải
n3 1 2 1  1 
Ta có un1  2un   un1  2un    un1   2  un  
 n  1 n  2  n2 n1 n2  n1
 1 1
1  v1  u1   
Đặt vn  un  , n  * , suy ra  2 2
n1  vn  1  2 vn , n  *
1
Do đó dãy số  vn  là cấp số nhân có công bội q  2 và v1   .
2
 1 1
Suy ra vn     .2 n1  2 n2 , n  *  un   2 n2 , n  * .
 2  n  1
1
Vậy u2018   2 2016   2 2016 ; 2 2017  .
2019
 2
u1  3 u u u 
Câu 24. Cho dãy số  un  xác định  . Tính L  lim  1  22   nn 
u  2nun ; n  *  2 2 2 
n 

 n1
n3
1 3 3
A. L  B. L  C. L  1 D. L 
2 4 2
Lời giải
2nun
Ta có un1    n  1 n  2  n  3  un1  2n  n  1  n  2  un ; n  *
n3
Đặt vn  n  n  1  n  2  un ta được dãy  vn  thỏa mãn v1  4; vn 1  2 vn ; n  * nên dãy
2 n1
 vn  là một cấp số nhân có công thức vn  4.2 n1
2 n1
. Vậy un 
n  n  1 n  2 
un 2 1 1 1 1   1 1 
        .
2 n
n  n  1 n  2  n  n  1  n  1  n  2   n n  1   n  1 n  2 
u u un  1 1 1  1
 L  lim  1  22   n 
 lim    
n 
 2 2 2  n  2 n  1 n  2  2

Tinh hoa của toán học nằm ở tự do của nó – Georg Cantor Chinh phục olympic toán | 39
CÁC BÀI TOÁN VẬN DỤNG CAO DÃY SỐ

(3xn  1)2019
Câu 25. Cho dãy số  xn  được xác định bởi: x1  1; xn1   xn với n là số
2019
 3x 1  1   3x 2  1   3x 3  1   3xn  1 
2018 2018 2018 2018

nguyên dương. Đặt un     ...  . Tính


3x 2  1 3x 3  1 3x 4  1 3 xn  1  1
lim un
2019 2019 673 673
A. B. C. D.
4 3 3 4
Lời giải
(3xn  1)2019
Ta có xn1  xn  , n  1
2019
3  xn  1  xn   3xn  1 
2018
1 1
   
3xn  1 3xn1  1  3xn  1  3xn1  1  673  3xn1  1 

 3x i 1
2018
n n
 1 1   1 1 
 un    673     673   
i 1 3xi  1  1 i  1  3xi  1 3 xi  1  1   3 x1  1 3 xn  1  1 
 3xn  1 
2019

Mặt khác: xn1  xn   0 nên dãy  xn  là dãy số tăng n  1 .


2019
Nếu  xn  bị chặn thì lim xn tồn tại hữu hạn.

( a  1)2019
Giả sử lim xn  a   a  1 và a   a - Điều này vô lý
2019
Suy ra  xn  không bị chặn trên hay lim xn   .
1 673 673
Do đó lim  0 . Suy ra lim un   .
3x n  1  1 n  3x1  1 4

u1  2019

Câu 26. Cho dãy số thực  un  tăng xác định bởi  2
un  2018un  2020un1  1  0, n  1  1

1 1 1
Đặt Sn    ...  . Tính lim Sn
u1  2019 u2  2019 un  2019
1 1
A. 2018 B. C. 2019 D.
2018 2019
Lời giải
Do  un  là dãy tăng nên un  2018, n  1 .
un2  2018un  1
Ta có un2  2018un  2020un  1  1  0  un1 
2020
un2  2018un  2019
 un1  1   2020  un 1  1    un  1  un  2019 
2020
2020 1 1 1 1
     *
 un  1 un  2019  un1  1 un  2019 un  1 un1  1
Thay n bởi 1, 2, 3,..., n vào (*) và cộng vế với vế các đẳng thức ta suy ra

40 | Chinh phục olympic toán Điều ta biết là giọt nước, điều ta chưa biết là đại dương - Newton
TUYỂN TẬP MỘT SỐ NHÓM CÂU HỎI VẬN DỤNG CAO MÔN TOÁN

1 1 1 1 1 1 1
Sn    ...     
u1  2019 u2  2019 un  2019 u1  1 un1  1 2018 un1  1
Do  un  là dãy số tăng nên có hai trường hợp xảy ra.
 Dãy  un  bị chặn trên suy ra tồn tại lim un . Giả sử lim un  x thì x  2018 . Chuyển
qua giới hạn hệ thức (1) khi n   ta có
x 2  2018x  2020x  1  0  x 2  2 x  1  0  x  1 - Điều này vô lý
 Dãy  un  không bị chặn trên, do  un  tăng và không bị chặn trên nên
1
lim un    lim  un1  1    lim 0
un1  1
 1 1  1
Do vậy, lim Sn  lim   
 2018 un1  1  2018
u1  1

Câu 27. Cho dãy số:  un   un 1 . Tìm lim u n .
 un  , n  2
 1  5 .u n 1
n

A. k  1616 B. k  808 C. k  404 D. k  1212


Lời giải
un1 1 1
Từ hệ thức truy hồi ta có un     5n .
1  5n.u n1 un un1
1
Đặt dãy số vn   vn  vn1  5n  vn  vn 1  5n
un
 vn   vn  vn1    vn1  vn 2   ...   v2  v1   v1
5 n 21 1
 vn  5n  5n 1  ...  52  1 
.5   un   limu n  0 .
4 4 5 n 21
.5 
4 4
u1  1, u2  3 u
Câu 28. Cho dãy số  un  được xác định  . Tính lim n2
un 2  2un1  un  1, n 
* n  n

1 1 1
A. 1 B. C. D.
6 3 2
Lời giải

Ta có un 2  un 1  un 1  un  1, n  1, 2,...


 un 2  un 1  un 1  un  1  un  un1  2  ...  u2  u1  n  un  2  un  1  n  2
Do đó un  u1   un  un1    un1  un 2   ...   u2  u1    n    n  1   ...   2 
n  n  1 u n  n  1 1
 un  1  2  ...  n   lim n2  lim 
2 n  n n  2 n2 2

Tinh hoa của toán học nằm ở tự do của nó – Georg Cantor Chinh phục olympic toán | 41
CÁC BÀI TOÁN VẬN DỤNG CAO DÃY SỐ

u1  4

Câu 29. Cho dãy số (un ) xác định như sau:  un 2 . Giả sử giới hạn
u
 n1  un  , n *

 2018
u u u  a
lim  1  2  ...  n    a , b  *
 và ba tối giản. Tính a  3b.
 u2 u3 un1  b
A. 1012 B. 1021 C. 1015 D. 1018
Lời giải
Từ cách xác định dãy số suy ra (un ) là dãy số tăng, nên tồn tại giới hạn hữu hạn hoặc vô
hạn.
Giả sử tồn tại giới hạn hữu hạn l  lim un Khi đó l  4.

un 2 l2
Từ un1  un  , n *
lấy giới hạn hai vế ta có l  l   l  0 (mâu thuẫn).
2018 2018
Vậy lim un  .
un u2 2018  un1  un   1 1 
Từ công thức truy hồi ta có  n   2018   
un1 unun1 unun1  un un1 
u u u   1 1  1 1  1009
 lim  1  2  n   lim 2018     lim 2018    
 u2 u3 un1   u1 un   4 un  2
Vậy a  3b  1015.
2 xn
Câu 30. Cho dãy số  xn  được xác định như sau x1  ; xn  1  ,  n  1, 2....
3 2  2 n  1  xn  1
Hỏi tổng của 2018 số hạng đầu tiên là bao nhiêu?
4035 2017 2018 4036
A. B. C. D.
4036 2018 2019 4037
Lời giải
Dễ thấy xn  0, n  1, 2,... . Nên theo giả thiết ta có
1 1 1
xn  1    2  2n  1  , n  *.
1 xn  1 xn
2  2n  1 
xn
2
Đặt un   u1  3; un1  4  2n  1   un , n  *
xn
 un  1  un  8n  4, n  1, 2,....
 un  un 1  8  n  1   4  un  2  8  n  1    n  2    2.4
 .....  u1  8  n  1    n  2   ....  2  1  n.4
  2 n  1  2 n  1 
2 2 1 1
Do đó xn     , n  1, 2....
un  2n  1 2n  1 2n  1 2n  1
1 1 1 1  1 1  4036
 x1  x2  ......  x2018          ....    
1 3 3 5  4035 4037  4037
42 | Chinh phục olympic toán Điều ta biết là giọt nước, điều ta chưa biết là đại dương - Newton
TUYỂN TẬP MỘT SỐ NHÓM CÂU HỎI VẬN DỤNG CAO MÔN TOÁN

u  1; u2  2
Câu 31. Cho dãy số  un   1 . Tổng S  1  2  ...  2017  u2018  u2019
un1  2un  un1  1; n  2
có giá trị bằng bao nhiêu?
A. 2039190 B. 2035153 C. 2037171 D. 2033136
Lời giải
u1  1
u  2
 2
u3  2 u2  u1  1
Cách 1. Từ công thức truy hồi suy ra 
 u 4  2 u3  u2  1
....

un  2 un 1  un  2  1
Cộng n đẳng thức trên theo vế ta được u1  un  un1  2  n  1  un  un 1  n  * 
u1  1
u  u  1
 2 1

u3  u2  2
Từ đề bài và  *  ta lại suy ra 
u4  u3  3
....

un  un 1  n  1
Cộng n đẳng thức trên theo vế ta được
 n  1 n  1
un  1  1  2  3  ...   n  1   1 
2 2
n 2
 n  2

1
Vậy số hạng tổng quát của dãy số đã cho là un   n2  n  2  , n  1
2
 1
u2018  2  2018  2018  2   2035154
2


u  1  2019 2  2019  2   2037172
 2019 2
 S  1  2  ...  2017  u2018  u2019  1  2  ...  2017  2035154  2037172
2018.2019
 1  2  ...  2017  2018   2037171
2
Cách 2.
Ta có un1  2un  un 1  1; n  2  un 1  un  un  un 1  1; n  2  * 
Đặt vn  un  1  un , n  2 và v1  u2  u1  1
Khi đó  *   vn  vn1  1, n  2 là cấp số cộng có v1  1 công sai d  1
 vn  1   n  1  .1  n , n  1
 S  1  2  ...  2017  u2018  u2019  1  2  ...  2017   u2019  u2018   1  2  ...  2017  v2018
2018.2019
 1  2  ...  2017  2018   2037171
2

Tinh hoa của toán học nằm ở tự do của nó – Georg Cantor Chinh phục olympic toán | 43
CÁC BÀI TOÁN VẬN DỤNG CAO DÃY SỐ

 4
u1  3
Câu 32. Cho dãy số  un  xác định bởi  , n  1 . Tìm lim un .
 n  2  u   n  1  u u  n u
2 2
 n n n1 n1

3
A. lim un  2 B. lim un  4 C. lim un  D. lim un  3
4
Lời giải
n  2
2
n2
Dễ thấy un  0, n  *
. Từ giả thiết ta có    n  1
un 1 un
1 1
Với mỗi n  *
, đặt vn   ta có v1  1 và
un 4
 1 2 1 n2
 n  2   vn1    n  vn     n  1   n  2  vn1  n vn  vn1 
2 2 2
v
n  2
2 n
 4  4
2 2 2 2 2 2
 n1  n2   n3  3  2  1 4 4
 vn        ...       v1  v 
 n  1 n  n  1 n2
2 2 1
 n1  n   n1  5  4  3
2

 lim vn  0 .
 1 1 1 1 1 1
Ta có lim vn  lim     lim   0  lim    lim un  4 .
 un 4  un 4 un 4
2 n  5n
Câu 33. Cho dãy  un  với un  n . Giả sử ta có tổng sau
2n  5
100
a
1 1 1 1   c
b
S    ....  
u1  1 u2  1 u3  1 u100  1 ba
Trong đó a, b c là các số nguyên dương và a, b là hai số dương nguyên tố cùng nhau . Khi
đó S  a  c  ?
A. 151 B. 153 C. 152 D. 154
Lời giải
2 n  5n 1   2  
n
2 n  5n 2.5n 1
Ta có un  1   1     1 
   
2 n  5n 2 n  5n un  1 2.5n 2   5  

1  2 
1 2 100
1 1 1 1 2 2
S     100          
u1  1 u2  1 u3  1 u100  1 2  5 5  5  
 2 
100
2
100

 1     1   151
2   2     5 
100
1  2  5 
 100    100   1      
2  5 2  2  3   5    3
 1  
 5 

Từ đó suy ra a  2, b=5, c=151 nên : a  c  153.

44 | Chinh phục olympic toán Điều ta biết là giọt nước, điều ta chưa biết là đại dương - Newton
TUYỂN TẬP MỘT SỐ NHÓM CÂU HỎI VẬN DỤNG CAO MÔN TOÁN

u1  9
Câu 34. Cho dãy số  un  được xác định bởi  n1 n1 n1
.
u
 n  n
un1  3.2  2.3 , n  2; 3....
Tính giá trị của u2018 ?

A. u2018  2018 3.2 2018  2.32018 B. u2018  2018 9  3.2 2018  2.32018

C. u2018  2018 3.2 2017  2.32017 D. u2018  2018 3.2 2018  32018 .
Lời giải
unn  unn11  3.2 n 1  2.3n 1
 n1 n2 n2 n2
un1  un 2  3.2  2.3
un  2  un  3  3.2 n  3  2.3n  3

Ta có  n 2 n3
 unn  u11  3.  2 1  2 2  ...  2 n1   2.  31  32  ...  3n1  .
 . . .
u 3  u2  3.2 2  2.32
 3 2

u2  u11  3.2 1  2.31


2

1  2 n1 1  3n  1
 9  3.2.  2.3.  3.2 n  3n
12 13
Vậy u2018  2018 3.2 2018  32018 .
 a1  2008
Câu 35. Cho dãy số thực a1 ; a2 ;...; an được xác định bởi  . Tính
 a1  a2  ...  an  n .an , n  1
2

giá trị của a2008 .


1 2 1 2
A. B. C. D.
2009 2007 2007 2009
Lời giải
Ta có a1  a2  ...  an1   n  1  an1 .
2

Do đó a1  a2  ...  an   a1  a2  ...  an1   an   n  1  an1  an .


2

n1
Ta có phương trình  n  1  an1  an  n2 an  an 
2
an 1 .
n1
n1 n2 n3 2 1 2 a1
Suy ra an  . . ... . .a1  .
n1 n n1 4 3 n  n  1
2.2008 2
Cho n  2008 ta được a2008   .
2008.2009 2009
 u1  1

Câu 36. Cho dãy số  un  xác định bởi  1
n
. Có bao nhiêu số nguyên
un  1  un    , n 
*

 2
1999
dương n sao cho un  .
1000
A. 11 B. 10 C. 15 D. Vô số
Lời giải

Tinh hoa của toán học nằm ở tự do của nó – Georg Cantor Chinh phục olympic toán | 45
CÁC BÀI TOÁN VẬN DỤNG CAO DÃY SỐ

u1  1 
 
u  u  1 
 2 1
2 
  1
2 1 n
 1  1 1 1 2  21 1   2  1
Ta có u3  u2      un  1   2  ...  n 1   
 2  2 2 2 1  2n  2 n 1
1
............  2
 n1

 1 
u
 n  u n1    
 2 
Theo giả thiết ta có
1999 1 1999 1 1  1 
un   2  n1   n1   n  1  log 1    10  n  11
1000 2 1000 2 1000 2  1000 

Suy ra có 10 số nguyên dương n thỏa mãn đề bài.


1
Câu 37. Cho dãy số  xn  xác định bởi x1  . Biết rằng
4
x1  4x2  9x3  ...   n  1 xn1
2

xn  , n  2, 3....
n2  n  1 
Tính lim  30n2  12 n  2018  xn
15 15
A. 15 B. 30 C. D.
4 2
Lời giải
 x1  4 x2  9 x3  ...   n  1 2 xn 1   n 2 xn n 2  n  1  xn  n 2 xn
Ta có xn  1     xn  1 
 n  1 n  n  1 n
2 2

n 3 xn n2
xn   n  1  x n  1  n 2 x n .
2
 xn  1   xn  1 
 n  1  n  1
2 2
n
1 1
 n2 xn   n  1 xn1  ...  4x2  1.x1 
2
 xn  2
4 4n
30n2  12n  2018  15 3 1009  15
 lim  30n2  12n  2018  xn  lim 2
 lim    2 

4n  2 n 2n  2

u1  2
Câu 38. Cho dãy số  un  được xác định bởi công thức  . Tìm
 2019 un1  un
2
 2018 u n ,  n  1
u1 u un
giới hạn của dãy số  Sn  xác định bởi công thức Sn   2   .
u2  1 u3  1 un 1  1
2018
A. lim Sn  2018 B. lim Sn  2019 C. lim Sn  D. lim Sn  1
2019
Lời giải
46 | Chinh phục olympic toán Điều ta biết là giọt nước, điều ta chưa biết là đại dương - Newton
TUYỂN TẬP MỘT SỐ NHÓM CÂU HỎI VẬN DỤNG CAO MÔN TOÁN

Trước tiên ta có hai nhận xét sau


 u1  1  un 1  un2  2018un  2019, n  1
 un  2, n  1 nên un2  un  1, n  1 .
un u  un
Theo giả thiết ta có 2019un1  un2  2018un  2019  un1  un   un  un  1    n1
2019 un  1
un un1  un un  1 1 
    2019   
2019  un1  1  un  1 un1  1 un1  1  un  1 un1  1 
 1 1 
 Sn  2019   
 u1  1 un1  1  .
1
Để tính lim , ta chứng minh mệnh đề  *  : 2019un  1  4036  n , n  1 bằng quy nạp.
un 1  1
Từ 2019u2  4  4036  4040  4037 suy ra mệnh đề  *  đúng khi n  1 .
Giả sử 2019uk  1  4036  k , k  1 . Khi đó
2019uk  2  uk2 1  2018uk  1  uk  1  1  2018uk  1  1  2019uk  1  1  4036  k
.
Suy ra  *  đúng khi n  k  1 . Hay 2019un  1  4036  n , n  1 .
1 2019
Do đó 2019  un1  1   2019  n   .
un 1  1 n  2019
2019 1
Ta lại có lim  0 nên lim 0.
n  2019 un1  1
 1 1 
Vậy lim Sn  2019   lim   2019 .
u
 1  1 un1  1 
un
Câu 39. Cho dãy số  un  được xác định bởi: u1  1, un1  , n  1, 2, 3,...
un  1
2018  u1  1  u2  1  ...  un  1 
Tính lim .
2019n
2018
A. lim Sn  2018 B. lim Sn  2019 C. lim Sn  D. lim Sn  1
2019
Lời giải
un 1 1 1
Do u1  0  un  0, n  * . Ta có un1     1  un  , n  1, 2,...
un  1 un1 un n
 2 3 n1
 u1  1 u2  1 ...  un  1    1  1
1
 1   1  ...   1   . ...  n1
1  2  n  1 2 n
 1
2018  1  
2018  u1  1  u2  1  ...  un  1  2018  n  1   n  2018
 lim  lim  lim 
2019n 2019n 2019 2019
2018  u1  1 u2  1  ...  un  1  2018
Vậy lim  .
2019n 2019

Tinh hoa của toán học nằm ở tự do của nó – Georg Cantor Chinh phục olympic toán | 47
CÁC BÀI TOÁN VẬN DỤNG CAO DÃY SỐ

Câu 40. Cho các số a1 , a2 , a3 , a4 , a5  0 lập thành cấp số cộng với công sai d và
b1 , b2 , b3 , b4 , b5  0 lập thành cấp số nhân với công bội q . Biết rằng a1  b1 và a5  b5 . Hỏi
có bao nhiêu khẳng định luôn đúng trong các khẳng định sau?
i) a2  b2 ii) a3  b3 iii) a4  b4 iv) d  q
A. 1 B. 2 C. 3 D. 4
Lời giải
a4  3a1 3x  y
Đặt a1  b1  x , a5  b5  y , mà a5  a1  4d và a2  a1  d nên a2   .
4 4
xy x  3y
Tương tự ta tính được a3  và a4  .
2 4
Lập luận tương tự với CSN, ta cũng có b2  4 x 3 y , b3  xy , b4  4 xy 3 .
Theo bất đẳng thức AM – GM ta có
xy 3x  y 4 3 x  3y 4 3
a3   xy  b3 , a2   x y  b2 , a4   xy  b4
2 4 4
Do đó, cả i), ii) và iii) đều đúng. Tuy nhiên, điều kiện iv) không luôn đúng, chẳng hạn khi
x  y thì d  0 nhưng q  1.
3  n  1
Câu 41. Cho dãy số  un  biết : u1  1 , un1  un  2n3  3n  1  n  *  . Giá trị nhỏ
n
nhất của n để un  n3  n.32018 là bao nhiêu?
A. n  2019 B. n  2018 C. n  2017 D. n  2020
Lời giải
Biến đổi giả thiết ta có
3  n  1 un1 u u u 
 3 n  2n2  2n  1  n 1   n  1   3  n  n2 
2
un1  un  2n3  3n  1 
n n1 n n1  n 
un
Đặt Sn   n2 .Ta có Sn 1  3Sn ,n  *
n
 Dãy số  Sn  là cấp số nhân với công bội q  3 và S1  2  Sn  2.3n1 , n  N *
un 1
Theo bài ra, un  n3  n.32018   n2  32018  2.3n1  32018  3n 2019 
n 2
1 1
 n  2019  log 3  n  log 3  2019  2018, 369...
2 2
Vậy giá trị nhỏ nhất của n là n  2019 .

48 | Chinh phục olympic toán Điều ta biết là giọt nước, điều ta chưa biết là đại dương - Newton
TUYỂN TẬP MỘT SỐ NHÓM CÂU HỎI VẬN DỤNG CAO MÔN TOÁN

Câu 42. Cho dãy số không âm  un  ,  n  *


 được xác định bởi công thức sau
u2  1

 2 1 2  m, n  , m  n 
u
 m  n  1  u 2
m n  1 
2
 u2 m1  u 2
2 n1 
Khi đó tổng của 2019 số hạng đầu tiên của dãy khi viết dưới dạng thập phân có chữ số ở
hàng đơn vị bằng bao nhiêu?
A. 1 B. 2 C. 3 D. 4
Lời giải
1 2
Cho n  m ta có u22m1  u12 
2
 u2 m1  u22m1   u1  0  1
1
Cho n  0 ta có um2 1  um2 1   u22m1  u12   u22m1  4um2 1  u2 m1  2um1  2 
2
u  u2.11  2.u11  2u2  2. 1
Vì u2  1 nên  3  u2211  u2211   u421  u221   u42  9  u4  3 .
u5  u2.2 1  2u2 1  2u3  4. 2
Ta chứng minh un 1  n , n   3
Thật vậy, với n  0, n  1, n  2, n  3 thì  3  đúng.
Giả sử  5  đúng đến n  k , k  , k  3 , tức là uk  1  k và uk  k  1 . Khi đó
1 2 1
uk2 1 1  uk21 1 
2
 u2 k  1  u32    4uk2 1  4 
2
 2 k  2   k  1  k  2 k  1   k  1
2 2
 uk2 2 2 2

 uk  2  k  1
Vậy tổng của 2019 số hạng đầu tiên của dãy là S2019  0  1  2  ...  2018  2037171 .
Do đó chữ số ở hàng đơn vị là 1.
Câu 43. Cho dãy số  xn  được xác định bởi x1  2019, xn 1  xn2  xn  1, n  1, 2, 3,... . Với
 1 1 1
mỗi số nguyên dương n , đặt yn  2019    ...   . Khi đó lim y n bằng?
 x1 x 2 xn 
2018 2019
A. B. C. 2018 D. 2019
2019 2018
Lời giải
Ta có xn1  xn  xn2  2 xn  1   xn  1   0  xn  1  xn , n  1. Do đó  xn  tăng.
2

Giả sử dãy  xn  có giới hạn hữu hạn bằng A  lim xn  lim xn 1  A  2019 . Chuyển qua
giới hạn hai vế phương trình xn 1  xn2  xn  1 ta được A  A2  A  1  A  1  2019 vô lý.
Vậy lim xn   .
1 1 1 1 1 1 1
Ta có xn 1  1  xn  xn  1         .
xn  1  1 xn  xn  1  xn  1 xn xn xn  1 xn  1  1
 1 1 1  1 1   1 1 
Do đó yn  2019    ...    2019     2019   
 x1 x 2 xn   x1  1 x n  1  1   2018 xn1  1 
Tinh hoa của toán học nằm ở tự do của nó – Georg Cantor Chinh phục olympic toán | 49
CÁC BÀI TOÁN VẬN DỤNG CAO DÃY SỐ

1 2019
Từ lim xn    lim  0 . Vậy lim yn  .
xn 2018
u1  2020
Câu 44. Cho dãy số (un) được xác định bởi  .
 4 n 2
 16 n  un  1   n 2
 6 n  5  u n , n  1

 4n 
Gọi k  lim  2 .un  thì k có giá trị là?
n 
A. k  1616 B. k  808 C. k  404 D. k  1212
Lời giải
Ta có  4n2  16n  un 1   n2  6n  5  un  4  n2  4n  un 1   n  1  4  n  1  un
2
 
1  n  1  4  n  1
2
un  1 1 u
 un  1  . un   . 2 n
n  4n  n  1   4  n  1  4 n  4n
2 2
4
un 1
Đặt dãy số vn   vn 1  vn
n  4n
2
4
1
  vn  là cấp số nhân có công bội q  và số hạng đầu
4
n1 n1
u 1 1 1
v1  1  .2020  404  vn  404.  
5 5 4
 un  404.  
4
n 2
 4n 

 4n 1
n1
  n 2  4n   4
 k  lim  2 .404.    n  4n    lim 
2
.4.404   lim  1   .1616  1616
n 4   n
2
  n
 
u1  1

Câu 45. Cho dãy  un  được xác định bởi  1  un21  1 , đặt
u
 n  ; n  2, n 
 un1
Sn  u1  u2  ...  un . Hãy chọn mệnh đề sai trong các mệnh đề sau?

 1 
n 1

A.  un  là dãy bị chặn. B. Sn  1  1    
4   2  
C.  un  là dãy giảm D. Sn  n , n  
.
Lời giải
 
1  tan 2  1 1  cos
 4 4  tan  .
Cách 1. Ta có u1  1  tan ; u2  
4   2.4
tan sin
4 4

Từ đây ta dự đoán un  tan n1  1 .
, n  N 
2 .4

Thật vậy, giả sử uk  tan k 1 , k  1 , khi đó
2 .4

50 | Chinh phục olympic toán Điều ta biết là giọt nước, điều ta chưa biết là đại dương - Newton
TUYỂN TẬP MỘT SỐ NHÓM CÂU HỎI VẬN DỤNG CAO MÔN TOÁN

 
1  tan 2 1 1  cos
2 .4  tan  .
k 1 k 1
2 .4
uk  1  
  2 k.4
tan k 1 sin k 1
2 .4 2 .4
Theo nguyên lý quy nạp suy ra công thức  1  đúng.

 Vì 0  n1
 0  un  1 nên  un  là dãy bị chặn.

2 .4 4
 
 Vì tan x là hàm số đồng biến trên  0;  suy ra  un  là dãy giảm.
 4
 Ta có Sn  u1  u2  ...  un  nu1  n.
  1  
 Xét hàm f ( x )  tan x  x ; x  0;  , có f ( x )  2
 1  tan 2 x  0, x  0;  .
 4 cos x  4
 
 tan x  x , x  0;  đẳng thức xảy ra khi và chỉ khi x  0 . Do đó
 4
 1 
n1
  
Sn  1   2   n 1  1  1    
2.4 2 .4 2 .4 4   2  
Cách 2.

 1  un21  1 un 1
Từ giả thiết suy ra un  0, n  . Ta có un    un 1 .
un  1 1  un21  1
Suy ra  un  giảm (C đúng) và un   0; 1 , n  
hay A đúng. Và khi đó Sn  n tức D
đúng. Vậy chọn B.
u1  1

Câu 46. Cho dãy số  un  thỏa mãn  2  n  1  un 1 2n
un   , n  *
.

 
2
n n 2
 n  1  1

Tìm giới hạn của dãy số  sn  với sn  n3 un , n  *
.
A. lim  sn    B. lim  sn   0

C. lim  sn   1 1
D. lim  sn   . .
2
Lời giải
Ta có  n2  n  1  1   n2  1   2n  n2  1   n2  1   n2  1   n  1   1
2 2
 2

Theo giả thiết ta có
2  n  1  un 1 2n 2n  n2
un    nun  2  n  1  un  1 
n  n2  n  1  1
2
 n2  1   n  1 2  1  
1 2 1 1 1 
 nun  2  n  1 un1     n  1 un1    nun  2   1
n  1  n  1  1
2 2
 n  1  1 2 
2
n 1

Tinh hoa của toán học nằm ở tự do của nó – Georg Cantor Chinh phục olympic toán | 51
CÁC BÀI TOÁN VẬN DỤNG CAO DÃY SỐ

1 1
Đặt vn  nun  , n  *
. Khi đó  1  trở thành vn1  vn , n  *
  vn  là một cấp
n 1
2
2
n1 n
1 1 1 1 1
số nhân với công bội q  , v1  u1    vn  v1 .    
2 2 2 2 2
1 1 n3 n2
 un    sn  n 3
un   .
n3  n 2 n n n3  n 2 n
Ta có
n3 1
 lim  lim  1;
n n
3
1
1 2
n
 Với mọi số nguyên dương n  3 , ta có
n3  5n  6 n3  5n  6
2  C  C  C  C  ...  C  C  C  C  C 
n 0
n
1
n
2
n
3
n
n
n
0
n
1
n
2
n 2 
n 3
n 2
6 6
n3  5n  6
Mặt khác  2  vẫn đúng mới n  1; 2 . Do vậy nên 2 n  , n  * .
6
6
n2 n2 6n 2 6 n 2
n n2
 n  n 3 , n  * ; lim 3  lim  0  lim n  0.
2 2 n  5n  6 n  5n  6 5 6 2
1 2  3
n n
 n3 n2 
Vậy lim sn  lim  3  n   1.
n n 2 
un  1  4un2  4un  0

Câu 47. Cho các dãy  un  thỏa:  1  n  *
 . Khi đó u 1 có thể nhận tất cả
u2018 
 2
bao nhiêu giá trị?
A. 2 2017 B. 2 2018
C. 2 2019 D. 2 2018  1 .
Lời giải
Xét hàm số: f  x   4x  4x với x   0;1  ta có f  x    0; 1 
2

1
Mặt khác u2018  f  u2017  
  0; 1   u2017   0; 1   ...  u1   0; 1 
2
Ta chứng minh bằng quy nạp 0  un  1 .
Theo trên có u1   0; 1  ; u2   0; 1  . Giả sử uk   0; 1  khi đó do uk  1  f  uk    0; 1  nên có
điều chứng minh.
 
Vì 0  u1  1 nên tồn tại số   0;  sao cho u1  sin 2 
 2
Khi đó u2  4u1  1  u1   4 sin 2   1  sin 2    sin  2 2  

Theo nguyên lý quy nạp ta có un  sin 2  2 n1  

52 | Chinh phục olympic toán Điều ta biết là giọt nước, điều ta chưa biết là đại dương - Newton
TUYỂN TẬP MỘT SỐ NHÓM CÂU HỎI VẬN DỤNG CAO MÔN TOÁN

1 1 1 1 1
Theo giả thiết ta có u2018   sin 2  2 2017      cos  2 2018   
2 2 2 2 2
 
 cos  2 2018    0    2019  k 2018  k  
2 2
     1 1
Vì   0;  nên 0  2019  k 2018     k  2 2017 
 2 2 2 2 2 2
Vậy k  {0; 1; 2;...; 2 2017  1}
 
Do đó có 2 2017 giá trị u1  2019
k 2018
với k  0; 1; 2;  ; 2 2017  1 thỏa yêu cầu.
2 2
2 2
Câu 48 . Cho dãy số  un  thỏa mãn: u1  1 ; un1  un  a , n  *
.
3
Biết rằng lim  u12  u22  ...  un2  2n   b . Giá trị của biểu thức T  ab là?
A. 2 B. 1 C. 1 D. 2
Lời giải
2 2 2
Biến đổi giả thiết ta có un1  un  a  un21  3a   un2  3a 
3 3
2
Đặt vn  un2  3a thì  vn  là cấp số nhân với v1  1  3 a và công bội q  .
3
n 1 n 1
2
 1  3a   u  vn  3a   
2
Do đó vn    2
n  1  3a   3a .
3 3
n
2
1 
 3   2 n 
 u1  u2  ...  un  2n   1  3a 
2 2 2
 2n  3na  3  1  3a   1      n  3a  2  .
2  3 
1  
3
Mặt khác ta lại có lim  u12  u22  ...  un2  2n   b

   2 n    2
3a  2  0 a 
 lim  3  1  3 a   1      n  3 a  2    b    3 ,


 3 
 

 
 b  3  1  3 a  b  3

Vậy T  ab  2 .
Câu 49. Cho 2 dãy cấp số cộng un  u1 ; u2 ;...un có công sai d 1 và vn  v1 ; v2 ;...vn có công sai
d2 . Gọi tổng của n số hạng đầu của mỗi cấp số theo thứ tự là
u11
Sn  u1  u2  ...  un  7 n  1 và Tn  v1  v2  ...  vn  14n  27 . Tính tỉ số của
v11
5 4 9 5
A. B. C. D.
3 3 4 4
Lời giải
n  2u1   n  1 d1  n  2 v1   n  1 d2 
Từ giả thiết, ta có Sn   và Tn  
2 2

Tinh hoa của toán học nằm ở tự do của nó – Georg Cantor Chinh phục olympic toán | 53
CÁC BÀI TOÁN VẬN DỤNG CAO DÃY SỐ

 Sn 2 u1   n  1  d1 7n  1
    1
T 2 v1   n  1  d2 4n  27
 n .
 u11  u1  10 d1  2 u1  20 d1 2
 v11 v1  10 d2 2 v1  20 d2
u11 148 4
So sách (1) và (2) bằng cách đồng nhất  n  1  20  n  21   
v11 111 3
Câu 50. Cho dãy số  an  xác định bởi a1  5, an 1  q.an  3 với mọi n  1 , trong đó q là
hằng số, q  0 , q  1 . Biết công thức số hạng tổng quát của dãy số viết được dưới dạng

n1 1  q n1
an  .q  . Tính   2 ?
1q

A. 13 B. 9 C. 11 D. 16

Lời giải
3
Ta có an 1  k  q  an  k   k  kq  3  k 
1q
Đặt vn  an  k  vn 1  q.vn  q 2 .vn  1  ...  q n .v1
 3 
Khi đó vn  q n1 .v1  q n1 .  a1  k   q n1 .  5  
 1q 
 3  n1  3  3 1  q n1
Vậy an  vn  k  q n1 .  5    k  q .  5     5.q n1
 3. .
 1  q   1  q  1  q 1  q
Do đó   5;   3    2  5  2.3  11 .
Câu 51. Cho cấp số nhân u1 , u2 , u3 ,.., un ; trong đó ui  0, i  1, 2,..., n . Biết rằng
1 1 1 1 1
Sn  u1  u2  u3  ...  un  2018 , Tn     ...   2019 và P  u1 .u2 .u3 ....un  .
u1 u2 u3 un 100
Hỏi số tự nhiên nhỏ nhất thỏa mãn P là?

A. 9295 B. 9296 C. 18592 D. 18591

Lời giải
u1  q n  1
Ta có Sn  u1  u2  u3  ...  un   2018  1
q1


1 1 1
Tn     ... 
1

 1  q   2019 n

2
u1 u2 u3 un u1q n1  1  q 
Tn 2018
Từ  1  và  2  suy ra  u12 q n1 
Sn 2019
n
n n  1 n
 2018  2
Ta có Qn  u1 .u2 .u3 ....un  u1 .  u1 .q  .  u1 .q 2  ....  u1 .q n 1   u1nq 2
 u q
1 
2 n 1 2
 
 2019 

54 | Chinh phục olympic toán Điều ta biết là giọt nước, điều ta chưa biết là đại dương - Newton
TUYỂN TẬP MỘT SỐ NHÓM CÂU HỎI VẬN DỤNG CAO MÔN TOÁN

n
 2018  2 1  1 
Theo đề     n  2 log 2018    18591, 1  nmin  18592
 2019  100 2019  100 

4
Câu 52. Gọi q là công bội của một cấp số nhân , biết tổng ba số hạng đầu bằng 16 , đồng
9
thời theo thứ tự , chúng là số hạng thứ nhất , thứ tư và thứ tám của một cấp số cộng . Hỏi
q thuộc khoảng nào sau đây?
A. q   3; 4  B. q   1; 2  C. q   2; 3  D. q   0; 1 
Lời giải
Gọi : u1 , u2 , u3 là 3 số hạng đầu tiên của cấp số nhân , với công bội q . Gọi  vn  là cấp số
cộng tương ứng với công sai là d . Theo giả thiết ta có :
 4  4
u1  u2  u3  16 9 u1  u1q  u1q  16 9
2
 1
u  v 
 1 1  u1q  u1  3d 2
u2  v4  v1  3d  2
 u1q  u1  7 d 3
u
 3  v 8  v 1  7 d 
Khử d từ (2) và (3) ta được : u1  3q 2  7 q  4   0  4 .
q  1
4
Do (1) nên : u1  0   4    4 . Theo định nghĩa thì q  1 , do vậy q 
q  3
 3
n
Câu 53. Cho dãy số  un  như sau: un  , n  1 , 2 , ... Tính giới hạn của tổng
1  n2  n4
lim  u1  u2  ...  un  .
x 

1 1 1
A. B. 1. C. D.
4 2 3
Lời giải
n n 1 1 1 
Ta có un     2  2 
1  n 
2 2
 n2 n 2
 n  1  n  n  1 
2
2 n n1 n n1

1 1 1 1 1 1 1 1 1 1 
Ta có u1  u2  ...  un   1         ...  2  2 
2 3 3 7 7 13 13 21 n n1 n n1
1
1
 1 n n
2
1 1 1 1 n

1 2   lim  u1  u2  ...  un   lim  .
n n1 2 n n1 2
2 1 1
2 1  2 2
n n

 x  10 khi x  2018
Câu 54. Cho hàm số f  x    . Tính giá trị f  1   f  2018  .
 f  f  x  11  khi x  2018

A. 1999 B. 2009 C. 4018 D. 4036
Lời giải
Ta có
Tinh hoa của toán học nằm ở tự do của nó – Georg Cantor Chinh phục olympic toán | 55
CÁC BÀI TOÁN VẬN DỤNG CAO DÃY SỐ

f  2018   f  f  2018  11    f  f  2029    2019  10  2009


f  2017   f  f  2017  11    f  f  2028    f  2018   2009
...
f  2009   f  f  2009  11    f  f  2020    f  2010   2009
f  2008   f  f  2008  11    f  f  2019    f  2009   2009
f  2007   f  f  2007  11    f  f  2018    f  2009   2009
...
f  1   f  f  1  11    f  f  12    2009
Do đó ta có f  2018   f  2018   ...  f  1  2009  f  1  f  2018   4018

Câu 55. Cho dãy un  thỏa mãn 25.2 2 u5  1  15.2 u1  u5  2  5.2 u5  15.2 u1  4  0 và un  1  un  8.
Giá trị nhỏ nhất của n để un  2019.
A. 512. B. 258. C. 511. D. 257.
Lời giải
Từ un 1  un  8.   un  là CSC công sai d  8  un  u1  8  n  1   u5  u1  32
Thay vào giả thiết ta được:
2  5.2 32  3  2 u1    5.2 32  3  2 u1  4  0
2

1  33  1  33 
Có dạng phương trình bậc 2 suy ra:  5.2 32  3  2 u1   u1  log 2  
4  4  5.2  3  
32

2019  u1
un  u1  8  n  1  2019  n   1  257, 63  nmin  258
8
Câu 56. Cho một cấp số cộng : u1 , u2 , u3 , u4 thỏa u1u4  u2 u3  6 . Tìm tập xác định D của

hàm f  x    x  u1  x  u2  x  u3  x  u4   9
A. D    ; 6  B. D   6;   C. D  D. D   6; 6 
Lời giải
Theo tính chất của cấp số cộng , ta có : u1  u4  u2  u3
Do đó  x  u1  x  u2  x  u3  x  u4    x 2   u1  u4  x  u1u4  x 2   u2  u3  x  u2 u3   * 
Đặt t  x 2   u1  u4  x  x 2   u2  u3  x , khi đó :
 *   f  t    t  u1u4  t  u2 u3   9  t 2   u1u4  u2 u3  t  u1u4u2u3  9
Với : t   u1u4  u1u3   4u1u2 u3u4  36   u1u4  u2 u3   36 .
2 2

Rõ ràng u1u4  u2 u3  6   t  0  f  t   0, t  f  x  có nghĩa với mọi x

56 | Chinh phục olympic toán Điều ta biết là giọt nước, điều ta chưa biết là đại dương - Newton
TUYỂN TẬP MỘT SỐ NHÓM CÂU HỎI VẬN DỤNG CAO MÔN TOÁN

2 2 2
 1  1  1 
Câu 57. Biết tổng Sn   2     2 2  2   ...   2 n  n  . Giá trị nhỏ nhất của n để
 2  2   2 
399  2n4n
Sn  , n *

4n
A. 41 B. 40 C. 51 D. 50
Lời giải
 1   4 1   2n 1 
Ta có Sn   2 2  2  2    2  2  4   ...   2  2  2 n 
 2   2   2 
 1 1 1 
  2 2  2 4  ..  2 2 n   2n   2  4  ..  2 n 
2 2 2 
qn  1
Áp dụng công thức tính tổng của n số hạng đầu của một cấp số nhân : Sn  u1 :
q1
n
1
1
Sn  4.
4 1
n
1  4 
 2n  .  2n 
 4n  1  4 n 1  1 
3 4 1 1 3.4n
4
Theo đề bài ta có:

2n 
4 n
 1 4n1  1 
399  2n4n
  4n  1 4n1  1   3100  n  39, 124...  nmin  40

n n
3.4 4
Câu 58. Cho dãy ( xn ) thỏa mãn x1  5, xn 1  xn2  2, n  1 . Tính giá trị của
 1 1 1 
M  lim    ........  
 x1 x1 x 2 x1x2 ...xn 
5  21 5  21 3  31 3  15
A. M  B. M  C. M  D. M 
2 2 3 3
Lời giải
Đầu tiên dễ thấy xn  
Ta có xn2 1   xn2  2   xn21  4  xn2  xn2  4   ...   x1 .x2 ....xn   x1 2  4 
2 2

xn  1 4 xn  1 4
   21  lim  lim  21  21
 x1 .x2 ....xn   x1 .x2 ....xn 
2 2
x1 .x2 ....xn x1 .x2 ....xn

xn  1 x 2 2 xn 2  1 1 1 
Lại có  n    ...  x1  2    ........  
x1 x2 ...xn x1 x2 ...xn x1 ...xn1 x1x2 ...xn  x1 x1 x 2 x1x2 ...xn 
1 1 1 1 xn  1 
   ........    x1  
x1 x1 x 2 x1 x2 ...xn 2  x1 x2 ...xn 
 1 1 1  1 xn 1  5  21
 lim    ........     5  lim 
 x1 x1 x 2 x1 x2 ...xn  2  x1 x2 ...xn  2

Tinh hoa của toán học nằm ở tự do của nó – Georg Cantor Chinh phục olympic toán | 57
CÁC BÀI TOÁN VẬN DỤNG CAO DÃY SỐ

 1 
Câu 59. Cho hàm số y  f  x   ln  1  2  . Biết rằng :
 x 
f  2   f  3   ...  f  2018   ln a  ln b  ln c  ln d
trong đó a , c , d là các số nguyên tố và a  b  c  d . Tính P  a  b  c  d
A. 1986 B. 1698 C. 1689 D. 1989
Lời giải
 x 1 2
Ta có y  ln  2   ln  x  1  ln  x  1  2 ln x
 x 
Khi đó:
f  2   ln 1  ln 3  2 ln 2
f  3   ln 2  ln 4  2 ln 3
f  4   ln 3  ln 5  2 ln 4
..........
f  2017   ln 2016  ln 2018  2 ln 2017
f  2018   ln 2017  ln 2019  2 ln 2018
 f  2   f  3   f  4   ...  f  2017   f  2018 
  ln 2  ln 2018  ln 2019  ln 3  ln 4  ln 673  ln 1019

Câu 60. Cho dãy số  un  thỏa mãn log u1  2  log u1  2 log u10  2 log u10 và un 1  2un với
mọi n  1 . Giá trị nhỏ nhất để un  5100 bằng

A. 247 B. 248 C. 229 D. 290

Lời giải
Vì un 1  2un nên dễ thấy dãy số  un  là cấp số nhân có công bội q  2 .
Ta có u10  u1 .q 9  2 9.u1 . Xét log u1  2  log u1  2 log u10  2 log u10

 log u1  2 log  2 9.u1   2  log u1  2 log  2 9.u1   0

 log u1  18 log 2  2 log u1  2  log u1  18 log 2  2 log u1  0


  log u1  18 log 2  2  log u1  18 log 2  0
Đặt 2  log u1  18 log 2  t  t  0 . Phương trình trên trở thành
t  1
t2  2  t  0  t2  t  2  0  
t  2  L 
5
Với t  1  2  log u1  18 log 2  1  2  log u1  18 log 2  1  u1 
2 17
5 n1
Trong trường hợp này ta có: un  .2  5100  2 n18  599  n  99 log 2 5  18
2 17
Mà n  *
nên giá trị nhỏ nhất trong trường hợp này là n  248 .

58 | Chinh phục olympic toán Điều ta biết là giọt nước, điều ta chưa biết là đại dương - Newton
TUYỂN TẬP MỘT SỐ NHÓM CÂU HỎI VẬN DỤNG CAO MÔN TOÁN

Chọn ý B.
Câu 61. Cho dãy số  un  thỏa mãn ln 2 u6  ln u8  ln u4  1 và un 1  un .en  1 . Tìm u1

A. e B. e 2 C. e 3 D. e 4

Lời giải
Từ giả thiết suy ra dãy số  un  là cấp số nhân với công bội e và un  0n  1 .
Ta có u6  u1 .e 5 ; u8  u1 .e7 ; u4  u1 .e 3 . Do đó ta có:
ln 2 u6  ln u8  ln u4  1  ln 2  u1 .e 5   ln  u1 .e7   ln  u1 .e 3   1
  ln u1  5    ln u1  7    ln u1  3   1   ln u1   8  ln u1   16  0
2 2

 ln u1  4  u1  e 4
Chọn ý D.
Câu 62. Cho dãy số  un  thỏa mãn e u18  5 e u18  e 4 u1  e 4 u1 và un  1  un  3 với mọi n  1 .
Giá trị lớn nhất của n để log 3 un  ln 2018 bằng?

A. 1419 B. 1418 C. 1420 D. 1417

Lời giải
Ta có un  1  un  3 với mọi n  1 nên  un  là cấp số cộng có công sai d  3

e u18  5 e u18  e 4 u1  e 4 u1  5 e u18  e 4 u1  e 4 u1  e u18  1 


Đặt t  e u18  e 4 u1  t  0  Phương trình  1  trở thành

5 t  t  t  5 t  0  t  
t 5 0 t 0 t 0

Với t  0 ta có e u18  e 4 u1  u18  4u1  u1  51  4u1  u1  17


Vậy un  u1   n  1  d  17   n  1 3  3n  14
3ln 2018  14
Khi đó ta được log 3 un  ln 2018  un  3 ln 2018  3n  14  3 ln 2018  n   1419, 98
3
Vậy giá trị lớn nhất của n là 1419 .
Chọn ý A.

3
Câu 63. Cho dãy số  an  thỏa mãn a1  1 và 5an1  an  1  , với mọi n  1 . Tìm số
3n  2
nguyên dương n  1 nhỏ nhất để an là một số nguyên.

A. n  123 B. n  41 C. n  39 D. n  49

Lời giải
3 3n  5 3n  5
Từ giả thiết ta có 5an1  an 1   5an1  an   an1  an  log 5
3n  2 3n  2 3n  2
Từ đó suy ra

Tinh hoa của toán học nằm ở tự do của nó – Georg Cantor Chinh phục olympic toán | 59
CÁC BÀI TOÁN VẬN DỤNG CAO DÃY SỐ

3n  2 3n  1 3n  2
an  an 1  log 5  an  2  log 5  log 5
3n  1 3n  4 3n  1
...
8 11 3n  1 3n  2
 a1  log 5  log 5  ...  log 5  log 5
5 8 3n  4 3n  1
 8 11 3n  1 3n  2  3n  2
 1  log 5  . ... .   1  log 5  log 5  3n  2 
 5 8 3n  4 3 n  1  5
Do đó an  log 5  3n  2  . Vì n  1 nên an  log 5  3n  2   log 5 5  1 , đồng thời dễ thấy  an 
5an  2
là dãy tăng. Lại có an  log 5  3n  2   n  .
3
Lần lượt thử các giá trị an  2; 3; 4;... ta có an  3 là giá trị nguyên, lớn hơn 1, nhỏ nhất, cho
giá trị tương ứng n  41 .
Vậy n  41 .
Chọn ý B.
 2u u u u
4e 9  2 e 9  4e 1 9  e 1  e 1  3
u 2u

Câu 64. Cho dãy số  un  thỏa mãn  . Giá trị nhỏ nhất của
 u
 n1  un  3, n  *

số n để un  1 ?

A. 725 B. 682 C. 681 D. 754

Lời giải
Từ giả thiết ta suy ra  un  là CSC có công sai d  3  u9  u1  24 .
Biến đổi giả thiết tương đương
4 e 2 u9  2 e u9  4 e u1  u9  e u1  e 2 u1  3
 4 e 2 u1  48  2 e u1  24  4 e 2 u1  24  e u1  e 2 u1  3  0

 2e  1  e 2 u1    2e 
 1  e 2 u1  3  0
2
 24 24

1  13  1  13 
  2 e 24  1  e 2 u1   u1  ln  
2  2  2 e  1  
24

Ta có un  u1  3  n  1   2018  n  681  n  682


Chọn ý B.
Câu 65. Cho dãy số  un  có số hạng đầu tiên u1  1 thỏa mãn đẳng thức sau :
log 22  5u1   log 22  7 u1   log 22 5  log 22 7 và un 1  7 un với mọi n  1 . Giá trị nhỏ nhất của n
để un  1111111 bằng?

A. 11 B. 8 C. 9 D. 10

Lời giải
Vì un 1  7 un nên dễ thấy dãy số  un  là cấp số nhân có công bội q  7 .
Biến đổi giả thiết tương đương

60 | Chinh phục olympic toán Điều ta biết là giọt nước, điều ta chưa biết là đại dương - Newton
TUYỂN TẬP MỘT SỐ NHÓM CÂU HỎI VẬN DỤNG CAO MÔN TOÁN

log 22  5u1   log 22  7 u1   log 22 5  log 22 7


  log 2 5  log 2 u1    log 2 7  log 2 u1   log 22 5  log 22 7
2 2

 2 log 2 5.log 2 u1  2 log 22 u1  2 log 2 7.log 2 u1  0


log 2 u1  0 u  1  L  1
  1  u1 
 2 log 2 5  2 log 2 u1  2 log 2 7  0 log 2 35u1  0 35
1 n1
Ta có un  u1 .7 n 1 . un  1111111  .7  1111111  7 n 1  35.1111111
35
 n  log 7  35.1111111   1 . Mà n  * nên giá trị nhỏ nhất trong trương hợp này là
n  10 .
Chọn ý D.
Câu 66. Có bao nhiêu giá trị nguyên của tham số a thuộc đoạn  0; 2018  sao cho ba số
a
5x 1  51 x ; ; 25x  25 x theo thứ tự đó, lập thành một cấp số cộng?
2
A. 2008 B. 2006 C. 2018 D. 2007

Lời giải
a
Ba số 5x  1  51 x ; ; 25x  25 x , theo thứ tự đó lập thành cấp số cộng khi và chỉ khi
2
a   5x  1  51 x    25x  25 x   2 5x  1  51 x  2 25x  25 x  12 .

 x 1
5  5
1 x

Dấu “=” xảy ra khi và chỉ khi  x x


x0.
25  25

Như vậy nếu xét a   0; 2018 thì ta nhận a   12; 2018  . Có 2007 số a thoả đề.
Chọn ý D.
8
Câu 67. Cho dãy số  un  thỏa mãn 2 2 u1  1  2 3u2  và un 1  2un với
1 
log 3  u32  4u1  4 
4 
mọi n  1 . Giá trị nhỏ nhất của n để Sn  u1  u2  ...  un  5100 bằng

A. 230 B. 231 C. 233 D. 234

Lời giải
Theo giả thiết ta có un 1  2un nên  un  là một cấp số nhân với công bội q  2 . Suy ra
un  u1 .2 n 1 với mọi n  *
, n  2 . Ta lại có :
8 8 8
2 2 u1  1  2 3u2   2.4 u1    1
1  1 4 u1

log 3  u32  4u1  4  log 3  u32  u3  4 
4  4 
8 8 8
Mà 2.4u1  u1  8 và  8
4 1 2   1 
2

log 3  u3  u3  4  log  u  1  3 
3  
4   2
3
 
Tinh hoa của toán học nằm ở tự do của nó – Georg Cantor Chinh phục olympic toán | 61
CÁC BÀI TOÁN VẬN DỤNG CAO DÃY SỐ

 u1 8
 2.4  8
4 u1
 1
Nên phương trình  1  tương đương  8  u1 
8 2
 1 2 
 log 3 u 3  u3  4 
 4 
1  2n 2n  1
Khi đó Sn  u1  u2  ...  un  u1 
12 2
2n  1 2n  1
Do đó, Sn  5 100
  5  log 5
100
 100  n  233
2 2
Chọn ý D.
Câu 68. Cho dãy số  un  thỏa mãn log 3  2u5  63   2 log 4  un  8n  8  , n  *
.
un .S2 n 148
Đặt Sn  u1  u2  ...  un . Tìm số nguyên dương lớn nhất n thỏa mãn  .
u2 n .Sn 75

A. 18 B. 17 C. 16 D. 19

Lời giải
Ta có n  *
, log 3  2u5  63   2 log 4  un  8n  8   log 3  2u5  63   log 2  un  8n  8  .
2u  63  3t
 2 u5  63  3t
Đặt t  log 3  2 u5  63    5    1  3t  2.2 t  t  2
un  8n  8  2 u5  32  2
t t

 un  8n  4  Sn  u1  u2  ...  un  4n2
un .S2 n  8n  4  .16n
2
148
Do đó    n  19 .
u2 n .Sn  16n  4  .4n 2
75
Chọn ý A.
1 1
1  m
x2  x  1 2
Câu 69. Cho hàm số f  x   e . Biết f  1  . f  2  . f  3  ... f  2017   e n
 m, n   với
m
là phân số tối giản. Tính P  m  n 2 .
n
A. 2018 B. 2018 C. 1 D. 1

Lời giải
Biến đổi giả thiết ta có
1 1 2 2 2
1  1 1  2 1 1  1 1  1 1  1 1 1 1
    1     2  1   1  1 
2
 x  1 2
 x x  1  x  x  1
f x  e
x  x x 1   x x 1   x x1 
e e e  ex x 1
 e.e x x 1
.
Do đó ta được:
1 1 1 1 1 1 1 1 1
1    
f  1   e.e 2
; f  2   e.e 2 3
; f  3   e.e 3 4
;<; f  2016   e.e 2016 2017
; f  2017   e.e 2017 2018
.
1 2017 20182  1
1  2017
 f  1  . f  2  . f  3  ... f  2017   e 2017
.e 2018
e 2018
e 2018

 m  20182  1 , n  2018 . Vậy P  1 .


Chọn ý D.
62 | Chinh phục olympic toán Điều ta biết là giọt nước, điều ta chưa biết là đại dương - Newton
TUYỂN TẬP MỘT SỐ NHÓM CÂU HỎI VẬN DỤNG CAO MÔN TOÁN

Câu 70. Cho cấp số cộng  un  có tất cả các số hạng đều dương thoả mãn điều kiện
u1  u2  ...  u2018  4  u1  u2  ...  u1009  . Tìm giá trị nhỏ nhất của biểu thức
P  log 23 u2  log 23 u5  log 23 u14 .
A. 3 B. 1 C. 2 D. 4
Lời giải
2018 1009
Ta có S2018   2u1  2017 d  , S1009   2u1  1008d  .
2 2
Theo giả thiết, ta có u1  u2  ...  u2018  4  u1  u2  ...  u1009 
2018 1009 d
  2u1  2017 d   4.  2u1  1008d   2u1  2017 d  2  2u1  1008d   u1 
2 2 2
d 3d 5d
Dãy số  un  : , , , ...
2 2 2
3d 9d 27 d
Ta có P  log 23 u2  log 23 u5  log 23 u14  log 23  log 23  log 23
2 2 2
2 2 2
 d  d  d
  1  log 3    2  log 3    3  log 3  .
 2  2  2
d
 x thì P   1  x    2  x    3  x   3x 2  12 x  14  2
2 2 2
Đặt log 3
2
2
Dấu bằng xảy ra khi x  2  d  . Vậy giá trị nhỏ nhất của P bằng 2.
9

Câu 71. Cho cấp số cộng  an  , cấp số nhân  bn  thỏa mãn a2  a1  0 và b2  b1  1 ; và hàm
số f  x   x 3  3x sao cho f  a2   2  f  a1  và f  log 2 b2   2  f  log 2 b1  . Số nguyên
dương n nhỏ nhất và lớn hơn 1 sao cho bn  2018 an là
A. 16 B. 15 C. 17 D. 18

Lời giải
Hàm số f  x   x 3  3x có bảng biến thiên như sau
x  1 1 
y'  0  0 

y 2
 2
 f  a2   2  f  a1  
  f  a 2   f  a1 
Theo giả thiết  
 a2  a1  0
  a2  a1  0

0  a1  a2  1
Từ đó suy ra  , hơn nữa f  x   2  0 x  0 . Ta xét các trường hợp
0  a1  1  a2

Tinh hoa của toán học nằm ở tự do của nó – Georg Cantor Chinh phục olympic toán | 63
CÁC BÀI TOÁN VẬN DỤNG CAO DÃY SỐ

 f  a2   2  0 
  f  a2   2  a2  1
 Nếu 0  a1  a2  1 thì    .

 f  a1   0 
 f  a1   0  a1  0
 f  a2   2  0
 Nếu 0  a1  1  a2 thì  điều này là không thể.
 f  a1   0
Do đó chỉ xảy ra trường hợp a1  0; a2  1 .
Từ đó suy ra an  n  1  n  1  . Tương tự vì b2  b1  1 nên log 2 b2  log 2 b1  0 , suy ra
log 2 b2  1 b2  1
   bn  2 n1  n  1 .
log
 2 1 a  0 b
 1  1
Xét hàm số g  x   2 x  2018x trên nữa khoảng  0;   , ta có bảng biến thiên
2018
x  log 2 
ln 2
g ' x   0 

1
g x
 2018 
g  log 2 
 ln 2 
  2018 
 g  log 2 ln 2   0
  
 2018
log 2 ln 2  11

Ta có  g  12   20120 nên số nguyên dương nhỏ nhất n thỏa g  n  1  0

 g  13   18042

 g  14   11868
 g  15   2498  0

là n  1  15  n  16 .
Chọn ý A.
Câu 72. Cho cấp số nhân  bn  thỏa mãn b2  b1  1 và hàm số f  x   x 3  3x sao cho
f  log 2  b2    2  f  log 2  b1   . Giá trị nhỏ nhất của n để bn  5100 bằng

A. 234 B. 229 C. 333 D. 292

Lời giải
Xét hàm số f  x   x 3  3x .
Có f   x   3x 2  3 , f   x   0  x  1 .
Ta có bảng biến thiên sau

64 | Chinh phục olympic toán Điều ta biết là giọt nước, điều ta chưa biết là đại dương - Newton
TUYỂN TẬP MỘT SỐ NHÓM CÂU HỎI VẬN DỤNG CAO MÔN TOÁN

x  1 1 
y'  0  0 

y 2
 2

Mặt khác, ta có b1  b2  1 . Đặt a  log 2 b2  log 2 b1  b  0 . Ta có: a 3  3 a  2  b 3  3b  1  .


 Nếu b  1  a  b  1  a 3  3 a  b 3  3b   1  vô nghiệm.

Nếu 0  b  1  2  b 3  3b  0  a 3  3 a  2  0   a  1   a  2   0 .
2


b1  2  1
0

Suy ra a  1  b  0 . Khi đó   bn  2 n1  5100  n  1  100 log 2 5  n  234 .


b2  2  2
1

Vậy giá trị nhỏ nhất của n là 234 .
Chọn ý A.
  2 1 1  4 u2 7 6 u1 6
log 1  u1  u3  4 u1  8   e e 3
 3  
Câu 73. Cho dãy số  un  thỏa mãn 
u  3  u  n  4  , n  *
 n  1 2  n n 2  3n  2 

3   n  1 2 2018
Giá trị lớn nhất của số n để un 
n1
A. 3472 B. 3245 C. 3665 D. 3453

Lời giải
3 3 2  3 3 3 
Biến đổi giả thiết ta có un1   un     un1    un  
2 n1 n2 n2 2 n1
3 3 3
Đặt vn  un   vn1  vn   vn  là CSN với công bội q  .
n1 2 2
n 1 n 1 n 1
3 3  3 3 3  3
Khi đó vn    v1     u1    un      u1  
2 2  2 n1  2  2
33 9 13 3
Ta có u3   u1 , u2   u1 , thay vào giả thiết ta được
8 4 4 2
log 1  u12  2u1  4   e 6 6 u1  e 6 u1 6  3
3

Theo bất đẳng thức AM – GM ta có e 6 6 u1  e 6 u1 6  3  2 e6 6 u1 .e6 u1 6  3  1


Mặt khác ta cũng có log 1  u12  2u1  4   log 1
3 3
 u  1  3  1
1
2

n 1
3 13
Do đó VT  VP , đẳng thức xảy ra khi và chỉ khi u1  1  un    
n1 2 2

Tinh hoa của toán học nằm ở tự do của nó – Georg Cantor Chinh phục olympic toán | 65
CÁC BÀI TOÁN VẬN DỤNG CAO DÃY SỐ

3   n  1 2 2018 3   n  1  2 2018
n 1
3 13
Để un        n  3453
n1 n1 2 2  n1
Chọn ý D.

f  1 . f  3  ... f  2n  1 
Câu 74. Cho f  n    n2  n  1   1 n  N * . Đặt un 
2
.
f  2  . f  4  ... f  2n 
10239
Tìm số n nguyên dương nhỏ nhất sao cho un thỏa mãn điều kiện log 2 un  un  .
1024
A. n  23 B. n  29 C. n  21 D. n  33

THPT Chuyên Biên Hòa – Hà Nam lần 1 năm học 2017 – 2018
Lời giải
Từ giả thiết ta có f  n    n2  n  1   1   n2  1  n  1  1 .
2 2
 
1
 1  2 2  1  32  1  4 2  1  ...  2 n  1   1  4n2  1 
2

2

Khi đó ta có un 
2
2
 1  3  1  4  1  5  1  ...  4n  1   2 n  1   1 
2 2 2 2

2

2 1
  2
 2n  1  1 2n  2n  1
2

10239 1 10239
Theo đề bài ta có log 2 un  un    log 2  2n2  2n  1  2  0.
1024 2n  2n  1 1024
1 10239
Xét hàm số g  n    log 2  2n2  2n  1  2  với n  1 .
2n  2n  1 1024
4n  2 4n  2
Ta có g  n      0 với n  1  g  n  nghịch biến.
 2n  2n  1 ln 2  2n2  2n  12
2

 1  2047  1 10239
  0 nên  log 2  2n  2n  1   2  0
2
Mà g 
 2 2n  2n  1 1024
 
1  2047
n . Do n nguyên dương nhỏ nhất thỏa mãn nên n  23
2
Chọn ý A.

  
Câu 75. Cho biểu thức A  log 2017  log 2016  log 2015  log ...  log  3  log 2  ...  
Biểu thức A có giá trị thuộc khoảng nào trong các khoảng dưới đây?
A.  log 2017; log 2018  B.  log 2019; log 2020 
C.  log 2018; log 2019  D.  log 2020; log 2021 
Lời giải

  
Đặt An  log 2017  log 2016  log 2015  log  ...  log  3  log 2 ...   A n   n  An1 

Ta có

66 | Chinh phục olympic toán Điều ta biết là giọt nước, điều ta chưa biết là đại dương - Newton
TUYỂN TẬP MỘT SỐ NHÓM CÂU HỎI VẬN DỤNG CAO MÔN TOÁN

0  log 2  1  0  A2  1
 0  log 3  A3  log  3  A2   log 4  1
...
 0  log 9  A9  log  9  A8   log 10  1
 1  log 10  A10  log  10  A9   log 11  2
 1  log 12  A11  log  11  A10   log 13  2
...
 2  log 999  A997  log  997  A996   log 1000  3
 3  log 1000  A998  log  998  A997   log 1001  4
 3  log 1002  A 999  log  999  A998   log 1003  4
...
 3  log 2020  A2017  log  2017  A2016   log 2021  4
Vậy A2017   log 2020; log 2021 
Câu 76. Cho dãy số  un  xác định bởi un  ln  2n2  1   ln  n2  n  1  , n  1 . Tìm số
2
nguyên n lớn nhất sao cho un   un   . Biết  a kí hiệu phần nguyên của số a là số tự
3
nhiên nhỏ nhất không vượt quá a.

A. 37 B. 36 C. 38 D. 40

Lời giải
2n  1
2
Ta có un  ln   0; ln 2    un   0
n2  n  1
2 2  2 n2  1  2 2 n2  1
 un   un    un   ln  2     3 e 2  n  37.462
n n1 3 n n1
2
3 3
Chọn ý A.
Câu 77. Cho dãy số  un  có tất cả số hạng đều dương thỏa mãn un 1  2un và đồng thời

4
u12  u22  ...  un2  un21  un 2  1  , n  1 . Số tự nhiên n nhỏ nhất để un  5100 là?
3
A. 232 B. 233 C. 234 D. 235

Lời giải
n1
Ta có un 1  2un  un  2 u1 , đẳng thức đúng với mọi n  1 nên đúng với n  1 nên
4 4
u12  u22  u3  1 
 u12  4u12  4u1  1 
3 3
4 4 1
 u12  2u1  1   u1  1   u1 
3 3 3
2 n 1
Do đó un   5100  2 n1  3.5100  n  log 2 3  100 log 2 5  233 .
3
Chọn ý C.
Tinh hoa của toán học nằm ở tự do của nó – Georg Cantor Chinh phục olympic toán | 67
CÁC BÀI TOÁN VẬN DỤNG CAO DÃY SỐ

Câu 78. Cho dãy số  un  thỏa mãn ln  u12  u22  10   ln  2u1  6u2  và đồng thời
un  2  un  2un  1  1, n  1 . Giá trị nhỏ nhất của n để un  5050

A. 100 B. 99 C. 101 D. 102

Lời giải
Biến đổi giả thiết ta có
u1  1
ln  u12  u22  10   ln  2u1  6u2    u1  1    u2  3   0  
2 2

u2  3
Mặt khác ta có un 2  un  2un  1  1  un  2  un  1  un  1  un  1 .
Đặt vn  un 1  un  vn 1  vn  1   vn  là CSC có công sai d  1
u2  u1  2
u  u  3
 3
n
n  n  1
 un  u1   i 
2
Khi đó  vn  n  1  un  1  un  n  1  
................... i 2 2
un  un 1  n

n  n  1
Vậy để un  5050   5050  n  100
2
Chọn ý C.
  391  1 39 
log  u2    log  u1    2
  40  4 4 
Câu 79. Cho dãy số  un  thỏa mãn  .
u  2  n  1  un  1  2n
, n  *

  
n 2
n n 2
 n  1  1

5100  n2  1
Giá trị nhỏ nhất của n để un  100 3 .
5  n  n

A. 235 B. 255 C. 233 D. 241

Lời giải
Ta có  n2  n  1  1   n2  1   2n  n2  1   n2  1   n2  1   n  1   1
2 2
 2

Biến đổi giả thiết tương đương
 n  1  2  n2  1   1
2
2n  n2
nun  2  n  1  un  1   2  n  1  un  1 
n 2

 1  n  1  1
2
 n 2

 1  n  1  1
2

1 2 1 1 1 
 nun  2  n  1  un  1     n  1  un  1    nun  2 
n  1  n  1  1
2 2
 n  1  1 2 
2
n 1
1 1 1
Đặt vn  nun   vn1  vn   vn  là CSN có công bội q 
n 1
2
2 2
n1 n1
1 1  1 1 1  1
Từ đó suy ra vn    v1     u1    un  3  n1  u1  
2 2  2 n n 2 n 2

68 | Chinh phục olympic toán Điều ta biết là giọt nước, điều ta chưa biết là đại dương - Newton
TUYỂN TẬP MỘT SỐ NHÓM CÂU HỎI VẬN DỤNG CAO MÔN TOÁN

1 1
Thay u2    u1 vào giả thiết ta được
40 4
1 39  1 39  1 1
log  u1    log  u1    2  u1  1  un  3  n
4 4  4 4  n n 2 n
5100  n2  1
Để un   n  100 log 2 5  n  233
5100  n3  n 
Chọn ý C.
4
Câu 80. Gọi q là công bội của một cấp số nhân , biết tổng ba số hạng đầu bằng 16 , đồng
9
thời theo thứ tự , chúng là số hạng thứ nhất , thứ tư và thứ tám của một cấp số cộng . Hỏi
q thuộc khoảng nào sau đây?

A. q   3; 4  B. q   1; 2  C. q   2; 3  D. q   0; 1 

Lời giải
Gọi : u1 , u2 , u3 là 3 số hạng đầu tiên của cấp số nhân , với công bội q . Gọi  vn  là cấp số
cộng tương ứng với công sai là d . Theo giả thiết ta có :
 4  4
u1  u2  u3  16 9 u1  u1q  u1q  16 9
2
 1
 
 u1  v1  u1q  u1  3d 2 
u2  v4  v1  3d  2
 u1q  u1  7 d 3
u3  v8  v1  7 d 
Khử d từ (2) và (3) ta được : u1  3q 2  7 q  4   0  4 .
q  1
4
Do (1) nên : u1  0   4    4 . Theo định nghĩa thì q  1 , do vậy q 
q  3
 3

I n 2
Câu 81. Cho I n   2 sin n xdx với n nguyên dương. Tính lim .
0 In
A. 1 B. 1 C. 2 D. 
Lời giải
 
Xét I n 2   2 sin n 2 xdx   2 sin n1 x.sin xdx
0 0

u  sin x du   n  1 sin x.cosxdx



n1 n

Đặt  
dv  sin xdx 
 v   cos x
 
 I n 2   cos x.sin n1
x   cos x.  n  1 sin n x.cos xdx
2 2
0 0
 
 I n 2  0   2  n  1 sin n x.cos 2 xdx   n  1   2 sin n x.  1  sin 2 x  dx
0 0
 
 I n 2   n  1  sin n xdx   n  1  sin n 2 xdx   n  1  .I n   n  1  .I n 2
2 2
0 0

Tinh hoa của toán học nằm ở tự do của nó – Georg Cantor Chinh phục olympic toán | 69
CÁC BÀI TOÁN VẬN DỤNG CAO DÃY SỐ

I n 2 n  1
  n  2  .I n 2   n  1  .I n  
In n2
I n 2 n1
 lim  lim  1.
In n2
Chọn ý B.
1
I n1
Câu 82. Với mỗi số nguyên dương n ta kí hiệu I n   x 2  1  x 2  dx . Tính lim
n
.
n  I
0 n

A. 1 B. 2 C. 3 D. 5
Lời giải
Cách 1. Tự luận
du  dx
1 u  x 
Xét I n   x 2  1  x 2 n
  1  x2  .
n1
dx . Đặt  
dv  x  1  x  dx
2 n
v 
2  n  1
0

n1 1
x  1  x 2  1
1
1
1

 1  x 2  dx   1  x 2  dx
n1 n1
 In 
n1
 
2  n  1 0 
2  n  1 0
0

1
1
1 1 1

 I n1  
2 n  2 0
 1  x  1  x  dx  I n1 
2 2 n1
   1  x  dx   x  1  x  dx 
2  n  2   0
2 n1 2 2 n1

0 
1 I 2n  1 I
 I n1   2  n  1 I n  I n1   n1   lim n1  1 .
2 n  2 In 2n  5 n  In
Cách 2. Trắc nghiệm
Ta thấy 0   1  x 2   1 với mọi x   0; 1  , nên
1 1 1

I n1   x  1  x
2

2 n1
dx   x  1  x
2
  1  x  dx   x  1  x 
2 n 2 2 2 n
dx  I n ,
0 0 0

I n1 I
Suy ra  1 , nên lim n 1  1 . Dựa vào các đáp án, ta chọn A.
In In
Chọn ý A.


  x  12 n 2 x 2  1
Câu 83. Đặt I n    n
1   2x 2  1  dx. Tính lim In .
0 
 x 2  1  x 2  1  
2 n1 n n1
 I n1
 
1 3
A. 1 B. C. 1 D.
2 2
Lời giải
 
2x 2  1  x  1
2

1 2x2  1 1 
Ta có bước biến đổi sau I n   n .  n . dx
0  x 2  1  x 2  1 2 x2  1 x2  1 
 

70 | Chinh phục olympic toán Điều ta biết là giọt nước, điều ta chưa biết là đại dương - Newton
TUYỂN TẬP MỘT SỐ NHÓM CÂU HỎI VẬN DỤNG CAO MÔN TOÁN

2x 2  1  x  1   x  1
2 2
1 1 2 x 2  1 2 xdx
 n . dx   n .
x2  1  x 2  1 x 2  1  x 2  1 2
0 2 0

2x2  1 2 xdx
Đến lúc này ta sẽ đổi biến. Đặt u   du  2
x 1
2
 x  1
3
n1 2

n  3 n 
n1
3 3 1
u n
 In   2 n
udu   2
u du 
n
 .  1
1 1 n1 n1  2 
 
n 1

n  3 
n1
n
.  1
In n1  2 
     lim I n  1
n  1  3 n 1 
n 2
I n1 I n1
. 1 
n2  2 
 
Chọn ý A.
x  xn
n
Câu 84. Ta đặt Fn  x    dx . Biết Fn  1   0 n . Tính lim Fn  2  .
x n1 n 

A. 1 B.  C. 1 D. 
Lời giải

 1  1
x n  n1  1 
n n 1
n
xx n
 x  x n1
Ta có Fn  x    dx   dx   x n dx
x n1 x n1
1 1n dx du
Đặt u  n1  1  du  n dx  n 
x x x 1n
n1
n
u 1 u n
 Fn  x   Gn  u    du  . C
1n 1n n1
n
n1
n  1  n
 Fn  x   .
2  n1
 1  C . Mà Fn  1   0 n  C  0 n
1n  x 
 n
 nlim  
 1  n 2
n1 
n  1 n   1 
 Fn  2   2  n1
 1  . Có  lim  n 1  1   1  lim Fn  2   
1n  2  
n  2
  n 

 n1
 nlim 1
  n
Chọn ý D.

Tinh hoa của toán học nằm ở tự do của nó – Georg Cantor Chinh phục olympic toán | 71
CÁC BÀI TOÁN VẬN DỤNG CAO DÃY SỐ

1
e  nx
Câu 85. Cho tích phân I n   dx với n  .
0
1  e x
Đặt un  1.  I 1  I 2   2  I 2  I 3   3  I 3  I 4   ...  n  I n  I n 1   n . Biết lim un  L . Mệnh đề nào
sau đây là đúng?
A. L   1; 0  B. L   2; 1  C. L   0; 1  D. L   1; 2 
Lời giải
Với n  , biến đổi giả thiết ta có
e 
1  n1 x 1 1 1 1
e  nx .e  x e  nx
I n1  x
dx   x
dx   e dx  
 nx
x
dx   e  nx dx  I n
0
1e 0
1e 0 0
1e 0
1
1
 I n 1   e  nx dx  I n  I n1  I n 
n
 1  en 
0

Do đó un   1  e 1    1  e 2    1  e 3   ...   1  e  n   n  un  e 1  e 2  e 3  ...  e  n
1
Ta thấy un là tổng n số hạng đầu của một cấp số nhân lùi vô hạn với u1  e 1 và q  ,
e
e 1 1
nên lim un  L  L   1; 0  .
1 e  1
1
e
Chọn ý A.
Câu 86. Có bao nhiêu giá trị nguyên dương n thỏa mãn tích phân
2

 1  n  2 x  3x 2  4x 3  ...  nx n1  dx  2
2

A. 1 B. 2 C. 0 D. 3
Lời giải
Biến đổi giả thiết ta có:
2

 1  n  2 x  3x 2  4x 3  ...  nx n1  dx  2   x  n2 x  x 2  x 3  x 4  ...  x n   2


2
2
0
0

 2  2 n2  2 2  2 3  2 4  ...  2 n  2  1  2  2 2  ...  2 n 1  n2  1
 2 n  1  n2  1  2 n  n2  2  0 .
Thử với các giá trị n  1; 2; 3; 4 đều không thỏa mãn.
Với n  , n  5 ta chứng minh 2 n  n 2  2  1  . Dễ thấy n  5 thì  1  đúng.
Giả sử  1  đúng với n  k với k  , k  5 . Khi đó 2 k  k 2  2 .

Khi đó: 2 k  1  2  k 2  2   k 2  k 2  2  2  k 2  2 k  1  2   k  1   2 .
2

Do đó  1  đúng với n  k  1 . Theo nguyên lý quy nạp thì  1  đúng.


Vậy không tồn tại số nguyên n .
Chọn ý C.

72 | Chinh phục olympic toán Điều ta biết là giọt nước, điều ta chưa biết là đại dương - Newton
TUYỂN TẬP MỘT SỐ NHÓM CÂU HỎI VẬN DỤNG CAO MÔN TOÁN

Câu 87. Cho hàm số f  x  có đạo hàm liên tục trên đoạn  0; 1 thỏa mãn điều kiện
1
f  2018x  2017   2018 f  x  , x   f  x   dx ?
2
. Tính tích phân 0 

4 5 7 8
 f  1    f  1    f  1    f  1  
2 2 2 2
A. B. C. D.
3 3 3 3
Lời giải
Xét biểu thức f  2018x  2017   2018 f  x  . Lấy đạo hàm 2 vế ta được
2018 f '  2018x  2017   2018 f '  x 
 x  2017 
 2018  2018  1   x  20182  1 
Thay x bởi 2018 x  2017 , ta được f '  x   f '    f ' 
 2018   20182 
 
Thay đến n lần và bằng quy nạp ta chứng minh được
 x  2018n  1   x 1 
f ' x   f ' n   f ' n
1 
 2018   2018 2018n 
Khi n    f '  x   f '  1   f  x   f '  1  x  C  * 
Thay x  1  f  1   2018 f  1   f  1   0
Thay x  1   *  : f  1    f '  1   C  0  f '  1   C
1 7
Vậy f  x   f '  1  x  1     f  x   dx   f  1 
2 2

0 3
Chọn ý C.
Câu 88. Cho I n   tan n xdx với n  . Khi đó I 0  I 1  2  I 2  I 3  ...  I 8   I 9  I 10 bằng?

 tan x   tan x   tan x   tan x 


9 r 9 r 1 10 r 10 r 1

A. 
r 1 r
C B. 
r 1 r 1
C C. 
r 1 r
C D. 
r 1 r 1
C

Lời giải
Biến đổi tích phân ban đầu ta có
 1  tan n1 x
I n   tan n 2 x.tan 2 xdx   tan n2 x.   1  dx   I n2  C
2
 cos x  n1
tan n1 x
  tan n 2 x.  tan x  dx  I n 2  I n  I n 2  C .
n1
Khi đó I 0  I 1  2  I 2  I 3  ...  I 8   I 9  I 10 =  I 10  I 8    I 9  I 7   ...   I 3  I 1    I 2  I 0 
9
tan 9 x tan 8 x tan 2 x tan r x
   ....   tan x  C   C .
9 8 2 r 1 r

Tinh hoa của toán học nằm ở tự do của nó – Georg Cantor Chinh phục olympic toán | 73
CÁC BÀI TOÁN VẬN DỤNG CAO DÃY SỐ

 6 2
U1  U
Câu 89. Cho dãy số xác định bởi  4 , n  1, n  N * . S= lim n có giá trị là ?
U  2.U 2  1 n
 n1 n

1 1
A. 1 B. C. 0 D.
2 4
Lời giải
Đây là một bài toán lượng giác hoá quen thuộc, với ý tưởng gợi mở từ công thức biến đổi
hạ bậc cos 2 x  2 cos 2 x  1
 6 2 
U 1   cos
 4 12
  
U 2  2.U 12  1  2.cos 2  1  cos 2.
Nhận thấy  12 12
....

 n1 
U n  2.U 2
n  1  1  cos 2
12

cos 2 n1
Un 12  1 và lim 1  0  lim U n  0
Lại có 0  
n n n n n

 1
U 1  2
Câu 90. Cho dãy số Un xác định bởi  ,n  1
U  U 2
n  n  U n  1   n 2

 n  1 n
 1 1 1 1 
Khi đó S  lim      thuộc khoảng nào sau đây?
 U1 U 2 U 3 Un 
A.  3; 1  B.  1; 2  C.  1; 2  D.  1; 1 
Lời giải
Đây là một bài toán khá khó. Với những dạng toán nhưng này, hướng biến đổi nằm ở câu
1
hỏi, tức là ta phải tìm cách đưa dãy về dạng
Uk
Để có được điều này, hướng giải quyết cơ bản và ưu tiên là thêm vào 2 vế f  x  sao cho
khi chia U k xuống mẫu, ta được một dãy có khả năng triệt tiêu
U k2  k U k  1  k 2
U k2  kU k  k  k 2 U k2  kU k
Cụ thể, ta thấy U k 1    k1
k k k
Thêm vào 2 vế   k  1  ( có thể nói là chuyển  k  1  sang trái , nhưng mình dùng từ theo
đúng phương pháp trên vì phương pháp này còn áp dụng vào nhiều dạng toán) , ta được:

74 | Chinh phục olympic toán Điều ta biết là giọt nước, điều ta chưa biết là đại dương - Newton
TUYỂN TẬP MỘT SỐ NHÓM CÂU HỎI VẬN DỤNG CAO MÔN TOÁN

U k2  kU k 1 k 1 1
U k 1   k  1    2  
k U k 1   k  1 U k  kU k U k  k U k
1 1 1
  
U k U k  k U k 1   k  1
 1 1 1
U  U  1  U  2
 1 1 2

 1 1 1
U  U  2  U  3
Áp dụng vào dãy số  2 2 3
....

 1  1  1
U
 n Un  n Un1   n  1
Cộng vế với vế ta được
1 1 1 1 1 1 1
      2 
U1 U2 U3 U n U 1  1 U n 1   n  1  U n 1   n  1 
Lại có U k  3 k - Chứng minh qua quy nạp
1 1
 Un1  3  n  1  Un   n  1  2 n  2  0  
Un 1   n  1  2n  2

1 1 1
Mà lim  0  lim 0 2  2  S  2
2n  2 Un1   n  1  Un1   n  1
Câu 91. Trong dịp hội trại hè 2017, bạn Anh thả một quả bóng cao su từ độ cao 6  m  so
với mặt đất, mỗi lần chạm đất quả bóng lại nảy lên một độ cao bằng ba phần tư độ cao lần
rơi trước. Biết rằng quả bóng luôn chuyển động vuông góc với mặt đất. Tổng quãng
đường quả bóng đã bay (từ lúc thả bóng cho đến lúc bóng không nảy nữa) khoảng ?
A. 44  m  B. 45  m  C. 42  m  D. 43  m 
Lời giải
Ta có quãng đường bóng bay bằng tổng quảng đường bóng nảy lên và quãng đường bóng
rơi xuống.
3
Vì mỗi lần bóng nảy lên bằng lần nảy trước nên ta có tổng quãng đường bóng nảy lên
4
2 3 n
3 3 3 3
là S1  6.  6.    6.    ...  6.    ...
4 4 4 4
3 9 3
Đây là tổng của cấp số nhân lùi vô hạn có số hạng đầu u1  6.  và công bội q  .
4 2 4
9
Suy ra S1  2  18 .
3
1
4

Tinh hoa của toán học nằm ở tự do của nó – Georg Cantor Chinh phục olympic toán | 75
CÁC BÀI TOÁN VẬN DỤNG CAO DÃY SỐ

Tổng quãng đường bóng rơi xuống bằng khoảng cách độ cao ban đầu và tổng quãng
2 n
3 3 3
đường bóng nảy lên nên là S2  6  6.    6.    ...  6.    ...
4 4 4
3
Đây là tổng của cấp số nhân lùi vô hạn với số hạng đầu u1  6 và công bội q  .
4
6
Suy ra S2   24 .
3
1
4
Vậy tổng quãng đường bóng bay là S  S1  S2  18  24  42 .
Câu 92. Có hai cơ sở khoan giếng A và B. Cơ sở A giá mét khoan đầu tiên là 8000 (đồng)
và kể từ mét khoan thứ hai, giá của mỗi mét sau tăng thêm 500 (đồng) so với giá của mét
khoan ngay trước đó. Cơ sở B: Giá của mét khoan đầu tiên là 6000 (đồng) và kể từ mét
khoan thứ hai, giá của mỗi mét khoan sau tăng thêm 7% giá của mét khoan ngay trước
đó. Một công ty giống cây trồng muốn thuê khoan hai giếng với độ sâu lần lượt là 20  m 
và 25  m  để phục vụ sản xuất. Giả thiết chất lượng và thời gian khoan giếng của hai cơ
sở là như nhau. Công ty ấy nên chọn cơ sở nào để tiết kiệm chi phí nhất?
A. luôn chọn A.
B. luôn chọn B.
C. giếng 20  m  chọn A còn giếng 25  m  chọn B.
D. giếng 20  m  chọn B còn giếng 25  m  chọn B.
Lời giải
Cơ sở A giá mét khoan đầu tiên là 8000 (đồng) và kể từ mét khoan thứ hai, giá của mỗi
mét sau tăng thêm 500 (đồng) so với giá của mét khoan ngay trước đó. Do đó theo tổng
của một cấp số cộng ta có:
20
+ Nếu đào giếng 20  m  hết số tiền là: S20   2.8000   20  1  500   255000 (đồng).
2 
25
+ Nếu đào giếng 25  m  hết số tiền là: S25   2.8000   25  1  500   350000 (đồng).
2 
Cơ sở B giá của mét khoan đầu tiên là 6000 (đồng) và kể từ mét khoan thứ hai, giá của
mỗi mét khoan sau tăng thêm 7% giá của mét khoan ngay trước đó. Do đó theo tổng của
một cấp số nhân ta có:

1   1, 07 
20

+ Nếu đào giếng 20  m  hết số tiền là: S20


  6000  245973 (đồng).
1  1, 07
1   1, 07 
25

+ Nếu đào giếng 25  m  hết số tiền là: S25


  6000  379494 (đồng).
1  1, 07
  S25 nên giếng 20  m  chọn B còn giếng 25  m  chọn A.
  S20 , S25
Ta thấy S20

76 | Chinh phục olympic toán Điều ta biết là giọt nước, điều ta chưa biết là đại dương - Newton
TUYỂN TẬP MỘT SỐ NHÓM CÂU HỎI VẬN DỤNG CAO MÔN TOÁN

Câu 93. Cho cấp số cộng  un  có các số hạng đều dương, số hạng đầu u1  1 và tổng của
100 số hạng đầu tiên bằng 14950 . Tính giá trị của tổng sau?
1 1 1
S   ... 
u2 u1  u1 u2 u3 u2  u2 u3 u2018 u2017  u2017 u2018
1 1  1
A.  1  B. 1  C. 2018 D. 1
3 6052  6052
Lời giải
Gọi d là công sai của cấp số cộng. Khi đó
100.99
S100  100u1  d  100  4950d  14950  d  3 .
2
Do đó u2018  u1  2017 d  6052 .

1 1 1 u  uk 1  1 1 
Ta có   . k 1  .  .
uk  1 uk  uk uk  1  
uk . uk  1 . uk  uk  1 d uk . uk  1 
d  uk uk  1 

1  1 1  1  1 1  1  1 1 
 S  .    .    ...  .   

d  u1  
u2  d  u2 
u3  
d  u2017 u2018 

1  1 1  1 1 
 .    1 
d  u1 u2018  3  6052 
Câu 94. Giá trị của tổng 4  44  444  ...  44...4 (tổng đó có 2018 số hạng) bằng?
40
A.
9
 10 2018  1   2018 . B. 
4  10 2019  10 
 2018  .
9 9 
4
4  10 2019  10
C. 

 2018  . D.
9
 10 2018  1  .
9 9 
Lời giải
Cách 1. Đặt S  4  44  444  ...  44...4 (tổng đó có 2018 số hạng). Ta có
9
S  9  99  999  ...  99...9   10  1    10 2  1    10 3  1   ...  10 2018  1 
4
9
 S   10  10 2  10 3  ...  10 2018   2018  A  2018 .
4
Với A  10  10  10  ...  10 2018 là tổng 2018 số hạng của một cấp số nhân có số hạng đầu
2 3

1  q 2018 1  10 2018 10 2019  10


u1  10 , công bội q  10 nên ta có A  u1  10  .
1q 9 9
9 10 2019  10 4  102019  10 
Do đó S  2018  S    2018  .
4 9 9 9 
u1  4
u1  4 
Cách 2. Xét dãy số có   4  4
un 1  10un  4  un  1   10  un  
 9  9

Tinh hoa của toán học nằm ở tự do của nó – Georg Cantor Chinh phục olympic toán | 77
CÁC BÀI TOÁN VẬN DỤNG CAO DÃY SỐ

 40
4  v1 
Đặt vn  un    9  v  n  là cấp số nhân.
9  vn 1  10 vn
4 v 4 2018.4
Ta có Sn  u1  u2  .......  u2018  v1   v2  ...  v2018   v1  v2  ...  v2018 
9 9 9 9
1  10 2018 40 40.  10  1
2018
1  qn
Trong đó Sv 2018  .v1  . 
1q 1  10 9 81
40 4 4  102019  10 
Vậy tổng là S 
81
 10 2018
 1  
9
.2018 
9

9
 2018  .

Câu 95. Cho dãy số  un  thỏa mãn un  un 1  6 , n  2 và log 2 u5  log 2
u9  8  11 . Đặt
Sn  u1  u2  ...  un . Tìm số tự nhiên n nhỏ nhất thỏa mãn Sn  20172018 .
A. 2587 B. 2590 C. 2593 D. 2584
Lời giải
Ta có dãy số  un  là cấp số cộng có công sai d  6 .
log 2 u5  log 2
u9  8  11  log 2 u5  u9  8   11  *  với u5  0 .
Mặt khác u5  u1  4d  u1  24 và u9  u1  8d  u1  48 .
u  8  u5  32
Thay vào  *  ta được  1 . Suy ra u1  8 .
u1  88  u5  64
n
Sn  20172018   2u1   n  1  d   20172018  3n2  5n  20172018  0 .
2
Vậy số tự nhiên n nhỏ nhất thỏa mãn Sn  20172018 là n  2593 .
Câu 96. Cho hai cấp số cộng  an  : a1  4 ; a2  7 ;...; a100 và  bn  : b1  1 ; b2  6 ;...; b100 . Hỏi có
bao nhiêu số có mặt đồng thời trong cả hai dãy số trên?
A. 32 B. 20 C. 33 D. 53
Lời giải
Cấp số cộng  an  : a1  4 ; a2  7 ;...; a100 có số hạng tổng quát: an  4   n  1  3  3n  1 .
Cấp số cộng  bn  : b1  1 ; b2  6 ;...; b100 có số hạng tổng quát: bm  1   m  1  5  5m  4 .
Các số có mặt đồng thời trong cả hai dãy số trên thỏa mãn hệ
 3n  1  5 m  4  3n  5  m  1 
 
1  n  100  1  n  100
1  m  100 1  m  100
 
Vì 3n  5  m  1  nên n 5 và m  1 3 với m  1  0
Ta lại có n  100  3n  300  5  m  1   300  m  61 .
Có m  1 3  m  3t  1 , t  * . Vì 1  m  61  1  3t  1  61  0  t  20 .
Vì t  *  t  1; 2; 3;...; 20 .
Vậy có 20 số hạng có mặt đồng thời ở hai dãy số trên.
78 | Chinh phục olympic toán Điều ta biết là giọt nước, điều ta chưa biết là đại dương - Newton
TUYỂN TẬP MỘT SỐ NHÓM CÂU HỎI VẬN DỤNG CAO MÔN TOÁN

Câu 97. Cho tam giác ABC cân tại A . Biết rằng độ dài cạnh BC , trung tuyến AM và độ
dài cạnh AB theo thứ tự đó lập thành một cấp số nhân có công bội q . Tìm công bội q của
cấp số nhân đó?
1 2 22 2 1  2 2  2 2
A. q  B. q  C. q  D. q 
2 2 2 2
Lời giải
2  AB  AC
2 2
  BC  1 .
2

Ta có AM 2 
4
Do ba cạnh BC , AM , AB lập thành cấp số nhân nên ta có: BC . AB  AM 2  2 
2  AB2  AC 2   BC 2
Thay  2  vào  1  ta được  BC.AB  4 AB2  4 AB.BC  BC 2  0
4
 AB 1  2
2  
 AB  AB  BC 2
 4  4 1  0 
 BC  BC  AB 1  2
   loai 
 BC 2
AB 1  2 1 2 22 2
  q  .
BC 2 2 2
Câu 98. Cho hàm số f  x    x 2  3x  2 
cos 2017 x 
và dãy số  un  được xác định bởi công thức
tổng quát un  log f  1   log f  2   log f  n  Tìm tổng tất cả các giá trị của n thỏa mãn
điều kiện un2018  1
A. 21 B. 18 C. 3 D. 2018
Lời giải
n n
Ta có un   log f  k    cos  2017 k  log  k  1   log  k  2     k chan    k le 
k 1 k 1

 Trường hợp 1: n  2 p khi đó ta có khai triển


un   log 3  log 4  log  2 p  1   log  2 p  1     log 2  log 3  log  2 p   log  2 p  1  
Như vậy un  log  p  1  un2018  1  p  9  n  18
 Trường hợp 2: n  2 p  1 khi đó ta có khai triển
un   log 3  log 4  log  2 p  1   log  2 p  1     log 2  log 3  log  2 p  2   log  2 p  3  
Như vậy un   log  4 p  6   un2018  1  p  1  n  3
Tổng các giá trị của n thỏa mãn điều kiện un2018  1 là 21.

Tinh hoa của toán học nằm ở tự do của nó – Georg Cantor Chinh phục olympic toán | 79
CÁC BÀI TOÁN VẬN DỤNG CAO DÃY SỐ

un  u4 n  u42 n  u42018 n a 2019  b


Câu 99. Biết rằng L  lim  Trong đó  un  xác định
un  u2 n  u2 2 n  u22018 n c

bởi u1  0; un 1  un  4n  3 và a b c , , là các số nguyên dương và b  2019 . Tính S  a  b  c


A. 1 B. 0 C. 2017 D. 2018
Lời giải
Ta có un  un1  4n  1  un  2n  n  3 2

Ta xét S1  n , 4n , 4 2 n , .4 2018 n , S2  n, 2n , 2 2 n , 2 2018 n


k3
Có uk  2 k 2  k  3  2  k  2.k   2k
2k  k  3  2k
2

k3  4 2019  1 
  2n  
3
 2019
1
Vậy L  lim
kS1 2k2  k  3  2  k  2
k3
  2 n   2 2019  1  3
kS2 2k  k  3  2  k
2

Câu 100. Cho ba số dương a , b , c theo thứ tự lập thành cấp số cộng. Giá trị lớn nhất của
a 2  8bc  3
biểu thức P  có dạng x y  x , y   Hỏi x  y bằng bao nhiêu?
 2a  c 
2
1
A. 9 B. 11 C. 13 D. 7
Lời giải
Ta có a  c  2b  a  2b  c  a 2   2b  c   a 2  8bc  4b 2  4bc  c 2  a 2  8bc   2b  c 
2 2

2b  c  3 t3
P   10  t  2b  c 
 2b  c  t2  1
2
1
1
Dấu bằng xảy ra khi 2b  c   x  y  11
3
Câu 101. Cho các số hạng dương a, b, c là số hạng thứ m, n, p của một cấp số cộng và một
cấp số nhân. Tính giá trị của biểu thức log 2 ab c  bc  a  c a b
A. 0 B. 2 C. 1 D. 4
Lời giải
Ta có a, b, c là số hạng thứu m, n, p của một cấp số cộng và một cấp số nhân nên
 a  u1   m  1  d  a1q n1 a  b   m  n  d
 
b  u1   n  1  d  a1q
n1
 b  c   n  p  d
c  u   p  1  d  a q p 1 c  a   p  m  d
 1 1 

a q 
 n p d  mnd
 P  log 2 ab c  bc  a  c a b  log 2  a1q m1  1
p 1
 log 2 a10 q 0  0

80 | Chinh phục olympic toán Điều ta biết là giọt nước, điều ta chưa biết là đại dương - Newton
TUYỂN TẬP MỘT SỐ NHÓM CÂU HỎI VẬN DỤNG CAO MÔN TOÁN


Câu 102. Cho a  b  c  và cot a , cot b , cot c Tạo thành cấp số cộng. Giá trị của cot a.cot c
2
bằng?
A. 1 B. 2 C. 3 D. 4
Lời giải
Ta có
    cot a  cot b  1 1
 abc   a  b   cot  a  b   cot   c   tan c  
2 2 2  cot a  cot b cot c
    cot a  cot b  1 1
 abc   a  b   cot  a  b   cot   c   tan c  
2 2 2  cot a  cot b cot c
 cot a  cot b  cot c  cot a  cot b  cot c
Mặt khác cot a  cot c  2 cot b  cot a  cot b  cot c  3 cot b  cot a  cot c  3
Ta có a  c  2b  sin A  sin C  2 sin B
AC A C B B A C A C
 2 sin cos  4 sin  cos  4 sin  cos
2 2 2 2 2 2
A C AC A C A C A C A C
 cos  2 cos  cos cos  sin sin  2 cos cos  2 sin sin
2 2 2 2 2 2 2 2 2 2
A C A C A C A C 1
 3 sin sin  cos cos  3 tan tan  1  tan tan 
2 2 2 2 2 2 2 2 3

Tinh hoa của toán học nằm ở tự do của nó – Georg Cantor Chinh phục olympic toán | 81
LỜI KẾT

Vậy là ta đã đi đến những trang cuối cùng của tuyển tập này với hơn 100 bài toán
đa dạng chắc hẳn đã mang tới cho bạn đọc một cái nhìn khác và mới lạ hơn về
chủ đề dãy số này. Các bạn thấy đó với hình thức thi trắc nghiệm như thế này sẽ
xuất hiện rất nhiều các dạng toán mới lạ mà nó liên kết nhiều mảng kiến thức với
nhau yêu cầu chúng ta cần phải tìm hiểu kỹ, sâu và rộng thì mới có thể giải quyết
được chúng. Hy vọng qua ebook này các bạn đã học thêm được nhiều điều và rút
ra được kinh nghiệm cho bản thân trong việc giải quyết các dạng toán mà bọn
mình đưa ra và nhiều dạng toán có liên quan khác. Sau đây bọn mình sẽ giới
thiệu cho các bạn một số tài liệu và sách tham khảo, trang web có thể giúp ích
được cho các bạn trong quá trình học tập.

1. Chuyên khảo dãy số - Nguyễn Tài Chung


2. Trắc nghiệm nâng cao chuyên đề dãy số - Đặng Việt Đông
3. Đi tìm công thức tổng quát của dãy số – Trần Duy Sơn
4. Các dạng toán phương pháp quy nạp toán học, dãy số, cấp số cộng và cấp
số nhân – Trần Quốc Nghĩa
5. Dãy số và giới hạn của dãy số – Nguyễn Tất Thu
6. Bài tập trắc nghiệm xác định số hạng thứ n của dãy số – Nguyễn Chiến
7. Tài liệu dãy số – cấp số dành cho học sinh khối chuyên – Lê Quang Ánh
8. Website toanmath.com
9. Website lovetoan.wordpress.com
Blog Chinh Phục Olympic Toán
https://lovetoan.wordpress.com/
Email: tuangenk@gmail.com
Blog chuyên chia sẻ tài liệu ôn học sinh giỏi môn toán
với rất nhiều tài liệu chất và thư viện tài liệu được
xây dựng rất đồ sộ

Ngoài ấn phẩm các bạn đang đọc thì các bạn có thể tìm hiểu thêm một số ấn
phẩm khác được đăng miễn phí trên blog sau

- Tham khảo thêm - Tài liệu được chia sẻ miễn phí


Bên cạnh blog của chúng tôi các bạn có thể theo dõi
trang fanpage: Tạp chí và tư liệu toán học – Đây là một
trang chúng tôi đăng free các tài liệu liên quan tới toán
VDC, VD, Ôn Olympic, HSG… Cảm ơn mọi người

You might also like